Está en la página 1de 598

INTRODUCCION A LA MECANICA

Herbert Massmann
V
ctor Muoz
n

AGRADECIMIENTOS (1997)
Aproximadamente la mitad de los problemas que guran al nal de cada cap
tulo no son originales del autor, sino que provienen de controles, pruebas y listas de
ejercicios que han circulado en la Facultad de Ciencias y en la Facultad de Ciencias
F
sicas y Matemticas de la Universidad de Chile durante la ultima dcada. Lamena

e
tablemente resulta casi imposible establecer quines son los autores intelectuales de
e
estos problemas para as poder darles el debido crdito.

e
Deseo agradecer a V
ctor Muoz, Miguel Kiwi, Fernando Lund, Patricia Lpez,
n
o
Pamela Weber, Claudio Romero y Lorena Cspedes, que con sus comentarios han mee
jorado el texto y permitieron pesquisar muchos de los errores de las versiones preliminares. Finalmente tambin mis agradecimientos al Dr. Hugo Arellano que gentilmente
e
permiti incorporar a estos apuntes la lista de problemas que l confeccion para el
o
e
o
curso.
Herbert Massmann

Notas adicionales (2008)


Los apuntes aqu presentados han servido de base, durante mucho tiempo, pa
ra los cursos de Mecnica I y II, correspondientes al Primer y Segundo Semestre
a
de Licenciatura en F
sica de la Facultad de Ciencias de la Universidad de Chile. La versin presentada en este archivo se basa en la ultima preparada por el
o

Dr. Herbert Massmann de que disponemos, correspondiente al ao 1997. No todas


n
las fuentes estaban disponibles, sin embargo, y completar estos apuntes ha signicado reconstruir algunos cap
tulos (1215) a partir de copias en papel. De hecho, en
esta versin dos cap
o
tulos no estn terminados: Gravitacin (Cap. 11, con problea
o
mas en algunas guras) y Ondas Sonoras (Cap. 15, ausencia de guras y algunas
referencias cruzadas). Ambos se pueden encontrar como documentos separados, en
http://llacolen.ciencias.uchile.cl/~vmunoz/cursos/mecanica2/mecanica2.html .
Los agradecimientos que estn ms arriba fueron copiados del texto original del
a
a
Dr. Massmann. Por mi parte, quisiera agregar agradecimientos a Areli Zniga, Max
u
Ram
rez y Felipe Gonzlez, que, en aos posteriores, tambin han colaborado en
a
n
e
la escritura de algunas secciones o en la confeccin de guras. Tambin a Nicols
o
e
a
Rojas, Felipe Asenjo, Denisse Pastn y Macarena Dom
e
nguez, que han le el texto
do
y encontrado aun ms errores.
a
Esperamos completar y mejorar estos apuntes en la medida que el tiempo y la
colaboracin de nuestros colegas y alumnos lo permita.
o
V
ctor Muoz
n


Indice general
1. Expansiones y Trigonometr
a
1.1. Expansiones y series . . . . . . . . .
1.2. Elementos de trigonometr . . . . .
a
1.3. Problemas . . . . . . . . . . . . . . .
1.4. Solucin a algunos de los problemas
o

.
.
.
.

.
.
.
.

.
.
.
.

.
.
.
.

.
.
.
.

.
.
.
.

.
.
.
.

.
.
.
.

.
.
.
.

.
.
.
.

.
.
.
.

.
.
.
.

.
.
.
.

.
.
.
.

.
.
.
.

1
1
8
19
31

2. Cinemtica en una dimensin


a
o
2.1. Posicin, velocidad y aceleracin . . . . . . .
o
o
2.2. El camino inverso . . . . . . . . . . . . . . . .
2.3. Mximos y m
a
nimos . . . . . . . . . . . . . .
2.4. Elementos del clculo innitesimal e integral .
a
2.5. Problemas . . . . . . . . . . . . . . . . . . . .
2.6. Solucin a algunos de los problemas . . . . .
o

.
.
.
.
.
.

.
.
.
.
.
.

.
.
.
.
.
.

.
.
.
.
.
.

.
.
.
.
.
.

.
.
.
.
.
.

.
.
.
.
.
.

.
.
.
.
.
.

.
.
.
.
.
.

.
.
.
.
.
.

.
.
.
.
.
.

.
.
.
.
.
.

.
.
.
.
.
.

.
.
.
.
.
.

36
36
56
60
61
64
85

.
.
.
.
.

94
94
104
112
115
132

.
.
.
.
.
.

138
138
141
144
152
159
175

.
.
.
.

3. Cinemtica en dos y tres dimensiones


a
3.1. Vectores . . . . . . . . . . . . . . . . .
3.2. Cinemtica . . . . . . . . . . . . . . .
a
3.3. Coordenadas polares . . . . . . . . . .
3.4. Problemas . . . . . . . . . . . . . . . .
3.5. Solucin a algunos de los problemas .
o
4. Las
4.1.
4.2.
4.3.
4.4.
4.5.
4.6.

leyes de Newton
Espacio y tiempo . . . . . . . . . . .
Las leyes de Newton . . . . . . . . .
Uso de las leyes de Newton . . . . .
Roce cintico y esttico . . . . . . .
e
a
Problemas . . . . . . . . . . . . . . .
Solucin a algunos de los problemas
o

.
.
.
.
.
.

.
.
.
.

.
.
.
.
.
.
.
.
.
.
.

.
.
.
.

.
.
.
.
.
.
.
.
.
.
.

.
.
.
.

.
.
.
.
.
.
.
.
.
.
.

.
.
.
.
.
.
.
.
.
.
.

.
.
.
.
.
.
.
.
.
.
.

.
.
.
.
.
.
.
.
.
.
.

.
.
.
.
.
.
.
.
.
.
.

.
.
.
.
.
.
.
.
.
.
.

.
.
.
.
.
.
.
.
.
.
.

.
.
.
.
.
.
.
.
.
.
.

.
.
.
.
.
.
.
.
.
.
.

.
.
.
.
.
.
.
.
.
.
.

.
.
.
.
.
.
.
.
.
.
.

.
.
.
.
.
.
.
.
.
.
.

.
.
.
.
.
.
.
.
.
.
.

.
.
.
.
.
.
.
.
.
.
.

.
.
.
.
.
.
.
.
.
.
.

5. Trabajo y energ
a
187
5.1. Trabajo y energ para movimientos en una dimensin . . . . . . . . . 188
a
o
5.2. Trabajo para un movimiento en tres dimensiones . . . . . . . . . . . . 200
i


INDICE GENERAL
5.3.
5.4.
5.5.
5.6.
5.7.

ii

Potencia . . . . . . . . . . . . . . . . . . . . .
Ejemplos . . . . . . . . . . . . . . . . . . . .
Resultados utiles para sistemas conservativos

Problemas . . . . . . . . . . . . . . . . . . . .
Solucin a algunos de los problemas . . . . .
o

6. Momento lineal y colisiones


6.1. Conservacin del momento lineal . .
o
6.2. Colisiones . . . . . . . . . . . . . . .
6.3. Impulso . . . . . . . . . . . . . . . .
6.4. Problemas . . . . . . . . . . . . . . .
6.5. Solucin a algunos de los problemas
o
6.6. Colisin de dos discos . . . . . . . .
o

.
.
.
.
.
.

.
.
.
.
.
.

.
.
.
.
.
.

.
.
.
.
.
.

.
.
.
.
.
.

.
.
.
.
.

.
.
.
.
.

.
.
.
.
.

.
.
.
.
.

.
.
.
.
.

.
.
.
.
.

.
.
.
.
.

.
.
.
.
.

.
.
.
.
.

.
.
.
.
.

.
.
.
.
.

.
.
.
.
.

.
.
.
.
.

.
.
.
.
.

205
206
207
213
230

.
.
.
.
.
.

.
.
.
.
.
.

.
.
.
.
.
.

.
.
.
.
.
.

.
.
.
.
.
.

.
.
.
.
.
.

.
.
.
.
.
.

.
.
.
.
.
.

.
.
.
.
.
.

.
.
.
.
.
.

.
.
.
.
.
.

.
.
.
.
.
.

.
.
.
.
.
.

.
.
.
.
.
.

238
238
242
246
248
255
261

7. Torque, centro de masas y equilibrio


7.1. Producto vectorial . . . . . . . . . . . . .
7.2. Torque . . . . . . . . . . . . . . . . . . . .
7.3. Centro de masas . . . . . . . . . . . . . .
7.4. Evaluacin numrica del centro de masas
o
e
7.5. Equilibrio . . . . . . . . . . . . . . . . . .
7.6. Problemas . . . . . . . . . . . . . . . . . .
7.7. Solucin a algunos de los problemas . . .
o

.
.
.
.
.
.
.

.
.
.
.
.
.
.

.
.
.
.
.
.
.

.
.
.
.
.
.
.

.
.
.
.
.
.
.

.
.
.
.
.
.
.

.
.
.
.
.
.
.

.
.
.
.
.
.
.

.
.
.
.
.
.
.

.
.
.
.
.
.
.

.
.
.
.
.
.
.

.
.
.
.
.
.
.

.
.
.
.
.
.
.

.
.
.
.
.
.
.

.
.
.
.
.
.
.

.
.
.
.
.
.
.

270
270
274
278
281
283
287
295

8. Momento angular
8.1. Momento angular de una part
cula . .
8.2. Momento angular de varias part
culas
8.3. Problemas . . . . . . . . . . . . . . . .
8.4. Solucin a algunos de los problemas .
o

.
.
.
.

.
.
.
.

.
.
.
.

.
.
.
.

.
.
.
.

.
.
.
.

.
.
.
.

.
.
.
.

.
.
.
.

.
.
.
.

.
.
.
.

.
.
.
.

.
.
.
.

.
.
.
.

.
.
.
.

.
.
.
.

.
.
.
.

.
.
.
.

306
306
310
316
321

9. Rotacin de un cuerpo r
o
gido
9.1. Las ecuaciones bsicas . . . . . . . .
a
9.2. Una notacin util . . . . . . . . . . .
o
9.3. Momento de inercia . . . . . . . . .
9.4. Problemas . . . . . . . . . . . . . . .
9.5. Solucin a algunos de los problemas
o

.
.
.
.
.

.
.
.
.
.

.
.
.
.
.

.
.
.
.
.

.
.
.
.
.

.
.
.
.
.

.
.
.
.
.

.
.
.
.
.

.
.
.
.
.

.
.
.
.
.

.
.
.
.
.

.
.
.
.
.

.
.
.
.
.

.
.
.
.
.

.
.
.
.
.

.
.
.
.
.

.
.
.
.
.

.
.
.
.
.

.
.
.
.
.

325
325
331
334
341
357

10.Fuerzas cticias
10.1. Referencial uniformemente acelerado
10.2. Referencial en rotacin uniforme . .
o
10.3. Problemas . . . . . . . . . . . . . . .
10.4. Solucin a algunos de los problemas
o

.
.
.
.

.
.
.
.

.
.
.
.

.
.
.
.

.
.
.
.

.
.
.
.

.
.
.
.

.
.
.
.

.
.
.
.

.
.
.
.

.
.
.
.

.
.
.
.

.
.
.
.

.
.
.
.

.
.
.
.

.
.
.
.

.
.
.
.

.
.
.
.

.
.
.
.

367
368
371
375
379


INDICE GENERAL

iii

11.Gravitacin
o
11.1. Elipses . . . . . . . . . . . . . . . . . . . . .
Elipse en coordenadas cartesianas . . . . . .
Elipse en coordenadas polares . . . . . . . .
11.2. Las leyes de Kepler . . . . . . . . . . . . . .
11.3. Satlites . . . . . . . . . . . . . . . . . . . .
e
11.4. Potencial efectivo . . . . . . . . . . . . . . .
11.5. Trayectorias de los satlites . . . . . . . . .
e
11.6. El campo y potencial gravitacional . . . . .
11.7. El caso elctrico: la ley de Coulomb . . . . .
e
11.8. Campo gravitacional de una cscara esfrica
a
e
11.9. Campo gravitacional de una esfera slida .
o
Densidad media de la Tierra . . . . . . . . .
11.10. roblemas . . . . . . . . . . . . . . . . . . .
P
11.11. olucin a algunos de los problemas . . . .
S
o

.
.
.
.
.
.
.
.
.
.
.
.
.
.

.
.
.
.
.
.
.
.
.
.
.
.
.
.

.
.
.
.
.
.
.
.
.
.
.
.
.
.

.
.
.
.
.
.
.
.
.
.
.
.
.
.

.
.
.
.
.
.
.
.
.
.
.
.
.
.

.
.
.
.
.
.
.
.
.
.
.
.
.
.

.
.
.
.
.
.
.
.
.
.
.
.
.
.

.
.
.
.
.
.
.
.
.
.
.
.
.
.

.
.
.
.
.
.
.
.
.
.
.
.
.
.

.
.
.
.
.
.
.
.
.
.
.
.
.
.

.
.
.
.
.
.
.
.
.
.
.
.
.
.

.
.
.
.
.
.
.
.
.
.
.
.
.
.

.
.
.
.
.
.
.
.
.
.
.
.
.
.

.
.
.
.
.
.
.
.
.
.
.
.
.
.

.
.
.
.
.
.
.
.
.
.
.
.
.
.

386
387
387
388
391
395
399
404
406
410
411
415
417
418
429

12.Fluidos
12.1. Conceptos preliminares . . . . . . . . .
12.2. La presin atmosfrica P0 . . . . . . .
o
e
12.3. Principio de Arqu
medes . . . . . . . .
12.4. La frmula baromtrica . . . . . . . .
o
e
12.5. Tensin supercial . . . . . . . . . . .
o
12.6. Capilaridad . . . . . . . . . . . . . . .
12.7. Fluidos en movimiento . . . . . . . . .
12.8. Aplicaciones del Principio de Bernoulli
12.9. Viscosidad . . . . . . . . . . . . . . .
12.10. roblemas . . . . . . . . . . . . . . . .
P
12.11. olucin a algunos de los problemas .
S
o

.
.
.
.
.
.
.
.
.
.
.

.
.
.
.
.
.
.
.
.
.
.

.
.
.
.
.
.
.
.
.
.
.

.
.
.
.
.
.
.
.
.
.
.

.
.
.
.
.
.
.
.
.
.
.

.
.
.
.
.
.
.
.
.
.
.

.
.
.
.
.
.
.
.
.
.
.

.
.
.
.
.
.
.
.
.
.
.

.
.
.
.
.
.
.
.
.
.
.

.
.
.
.
.
.
.
.
.
.
.

.
.
.
.
.
.
.
.
.
.
.

.
.
.
.
.
.
.
.
.
.
.

.
.
.
.
.
.
.
.
.
.
.

.
.
.
.
.
.
.
.
.
.
.

.
.
.
.
.
.
.
.
.
.
.

.
.
.
.
.
.
.
.
.
.
.

.
.
.
.
.
.
.
.
.
.
.

.
.
.
.
.
.
.
.
.
.
.

436
436
438
440
442
446
448
449
452
456
459
471

13.Oscilador armnico
o
2
13.1. La ecuacin diferencial x(t) + 0 x(t) = 0
o

13.2. El oscilador armnico simple . . . . . .


o
13.3. El oscilador armnico atenuado . . . . .
o
13.4. El oscilador armnico forzado . . . . . .
o
13.5. Osciladores armnicos acoplados . . . .
o
13.6. Modos normales de una cuerda . . . .
13.7. Problemas . . . . . . . . . . . . . . . . .
13.8. Solucin a algunos de los problemas . .
o

.
.
.
.
.
.
.
.

.
.
.
.
.
.
.
.

.
.
.
.
.
.
.
.

.
.
.
.
.
.
.
.

.
.
.
.
.
.
.
.

.
.
.
.
.
.
.
.

.
.
.
.
.
.
.
.

.
.
.
.
.
.
.
.

.
.
.
.
.
.
.
.

.
.
.
.
.
.
.
.

.
.
.
.
.
.
.
.

.
.
.
.
.
.
.
.

.
.
.
.
.
.
.
.

.
.
.
.
.
.
.
.

.
.
.
.
.
.
.
.

.
.
.
.
.
.
.
.

.
.
.
.
.
.
.
.

475
475
478
481
485
488
493
498
512


INDICE GENERAL

14.Ondas
14.1. La ecuacin de ondas . . . . . . . . . . . . . .
o
14.2. Solucin de la ecuacin de ondas . . . . . . .
o
o
14.3. Ondas estacionarias en una cuerda de largo L
14.4. Desarrollo de Fourier . . . . . . . . . . . . .
14.5. Problemas . . . . . . . . . . . . . . . . . . . .
14.6. Solucin a algunos de los problemas . . . . .
o

.
.
.
.
.
.

.
.
.
.
.
.

.
.
.
.
.
.

.
.
.
.
.
.

.
.
.
.
.
.

.
.
.
.
.
.

.
.
.
.
.
.

.
.
.
.
.
.

.
.
.
.
.
.

.
.
.
.
.
.

.
.
.
.
.
.

.
.
.
.
.
.

.
.
.
.
.
.

.
.
.
.
.
.

518
518
520
528
531
533
542

15.Ondas sonoras
15.1. Propagacin del sonido . . . . . . . .
o
15.2. Velocidad del sonido . . . . . . . . .
15.3. La ecuacin de ondas . . . . . . . . .
o
15.4. Frecuencia . . . . . . . . . . . . . . .
15.5. Intensidad . . . . . . . . . . . . . . .
15.6. Propagacin del sonido . . . . . . . .
o
15.7. Timbre . . . . . . . . . . . . . . . . .
15.8. Consonancia y disonancia . . . . . .
15.9. Oscilaciones de una columna de aire
15.10. fecto Doppler . . . . . . . . . . . .
E
15.11. roblemas . . . . . . . . . . . . . . .
P
15.12. olucin a algunos de los problemas
S
o

.
.
.
.
.
.
.
.
.
.
.
.

.
.
.
.
.
.
.
.
.
.
.
.

.
.
.
.
.
.
.
.
.
.
.
.

.
.
.
.
.
.
.
.
.
.
.
.

.
.
.
.
.
.
.
.
.
.
.
.

.
.
.
.
.
.
.
.
.
.
.
.

.
.
.
.
.
.
.
.
.
.
.
.

.
.
.
.
.
.
.
.
.
.
.
.

.
.
.
.
.
.
.
.
.
.
.
.

.
.
.
.
.
.
.
.
.
.
.
.

.
.
.
.
.
.
.
.
.
.
.
.

.
.
.
.
.
.
.
.
.
.
.
.

.
.
.
.
.
.
.
.
.
.
.
.

.
.
.
.
.
.
.
.
.
.
.
.

546
546
548
549
552
555
558
565
569
571
575
576
580

.
.
.
.
.
.
.
.
.
.
.
.

.
.
.
.
.
.
.
.
.
.
.
.

.
.
.
.
.
.
.
.
.
.
.
.

.
.
.
.
.
.
.
.
.
.
.
.

.
.
.
.
.
.
.
.
.
.
.
.

16.Relatividad especial
586
16.1. Problemas (Galileo-Newton) . . . . . . . . . . . . . . . . . . . . . . . . 586
16.2. Problemas (Einstein) . . . . . . . . . . . . . . . . . . . . . . . . . . . . 588

Cap
tulo 1

Expansiones y Trigonometr
a
versin 8 mayo 2012
o

En este primer cap


tulo se recopilarn algunos resultados de las Matemticas que
a
a
son bsicos para los cap
a
tulos que siguen. Primero, en la Sec. 1.1, revisaremos resultados relevantes para describir de manera aproximada cantidades f
sicas. Luego, en la
Sec. 1.2, recordaremos algunos resultados bsicos de trigonometr En este cap
a
a.
tulo,
particularmente en la Sec. 1.1, hay armaciones que no sern demostradas rigurosaa
mente. La idea es simplemente dar una intuicin acerca de resultados matemticos
o
a
cuya demostracin rigurosa es materia de cursos de Clculo especialmente, as como
o
a

introducir lenguaje formal que necesitaremos en el resto de nuestro curso.

1.1.

Expansiones y series

La F
sica es, ante todo, una ciencia experimental. Necesariamente, entonces, en
su estudio tendremos que lidiar con nmeros concretos, con mediciones. Por otro
u
lado, la Matemtica nos proporciona resultados abstractos que, en algn sentido,
a
u
representan la realidad. Por ejemplo, el nmero se puede conocer con innitos
u
decimales en principio, y el per
metro de un c
rculo de radio r siempre es 2r. Ingenuamente, uno podr pensar entonces que la circunferencia del planeta Tierra se
a
puede determinar con innitos decimales tambin. Sin embargo, ello no es cierto, pues
e
las imprecisiones experimentales al determinar el radio de la Tierra estropean toda
la innita precisin conque se conoce . Esto puede ser frustrante desde el punto de
o
vista matemtico, pero son numerosas las situaciones en que, en general, no es tan
a
importante tener un nmero exacto, pero s es crucial tener una buena aproximacin.
u

o
Por ejemplo, sabemos que un partido de ftbol signica esencialmente 45 minutos
u
de juego por lado, y 15 de descanso. Sabemos que nunca es exactamente as pero
,
no importa, porque esa informacin nos basta para tomar decisiones tales como si
o
alcanzamos a ir al supermercado antes de que comience el segundo tiempo, o si podemos ver el partido hasta el nal sin perdernos el comienzo de nuestra serie favorita
1

CAP
ITULO 1. EXPANSIONES Y TRIGONOMETR
IA

en otro canal. Tampoco necesitamos comprar una balanza de precisin innita para
o
controlar nuestro peso: ser absurdo que nos preocupramos porque hemos subido
a
a
una milsima de gramo desde el desayuno. . .
e
Cuando hacemos F
sica, nunca nos interesan los nmeros exactos, innitamente
u
precisos. Pero tener una buena aproximacin es fundamental siempre. Por ello dedio
caremos la primera seccin de este curso a exponer algunos resultados matemticos
o
a
aproximados, pero que utilizaremos frecuentemente en adelante.
Para comenzar, consideremos las expansiones siguientes:
(1 + x)1 = 1 + x
(1 + x)2 = 1 + 2x + x2
(1 + x)3 = 1 + 3x + 3x2 + x3
(1 + x)4 = 1 + 4x + 6x2 + 4x3 + x4
(1 + x)5 = 1 + 5x + 10x2 + 10x3 + 5x4 + x5
En general, para un entero n positivo arbitrario, la expansin del binomio (1 + x)n
o
puede escribirse en la forma
n (n 1) 2 n (n 1) (n 2) 3
n
x+
x +
x
1!
2!
3!
n (n 1) (n 2) (n 3) 4
x + + nx(n1) + xn ,
+
4!

(1 + x)n = 1 +

(1.1)

donde n! 1 2 3 (n 1) n, es decir, el producto de los n primeros nmeros


u
enteros, y se denomina el factorial del nmero entero n. Se dene tambin el factorial
u
e
de cero: 0! 1. (Esto se hace por razones de conveniencia matemtica, aunque en este
a
caso ya no tiene sentido hablar del factorial como el producto de los cero primeros
nmeros enteros.)
u
La expansin (1.1) es vlida para cualquier valor de x y cualquier valor de n
o
a
entero no negativo. Observamos adems que los coecientes de dicha expansin son
a
o
simtricos: los coecientes de la potencia ms baja (x0 ) y la ms alta (xn ) son 1; los
e
a
a
coecientes de la segunda potencia ms baja (x1 ) y la segunda ms alta (x(n1) ) son
a
a
n, etc. Adems, la expansin consta de un nmero nito de trminos, exactamente
a
o
u
e
igual a n + 1.
Lo anterior se puede entender fcilmente, ya que (1.1) es simplemente el producto
a
de n factores iguales. Lo interesante es que es posible generalizar dicha expresin al
o
caso en que el exponente no es entero, sino un nmero real arbitrario. En efecto,
u
en ese caso
( 1) 2 ( 1) ( 2) 3

x+
x +
x
1!
2!
3!
( 1) ( 2) ( 3) 4
x + .
+
4!

(1 + x) = 1 +

(1.2)

CAP
ITULO 1. EXPANSIONES Y TRIGONOMETR
IA

Observemos que si es un entero n, positivo o cero, (1.2) se reduce efectivamente a


(1.1), por cuanto existe un factor ( n) que anula todos los trminos desde en n + 1e
simo en adelante. [Naturalmente, esto no demuestra que (1.2) es la generalizacin
e
o
correcta de (1.1), pero es un indicio de que puede serlo.] As cuando es un entero
,
no negativo (1.2) tiene una cantidad nita de trminos, y siempre se puede calcular,
e
independiente del valor de x. Sin embargo, cuando es cualquier otro nmero (entero
u
negativo o un nmero real arbitrario), (1.2) posee innitos trminos.
u
e
Lo anterior es un problema, porque en general una suma innita de trminos
e
puede ser un nmero innito. Por ejemplo, 1 + 1 + 1 + es innito. Pero tambin
u
e
podemos construir sumas que, aunque consten de innitos trminos, sean un nmero
e
u
sica no nos interesan las cantidades
nito: 1 + 0.1 + 0.001 + 0.0001 + = 1.1. En F
innitas. Toda cantidad medible es un nmero nito. Qu signica entonces (1.2),
u
e
considerando que a veces el lado derecho puede ser innito o, en general, no estar bien
denido? En lenguaje tcnico, se dice que la serie (suma innita) 1 + 0.1 + 0.001 +
e
0.0001 + converge, esto es, es una serie convergente. La serie 1 + 1 + 1 + , en
tanto, no converge, es decir, es una serie no convergente.
Matemticamente es posible encontrar criterios de convergencia, de modo que
a
es posible saber de antemano si una serie dada converge o no. Un criterio sencillo
es notar si cada trmino es menor en mdulo que el anterior. Es claro que si tengo
e
o
innitos trminos iguales, o si dichos trminos son cada uno mayor que el anterior, la
e
e
suma de todos ellos debe ser innita. En cambio, si cada trmino es menor en mdulo
e
o
que el anterior, existe al menos alguna posibilidad de que el resultado sea un nmero
u
nito (as sucede al menos en nuestro segundo ejemplo, 1 + 0.1 + 0.001 + 0.0001 +

). Sin embargo, no siempre sumas innitas con trminos cada vez menores son
e
convergentes. Por ello, se dice que esta condicin (cada trmino menor en mdulo
o
e
o
que el anterior) es necesaria, pero no suciente. (Vale decir, toda serie convergente
satisface esta condicin, pero no todas las series que la satisfacen son convergentes.
o
En la prctica, esto signica que es un criterio denitivo para establecer que una serie
a
es no convergente, pero no es conclusivo para establecer que converja.)
De lo dicho hasta ahora, entonces, se sigue que lo que necesitamos es que la
expresin (1.2) sea convergente para que tenga sentido. Por ejemplo, si = 1/2,
o

u
el lado izquierdo de (1.2) es 1 + x, un nmero completamente normal (mientras
x > 1). Sin embargo, el lado derecho de (1.2) es una serie, y no necesariamente
converge. De hecho, se puede mostrar que, para un arbitrario, converge slo si
o
|x| < 1.
Por ejemplo, calculemos los lados derecho e izquierdo de (1.2) para = 1/2, con
dos valores distintos de x.
a) Si x = 3, el lado izquierdo es
(1 + x) =

4=2.

CAP
ITULO 1. EXPANSIONES Y TRIGONOMETR
IA

El lado derecho, en tanto, es


1
1 1
1
1
3
1 1
3 9 27
1+ 3+

32 +

33 + = 1+ + +
2
2 2
2
6 2
2
2
2 8 16
Sumando los cuatro primeros trminos de la serie, el resultado es 3.07, bastante
e
alejado de 2. Uno podr pensar que esto mejorar al sumar ms trminos, pero no
a
a
a e
es as con diez trminos, por ejemplo, el resultado es 155.68. Y as sucesivamen,
e

te, cada vez ms alejando del resultado correcto. Podemos ver que efectivamente
a
la serie no converge para este valor de x > 1.

b) Si x = 0.1, en cambio, el lado izquierdo es 1.1 1.04880884817015 . . ., y el


lado derecho, al sumar los cuatro primeros trminos de la serie, por ejemplo, es
e
1.0488125, bastante parecido al resultado exacto; y con los diez primeros trminos
e
es 1.04880884817101, an ms parecido. Vemos que |x| < 1, y la serie no slo
u
a
o
converge, sino que converge al resultado correcto.
Estamos acostumbrados a que el signo = signique que dos cantidades son exactamente iguales, pero cuando tenemos series innitas hay que tomar al signo igual
con cuidado: la igualdad existe slo si la serie converge. De otro modo, el lado derecho
o
de (1.2) ni siquiera se puede usar porque no tiene sentido, aunque, para los mismos
valores de x y , el lado izquierdo sea un nmero perfectamente razonable.
u
Otro ejemplo: para = 1, con (1.2) se obtiene la llamada serie geomtrica:
e
(1 x)1 =

1
= 1 + x + x2 + x3 + x4 +
1x

(1.3)

Nuevamente, en este caso, si bien el lado izquierdo est bien denido para cualquier
a
valor de x, el lado derecho slo da un resultado nito si |x| < 1.
o
Para x = 1/2 el lado izquierdo es igual a 2, mientras que el lado derecho da la
serie
1
1 1 1
+
1+ + + +
2 4 8 16
que, obviamente, al sumarla, tambin da 2. Si no estamos convencidos, podemos
e
recurrir al siguiente argumento geomtrico: representando, como es tradicional, la
e
unidad por un cuadrado, y sus fracciones por subreas de l, podemos ver que la
a
e
suma de los 5 primeros trminos de la serie geomtrica es:
e
e

1/8
1/16

1/2
1/4

CAP
ITULO 1. EXPANSIONES Y TRIGONOMETR
IA

Claramente, al continuar sumando trminos de la serie geomtrica, se obtienen


e
e
cuadrados cada vez ms pequeos, cada uno de rea igual a la mitad del anterior,
a
n
a
y podemos ver que con innitos trminos terminaremos llenando completamente el
e
segundo cuadrado.
Para x = 1/10, en tanto, el lado izquierdo es igual a 10/9, mientras que el
lado derecho da la serie (que ya hab
amos utilizado antes como ejemplo de serie
convergente)
1 + 0.1 + 0.01 + 0.001 + = 1.1111 . . . ,
que es el desarrollo decimal de 10/9.
Seamos un poco ms generales. Hasta el momento slo hemos resuelto ejemplos
a
o
numricos, usando, sin demostrar, que (1.2) es correcta. Sin embargo, podemos dee
mostrar, al menos para la serie geomtrica ( = 1), que lo es. Evaluemos primero
e
la suma nita
S N = 1 + x + x2 + x3 + + xN ,
que corresponde a los primeros N +1 trminos de la serie geomtrica. Deseamos saber
e
e
qu le ocurre a SN cuando N crece. Para calcular SN restemos de esta serie la misma
e
serie, pero multiplicada por x, es decir:
SN
x SN

= 1 + x + x2 + x3 + + xN

x + x2 + x3 + + xN + xN +1 .

Al restar, al lado izquierdo queda (1 x) SN , mientras que al lado derecho queda


1 xN +1 , o sea,
(1 x) SN = 1 xN +1 .
Despejando SN se obtiene

1 xN +1
.
1x
Ahora, si hacemos N cada vez ms grande, es decir lo hacemos tender a innito, el
a
N +1 es distinto dependiendo de si |x| es mayor o menor que 1, Si
comportamiento de x
|x| > 1, entonces cada potencia de x es ms grande que la anterior, y por lo tanto el
a
lado derecho es innito: la serie no converge. En cambio, si |x| < 1, cada potencia de
x es ms pequea que la anterior, de modo que, cuando N es innitamente grande,
a
n
xN +1 es innitamente pequeo. Formalmente, decimos que el lmite de xN +1 , cuando
n

N tiende a innito, es 0, y lo denotamos as


:
SN =

l xN +1 = 0 .
m

Por lo tanto,
l SN = 1 + x + x2 + x3 + =
m

resultado consistente con (1.3).

1
,
1x

CAP
ITULO 1. EXPANSIONES Y TRIGONOMETR
IA

Para determinar completamente las condiciones de convergencia de la serie falta


considerar los casos x = 1 y x = 1. Si x = 1 es evidente que no hay convergencia,
pues la serie geomtrica es una suma innita de unos. Tambin ello es evidente al
e
e
reemplazar x = 1 en el lado izquierdo de (1.3). Pero, qu pasa si x = 1? El lado
e
izquierdo de (1.3) es 1/2, un nmero nito. Revisemos entonces la serie del lado
u
derecho. Si x = 1, en cambio, tenemos una suma alternada de 1 y 1, de modo que
SN = 0 si N es par, y SN = 1 si N es impar. Tenemos entonces que la suma nita,
SN , a medida que N crece, salta eternamente entre 0 y 1. La serie no se hace innita,
pero tampoco tiende a algn valor denido, y tampoco converge.
u
Vemos entonces que hay dos modos en que una serie puede ser no convergente:
una, es que la serie explote; la otra, es que la serie, al ir agregando cada vez nuevos
trminos, no alcance nunca un valor dado.
e
Con esto terminamos de mostrar que la expresin (1.2) es, al menos para = 1,
o
convergente slo para |x| < 1.
o
Volvamos ahora a (1.2) con = 1/2. En ese caso se obtiene
(1 + x)1/2 =

1
1
1
1 + x = 1 + x x2 + x3
2
8
16

Ya hab
amos calculado este caso con x = 0.1. Repitamos el clculo, pero con ms
a
a
detalle. Observemos lo que ocurre con la suma en el lado derecho de (1.2), a medida
que consideramos cada vez ms trminos. La tabla adjunta muestra este anlisis para
a e
a
x = 0.1:
lado izquierdo
1.04880884817

lado derecho
1.0
1.05
1.04875
1.0488125

No de trminos
e
1
2
3
4

error
4.9 %
0.11 %
0.0059 %
0.00037 %

Esto es muy importante. La tabla anterior nos permite vericar que no es necesario

tener una calculadora para saber el valor de 1.1, sino que basta con saber las operaciones bsicas. Sumando slo trminos de una serie que en principio tiene innitos
a
o 4 e
trminos, es posible calcular 1.1 con cuatro decimales exactos. Esto es extraordinae
rio. La expansin en serie (1.2) no es slo un resultado matemtico abstracto, formal,
o
o
a
sino que nos da el gran poder de ser capaces de calcular, en este caso, ra cuadradas
ces
con varios decimales exactos, sin necesidad de sumar innitos trminos, y sin usar
e
una calculadora. Recordemos: en F
sica no nos interesan las cantidades con innitos
decimales, porque no son medibles con instrumentos reales; pero s nos interesa tener

buenas aproximaciones, y acabamos de encontrar una.


De hecho, se puede vericar (Ejercicio para el lector) que mientras ms pequeo
a
n
es x, la convergencia de la serie truncada al resultado exacto es aun ms rpida, de
a a

modo que son necesarios an menos trminos para calcular 1 + x con gran preu
e
cisin. Esto es, para x sucientemente pequeo, podemos decir, sin gran error, que
o
n

CAP
ITULO 1. EXPANSIONES Y TRIGONOMETR
IA

1 + x = 1 + x/2 (slo los dos primeros trminos de la serie innita), y para todos
o
e
los efectos prcticos ello ser correcto. Este resultado es muy importante, y lo usarea
a
mos frecuentemente en el resto de este curso. Queda, por cierto, an la pregunta de
u
qu signica sucientemente pequeo. La respuesta en detalle depende del problee
n
ma que se est trabajando, pero en general se puede decir que un 5 % es adecuado.
e
(Para la serie que analizamos, x 0.05 ser sucientemente pequeo, aunque vemos
a
n
que x = 0.1 ya funciona bastante bien.)
Ejemplos:
1. Sea = 0 un nmero real arbitrario y evaluemos [(1+x) 1]/x para valores de
u
x muy pequeos. Observe que para valores de x cada vez ms pequeos, tanto
n
a
n
el numerador como el denominador tienden a cero, de modo que la expresin,
o
en principio, carece de sentido. Pero veremos que, gracias a la aproximacin
o
derivada de (1.2), podemos calcular esta expresin sin problema.
o
De acuerdo a la ecuacin (1.2), para x muy pequeo vale la aproximacin
o
n
o
(1 + x) 1 + x
(o sea, estamos despreciando todos los trminos de la serie excepto los dos
e
primeros). Usando esta aproximacin se encuentra que (para x muy pequeo)
o
n
1+x1
x
(1 + x) 1

=
= .
x
x
x
Verique numricamente este resultado usando una calculadora.
e
2. Problema resuelto en clases: 1.2 (Cap
tulo 1, Problema 2)
Mejor an: Anlogo a 1.2, pero con (x + )3 .
u
a

Algunas aproximaciones que se obtienen a partir de la ecuacin (1.2) para |x| pequeo,
o
n
que se usarn con frecuencia, y conviene tener siempre presentes, son:
a
(1 + x) 1 + x ,
1
1x ,
1+x
1
1+x ,
1x

x
1+x1+ .
2

(1.4)
(1.5)
(1.6)
(1.7)

CAP
ITULO 1. EXPANSIONES Y TRIGONOMETR
IA

Concluyamos esta seccin con unas palabras acerca de la notacin. Para abreviar
o
o
la escritura de series, se usa frecuentemente la letra griega sigma mayscula ( ).
u
Ilustramos el uso de este s
mbolo con algunos ejemplos:
6

j = 1 + 2 + 3 + 4 + 5 + 6 = 21 ,
j=1
4

j 2 = 12 + 22 + 32 + 42 = 30 ,
j=1
2

j k = j 2 + j 1 + 1 + j + j 2 ,
k=2

n=0

1
2

=1+

1 1 1
+ + + = 2 .
2 4 8

En todas estas expresiones hay un


ndice (j, k, n, etc.), que var slo entre nmea o
u
ros enteros. En el s
mbolo
se indican los valores m
nimo y mximo que puede
a
tomar. Aunque en principio uno podr usar cualquier letra para denotar estos ndia

ces de suma, lo usual es utilizar slo letras latinas minsculas, desde i en adelante
o
u
(exceptuando la o, que se puede confundir con el nmero 0).1
u

1.2.

Elementos de trigonometr
a

Consideremos los tringulos rectngulos (ABC) y (AB C ) mostrados en


a
a
la gura 1.1. De acuerdo a un teorema de la geometr elemental, la razn (entre
a
o
o
e
o
trazos) AC : AB es igual a la razn AC : AB , dependiendo sta slo del valor del
ngulo . Se ha convenido llamar a tal razn cos ; o sea, en un tringulo rectngulo,
a
o
a
a
el cuociente entre el cateto adyacente y la hipotenusa dene el coseno del ngulo que
a
forman esos dos lados:
AC
longitud del lado adyacente
.
=
longitud de la hipotenusa
AB
Tambin el cuociente entre el cateto opuesto al ngulo y la hipotenusa es indee
a
pendiente del tamao del tringulo rectngulo y slo depende del valor de . A esta
n
a
a
o
razn se la llama seno del ngulo, tenindose
o
a
e
cos =

Conviene resaltar la conveniencia de no ser creativo con la notacin matemtica. Un texto


o
a
cient
co se puede convertir rpidamente en inentendible si utiliza notacin no convencional. La
a
o
notacin matemtica debe facilitar la comunicacin de resultados, no entorpecerla.
o
a
o

CAP
ITULO 1. EXPANSIONES Y TRIGONOMETR
IA

9
B

Figura 1.1

BC
longitud del lado opuesto
=
.
longitud de la hipotenusa
AB
Es util denir tambin la funcin tangente:

e
o
sin =

tan

sin
longitud del lado opuesto
=
.
longitud del lado adyacente
cos

Evaluemos sin2 + cos2 . Se tiene:


2

cos2 + sin2 =
=

AC
BC
+
AB
AB
2 + (BC)2
(AC)
.
(AB)2

Pero, de acuerdo al teorema de Pitgoras, (AC)2 + (BC)2 = (AB)2 , luego


a
cos2 + sin2 = 1 .
Observemos que cos y sin son ambos menores o iguales que 1. Esto se puede
ver tanto de la igualdad recin obtenida como de la denicin de coseno y seno.
e
o
Dos relaciones trigonomtricas importantes son:
e
sin( + ) = sin cos + sin cos

(1.8)

cos( + ) = cos cos sin sin .

(1.9)

Demostremos al menos una de ellas; la primera. Para ello consideremos la gura 1.2.
Partiendo del tringulo (ABC), prolongamos el lado BC y gracamos las alturas
a

CAP
ITULO 1. EXPANSIONES Y TRIGONOMETR
IA

10

Figura 1.2
CD y AE. Note que el ngulo ACE resulta ser igual a + . El rea de un
a
a
tringulo es la mitad del producto de su base por la altura. De la gura 1.2, para el
a
rea del (ABC), obtenemos
a

2 Area [ (ABC)] = BC EA = AB CD .
En la ultima ecuacin hemos escrito el producto base por altura del tringulo

o
a
(ABC) de dos maneras distintas: en la primera igualdad, BC es la base y EA
la altura, mientras que en la segunda, AB es la base y CD la altura.
Luego,
sin( + ) =

(AD + DB) CD 1
AE
AB CD 1
AD CD DB CD
=
=
=

.
AC
BC AC
BC
AC
AC BC BC AC

Usando las deniciones de seno y coseno, se deduce nalmente que


sin( + ) = sin cos + sin cos .
Como casos particulares de las ecuaciones (1.8) y (1.9), se encuentra
cos(2) = cos2 sin2

(1.10)

sin(2) = 2 cos sin .

(1.11)

Existen muchas identidades trigonomtricas de este tipo que resultan ser utiles pae

ra llevar adelante diferentes tipos de clculos. Dejamos que el lector demuestre las
a
siguientes identidades:
sin sin = 2 sin

cos

(1.12)

cos + cos = 2 cos

+
2

cos

(1.13)

CAP
ITULO 1. EXPANSIONES Y TRIGONOMETR
IA

11

^
y
A

xp

^
x

yp

Figura 1.3
+
2

cos cos = 2 sin


tan 2 =

sin

2 tan
.
1 tan2

(1.14)
(1.15)

Problema resuelto en clases: 1.10


La denicin del seno y coseno que hemos dado es vlida para ngulos entre 0
o
a
a
y 90 grados. Para denir estas funciones para otros ngulos es conveniente considerar
a
un c
rculo de radio R = 1 centrado en el origen (ver gura 1.3). Por convencin, los
o
ngulos se miden desde el eje x en el sentido contrario a los punteros del reloj.
a

Consideremos el punto A sobre el c


rculo, formando un ngulo con el eje x.
a

Usando el hecho que la hipotenusa vale 1, es fcil convencerse de que las coordenadas
a
x e y del punto A coinciden con los valores de cos y sin , respectivamente.
Es sta la propiedad que se usa para denir el valor del seno y coseno para
e
cualquier ngulo . El procedimiento es el siguiente: i) Encontrar el punto P sobre
a
el c
rculo que forma un ngulo con el eje x (en la gura 1.3, esto se muestra para
a

un ngulo > 180 ; ii) luego, proyectar el punto P sobre los ejes para encontrar xp
a
e yp . Entonces cos = xp y sin = yp . Por ejemplo, si en la gura 1.3 = 30 y
= 210 (de modo que los puntos A y P estn unidos por una recta que pasa por el
a

origen), entonces cos(210 ) = 3/2 = 0, 8660 . . . y sin(210 ) = 1/2. Es evidente


que, para todos los ngulos , siempre se cumple
a
1 cos 1
y
1 sin 1 .
La denicin anterior nos permite conocer el seno y el coseno de ngulos negativos:
o
a

CAP
ITULO 1. EXPANSIONES Y TRIGONOMETR
IA

12

^
y

xp

yp

^
x
P

De la gura es inmediato que


sin() = sin ,

cos() = cos .

Se dice en este caso que el seno es una funcin impar (cambia de signo cuando el
o
argumento lo hace), y el coseno es una funcin par (no cambia de signo cuando el
o
argumento lo hace). Esta propiedad, por ejemplo, permite deducir el seno y el coseno
de la diferencia de dos ngulos. Basta con usar (1.8) y (1.9), y las propiedades de
a
paridad anteriores, para encontrar que
sin( ) = sin cos() + cos sin()
= sin cos cos sin .

Es usual medir los ngulos en grados, pero aunque uno puede tener mayor intuia
cin con esta unidad, precisamente porque es ms usual en la vida diaria (bsicamente
o
a
a
es parte de la cultura general que 90 corresponde a un ngulo recto, uno puede imagia
narse que 1 es un ngulo pequeito, etc.), los grados no son una unidad muy prctica
a
n
a
desde el punto de vista f
sico, porque nacen por una convencin arbitraria, como es
o
dividir una circunferencia en 360 partes iguales. 360 es un nmero completamente
u
arbitrario, y podr haber sido cualquier otro (por ejemplo, dividir el ngulo recto en
a
a
100, que da origen a los grados centesimales o gradianes). Resulta ms conveniente
a
denir una unidad angular que no tenga dicha arbitrariedad.
Consideremos un ngulo recto, de 90 :
a

s
r

CAP
ITULO 1. EXPANSIONES Y TRIGONOMETR
IA

13

La longitud del arco subtendido por es un cuarto del per


metro de la circunferencia completa, s = r/4. Luego, no importa qu radio r hayamos elegido, el
e
cuociente s/r es independiente del radio (en este caso, s/r = /4). Breves momentos
de reexin deber convencernos de que lo anterior debe ser cierto para cualquier
o
an
subdivisin de . Independiente del ngulo, s/r es independiente del radio, y por
o
a
tanto depende slo del ngulo. Es natural entonces denir s/r como el ngulo en s
o
a
a
.
Cuando expresamos el ngulo de esta manera, decimos que est medido en radianes
a
a
(abreviada rad).
Denimos, pues, el valor del ngulo , en radianes, como el largo del arco suba
tendido sobre el c
rculo unitario desde donde lo cruza el eje x hasta el punto A (ver

gura 1.3),
s
= .
r
De acuerdo a la denicin, un ngulo de 360 , o sea, la circunferencia completa,
o
a
corresponder a un ngulo igual a 2 rad. El ngulo recto es igual a /2 rad; el
a
a
a
extendido a rad. No es dif vericar que
cil
1 rad =

360
= 57.3 .
2

Si bien es cierto se puede abreviar radianes por rad, debemos notar que en realidad
los radianes son una unidad adimensional, pues se denen como el cuociente de dos
longitudes. Por tanto, en realidad es innecesario escribir expl
citamente rad, a menos
que sea necesario para evitar confusin, o para marcar la diferencia con otras unidades,
o
como en la expresin anterior. As es tambin correcto, y es la forma preferida, decir
o
,
e
que el ngulo recto es simplemente igual a /2.
a
Para llegar al punto P (gura 1.3) originalmente se recorri un ngulo desde
o
a
el eje x positivo. Al continuar y dar una vuelta completa para volver al punto P ,

habremos recorrido desde el eje x un ngulo 2 + . Sucesivas rotaciones nos llevarn

a
a
nuevamente al punto P , habindose recorrido ngulos 4 + , 6 + , etc. Cada vez
e
a
que, desde el eje x positivo, recorremos un ngulo ms un mltiplo de 2, estaremos

a
a
u
en el punto P . Se trata de un movimiento que se repite y se dice que es peridico en
o
el ngulo , con per
a
odo igual a 2. Se tiene, en general, que, para cualquier ngulo
a
,
cos( + n 2) = cos
y
sin( + n 2) = sin ,
donde n es un entero. Relaciones se pueden obtener ya sea usando geometr sobre el
a
c
rculo trigonomtrico (Fig. 1.3), o algebraicamente a partir de (1.8) y (1.9):
e
sin( ) = sin
sin(/2 ) = cos

CAP
ITULO 1. EXPANSIONES Y TRIGONOMETR
IA

sin

14

cos

1
Figura 1.4

B
c
s

Figura 1.5
cos( ) = cos
cos(/2 ) = sin
cos( + /2) = sin
sin( + /2) = cos .
Finalmente, observemos que si variamos en la Fig. 1.3 de modo que P recorra
todo el c
rculo unitario varias veces, podemos gracar sus proyecciones vertical y
vertical, que correspondern precisamente al grco de las funciones seno y coseno
a
a
(ver gura 1.4). Se aprecia que el seno y el coseno son escencialmente la misma
funcin, pero desplazadas una respecto a la otra en /2, lo cual es consistente con las
o
relaciones algebraicas anteriores para el seno y el coseno de /2 .
Cuando el argumento (en radianes) de una funcin trigonomtrica es muy peo
e
queo, sta puede aproximarse con una expresin simple. En efecto, consideremos
n e
o
el tringulo rectngulo ABC mostrado en la gura 1.5. A medida que decrece, el
a
a
cateto opuesto a se hace cada vez ms parecido al arco de c
a
rculo s con centro en A.
Usando la denicin de la funcin seno se tiene
o
o
sin =

a
s

.
c
c

CAP
ITULO 1. EXPANSIONES Y TRIGONOMETR
IA

15

Pero el cuociente s/c es precisamente el ngulo en radianes, luego, para ngulos


a
a
pequeos (y stos expresados en radianes)
n
e
sin .

(1.16)

Esto signica que al gracar sin como funcin de , deber obtenerse aproxio
a
madamente una l
nea recta. Ello ocurre efectivamente, como se aprecia en la Fig. 1.4,
cerca del origen. Pero evidentemente la aproximacin falla para ngulos sucienteo
a
mente grandes.
Usando (1.16) es posible tambin obtener una expresin aproximada para cos .
e
o
Sabemos que
cos2 = 1 sin2 .
Luego, para ngulos pequeos
a
n
cos2 1 2 ,
o sea,

1
1 2 1 2 .
2

cos

(1.17)

Ejemplo:
Evale, usando una calculadora, las funciones sin y cos para = 5 . Compare
u
los valores obtenidos con aqullos que resultan de usar las expresiones aproximadas
e
escritas ms arriba.
a
Ingresando el valor = 5 = 5 2/360 rad en una calculadora, obtenemos:
sin 5 = 0.0871557
y
cos 5 = 0.9961947 .
Si ahora hacemos uso de las expresiones aproximadas, obtenemos

sin 5
cos 5 = 1

5 2
= 0.087266
360

5 2
360

= 0.9961923

Note que los valores aproximados dieren poco de los obtenidos con la calculadora.
Para el coseno el error es inferior al 0.003 %. (La razn por la cual el error en el
o
coseno es mayor es porque hemos usado una expansin hasta orden 2 , que es mejor
o
que para el seno, donde la expansin es hasta una potencia menor, .)
o

CAP
ITULO 1. EXPANSIONES Y TRIGONOMETR
IA

16

Cabe destacar que las funciones sin y cos pueden ser expresadas como una
suma innita de trminos proporcionales a diferentes potencias del ngulo :
e
a
cos = 1

2 4 6
+

+ ,
2!
4!
6!

(1.18)

3 5 7
+

+ .
3!
5!
7!
Para || 1, estas series convergen rpidamente, lo que permite representar las
a
funciones seno y coseno con pocos trminos, como comprobamos con el ejemplo de la
e
pgina anterior.
a
Es importante destacar que estas expansiones en serie, y por tanto las aproximaciones (1.16) y (1.17), slo tienen sentido si el ngulo se mide en radianes, lo cual es
o
a
otro argumento para preferir esta unidad en vez de los grados.
sin =

Ejemplo:
Representemos en un mismo grco, para el intervalo t [, 2] , las siguientes
a
cinco funciones:
i)

f0 (t) = cos t

ii)

f1 (t) = 1

iii)

f2 (t) = 1 t2 /2!

iv)

f3 (t) = 1 t2 /2! + t4 /4!

v)

f4 (t) = 1 t2 /2! + t4 /4! t6 /6!

Observe que de acuerdo a la ecuacin (1.18), las funciones f1 (t), f2 (t), etc., para t
o
pequeo son aproximaciones cada vez mejores de f0 (t) = cos t. Este comportamiento
n
se observa claramente en la gura 1.6 donde se han gracado las diversas funciones.
Problema resuelto en clases: 1.20
Funciones trigonomtricas inversas
e
En ocasiones, lo que conocemos es x = sin , y lo que se desea conocer es el ngulo
a
. Por ejemplo en la Fig. 1.1, si conocemos los lados del tringulo, es posible conocer
a
el ngulo . Para ello necesitamos la nocin de funcin inversa. Dada una funcin
a
o
o
o
y = f (x), la funcin inversa es aquella funcin que, dado un valor de y, entrega el
o
o

CAP
ITULO 1. EXPANSIONES Y TRIGONOMETR
IA

17

f ( t)
2

f3 ( t)

f1 ( t)

1
2

f2 ( t)

2
f 0 ( t ) = cos( )
t

f4 ( t)

Figura 1.6
valor de x correspondiente. En el caso del seno y el coseno, sus funciones inversas son
el arcoseno y el arcocoseno, respectivamente, y se denotan:
= arcsin x ,

= arc cos x .

Desafortunadamente, el seno no es una funcin montona del ngulo (Fig. 1.4), y


o
o
a
por lo tanto, dado un valor y (|y| < 1), no existe un unico valor de tal que sin = y.

Esto signica que el arcoseno es una funcin multivaluada. Esto es un problema desde
o
el punto de vista matemtico, formal, porque una funcin, por denicin, deber
a
o
o
a
ser monovaluada. Adems es un problema prctico, porque es claro que, dado un
a
a
tringulo rectngulo como en la Fig. 1.4, si los lados son conocidos, el ngulo
a
a
a
deber ser unico. Por otro lado, si tomamos cualquier calculadora y calculamos el
a

arcoseno de un nmero arbitrario (mientras tenga mdulo menor que 1), obtendremos
u
o
en pantalla un unico resultado. Cmo hacemos para que una funcin multivaluada

o
o
se vuelva monovaluada, y cmo elegimos, entre los muchos valores, el correcto?
o
La estrategia es sencilla. Si observamos la Fig. 1.4, notamos que si nos restringimos a valores de entre /2 y /2, el seno es una funcin montona creciente
o
o
de , y por lo tanto el arcoseno es una funcin monovaluada. Anlogamente, para
o
a
entre 0 y el coseno es una funcin montona decreciente, y el arcocoseno es moo
o
novaluado. Naturalmente, podr
amos haber tomado otros intervalos de . Con otras
elecciones, el arcoseno y arcocoseno tendr tambin valores unicos, pero distintos
an
e

a los obtenidos con los primeros intervalos escogidos. A cada uno de los intervalos
en los cuales las funciones quedan monovaluadas se les denomina ramas. Entonces,
la manera de convertir una funcin multivaluada en monovalueada es escogiendo una
o
rama adecuada. Qu signica adecuada? Depende de la aplicacin. Lo usual para
e
o
el arcoseno y el arcoseno son las ramas que escogimos antes, [/2, +/2] para el

CAP
ITULO 1. EXPANSIONES Y TRIGONOMETR
IA

18

arcoseno, [0, ] para el arcocoseno. Es la eleccin de las calculadoras tambin. Y es


o
e
la eleccin correcta para el caso de la Fig. 1.1, ya que sabemos que en el tringulo
o
a
rectngulo 0 < < /2. Pero cuidado!, no hay ninguna manera, en general, de asea
gurar que esta eleccin es la adecuada para cualquier problema f
o
sico, y por tanto la
eleccin de rama es una decisin que debe tomarse caso a caso. No hay que dejarse
o
o
engaar por el resultado que entrega una calculadora o un computador, y siempre
n
hay que cuestionarse si el nmero que nos da es la solucin f
u
o sicamente aceptable.
Algo similar ocurre cuando uno extrae ra
ces: puede ocurrir que la ra de 9 de
z
inters f
e sico sea 3 y no la solucin que entrega la calculadora (que es +3).
o
Ejercicio: Sea |x| 1 cierto valor dado y suponga que deseamos encontrar todos los
a
ngulos (en radianes) para los cuales cos = x. Suponga adems que hemos, de
a
alguna manera, encontrado una solucin = 0 (por ejemplo, el ngulo que muestra
o
a
la calculadora al evaluar arccos(x) ). Demuestre que todas las dems soluciones a
a
nuestro problema vienen dadas por = 0 + j 2 y = 0 + j 2, con j cualquier
valor entero.
Ejercicio: Sea |x| 1 cierto valor dado y suponga que deseamos encontrar todos los
a
ngulos (en radianes) para los cuales sin = x. Suponga adems que hemos, de
a
alguna manera, encontrado una solucin = 0 (por ejemplo, el ngulo que muestra
o
a
la calculadora al evaluar arccos(x) ). Demuestre que todas las dems soluciones a
a
nuestro problema vienen dadas por = 0 + j 2 y = ( 0 ) + j 2, con j
cualquier valor entero.
Por ser frecuentemente fuente de errores reiteramos lo dicho unos prrafos antes:
a
al evaluar funciones trigonomtricas inversas la solucin entregada por la calculadora
e
o
no es siempre la f
sicamente aceptable. El alumno debe asegurarse de que la respuesta
mostrada por la calculadora efectivamente resuelve completamente su problema, en
caso contrario, debe analizar si alguna de las otras soluciones, que se obtuvieron en
los dos ejercicios anteriores, sirve.

CAP
ITULO 1. EXPANSIONES Y TRIGONOMETR
IA

1.3.

19

Problemas

1. Evale las siguientes sumatorias


u
nm

S=

a)

n = 1, 2
m = 1, 2, 3

S=

b)

1
j=3,...,8
N

c)

S=
j=0

S=

d)

i, j = 1, . . . , 4
i>j

1
|i j|

Respuestas: a) 17 , b) 12 , c) N (N + 1)/2 , d) 13/3


2. Encuentre una expresin para [ (x+) x ]/, en el lmite en que tiende a
o

cero. En otras palabras, tiene un valor nito pero peque


nsimo (tan pequeo
n
como se quiera); al nal del clculo se permite poner = 0.
a
Usando una notacin y un lenguaje ms tcnico, el enunciado de este problema
o
a e
ser
a:
Evale
u

f (x) = l
m

1
[ (x + ) x ] .

Respuesta: f (x) = x1 .
cos(x + ) cos x

3. Evale
u
Respuesta:

para || 1 .

sin x.

4. Represente en forma cuidadosa, en un mismo grco, para el intervalo t


a
[1, 1] , las siguientes cuatro funciones:
a)

f0 (t) = 1/(1 t)

CAP
ITULO 1. EXPANSIONES Y TRIGONOMETR
IA

Figura 1.7

20

Figura 1.8

b)

f1 (t) = 1 + t

c)

f2 (t) = 1 + t + t2

d)

f3 (t) = 1 + t + t2 + t3

Observe que, de acuerdo a la ecuacin (1.3), f1 (t), f2 (t) y f3 (t) son sucesivao
mente aproximaciones cada vez mejores (para t pequeo) de la funcin f0 (t).
n
o

5. Demuestre las siguientes relaciones trigonomtricas:


e
(a)

sin =

(b)

tan( + ) =

(c)

sin + sin = 2 sin

tan
1 + tan2

tan + tan
1 tan tan
+
2

cos

6. Sea r el radio del c


rculo circunscrito de un pentgono regular (ver gura 1.7).
a
a) Cunto mide el ngulo interior (en radianes)?
a
a
b) Determine el largo del lado s en funcin de r.
o
c) Determine el rea del pentgono.
a
a
Respuestas: a) = 3/5 radianes ;

c) rea =
a

5
2

r 2 sin(2/5).

CAP
ITULO 1. EXPANSIONES Y TRIGONOMETR
IA

21

7. Una camionada de arena seca se descarga formando un cono de 4 metros de


dimetro. Si la densidad de la arena seca es =1.7 g/cm3 y el el ngulo del
a
a
cono (ver gura 1.8) es de = 32 , calcule la masa de la arena (en toneladas).
8. Encuentre todos los valores de x en el intervalo [5, +5] (cuando no se especica
nada se asume que las unidades son radianes) para los cuales se cumple la
relacin
o
3
sin x tan x = .
2
Respuesta: x = 4/3 , 2/3 , 2/3 , 4/3 .
9. Represente en un mismo grco, para t en el intervalo [, 2] , las siguientes
a
cuatro funciones:
a)

f0 (t) = sin t

b)

f1 (t) = t

c)

f2 (t) = t t3 /3!

d)

f3 (t) = t t3 /3! + t5 /5!

Aqu nuevamente f1 (t), f2 (t) y f3 (t) son sucesivamente aproximaciones cada

vez mejores (para t pequeo) de la funcin f0 (t).


n
o
10. Al incidir luz sobre una interfase, por ejemplo, al pasar del aire al vidrio o
viceversa, sta generalmente sufre un cambio de direccin (ver gura 1.9). Este
e
o
fenmeno se conoce con el nombre de refraccin de la luz. La ecuacin que
o
o
o
describe este fenmeno es la Ley de Snell:
o
v
sin
= aire ,
sin
vvidrio
donde vaire y vvidrio corresponden a la velocidad de la luz en el aire y el vidrio,
respectivamente. (Para el vidrio comn se tiene vaire /vvidrio 1.5 .)
u
a) Supongamos que un haz de luz incide sobre un vidrio de 2 cm de espesor,
con un ngulo de incidencia = 40 . Encuentre la distancia d por la cual
a
el haz de luz emergente se encontrar paralelamente desplazado respecto
a
al haz incidente (ver gura 1.10).
b) Considere ahora un haz de luz incidiendo sobre un prisma en la forma que
se muestra en la gura 1.11. Encuentre el ngulo para = 20 , 40 , 50
a
y 70 . Para qu ngulo = 0 se obtiene = 90 ? Para > 0 el haz de
ea
luz es reejado especularmente (como si fuese un espejo) por la supercie
interior del prisma, fenmeno conocido con el nombre de reexin total.
o
o

CAP
ITULO 1. EXPANSIONES Y TRIGONOMETR
IA

22

Figura 1.9

Figura 1.10

Figura 1.11

11. La gura 1.12 adjunta indica la diferencia entre un da sideral y un da solar.

Para facilitar la explicacin supongamos que es posible observar las estrellas


o
durante el d (Por supuesto que las estrellas estn all y de hecho los radioa.
a

astrnomos observan algunas de ellas.)


o
Para un observador en el Ecuador, el d solar es el per
a
odo que transcurre entre
dos pasos consecutivos del Sol por el zenit (posicin del Sol justo sobre nuestras
o
cabezas). El d sideral consiste en el mismo fenmeno pero que ahora ocurre
a
o
con una estrella muy lejana. La diferencia entre ambas deniciones se debe a la
traslacin de la Tierra alrededor del Sol. Determine el valor del ngulo que
o
a
se muestra en la gura y calcule la diferencia entre el d sideral y el d solar
a
a
en segundos.
12. Un tambor de 50 cm de radio y 1.5 m de largo se encuentra acostado y lleno
con parana hasta una altura h =60 cm (ver gura 1.13). Cuntos litros de
a
parana hay en el tambor?
13. La esfericidad de la Tierra fue postulada por Pitgoras y conrmada por Aristtea
o
les al observar la forma circular de la sombra que proyecta la Tierra en la supercie de la Luna durante un eclipse lunar.
El primer clculo que se conoce del radio de la Tierra se debe a Eratstenes (276
a
o

CAP
ITULO 1. EXPANSIONES Y TRIGONOMETR
IA

Figura 1.12

23

Figura 1.13

Figura 1.14
A.C.194 A.C.), quien a la fecha estaba a cargo del Museo de Alejandr El
a.
mtodo que us se bas en observar el ngulo con que inciden los rayos solares
e
o
o
a
sobre la supercie de la Tierra, el mismo d y a la misma hora, en dos lugares
a
separados entre s por una gran distancia. Los lugares elegidos fueron Siena (S)

(hoy Asun) y Alejandr (A).


a
a
Eratstenes sab que al mediod del 22 de junio el Sol ca verticalmente en
o
a
a
a
Siena, pues la luz se reejaba directamente en el fondo de una noria. El mismo
d a la misma hora, midi la sombra que proyectaba en Alejandr un alto
a,
o
a
obelisco, que le indic que los rayos solares formaban un ngulo de 7.2 con la
o
a
vertical (ver gura 1.14).
Dado que el Sol est a gran distancia de la Tierra se puede suponer que los rayos
a
que llegan a ambas ciudades son paralelos. Eso quiere decir que la separacin
o
angular entre Siena y Alejandr medida con respecto al centro de la Tierra es
a
tambin 7.2 (demustrelo). Sabiendo que la distancia entre Siena y Alejandr
e
e
a
(arco de c
rculo) es de aproximadamente 800 km, estime el radio de la Tierra.

CAP
ITULO 1. EXPANSIONES Y TRIGONOMETR
IA

24

Respuesta: Radio 6400 km. (El resultado que obtuvo Eratstenes en su poca
o
e
fue incorrecto, debido a la imprecisin con que estim la distancia entre los dos
o
o
lugares.)
14. Una persona ubicada en el punto P observa dos montaas que la rodean, una
n
a la derecha y la otra a la izquierda. Sean y los ngulos de elevacin,
a
o
respectivamente (ver gura 1.15). Si la montaa de la izquierda tiene una altura
n
h y la separacin entre las proyecciones de las cimas sobre el nivel de la supercie
o
terrestre es D, calcule la altura del otro monte.
15. En el ao 1752 los astrnomos Landale y Lacaille determinaron en Berl (B) y
n
o
n
en la ciudad del Cabo (C), a la misma hora, el ngulo entre la normal y la recta
a
entre su posicin y un punto predeterminado del borde de la Luna. Los ngulos
o
a
que determinaron fueron = 32.08 en Berl y = 55.72 en El Cabo. Amn
bas ciudades se ubican en el mismo meridiano y se encuentran en las latidudes
B = 52.52 y C = 33.93 , respectivamente (ver gura 1.16). Usando para
el radio terrestre el valor de 6370 km, determine la distancia entre la Tierra y
la Luna.

16. Encuentre el ngulo entre dos diagonales de un cubo.


a
17. a) Teorema del seno. Demuestre que en un tringulo cualquiera se cumplen
a
las siguientes relaciones:
b
c
a
=
=
,
sin
sin
sin
donde , y son los ngulos interiores del tringulo y a, b y c los lados
a
a
opuestos a cada uno de estos ngulos.
a
b) Teorema del coseno. Demuestre que en un tringulo cualquiera se cumplen
a
las siguientes relaciones:
c2 = a2 + b2 2ab cos ,

Figura 1.15

CAP
ITULO 1. EXPANSIONES Y TRIGONOMETR
IA

25

Figura 1.16
b2 = a2 + c2 2ac cos ,

a2 = b2 + c2 2cb cos .
18. Determine el largo m
nimo que debe
tener una cadena para unir dos poleas de radios R y r, separadas por
una distancia D (ver gura 1.17).

Respuesta:
L = 2 (R r) arcsin

Figura 1.17
Rr
D

+2

D 2 (R r)2 + (r + R) .

19. Un tetraedro regular es la gura geomtrica que se obtiene al formar una pirmie
a
de con cuatro tringulos equilteros idnticos. Encuentre el ngulo entre dos de
a
a
e
a
sus caras.
20. La altura de un edicio se puede determinar midiendo su angulo de elevacin y

o
la distancia a la que uno se encuentra del edicio. Suponga que el instrumento
que tiene a disposicin le permite medir ngulos con un error de 1 . Determine
o
a

CAP
ITULO 1. EXPANSIONES Y TRIGONOMETR
IA

26

el menor error porcentual con que, con tal instrumento, usted puede medir la
altura de un edicio.
21. Dos observadores A y B miden
a
ngulos de elevacin de un avin que
o
o
los sobrevuela a una altura constante. En cierto instante los ngulos
a
medidos por A y B son = 60 y
= 40 , respectivamente. Diez segundos ms tarde, A mide un ngua
a
lo de elevacin = 110 (ver guo
ra 1.18). La separacin entre A y B
o
es D = 1 km. A qu altura vuela el
e
avin? Cul es su velocidad?
o
a

Figura 1.18

22. Graque, usando un computador, la funcin f (t) = cos(t) + cos(0, 9t) para
o
t [0, 40] y observe el fenmeno de pulsaciones.
o
23. Para qu latitud el paralelo terrestre tiene 1/3 de la longitud del Ecuador?
e

24. Una cuneta de forma angular


est caracterizada por los ngulos
a
a
y respecto a la horizontal. Una bola de acero de radio R posa sobre la
cuneta, ver gura 1.19. Determine el
nivel m
nimo de agua, medido desde el punto ms bajo de la cuneta,
a
necesario para cubrir la bola completamente.

Figura 1.19

25. Son las 12 del d Determine en cunto rato ms se vuelven a juntar los punteros
a.
a
a
del reloj.
26. a) Calcule la razn entre las reas del c
o
a
rculo y del tringulo equiltero que lo
a
a
circunscribe (ver gura 1.20a).
b) Haga el mismo clculo anterior pero para el caso en que el tringulo contenga
a
a
n(n + 1)/2 discos de radio R dispuestos como se muestra en la gura 1.20b.

CAP
ITULO 1. EXPANSIONES Y TRIGONOMETR
IA

Figura 1.20a

27

Figura 1.20b

27. Usted se plantea tener un atardecer de 24 horas de duracin en el Ecuador, para


o
lo cual cuenta con un aeroplano. Calcule la velocidad con que deber volar y
a
la direccin que debe tomar para lograr su propsito. Si un amigo viaja a la
o
o
misma velocidad relativa a la Tierra, pero en sentido opuesto, calcule el tiempo
que transcurrir hasta encontrarse nuevamente con l.
a
e
28. Hay que decidir el tipo de empaque que se le va a dar a pelotas de tenis en una
bandeja de forma cuadrada. Decida cul de las dos conguraciones mostradas en
a
la gura 21 resulta ms conveniente. Justique su respuesta cuantitativamente.
a

Figura 1.21a

Figura 1.21b

29. Si 2 es el ngulo entre dos posiciones de una estrella, vista desde la Tierra con
a
seis meses de separacin, entonces se dene la paralaje anual de dicha estrella
o
como .

CAP
ITULO 1. EXPANSIONES Y TRIGONOMETR
IA

28

Muestre que la paralaje es igual al ngulo que subtiende el radio de la rbita


a
o
terrestre en torno al Sol (ver gura) vista desde la estrella, y que

R
,
d

donde R es el radio de la rbita terrestre y d es la distancia entre el Sol y la


o
estrella.
Use este resultado para encontrar la distancia a la estrella Proxima Centauri ,
sabiendo que su paralaje es = 0.772 segundos de arco (1 es 1/3600 de grado),
y que la distancia de la Tierra al Sol es aproximadamente 150 millones de
kilmetros.
o
1 de enero

1 de julio

d
2

30. Comienza un nuevo d Una ducha para despertar. Un desayuno con cereales
a.
preparado por mam. Sof como cada maana, se prepara para ir a la Univera
a,
n
sidad que sus padres pagan con esfuerzo. Su casa, emplazada sobre una suave
colina en los alrededores de la ciudad, delata el origen humilde del cual se siente orgullosa. Hoy tendr su primera prueba, pero no est nerviosa. Sus padres
a
a
s pero callan para no preocuparla. Ya tendrn tiempo, durante la cena, para
,
a
preguntarle cmo le fue.
o
Sof se despide de ambos con un beso, toma la tarjeta bip! desde el escritorio,
a
y sale de su casa. Ante ella se extiende el camino, levemente inclinado, que la
separa del paradero. El sol ha salido t
midamente, y ya se eleva un ngulo
a
sobre el horizonte. Sof desciende decidida por la colina, que tiene un ngulo de
a
a
inclinacin . El sol le da de frente, pero no le molesta. Por el contrario, Sof
o
a
siente cmo ese suave sol matinal le devuelve poco a poco la energ la llena
o
a,
de vida, la anima. Su sombra se extiende tras ella, acompaandola como sus
n
padres quisieran hacerlo. Sof no sabe que, tras ella, sus padres se han quedado
a
observndola por la ventana, y ellos tambin reparan en la sombra de su hija.
a
e

CAP
ITULO 1. EXPANSIONES Y TRIGONOMETR
IA

29

Ambos se preguntan: si la altura de nuestra hija es h, cul es la longitud L de


a
su sombra? Y si la inclinacin de la colina fuera muy pequea, 1, cul
o
n
a
ser el resultado aproximado para L? (Considerar cos 1.)
a

bip !

Sof va a la Universidad.
a
31. Evale
u

tan(x + ) tan x
,

para || 0.

32. Angela estaba triste. Oscar, ese muchacho que la conquist 6 aos atrs, se
o
n
a
hab convertido en un extrao. En una casa que casi era una mansin, llena de
a
n
o

objetos valiosos pero sin hijos, Angela se sent sola. Poco antes, poniendo orden
a
en el caos que dej el terremoto en algunas habitaciones, hab descubierto que
o
a

Oscar le era inel. Durante aos Angela hab tolerado las excentricidades de
n
a
su marido, como esa piscina temperada con el fondo cubierto de espejos, como
parte de eso que ella cre encanto cuando lo conoci. Slo ahora comprend
a
o o
a
que en realidad eran la evidencia, tantas veces negada frente a sus amigas, de

que Oscar era en realidad un tipo obsesivo, presuntuoso y supercial. Pero la


traicin que acababa de descubrir era diferente.
o

Sentada frente a la piscina, Angela buscaba la manera de terminar esta mentira.


No era fcil. Miraba jamente el agua de la piscina como si desde ah pudiera
a

emanar la fuerza que necesitaba para enfrentar al que nunca fue realmente
su marido. Ya era de noche. En los espejos del fondo se reejaba la luz de la

ventana de la habitacin matrimonial. Qu iron Seguro que Oscar la esperaba


o
e
a.
para abrazarla hipcritamente. Como era irnico tambin que esa imagen se
o
o
e
reejara justamente sobre el fondo de esa piscina, que se hab inclinado un
a
a
ngulo respecto a la horizontal despus del terremoto, se gracias al cual
e
e
termin enterndose de que su felicidad no era sino un engao.
o
a
n
Sabiendo que el
ndice de refraccin del agua es n, que el ngulo de incidencia
o
a
de la luz sobre el agua es , y el ngulo de inclinacin del fondo de la piscina, ,
a
o

CAP
ITULO 1. EXPANSIONES Y TRIGONOMETR
IA

30

Angela, con los ojos nublados por el llanto, se dio cuenta de que pod calcular
a
el ngulo de salida de los rayos desde el agua. Encuntrelo, teniendo presente
a
e
que el ngulo de reexin es igual al de incidencia en el fondo de la piscina.
a
o
Muestre, adems, que si 1, entonces sin sin + A , y encuentre A.
a

Quizs esto ayude a que Angela recupere la fuerza interior para ser valiente y
a
dejar atrs para siempre su vida actual.
a

La piscina que recoge las lgrimas de Angela.


a
33. El cuarto Campeonato Panamericano de Canopy contemplaba una novedad en
el circuito: una tirolesa entre las dos ms altas montaas del Parque Seilrutsa
n
che. Como parte de la agresiva campaa publicitaria para reposicionar esta
n
actividad, dos d antes de la competencia el Parque Seilrutsche fue abierto al
as
pblico para que los visitantes pudieran experimentar esta tirolesa, la ms larga
u
a
del continente.
La tirolesa un las cimas de las montaas Vuori y Gunung. La altura de la
a
n
montaa ms pequea, Gunung, es h; la altura de la mayor, Vuori, es desconon
a
n
cida. Se sabe, sin embargo, que a las 17 horas Vuori proyecta una sombra de
longitud L en el suelo, siendo el ngulo asociado a la sombra respecto a la
a
vertical (ver gura). Si la distancia horizontal entre ambas cimas es D, cul es
a
la longitud de la tirolesa?

CAP
ITULO 1. EXPANSIONES Y TRIGONOMETR
IA

31

instructor
visitante VIP

Vuori

Gunung
h

L
D

La nueva atraccin del Parque Seilrutsche.


o

1.4.

Solucin a algunos de los problemas


o

Solucin al problema 15
o
Inspeccionando la gura 1.22 se deduce de inmediato que
= +

Figura 1.22
y
= + B |C | .

CAP
ITULO 1. EXPANSIONES Y TRIGONOMETR
IA

32

Usando el teorema del seno (ver problema 17) en los tringulos OBL y OLC, se
a
obtienen las expresiones
sin
sin( )
=
R
D
y
sin
sin( )
=
.
R
D
Como y son ngulos pequeos podemos usar las aproximaciones
a
n
sin
y
sin .
De esta manera se obtienen

R
sin
D

R
sin .
D
Sumando estas ecuaciones se deduce que

= +

R
(sin + sin ) ,
D

o sea,
D

R (sin + sin )
R (sin + sin )
=
.

+ B |C |

Sustituyendo en esta ecuacin los valores numricos se encuentra que


o
e
D 367.000 km ,
valor muy cercano al actualmente aceptado para el radio de la rbita lunar, que es
o
de 384.000 km.
Solucin al problema 16
o
Consideremos un cubo de lados a. Sea A un vrtice de una diagonal y B el vrtice
e
e
de otra diagonal del cubo. De los dos vtices de la segunda diagonal, denotaremos
e
por B al vrtice que est a una distancia a de A (el otro vrtice se encontrar a una
e
a
e
a

distancia a 2 de A). Sea O el punto central del cubo.

El tringulo AOB es issceles: con base AB = a y lados b AO = BO = 23 a.


a
o
El ngulo = (AOB) es el ngulo buscado. Se tiene que
a
a
sin

a/2
1

=
= ,
2
b
3

CAP
ITULO 1. EXPANSIONES Y TRIGONOMETR
IA

33

de donde se deduce que


= 70.529 .
(AOC) = 109.47 .

El ngulo complementario
a

Figura 1.23
Solucin al problema 21
o
Sea a = AP y d = P Q. Usando el teorema del seno en el tringulo AP B se obtiene
a
sin ( )
sin
=
,
a
D
o sea,
a=D

sin
.
sin( )

Usando el teorema del seno en el tringulo AQP se deduce que


a
sin( )
sin( )
=
.
a
d
Usando las dos ecuaciones anteriores se obtiene para d la expresin
o
d=D

sin
sin( )
.
sin( )
sin

Reemplazando los valores numricos se encuentra que la distancia recorrida por el


e
avin en 10 segundos es d = 1, 53 km. La velocidad del avin es, por lo tanto, v =
o
o
552 km/h. La altura a la que vuela el avin viene dada por
o
h = a sin = 1628 [m] .

CAP
ITULO 1. EXPANSIONES Y TRIGONOMETR
IA

34

Figura 1.24

Solucin al problema 24
o
Primero giremos la cuneta de manera que quede simtrica respecto a la horizontal,
e
es decir, con un ngulo ( + )/2 a cada lado (ver gura 25a).
a

Figura 1.25a

Figura 1.25b

El ngulo <
a
)ABC tambin es ( + )/2, luego
e
AB =

R
cos +
2

CAP
ITULO 1. EXPANSIONES Y TRIGONOMETR
IA

35

Para volver a poner la cuneta en la orientacin original debemos girarla en un


o
a
ngulo ( )/2. Por lo tanto, (ver gura 1.25b)
BD = AB cos

=R

cos
2
cos +
2

Para la altura del nivel de agua se obtiene nalmente la expresin


o

h = R 1 +

cos
2
cos +
2

Cap
tulo 2

Cinemtica en una dimensin


a
o
versin 8 mayo 2012
o

La Cinemtica es la descripcin del movimiento de un cuerpo sin considerar las


a
o
causas que lo producen. Ser capaces de describir adecuadamente el movimiento es el
primer paso para estudiarlo y entender sus causas. En este Cap
tulo introduciremos
conceptos bsicos para la descripcin del movimiento: posicin, velocidad, aceleracin.
a
o
o
o
Mantendremos la discusin simple al considerar slo movimientos en una dimensin.
o
o
o
Sin embargo, incluso con esta simplicacin ser necesario introducir conceptos mao
a
temticos nuevos, sin los cuales nos ser muy dif avanzar. Por ello, en este Cap
a
a
cil
tulo
tambin se revisarn algunas nociones bsicas de clculo integro-diferencial. Como en
e
a
a
a
el Cap
tulo anterior, la idea es introducir estos conceptos de manera intuitiva, como
necesidades naturales para la descripcin del movimiento, dejando los aspectos ms
o
a
formales para el curso de Clculo correspondiente.
a

2.1.

Posicin, velocidad y aceleracin


o
o

Para simplicar la discusin, comenzaremos por estudiar el movimiento de obo


jetos cuya ubicacin queda determinada especicando la posicin de un solo punto.
o
o
Este tipo de objeto recibe el nombre de partcula. Contrariamente a lo que pudiera

pensarse, no es necesario que los objetos sean pequeos para que puedan ser considen
rados part
culas. Por ejemplo, cuando se estudia el movimiento de la tierra en torno
al sol, la distancia relevante es la distancia Tierra-Sol. En este caso, el tamao de la
n
Tierra no es importante, pudindose tratar como una part
e
cula ubicada en el centro
de la tierra.
El movimiento ms simple de una part
a
cula se tiene cuando la posicin de sta
o
e
viene descrita por una unica coordenada; por ejemplo, el movimiento de una part

cula
que se traslada a lo largo de una l
nea recta. Ser ste tipo de movimientos del que nos
ae
ocuparemos en este Cap
tulo. No nos basta con saber que el movimiento es a lo largo
de una recta, sin embargo. Para describir el movimiento de la part
cula necesitamos
36

CAP
ITULO 2. CINEMATICA EN UNA DIMENSION

37

elegir un origen, en alguna parte arbitraria de la recta. Adems, necesitamos decidir


a
qu lado de la recta ser el lado positivo, y cul el negativo. Lo usual es escoger el
e
a
a
lado positivo como el lado derecho (Fig. 1.1).

x
Figura 2.1

En estas condiciones, la posicin de una part


o
cula queda determinada dando
simplemente un nmero (la coordenada x). La descripcin de su movimiento es
u
o
completa si conocemos la funcin x(t) que indica la posicin que ocupa en cada
o
o
instante t.
La diferencia entre la coordenada de una part
cula entre dos instantes t1 y t2 (con
t2 > t1 ) se denomina desplazamiento:
Desplazamiento x2 x1 x .

El desplazamiento es una cantidad con unidades, que se mide, en el sistema internacional, en metros (m). Adems, el desplazamiento tiene signo. Si la coordenada x de la
a
part
cula se incrementa durante cierto intervalo de tiempo, entonces el desplazamiento es positivo; si, por el contrario, decrece, el desplazamiento es negativo. Notemos
que un desplazamiento positivo signica que la part
cula est ms a la derecha que
a a
lo que estaba inicialmente, no que est ms lejos del origen.
a a
En todo caso, es claro que, si bien el desplazamiento es un primer concepto util

para describir el movimiento, no es en absoluto capaz de hacerlo completamente.


No es lo mismo cubrir la distancia entre dos ciudades en automvil que en auto. Es
o
necesario, adems, introducir el concepto de tiempo en esta discusin.
a
o
Se dene la velocidad media de una part
cula durante el intervalo [t1 , t2 ] como la
razn entre el desplazamiento y la duracin del intervalo de tiempo,
o
o
v(t1 , t2 ) =

x(t2 ) x(t1 )
,
t2 t1

y sus unidades, en el sistema internacional, son de metros sobre segundos (m/s).


La velocidad media entrega una informacin global sobre el movimiento que reao
liza una part
cula en un cierto intervalo de tiempo. En cualquier viaje en automvil
o
a veces vamos ms rpido o ms lento, por ejemplo. La velocidad media no pretende
a a
a
dar estos detalles. Pero si la velocidad es constante, entonces uno puede estar seguro
de que la velocidad siempre ser igual a la velocidad media, pues el cuociente entre
a
x y t ser el mismo, independiente del intervalo de tiempo escogido.
a
Consideremos entonces el caso en que la velocidad es constante, v0 . De la ecuacin
o
anterior se sigue que, si denominamos x0 a la posicin inicial en el tiempo t0 , y a x(t)
o

CAP
ITULO 2. CINEMATICA EN UNA DIMENSION

38

a la posicin en un tiempo t cualquiera, entonces


o
x(t) = x0 + v0 (t t0 ) .
Uno dice que sta es una ecuacin de itinerario, porque nos permite saber, si conozco
e
o
las condiciones iniciales (t0 , x0 ), dnde se encuentra la part
o
cula en cada instante. La
ecuacin de itinerario anterior, entonces, contiene toda la informacin necesaria para
o
o
describir el movimiento si la velocidad es constante.
Si gracamos la ecuacin de itinerario anterior, resultar una l
o
a
nea recta:
x ( t)
x1
x0

t0

t1

Observemos dos cosas: primero, t0 es simplemente un tiempo inicial, arbitrario.


No es necesariamente el tiempo en que el movimiento comenz (como vemos en la
o
gura anterior, en que la part
cula ya estaba en movimiento antes de t0 ). Es slo un
o
instante de referencia. Ni siquiera t = 0 tiene que ser el instante en que comienza el
movimiento; t = 0 puede ser el tiempo en el comenz a funcionar nuestro cronmetro,
o
o
o cualquier otro instante de referencia adecuado.
Segundo, de la ecuacin de itinerario, o bien de la gura anterior, se sigue que v0
o
es la pendiente de la recta x(t). Observemos las siguientes guras esquemticas:
a
x(t)

(a)

x(t)

(b)

x(t)

(c)

La Fig. (a) representa a una part


cula movindose con velocidad positiva, vale
e
decir, hacia la derecha [de la denicin de velocidad media, sta es mayor que cero si
o
e
x(t2 ) > x(t1 )]. La Fig. (b), en cambio, representa a una part
cula en reposo (velocidad

CAP
ITULO 2. CINEMATICA EN UNA DIMENSION

39

cero). Finalmente, la Fig. (c) representa a una part


cula con velocidad negativa, es
decir, movindose hacia la izquierda. Observemos que, en mdulo, la pendiente (c) es
e
o
mayor que la pendiente (a), por tanto podemos armar que la part
cula que se mueve
hacia la izquierda se mueve ms rpidamente que la part
a a
cula hacia la derecha.
Si gracamos ahora la velocidad como funcin del tiempo, para las mismas tres
o
part
culas anteriores, obtendremos las siguientes guras:
v(t)

(a)

(b)

v(t)

(c)

v(t)

Las l
neas horizontales indican que se trata de movimientos con velocidad constante en todos los casos, primero positiva, luego cero, y luego negativa, pero de mdulo
o
mayor que en el caso (a).
Si bien es cierto tenemos bastante informacin en estos grcos, no tenemos toda
o
a
la informacin necesaria para describir el movimiento de la part
o
cula. Por ejemplo,
si la part
cula (a) est inicialmente a la derecha del origen, entonces sabemos que se
a
est alejando del mismo; si est a la izquierda del origen, sabemos que se est acera
a
a
cando. As que no podemos determinar completamente su movimiento a menos que

tengamos informacin sobre su posicin inicial. Esto es, por supuesto, consistente con
o
o
nuestra ecuacin de itinerario anterior.
o
Sin embargo, observemos el siguiente interesante hecho: Consideremos, en el gra
co de velocidad versus tiempo (a) anterior, dos instantes t1 y t2 :

v ( t)

v0

111111111111
000000000000
111111111111
000000000000
111111111111
000000000000
111111111111
000000000000
111111111111
000000000000
111111111111
000000000000
111111111111
000000000000
A
111111111111
000000000000
111111111111
000000000000
111111111111
000000000000
111111111111
000000000000
111111111111
000000000000
111111111111
000000000000
111111111111
000000000000
t1

t2

Dichos instantes determinan un rea bajo la curva, A, igual a


a
A = v0 t ,

CAP
ITULO 2. CINEMATICA EN UNA DIMENSION

40

que es precisamente igual a x, el desplazamiento de la part


cula entre los instantes t1
y t2 . Notemos tambin que si v0 < 0, entonces la expresin algebraica anterior nos da
e
o
un rea negativa. Esto puede parecer inconsistente con la nocin intuitiva de rea,
a
o
a
pero matemticamente no lo es. De hecho, resulta adecuado decir que el rea bajo
a
a
una curva, cuando dicha curva est por debajo del eje de las abscisas, es negativa.
a
Por tanto, tambin es cierto, para velocidades negativas, que el desplazamiento x
e
es igual al rea bajo la curva de velocidad versus tiempo.
a
De lo anterior se sigue que, dado un grco velocidad versus tiempo, no slo podea
o
mos conocer su velocidad, sino el desplazamiento, lo cual es bastante informacin. Si
o
adems conocemos su posicin inicial, entonces podremos determinar completamente
a
o
la trayectoria de la part
cula.
Problema resuelto en clases: 2.13
Hasta ahora tenemos, entonces, una descripcin completa del movimiento de
o
part
culas con velocidad constante. Sin embargo, la mayor de las veces la velocidad
a
es constante. Por analog a lo ya hecho, donde denimos una velocidad media como el
a
cuociente entre un intervalo de posiciones y un intervalo de tiempos, ahora deniremos
el concepto de aceleracin.
o
La aceleracin media (o promedio) que tiene la part
o
cula durante el intervalo
[t1 , t2 ] es igual al cambio de velocidad que ocurre durante el intervalo, dividido por la
duracin de ste, es decir
o
e
v(t2 ) v(t1 )
a(t1 , t2 ) =
.
t2 t1
Las unidades de la aceleracin son m/s2 .
o
Observemos que una aceleracin negativa simplemente signica que la velocidad
o
disminuye. Sin embargo, ello no implica que la part
cula vaya ms lento. Esto ser
a
a
cierto slo si la velocidad es positiva (es decir, si se mueve hacia la derecha). Si se
o
mueve hacia la izquierda, la velocidad es negativa, y por lo tanto una aceleracin
o
negativa signica que la velocidad disminuye, esto es, se vuelve ms negativa. Vale
a
decir, la part
cula va aun ms rpido hacia la izquierda.
a a
Si la aceleracin es constante, igual a a, entonces la aceleracin media es igual a la
o
o
aceleracin a en todo intervalo de tiempo. (Esto es lo que se denomina un movimiento
o
uniformemente acelerado.) De la denicin se sigue que, en este caso, llamando al
o
instante inicial t1 = t0 , la velocidad inicial v(t0 ) = v0 , y a t2 = t,
v(t) = v0 + a(t t0 ) .
Conocida la aceleracin, entonces, podemos determinar la velocidad en todo instante,
o
conocida la velocidad inicial.
La ecuacin anterior, adems, nos dice que el grco de velocidad versus tiemo
a
a
po, cuando la aceleracin es constante, es una l
o
nea recta, de pendiente igual a la
aceleracin:
o

CAP
ITULO 2. CINEMATICA EN UNA DIMENSION

41

v ( t)
v1
v0

t0

t1

De manera anloga a lo que hicimos para velocidad constante, podemos ahora


a
gracar la velocidad como funcin del tiempo, para distintas aceleraciones:
o
v(t)

(a)

v(t)

(b)

v(t)

(c)

En el caso (a), tenemos una part


cula cuya velocidad aumenta (aceleracin posio
tiva). Nuevamente hay que tener cuidado con las interpretaciones: que la velocidad
aumenta signica, en el caso de la gura (a), que la part
cula va cada vez ms rpido
a a
(recorre, en intervalos de tiempo iguales, cada vez mayor distancia), Sin embargo, si
la recta estuviera bajo el eje de las abscisas, que la velocidad aumente signica que se
hace cada vez menos negativa, lo cual implica que la part
cula va cada vez ms lento
a
(recorre, en intervalos de tiempo iguales, cada vez menor distancia).
Luego, en el caso (b), la recta tiene pendiente cero, es decir la aceleracin es
o
cero. Corresponde precisamente al caso que hab
amos analizado antes, con velocidad
constante.
Por ultimo, en el caso (c), la part

cula tiene aceleracin negativa, es decir, su


o
velocidad disminuye (lo cual no signica necesariamente que vaya ms lento).
a
Tambin podemos gracar la aceleracin como funcin del tiempo, para cada uno
e
o
o
de los casos anteriores:

CAP
ITULO 2. CINEMATICA EN UNA DIMENSION
a(t)

(a)

(b)

a(t)

42
a(t)

(c)

Recapitulando: en un grco de posicin versus tiempo, la pendiente de la recta


a
o
x(t) representa la velocidad; en uno de velocidad versus tiempo, la pendiente de la
recta v(t) representa la aceleracin
o
Observemos que, anlogamente a los grcos v(t) analizados anteriormente, en
a
a
este caso el rea bajo la curva de un grco a(t) nos da v = v(t2 ) v(t1 ), el cambio
a
a
de velocidad en un intervalo de tiempo dado.
Ya hab
amos encontrado que, al menos para el caso en que la velocidad es constante, el rea bajo la curva en el grco de v(t) es el desplazamiento. Ser cierto
a
a
a
aun en el caso en que la aceleracin es distinta de cero? Claro que s La razn es que
o
.
o
siempre ser posible dividir el movimiento en intervalos sucientemente pequeos en
a
n
que la velocidad se pueda considerar constante, y para cada uno de esos intervalos
ser entonces cierto, como vimos, que el rea bajo la curva es el desplazamiento.
a
a
Ahora estamos en condiciones de encontrar la posicin de una part
o
cula con aceleracin constante a. Consideremos la curva v(t) siguiente, y calculemos el rea bajo
o
a
la curva entre dos instantes t1 y t2 :
v ( t)
v1
v0

1111111111111
0000000000000
1111111111111
0000000000000
1111111111111
0000000000000
A2
1111111111111
0000000000000
1111111111111
0000000000000
1111111111111
0000000000000
1111111111111
0000000000000
1111111111111
0000000000000
1111111111111
0000000000000
1111111111111
0000000000000
A1
1111111111111
0000000000000
1111111111111
0000000000000
1111111111111
0000000000000
1111111111111
0000000000000
1111111111111
0000000000000
1111111111111
0000000000000
t0

t1

El desplazamiento ser la suma de las reas A1 y A2 :


a
a
1
x = A1 + A2 = v0 t + tv .
2
De la denicin de aceleracin, v = at, luego
o
o
1
x = v0 t + at2 .
2

CAP
ITULO 2. CINEMATICA EN UNA DIMENSION

43

Deniendo, para simplicar la notacin, t1 = t, x(t0 ) = x0 , queda


o
1
x(t) = x0 + v0 (t t0 ) + a(t t0 )2 ,
2
que es entonces la ecuacin de itinerario para aceleracin constante. Si slo conocemos
o
o
o
el grco de v(t), entonces para determinar completamente la posicin necesitamos
a
o
una informacin adicional, x(t0 ) = x0 .
o
Se sigue que el grco de x(t) es, si hay aceleracin constante, una parbola. En
a
o
a
particular, mientras ms grande sea la aceleracin a, la parbola ser ms abierta.
a
o
a
a a
Adems, si a > 0, se tiene una parbola apuntando hacia arriba, y si a < 0
a
a
ser una parbola invertida. Matemticamente, el concepto a usar es el de curvatura.
a
a
a
Consideremos, por ejemplo, las siguientes curvas:

x(t)

(a)

x(t)

(b)

El caso (a) corresponde a una parbola con curvatura positiva (curva cncava),
a
o
de modo que describe a una part
cula con aceleracin positiva, mientras que en (b)
o
la part
cula tiene curvatura (aceleracin) negativa (curva convexa). En mdulo, la
o
o
aceleracin del caso (a) es menor que en (b). Observemos que una recta es una curva
o
con curvatura cero, es decir, la aceleracin es cero, lo cual por supuesto es consistente
o
con nuestros resultados anteriores, que una recta en el grco x(t) corresponde a un
a
movimiento con velocidad constante.
Recapitulamos brevemente la correspondencia entre conceptos geomtricos y f
e
sicos en los grcos que hemos estudiado:
a
Concepto geomtrico
e
pendiente
curvatura
a
rea bajo la curva

x(t)
velocidad
aceleracin
o

Concepto f
sico
v(t)
aceleracin
o

desplazamiento cambio

a(t)

de velocidad

Ejemplo:
Consideremos una piedra que se deja caer desde una altura h. Cunto tiempo se
a
demora en caer, y con qu velocidad llega al suelo? Representemos grcamente este
e
a
problema:

CAP
ITULO 2. CINEMATICA EN UNA DIMENSION

44

t =0

y =h

1111111111111111111111
y =0 0000000000000000000000
1111111111111111111111
0000000000000000000000
1111111111111111111111
0000000000000000000000
1111111111111111111111
0000000000000000000000

Debemos ser cuidadosos, en todo este tipo de problemas, al elegir el origen del
tiempo, y el origen de coordenadas. Por supuesto, el resultado nal no deber dea
pender de dicha eleccin, pero es necesario hacerla para que cualquier ecuacin de
o
o
itinerario tenga sentido. En este caso, hemos escogido t = 0 como el instante en que
se deja caer la pelota, el eje de coordenadas es en la direccin vertical, que es donde
o
ocurre el movimiento, con y = 0 en el suelo y con la direccin positiva apuntando
o
hacia arriba.
Galileo fue quien observ que todos los cuerpos, en una situacin como la deso
o
crita en la gura, estn sujetos a la misma aceleracin, que llamamos aceleracin de
a
o
o
gravedad, e igual a a = g 9.8 m/s2 . Debemos tener cuidado con el signo, para
no cometer un error t
pico: que la aceleracin sea negativa no tiene relacin con que
o
o
la piedra cae. No es tan sencillo. Una aceleracin negativa signica que la velocidad
o
disminuye. Si inicialmente la velocidad es cero, como en la gura, entonces que disminuya signica que la velocidad se vuelve negativa. Una velocidad negativa signica
que el desplazamiento es negativo, es decir, que su coordenada de posicin disminuye
o
con el tiempo. En el caso de la gura, inicialmente y = h > 0, luego y se hace menos
positiva con el tiempo, es decir, cae. En este caso particular, que la aceleracin sea
o
negativa signica que cae. Pero qu sucede si la part
e
cula es lanzada desde la altura h
hacia arriba? En tal caso, sigue siendo cierto que la velocidad debe disminuir, porque
la aceleracin es negativa. Pero ahora, inicialmente, la velocidad es positiva (porque
o
su desplazamiento es positivo, es decir, su posicin y aumenta, se hace ms positiva
o
a
respecto a h), de modo que en un instante posterior su velocidad es menos positiva.
La part
cula entonces no cae, sino simplemente va cada vez ms lento.
a
Ya que hemos jado el signicado de t = 0, y = 0 e y > 0, escribimos las
ecuaciones de itinerario:
1
y(t) = y0 + v0 t + at2 ,
2
v(t) = v0 + at ,
donde y0 = y(t = 0), v0 = v(t = 0). En nuestro ejemplo, adems, y0 = h, v0 = 0,
a

CAP
ITULO 2. CINEMATICA EN UNA DIMENSION

45

a = g, luego
1
y(t) = h gt2 ,
2
v(t) = gt .
Podemos entonces responder la primera pregunta, cundo la part
a
cula llega al suelo.
Esto sucede para un tiempo t = T tal que y(T ) = 0, es decir:
1
y(T ) = h gT 2 ,
2
que tiene dos soluciones:
T =

2h
.
g

Nos encontramos aqu frente al mismo problema que con las funciones inversas tri
gonomtricas: ms de una solucin matemtica, pero slo una puede ser f
e
a
o
a
o
sicamente
correcta, ya que sabemos que si dejamos caer esta piedra, llegar al suelo en un unico
a

instante bien determinado. Cul es la solucin f


a
o sicamente aceptable? En este caso,
es evidente que la piedra debe llegar al suelo despus de soltarla, por lo tanto la
e
solucin correcta es la positiva, T = 2h/g.
o
Toda esta discusin puede parecer una trivialidad, pero no es dif imaginar un
o
cil
problema en que la solucin correcta no sea la anterior. En efecto, digamos que el
o
experimento ahora es lanzar la piedra, con cierta velocidad inicial, desde el suelo, y
que llega al punto ms alto de su trayectoria, antes de caer nuevamente, en t = 0.
a
Recordemos: la eleccin de t = 0 es completamente arbitraria, y nada nos impide jar
o
que t = 0 es el instante en que la piedra alcanza su mayor altura. La pregunta ahora
es: Cundo fue lanzada la piedra? No es dif convencerse de que las ecuaciones de
a
cil
itinerario son exactamente las mismas, y por lo tanto la solucin matemtica ser T =
o
a
a
2h/g. Sin embargo, en este caso la piedra fue lanzada antes de que llegara arriba,
y por lo tanto la solucin f
o sica al problema es el tiempo negativo, T = 2h/g. Por
supuesto, un tiempo negativo no es nada extico. Signica simplemente que es un
o
tiempo anterior a t = 0, cuya eleccin a su vez es completamente arbitraria.
o
Ahora podemos responder la siguiente pregunta: con qu velocidad llega al suelo?
e
Como sabemos el tiempo que tarda en caer, la velocidad nal ser
a
v(T ) = aT = g

2h
=
g

2gh .

Que la velocidad sea negativa signica, dada la eleccin de nuestro eje de coordenadas,
o
que es hacia abajo, como debe ser.
Hay una manera un poco ms directa de obtener el mismo resultado. Consia
deremos ahora un caso general de un movimiento acelerado, y las correspondientes

CAP
ITULO 2. CINEMATICA EN UNA DIMENSION

46

ecuaciones de itinerario, que escribiremos de la siguiente forma:


1
x = v1 t + at2 ,
2
v = at ,
con t = t2 t1 , x = x(t2 ) x(t1 ), v = v2 v1 , v1 = v(t1 ) y v2 = v(t2 ). Las
dos ecuaciones dependen del tiempo, pero podemos manipularlas para obtener una
expresin que no dependa del tiempo. Basta con despejar t de la segunda ecuacin,
o
o
y reemplazar el resultado en la primera:
x = v1

v 1
+ a
a
2

v
a

v
a

1
v1 + v
2

v2 v1 1
(v1 + v2 ) ,
a 2

es decir

2
2
v2 v1
.
2a
Tenemos aqu una expresin que relaciona directamente la distancia recorrida, la

o
aceleracin, y las velocidades inicial y nal de una part
o
cula, si la aceleracin es
o
constante. El tiempo que transcurre entre medio no es relevante.
Usando esta expresin podemos recalcular la velocidad con que la piedra llega
o
al suelo en nuestro ejemplo anterior. En este caso, t1 = 0, t2 = T , de modo que
x = 0 h = h, v1 = 0, a = g, y v2 = v(T ) es lo que queremos calcular. Se tiene
entonces
2
v2 = 2ax = 2gh ,

x =

resultado por supuesto consistente con lo anterior, pero ahora obtenido de un modo
mucho ms sencillo. Nuevamente ac tenemos dos soluciones matemticamente acepa
a
a

cula, en t = T , se mueve hacia abajo,


tables, v2 = 2gh. Pero sabemos que la part
por tanto (dada la eleccin de eje coordenado que hicimos) su velocidad es negativa,
o

de modo que la solucin f


o sicamente aceptable es la solucin negativa, v2 = 2gh.
o
Problema resuelto en clases: 2.12
Hasta el momento, hemos denido ciertos conceptos bsicos, y relacionado ala
gunos elementos geomtricos con f
e
sicos, para movimientos sencillos, con velocidad
o aceleracin constante. Sin embargo, en general los cuerpos se mueven de manera
o
ms compleja, y debemos por tanto extender los resultados anteriores para describir
a
dichos movimientos tambin.
e
Lo primero que observamos es que la velocidad media, si bien es cierto nos da
una primera idea acerca del movimiento, no es un buen descriptor de los detalles
del mismo. En la Fig. 2.2 hemos representado la trayectoria de una part
cula. En un
grco x(t) en funcin de t, la velocidad media entre t1 y t2 corresponde a la tangente
a
o
del ngulo que forma la recta que une (x1 , t1 ) y (x2 , t2 ) con el eje del tiempo.
a

CAP
ITULO 2. CINEMATICA EN UNA DIMENSION

47

x
P2

x2

x1

x2 _ x1

P1
t2 _ t1
t2

t1

Figura 2.2
Si la part
cula se moviera realmente con la velocidad media, su trayectoria ser
a
el trazo que une a los puntos P1 y P2 , pero es claro que, en el intervalo [t1 , t2 ], el
movimiento es algo ms complicado.
a
Si se desea tener una informacin ms precisa acerca de la velocidad durante el
o
a
movimiento, es necesario subdividir el intervalo de tiempo original en subintervalos
y calcular en cada uno de ellos una velocidad media. Mientras ms pequeo es el
a
n
tamao de esos subintervalos, ms precisa es la informacin acerca de las variaciones
n
a
o
que experimenta la velocidad de la part
cula mientras se desplaza. El valor que se
mide para la velocidad media en un cierto intervalo de tiempo pequeo, donde es
n
nito pero tan pequeo como nosotros deseamos, se denomina velocidad instantnea.
n
a
Para determinar la velocidad instantnea de la part
a
cula en un instante t, se
evala la velocidad promedio durante un intervalo muy pequeo que comienza en t y
u
n
termina en t + , donde es un incremento de tiempo innitesimal (ms adelante, al
a
nalizar el clculo, haremos 0). Expl
a
citamente:
v(t, t + ) =

x(t + ) x(t)
.

Al hacer 0, se obtiene la velocidad instantnea de la part


a
cula en el instante t.

Esta la denotaremos por v(t) o x(t). Se tiene

v(t) = l
m

x(t + ) x(t)
= x(t) .

(2.1)

Este proceso l
mite est ilustrado en la Figura 2.3. All se observa cmo cambia
a

o
el valor de la velocidad media de la part
cula en un intervalo [t, t + t] cuando es
evaluada para diferentes valores de t. Mientras ms pequeo es t, la recta que une
a
n
los dos puntos de la curva se parece ms a la curva real: en la gura, P P3 es mucho
a
ms parecida a la curva x(t) en el intervalo t3 , que el trazo P P2 en el intervalo t1 .
a
(Esto no es sino una consecuencia de que, en intervalos de tiempo sucientemente

CAP
ITULO 2. CINEMATICA EN UNA DIMENSION

48

cortos, siempre se puede considerar que la velocidad es constante.) En el caso l


mite,
cuando 0, se observa que la velocidad instantnea queda representada por la
a
tangente del ngulo (es decir, la pendiente) que forma la recta tangente a la curva
a
x(t) vs. t con el eje del tiempo.
De aqu en adelante el trmino velocidad siempre se referir a la velocidad ins
e
a
tantnea.
a

P2

tangente
en P

P1

P3

x1

P
t3
t2
t1

t1

Figura 2.3
Es interesante notar cmo la informacin anterior nos permite describir de manera
o
o
bastante detallada el movimiento de una part
cula, simplemente observando su grco
a
de posicin versus tiempo. Por ejemplo, en la Fig. 2.3, podemos decir que, en el
o
instante t, la part
cula tiene velocidad positiva (la pendiente de la recta tangente a
la curva en P es positiva), de modo que se mueve hacia la derecha, alejndose del
a
origen; luego, a medida que avanza el tiempo, su velocidad se hace cada vez ms
a
pequea, hasta hacerse cero un poco antes que P2 (la recta tangente a la curva en ese
n
punto es horizontal, de modo que su pendiente es cero), momento en el cual alcanza
su mxima distancia respecto al origen. Ms adelante, la tangente a la curva en P1
a
a
tiene pendiente negativa, de modo que la part
cula tiene velocidad negativa en t = t1 ,
esto es, se mueve hacia la izquierda, acercndose ahora al origen.
a
Ejemplos:
1. Supongamos que la posicin de una part
o
cula viene dada por x(t) = x0 + v0 t,
m . El grco x(t) en funcin de t da lugar a la recta
a
o
con x0 = 1 m y v0 = 0.5 s
que se muestra en la gura 2.4.

CAP
ITULO 2. CINEMATICA EN UNA DIMENSION

49

Evaluemos expl
citamente la velocidad en un instante t cualquiera. Usando la
ecuacin (2.1) y la expresin para x(t) de este ejercicio, se obtiene
o
o
[x0 + v0 (t + )] [x0 + v0 t]
x(t + ) x(t)
= l
m
0
0

v0
= l
m
= l v0 = v0 .
m
0
0

v(t) = l
m

x ( t ) [m]
2
1
0
1

t [s]

Figura 2.4
Este resultado indica que la expresin para x(t) escrita ms arriba efectivamente
o
a
corresponde al movimiento de una part
cula con velocidad constante v0 (i.e.,
independiente del tiempo).
2. Supongamos ahora que la posicin de una part
o
cula viene dada por
z(t) = z0

1 2
gt ,
2

con z0 = 10 m y g = 9.81 m . Esto describe a una part


cula en ca libre en
da
s2
el campo gravitatorio terrestre, que se deja caer en t = 0 desde una altura z0 .
Al gracar la posicin en funcin del tiempo se encuentra la curva (parbola)
o
o
a
mostrada en la gura 2.5.
Evaluemos la velocidad en un instante t cualquiera. Usando la ecuacin (2.1),
o
se obtiene
1
[z0 2 g (t + )2 ] [z0 1 g t2 ]
z(t + ) z(t)
2
= l
m
0
0

1 g (2t + )
g (2t + )
= l
m
= g t .
= l
m 2
0
0

v(t) = l
m

CAP
ITULO 2. CINEMATICA EN UNA DIMENSION

50

La gura 2.6 muestra el grco de la velocidad instantnea en funcin del tiema


a
o
po. Se observa que sta decrece linealmente a medida que transcurre el tiempo.
e
El signo negativo de la velocidad signica que la part
cula se est desplazando
a
en el sentido negativo del eje z.
Sin embargo, el mdulo de la velocidad de la part
o
cula (magnitud que en algunos
textos es denominada rapidez) aumenta a medida que transcurre el tiempo:
|v(t)| = g t .
z ( t ) [m]
10
0
1

t [s]

10
20
30

Figura 2.5
v ( t ) [m/s]
10
0
1

t [s]

10
20
30

Figura 2.6

As como generalizamos el concepto de velocidad al caso de velocidad no uniforme,

a travs de un proceso l
e
mite, podemos hacerlo para el caso en que la aceleracin no es
o
uniforme. Por los mismos argumentos dados anteriormente, en este caso la aceleracin
o
media da slo informacin aproximada sobre el cambio de velocidad. Pero intuimos
o
o
que, para intervalos de tiempo sucientemente pequeos, la aceleracin media deber
n
o
a
ser la aceleracin correcta. Denimos entonces la aceleracin instantnea en un
o
o
a
instante t, como la aceleracin media durante un intervalo muy pequeo que comienza
o
n

CAP
ITULO 2. CINEMATICA EN UNA DIMENSION

51

en t. Sea [t, t + ] ese intervalo, donde es un tiempo innitesimal. Entonces


a(t, t + ) =

v(t + ) v(t)
.

Al hacer 0 se obtiene la aceleracin instantnea de la part


o
a
cula (en el instante t),
que denotaremos con a(t), x(t) o v(t):

a(t) = l
m

v(t + ) v(t)
= v(t) = x(t) .

(2.2)

De aqu en adelante el trmino aceleracin siempre se referir a la aceleracin ins


e
o
a
o
tantnea.
a
Ya hab
amos observado antes que, al menos para aceleracin constante, la aceleo
racin se relaciona con la curvatura de x(t). Ahora podemos entender que esto es un
o
hecho general. Consideremos la siguiente gura:

B
B

A
C

A la izquierda, tenemos una curva con curvatura positiva. Es claro que, a medida
que la curva avanza de A a B a C, la pendiente de la tangente aumenta (se hace
menos negativa). Si la curva representa posicin en funcin del tiempo, entonces
o
o
lo que sucede es que su velocidad aumenta, es decir, su aceleracin es positiva. A
o
la inversa, en la gura de la derecha, al pasar de A a B a C, la pendiente de la
tangente disminuye (se hace menos positiva). Si se trata de posicin en funcin del
o
o
tiempo, entonces, es la velocidad la que est disminuyendo, vale decir, su aceleracin
a
o
es negativa. Concluimos entonces que, efectivamente, para una curva arbitraria (en
particular si la aceleracin no es constante), el signo de la curvatura es igual al signo
o
de la aceleracin.
o
Ejemplos:
1. Para el movimiento rectil
neo uniforme, la posicin de una part
o
cula viene dada
por x(t) = x0 + v0 t. Ya hemos visto que, en ese caso, su velocidad es constante
e igual a v0 . Demostremos ahora, usando la ecuacin (2.2), que en este caso la
o
part
cula efectivamente no tiene aceleracin:
o
a(t) = l
m

v0 v0
v(t + ) v(t)
= l
m
= l 0 = 0 .
m
0
0

CAP
ITULO 2. CINEMATICA EN UNA DIMENSION

52

2. En un ejemplo anterior vimos que la posicin y velocidad de una part


o
cula que
cae libremente bajo la accin de la aceleracin de gravedad terrestre estn dadas
o
o
a
por las siguientes ecuaciones
z(t) = z0

1 2
gt
2

y
v(t) = g t .
Evaluemos la aceleracin:
o
v(t + ) v(t)
[g (t + )] (g t)]
= l
m
0

g
= l
m
= l (g) = g .
m
0
0

a(t) = l
m

El resultado indica que la aceleracin es constante y negativa. Eso signica que


o
la part
cula acelera en el sentido negativo del eje z.
En general, para una funcin arbitraria, se dene la derivada de dicha funcin
o
o
respecto a su variable de la siguiente manera:
f (t + ) f (t)
df (t)
= l
m
.
f(t) =
0
dt

Podemos decir, entonces, que la velocidad de una part


cula es la derivada de la posicin respecto al tiempo; anlogamente, la aceleracin de una part
o
a
o
cula es la derivada
de la velocidad respecto al tiempo. La notacin f(t) se debe a Newton, es ms usual
o
a
en textos de F
sica, y se reserva espec
camente a derivadas respecto al tiempo. La
notacin df /dt se debe a Leibniz, y es ms general, usndose para derivadas respecto
o
a
a
a cualquier tipo de argumento.
Observemos, en las Ecs. (2.1) y (2.2), que tanto el numerador como el denominador tienden a cero al hacer el l
mite. El resultado del cuociente queda indeterminado,
entonces. Pero al hacer el l
mite con cuidado, como vemos en todos los ejemplos
anteriores, se llega a un resultado denido.
Notemos tambin que la aceleracin es la derivada de una derivada (la velocidad).
e
o
A esto se le llama una segunda derivada. Tanto las notaciones de Newton como de
Leibniz sugieren rpidamente una notacin especial para la segunda derivada:
a
o
a(t) = v(t) = (x(t)) x(t) ,

a(t) =

d
dv(t)
=
dt
dt

dx(t)
dt

d2 x(t)
.
dt2

Ya tendremos oportunidad de profundizar en el concepto matemtico de derivaa


da, pero ahora nos interesa destacar la enorme importancia que la derivada tiene para

CAP
ITULO 2. CINEMATICA EN UNA DIMENSION

53

nuestra descripcin del movimiento. Sabemos que, por ejemplo, para un movimiento
o
con aceleracin uniforme, podemos escribir ecuaciones de itinerario para la posicin,
o
o
la velocidad y la aceleracin. Con ellas es posible describir completamente todo el
o
movimiento de la part
cula. Sin embargo, ahora es claro que, en realidad, slo neceo
sitamos una sola funcin, la posicin. Las ecuaciones de itinerario para la velocidad
o
o
y la posicin se pueden obtener a partir de ella usando una herramienta matemtica,
o
a
la derivada. Esto no es sino un ejemplo de algo que encontraremos frecuentemente a
lo largo de este curso: en F
sica, en general, nos interesa tener una descripcin tan
o
sencilla como sea posible de la Naturaleza, y usualmente obtener dicha descripcin,
o
para que sea sencilla, requiere la utilizacin de alguna Matemtica adicional. En este
o
a
caso hemos logrado, gracias a la derivada, reemplazar una descripcin complicada, en
o
trminos de tres ecuaciones de itinerario, por una descripcin sencilla, con una sola
e
o
ecuacin de itinerario, x(t). Y por qu es importante tener una descripcin sencilla?
o
e
o
Porque sabemos, y dicindolo comenzamos este cap
e
tulo, que la descripcin compleo
ta del movimiento se tiene si se conoce x(t) para todo tiempo. Tal intuicin f
o sica
hubiera sido traicionada, en algn sentido, si hubiramos tenido que mantener la
u
e
descripcin matemtica en trminos de tres ecuaciones de itinerario. Ahora, usando
o
a
e
la derivada, podemos ser eles a la intuicin original.
o

Ejemplos adicionales:
1. Consideremos una part
cula de masa m, cuya posicin a medida que transcurre
o
el tiempo viene dada por
z(t) = A cos(t) ,
donde A y son constantes. Tal movimiento de la part
cula es un movimiento
oscilatorio peridico. La amplitud de las oscilaciones es A y el per
o
odo del movimiento (es decir, el tiempo que debe transcurrir hasta que una conguracin
o
se vuelva a repetir) es
T = 2/ .
Al inverso de T se le llama frecuencia: = 1/T . A la magnitud se le llama
frecuencia angular. Se tiene que = 2.

CAP
ITULO 2. CINEMATICA EN UNA DIMENSION

54

Evaluemos la velocidad de la part


cula:
z(t + ) z(t)

1
l
m [A cos((t + )) A cos(t)]
0
A
l
m [cos(t) cos() sin(t) sin() cos(t)]
0
A
2 2
l
m
cos(t) 1
sin(t) () cos(t)
0
2
A
2 2
l
m
sin(t) ()
cos(t)
0
2
2
sin(t)
l A cos(t)
m
0
2
A sin(t)

v(t) = l
m

=
=

=
=
=

Una vez conocida la velocidad podemos, en forma anloga, calcular la aceleraa


cin:
o
v(t + ) v(t)

1
l
m [A sin((t + )) (A) sin(t)]
0
A
[sin(t) cos() + cos(t) sin() sin(t)]
l
m
0

A
2 2
l
m
sin(t) 1
+ cos(t) sin(t)
0

2
2
l A sin(t)
m
+ cos(t)
0
2

a(t) = l
m

=
=

= A 2 cos(t)
La gura 2.7 muestra la posicin, velocidad y aceleracin de la part
o
o
cula en
funcin del tiempo.
o

CAP
ITULO 2. CINEMATICA EN UNA DIMENSION

55

x ( t)
A
3

4 t

_A
v ( t)
A
3

_ A

4 t

a( t )
A2

3
2

4 t

_ A2

Figura 2.7
2. Una persona levanta un peso P , sujetando una cuerda que pasa por
una polea y caminando horizontalmente con velocidad v0 . Cul es la
a
velocidad del peso P ?
Supongamos que el largo de la cuerda es 2h (o sea, cuando la persona
est en x = 0, el cuerpo P est en
a
a
el suelo encontrndose la cuerda esa
tirada). Se tiene

1111111111111111111
0000000000000000000
11111111111 1111111111111111111
00000000000 0000000000000000000
11111111111 1111111111111111111
00000000000 0000000000000000000

h
v0
y

11111111111 11111111111111111111
00000000000 00000000000000000000
11111111111 11111111111111111111
00000000000 00000000000000000000
O

Figura 2.8
(h y) +

h2 + x2 = 2h ,

o sea,
y(t) =

h2 + x2 (t) h =

Para la velocidad obtenemos

2
h2 + v0 t2 h .

CAP
ITULO 2. CINEMATICA EN UNA DIMENSION

56

y(t + ) y(t)
0

1
2
2
= l
m
h2 + v0 (t + )2 h
h2 + v0 t2 h
0
1
2
2
2
2
= l
m
(h2 + v0 t2 ) + (2v0 t + v0 2 ) h2 + v0 t2
0

y(t) = v(t) = l

1
0

2
h2 + v0 t2

1
0

2
h2 + v0 t2

= l
m

= l
m

1+
1+

2
2
2v0 t + v0 2
1
2
h2 + v0 t2

2
2
1 2v0 t + v0 2
1
2
2 h2 + v0 t2

2
2
1 1 2v0 t + v0 2
2
0 2
h2 + v0 t2
2
v0 t

= l
m
=

2
h2 + v0 t2

Ejercicio: Demuestre que la aceleracin de P viene dada por:


o
2
a(t) = y (t) = v0

2.2.

h2
2
h2 + v0 t2

3/2

El camino inverso

En la seccin anterior se present el procedimiento que permite evaluar, partieno


o
do del conocimiento de la posicin en funcin del tiempo, la velocidad y luego la
o
o
aceleracin. Sabemos tambin, sin embargo, que es posible, al menos en el caso de
o
e
aceleracin constante, seguir el camino inverso, es decir, conociendo la aceleracin en
o
o
funcin del tiempo, calcular la velocidad y posicin. En esta seccin profundizareo
o
o
mos en esto, encontrando que este camino es posible tambin si la aceleracin no es
e
o
constante.
Supongamos que la velocidad de una part
cula en funcin del tiempo viene dada
o
por el grco mostrado en la gura 2.9.
a
Recordemos primero algo ya calculado, el caso de velocidad constante. Cul
a
ser la distancia recorrida por la part
a
cula entre los instantes ti y tf ? Entre esos dos
instantes la velocidad de la part
cula es constante (igual a v0 ), por lo tanto la distancia
recorrida ser x(tf ) x(ti ) = v0 (tf ti ). Podemos escribir
a
x(tf ) = x(ti ) + v0 (tf ti ) ,

CAP
ITULO 2. CINEMATICA EN UNA DIMENSION

57

v(t )

1111111111
0000000000
1111111111
0000000000
1111111111
0000000000
1111111111
0000000000
1111111111
0000000000
1111111111
0000000000
1111111111
0000000000
1111111111
0000000000
1111111111
0000000000
1111111111
0000000000
1111111111
0000000000
1111111111
0000000000
1111111111
0000000000
1111111111
0000000000
1111111111
0000000000
1111111111
0000000000
1111111111
0000000000
t
t

v0

v3
v2
v1

11
00
11
00
11
00
11
00
111
000
11
00
111
000
11
00
111
000
111 11
000 00
111
000
111 11
000 00
111
000
111 11
000 00
111
000
111 11
000 00
111
000 t
111 11
000 00

111
000
111 11
000 00
111
000
111 11
000 00
111
000
111 11
000 00
111
000
111 11
000 00
111
000
111 11
000 00
111
000
11
111 00
000 00
111
000
111 00
000 11
111
000
11

t1 t2 t3 t4

Figura 2.9
o sea, si una part
cula entre dos instantes (inicial y nal) se mueve a una velocidad
constante, entonces la posicin nal es igual a la posicin inicial ms el rea de la
o
o
a
a
funcin v(t) entre los instantes ti y tf .
o
Cuando la funcin v(t) no es constante la situacin es ms compleja. Intentemos
o
o
a
evaluar la distancia que recorre la part
cula entre los instantes t1 y t4 . Como la
velocidad no es constante, tomaremos algunas mediciones intermedias, separadas por
un intervalo de tiempo t. Entre t1 y t2 la distancia recorrida ser aproximadamente
a
v(t1 ) (t2 t1 ) = v(t1 ) t, entre t2 y t3 ser v(t2 ) (t3 t2 ) = v(t2 ) t, y nalmente
a
entre t3 y t4 ser aproximadamente v(t3 ) (t4 t3 ) = v(t3 ) t. La distancia total
a
recorrida ser aproximadamente
a
3

x(t4 ) x(t1 )

j=1

v(tj ) t ,

(2.3)

donde t = (t4 t1 )/3. Observe que el lado derecho de la ecuacin (2.3) es igual
o
al rea de los rectngulos mostrados en la gura 2.10. Evidentemente el resultado
a
a
anterior es slo aproximado: hemos tomado 3 mediciones intermedias y hemos suo
puesto que entre las mediciones la velocidad es constante (igual al valor de la ultima

medicin). Tambin es claro que si aumentamos el nmero de mediciones intermeo


e
u
dias obtendremos un resultado ms preciso. Para un nmero muy grande (innito)
a
u
de mediciones intermedias, el procedimiento ser exacto; en ese caso el rea de los
a
a

rectngulos ser igual al rea entre la funcin v(t) y el eje t. Si dividimos el intervalo
a
a
a
o
[ti , tf ] en N intervalos de ancho t = (tf ti )/N , entonces el proceso l
mite anterior

CAP
ITULO 2. CINEMATICA EN UNA DIMENSION

58

se escribe en la forma:
N

tf

l
m

i=1

v(ti )t

v(t) dt ,
ti

donde
tf

v(t) dt = (Area delimitada por v(t) y el eje t entre t = ti y tf ) .

ti
t

a
a
Otro modo de pensarlo es que tif signica sume las contribuciones que estn detrs
del s
mbolo desde t = ti hasta t = tf [en este caso, sume contribuciones de tamao
n
v(t) dt, que representa el rea de un rectngulo con un lado igual a v(t) y el otro igual
a
a
a una magnitud innitesimal dt].
Observemos la lgica de la notacin utilizada. Hacer el proceso l
o
o
mite anterior
signica bsicamente convertir letras griegas en latinas: t es ahora dt, y es ahora
a
una S estilizada, .
De esta manera hemos encontrado un resultado completamente general:

x(tf ) = x(ti ) + (Area entre v(t) y el eje t entre t = ti y tf ) ,

o bien,

(2.4)

tf

x(tf ) = x(ti ) +

v(t) dt .

(2.5)

ti

El s
mbolo se denomina integral. Decimos entonces que el desplazamiento de una
part
cula entre dos instantes ti y tf es la integral de la velocidad entre ti y tf .
Ejemplos:
1. Movimiento uniforme:
Si v(t) = v0 , constante, se tiene que el rea entre la curva v(t) y el eje t es:
a
tf
ti

v0 dt = v0 (tf ti ) .

Entonces,
x(tf ) = x(ti ) + v0 (tf ti ) .
2. Movimiento uniformemente acelerado:
Consideremos una part
cula cuya velocidad viene dada por
v(t) = v0 t + a0 t ,

CAP
ITULO 2. CINEMATICA EN UNA DIMENSION

59

(ver gura 2.10). Observe que v0 es la velocidad de la part


cula en el instante
t = 0. El rea bajo la curva entre los instantes ti = 0 y tf es el rea de una
a
a
seccin rectangular y una triangular, resultando:
o
tf
0

Es decir,

1
v(t) dt = v0 tf + (v(tf ) v0 ) tf
2
1
= v0 t f + a 0 t 2 .
f
2
1
x(tf ) = x(0) + v0 tf + a0 t2 .
f
2

v( t )

v ( tf )

111111111111111111111
000000000000000000000
111111111111111111111
000000000000000000000
111111111111111111111
000000000000000000000
111111111111111111111
000000000000000000000
111111111111111111111
000000000000000000000
111111111111111111111
000000000000000000000
111111111111111111111
v0000000000000000000000
111111111111111111111
000000000000000000000
111111111111111111111
000000000000000000000
111111111111111111111
000000000000000000000
111111111111111111111
000000000000000000000
111111111111111111111
000000000000000000000
111111111111111111111
000000000000000000000
111111111111111111111
000000000000000000000
111111111111111111111
000000000000000000000
t

Figura 2.10
En un caso un poco ms general, en que el instante inicial es t = ti = 0, y la
a
velocidad inicial es v(ti ) = vi , se tiene, para un movimiento acelerado uniforme
con aceleracin a,
o
tf
ti

con lo cual

1
(vi + at) dt = vi (tf ti ) + a(tf ti )2 ,
2

1
x(tf ) = x(ti ) + vi (tf fi ) + a(tf ti )2 .
2

Conociendo la posicin x(t) de una part


o
cula, siempre es posible determinar su
velocidad. El rec
proco no es cierto: si se conoce la velocidad v(t) no es posible determinar la posicin; lo unico que se puede determinar es el desplazamiento entre dos
o

instantes. En otras palabras, si conocemos v(t), debemos conocer adems la posicin


a
o
en algn instante para poder determinar x(t).
u

CAP
ITULO 2. CINEMATICA EN UNA DIMENSION

60

Las relaciones que permiten obtener la velocidad si se conoce la aceleracin a(t),


o
son anlogas a las que relacionan la posicin con la velocidad:
a
o

v(tf ) = v(ti ) + Area entre a(t) y el eje t entre t = ti y tf .

(2.6)

tf

a(t) dt .

v(tf ) = v(ti ) +

(2.7)

ti

Ejemplo: Movimiento uniformemente acelerado.


Suponga que la aceleracin de una part
o
cula es constante, a(t) = a0 . Usando (2.6) se
deduce que
v(t) = v(0) + a0 t .
Haciendo uso del resultado obtenido en el ejemplo anterior se obtiene nalmente que
1
x(t) = x(0) + v(0) t + a0 t2 .
2
Observe que x(0) y v(0) son la posicin y la velocidad de la part
o
cula en el instante
t = 0.

2.3.

Mximos y m
a
nimos

Considere una funcin f (t) suave (o sea, sin saltos ni puntas). Ya sabemos (ver
o
ultimo problema de la seccin anterior) que f(t) est relacionado con la pendiente de

o
a
las tangentes de la funcin f (t). Observemos que para valores de t en los cuales f(t) =
o
0, la funcin f (t) tiene un mximo o m
o
a
nimo (local). Tambin podemos invertir la
e
argumentacin: encontrar los mximos y m
o
a
nimos de una funcin f (z) es equivalente
o
a encontrar los ceros de la funcin derivada
o
g(z) = l
m

f (z + ) f (z)
.

Ejemplo: Suponga que un agricultor tiene L metros de malla para construir un corral rectangular. El agricultor desea aprovechar una muralla de piedra (recta) para
obtener un corral mayor. Qu dimensioe
nes deber tener el corral para que su rea
a
a
sea mxima?
a

b
a

malla
pared

11111111111111111111111
00000000000000000000000
11111111111111111111111
00000000000000000000000
11111111111111111111111
00000000000000000000000

Figura 2.11

CAP
ITULO 2. CINEMATICA EN UNA DIMENSION

61

Solucin: Sean a y b los largos del gallinero (ver gura 2.11). El largo de la malla es
o
L = 2a + b, mientras que el rea del gallinero es A = a b. Despejando b de la primera
a
ecuacin y sustituyndolo en la segunda se obtiene:
o
e
A = a (L 2a) .
El rea es una funcin de a. Tanto para a = 0 como para a = L/2 se tiene que
a
o
A = 0. Para algn valor intermedio el rea del gallinero ser mxima. Para resolver
u
a
a a
el problema debemos encontrar el mximo de la funcin f (a) = a (L 2a). Para ello
a
o
encontremos los ceros de la funcin derivada
o
g(a) = l
m

1
f (a + ) f (a)
= l
m [(a + ) (L 2(a + )) a (L 2a)] = L 4a .
0

La funcin g(a) tiene un (nico) cero para a = L/4. Luego para ese valor de a el rea
o
u
a
del gallinero ser mxima.
a a

2.4.

Elementos del clculo innitesimal e integral


a

A continuacin se presenta un resumen de algunos resultados del clculo que se


o
a
usarn extensivamente en lo que sigue. Se dejar para los cursos de matemticas la
a
a
a
demostracin rigurosa de los resultados. Supondremos impl
o
citamente que las funciones que se usan ms abajo tienen todas las propiedades necesarias para que los
a
teoremas planteados sean vlidos (por ejemplo, sean funciones continuas, derivables,
a
acotadas, etc.).
Sean f (t) y g(t) dos funciones y un nmero (real o complejo). La funcin
u
o
derivada df (t)/dt, relacionada con la pendiente de la funcin f (t), por denicin es
o
o
1
df (t)
= f(t) = l
m
[f (t + ) f (t)] .
0
dt
Propiedades:
a)
b)
c)
d)

d(f (t))
= f(t) .
dt
d(f (t) + g(t))
= f(t) + g(t) .

dt
d(f (t) g(t))
= f(t) g(t) + f (t) g(t) .

dt
df (g(t))
= f(g(t)) g(t) .

dt

CAP
ITULO 2. CINEMATICA EN UNA DIMENSION

62

Demostracin de c):
o
De la denicin de la derivada se deduce que, para muy pequeo
o
n
f (t + ) = f (t) + f(t) .
Con esta relacin, y una anloga para la funcin g(t), se deduce que
o
a
o
d(f (t) g(t))
dt

1
[f (t + ) g(t + ) f (t) g(t)]
0
1
(f (t) + f(t)) (g(t) + g(t)) f (t) g(t)

= l
m
0
1
= l
m
f(t) g(t) + f (t) g(t) + 2 f(t) g(t)

0
= f(t) g(t) + f (t) g(t) .

= l
m

Demostracin de d):
o
1
d
f (g(t)) = l
m
[f (g(t + )) f (g(t))]
0
dt
1
= l
m
[f (g(t) + g(t)) f (g(t))]

0
Pero, usando nuevamente la ecuacin (), se tiene
o
f (g + g) = f (g) + ( g) f(g) ,

luego
d
1
f (g(t)) + g(t) f(g(t)) f (g(t))

f (g(t)) = l
m
0
dt
= f(g(t)) g(t) .

De los ejemplos que hemos desarrollado hasta el momento, se sigue que


d(c)
= 0 , c = constante
dt
d(t)
=1,
dt
d(t2 )
= 2t .
dt
En general, se puede mostrar que
dt
= t1 ,
dt

()

CAP
ITULO 2. CINEMATICA EN UNA DIMENSION

63

para todo nmero real . Adems,


u
a
d sin t
= cos t ,
dt
d cos t
= sin t .
dt
Con estos resultados bsicos, ms las propiedades enunciadas de la derivada, se
a
a
pueden encontrar las derivadas de much
simas funciones. Consideremos, por ejemplo,
f (t) = A cos t. Observando que A es un escalar, y que f (t) = g(h(t)), con g(t) = cos t
y h(t) = t, encontramos:
df
d(A cos t)
d(cos t)
d(cos h)
dh
=
=A
=A
dt
dt
dt
dh
h=t dt
d(t)
d(t)
= A sin(h(t))
= A sin(h(t))
dt
dt
= A sin(t) ,
resultado consistente con lo obtenido en la Sec. 2.1.
En un grco de la funcin f (t) en funcin
a
o
o
de t, la expresin (integral)
o

f (t)

f (t)

f (t) dt

A=
a

111111111
000000000
111111111
000000000
111111111
000000000
111111111
000000000
111111111
000000000
111111111
000000000
111111111
000000000
111111111
000000000
111111111
000000000
111111111
000000000
111111111
000000000
111111111
000000000
111111111
000000000
111111111
000000000
111111111
000000000
111111111
000000000
111111111
000000000
111111111
000000000
111111111
000000000
111111111
000000000
111111111
000000000
111111111
000000000

representa al rea delimitada por la funa


cin f (t) y el eje t entre t = a y t = b (ver
o
gura).
0

Figura 2.23
Propiedades:
b

a)

f (t) dt .

f (t) dt =
a

b)

f (t) dt .

f (t) dt +

f (t) dt =
a

c)

g(t) dt .

f (t) dt +

[ f (t) + g(t) ] dt =

CAP
ITULO 2. CINEMATICA EN UNA DIMENSION

64

En muchos casos es posible evaluar la integral A anal


ticamente. Para ello, se debe
encontrar una funcin F (t) tal que su derivada sea la funcin que aparece tras el
o
o
s
mbolo integral, o sea, tal que dF (t)/dt = f (t). Entonces
b

f (t) dt = F (t)

A=
a

F (a) F (b) .

En la Sec. 2.1 encontramos ejemplos inmediato de esto. En efecto, v(t) = dx(t)/dt,


de modo que
b

v(t) dt = x(t)
a

= x(b) x(a) ,

que no es sino el resultado ya conocido de que la integral de la velocidad es el desplazamiento.


Usando las derivadas de funciones ya conocidas, podemos escribir algunas integrales utiles:

t dt =

t+1
+1

sin t dt =

cos t

cos t dt =

sin t

a
b
a

2.5.

= 1 ,

b
a

b
a

Problemas

1. Suponga que la altura de cierto proyectil en funcin del tiempo viene dada por
o
2 + z , con z = 125 m, t = 5 s y a = 5 m/s2 .
la relacin z(t) = a0 (t t0 )
o
0
0
0
0
a) Graque la altura del proyectil en funcin del tiempo desde t = 0 hasta
o
t = 12 s.
b) En qu instante choca el proyectil contra el suelo?
e
c) Encuentre grcamente la velocidad instantnea (es decir, mida las pena
a
dientes de las tangentes) en los instantes t=0 s, t=2 s, t=4 s, t=6 s, t=8 s
y t=10 s. Graque su resultado.
2. Un conductor maneja su coche 10 km a una velocidad de 90 km/h y luego otros
10 km a 70 km/h. Cul es la rapidez promedio durante el trayecto de 20 km?
a
(La respuesta no es 80 km/h.)

CAP
ITULO 2. CINEMATICA EN UNA DIMENSION

65

3. La gura 2.12 muestra la posicin de una part


o
cula en funcin del tiempo.
o
Encuentre la velocidad promedio durante los siguientes intervalos de tiempo:
a)

0s < t <4s

b)

7 s < t < 10 s

c)

0 s < t < 13 s

d)

10 s < t < 13 s

(Respuesta: v = 0.154 m/s )

Figura 2.12

CAP
ITULO 2. CINEMATICA EN UNA DIMENSION

66

Figura 2.13
4. La gura 2.13 muestra la posicin de una part
o
cula en funcin del tiempo. En
o
qu instantes o en qu intervalos de tiempo
e
e
a) la velocidad (instantnea) es cero?
a
b) la velocidad es positiva?
c) la velocidad es negativa?
d) el mdulo de la velocidad es mximo?
o
a
e) la velocidad es constante?
f ) la aceleracin es negativa?
o
5. Suponga que la posicin de una part
o
cula en funcin del tiempo viene dada por
o
z(t) =

t
.
1 + t2

a) Graque z(t) en el intervalo de tiempo 4 < t < +4.

b) Encuentre la velocidad instantnea en funcin del tiempo evaluando


a
o
z(t + t) z(t)
.
t0
t

v(t) = l
m
Graque v(t).

CAP
ITULO 2. CINEMATICA EN UNA DIMENSION

67

Figura 2.14
6. La gura 2.14 muestra la posicin de una part
o
cula en funcin del tiempo.
o
a) Encuentre la velocidad promedio en el intervalo de tiempo 2 s < t < 10 s.
b) Encuentre la velocidad instantnea para t = 10 s.
a
c) En qu instante o instantes la velocidad (instantnea) de la part
e
a
cula es
nula?
d) En qu instante la rapidez es mxima?
e
a
e) En qu instante la aceleracin es nula?
e
o
7. Suponga que la posicin de una part
o
cula en funcin del tiempo viene dada por
o
z(t) = t 4 cos t .
a) Graque z(t) en el intervalo de tiempo 0 < t < +6.
b) A partir del grco responda las siguientes preguntas:
a
1)
2)
3)
4)

En
En
En
En

qu
e
qu
e
qu
e
qu
e

instante la velocidad es nula?


instantes la part
cula se encuentra en el origen?
intervalos de tiempo la velocidad es negativa?
intervalos de tiempo la aceleracin es positiva?
o

c) Encuentre la velocidad instantnea en funcin del tiempo evaluando


a
o
v(t) = l
m

t0

z(t + t) z(t)
.
t

d) Graque v(t) encontrada en la parte anterior. A partir del grco responda


a
las siguientes preguntas:
1) En qu instante la velocidad es nula?
e

CAP
ITULO 2. CINEMATICA EN UNA DIMENSION

68

2) En qu intervalos de tiempo la velocidad es negativa?


e
3) En qu intervalos de tiempo la aceleracin es positiva?
e
o
(Compare las respuestas con las de la parte b)).
8. La gura 2.15 muestra la velocidad de una part
cula en funcin del tiempo.
o
En qu instantes o en qu intervalos de tiempo:
e
e
a) La velocidad es cero?
b) La velocidad es constante?
c) La velocidad es positiva?
d) La aceleracin es nula?
o
e) La aceleracin es positiva?
o
f ) El mdulo de la velocidad es mximo?
o
a
g) El mdulo de la aceleracin es mximo?
o
o
a
h) Cul es la distancia que recorre la part
a
cula entre t = 2 s y t = 4 s?
i ) Si en el instante t = 0 la part
cula se encuentra en el origen (es decir, si
s(0) = 0), haga un grco aproximado del desplazamiento s(t).
a
j ) Haga un grco aproximado de s(t) si s(0) = 4 m.
a
Respuestas: a) En t = 2 s y t = 8.5 s; b) A partir de t = 10 s, se podr decir
a
tambin que en el instante t = 6 s la velocidad es constante; c) Entre t = 2 s y
e
t = 8.5 s; d) Misma respuesta de la parte b); e) Entre t = 0 s y t = 6 s; f) En
t = 6 s; g) Entre t = 7 s y t = 9 s; h) Entre t = 2 s y t = 4 s la velocidad media
es de 1 m/s, luego la distancia recorrida es de 2 m (note que esto coincide con
el rea bajo la curva).
a

Figura 2.15

CAP
ITULO 2. CINEMATICA EN UNA DIMENSION

69

a ( t ) [m/s 2]

2
I
III
0
4

8
II

12
t [s]

Figura 2.16
9. La gura 2.16 muestra la aceleracin de una part
o
cula en funcin del tiempo.
o
a) Si en el instante t = 0 s la part
cula est en reposo, encuentre la velocidad
a
de la part
cula en cada instante. Graque!
b) Calcule el tamao de las reas I, II y III. Qu unidades tienen? Qu ren
a
e
e
lacin hay entre estas reas y la parte a) de este problema?
o
a
c) Repita lo hecho en la parte a), pero suponiendo que en el instante t = 0 la
part
cula tiene una velocidad v0 = 8 m/s. Graque!
10. En cada una de las siguientes expresiones para la posicin s(t) de una part
o
cula,
encuentre una expresin anal
o
tica para la velocidad instantnea:
a
a) s(t) = at2 + bt + c
b) s(t) = at
c) s(t) = a cos (t + )
En las ecuaciones anteriores a, b, c, , y son constantes.
11. Para cada una de las siguientes expresiones para la aceleracin a(t) de una
o
part
cula (a en m/s2 y t en s), encuentre la expresin ms general para la
o
a
velocidad v(t) y la posicin x(t).
o
a)

a(t) = a0

b)

a(t) = a0 cos (t)

CAP
ITULO 2. CINEMATICA EN UNA DIMENSION

70

v ( t ) [m/s ]

0
4

12
t [s]

Figura 2.17
En las expresiones anteriores, a0 y son constantes.
12. Un observador suelta una piedra desde el techo de un edicio. El sonido de la
piedra chocando contra el suelo se escucha despus de t0 = 6 s.
e
a) Si la velocidad del sonido es c = 340 m/s, encuentre la altura del edicio.
(Ignore los efectos del roce del aire, que en la prctica, para este problema,
a
no son despreciables.)
b) Demuestre que si gt0 /c 1, entonces la altura del edicio viene aproximadamente dada por
h=

1 2
gt
2 0

gt0
c

13. Dos trenes A y B, inicialmente separados por una distancia de 13 km, viajan
hacia su encuentro a una velocidad de 30 km/h respecto a la v Desde A parte
a.
una paloma mensajera que tarda 10 minutos en llegar al tren B. Calcule la
velocidad con que vuela la paloma respecto al tren A. Resuelva el problema en
forma grca y luego en forma anal
a
tica.
14. La gura 2.17 muestra la velocidad de una part
cula en funcin del tiempo.
o
a) Si en el instante t = 0 s la part
cula se encuentra en el origen (es decir,
x(0) = 0), encuentre la posicin de la part
o
cula en cada instante. Graque.

CAP
ITULO 2. CINEMATICA EN UNA DIMENSION

71

b) Repita lo hecho en la parte a), pero suponiendo que en el instante t = 0 se


tiene x(0) = 3 m.
15. Desde un puente de 60 m de altura se deja caer una piedra. Una segunda piedra
se arroja verticalmente hacia abajo 1 s ms tarde. Ambas piedras llegan al suelo
a
simultneamente. Cul fue la velocidad inicial de la segunda piedra? (Desprecie
a
a
el roce del aire.)
16. Un cohete se dispara verticalmente, subiendo con aceleracin constante de 20 m/s2
o
respecto a la plataforma de lanzamiento durante 1 minuto. En ese momento se
agota su combustible y contina movindose slo bajo la accin de la aceleracin
u
e
o
o
o
de gravedad.
a) Cul es la mxima altura que alcanza?
a
a
b) Cul es el tiempo transcurrido desde que despega hasta volver a caer
a
sobre la plataforma?
c) Graque la posicin y velocidad en funcin del tiempo.
o
o

17. Panchito deja caer una pelota desde una altura h. La pelota, cada vez que choca
contra el suelo, rebota con una rapidez igual a aqulla con la cual lleg al suelo
e
o
multiplicada por , donde es una constante 0 < < 1. Encuentre:
a) La altura que alcanza la pelota despus del primer rebote.
e
b) La altura que alcanza despus del segundo rebote.
e
c) La altura que alcanza despus del k-simo rebote.
e
e
d) La distancia total recorrida desde que se solt la pelota hasta el k-simo
o
e
rebote.
e) La distancia total recorrida por la pelota hasta que se detiene (tome k
en la expresin anterior).
o
2(k1) 1
2 1

Respuestas: c) 2k h ; d) h + 2h2

18. Un automovilista pasa a exceso de velocidad frente a un retn policial. 5 minutos


e
ms tarde sale en su persecusin un polic motorizado a una velocidad de
a
o
a
120 km/h. Despus de 40 minutos, el polic da alcance al infractor. Cul era
e
a
a
la velocidad del infractor?
19. Consideremos el movimiento de una esfera en un medio viscoso (en ausencia de
fuerzas gravitacionales). La aceleracin que sufre la esfera es proporcional a su
o
velocidad, pero en direccin contraria, es decir a(t) = v(t), donde es una
o

CAP
ITULO 2. CINEMATICA EN UNA DIMENSION

72

constante. Supongamos que = 0.01 s1 y la velocidad inicial de la esfera es


|v0 | = 50 m/s. Encuentre numricamente la distancia s(t) recorrida por la esfera
e
y graf
quela. Para resolver el problema note que, si es un pequeo intervalo
n
de tiempo, entonces
v(t + ) v(t) + a(t)
s(t + ) s(t) + v(t)

20. Considere dos varillas muy largas: una ja horizontalmente y la otra formando
un ngulo constante con la primera, y movindose verticalmente con rapidez
a
e
v0 constante (ver gura 2.18). Determine la velocidad con que se mueve el punto
de interseccin de las dos varillas (tal punto de interseccin no corresponde al
o
o
movimiento de algn objeto f
u
sico real).

Figura 2.18
21. Un pasajero corre con velocidad de 4 m/s para alcanzar un tren. Cuando est a
a
una distancia d de la portezuela ms prxima, el tren comienza a moverse con
a
o
una aceleracin constante a=0.4 m/s2 , alejndose del pasajero.
o
a
a) Si d=12 m y el pasajero sigue corriendo, alcanzar a subirse al tren?
a
En caso armativo, cunto tiempo tarda en hacerlo, desde que el tren
a
comienza a moverse?
b) Haga un grco de la funcin xt (t) del tren. En el mismo grco dibuje
a
o
a
la funcin xp (t) correspondiente al pasajero para diversos valores de la
o
distancia de separacin d. Encuentre el valor cr
o
tico dc para el cual el
pasajero alcanza apenas el tren.
c) Para la separacin cr
o
tica dc , cul es la velocidad del tren cuando el paa
sajero lo alcanza?
22. Desde un edicio se lanza una piedra A con una velocidad inicial vertical hacia
abajo v0 = 30 m/s. Desde el suelo, al pie del edicio y en el mismo instante, se

CAP
ITULO 2. CINEMATICA EN UNA DIMENSION

73

lanza una piedra B hacia arriba. Las dos piedras chocan a una altura h = 30 m,
siendo en ese instante la rapidez de ambas piedras la misma. Encuentre el tiempo
que transcurre entre el lanzamiento y la colisin. (Use para g el valor 10 m/s2 .)
o

Respuesta: t = 3 1 s.
23. Considere un avin de pasajeros cuya velocidad de aterrizaje es de unos 400 km/h.
o
Suponga que la desaceleracin del avin es uniforme. Encuentre el valor que deo
o
be tener sta para que el avin llegue al reposo en una pista de 1200 m.
e
o
Respuesta: a =5,15 m/s2
24. Cules son las dimensiones del cilindro de mximo volumen que puede ser
a
a
inscrito en una esfera de radio R?
25. En Paine un agricultor tiene la posibilidad de realizar una (y slo una) exporo
tacin de sand de su plantacin. Al comienzo de la temporada el precio es
o
as
o
bueno, pero la produccin no es grande. En efecto, al comienzo tiene 6 toneladas
o
para vender y el precio es de $40.000/ton. Por cada d que demore la exportaa
cin puede exportar 0.5 toneladas adicionales; sin embargo, el precio disminuye
o
en aproximadamente $800/ton. Cunto tiempo deber esperar para realizar
a
a
la exportacin si desea maximizar las entradas?
o
Respuesta: 19 d
as.
26. A partir de un tronco de 27 cm de dimetro se desea aserrar una viga de seccin
a
o
rectangular que tenga la mayor resistencia posible. La resistencia de una viga
horizontal apoyada en sus extremos, en primera aproximacin, es proporcional
o
al ancho y proporcional al cuadrado de su altura. Cules sern las dimensiones
a
a
de la viga?
27. Un salvavidas ubicado en el punto A en
una playa debe socorrer a un nadador ubicado en el punto B (ver gura 2.19). La
velocidad con que puede correr el salvavidas en la arena es v1 y la velocidad con
que avanza en el agua es v2 . Sea P el lugar ptimo en el cual el salvavidas debe
o
ingresar al agua para que tarde el menor
tiempo posible en el trayecto de A a B.
Demuestre que en ese caso se satisface
v1
sin
=
.
sin
v2

Figura 2.19

CAP
ITULO 2. CINEMATICA EN UNA DIMENSION

74

Notemos que esta expresin es anloga a la ley de Snell para la refraccin de


o
a
o
un rayo de luz.
28. Qu dimensiones (interiores) tiene un recipiente cil
e
ndrico, cuya capacidad es
de un litro, si la forma se ha elegido de tal manera que en su confeccin se use
o
la menor cantidad de material posible?
29. Considere cierto objeto A que se mueve a lo largo del eje x tal como se describe

a continuacin:
o
i) En el instante t = 0 se encuentra en x0 = 4 [m] y su velocidad es
v0 = 2 [m/s].
ii) Durante los primeros cuatro segundos la velocidad permanece constante.
iii) A partir del instante t = 4 [s], el objeto frena uniformemente hasta quedar
con la mitad de la velocidad. Durante este proceso de frenado la part
cula
avanza 3 [m].
iv) Luego mantiene esa velocidad durante 2 [s].
v) Luego la part
cula acelera (en sentido negativo) con una aceleracin conso
2 ] hasta que la velocidad sea v = 3 [m/s].
tante a0 = 2 [m/s
1

vi) A continuacin se desplaza con la velocidad v1 hasta llegar a dos metros


o
del punto de partida.

vii) Finalmente la part


cula A frena uniformemente hasta quedar en reposo en
el punto de partida (x0 = 4 [m]).
a) Haga un grco detallado de x(t) y v(t).
a
b) Encuentre la velocidad media de la part
cula A entre los instantes t = 6 [s]
y t = 13 [s].
c) En qu instante el alejamiento desde el punto de partida es mximo y
e
a
cunto es ese alejamiento?
a
d) Un segundo mvil B parte en t = 0 desde el origen y se deplaza con
o
velocidad constante vB = 1 [m/s] a lo largo de la misma recta que A.
Suponga que cuando los dos mviles se encuentran por primera vez, B se
o
detiene. En qu instante volvern a encontrarse?
e
a
30. Un malabarista desea hacer piruetas manteniendo en forma rotativa, con una
mano, tres manzanas en el aire. Si el malabarista desea hacer lanzamientos cada
0,5 s, determine la altura a la cual usted le aconsejar lanzar cada manzana.
a

CAP
ITULO 2. CINEMATICA EN UNA DIMENSION

75

31. Desde la altura H con respecto al piso se deja caer un macetero. En ese instante,
y desde el primer piso, un ascensor acelera hacia arriba con aceleracin g
o
( < 1). Si el ascensor tiene una altura h (h < H) y parte del reposo, calcule el
tiempo que demora el macetero en pasar desde el techo al piso del mismo. Para
no hacer trgica la situacin, suponga que la trayectoria (recta) del macetero
a
o
pasa al lado del ascensor.
32. Dos mviles A y B (puntuales) estn restringidos a moverse sobre el eje x de
o
a
cierto sistema de coordenadas. Inicialmente A se desplaza a 10 m/s, mientras
que B se encuentra en reposo en el origen del sistema de coordenadas. En t = 0
cuando A se encuentra en xA = 100 m, el mvil B comienza a ser uniformemente
o
acelerado en la direccin positiva del eje x con aceleracin a1 = 1 m/s2 . Este
o
o
movimiento contina hasta que B se encuentra a 22 m de A. Entonces B deja
u
de acelerar y simultneamente env un mensaje al mvil A, que demora 0,5 s
a
a
o
en llagar a destino. Tan pronto A recibe el mensaje, se detiene.
a) Cul es la velocidad c con que se propaga el mensaje entre A y B? Suponga
a
que la velocidad con que viaja el mensaje es constante.
b) Cul es la velocidad de B en el instante en que env el mensaje?
a
a
c) Cul es el desplazamiento de B entre t = 0 y el instante en que choca con
a
A?
d) Cul es la velocidad media de B entre t = 0 y el instante en que choca
a
con A?
33. Un mvil describe un movimiento rectil
o
neo. En la gura se muestra su velocidad
en funcin del tiempo. Sabiendo que en el instante t = 0 el mvil est en x = 0:
o
o
a
a) Graque su aceleracin en funcion del tiempo.
o
b) Calcule el desplazamiento total del mvil hasta el instante t = 8 s.
o
c) Escriba la expresin de la posicin x(t) en funcin del tiempo, entre los
o
o
o
tiempos t = 1 s y t = 3 s, y desde t = 3 s a t = 4 s.
d) Encuentre la aceleracin media entre t = 1 s y t = 8 s.
o

CAP
ITULO 2. CINEMATICA EN UNA DIMENSION

76

v [m/s]

20

10

t [s]
1

34. Suponga que la posicin de una part


o
cula en funcin del tiempo viene dada por
o
z(t) =

t
.
1 + cos2 t

a) Graque z(t) en el intervalo de tiempo 10 < t < +10.

b) Encuentre la velocidad instantnea en funcin del tiempo evaluando


a
o
z(t + t) z(t)
.
t0
t

v(t) = l
m
Graque v(t).

35. Repita el problema anterior, pero para la funcin


o
z(t) =

sin t
.
1 + t2

36. Dos caracoles de mala clase asaltaron el Banco del Jard justo antes de que
n,
lo cerraran. Una vez perpetrado el delito, justo a las 14.00 hrs., se lanzaron en
una loca carrera hacia su escondite (ver gura). El caracol l
der avanzaba con
una velocidad de 10 cm/min, y su cmplice a 8 cm/min.
o
A las 14.05 hrs., al percatarse de los hechos, una tortuga se abalanz sobre los
o
malhechores, partiendo desde el Banco y desde el reposo, con una aceleracin
o
de 0.8 cm/min2 . Tras una vertiginosa persecucin, la tortuga da alcance al
o
caracol cmplice, y le ordena al otro: Detngase! El caracol l
o
e
der desacelera
inmediatamente, detenindose al cabo de 30 cm.
e
a) A qu hora alcanza la tortuga al caracol cmplice?
e
o
b) A qu distancia del Banco se encuentran la tortuga y los caracoles cuando
e
aqulla da alcance al caracol cmplice?
e
o

CAP
ITULO 2. CINEMATICA EN UNA DIMENSION

77

c) Qu aceleracin tiene el caracol l


e
o
der luego de que la tortuga le ordena
detenerse?
d) A qu hora se detuvo el caracol l
e
der?
e) Represente, en un mismo grco, la posicin de ambos caracoles y la tora
o
tuga como funcin del tiempo.
o
Banco del Jardn

Escondite

Asalto al Banco del Jard


n
Sugerencia: Puede serle util, para determinar la solucin con sentido f

o
sico,
estudiar el l
mite t0 0, donde t0 es la diferencia de tiempo entre la salida de
los caracoles y la tortuga.
37. Considere un mvil A que se desplaza a lo largo de una recta. De este mvil se
o
o
sabe lo siguiente:
i) En el instante t = 0 se encuentra a 4 m del origen, o sea, x(0) = 4 m.

ii) Entre t = 0 y t = 3 s se mueve con velocidad constante v = 2 m/s.

iii) A partir de t = 3 s el mvil frena uniformemente durante 2 s hasta quedar


o
en reposo (en t = 5 s).
iv) Entre t = 5 y t = 7 s el mvil se mantiene en reposo.
o
v) Finalmente, a partir de t = 7 s el mvil acelera uniformemente hasta
o
adquirir una velocidad nal vf = 6 m/s. Durante este ultimo proceso de

aceleracin, el mvil A se desplaza 9 m.


o
o
Usando la informacin anterior:
o
a) Haga un grco de la velocidad del mvil en funcin del tiempo.
a
o
o
b) Haga un grco de la posicin en funcin del tiempo.
a
o
o
c) Determine la velocidad media del mvil entre los instantes t = 2 y t = 7 s.
o
d) Determine la distancia recorrida por el mvil A entre t = 3 y t = 4 s.
o
e) Suponga que un segundo mvil B, que se mueve con velocidad constante,
o
se encuentra con el mvil A en los instantes t = 3 y t = 6 s. Determine la
o
posicin que el mvil B ten en el instante t = 0.
o
o
a

CAP
ITULO 2. CINEMATICA EN UNA DIMENSION

78

38. El rodaje de la ultima pel

cula de James Bond hab causado gran expectacin


a
o
entre los habitantes de un pequeo pueblito en el norte de Chile. Pero aunque la
n
mitad del pueblo estaba emocionado con la perspectiva de aparecer en una gran
produccin de Hollywood, la otra mitad, encabezada por el alcalde, protestaba
o
porque el pueblo aparecer en la pel
a
cula con otro nombre, y ni siquiera aparecer como chileno. El alcalde, encendido nacionalista, decidi tomar cartas en
a
o
el asunto y protest ocialmente ante la produccin de la pel
o
o
cula, pero nadie
lo tom muy en serio.
o
Contrariado por su fracaso, el alcalde se present de improviso en la lmacin,
o
o
justo cuando Daniel Craig, caracterizado como el agente secreto, caminaba por
una calle durante una toma. El alcalde, dispuesto a todo, rompi el cerco de
o
seguridad y corri hacia Bond, arrebatndole su arma de utiler Bond, soro
a
a.
prendido, se detuvo, limitndose a ver cmo el alcalde se alejaba de l con veloa
o
e
cidad constante vA . Slo despus de un tiempo T Bond reaccion, y comenz a
o
e
o
o
acelerar con aceleracin constante aJB .
o
Durante cunto tiempo debe acelerar Bond, James Bond, para alcanzar al
a
alcalde?
Cunta distancia alcanza a recorrer el alcalde despus de arrebatar el arma,
a
e
antes de ser alcanzado por Bond, James Bond?

vA

vA

(a)

vA

(c)

aJB

(b)

vA

(d)

39. Dos part


culas A y B que se mueven en movimiento unidimensional parten desde
el origen. La part
cula A parte en t = 0 con velocidad vA = 10 m/s. La part
cula
B parte un segundo despus con velocidad vB = 10 m/s. Ambas part
e
culas
desaceleran con aceleracin a = 6 m/s2 . Determine la mxima distancia entre
o
a
ellos antes de que se crucen.
40. Una part
cula se mueve a lo largo de una l
nea recta. Inicialmente se encuentra
en el origen, con velocidad v0 > 0 y sujeta a una aceleracin a < 0. Cul es el
o
a

CAP
ITULO 2. CINEMATICA EN UNA DIMENSION

79

mximo valor de la posicin que alcanza dicha part


a
o
cula? Cundo pasa por el
a
origen nuevamente?
41. Hormiguita se sent sola. Con cinco millones de hermanos es dif tener la
a
cil
atencin de mam. A Hormiguita nunca le leyeron un cuento, nunca la arroparon
o
a
en la noche, nunca la mimaron cuando estaba enferma. Pero a pesar de que su
madre siempre parec estar ocupada poniendo nuevos huevos, Hormiguita la
a
segu queriendo, y ahora que se acercaba el D de la Madre Hormiga, se lo
a
a
dir Se acerc a la Galer Real de la colonia, hasta estar a una distancia L
a.
o
a
de mam. Pero, en t = 0, mam se comenz a mover con velocidad vA hacia la
a
a
o
derecha. Hormiguita se puso muy triste al ver a mam alejarse, pero en t = t0
a
Hormiguita reaccion, y comenz a moverse con velocidad vB > vA hacia la
o
o
derecha.

vA
Hormiga Reina

Hormiguita

t=0
L
vB

vA
t= t0

Hormiguita al encuentro de mam.


a
Cunto tiempo tarda Hormiguita (desde que comenz a moverse) en alcanzar
a
o
a mam? Qu distancia d alcanz a recorrer mam antes de ser alcanzada por
a
e
o
a
su tierna hija?
Represente, en el mismo grco, la posicin en funcin del tiempo para Hormia
o
o
guita y su mam. Repita lo anterior para la velocidad en funcin del tiempo.
a
o
42. Marcela caminaba absorta por el pasillo. Hab sido un mes muy ocupado en
a
el trabajo, y eso la hab ayudado a olvidar a Daniel. Alguna vez se hab proa
a
metido no involucrarse con nadie de la ocina, pero sucedi, y ahora no sab
o
a
muy bien cmo terminar. . . Eran casi las 9, y Marcela se dirig a una reunin
o
a
o
al Departamento de Recursos Humanos, movindose con velocidad vM hacia la
e

CAP
ITULO 2. CINEMATICA EN UNA DIMENSION

80

derecha. Hac semanas que no visitaba el cuarto piso, y quizs por eso no se
a
a
dio cuenta de que, desde el otro lado del pasillo, Daniel se dirig directamente
a
hacia ella, con velocidad vD hacia la izquierda. Ese ritmo familiar, pausado al
caminar, la sac de su distraccin, y en t = 0 Marcela not que Daniel estaba a
o
o
o
una distancia R de ella. Quiso disimular y sigui caminando, esperando quizs
o
a
que Daniel entrara a la ocina de Contabilidad. Mas no lo hizo, y Daniel segu
a
aproximndose, incomodndola. En t = T , Marcela se detuvo instantneamena
a
a
te, para evitar el encuentro. En ese mismo instante, Daniel comenz a frenar
o
u
con aceleracin constante a. El an cre que Marcela pod ser la mujer de
o
a
a
su vida, si slo pudieran hablar una vez ms. . . Marcela segu detenida, frente
o
a
a
al escritorio de la chismosa de Lorena, de hecho, quien segu atenta toda la
a
escena, reuniendo material para amenizar la usualmente aburrida hora de colacin. Daniel continuaba acercndose, cada vez ms lentamente, hasta detenerse
o
a
a
completamente justo al encontrarse con Marcela. En qu tiempo tf Daniel se
e
encuentra con Marcela, y cul fue su aceleracin a?
a
o
Marcela y Daniel quedaron frente a frente unos segundos, sin hablar. Lorena
agudizaba sus sentidos para no perder detalle. La puerta de Recursos Humanos se abri, y Marcela escuch su nombre. Las 9 en punto. El trabajo deb
o
o
a
comenzar.

vD

vM

t= 0

R
a
t= T

Marcela y Daniel se reencuentran casualmente en Recursos Humanos.


43. Carros de fuego (Imaginar msica de Vangelis aqu
u
.)
Esta competencia hab que ganarla. Sus motivaciones no eran exactamente las
a
mismas, pero Eric y Harold sab que la victoria era el unico nal posible.
an

La posta de 4 por 400 era su especialidad, y ah estaban ambos, en la pista.

CAP
ITULO 2. CINEMATICA EN UNA DIMENSION

81

Eric llevaba el testimonio en la tercera etapa, con una leve ventaja sobre su
ms cercano competidor. Un poco ms all, en la meta, lo espera Harold, a
a
a
a
quien le corresponder cerrar la carrera y consolidar la ventaja. Eric lleva una
a
velocidad constante vE y, cuando se encuentra a una distancia LEH de Harold,
ste comienza a correr con velocidad constante vH . Queda poco. Son segundos
e
interminables. (Ms msica de Vangelis aqu Eric est a pocos cent
a
u
.)
a
metros de
Harold, sus manos casi se alcanzan. Eric hace un ultimo esfuerzo, y nalmente

entrega el testimonio a Harold. Harold mantiene su velocidad vH , mientras Eric

vuelve caminando hacia la meta, con velocidad vE .


a) Cunta distancia alcanz a recorrer Harold desde que parti hasta que
a
o
o
recibi el testimonio?
o
b) Si R es la longitud de una vuelta completa, a qu distancia de la meta se
e
encuentra Eric cuando Harold completa su vuelta a la pista?
vE

vH

(a)

vH

v
E

LEH
META

META

ENTREGA
TESTIMONIO

Superacin personal en la pista atltica.


o
e
R

META

La pista atltica.
e

(b)

CAP
ITULO 2. CINEMATICA EN UNA DIMENSION

82

44. Contacto en Francia (Imaginar msica de Don Ellis aqu


u
.)
(Continuacin del Problema 43.)
o
Para sorpresa de todos, Harold, al terminar su vuelta completa a la pista no
se detuvo a celebrar su victoria, sino que continu corriendo hasta la salida del
o
estadio, subiendo a su automvil con el testimonio an en la mano. Era un cono
u
vertible rojo, que de seguro no pod nanciar con sus exiguos ingresos como
a
aliado al programa de Deportistas de Alto Rendimiento del Gobierno. El perspicaz agente Jimmy Doyle se dio cuenta rpidamente de ello, y comprendi de
a
o
inmediato: Harold era pieza fundamental de una red global de narcotrco, y
a
el testimonio de la posta llevaba oculto en su interior informacin clave que la
o
polic necesitaba para desbaratarla desde su origen.
a
Doyle subi raudo a su auto policial, y comenz una frentica persecusin por
o
o
e
o
las calles de la ciudad, atestada de gente y automviles. Ahora Doyle y Harold
o
van por una calle muy larga y recta. El convertible de Harold lleva una velocidad constante vC , mientras el automvil policial de Doyle lleva una velocidad,
o
tambin constante, vD = vC . Harold aventaja a Doyle en una distancia LCD .
e
Sin embargo, una tragedia parece avecinarse. En el siguiente cruce de peatones,
una seora avanza empujando un coche con su beb. Justo cuando Doyle se
n
e
encuentra a una distancia LDS del paso peatonal, ve a la seora, y aplica los
n
frenos. El chirrido ensordecedor de los frenos se confunde con el grito despavorido de la seora, quien slo atina a mirar a su izquierda y ver el auto que se
n
o
aproxima a gran velocidad.
a) Cul debe ser la aceleracin del auto de Doyle para que alcance a detenerse
a
o
justo antes del paso peatonal?
b) Evidentemente, Doyle ha perdido la oportunidad de atrapar a Harold. Si
ste ha mantenido su velocidad, a qu distancia del paso peatonal se
e
e
encontrar Harold cuando Doyle nalmente se detiene?
a

LDS

vC
vD

LCD

Doyle ve
a la seora
Una persecusin vertiginosa.
o

CAP
ITULO 2. CINEMATICA EN UNA DIMENSION

83

45. Marlin, el pez payaso, hab perdido a su hijo. D y d de bsqueda incesante


a
as
as
u
ya lo hab agotado. Ningn pez pod resistir tanta tensin. Incluso Dory, que
an
u
a
o
todo el tiempo intentaba animar a su amigo con su encantador pero irracional
optimismo, tambin estaba agotada. Con un ocano tan grande, cmo tener la
e
e
o
menor esperanza de encontrar a Nemo alguna vez? Habiendo ya seguido la unica

pista que ten llegando a Sidney, Marlin y Dory se detuvieron a descansar en


an
el coral P (ver gura).
El cansancio no les permiti advertir que, a una distancia D, en el coral O,
o

lloraba un pequeo pez payaso con una aleta defectuosa. El tambin hab pan
e
a
sado por muchas aventuras, y ahora estaba solo en mar abierto, sin su padre.
Usando sus ultimas fuerzas, Nemo se sec las lgrimas de sus ojos con su ale
o
a
ta, y mir hacia adelante. . . Ser posible? Pap. . . eres t? Pap!. . . Pap!
o
a
a
u
a
a
Nemo, emocionado, sali nadando en el tiempo t = 0 con velocidad constante
o
vN , directamente hacia el coral P .
Los gritos despertaron a Marlin. Nemo? Es cierto? Nemo! Marlin no lo
pod creer. S era l! En t = T , Marlin comenz a nadar en l
a
,
e
o
nea recta hacia
el coral O, con velocidad vM , mientras la dormilona Dory segu descansando
a
en P .
En qu instante T0 se encuentran Nemo y su padre? A qu distancia de Dory
e
e
ocurre el encuentro?
Cuando padre e hijo se encuentran, Dory se despierta, pero con el sueo que
n
ten y la poca visibilidad del mar en esa zona, Dory no pod ver ms all que
a
a
a
a
una distancia D/3. En qu tiempo T debe comenzar a nadar Marlin para que
e
su encuentro con Nemo ocurra a una distancia D/3 de Dory, y as ella tambin

e
pueda disfrutar de tan feliz momento?

vM

vN

P
D
Encuentro en la bah de Sidney.
a

46. Rick, el capitn de la nave, ten rdenes precisas: destruir completamente la


a
a o
base alien
gena. En este planeta hostil, con todo tipo de alimaas salvajes acen
chando en los bosques que cubr casi toda la supercie del planeta, la unica
an

posibilidad de atacar era por aire. Rick lo sab y no hizo ningn intento por
a,
u

CAP
ITULO 2. CINEMATICA EN UNA DIMENSION

84

aterrizar su nave. En vez de ello, se mantuvo a una altura h de la torre de comunicaciones Delta, el punto ms dbilmente vigilado por los enemigos. No iba
a e
a ser un paseo de campo, pero ten al mejor para hacer el trabajo, el teniente
a
Rick. Curioso: ahora que lo pensaba, varios en la tripulacin se llamaban Rick.
o
Ser por eso que a veces costaba tanto comunicarse y hab fracasado dos
a
an
misiones anteriores?
Como sea, no hab tiempo de losofar. Rick, el teniente Rick, se dej caer
a
o
desde la nave en el tiempo t = 0, comenzando a caer verticalmente hacia abajo,
con aceleracin a. Transcurrido un tiempo t0 desde su salto, Rick alcanza una
o
cierta velocidad v, y abre su paraca
das, manteniendo su velocidad hasta llegar
al suelo, en las cercan de la base Delta.
as
Si Rick, el teniente, demora un tiempo T en llegar al suelo, determine su aceleracin a y su velocidad mxima.
o
a
Mientras, en la nave, Rick el capitn registraba la operacin con minucioa
o
sidad, haciendo grcos de posicin versus tiempo, velocidad versus tiempo y
a
o
aceleracin versus tiempo, para la trayectoria del teniente Rick. Haga los gro
a
cos que el capitn Rick debi haber hecho.
a
o
47. En un antiguo monasterio, perdido en los Himalayas, un monje se dispone a
comenzar un nuevo d de estudio y meditacin. Pausadamente, abandona la
a
o
celda en la cual ha dormido y desayunado frugalmente, y se dirige a la sala de
lectura. Hoy escoge un pergamino del ala norte, y lo extiende sobre uno de los
tablones que hacen las veces de escritorio. Con la an dbil luz matinal, percibe
u e
que se trata de un problema de F
sica, su tpico favorito:
o
Desde el piso, se lanza hacia arriba una pelota con una velocidad
v. Determine el tiempo transcurrido entre los dos instantes en que el
mdulo de su velocidad es |v|/N (con N un nmero entero arbitrario).
o
u
Determine tambin la distancia respecto al piso a la que se encuentra
e
la pelota en dichos instantes.
Un bello desaf para comenzar una nueva jornada de crecimiento personal.
o
Resuelva Ud. tambin el problema que el monje debe solucionar.
e
48. En la Torre Central de Metro City, el drama est a punto de cobrar una v
a
ctima.
Titn tiene a Roxanne de rehn en lo ms alto de la Torre, de altura h, dispuesto
a
e
a
a arrojarla si la ciudad no cede a sus demandas. Abajo, en el suelo, Megamente
observa incrdulo. El inepto de Titn parece decidido. Antes de que pudiera
e
a
reaccionar, Titn, ciego de maldad, empuja a Roxanne con velocidad v0 hacia
a
abajo. En ese mismo instante, Megamente se eleva desde el reposo, acelerando
con aceleracin a.
o

CAP
ITULO 2. CINEMATICA EN UNA DIMENSION

85

Cunto tiempo tarda Megamente, desde que parte del suelo, en encontrarse
a
con Roxanne y salvarla as de una muerte segura?

Cuando Roxanne comienza a caer, el cobarde Titn huye de la escena, con vea
locidad constante vT hacia arriba. A qu distancia del suelo est Titn cuando
e
a
a
Megamente y Roxanne se encuentran?

Terror en Metro City

2.6.

Solucin a algunos de los problemas


o

Solucin al problema 19
o
Sea x la direccin a lo largo de la cual ocurre el movimiento y denotemos, reso
pectivamente, con s(t), v(t) y a(t) a la posicin , velocidad y aceleracin que tiene la
o
o
part
cula en el instante t. Las condiciones iniciales son s(0) = 0 y v(0) = 50 m/s.
Conociendo s(0), v(0) podemos encontrar a(0). En efecto a(0) = v(0).
Usando las expresiones
v(t + ) v(t) + a(t)
s(t + ) s(t) + v(t)

() .

CAP
ITULO 2. CINEMATICA EN UNA DIMENSION

86

y eligiendo cierto valor pequeo para , podemos encontrar s() y v().


n
Conociendo s() y v() podemos encontrar a(). En efecto a() = v().
Usando nuevamente las relaciones (*) (pero ahora con t = ), podemos encontrar
s(2) y v(2), y a partir del ultimo tambin a(2). Etc...

e
Todo el proceso anterior se puede automatizar. En la prxima pgina se presenta
o
a
un programa en QUICKBASIC (para un PC compatible) que resuelve numricamente
e
el problema y graca los resultados en la pantalla del computador.
Al resolver numricamente el problema, repita el clculo con distintos valores
e
a
de y observe cmo el resultado no depende de este parmetro cuando es lo
o
a
sucientemente chico. Tambin repita el clculo para distintos valores de y analice
e
a
cmo este parmetro afecta al resultado.
o
a

CAP
ITULO 2. CINEMATICA EN UNA DIMENSION
CLS
SCREEN 12
VIEW (160, 20)-(580,310)
TMIN = 0
TMAX = 500
YMIN = 0
YMAX = 6000
WINDOW (TMIN, YMIN)-(TMAX, YMAX)
LINE (TMIN, YMIN)-(TMAX, YMAX), , B
FOR I = 0 TO 6
YP = I * 1000
PSET (TMIN, YP)
DRAW "R8"
PSET (TMAX - 10, YP)
DRAW "R8"
NEXT I
LOCATE 2, 17
PRINT "60"
LOCATE 2, 74
PRINT YMAX
LOCATE 2, 18
PRINT "0"
LOCATE 20, 74
PRINT YMIN
LOCATE 11, 17
PRINT "30"
LOCATE 11, 76
PRINT "X"
LOCATE 2, 13
PRINT "V"
FOR I = 0 TO 10
XP = TMIN + I * (TMAX - TMIN) / 10
PSET (XP, YMIN)
DRAW 5"
U
NEXT I
LOCATE 21, 20
PRINT TMIN
LOCATE 21, 71
PRINT TMAX
LOCATE 23, 44
PRINT "TIEMPO"
DT = 1
T = 0
X = 0
V = 40
ETA = 0.01
TF = 500
LOCATE 1, 36
PRINT "DT="; DT; .ETA="; ETA;
10

T = T + DT
IF T >TF THEN STOP
A = -ETA * V
X = X + V * DT
V = V + A * DT
PSET (T, X), 12
PSET (T, V * 100), 14
GOTO 10

LIMPIA PANTALLA
ELIGE SUPERVGA COLOR
DEFINE AREA DE TRABAJO
MINIMO DE ABSISA
MAXIMI DE ABSISA
MINIMO DE ORDENADA
MAXIMO DE ORDENADA
fIJA VALORES ANTERIORES
GRAFICA EJES (CAJA)
EVALUA POSICION DE TIC
POSICIONA EL LAPIZ EN ORDENADA (IZQ)
GRAFICA TIC
POSICIONA EL LAPIZ EN ORDENADA (DER)
GRAFICA TIC
POSICIONA LAPIZ
IMPRIME 60 EN ORDENADA IZQUIERDA
POSICIONA LAPIZ
IMPRIME EN ORDENADA DERECHA
POSICIONA LAPIZ
IMPRIME
POSICIONA LAPIZ
IMPRIME
POSICIONA LAPIZ
IMPRIME
POSICIONA LAPIZ
IMPRIME LEYENDA DE ORDENADA DERECHA
POSICIONA LAPIZ
IMPRIME LEYENDA DE ORDENADA IZQUIERDA
EVALUA POSICION DE TICS DE ABSISA
POSICIONA LAPIZ
GRAFICA TIC
POSICIONA LAPIZ
IMPRIME
POSICIONA LAPIZ
IMPRIME
POSICIONA LAPIZ
IMPRIME LEYENDA DE ABSISA
SE ELIGE DT
TIEMPO INICIAL
POSICION INICIAL
VELOCIDAD INICIAL
SE FIJA PARAMETRO DE FRICCION
TIEMPO FINAL
POSICIONA LAPIZ
IMPRIME TITULO
EL CALCULO EMPIEZA AQUI !!
SE INCREMENTA EL TIEMPO
SI T>TF EL CALCULO TERMINA
EVALUACION DE LA ACELERACION
NUEVA POSICION
NUEVA VELOCIDAD
GRAFICA PUNTO (T,X)
GRAFICA PUNTO (T,V)

87

CAP
ITULO 2. CINEMATICA EN UNA DIMENSION

88

Solucin al problema 27
o
Los tiempos t1 , que el salvavidas tarda para correr de A a P y t2 , que tarda para
nadar de P a B vienen dados por
t1 =

2
x2 + za
.
v1

y
t2 =

2
(L x)2 + zb

v1

Por lo tanto, el tiempo total que tarda en


ir de A a B es
T =

2
x2 + za
+
v1

2
(L x)2 + zb

v1

Figura 2.20

En la expresin anterior L, za y zb son jos; el valor de x se debe determinar de


o
manera que T sea m
nimo.
Encontrar el m
nimo de T en funcin de x es equivalente a encontrar los ceros de
o
la funcin derivada dT /dx:
o
T (x + ) T (x)
dT (x)
= l
m
=
0
dx

v1

x
2
x2 + za

(L x)

2
(L x)2 + zb

v2

La derivada tiene ceros si


x
v1
Pero

2
x2 + za

(L x)

2
(L x)2 + zb

v2
x

x2

2
+ za

= sin

y
(L x)

2
(L x)2 + zb

= sin ,

luego, T (x) tiene un extremo en funcin de x cuando


o
sin
sin
=
.
v1
v2
No es dif convencerse que tal extremo corresponde a un m
cil
nimo (y no a un mxia
mo).

CAP
ITULO 2. CINEMATICA EN UNA DIMENSION

89

Solucin al problema 29
o
a) Impl
citamente supondremos que las distancias estarn expresadas en metros, el
a
tiempo en segundos, las velocidades en m/s y las aceleraciones en m/s2 . De acuerdo
al enunciado se tiene:
Punto de partida: x(0) = 4, v(0) = 2
Entre t = 0 y 4, v(t) = 2, lo que corresponde a una l
nea horizontal en el grco v en
a
funcin de t (ver gura 2.21).
o
Entre t = 0 y 4 se tiene una recta con pendiente 2, en el grco x(t) en funcin de t
a
o
(ver gura 2.22). La posicin en t = 4 es x(4) = x(0) + v0 4 = 4 + 2 4 = 4.
o
A partir de t = 4, en el grco v en funcin de t, la velocidad estar representada
a
o
a
por una recta hasta llegar a v0 /2 = 1. Durante el proceso de frenado que tarda hasta

cierto instante t, la part


cula avanza 3 metros, o sea, el rea bajo la curva v(t) entre
a
debe ser 3. No es dif darse cuenta de que t debe ser 6.

t=4yt
cil
La aceleracin entre t = 4 y t = 6 es a1 = 0.5 (es la pendiente en el grco 2.21).
o
a
De acuerdo al enunciado, la part
cula avanza 3 metros durante el frenado, o sea,

x(6) = x(4) + 3 = 7. El grco de x(t), entre t = 4 y t = t = 6 ser parablico


a
a
o
con curvatura negativa. Otra forma de encontrar la posicin en t = 6 es usando la
o
expresin x(6) = x(4)+v(4)(64)+0.5 a1 (64)2 , o sea, x(6) = 4+220.50.522 =
o
7.
De t = 6 hasta t = 8 (durante 2 segundos) la velocidad se mantiene constante. El
grco de v(t) es una recta horizontal con velocidad 1.
a
El rea bajo el grco v(t) entre t = 6 y 8 nos da la distancia que A avanza en
a
a
ese intervalo. Tal rea es 2, luego x(8) = 7 + 2 = 9. Durante este intervalo x(t) es
a
representado por una recta (velocidad constante).
Se tiene que v(8) = 1. La part
cula desacelera con aceleracin a0 = 2 hasta que la
o
velocidad sea 3. Se observa inmediatamente que para ello debe desacelerar durante
2 segundos. Entonces v(10) = v(8) + a0 (10 8) = 1 2 (10 8) = 1 4 = 3.
Entre t = 8 y 10 el grco de v(t) es una recta (aceleracin constante).
a
o
Podemos encontrar la posicin de la part
o
cula en t = 10: x(10) = x(8) + v(8) (10
8) + 0.5 a1 (10 8)2 , o sea, x(10) = 9 + 1 2 + 0.5 (2) 22 = 7.
En t = 10 la part
cula se encuentra en x(10) = 7 y su velocidad es v(10) = 3. La
part
cula sigue a velocidad constante hasta llegar a dos metros del punto de partida
(o sea, hasta llegar a 2 metros). La part
cula, por lo tanto, deber recorrer 9 metros.
a
Con v1 = 3 [m/s] tardar para ello 3 segundos. O sea, entre t = 10 y t = 13 la
a
velocidad ser constante (linea horizontal) en el grco v en funcin de t.
a
a
o
A partir de t = 13 la part
cula frena uniformemente hasta quedar en reposo en el

CAP
ITULO 2. CINEMATICA EN UNA DIMENSION

90

punto de partida. El grco de v(t) es por lo tanto una recta hasta cero. El rea bajo
a
a
la curva entre t = 13 y el instante en que queda en reposo debe ser 3 (la part
cula
A debe recorrer an dos metros hacia la izquierda para llegar al punto de partida).
u
Es claro que para ello tardar 4/3 segundos.
a
cula recorre 2 metros. El grco de x(t) es una
a
Entre t = 13 y t = 14, 3, la part
parbola curvada hacia arriba que llega a t = 14, 3 con pendiente nula.
a

Figura 2.21

CAP
ITULO 2. CINEMATICA EN UNA DIMENSION

91

Figura 2.22
b) En t = 6 y t = 13 la part
cula A se encuentra en x(6) = 7 y x(13) = 2,
respectivamente. La velocidad media entre esos dos instantes es
v=

(2) 7
= 9/7 m/s .
13 6

c) En t = 8 la velocidad es 1 m/s. A partir de ese instante la part


cula acelera con
aceleracin a0 = 2, o sea, tarda 0.5 s para quedar temporalmente en reposo. En ese
o
instante (8,5 s) ocurre el alejamiento mximo. Se tiene
a
1
x(8, 5) = x(8) + v(8) (8, 5 8) + a0 (8, 5 8)2
2
1
= 9 + 1 0, 5 2 0, 52 = 9, 25 [m] .
2
d) Gracando xB (t) en la gura 2.21 se encuentra que los dos mviles se vuelven a
o
encontrar en el instante t = 11 s.
Solucin al problema 30
o
Cada manzana debe tardar t0 = 30, 5 = 1, 5 segundos en subir y bajar. Al lanzar
un objeto con velocidad v0 hacia arriba tarda un tiempo v0 /g hasta llegar arriba y
un tiempo igual hasta volver al punto de partida. Tenemos
t0 =

2v0
= 1, 5 [s] .
g

CAP
ITULO 2. CINEMATICA EN UNA DIMENSION

92

Esta ecuacin nos permite evaluar la velocidad con que se debe lanzar la manzana,
o
v0 = t0 g/2.
2
La altura a la que llega es un objeto lanzado con velocidad v0 es h = v0 /(2g).
Combinando las dos ultimas ecuaciones se encuentra para h la expresin

o
h=

1 2
gt .
8 0

Con g 10 [m/s2 ] se encuentra h 3 metros.


Solucin al problema 32
o
a) Cuando B env el mensaje se encuentra a 22 m de A. El mensaje tarda 1/2
a
s en llegar a su destino. Durante ese intervalo el mvil A seguir movindose
o
a
e
desplazndose 10 0, 5 = 5 metros. El mensaje deber recorrer en 0,5 s una
a
a
distancia de (22+5)=27 metros. La velocidad del mensaje ser c = 27/0, 5 =
a
54 [m/s].
b) Las ecuaciones de movimiento de los mviles, para 0 < t y el instante en que B
o
env el mensaje (llammoslo t1 ), son
a
e
xA (t) = xA (0) + vA (0)t = 100 + 10 t
1 2 1 2
a1 t = t
2
2
vA (t) = vA (0) = 100

xB (t) =

vB (t) = a1 t = t .
(En las expresiones anteriores estamos suponiendo que los tiempos estn dados
a
en segundos, las distancias en metros, las velocidades en [m/s] y las aceleraciones
en [m/s2 ].)
Sabemos que en t = t1 la separacin entre A y B es de 22 metros, o sea,
o
xA (t1 ) xB (t1 ) = 100 + 10 t1

1 2
t = 22 .
2 1

Resolviendo esta ecuacin cuadrtica para t1 se encuentra que t1 = 10 16. En


o
a
el contexto del problema slo la solucin positiva tiene sentido, o sea, t1 = 26 [s].
o
o
La velocidad de B en el instante t1 es vB (t1 ) = 26 [m/s].
c) Desde que B env el mensaje hasta chocar con A, el mvil B debe recorrer
a
o
una distancia de 22+5=27 metros. En el instante t1 se encuentra a xB (t1 ) =
(26)2 /2 = 338 m del origen. La distancia total que B debe recorrer desde que
parte del origen hasta que choca con A es (338+27)=365 m.

CAP
ITULO 2. CINEMATICA EN UNA DIMENSION

93

d) Desde que B env el mensaje hasta chocar con A, el mvil B debe recorrer una
a
o
distancia de 22+5=27 metros. Como su velocidad (a partir de t1 ) es de 26 m/s,
tardar 27/26 segundos. El tiempo total, desde que B parte del origen hasta
a
que choca con A es (26+27/26) s. Para la velocidad media de B se encuentra
v=

365
13, 5 [m/s] .
26 + 27
26

Cap
tulo 3

Cinemtica en dos y tres


a
dimensiones
versin 28 mayo 2012
o

En este cap
tulo extenderemos la descripcin del movimiento de una part
o
cula
a dos y tres dimensiones. Esto nos lleva a introducir el concepto de vector, cuya
denicin y propiedades ilustraremos con los vectores desplazamiento, velocidad y
o
aceleracin.
o

3.1.

Vectores

Para movimientos en una dimensin, basta un nmero para describir completao


u
mente la posicin de una part
o
cula, a saber, la distancia entre dicho punto y el origen.
Como el eje es orientado, el signo de dicho nmero indica si la part
u
cula est a la
a
derecha o a la izquierda del origen, y sa es toda la informacin que necesitamos.
e
o
Cuando el movimiento es en ms dimensiones, no basta un nmero. Si deseamos
a
u
indicar completamente la posicin de, digamos, Valpara respecto a Santiago, neo
so
cesitamos indicar no slo la distancia que los separa, sino alguna indicacin de en
o
o
qu direccin se encuentra desde Santiago. Si imaginamos entonces una echa cue
o
yo origen est en Santiago, y su punta est en Valpara esa echa tiene toda la
a
a
so,
informacin necesaria para llegar de una ciudad a la otra.
o
Esta echa corresponde a un vector . Un vector ser una entidad que no solamena
te tiene magnitud, sino tambin direccin. Decimos entonces que la posicin de una
e
o
o
part
cula es un vector. En este cap
tulo y los siguientes encontraremos diversas cantidades f
sicas para cuya descripcin completa no basta slo un nmero, sino adems
o
o
u
a
una direccin (la velocidad, por ejemplo; su direccin es importante: puede hacer la
o
o
diferencia entre una colisin entre dos veh
o
culos o no). Otras cantidades, en cambio,
pueden ser descritas con slo un nmero: la masa de una part
o
u
cula, por ejemplo; o la
cantidad de libros en nuestra biblioteca. A dichas cantidades las llamamos escalares.
94


CAP
ITULO 3. CINEMATICA EN DOS Y TRES DIMENSIONES

95

Un vector se denota con una echa: A. Se denomina mdulo del vector A a la


o
longitud de la echa que lo representa, y se le denomina |A |, o simplemente A.
Si bien es cierto la imagen geomtrica de un vector como una echa es suciente,
e
necesitamos, para analizar el movimiento, ser ms precisos. Introduciremos, entonces,
a
igual como lo hicimos en el Cap
tulo anterior, un origen y ejes coordenados.
Consideremos el movimiento de una part
cula en
un plano. La posicin de la part
o
cula podr ser
a
claramente especicada si se introduce un sistema de ejes perpendiculares que se intersectan
en un punto, que llamaremos el origen (ver
gura 3.1).
Por ejemplo, el punto P en la gura 3.1 se encuentra a 3 m a la derecha del origen, medidos
a lo largo de la direccin del eje x, a 2 m soo
bre el origen, medidos a lo largo del eje y. En
general, la posicin de un punto cualquiera queo
da determinada dando un par ordenado (x, y)
de nmeros, en el sentido que siempre el primer
u
nmero corresponder a la proyeccin sobre el
u
a
o
eje x y el segundo nmero a aqulla sobre el eje

u
e
y.

y
4
yQ

Q
r

yP

1
_1

rP
xQ

xP

_1

Figura 3.1

El trazo que une el origen O con el punto P , en el sentido que indica la punta de
echa en la gura 3.1, se denomina el vector de posicin rp del punto P . La magnitud
o
de este vector es igual a la longitud del trazo OP y se denota por |rp | o simplemente
como rp (sin echa).
La descripcin de un vector como un par ordenado es equivalente a su descripcin
o
o
como una echa, pero sirven propsitos distintos. Es la misma diferencia entre hacer
o
Geometr Euclidiana (en que una recta, por ejemplo, es un objeto geomtrico tal
a
e
que pasa por dos puntos dados; para construir una recta no necesitamos otra cosa
que lpiz y papel) y Geometr Anal
a
a
tica (en que una recta es el conjunto de puntos
que satisface una ecuacin del tipo y = mx + n, de modo que adems de lpiz y papel
o
a
a
necesitamos un par de ejes coordenados respecto a los cuales medir x e y.)


CAP
ITULO 3. CINEMATICA EN DOS Y TRES DIMENSIONES
Es importante notar que dos vectores son
iguales si tienen la misma magnitud y
que apuntan en la misma direccin. En la
o
Fig. 3.2 se muestra un conjunto de vectores iguales, dibujados en diferentes posiciones del plano x-y.

96

Figura 3.2
Supongamos ahora que la part
cula en un instante t se encuentra en P y en un
instante posterior t > t se encuentra en el punto Q (ver gura 3.1). El vector que
une el origen O con Q es el nuevo vector de posicin de la part
o
cula. Llammoslo
e
rQ . Por analog al caso unidimensional, deber
a
amos denir el vector desplazamiento
a la diferencia entre los dos vectores posicin, rQ rP . Pero entonces vemos que,
o
para describir el movimiento, necesitamos denir la resta (es decir, en general, la
suma) entre vectores. Por ahora podemos decir que, en el caso de la Fig. 3.1, el vector
desplazamiento corresponde al vector conformado por el trazo P Q y cuyo sentido va
desde P hacia Q, y corresponde precisamente a r rQ rP . Pero para formalizar
este hecho necesitamos denir antes la suma de vectores y estudiar sus propiedades.
Suma de vectores
Sean A y B dos vectores. Traslademos paralelamente a s mismo al vector B hasta

que su extremo romo se superponga con


el extremo aguzado (punta de echa) del
vector A. El vector suma A + B C se
dene como el trazo que comienza en el extremo romo de A y termina en el extremo
aguzado de B. Esta denicin se conoce
o
con el nombre de regla del paralelgramo.
o

A
C

B
A
A + B =C

Figura 3.3

Equivalentemente, podemos decir que para sumar dos vectores basta colocar un
vector a continuacin del otro. Al unir el origen del primer vector con la echa del
o
segundo vector, se obtiene el vector suma (en el caso de la Fig. 3.3, corresponde a
considerar slo el semi-paralelgramo inferior; claramente el resultado es el mismo).
o
o


CAP
ITULO 3. CINEMATICA EN DOS Y TRES DIMENSIONES
Ejemplo:
Un excursionista parte desde una cierta
posicin y camina 4 km hacia el Este y
o
luego 3 km hacia el Sur. Cul es el vector
a
desplazamiento resultante C?
El vector C es la suma vectorial de los
desplazamientos parciales realizados por
el excursionista, hacia el este A y luego
hacia el sur B. Grcamente la situacin
a
o
est ilustrada en la gura 3.4. La magnia
tud del desplazamiento resultante se calcula utilizando el teorema de Pitgoras
a
C=

A2 + B 2 =

97

Norte
km
5

A
4

2
1
0
0

Este
5 km

Figura 3.4

9 + 16 = 5 km .

La direccin de C queda denida por el ngulo que forma el vector C con la direccin
o
a
o
OesteEste. Consideraremos un ngulo positivo cuando se mide en sentido contrario
a
a los punteros del reloj, luego
3
tan = = 0.75 , es decir,
4

= 36.9 .

Que el ngulo sea negativo signica que est medido en el mismo sentido de los
a
a
punteros del reloj.
La suma de vectores tiene las siguientes propiedades:
i) Conmutatividad:
ii) Asociatividad:

A+B = B+A .
A + (B + C) = (A + B) + C .

iii) Existe un vector nulo tal que

A+0 = A .

iv) Para cada vector A existe un vector opuesto, que denotaremos por A, tal que
A + (A) = 0 .
De hecho, son estas propiedades las que permiten llamar suma a la operacin
o
denida en la Fig. 3.3. En general, a cualquier operacin que satisfaga las cuatro
o
propiedades anteriores se le puede llamar suma, y asignarle el s
mbolo +.
Y cmo se restan dos vectores? Por una parte, podemos calcularlo sumando un
o
vector con el opuesto aditivo del otro. Alternativamente, podemos verlo como sugiere
la gura siguiente:


CAP
ITULO 3. CINEMATICA EN DOS Y TRES DIMENSIONES

98

C
A
B
Si C = A B, entonces C debe ser tal que C + B = A, esto es, al poner el vector C
a continuacin del vector B, deber obtenerse el vector A. Esto es lo que muestra la
o
a
gura anterior, y por tanto el vector A B es el vector que une las puntas de los dos
vectores involucrados, y apunta en direccin de B hacia A.
o
Tambin es posible denir la multiplicacin de un vector por un escalar, como
e
o
veremos a continuacin.
o
Multiplicacin de un vector por un escalar real.
o
La multiplicacin de un vector A por un nmero real (escalar real) se dene como
o
u
un nuevo vector B de magnitud |A|, cuyo sentido coincide con el de A si > 0 y es
opuesto al de ste si < 0.
e
Propiedades de la multiplicacin por un escalar real.
o
Sean y dos nmeros reales y A y B dos vectores, entonces:
u
i)

(A + B) = A + B.

ii)

( + )A = A + A.

iii)

()A = ( A).

iv)

Para todo vector A se cumple que

1 A = A.

Ejercicio: Compruebe grcamente, con algunos ejemplos concretos, que se cumplen


a
todas las propiedades de los vectores recin sealadas.
e
n
Las dos operaciones que hemos denido son fundamentales para un vector. De
hecho, desde el punto de vista matemtico, un vector es un objeto para el cual se
a
puede denir una suma y un producto por escalar (es decir, dos operaciones, una
entre vectores y otra entre un vector y un escalar, con las propiedades indicadas
ms arriba). Cualquier objeto que tenga tales operaciones con dichas propiedades,
a
podr ser llamado vector. La imagen de un vector como una echa es, entonces, slo
a
o


CAP
ITULO 3. CINEMATICA EN DOS Y TRES DIMENSIONES

99

pictrica, nos permite tener una intuicin, pero matemticamente podemos ser ms
o
o
a
a
abstractos an.
u
Componentes cartesianas y polares de un vector.
Para describir un
vocamente un vector en dos dimensiones necesitamos las dos
componentes del par ordenado que lo representa, por ejemplo. A dichos nmeros se
u
les denomina las componentes cartesianas del vector. Dos vectores A = (Ax , Ay ) y
B = (Bx , By ) son iguales si cada una de sus componentes son iguales: Ax = Bx , y
Ay = By .
Sin embargo, la anterior no es la unica posibilidad de descripcin. Como se ve

o
en la Fig. 3.4, tambin es posible caracterizar al vector dando otros dos nmeros, a
e
u
saber, su mdulo y su direccin, en la forma de un ngulo respecto al eje x. A stos
o
o
a
e
se les denomina las coordenadas polares del vector.
El vector es, desde luego, uno solo. Lo unico que cambia son los nmeros (las

u
coordenadas) con las cuales se decide describirlo. Debe existir, entonces, una relacin
o
entre ambos sistemas de coordenadas.
Sea A = (Ax , Ay ) un vector cualquiera del
plano x-y. Expresemos el vector, alternativamente, en funcin de su magnitud A
o
y del ngulo que forma con el semieje x
a
positivo. La gura 3.5 muestra que
Ax = A cos

Ay
A

Ay = A sin ,

Ax

y que
A = |A| =

A2 + A2
x
y

tan =

Ay
.
Ax

Figura 3.5

Los nmeros (Ax , Ay ) se denominan las coordenadas cartesianas del vector; los nmeu
u
ros (A, ) reciben el nombre de coordenadas polares del vector A. Cualquiera sea el
caso, un vector en un plano queda determinado si se conocen dos nmeros.
u
Notemos que al calcular las coordenadas polares en trminos de las cartesianas, es
e
necesario invertir una funcin trigonomtrica. Como el cuociente Ay /Ax no distingue
o
e
entre vectores en los cuadrantes I y III, por un lado, ni en los cuadrantes II y IV, por
otro, es importante tener presente esto y no usar la calculadora ciegamente.
Ahora que manejamos el concepto de coordenadas, es conveniente notar la forma
que tienen las dos operaciones que hemos denido sobre vectores, pero ahora en
coordenadas cartesianas. Esto, en la prctica, y dependiendo del problema en cuestin,
a
o
puede ser ms conveniente que manejar slo las deniciones geomtricas dadas hasta
a
o
e
ahora.


CAP
ITULO 3. CINEMATICA EN DOS Y TRES DIMENSIONES

100

En la siguiente gura, representamos la suma de dos vectores:


^
y
By

B
A

Ay

Ax

^
x

Bx

Es claro de la gura que, de la denicin de suma de vectores, sta es, en cooro


e
denadas cartesianas,
C = A + B = (Ax + Bx , Ay + By ) .
Anlogamente, para la multiplicacin por un escalar:
a
o
^
y
Ay

Ay
A
Ax

Ax

^
x

se tiene que
A = (Ax , Ay ) .
Los anlisis anteriores han sido realizados para vectores en dos dimensiones, pero
a
naturalmente pueden ser generalizados a tres. En este caso, el mdulo de un vector
o
en coordenadas cartesianas es
A=

A2 + A2 + A2 ,
x
y
z

la suma entre vectores es


A + B = (Ax + Bx , Ay + By , Az + Bz ) ,
y el producto por un escalar es
A = (Ax , Ay , Az ) .


CAP
ITULO 3. CINEMATICA EN DOS Y TRES DIMENSIONES

101

Vectores unitarios.
Habiendo denido la multiplicacin entre escalares y vectores, ser interesante
o
a
tener ciertos vectores elementales, tal que un vector arbitrario se pudiera escribir
como un mltiplo de esos vectores elementales. Para ello denimos los vectores unitau
rios, que son los vectores de mdulo uno. Para distinguirlos del resto de los vectores,
o

los denotamos con un tongo: A. Para caracterizar una direccin, entonces, basta
o
con indicar el vector unitario que tiene dicha direccin (existe uno solo; todos los
o
otros vectores con la misma direccin no son unitarios).
o
Dado cualquier vector A es posible obtener el vector unitario con su misma direccin: basta dividirlo por su mdulo. En efecto, el vector
o
o
A=
A=
A

Ax Ay Az
,
,
A A A

es unitario:

|A| =

A2 + A2 + A2
x
y
z
=1 .
2
A

(Ax /A)2 + (Ay /A)2 + (Az /A)2 =

Es posible, entonces, encontrar innitos vectores unitarios. Existen, sin embargo,

tres vectores unitarios que merecen mencin especial. Estos son los vectores unitarios
o
x, y y z que apuntan en sentido positivo sobre cada uno de los ejes coordenados de un

sistema cartesiano en tres dimensiones. (En algunos libros estos vectores se designan
con , y k, respectivamente.)

Observemos la gura 3.6. Recordando que
^
z
la suma de vectores equivale a poner un
Az
vector a continuacin del otro, es claro que
o
el vector A se puede escribir como la suma
A
de tres vectores: Ax x (un vector en la di
reccin x, de mdulo Ax ), Ay y (direccin
o
o

o
Ay
y , mdulo Ay ), y Az z (direccin z , mdulo
o

o o
O
Az ). Es decir,
^
y
Ax

A = Ax x + Ay y + Az z .

^
x

Figura 3.6
Sabemos sumar vectores y multiplicarlos por escalares. Es posible denir otras
operaciones, anlogas a las conocidas entre nmeros reales? Por ejemplo, un proa
u
ducto entre vectores? La respuesta es s pero no hay una unica manera de hacerlo.
,

Revisemos a continuacin una de ellas.


o


CAP
ITULO 3. CINEMATICA EN DOS Y TRES DIMENSIONES

102

Producto escalar o producto punto de dos vectores


Sean A y B dos vectores arbitrarios. Se
dene el producto punto entre los vectores
A y B mediante la expresin
o

A B |A| |B| cos ,


donde es el ngulo entre los dos vectores
a
(ver gura 3.7).1

A
Figura 3.7

De la denicin se desprende que el producto punto de dos vectores es un nmero


o
u
real. Podemos reinterpretar geomtricamente el producto punto como el producto
e
entre el mdulo de un vector (A) y el mdulo de la proyeccin del otro vector sobre
o
o
o
ste (B cos ). Y viceversa.
e
Es claro que si los vectores A y B son rotados simultneamente, en el mismo
a
ngulo, su producto punto no cambia.
a
Se tiene tambin que si dos vectores son paralelos, el producto punto es simplee
mente el producto de sus mdulos. Por su parte, si dos vectores son perpendiculares
o
el producto punto entre ellos es cero.
En particular, para los vectores unitarios x, y , z :

xx=yy =zz =1

y
xy =xz =yz =0.

Otras caracter
sticas importantes del producto punto son su conmutatividad
AB =BA
y distributividad
A (B + C) = A B + A C .
Ejercicio: Demuestre las dos propiedades anteriores a partir de las deniciones
geomtricas de suma de vectores y producto punto.
e
Es claro que
A= AA .
1

A diferencia del producto entre nmeros reales, el punto que representa el producto escalar entre
u
vectores no se puede omitir.


CAP
ITULO 3. CINEMATICA EN DOS Y TRES DIMENSIONES

103

Notemos que, a diferencia del producto entre reales, no existe un elemento neutro.
Si existiera (llammoslo N ), ser un vector tal que A N sea igual a A, lo que
e
a
es imposible, ya que el primero es un escalar, y el segundo un vector. Por tanto,
tampoco existe un elemento inverso bajo el producto punto, y no es posible denir
una divisin entre vectores.
o
Usando las propiedades conocidas, evaluemos el producto punto entre los dos vectores
A y B en trminos de sus coordenadas cartesianas. Se tiene
e
A B = (Ax x + Ay y + Az z ) (Bx x + By y + Bz z )

= Ax Bx x x + Ax By x y + Ax Bz x z + Ay Bx y x + Ay By y y +





+Ay Bz y z + Az Bx z x + Az By z y + Az Bz z z ,



es decir,
A B = Ax Bx + Ay By + Az Bz .
Note que la ultima expresin permite evaluar el ngulo entre dos vectores si se conocen

o
a
sus componentes cartesianas.

Ejemplo
Evaluemos nuevamente el ngulo entre dos
a
diagonales de un cubo.
Sea A el vector a lo largo de la diagonal que une el punto (0,0,0) con el punto
(1,1,1) y B el vector a lo largo de la diagonal que une el punto (1,0,0) con el punto
(0,1,1). Los vectores A y B, por lo tanto,
pueden escribirse en coordenadas cartesianas de la forma
A=x+y+z

z
(0,1,1)

(1,1,1)

(0,0,1)

(1,1,0)
(0,0,0)

B = + y + z .
x

(1,0,0)

Figura 3.8

Evaluemos el producto punto de estos dos vectores. Se tiene



A B = |A| |B| cos = 3 3 cos ,
donde es el ngulo entre los dos vectores (o sea, el ngulo entre las dos diagonales).
a
a
Por otra parte, usando coordenadas cartesianas
A B = 1 (1) + 1 1 + 1 1 = 1 .
De las dos ecuaciones anteriores se deduce que cos = 1/3, o sea, = 70.53 .


CAP
ITULO 3. CINEMATICA EN DOS Y TRES DIMENSIONES

3.2.

104

Cinemtica
a

Habiendo ya denido los aspectos fundamentales de vectores, es directo generalizar los conceptos de la cinemtica de una a dos y tres dimensiones.
a
Supongamos que r (t) representa la posicin de cierta part
o
cula. Entonces su
velocidad y aceleracin (instantnea) vendrn dadas por
o
a
a
r(t + ) r(t)

v(t) = r (t) = l
m
0

y
v(t + ) v(t)

a(t) = v(t) = r (t) = l


m
.
0

De la expresin anterior se deduce que si


o
r (t) = x(t) + y(t) + z(t) ,
x
y
z
donde x(t), y(t) y z(t) son las componentes del vector de posicin, entonces
o
v(t) = l
m

y(t + ) y(t)
z(t + ) z(t)
x(t + ) x(t)
x+

y+

z ,

es decir
v(t) = x(t) + y(t) + z(t) ,
x y z
o sea, para encontrar la velocidad se puede derivar cada componente del vector posicin por separado. Anlogamente,
o
a
a(t) = x(t) + y (t) + z (t) .
x y z
Introduzcamos tambin el concepto de velocidad relativa. Supongamos que una
e
part
cula A se mueve con velocidad vA y otra part
cula B con velocidad vB , entonces
la velocidad con que A observa que se mueve B, viene dada por
v = vB vA .
Se dice que v es la velocidad relativa de B respecto a A.
Ejemplo:
Suponga que la corriente de un canal tiene una velocidad de 10 km/h en direccin
o
Este. Un transbordador navega en la direccin de 30 Noroeste, a una velocidad de 20
o
km/hora con respecto a la corriente del canal (ver gura 3.9). Cul es la velocidad
a
y direccin del transbordador segn un observador situado en la ribera?
o
u


CAP
ITULO 3. CINEMATICA EN DOS Y TRES DIMENSIONES

105

1111111111111111111111111111111111111
0000000000000000000000000000000000000
1111111111111111111111111111111111111
0000000000000000000000000000000000000
1111111111111111111111111111111111111
0000000000000000000000000000000000000
1111111111111111111111111111111111111
0000000000000000000000000000000000000
y
vt

30 o

corriente

1111111111111111111111111111111111111
0000000000000000000000000000000000000
1111111111111111111111111111111111111
0000000000000000000000000000000000000
1111111111111111111111111111111111111
0000000000000000000000000000000000000
Figura 3.9
Para resolver el problema introduciremos un sistema de coordenadas x, y cuyo

origen O se mueve junto al agua del canal. Para el observador O , un punto jo en
la orilla se mueve con velocidad
vA = [10, 0] km/h ,
mientras que el transbordador se aleja con una velocidad
vt = [20 sin(30 ), 20 cos(30 )] km/h = [10, 10

3] km/h .

Luego, la velocidad con que el observador parado en la orilla en el punto A ve alejarse


al transbordador (o sea, la velocidad relativa entre el transbordador y la orilla), ser
a

v = vt vA = [0, 10 3] km/h = 10 3 y km/h ,

un movimiento puramente transversal a la corriente del canal.


Analicemos ahora el problema de otra forma. Supongamos que nos damos un
intervalo de tiempo arbitrario, por ejemplo, 1 hora (porque es el ms fcil de usar en
a a
este caso) e imaginemos que durante ese intervalo la corriente del canal est detenida.
a
Calculamos el desplazamiento del transbordador en este caso. En una hora el ferry
se desplaza 20 km desde O hasta el punto P . En seguida y siempre en nuestra
imaginacin dejemos uir la corriente del canal durante una hora, pero ahora con
o
el ferry detenido (dejando que simplemente ote en la corriente). El desplazamiento
debido al arrastre del canal llevar al ferry desde el punto P hasta P (10 km hacia
a
la derecha), como mostramos en la gura 3.10.
El desplazamiento total del ferry es el vector de O hasta P . Este desplazamiento, como es fcil de demostrar, coincide con el que el ferry hubiese tenido en una
a
hora si los dos movimientos hubiesen estado presentes simultneamente. Es decir,
a
para resolver el problema podemos descomponer el movimiento en dos movimientos
separados, congelando uno y otro sucesivamente. El movimiento total es la superposicin de ambos movimientos. Esta operacin, slo posible en la imaginacin, arroja
o
o
o
o


CAP
ITULO 3. CINEMATICA EN DOS Y TRES DIMENSIONES

vc

106

10 km

vt
20 km

30 o
N
E

Figura 3.10
los mismos resultados que se observan en la vida real, y es extremadamente util para

describir movimientos complejos, al considerarlos como superposicin de dos o ms


o
a
movimientos ms sencillos.
a
Demos otro ejemplo del uso del principio de superposicin. Consideremos un anio
llo que rueda (sin resbalar) por una supercie horizontal con velocidad constante.
Tomemos un punto cualquiera sobre el anillo y analicemos su movimiento. Para un
observador O en reposo respecto a la supercie, el movimiento del punto tendr un
a
aspecto complicado. Sin embargo, al trasladarnos uniformemente con la misma velocidad que el centro del anillo, el movimiento del punto se tornar muy simple: es un
a
movimiento circular uniforme. As el movimiento complicado que observa O se pue,
de descomponer en dos movimientos simples, un movimiento de traslacin uniforme
o
superpuesto a un movimiento circular uniforme (ver problema 13).
Problema resuelto en clases: 3.19

Ca libre
da
Podemos utilizar el principio de superposicin para describir la ca de una
o
da
part
cula en el campo gravitatorio terrestre.
La gura 3.11, a la izquierda, muestra la posicin de una pelota en ca lio
da
bre durante varios instantes equiespaciados. A la derecha se muestra la situacin
o
que se observa si el cuerpo adems inicialmente tiene una velocidad horizontal. La
a
trayectoria en este caso es una parbola. Histricamente, los lsofos se esforzaron
a
o
o
mucho para intentar explicar este movimiento, sin xito. Fue Galileo Galilei quien
e
comprendi que la solucin era describir el movimiento de la manera ms sencilla y
o
o
a
directa. Fue l quien, por primera vez, estudi la ca de una part
e
o
da
cula como una
superposicin de dos movimientos: i) la tendencia natural de los cuerpos a mantener
o
su velocidad (ley de inercia) y ii) la ca libre de un cuerpo debida a la atraccin
da
o
gravitatoria. Cuando ambos movimientos se superponen simultneamente, se puea


CAP
ITULO 3. CINEMATICA EN DOS Y TRES DIMENSIONES

107

de mostrar, geomtricamente, que dan origen al movimiento parablico, tal como se


e
o
2
observa experimentalmente.

Figura 3.11
Analicemos ahora vectorialmente este problema. Para comenzar, especiquemos
el sistema de referencia. El eje x lo elejimos de manera que su direccin coincida

o
con la proyeccin de la velocidad sobre el plano horizontal, mientras que el eje z
o

lo elegimos hacia arriba (o sea, una part


cula al caer acelera en la direccin ).
o
z
De acuerdo a las observaciones del propio Galileo, la aceleracin en todo instante
o
es a(t) = g. Tambin supondremos que la velocidad en el instante t = 0 viene
z
e
(0)
(0)
dada por v(0) = vx x + vz z y que la part

cula se encuentra en el lugar r(0) =


r0 = x0 x + z0 z . Deseamos encontrar, con toda esta informacin, la posicin de la

o
o
= a(t), el problema corresponde,
part
cula, r(t), para todo instante t. Puesto que r(t)
matemticamente, integrar dos veces el vector a(t); la primera vez obtendremos la
a
velocidad, y la segunda vez la posicin. Pero ya sabemos que derivar un vector equivale
o
a derivar cada componente por separado; es claro que lo mismo debe ocurrir con
la operacin inversa: integrar un vector equivale a integrar cada componente por
o
separado [basta recordar que integrar a(t) es encontrar aquella funcin tal que su
o
derivada es a(t).] En la prctica, esto signica que podemos analizar cada una de las
a
componentes del vector a por separado.
2
Es importante notar la gran intuicin f
o sica que implica, de parte de Galileo, el haber sido capaz de
separar el movimiento de este modo, en una poca en que las condiciones para hacer experimentos de
e
precisin era muy limitada, y ni siquiera exist el concepto de la necesidad de realizar experimentos
o
a
de precisin.
o


CAP
ITULO 3. CINEMATICA EN DOS Y TRES DIMENSIONES

108

Componente x : La aceleracin no tiene componente en la direccin x, o sea,


o
o
ax = 0 .
La velocidad vx es, por lo tanto, constante, igual al valor inicial:
(0)
vx (t) = vx ,

para todo t .

Para el desplazamiento en la direccin x se encuentra que


o
(0)
x(t) = x0 + vx t .

Componente z : La aceleracin es
o
az = g .
La velocidad vz y el desplazamiento en la direccin z vendrn dados por
o
a
(0)
vz (t) = vz gt

1
(0)
z(t) = z0 + vz t gt2 .
2

Estos resultados los podemos condensar escribindolos en forma vectorial:


e
a(t) = g
z
v(t) = v (0) gt
z

r(t) = r0 + v (0) t gt2 z .


2
Con esto hemos resuelto completamente el problema. Es posible, a partir de estas
ecuaciones, mostrar que la trayectoria corresponde, efectivamente, a una parbola (ver
a
Problema 3.21).
Notemos, nalmente, cmo el hecho de usar vectores para describir el movimiento
o
nos permiti (por argumentos puramente matemticos), separar el movimiento en dos
o
a
componentes, que analizamos por separado. Pero se es precisamente el esp
e
ritu del
principio de superposicin que hab usado Galileo. Esto no es casualidad: el hecho de
o
a
que sea posible usar superposicin para describir movimientos complejos en trminos
o
e
de movimientos ms sencillos, y el hecho de que podamos describir el movimiento con
a
cantidades vectoriales, estn
a ntimamente relacionados.


CAP
ITULO 3. CINEMATICA EN DOS Y TRES DIMENSIONES

109

v0

1111111111111111111111111111111111111111111
0000000000000000000000000000000000000000000
1111111111111111111111111111111111111111111
0000000000000000000000000000000000000000000
O
L

Figura 3.12
Ejemplo
Un bombardero vuela con una velocidad horizontal v0 , constante, y a una altura h
en una trayectoria que pasa directamente por sobre su objetivo. A qu ngulo de
e a
visin debe soltar la bomba, de forma que sta llegue a su objetivo? (Ignore el efecto
o
e
debido al roce del aire.)
La bomba en el instante en que se deja libre tiene la misma velocidad que el bombardero. Denimos el sistema de coordenadas de acuerdo a lo que se observa en la
gura 3.12. Entonces la posicin y la velocidad inicial de la bomba vienen dadas por
o
r0 = h y v0 = v0 x, respectivamente. Cunto demora la bomba en caer? La bomba
z

a
llegar al suelo cuando z(t) = h gt2 /2 = 0. Esto ocurre en el instante = (2h/g).
a
Durante el intervalo de tiempo la bomba alcanza a recorrer una distancia horizontal
L = v0 . Luego para el ngulo de visin obtenemos
a
o
tan =

L
v0
=
h
h

2h
=
g

2
2v0
.
gh

Observemos que, mientras mayor es la velocidad del bombardero, mayor es el


ngulo al cual debe lanzarse la bomba. En particular, cuando v0 , = /2, lo
a
que signica que el lanzamiento debe producirse cuando el avin se encuentra innio
tamente lejos del objetivo. Otro l
mite interesante es h , en cuyo caso = 0,
lo cual dice que cuando el avin se encuentra a una altura demasiado grande, hay
o
que arrojar la bomba cuando el avin se encuentra directamente sobre el objetivo.
o
Lo cual suena contradictorio a primera vista, ya que si el avin se mueve, siempre
o
habr un desplazamiento horizontal de la bomba, y ste ser cada vez mayor precia
e
a
samente mientras ms alto se encuentre el avin. La solucin es que, precisamente
a
o
o
debido a que el movimiento es acelerado en la direccin vertical, para alturas demao
siado grandes la bomba tiene mucho tiempo para aumentar su velocidad, superando
cualquier velocidad horizontal que pueda haber tenido inicialmente. Esto hace que


CAP
ITULO 3. CINEMATICA EN DOS Y TRES DIMENSIONES

110

su desplazamiento horizontal termine siendo despreciable respecto al vertical, y eso


corresponde a despreciable, es decir cero.
Notemos tambin que si el campo gravitatorio disminuyera, ser necesario arrojar
e
a
la bomba desde ngulos cada vez mayores, lo cual tambin est de acuerdo con la
a
e
a
intuicin, ya que la bomba demorar ms en caer.
o
a a
Problema resuelto en clases: 3.34

Movimiento circular uniforme


Adems del movimiento parablico antes descrito, el movimiento circular es otro
a
o
movimiento sencillo que podemos describir en dos dimensiones.
Consideremos una part
cula que gira con rapidez constante sobre una trayectoria
circular de radio R (que dene el plano x-y). Eligiendo el origen al centro del c
rculo,
que el movimiento sea uniforme signica que el ngulo del vector posicin con el eje
a
o
x aumentar en la forma:

a
(t) = 0 + 0 t ,
donde 0 es el ngulo en el instante t = 0 y 0 es una constante que determina
a
cun rpido var el ngulo. Observemos la analog entre la expresin anterior y la
a a
a a
a
o
ecuacin de itinerario para la posicin en un movimiento uniforme en una dimensin.
o
o
o
La variable lineal x es reemplazada por el ngulo , y la velocidad lineal v por la
a
as llamada velocidad angular .

Las componentes x e y del vector posicin vienen dadas por


o
x(t) = R cos (t) = R cos(0 + 0 t)
e
y(t) = R sin (t) = R sin(0 + 0 t).
El vector posicin es, por lo tanto,
o

^
y

v (t)

y (t)
r (t)

r(t) = R cos(0 +0 t) +R sin(0 +0 t) .


x
y
Derivando r(t) se encuentra la velocidad

(t)

x (t)

v(t) = R0 sin(0 + 0 t)
x

+ R0 cos(0 + 0 t) .
y

Evaluemos el mdulo de la velocidad (rao


pidez):
Figura 3.13

^
x


CAP
ITULO 3. CINEMATICA EN DOS Y TRES DIMENSIONES

v = |v(t)| =
=

111

vx (t)2 + vy (t)2

2
2
R2 0 sin2 (0 + 0 t) + R2 0 cos2 (0 + 0 t) = R0 .

A pesar de que la rapidez es constante (no depende del tiempo), la velocidad no lo


es, ya que continuamente cambia de sentido (y por tanto, existe una aceleracin).
o
Esta ultima ecuacin ensea que la velocidad angular es la rapidez de la part

o
n
cula
dividida por el radio de giro. Lo cual es razonable, considerando lo siguiente: si la
velocidad angular es constante, entonces la part
cula debe dar una vuelta completa al
c
rculo (es decir, recorrer un ngulo 2), siempre en el mismo tiempo (el per
a
odo T ).
Equivalentemente, la part
cula debe recorrer la circunferencia completa (una distancia
2R, en el mismo tiempo T ). Es decir:
0 =

1 2R
1
2
=
= v.
T
R T
R

Si observamos la expresin para v recin obtenida, notaremos que, cuando la


o
e
part
cula se encuentra sobre el eje x ((t) = 0), su velocidad es R0 y , vertical hacia

arriba. Cuando se encuentra sobre el eje y ((t) = /2), su velocidad es R0 x,

horizontal hacia la izquierda. As sucesivamente, esto sugiere que la velocidad es un

vector siempre tangente a la circunferencia.


Podemos vericar lo anterior evaluando el producto punto entre r y v:
r(t) v(t) = x(t)vx (t) + y(t)vy (t) = 0 .
Como el producto punto de dos vectores no nulos vale cero slo si los dos vectores son
o
perpendiculares, se halla que la velocidad de una part
cula en un movimiento circular
uniforme es siempre perpendicular al radio.
Derivando la velocidad se encuentra la aceleracin:
o
2
2
a(t) = R0 cos(0 + 0 t) R0 sin(0 + 0 t) .
x
y

Note que en todo instante


2
a(t) = 0 r(t) ,

o sea, la aceleracin siempre apunta hacia el origen (razn por la cual se llama aceo
o
leracin centrpeta). La magnitud de la aceleracin siempre es constante y vale
o

o
2
a = |a(t)| = R0 .

Observemos que podr


amos haber concluido desde un comienzo que la aceleracin
o
deb ser radial. En efecto, la aceleracin es proporcional a la diferencia de velocidades
a
o
entre dos instantes innitamente cercanos, v(t) y v(t + ). Si la velocidad se debe
mantener tangente al c
rculo, breves segundos de reexin nos deber convencer
o
an
que el vector diferencia v(t) = v(t + ) v(t) debe ser puramente radial:


CAP
ITULO 3. CINEMATICA EN DOS Y TRES DIMENSIONES

112

v
v(t)
v(t+ )

3.3.

Coordenadas polares

Los vectores unitarios r y .



Hemos visto que el movimiento de un punto P en el plano x, y se puede especicar
usando dos funciones que describan sus coordenadas cartesianas del punto, o sea,
r(t) = x(t) x + y(t) y .

Tambin podemos especicar el movimiento P usando coordenadas polares, es decir,


e
dando las funciones r(t) y (t). Deber ser posible, por analog a las coordenadas
a
a
cartesianas, escribir el vector posicin en la forma:
o

r(t) = r(t) + (t) .


r
Qu son los vectores r y ?
e

Al menos para r es evidente cmo denirlo:

o
r=

r
,
r

un vector unitario en la direccin radial (ver Fig. 3.14). De la misma gura podemos
o
deducir que
r = cos x + sin y .

Y ? Deber ser un vector perpendicular a r , y por tanto tangente a la circunferencia


a

de radio r. Existen dos direcciones perpendiculares, pero elegimos de modo que


apunte en la direccin en que aumenta el ngulo , como se indica en la Fig. 3.14.
o
a
Nuevamente, podemos deducir de la gura que

= sin x + cos y .


CAP
ITULO 3. CINEMATICA EN DOS Y TRES DIMENSIONES
Ejercicio: Demuestre que los vectores

r y efectivamente son unitarios.

Tambien demuestre que son ortonormales, es decir, r .


^
y

Observe que estos vectores unitarios generalmente (cuando = (t)


depende del tiempo) son tiempo dependientes. El vector r apunta en la

direccin radial, mientras que el vector


o

es tangencial al c
rculo que pasa por
P y tiene su centro en el origen.

113

^
r
P

^
x

Figura 3.14
En principio, puede parecer incmodo el haber denido dos vectores unitarios que
o
dependen del tiempo. Y no slo que dependen del tiempo, sino que dependen de la
o
posicin de la propia part
o
cula cuyo movimiento se intenta describir. Efectivamente, es
menos sencillo trabajar en este sistema de coordenadas que en el sistema cartesiano,
que es independiente del tiempo y de la part
cula a observar. Pero nada, en principio,
nos prohibe trabajar en tal sistema, y de hecho encontraremos muchas situaciones en
que es la mejor opcin (a pesar de las apariencias).
o
Ya que los vectores unitarios var con el tiempo, calculemos su derivada teman
poral. Se tiene
d
[ cos x + sin y ]

dt
d sin (t)
d cos (t)
x+

=
dt
dt

= sin((t)) (t) x + cos((t)) (t) y

= (t) [ sin((t)) x + cos((t)) y ] =

d
r

=r =

dt

= =
dt

d
[ sin x + cos y ]

dt
d cos (t)
d sin (t)
x+

=
dt
dt

= cos((t)) (t) x sin((t)) (t) y

= (t) [ cos((t)) x + sin((t)) y ] = r

Resumen:

r=

(3.1)


CAP
ITULO 3. CINEMATICA EN DOS Y TRES DIMENSIONES

= r .

114
(3.2)

Los resultados anteriores son completamente generales, y no dependen de que el


movimiento sea uniforme, y ni siquiera circular. Notemos adems que son complea
tamente esperables. En efecto, de la Fig. 3.14 es evidente que si r cambia, la unica

posibilidad es que la part


cula cambie su ngulo . Cambios en su coordenada r(t)
a

mantienen el vector r constante. Por lo tanto, r deba ser un vector paralelo a la

direccin , y adems proporcional a (de modo que si es constante, r tambin lo


o
a

sea). Anlogamente, si r(t) cambia ello no altera en la Fig. 3.14. Nuevamente, la


a

unica manera de que cambie es que lo haga, de modo que debe ser proporcional

tambin a . Pero ahora, al cambiar el ngulo, , que es tangente a la circunferencia,


e
a
cambia en direccin radial hacia el centro del c
o
rculo (la razn es la misma que la que
o
hace que la aceleracin en un movimiento circular uniforme sea centr
o
peta), y por

deb ser proporcional a .


tanto
a
r
Movimiento circular (en coordenadas polares).
Estudiemos ahora el mismo problema anteriormente resuelto, de una part
cula
en movimiento circular, pero ahora en coordenadas polares.
Consideremos un punto P que se mueve en torno al origen sobre un c
rculo de
radio R y sea (t) el ngulo polar (medido respecto al eje x y en el sentido contrario
a

al avance del reloj). El vector posicin del punto P es:


o
r(t) = R r(t) .

La velocidad es:

v = r(t) = R r = R ,

y la aceleracin es:
o
d

a = v(t) = R ( )
dt


= R ( + )


= R R 2 r
En el caso particular en que el movimiento es uniforme, la velocidad angular

0 es constante. Entonces
r = R ,
r

v = R0 ,
2
a = R0 r .


CAP
ITULO 3. CINEMATICA EN DOS Y TRES DIMENSIONES

115

Estos resultados, por supuesto, son consistentes con los anteriormente obtenidos en
coordenadas cartesianas. De hecho, es trivial recuperar todos los hechos que ya discutimos sobre el movimiento circular uniforme simplemente inspeccionando las igualdades anteriores: La distancia al origen es constante (| r | es constante); la rapidez
es constante e igual a | v | = R0 ; el mdulo de la aceleracin es constante e igual
o
o
2
o
a | a | = R0 ; la velocidad es perpendicular a la posicin en todo momento (r es

proporcional a r , mientras que v lo es a ); y la aceleracin es centr

o
peta (a tiene
direccin ).
o
r
Podemos apreciar, entonces, que la descripcin del movimiento circular uniforo
me es considerablemente ms sencilla en coordenadas polares, a pesar de que, al
a
introducirlas, los vectores unitarios dependen del tiempo. Naturalmente, lo complejo
del movimiento circular, que antes se traduc en tener que derivar funciones trigoa
nomtricas, ahora se traduce en tener que derivar los vectores unitarios. Ah esta
e

encapsulada la no trivialidad del movimiento. Pero una vez que sabemos cmo deo
penden r y del tiempo, encontrar la posicin, la velocidad y la aceleracin es muy

o
o
sencillo, y la interpretacin es inmediata, no como en el caso de coordenadas carteo
sianas.
Notemos que si el movimiento ahora es no uniforme, sigue siendo cierto que la
velocidad es perpendicular a la posicin. Debe ser as porque la velocidad es, por
o
,
la propia denicin de derivada, tangente a la trayectoria, de modo que si sta es
o
e
un c
rculo, la velocidad debe tener direccin . En cambio, la aceleracin deja de ser
o
o

puramente centr
peta. Adems de una aceleracin radial R 2 r , aparece un trmino
a
o
e
. Esta aceleracin tangencial deb aparecer, puesto que un movimiento circular

R
o
a
slo puede ser no uniforme si el ngulo no var linealmente con el tiempo, en cuyo
o
a
a
= 0, y si dicha aceleracin es precisamente en la direccin angular, . La

caso
o
o
aceleracin, as tiene dos trminos, uno radial, que se encarga de que el cuerpo se
o
,
e
mueva en un c
rculo, y uno tangencial, que cambia la velocidad con la cual se recorre
el c
rculo.

3.4.

Problemas

1. Sean A, B y C los vectores A = 2 + y , B = 3 + y 2 y C = x + 3 z .


x
x
z

y
a) Encuentre el mdulo de A, B y C.
o
b) Encuentre el mdulo del vector suma, o sea, evale
o
u
D = |D| = |A + B + C| .
c) Cul vector es ms largo: A + B o A + C ? En vista de lo calculado en
a
a
la parte a), le sorprende este resultado?
d) Encuentre el ngulo entre los vectores B y C.
a


CAP
ITULO 3. CINEMATICA EN DOS Y TRES DIMENSIONES
Respuesta: d)

116

49,86 .

2. Demuestre que los vectores:


A = cos () + sin ()
x
y
B = cos () + sin ()
x
y
son vectores unitarios que forman un ngulo y con el eje x, respectivamente.
a

Evale A B y encuentre una frmula para cos ( ).


u
o
3. Considere los tres puntos cuyas coordenadas cartesianas vienen dadas por: P1 =
(1, 1, 1), P2 = (1, 2, 0) y P3 = (2, 3, 1). Demuestre que ellos denen los vrtices
e
de un tringulo rectngulo.
a
a

4. Encuentre un vector unitario A que sea simultneamente perpendicular a los


a

vectores u = 2 + y z y v = x y + z . Cuntos vectores unitarios A


x

a
existen con esta propiedad?
5. Denamos los vectores:

1
x
s = ( + y )
2
1
t = ( + y )
x
2

a) Graque s y t .
b) Evale s = |s | y t = |t | .
u

c) Encuentre el ngulo entre s y t .


a
Comentario: Note que s y t pueden considerarse como un nuevo conjunto
de ejes de referencia (, t ). Para indicar que s y t son vectores unitarios
s
se ha usado la convencin de reemplazar las echas por tongos.
o

d) Considere los vectores A = x + 2 y B = 2 3. Exprese estos vectores

y
x
y
en trminos de los nuevos vectores unitarios, es decir, escriba A y B de la
e
forma

A = as s + at t

B = bs s + bt t

y evale las constantes as , at , bs y bt .


u
e) Evale A B de dos maneras distintas: primero usando las componentes
u
respecto al sistema de referencia (, y ) y luego usando las componentes
x
respecto al sistema de referencia (, t ).
s


CAP
ITULO 3. CINEMATICA EN DOS Y TRES DIMENSIONES

117

6. Sea A = x + 3 2. Encuentre un vector B en el plano x, y que sea perpen


z
y

dicular a A.
Respuesta:

B = (2 + y ), donde es un nmero real no nulo.


x
u

7. Considere la siguiente situacin en nuestro espacio f


o
sico de tres dimensiones:
Desde cierto origen emergen cuatro vectores de igual tamao, de manera que
n
los ngulos entre cualquier par de vectores sean iguales. Encuentre el valor de
a
ese ngulo. (Para resolver este problema relacinelo con el de las diagonales de
a
o
un cubo considerado en la seccin 3.1.)
o
Comentario: Las puntas de los cuatro vectores forman los vrtices de un
e
tetraedro regular. La molcula de metano CH4 es un ejemplo de lo arriba
e
planteado. En tal molcula el tomo de carbono se encuentra al centro de los
e
a
cuatro tomos de hidrgeno que estn distribuidos de la manera ms regular
a
o
a
a
posible.
8. Encuentre el ngulo entre dos vectores de 8 y 10 unidades de largo, si el vector
a
suma forma un ngulo de 50 con el mayor de ellos. Encuentre tambin la
a
e
magnitud del vector suma.
9. La suma de dos vectores mide 30 unidades y forma ngulos de 25 y 50 con
a
ellos. Cul es la magnitud de cada uno de los vectores?
a
10. Suponga que la posicin r de una part
o
cula en funcin del tiempo t viene dada
o
por:
t
t
x + sin

y ,

r = r (t) = r0 cos
t0
t0
con t0 = 1 min y r0 = 3 cm. Qu trayectoria recorre la part
e
cula? Cunto
a
tiempo tarda la part
cula en volver al punto de partida?
11. Supongamos que la posicin r de una part
o
cula en funcin del tiempo t viene
o
dada por
r = at x + (b ct2 ) y ,

con a = 2 m/s, b = 10 m y c = 9.8 m/s2 . Graque la trayectoria. Qu tipo


e
de trayectoria es? En qu instante la part
e
cula cruza el eje x?


CAP
ITULO 3. CINEMATICA EN DOS Y TRES DIMENSIONES

118

12. Un barco a vapor se dirige hacia el sur


con una velocidad vb = 25 km/h en un
a
rea donde sopla un viento desde el suroeste con velocidad v0 = 18 km/h. Encuentre el ngulo 0 que forma el humo
a
emitido por el vapor con la direccin
o
nortesur (ver gura 3.15).
Respuesta: 0 18, 64

Figura 3.15
13. Considere un disco de radio R = 50 cm
que rueda sobre una recta (el eje x )

con una velocidad angular = 2 s1 .


Considere un punto P ubicado en el
per
metro del disco, y designe por r al
vector que va desde el origen hacia el
punto P . Encuentre una expresin pao
ra r = r (t); suponga que en el instante
t = 0 el punto P est en el origen.
a

Figura 3.16

Haga un grco de r (t) para el intervalo t [0 s , 10 s ]. Cunto tarda la rueda


a
a
en dar una vuelta completa?
14. Una part
cula recorre una trayectoria circular en el plano xy, cuyo radio es
R = 5 m con una velocidad constante v0 = 15 m/s y en el sentido del reloj.
Encuentre el vector posicin r(t), el vector velocidad v(t) y el vector aceleracin
o
o
a(t) (en coordenadas cartesianas) si en el instante t = 0 la part
cula se encuentra
en r0 = 5.
y
15. Considere un disco de radio R en el
plano xy. Sea el ngulo de un punto
a
ubicado en el borde del disco respecto al eje x. Suponga que el disco gira

con una aceleracin angular constante


o

0 (es decir, (t) = 0 ). Encuentre la


velocidad y aceleracin de P en funcin
o
o
del tiempo. Suponga que en el instante t = 0 el punto P se encontraba en
reposo sobre el eje x.

Figura 3.17

16. Estime (en m/s y km/h) la velocidad mxima con la que usted puede lanzar
a


CAP
ITULO 3. CINEMATICA EN DOS Y TRES DIMENSIONES

119

una piedra.
17. Una pelota sale rodando del descanso de una escalera con velocidad horizontal
v0 = 1.52 m/s. Los escalones son de 20 cm de alto y 20 cm de ancho. Cul
a
ser el primer escaln al que llegue la pelota? Dibuje una gura para ilustrar el
a
o
problema.
18. Un caon se encuentra a una distancia
n
D de un edicio. Encuentre el ngua
lo de elevacin 0 y la velocidad v0 de
o
la bala de manera que el proyectil entre horizontalmente por la ventana que
se encuentra a una altura h (ver gura
3.18).
Figura 3.18
19. Considere un r de ancho L en el cual
o
el agua uye con velocidad v0 . Un nadador recorre el trayecto A B
A, mientras que un segundo nada el
trayecto C D C (ver gura 3.19). Los puntos C y D estn ana
clados jamente al fondo del r y la
o
separacin entre C y D es la misma
o
que entre A y B. Si ambos nadan con
la misma velocidad v respecto al agua,
quin ganar la carrera?
e
a

Figura 3.19

20. Un pato vuela horizontalmente en


l
nea recta con velocidad vp a una
altura h. Un nio con una honda,
n
que puede disparar piedras con una
velocidad v0 , hace uso de su arma en
el instante que el pato lo sobrevuela.
a) Cul es el ngulo respecto a
a
a
la normal con el cual debe disparar la piedra?
b) Qu distancia d alcanza a ree
correr el pato antes de ser alcanzado por el proyectil?

Figura 3.20


CAP
ITULO 3. CINEMATICA EN DOS Y TRES DIMENSIONES

120

(c) Cul es la velocidad m


a
nima que debe tener el proyectil para que ste
e
llegue al pato?
21. Se lanza un proyectil con cierto
a
ngulo de elevacin 0 . El alcance
o
del proyectil es R (ver gura 3.21).
Si se desprecia el roce con el aire,
demuestre que la trayectoria viene
dada por la ecuacin
o
y(x) =

tan 0
R

x2 + x tan 0 .
Figura 3.21

Note que esta ecuacin corresponde a una parbola. Demuestre tambin que el
o
a
e
a
ngulo de la tangente en el punto x viene impl
citamente dado por
tan = 1

2x
tan 0 .
R

22. Graque en papel polar la trayectoria de una part


cula si su posicin en cooro
denadas polares, en funcin del tiempo, viene dada por:
o
a)

r(t) = r0
(t) = t/t0
con r0 = 1 [m] y t0 = 2 [s].

b)

r(t) = At
(t) = t/t0
con A = 1/(4) [m/s] y t0 = 2 [s].

c)

r(t) = r0 + B cos (t/2t0 )


(t) = t/t0
con r0 = 1 [m], t0 = 2 [s] y B = 0.5 [m].

23. Una part


cula se encuentra en el instante t = 0 en el lugar r(0) = 10 cm y
y
tiene una velocidad v(0) = 2 cm/s. La aceleracin en todo instante es
x
o
a = G

r
,
r3

con G=200 cm/s2 . Encuentre numricamente la trayectoria de la part


e
cula para
t [0, 3.5 s]. Graque!


CAP
ITULO 3. CINEMATICA EN DOS Y TRES DIMENSIONES

121

Indicacin: programe las siguientes relaciones


o
r(t + ) r(t) + v(t)
v(t + ) v(t) + a(t)

a(t + ) = Gr(t + )/r 3 (t + ) .


24. Calcule la mxima distancia que un
a
objeto puede alejarse del borde de un
peldao para evitar ser alcanzado
n
por los objetos lanzados con velocidad
v0 desde el punto A. La distancia desde
A al borde del peldao es L y la altura
n
de ste es H.
e
Figura 3.22
25. Un proyectil se lanza con velocidad inicial v0 y ngulo de lanzamiento , ama
bos conocidos. El proyectil sobrepasa
una barrera rectangular de ancho a conocido, pero altura h desconocida, rozando sus dos vrtices A y B (ver gue
ra 3.23). Encuentre la distancia d que
separa el punto de lanzamiento con la
pared ms cercana al obstculo. Tama
a
bin encuentre la altura h de la barree
ra.

Figura 3.23

26. Una part


cula tiene un vector posicin dado por r = 30 t x + (40 t 5 t2 ),
o

y
donde r est en metros y t en segundos. Encuentre los vectores velocidad y
a
aceleracin instantneas.
o
a
27. Desde una distancia d del borde recto de un tobogn se dispara una
a
bengala. Si el tobogn tiene una ala
tura h y un largo b, determinar ambas componentes de la velocidad inicial del proyectil para que ste atee
rrice sobre el vrtice superior del toe
bogn de manera que su velocidad
a
sea paralela al plano inclinado.
Figura 3.24


CAP
ITULO 3. CINEMATICA EN DOS Y TRES DIMENSIONES

122

Respuesta:
v=d

gb
x + (2 b + d)

2 h (b + d)

hg
z .

2 b (b + d)

28. Supongamos que r(t) y (t) son las coordenadas polares de un punto que se
mueve en un plano. Demuestre que la velocidad de tal punto, en coordenadas
cartesianas, viene dada por
v(t) =

dr
d
d
dr
cos r
sin x +

sin + r
cos y

dt
dt
dt
dt

= r cos r sin x + r sin + r cos y .

Encuentre la velocidad en coordenadas cartesianas para los tres casos del problema 22.
29. Una part
cula tiene aceleracin constante
o
a = (6 x + 4 y )[m/s2 ] .

En t = 0 la velocidad es cero y el vector posicin es x0 = 10 x [m].


o

a) Encuentre los vectores velocidad y posicin en un instante t cualquiera.


o

b) Encuentre la ecuacin de la trayectoria en el plano y dibjela.


o
u
30. De un caon se disparan dos proyectiles: el primero con un ngulo de elevacin
n
a
o
1 = 60 y el segundo con un ngulo de elevacin 2 = 45 . La velocidad de los
a
o
proyectiles, al emerger del caon es v0 = 250 m/s. Despreciando la resistencia
n
del aire, encuentre el intervalo de tiempo entre los dos disparos que asegure que
los proyectiles choquen.


CAP
ITULO 3. CINEMATICA EN DOS Y TRES DIMENSIONES
31. La gura indica la conexin en una caja
o
de cambios de un automvil. Encuentre
o
la razn entre los radios de ambos eno
granajes, que es la misma para ambos
pares, si uno desea que en la primera
marcha, con el motor a 2000 RPM, el
auto tenga una velocidad de 30 Km/h.
Por cada cinco vueltas en la salida de la
caja de cambios, las ruedas, cuyo radio
es de 50 cm, dan una vuelta.

123

Figura 3.25

32. Consideremos una turbina hidrulica.


a
Supongamos que el agua ingresa a la
turbina con una velocidad v, con v =
|v| = 15 m/s, formando un ngulo con
a
la tangente al rotor en el punto de entrada = 30 (ver gura 3.26). Suponga adems que el radio externo del
a
rotor es R = 2 m y que, en su estado
estacionario, el rotor gira a 30 RPM (o
sea, con frecuencia = 0, 5 s1 ).
La forma de las paletas de un rotor de
una turbina hidrulica es tal que la vea
Figura 3.26
locidad relativa entre el agua que ingresa a la turbina y la paleta en el punto
de entrada, sea tangente a la paleta (de
esta manera el agua ingresa a la turbina sin choques).
Determine el ngulo entre la paleta del rotor y la tangente al rotor en el
a
punto de entrada de agua. Encuentre tambin la velocidad relativa vr del agua
e
(respecto a la paleta) en ese punto.
Respuesta:

tan =

v sin
;
v cos 2R

vr = 10, 06 [m/s] .

33. Una part


cula se mueve en el plano x-y con una velocidad (que depende de la
posicin) v = a + bx, donde a y b son constantes. En el instante inicial la
o
x
y
part
cula se encuentra en el origen (x(0) = y(0) = 0). Encuentre la ecuacin de
o
la trayectoria y(x).
b 2
Respuesta:
x .
y(x) =
2a


CAP
ITULO 3. CINEMATICA EN DOS Y TRES DIMENSIONES

124

34. Un mono est colgado a una altura


a
h de un rbol. Un cazador apunta
a
con una cerbatana directamente al
mono desde una distancia d (ver gura 3.27). En el mismo instante en
que el cazador sopla el dardo envenenado el mono se suelta del rbol.
a
Sobrevivir el mono? (Desprecie el
a
efecto de friccin del dardo con el
o
aire)
Figura 3.27
35. Una rueda gira en torno a un eje horizontal a 30 rpm (1 rpm = una revolucin
o
por minuto = 1 vuelta por minuto), de manera que su parte inferior queda a
nivel del suelo, pero sin rozarlo. (O sea, la rueda gira sin rodar).
Sobre el borde de la rueda se han adosado dos piedrecitas, en posiciones diametralmente opuestas.
a) Suponga que cuando el dimetro que une a las piedras pasa por la posia
cin horizontal, stas se desprenden del borde, en forma simultnea (gura
o
e
a
3.28a), y una de ellas llega al suelo antes que la otra. Se observa que durante el intervalo de tiempo entre la llegada al suelo de una y otra piedra,
la rueda da una vuelta completa. Determine el radio de la rueda.
b) Qu ngulo debe formar la l
ea
nea que une a ambas piedras con la vertical
para que, si las piedras se desprenden en esa posicin, lleguen al suelo al
o
mismo tiempo?

Figura 3.28a

Figura 3.28b


CAP
ITULO 3. CINEMATICA EN DOS Y TRES DIMENSIONES

125

36. Un globo sonda es soltado desde la tierra y se aleja con velocidad constante
en trayectoria recta la cual forma un
a
ngulo de 30 con la vertical. La velocidad del viento con respecto al suelo
es de 10 [km/h], estable, hacia el norte.
a) Calcule la velocidad del globo
respecto al aire.
b) Calcule el tiempo que tarda el
globo en alcanzar una altura de
1 km con respecto al suelo.
37.

Figura 3.29

Una rueda de radio 0,25 [m] ha estado girando en forma uniforme a razn de
o
una revolucin por segundo. En cierto instante la rueda es frenada y se detiene,
o
uniformemente, despus de haber girado media vuelta. Calcule la aceleracin
e
o
tangencial y centr
peta de un punto jo en el borde de la rueda cuando sta
e
comienza a ser frenada.

38. Dos proyectiles son lanzados simultneamente desde el mismo punto


a
en un plano horizontal. Los proyectiles
son lanzados con igual rapidez y con
a
ngulos con respecto a la horizontal
y , respectivamente ( < ). Ambos
proyectiles llegan al mismo punto en la
horizontal pero a instantes diferentes.
Demuestre que lo descrito es posible y
encuentre la razn entre los tiempos
o
de llegada. (Expresar el resultado en
trminos de ).
e

Figura 3.30

39. Un proyectil es lanzado desde un plano


inclinado cuyo ngulo de inclinacin
a
o
con la horizontal es . Si el proyectil es
lanzado con rapidez v0 y con un ngua
lo de eyeccin con respecto al plano
o
(ver gura 3.31), calcule el alcance D
del proyectil a lo largo del plano.
Figura 3.31


CAP
ITULO 3. CINEMATICA EN DOS Y TRES DIMENSIONES

126

40. El avix, una apetitosa ave del tiempo de las cavernas, desarroll por un proceso
o
de evolucin, una coraza en la parte inferior de su cuerpo de manera que los
o
trogloditas no pod cazarlas con arcos y echas.
an
Og, un ingenioso troglodita, desarroll un mtodo para cazarla aprovechando
u
o
e
que el ave no tiene coraza sobre el dorso. El disparaba echas que impactaran
al avix por arriba.
Dados la velocidad del ave vave , la altura h a la que vuela, la velocidad v0 con
que la echa es impulsada por el arco y el ngulo (respecto a la horizontal)
a
con que el troglodita dispara la echa, calcular:
a) El tiempo que le toma a la echa pasar por la altura h la segunda vez.
b) El valor de la distancia d entre el ave y la vertical por el punto de lanzamiento, en el instante del lanzamiento, para que la echa impacte al
ave.

Figura 3.32
41. Se lanzan dos proyectiles A y B de modo que tienen igual alcance horizontal L.
A se lanza horizontalmente desde una altura h, que es igual a la altura mxima
a
que alcanza B durante su vuelo (ver gura 3.33)
a) Calcule la razn entre los tiempos
o
de vuelo de A y B.
b) Calcule la razn entre las compoo
nentes horizontales de la velocidad de los proyectiles.
c) Cul es la rapidez (magnitud de
a
la velocidad) de cada uno de ellos
al llegar al suelo?
Figura 3.33


CAP
ITULO 3. CINEMATICA EN DOS Y TRES DIMENSIONES

127

42. Una part


cula se desplaza sobre una trayectoria circular de radio R. Inicialmente
se encuentra en reposo en un punto A dado de dicha trayectoria. Para todo
tiempo t posterior, la part
cula ha recorrido una distancia s(t) = t desde
A, donde y son constantes, y tiene unidades de m/s . Determinar las
componentes tangencial y normal de la velocidad y la aceleracin de dicha
o
part
cula, para todo tiempo t > 0.
43. Un aguerrido topo con superpoderes decide saltar desde una altura h a su
madriguera.
a) Determine la velocidad inicial v0 con que debe impulsarse para llegar a
su madriguera en 3 segundos. Suponga que h = 1 [m] y d = 1.5 [m], y
considere g = 10 m/s2 .
b) Suponga ahora que el topo quiere hacer un salto ms elaborado. Desea
a
saltar con una velocidad inicial de 5 m/s, formando un ngulo de 45 con
a
la horizontal. El topo sabe (porque l construy la madriguera) que el
e
o
a
ngulo es 60 . A qu distancia horizontal d deber estar la madriguera
e
a
para que la velocidad del topo, al llegar a la madriguera, sea paralela al
tnel, y pueda as entrar cmodamente desde el aire?
u

v0

El topo y su madriguera

1111111
0000000
1111111
0000000
1111111
0000000
1111111
0000000
1111111
0000000
1111111
0000000
1111111
0000000
1111111
0000000
1111111
0000000

44. Romeo se acerc temeroso a la torre donde estaba Rapunzel, su amada. Ya hab
o
a
tenido abundantes problemas con el padre de Rapunzel, quien lo rechazaba como
yerno a causa de su humilde origen. De modo que Romeo slo pod encontrarse
o
a
con ella a escondidas. Ese d se acerc sigilosamente a la hora de la siesta. Tanto
a
o
miedo ten que se qued a una prudente distancia L de la torre. Arriba, por
a,
o
una ventana a una altura h, se asomar Rapunzel. Si slo pudiera avisarle
a
o
que l la esperaba abajo. Mir rpidamente a su alrededor, tom un guijarro
e
o a
o
y lo lanz hacia la ventana con velocidad v0 . El lanzamiento fue perfecto, pero
o
Romeo no se dio cuenta de que, debido al suave calor estival, Rapunzel hab
a
dejado la ventana abierta, y el guijarro mensajero no choc con la ventana, sino
o


CAP
ITULO 3. CINEMATICA EN DOS Y TRES DIMENSIONES

128

que continu su trayectoria, cayendo justo en el patio, donde su padre dorm


o
a
siesta.

v0

L
R
Romeo intenta llamar la atencin de Rapunzel.
o
Oops!, exclam Rapunzel. Y puesto que su padre siempre duerme siesta en el
o
mismo sitio, se propuso decirle a su galn, la prxima vez que lo viera, que si se
a
o
va a poner a una distancia L de la torre, no lance de nuevo piedritas con el mismo
a
ngulo de hoy. Ay, Romeo! Tan tierno, pero tan torpe, pens Rapunzel, y
o
se volvi a acostar mientras o los pasos de Romeo huyendo por las callejuelas
o
a
vecinas.
Si el padre de Rapunzel estaba durmiendo a una distancia horizontal R de
Romeo, cul es el ngulo con que Romeo lanz la piedra? Y a qu distancia
a
a
o
e
L de la torre estaba cuando lo hizo? Observar que hay dos soluciones posibles
a
para L. Qu signica cada solucin?
e
o
(Para simplicar el problema, suponga que el guijarro es lanzado desde el suelo.)
45. Thelma y Louise hab llegado al nal de su aventura. Fue un par de semanas
an
incre
bles, pero ahora eran prfugas de la justicia, y estaban acorraladas. Detrs
o
a
de ellas, toda la polic de Huechuraba las conminaba a rendirse. Delante de
a
ellas, un profundo acantilado las esperaba. Thelma y Louise se miraron por ulti
ma vez. Se tomaron de la mano. Thelma pisa el acelerador, y el auto comienza
a moverse con aceleracin a. Tras recorrer una distancia d, llega al borde del
o
acantilado y, ante la sorpresa de la polic y la prensa especializada apostada en
a
el sitio del suceso, el veh
culo cae al vac con las dos ocupantes en su interior.
o
Si el acantilado ten una profundidad h, a qu distancia L de la pared del
a
e
acantilado se estrell el veh
o
culo? Cunto tiempo transcurri desde que el auto
a
o
comenz a acelerar hasta el impacto nal?
o


CAP
ITULO 3. CINEMATICA EN DOS Y TRES DIMENSIONES

129

L
Thelma y Louise.
46. 2001: Odisea del Espacio (Msica de Richard Strauss aqu
u
.)
(Continuacin del Problema 2.44.)
o
Tras un largo camino, Harold llega nalmente al borde de una quebrada de profundidad H. Ha tenido tiempo de pensar. El incidente con la seora y su beb,
n
e
que vio por el espejo retrovisor, lo hizo cuestionarse su accionar. Hab traia
cionado a Eric, su amigo de infancia; hab puesto en peligro a inocentes; y se
a
hab traicionado a s mismo, al dejarse tentar por dinero mal habido. Sentado
a

al borde de la quebrada con el testimonio en la mano, sinti rabia, mucha rabia


o
consigo mismo. La informacin en ese testimonio val millones. Entregarla a
o
a
la maa ser inmoral; entregarla a la polic signicaba ser hombre muerto.
a
a
Harold no pod pensar con claridad. Agobiado por sus dudas, decidi deshaa
o
cerse del testimonio. Lo tom con fuerza entre sus manos, y lo azot contra una
o
o
piedra. . . una, dos, tres veces. El sonido de los golpes lo excitaba cada vez ms,
a
y lo obligaba a golpear con aun mayor fuerza. Harold estaba completamente
fuera de s Se puso de pie y con fuerza, con mucha fuerza, lanz el testimonio,
.
o
todo abollado, hacia el cielo. (Msica de Strauss ms fuerte.)
u
a
a) Si Harold lanza el testimonio con velocidad inicial v0 y con un ngulo
a
respecto a la horizontal, cunto tiempo transcurre desde que Harold lanza
a
el testimonio hasta que cae al fondo de la quebrada?
b) A qu distancia horizontal de Harold, LT H , quedar el testimonio una
e
a
vez que llegue al fondo de la quebrada?


CAP
ITULO 3. CINEMATICA EN DOS Y TRES DIMENSIONES

130

v0

L TH

La furia primordial.
Por unos segundos, Harold qued mirando al vac sintiendo la brisa que se
o
o,
levantaba con intensidad, acallando apenas el an dbil sonido de las sirenas
u
e
policiales que, con el agente Doyle a la cabeza, se acercaban a gran velocidad.
Harold se hab deshecho de la evidencia, pero no de su culpa.
a
Esta historia continuar. . .
a
47. Jack Sully ha llegado a una encrucijada en su vida. Los Navi desconf de l, y
an
e
sabe que su unica posibilidad es domesticar al Toruk. En las islas otantes, Jack

aguarda el momento oportuno, a una distancia horizontal L, y a una altura H


respecto al Toruk. En t = 0, el Toruk comienza a volar con velocidad horizontal
vT constante. En ese mismo instante, Sully salta con velocidad vS hacia el Toruk,
con un ngulo respecto a la horizontal. Si vT = vS , qu valor debe tener
a
e
para que Sully caiga justo sobre el Toruk y pueda controlarlo?


CAP
ITULO 3. CINEMATICA EN DOS Y TRES DIMENSIONES

131

vS

vT

H
L

48. Pitgoras tiene un problema grave: no sabe dibujar tringulos rectngulos. Ana
a
a
gustiado, sabiendo que corre el riesgo de no poder demostrar su Teorema, se
conecta a www.sos-trigono.gr, donde encuentra las siguientes instrucciones:
a) Elija dos puntos diferentes en el plano x-y (Pitgoras escogi los puntos P1
a
o
b

y P2 dados por los vectores A = a + b y B = 2a + 2 y , respectivamente.)


x y
x
b) Encuentre un vector perpendicular a la l
nea que los une, de mdulo C.
o
c) Con ese vector, encuentre un punto P3 tal que P1 , P2 y P3 sean los vrtices
e
de un tringulo rectngulo, y que uno de sus catetos tenga largo C.
a
a
Ayude Ud. a Pitgoras, y encuentre el punto P3 , siguiendo las instrucciones
a
anteriores, partiendo de los puntos escogidos por l.
e
49. Protegido por la oscuridad de la noche, el galn enamorado se acerca hasta una
a
distancia D del muro donde su amada duerme. A una altura h1 desde el suelo
se encuentra su ventana. Para asegurarse de llamar su atencin y confesarle por
o
n sus sentimientos, el mozalbete lanza un guijarro, con ngulo 1 respecto al
a
suelo, y un corto tiempo T despus, una segunda piedra. La primera piedrecilla
e
lleg a la ventana, pero tan pequea era que, aunque golpe la ventana, no
o
n
o
logr despertar a la joven.
o
En cuanto a la segunda piedra, sta era bastante ms grande, pero el muy torpe
e
a
la lanz con un ngulo 2 respecto al suelo, y la piedra termin golpeando en
o
a
o
otra ventana a una altura h2 < h1 del suelo, despertando a la madre, quien
dorm ah justamente para proteger la castidad de su primognita.
a
e
Si nuestro protagonista lanza ambos proyectiles con velocidades de mdulo v0 ,
o
cul es la diferencia de alturas, h = h1 h2 , entre ambas ventanas?
a


CAP
ITULO 3. CINEMATICA EN DOS Y TRES DIMENSIONES

132

g
h1
h2

D
50. Un cuerpo sube con velocidad constante v0 , en diagonal, de modo que su trayectoria forma un ngulo respecto a la horizontal. Al mismo tiempo que el cuerpo
a
comienza a subir, se lanza un proyectil con una velocidad inicial vp , formando
un ngulo > con la horizontal (ver gura).
a
Encuentre la distancia D que debe separar el punto inferior del plano inclinado
y el punto de lanzamiento del proyectil para que el cuerpo y el proyectil se
encuentren.

v0
vp

3.5.

Solucin a algunos de los problemas


o

Solucin al problema 18.


o
Coloquemos el origen en el lugar en que est ubicado el caon y sean x y z los
a
n

ejes horizontal y vertical, respectivamente. La posicin de la bala (siendo t = 0 el
o
instante del disparo) vendr dada por las coordenadas
a
y

x(t) = v0 cos 0 t


CAP
ITULO 3. CINEMATICA EN DOS Y TRES DIMENSIONES

133

1
z(t) = v0 sin 0 t gt2 .
2
La componente vertical de la velocidad de la bala ser
a
vz (t) = v0 sin 0 gt .
Sea t el instante en que la bala penetra por la ventana. En ese instante deben
cumplirse las relaciones
v0 cos 0 t = D
y
1
v0 sin 0 t gt2 = h .
2
La condicin de que la bala penetre en forma horizontal por la ventana exige que
o
en t la velocidad vertical de la bala sea nula. O sea, adems de las dos relaciones
a
anteriores, debe cumplirse que
v0 sin 0 gt = 0 .
Despejando t de la ultima relacin y reemplazndola en las dos anteriores se obtiene

o
a
2
v0 sin 0 cos 0 = Dg

2
v0 sin2 0 = 2hg .

(1)
(2)

Dividindo la ultima por la antepenltima se encuentra


e

u
tan 0 =

2h
.
D

Esta relacin permite encontrar el ngulo de elevacin del disparo 0 . Para determinar
o
a
o
el valor de v0 elevamos al cuadrado la ecuacin (1):
o
4
v0 sin2 0 (1 sin2 0 ) = D 2 g 2 .

Despejando sin2 0 de (2), sustituyndolo en la ultima ecuacin se encuentra para v0


e

o
la expresin
o
(D 2 + 4h2 )g
2
.
v0 =
2h
Solucin al problema 30.
o
Sea x-y el plano en que se mueven los proyectiles, z el eje que apunta hacia arriba

y coloquemos el origen en el lugar en que se encuentra el caon.


n


CAP
ITULO 3. CINEMATICA EN DOS Y TRES DIMENSIONES

134

Sea t el tiempo transcurrido desde el disparo de la bala # 1. La posicin de esa


o
bala viene dada por
z1 (t) = v0 sin 1 t 1 gt2
2
x1 (t) = v0 cos 1 t .
Sea t el tiempo transcurrido desde el disparo de la bala # 2. La posicin de la segunda
o
bala viene, anlogamente, dada por
a
z2 (t ) = v0 sin 2 t 1 gt 2
2
x2 (t ) = v0 cos 2 t .
Para que las balas choquen deben coincidir las dos coordenadas de ambas balas, o
sea, debe cumplirse
cos1 t = cos2 t
(3.3)
y

1
1
(3.4)
v0 sin 1 t gt2 = v0 sin 2 t gt2 .
2
2
Despejando t de la primera de estas ecuaciones y reemplazndola en la segunda se
a
obtiene
1
cos 1
1 cos2 1 2
v0 sin 1 t gt2 = v0 sin 2
t g
t .
2
cos 2
2 cos2 2

Luego dividimos por t, multiplicamos por cos 2 y reordenamos los trminos:


e
v0 (cos 2 sin 1 sin 2 cos 1 ) =

gt
(cos2 2 cos2 1 ) .
2 cos 2

(3.5)

Sea t el tiempo entre ambos disparos. Se tiene entonces que t = tt. Sustituyendo
esto en (5.14) se encuentra que
t=

cos 2
cos 2 cos 1

t .

Sustituyendo esta relacin a su vez en (5.17), se obtiene:


o
v0 (cos 2 sin 1 sin 2 cos 1 ) =
o sea,
t =

Solucin al problema 33.


o

g (cos2 2 cos2 1 )
t ,
2 cos 2 cos 1

2v0 sin(1 2 )
11 s .
g cos 1 + cos 2

(3.6)


CAP
ITULO 3. CINEMATICA EN DOS Y TRES DIMENSIONES

135

Sea r (t) = x(t) x + y(t) y la posicin de la part

o
cula. Derivando respecto al tiempo se
encuentra su velocidad:
v(t) = x(t) x + y(t) y .

Por otra parte, de acuerdo al enunciado, sabemos que
v(t) = a x + bx(t) y .

Igualando ambas expresiones, componente a componente, obtenemos


x(t) = a

y
y(t) = bx(t) .

La primera de estas expresiones indica que, para la componente a lo largo del eje x,

el movimiento es uniforme, o sea,


x(t) = x(0) + at .
Pero, de acuerdo al enunciado, x(0) = 0, luego x(t) = at. Sustituyendo esto en la
ecuacin para y(t) se encuentra
o

y(t) = bat .

De aqu se deduce que el movimiento a lo largo del eje y es uniformemente acelerado,

luego
1
1
y(t) = y(0) + bat2 = bat2 .
2
2
De esta manera hemos encontrado que las coordenadas x e y de la part
cula, en
funcin del tiempo, vienen dadas por
o
x = at
ab 2
t .
2
Despejando t de la primera de estas ecuaciones y reemplazndolo en la segunda, se
a
obtiene nalmente la ecuacin de la trayectoria
o
y=

y = y(x) =

b 2
x .
2a


CAP
ITULO 3. CINEMATICA EN DOS Y TRES DIMENSIONES

136

Solucin al problema 36.


o
a) Sea v0 la velocidad del globo respecto a un observador jo en la Tierra. La velocidad
vertical y horizontal sern
a

v0 3

vz = v0 cos 30 =
2
y

v0
,
2
respectivamente. La componente horizontal de la velocidad del globo debe coincidir
con la del viento, o sea, vx = v0 /2 = vv . De aqu se deduce que v0 = 2vv = 20 km/h.

La componente vertical de la velocidad del globo es precisamente la velocidad


con que ste se mueve respecto al aire (su movimiento horizontal se debe al viento).
e

Esta velocidad vertical viene dada por vz = v0 3/2 = 17, 3... km/h.
vx = v0 sin 30 =

b) Conociendo vz es fcil evaluar el tiempo t que demora el globo en alcanzar una


a
altura de h = 1 km. Este viene dado por
t =

h
1

[h] 3, 46 [minutos] .
vz
17, 3

Solucin al problema 37.


o
Sea 0 la velocidad angular de la rueda antes de ser frenada: 0 = 2 s1 . Sea la
aceleracin angular que sufre la rueda al ser frenada. Si t = 0 es el instante en que se
o
aplica el freno, se tiene que la velocidad angular vendr dada por
a
(t) = 0 + t ,
mientras que el ngulo que rotar la rueda ser
a
a
a
1
1
(t) = (0) + 0 t + t2 = 0 t + t2 .
2
2
Sea t el tiempo que tarda la rueda en quedar en reposo. De acuerdo al enunciado del
problema, debe cumplirse que (t ) = 0 y (t ) = , o sea,
1
= 0 t + t 2
2

0 + t = 0 .

De estas ecuaciones podemos despejar t y . En particular para la aceleracin angular


o
se obtiene
2
= 0 = 2 [s2 ] .
2


CAP
ITULO 3. CINEMATICA EN DOS Y TRES DIMENSIONES

137

La magnitud de la aceleracin tangencial y centr


o
peta (ver seccin 3.3) vienen dadas
o
2 . Usando estas expresiones con R = 0, 25 [m] y = =
por at = R y ac = R
0
2 s1 se encuentra que la aceleraciones tangencial y centr
peta de un punto jo en
el borde de la rueda, cuando sta comienza a ser frenada, son at = 1, 57 [m/s2 ] y
e
2 ].
ac = 9, 87 [m/s
Solucin al problema 41.
o
a) Lo que A tarda en llegar hasta el suelo es igual a lo que demora B desde su punto
mximo (ambos ah tienen una velocidad vertical nula). B demora lo mismo en subir
a

que en bajar, luego la razn entre los tiempos de vuelo de A y B es


o
tA
1
= .
tB
2
b) La velocidad horizontal de ambos proyectiles es constante. Ambos recorren la
misma distancia horizontal y como B para ello demora el doble que A, se deduce que
la velocidad horizontal de B debe ser la mitad de la de A.
c) La velocidad vertical con que A B llegan al suelo es la misma (la de una ca
y
da
da
libre de una altura h). Esta es vv = 2gh. El tiempo de ca de A es t = (2h/g).
En ese tiempo A avanza en direccin horizontal una distancia horizontal L. Como la
o
velocidad horizontal es uniforme se deduce que sta (para la part
e
cula A) debe ser
vh = L/t = L g/(2h). La rapidez de A cuando llega al suelo es, por lo tanto,
|vA (t )| =

2
2
vv + vh =

2gh +

L2 g
.
2h

Para la part
cula B la componente vertical de la velocidad es la misma, mientras que
la componente horizontal es la mitad de la de A, o sea,
|vB (t )| =

2
vv + (vh /2)2 =

2gh +

L2 g
.
8h

Cap
tulo 4

Las leyes de Newton


versin 28 mayo 2012
o

Hasta el momento, slo nos hemos preocupado de describir el movimiento. Ahora


o
que tenemos las herramientas adecuadas para ello, podemos ocuparnos del problema
siguiente, que es estudiar las causas del movimiento, es decir, estudiaremos la dinmia
ca del movimiento.
En principio, es sencillo: basta con enunciar las as llamadas tres Leyes de Newton.

Sin embargo, en realidad no es tan simple, y de hecho dedicaremos este cap


tulo, y
prcticamente todo el resto de este curso, a analizar sus consecuencias. El resultado,
a
es decir, la descripcin de la Naturaleza basada en las leyes de Newton, se denomina
o
Mecnica Clsica. En estricto rigor, es posible hacer ciertas objeciones desde el punto
a
a
de vista lgico a estudiar la mecnica partiendo de las leyes de Newton. Existen
o
a
alternativas a esta formulacin de la mecnica clsica, pero son ms abstractas y por
o
a
a
a
ende no tan adecuadas para un primer curso de Mecnica. Adems, esta forma de
a
a
proceder corresponde mejor al desarrollo histrico de la F
o
sica
Pero antes incluso de enunciar las famosas leyes de Newton, debemos discutir
algunos conceptos preliminares.

4.1.

Espacio y tiempo

En la Mecnica Newtoniana se supone que las part


a
culas, como tambin los obe
servadores, viven en un espacio euclidiano tridimensional. Que sea tridimensional
no es una suposicin realmente sorprendente, ya que nuestra experiencia avala que,
o
para determinar unicamente la posicin de una part

o
cula, necesitamos tres coordenadas. Por su parte, que sea euclidiano signica que se cumplen los cinco postulados de

Euclides. En particular, el quinto postulado. Este indica que por un punto externo

138

CAP
ITULO 4. LAS LEYES DE NEWTON

139

a una recta pasa slo una recta paralela a la recta original.1 Estos cinco postulados
o
determinan completamente lo que se denomina Geometr Euclidiana, que es la que
a
estudiaron los griegos naturalmente, y que nos parece completamente natural en la
vida cotidiana. Por ejemplo, a partir de estos postulados (y en particular el quinto!)
es posible demostrar que la suma de los ngulos interiores de cualquier tringulo es
a
a
siempre 180 . Otra caracter
stica de un espacio euclidiano es, por ejemplo, que la
suma de dos vectores de desplazamiento es conmutativa.
Es interesante notar que es posible considerar tambin espacios no euclidianos. Un ejemplo
e
es la supercie de una esfera. Sobre una esfera es imposible trazar una recta, en el sentido
tradicional. Podemos salvar este punto notando que una recta, en el espacio euclidiano, es
el camino de menor longitud entre dos puntos dados. Si usamos ese hecho para denir una
recta, entonces s podemos trazar rectas sobre una esfera: el camino de menor distancia

entre dos puntos dados corresponde en tal caso a los meridianos (tcnicamente, para una
e
supercie cualquiera se dice que el camino de menor longitud entre dos puntos dados es
una geodsica). Y ahora nos preguntamos: es posible trazar, por un punto externo a un
e
meridiano dado, otro meridiano paralelo (es decir, que no se cruce con el otro meridiano en
ningn punto)? La respuesta es no, y sabemos por qu: los dos meridianos se encontrarn
u
e
a
invariablemente en los polos. Vemos que, sobre una esfera, el quinto postulado de Euclides
no se cumple: no es posible trazar ninguna recta paralela a una recta dada. A pesar de
ello, es posible construir una geometr completamente consistente desde el punto de vista
a
matemtico, slo que el espacio resultante es no euclidiano. Este caso fue estudiado por
a
o
Riemann. En un espacio de Riemann se presentan varias situaciones curiosas. Por ejemplo,
al viajar en l
nea recta en ese espacio, en algn instante uno vuelve al punto de partida.
u
Adems, la suma de los ngulos interiores de un tringulo dibujado sobre tal esfera es
a
a
a
mayor a 180o y tambin la suma de dos vectores es no conmutativa.
e
Tambin es posible violar el quinto postulado de Euclides en el sentido opuesto, es decir,
e
proponer un espacio en el cual es posible trazar innitas rectas paralelas a una recta dada.
Esto da origen a la geometr de Lobachevsky, y corresponde a una supercie tipo silla de
a
montar.

El espacio que Newton usa para desarrollar la mecnica no slo es euclidiano sino
a
o
que tambin homogneo e istropo. Esto signica que todos los lugares del espacio son
e
e
o
equivalentes y que el espacio tiene las mismas propiedades en todas las direcciones.
Para desarrollar la mecnica tambin es indispensable decir algo sobre el concepa
e
to de tiempo. Newton us la suposicin de que: El tiempo matemtico, absoluto y
o
o
a
verdadero uye, debido a su propia naturaleza, parejamente y en forma independiente
a cualquier agente externo. Esta suposicin tambin nos parece completamente nao
e
tural, avalada por nuestra experiencia: el hecho de que el tiempo avanza homognea
e
y continuamente, independiente de la posicin de un observador, de su velocidad, de
o
cualquier cosa.
Ahora bien, notemos que Newton no nos entrega precisamente una nocin de
o
tiempo. De hecho, nos dice que el tiempo uye parejamente, pero sin conocer de
1
En realidad, Euclides formul el quinto postulado de otro modo, pero sta es la formulacin
o
e
o
quizs ms famosa, debida a Ptolomeo.
a
a

CAP
ITULO 4. LAS LEYES DE NEWTON

140

antemano lo que es el tiempo, la palabra parejamente no tiene sentido. De modo


que, desde el punto de vista estrictamente lgico, la concepcin del tiempo anterior es
o
o
insatisfactoria. Pero, de todos modos, no es fcil decir algo sobre la nocin tiempo que
a
o
sea mejor o que clarique lo expresado por Newton, consecuentemente, no intentaremos hacerlo aqu Ms bien apelaremos a nuestra intuicin y experiencia cotidiana
. a
o
sobre lo que entendemos como tiempo.
Hay muchos modos de vericar que la nocin Newtoniana del tiempo como algo
o
independiente de todos los observadores es correcta o por lo menos plausible. Si
dos personas se encuentran y sincronizan sus relojes en un momento dado, sabemos
que sus relojes permanecern sincronizados siempre, independiente de lo que hagan.
a
En eso se basan todas nuestras actividades. Si decimos que una clase comienza a
las 10 de la maana, un programa de televisin a las 9 de la noche, o quedamos de
n
o
encontrarnos con un amigo en 20 minutos ms, dichas armaciones tienen un carcter
a
a
absoluto para todos.
Este hecho, que relojes inicialmente sincronizados permanecen sincronizados, es
bsico en la mecnica newtoniana. Por ejemplo, el movimiento rotatorio de la Tierra
a
a
en torno al Sol se usa para denir la unidad de tiempo llamada ao; el movimiento
n
de la Tierra en torno a su propio eje puede usarse para denir da solar. Un pndulo,

e
o una masa colgada de un resorte, tambin puede usarse como reloj.
e
Supongamos que un observador O tiene numerosos relojes idnticos a su disposie
cin, que los ha sincronizado y que tales relojes no modican su ritmo si se los aleja,
o
cada uno de los dems. De esta manera el observador O puede tener en todos los
a
lugares del espacio relojes sincronizados con el que l posee. Para el observador O,
e
dos eventos que ocurren en lugares distintos, sern simultneos si los relojes ubicados
a
a
en los dos lugares marcan la misma hora al ocurrir los eventos. Una consecuencia
de la concepcin newtoniana del tiempo es que si dos eventos son simultneos para
o
a
un observador, tambin lo sern para todos los dems observadores. En la mecnica
e
a
a
a
newtoniana el concepto simultaneidad tiene una validez absoluta.
Como hemos dicho, todas stas son suposiciones. Razonables, pero suposiciones.
e
Validadas, en todo caso, por nuestra experiencia cotidiana. Es oportuno indicar, sin
embargo, que ms adelante (en futuros cursos) nos veremos forzados a abandonar
a
este concepto intuitivo del tiempo. Y no solamente del tiempo; en algn momento nos
u
veremos obligados a revisar muchos otros conceptos que ya cre
amos tener claramente
establecidos. La razn escencial es que nuestra experiencia cotidiana slo alcanza a
o
o
un conjunto limitado de fenmenos. Las leyes de Newton no son necesariamente (y
o
de hecho no lo son) aplicables a fenmenos que ocurren en el mundo microscpico
o
o
o en escalas cosmolgicas, fracasando estrepitosamente cuando los sistemas son muy
o
pequeos o las part
n
culas se desplazan a velocidades comparables a la velocidad de
la luz. No obstante, esto no invalida la necesidad de nuestro estudio: la mecnica
a
newtoniana es una buena descripcin para una gran diversidad de fenmenos, y sus
o
o
xitos (aunque ahora sepamos que estn restringidos a un cierto mbito de fenmenos)
e
a
a
o
son indiscutibles.

CAP
ITULO 4. LAS LEYES DE NEWTON

4.2.

141

Las leyes de Newton

Estos son los postulados fundamentales de la mecnica que Isaac Newton public en
a
o
su libro Principia Mathematica en 1687.
Primera ley:
Cada cuerpo material persiste en su estado de reposo o de movimiento
uniforme en l
nea recta, a menos que una fuerza, que acta sobre el cuerpo,
u
lo conmine a cambiar de estado.
Si miramos con cuidado, notamos que esta ley est escrita en trminos de un
a
e
concepto an no denido, la fuerza. Slo podemos apelar a nuestro conocimiento
u
o
intuitivo sobre qu es una fuerza: una fuerza es lo que hacemos, por ejemplo, al usar
e
nuestros msculos para empujar un objeto.
u
Estamos diciendo entonces que mientras no haya una fuerza sobre un objeeto,
ste va a seguir en reposo si est en reposo, o en movimiento rectil
e
a
neo uniforme si
est en movimiento. Es decir, si no hay fuerzas sobre un objeto ste tendr velocia
e
a
dad constante (nula o no). Este principio, tambin conocido como ley o principio de
e
inercia, ya hab sido enunciado por Galileo.
a
Ahora bien, consideremos la siguiente situacin: estamos dentro de un auto en
o
reposo, y observamos un rbol en la calle. Nadie empuja o tira del rbol, y ste no se
a
a
e
mueve. Eso es precisamente lo que dice la primera ley de Newton. Pero ahora el auto
acelera. Repentinamente, vemos que, a pesar de que nadie empuja al rbol, ste se
a
e
mueve. Es decir, en nuestro sistema de referencia, dentro de ese automvil, el rbol
o
a
acelera a pesar de que no hay fuerzas sobre l. Qu signica esto, entonces? Qu la
e
e
e
primera ley de Newton no es cierta? Efectivamente. La primera ley de Newton no se
cumple en sistemas de referencia acelerados (como nuestro auto cuando parte). Esto
no signica que un sistema de referencia acelerado sea malo; simplemente signica
que la primera ley de Newton no se cumple. Diremos que un sistema de referencia en
que se cumple la primera ley de Newton es un sistema de referencia inercial. Por lo
tanto, podemos considerar que la primera ley de Newton no es tanto una armacin
o
sobre lo que le ocurre a un cuerpo cuando aplico o no una fuerza, sino que, en realidad,
es la denicin de sistema de referencia inercial.
o
En general en F
sica, y lo haremos as en este curso, preferimos trabajar con

sistemas de referencias inerciales, porque la descripcin es ms sencilla. En nuestro


o
a
problema del rbol visto desde un auto acelerando, cualquier explicacin sobre las
a
o
causas del movimiento del rbol ser inveros
a
a
mil, pues de verdad no hay nada en
el Universo que haya intentado mover el rbol. Claro, si alguien ve esta situacin
a
o
desde la acera, va a ser evidente para ese observador adicional que, desde dentro del
auto, el rbol parece acelerar porque en realidad es el auto el que acelera, pero hacer
a
F
sica en el interior del auto signica trabajar slo sobre observaciones que se pueden
o

CAP
ITULO 4. LAS LEYES DE NEWTON

142

realizar desde el interior del auto. Es posible, sin embargo, hacer F


sica en sistemas
no inerciales, lo cual ser tema de un cap
a
tulo posterior en este mismo curso. No es
que los sistemas no inerciales no sirvan para hacer F
sica, sino simplemente que la
descripcin es ms complicada.
o
a
Para enunciar la segunda ley debemos denir previamente una cantidad f
sica
nueva: el concepto de cantidad de movimiento, momentum, momento o momento
lineal de una part
cula. El momentum de una part
cula es el producto de la masa de
la part
cula por su velocidad. Como el producto de un escalar (la masa) por un vector
(la velocidad), es un vector, el momentum de una part
cula es un vector:
p = mv .
Nuevamente estamos en dicultades con el lenguaje ac. La velocidad tiene un
a
signicado claro, pero qu es la masa? Y nuevamente tendremos que apelar a nuestra
e
intuicin cotidiana. La masa m de un cuerpo ser una magnitud que es proporcional
o
a
a su peso, es decir, dicho de alguna manera imprecisa, proporcional al esfuerzo que
es necesario realizar para levantarlo o suspenderlo. Si un cuerpo pesa ms que otro,
a
esto se debe a que el primero tiene una masa mayor que el segundo.
La unidad de masa en el sistema internacional de unidades SI es el kilgramo,
o
y corresponde a la masa del kilgramo patrn guardado en una ocina en Par Sin
o
o
s.
embargo, para la mayor de los efectos prcticos podemos denir a un kilgramo
a
a
o
como la cantidad de masa que posee un litro de agua dulce.
Con esta denicin, podemos enunciar enunciar la segunda ley de Newton.
o
Segunda ley:
El cambio de momentum p de una part
cula es proporcional a la fuerza
neta que acta sobre el cuerpo, como tambin al intervalo t durante el
u
e
cual ella se aplica, y apunta en la direccin y sentido de esta fuerza, o sea,
o
p = F t .

Ms adelante deniremos a la cantidad F t como el impulso, de modo que la


a
segunda ley de Newton se puede enunciar diciendo que el cambio de momentum es
igual al impulso.
Observamos de inmediato que esta ley slo es vlida si la fuerza F es constano
a
te durante el intervalo t y si las magnitudes son observadas desde un sistema de
referencia inercial.
Pero notemos tambin que en la expresin anterior, conocemos las deniciones de
e
o
tiempo, masa y velocidad, de modo que la segunda ley es, en realidad, la denicin
o
de fuerza.

CAP
ITULO 4. LAS LEYES DE NEWTON

143

Si una part
cula est sometida a una fuerza F durante un intervalo de tiempo
a
t, cambiar su velocidad y, por tanto, su momentum. De acuerdo a la segunda ley,
a
podemos denir la fuerza que acta sobre la part
u
cula haciendo el cuociente:
F =

p
,
t

donde con los parntesis indicamos que esto en realidad corresponde a la fuerza
e
media que siente la part
cula durante el tiempo t (de modo anlogo a los conceptos
a
de velocidad y aceleracin media en el Cap. 2).
o
La fuerza instantnea se obtiene en el l
a
mite t0, o sea, viene dada por
F

dp
.
dt

Esta ser nuestra denicin de fuerza. En todo momento podemos determinar expl
a
o
citamente el momentum de una part
cula como funcin del tiempo. Su derivada temo
poral es la fuerza. Notemos que es evidente que la fuerza es tambin una magnitud
e
vectorial.
Si la masa de una part
cula no var a medida que transcurre el tiempo, entonces
a
F =

dp
d( mv )
dv
=
=m
= ma .
dt
dt
dt

Es decir, la fuerza neta que acta sobre una part


u
cula es igual al producto de su masa y
su aceleracin. La anterior es ciertamente una formulacin popularmente conocida de
o
o
la segunda ley de Newton, pero enfatizamos que es slo vlida si la masa es constante.
o
a
Hay muchas situaciones en que la masa no es constante: por ejemplo, un cohete que
sube desde la supercie terrestre quemando combustible, un auto que va gastando
gasolina, etc. En tales casos, lo correcto es utilizar la expresin original, en trminos
o
e
del momentum.
En el sistema de unidades SI, si la masa se mide en kg y la aceleracin en m/s2 ,
o
entonces la fuerza viene dada en newtons (N):
1 N 1 kg 1

m
.
s2

Tercera ley:
Si un cuerpo A ejerce una fuerza sobre otro B, entonces este ultimo ejer
cer sobre A una fuerza de igual magnitud y en la misma direccin, pero
a
o
en sentido opuesto.

CAP
ITULO 4. LAS LEYES DE NEWTON

144

La tercera ley se conoce tambin como ley de accin y reaccin. De acuerdo a ella,
e
o
o
una fuerza nunca aparece en forma solitaria, sino que siempre vendr acompaada de
a
n
otras fuerzas, de manera que la suma vectorial de todas ellas sea nula. Por ejemplo,
la Tierra nos atrae hacia ella, y esto no nos genera ninguna complicacin conceptual,
o
pues la Tierra tiene mucho ms masa que nosotros. Pero la tercera ley de Newton
a
asegura que tambin es cierto que nosotros atraemos a la Tierra, con una fuerza igual
e
a nuestro peso. Estamos acostumbrados a atornillar tornillos en nuestras casas, pero
si hiciramos eso en ausencia de gravedad, al intentarlo el tornillo nos har girar a
e
a
nosotros en direccin contraria, con la misma fuerza que nosotros aplicamos sobre l,
o
e
pues no habr otras fuerzas sobre nosotros que lo impidieran.
a
Si no lo pensamos con cuidado, la tercera ley nos puede llevar a imaginar ciertas
paradojas. Por ejemplo, si un caballo intenta tirar una carreta, puesto que la carreta
ejerce la misma fuerza sobre el caballo, cmo es posible que el caballo mueva a la
o
carreta? No deber ser la suma de las fuerzas igual a cero? Bueno, la suma de esas
a
dos fuerzas es cero, pero es crucial notar que no actan sobre el mismo cuerpo, y
u
la primera ley de Newton asegura que un cuerpo comenzar a moverse si la fuerza
a
total sobre l es cero. En este caso, el caballo ejerce una fuerza sobre la carreta, y sta
e
e
sobre el caballo, de modo que sobre ambos, carreta y caballo, se ejerce una fuerza neta
distinta de cero, y por tanto es posible que se muevan. (El hecho de que en efecto se
muevan en la direccin que el caballo tira tiene que ver en realidad con otros efectos,
o
relacionados con el suelo; de hecho, si el suelo fuera muy resbaloso, el caballo tampoco
lograr mucho, pero dejaremos la discusin de estos efectos para ms adelante.)
a
o
a
A pesar de que no se menciona expl
citamente, al aplicar la tercera ley se supone que la
accin y reaccin aparecen en forma simultnea. Esto debe ser cierto independiente de la
o
o
a
distancia en que se encuentren. Por ejemplo, dos cuerpos pueden interactuar a distancia
a travs de la interaccin gravitacional, y los pares de fuerza de accin y reaccin deben
e
o
o
o
aparecen simultneamente tanto para una manzana cerca de la Tierra, el sistema Tierra-Sol,
a
entre galaxias, entre quasares. . . Por lo tanto, impl
citamente estamos aceptando que, en la
mecnica newtoniana, debe existir una manera de transmitir la informacin de un cuerpo
a
o
a otro con una velocidad innita. Ahora sabemos que en la naturaleza tales velocidades
innitas no existen; hoy en d sabemos que la velocidad de la luz en el vac es un l
a
o
mite
superior para las velocidades con que se puede trasladar algo material o informacin de un
o
lugar a otro. Por esta razn, la tercera ley es generalmente una muy buena aproximacin,
o
o
pero no tiene una validez universal; por ejemplo, en colisiones atmicas no es siempre
o
aplicable.

4.3.

Uso de las leyes de Newton

Para aprender a manejar las leyes de Newton y comprender su signicado, lo


mejor es ilustrar su uso en algunas situaciones concretas.

CAP
ITULO 4. LAS LEYES DE NEWTON

145

Ejemplos:
1. Analicemos las fuerzas que actan sobre un cuerpo que cae.
u
Debido a la atraccin gravitatoria, todo objeto sufrir una fuerza que apunta
o
a
hacia el centro de la Tierra. Es esta fuerza la que acelera al cuerpo durante su
ca
da.
Cul es el tamao de esta fuerza? Sabemos que al realizar experimentos con
a
n
cuerpos sobre la supercie terrestre, al soltarlos todos ellos caen con la misma
aceleracin hacia la supercie. Esta aceleracin constante, llamada aceleracin
o
o
o
de gravedad, se denota por g, y su valor es aproximadamente g = 9.81 m/s2 . (En
realidad, al realizar estos experimentos hay que asegurarse de que los efectos
de la densidad y viscosidad de la atmsfera sean despreciables. Ms an, el exo
a u
perimento debe realizarse sin alejarse demasiadoa lo ms unas pocas decenas
a
de kilmetrosde la supercie terrestre, adems de haber pequeas variacioo
a
n
nes con la latitud. Pero para todos los efectos prcticos, cerca de la supercie
a
terrestre la aceleracin de gravedad se puede considerar constante.)
o
Conociendo la aceleracin del cuerpo y su masa m podemos (usando la segunda
o
ley de Newton) establecer cul es la fuerza gravitacional que acta sobre el
a
u
cuerpo. Deniendo al vector unitario z como un vector que apunta hacia arriba,

el vector aceleracin del cuerpo vendr dado por a = g. La fuerza sobre el


o
a
z
cuerpo es entonces
F = m (g z ) = mg .

z
A la magnitud de esta fuerza gravitacional es lo que se llama peso del objeto.
Usando la letra W para denotar al peso se tiene
|F | W = m g = peso del objeto .
El peso de un objeto no es sino la fuerza que ejerce la Tierra sobre l. En el
e
lenguaje coloquial, es habitual decir que el peso se mide en kilogramos. Pero
vemos que el peso es una fuerza, y por tanto se mide en newtons; es la masa,
una cantidad escalar, la que se mide en kilogramos. Ambos conceptos pueden
considerarse intercambiables, sin embargo, mientras estemos en la Tierra, pues
masa y peso dieren en un factor constante para todos los cuerpos, igual a g.
Pero en rigor, siempre debemos hacer la distincin entre masa y peso.
o
2. Analicemos las fuerzas que actan sobre un libro de masa M , en reposo sobre
u
una mesa (supercie horizontal).
Ya sabemos que sobre el libro acta una fuerza, debido a la gravedad terrestre,
u
que es
W = M g .
z

CAP
ITULO 4. LAS LEYES DE NEWTON

146

Por otra parte, debido a que el libro se encuentra (y se mantiene) en reposo, la


fuerza neta sobre el libro debe ser nula. Quin o qu ejerce otra fuerza, igual a
e
e
W , sobre el libro? La respuesta es: la mesa. Efectivamente, el libro se apoya
sobre la mesa y la supercie de ella ejerce sobre el libro una fuerza hacia arriba,
R, cuya magnitud es igual al peso del libro, y de sentido opuesto, R = W .

A su vez, como R es la fuerza ejercida por la mesa sobre el libro, ste debe
e
ejercer una fuerza de igual magnitud y sentido opuesto, S1 = R, sobre la mesa.
Y tambin es cierto que el libro debe ejercer una fuerza de magnitud igual a
e
S2 = W sobre la Tierra. Pero aunque S1 y S2 existen en este problema, no
actan sobre el libro, y por tanto son completamente ignorables cuando se trata
u
de estudiar el comportamiento del libro. Ms an, el hecho mismo de que el libro
a u
sea atraido por la Tierra y de que est sobre la mesa es irrelevante, ya que lo
e
unico que le importa al libro es que sobre l actan fuerzas W y R. Toda la

e
u
inuencia de la Tierra y la mesa se reduce a la existencia de dichas fuerzas sobre
el libro. Esto sugiere que es posible estudiar al libro aislado de todos los otros
cuerpos del Universo, como si stos no existieran, y reemplazando aquellos que
e
tienen efecto sobre l por las fuerzas respectivas:
e
Al analizar las fuerzas que se ejercen sobre un cuerpo es conveniente
aislarlo del resto de los objetos que interactan con l. Para ello cada
u
e
objeto que interacta con este cuerpo es sustituido por una fuerza que
u
cumple con la tercera ley de Newton. El resultado de esta operacin
o
es el as llamado diagrama de cuerpo libre del objeto.

Para el caso del libro, la interaccin de


o
ste con la tierra se reemplaza por el
e
vector W que apunta hacia abajo y cuya magnitud coincide con el peso del
libro; el efecto de la mesa sobre el libro se reemplaza por el vector R (ver
gura 4.1). Si el libro se mantiene en
reposo, la segunda ley de Newton requiere que W + R = 0.

R
111111111111111111
000000000000000000
111111111111111111
000000000000000000
111111111111111111
000000000000000000
111111111111111111
000000000000000000

Figura 4.1

A la fuerza R se le denomina fuerza normal, pues es perpendicular a la supercie


sobre la cual est apoyado el objeto. La fuerza normal es la responsable de que
a
el cuerpo no atraviese la supercie. En el caso sencillo analizado recin, la
e
fuerza normal R = W . Un error usual es decir que la fuerza normal es la
reaccin al peso. Es el mismo vector que la fuerza de reaccin al peso, pero en
o
o
realidad son fuerzas distintas, pues la normal es una fuerza que ejerce la mesa
sobre el libro, y la reaccin al peso es la fuerza que ejerce el libro sobre la Tierra.
o

CAP
ITULO 4. LAS LEYES DE NEWTON

147

Y adems, la normal tiene igual magnitud que el peso slo en este caso, en que
a
o
la supercie de apoyo es horizontal.
3. Consideremos un objeto de masa m que cuelga del techo sujetado por una
cuerda ideal (ver gura 4.2). Cul es la fuerza que la cuerda ejerce sobre el
a
gancho en el techo y cul es la tensin de la cuerda?
a
o
Una cuerda ideal es una cuerda que, a
menos que se especique lo contrario,
no tiene masa, es perfectamente exible
y no es extensible. Que una cuerda sea
perfectamente exible quiere decir que
slo es capaz de transmitir una fuerza a
o
lo largo de ella; no puede ejercer fuerzas
transversales.

111111111111
000000000000
111111111111
000000000000
111111111111
000000000000
111111111111
000000000000
111111111111
000000000000

111111111111
000000000000
111111111111
000000000000
111111111111
000000000000
111111111111
000000000000
111111111111
000000000000

F
2

F2
F1

1111
0000
1111
0000
1111
0000

111
000
111
000
111
000

F
1

Figura 4.2

Sobre el objeto actan dos fuerzas; una es el peso W = mg y la otra es


u
z
la fuerza F1 ejercida por la cuerda. Como el objeto no acelera, la fuerza neta
(es decir, la suma de todas las fuerzas que actan sobre l) debe ser nula. Por
u
e
consiguiente, F1 = W .

Debido al principio de accin y reaccin, debe haber una fuerza ejercida por
o
o

el objeto sobre la cuerda igual a F1 = F1 . Sobre la cuerda tambin hay una


e

fuerza ejercida por el gancho, F2 .


Nuevamente, debido al principio de accin y reaccin, la cuerda debe ejercer
o
o

sobre el gancho una fuerza igual a F2 = F2 .

Ahora, debido a que la cuerda no tiene masa, las unicas fuerzas que actan

sobre ella sern F1 y F2 . Al estar en equilibrio (la cuerda no acelera), la suma


a

de ambas fuerzas debe ser cero, luego F2 = F1 . Resumiendo, tenemos que

mg = W = F1 = F1 = F2 = F2 ,
z

o sea, la fuerza F2 que la cuerda ejerce sobre el gancho es igual al peso mg.
z
Cada uno de los extremos de la cuerda ejerce una fuerza sobre los objetos a
los cuales est unida. Cuando la masa de la cuerda es nula, la magnitud de esa
a
fuerza es la misma. A esta magnitud se le llama tensin de la cuerda. A lo largo
o
de una cuerda ideal, que no tiene masa, la tensin no var Para la cuerda del
o
a.
presente problema, la tensin es = mg. La tensin es un escalar.
o
o
A primera vista, uno podr haber adivinado que si colgamos del techo una
a
masa, la fuerza que se va a ejercer sobre el gancho del que todo cuelga ser igual
a
al peso del objeto suspendido. Pero notemos la cadena de implicaciones que han
conducido a este resultado, en particular la importancia de que el sistema est en
e
reposo.

CAP
ITULO 4. LAS LEYES DE NEWTON

148

4. Mquina de Atwood.
a
Consideremos dos masas m1 y m2 unidas por una cuerda ideal sin masa que
pasa sobre una polea ideal (ver gura 4.3). Deseamos encontrar la aceleracin
o
de las masas y las tensiones de las cuerdas.
Con la expresin polea ideal nos estamos
o
reriendo a una polea que no tiene masa y gira sin roce. El objetivo de la polea
es simplemente cambiar la direccin de la
o
cuerda y, por lo tanto, de la fuerza (que
acta siempre a lo largo de la cuerda). La
u
tensin a la que est sometida una cuero
a
da no se modica al pasar por una polea
ideal.
Sea la tensin de la cuerda que une amo
bas masas y a1 = a1 z la aceleracin que

o
sufrir la masa 1. La fuerza neta que acta
a
u
sobre la masa 1 es (m1 g + ), luego, de
z
acuerdo a la segunda ley de Newton

1111111111111111111
0000000000000000000
1111111111111111111
0000000000000000000
1111111111111111111
0000000000000000000
1111111111111111111
0000000000000000000

(m1 g + ) = m1 a1 = m1 a1 z .
z

^
z
m2
m1
Figura 4.3

De esta relacin se deduce que


o
m1 g = m1 a1 .

(4.1)

Anlogamente, aplicando la segunda ley de Newton a la segunda masa se obtiene


a
la expresin
o
m2 g = m2 a2 .
(4.2)
Las ecuaciones (4.1) y (4.2) contienen toda la informacin que se puede extraer
o
a partir de las fuerzas. Sin embargo, no permiten encontrar la aceleracin de
o
cada masa, pues son dos ecuaciones para las tres incgnitas: a1 , a2 y . De
o
dnde se puede extraer una ecuacin adicional? La respuesta es t
o
o
pica de este
tipo de problemas, con poleas: la tercera ecuacin es una condicin geomtrica,
o
o
e
no dinmica, que se deduce del hecho de que la cuerda es inextensible. En efecto,
a
como el largo de la cuerda es constante, debe tenerse que el desplazamiento de
la masa 1 en un tiempo dado debe ser igual en mdulo y de signo opuesto que
o
el desplazamiento de la masa 2 en el mismo tiempo; es decir, lo que sube (baja)
m1 , debe bajarlo (subirlo) m2 :
z1 = z2 .

CAP
ITULO 4. LAS LEYES DE NEWTON

149

Dividiendo por el tiempo t durante el cual ha ocurrido este desplazamiento, y


tomando el l
mite t 0, esta expresin queda z1 = z2 . Derivando una vez
o

ms respecto al tiempo queda z1 = 2 , es decir,


a

z
a1 = a2 .

Esta es la tercera ecuacin que necesitamos para resolver el sistema.


o
Sea entonces
a1 = a2 = a0 z .

De las ecuaciones (4.1) y (4.2) podemos despejar las dos incgnitas a0 y :


o
=2

m1 m2
g
m1 + m2

m1 m2
g.
m1 + m2
Como la polea no tiene masa y sta no sufre aceleraciones, la tensin de la
e
o
cuerda que la sujeta deber ser igual a 2 .
a
a0 =

Casos particulares:
Si m1 = m2 , entonces a0 = 0 y = m1 g = m2 g. Tal como era de esperarse, si
las masas son iguales, ninguna de ellas acelera.
Si m1 > m2 entonces a0 resulta ser una magnitud negativa. Esto quiere decir
que a1 = a0 z es una aceleracin que apunta hacia abajo; tal como debe ser, la

o
masa 1 baja, mientras que la masa 2 sube.
Si m1 es muy parecida a m2 , entonces |a0 | g. O sea, cada una de las masas
realizar un movimiento uniformemente acelerado, pero con una aceleracin
a
o
mucho menor que g.
Si m1 = 0, entonces a0 = g y = 0. En este caso la cuerda deja de tener tensin,
o
y por consiguiente la part
cula 2 caer con aceleracin g.
a
o
El mtodo expuesto es muy general, permitiendo abordar una gran variedad
e
de problemas con poleas y cuerdas. Es interesante notar, sin embargo, que el
sistema analizado es particularmente sencillo, y existen otros modos de llegar a
la misma solucin. En efecto, notemos que, para todo efecto prctico, la cuerda
o
a
tiene slo el efecto de mantener unidas las dos masas, y que stas pueden ser
o
e
consideradas como un solo cuerpo. Si ponemos este sistema de dos masas unidas
por una cuerda en el suelo, horizontalment, podemos pensar que est sometido
a
a dos fuerzas: los pesos de cada masa, uno hacia la derecha, el otro hacia la
izquierda. Por lo tanto, uno podr escribir la fuerza neta sobre el sistema:
a
F = m1 g + m2 g ,
x
x

CAP
ITULO 4. LAS LEYES DE NEWTON

150

lo que debe ser igual, por la segunda ley de Newton, a:


F = (m1 + m2 )a0 x .

Despejando a0 de estas ecuaciones, se recupera el resultado ya obtenido. Este


procedimiento tan sencillo, sin embargo, es posible porque el problema es en
s sencillo. Para conguraciones ms complicadas, es mejor proceder del modo

a
general anteriormente expuesto, escribiendo cuidadosamente las fuerzas sobre
cada masa, y cualquier condicin geomtrica adicional sobre los desplazamientos
o
e
de las mismas.
5. Considere una cuerda exible de masa M
que cuelga entre dos paredes, siendo el
a
ngulo que forma la cuerda con la pared
(ver gura 4.4). Se desea encontrar la tensin que la cuerda tiene en el punto m
o
nimo. (En este caso, a diferencia de los anteriores, la tensin no es constante, precio
samente porque la cuerda tiene masa.)

^
z

F2
^
x

F1

Figura 4.4
Para resolver el problema consideremos como nuestro sistema slo la mitad
o
derecha de la cuerda. Este truco es en el mismo esp
ritu que el diagrama de
cuerpo libre. De hecho, es un diagrama de cuerpo libre, para la mitad derecha de
la cuerda. Estamos reemplazando, en el fondo, la mitad izquierda de la cuerda
por la fuerza que sta ejerce sobre la otra mitad, F1 . A la mitad derecha de
e
la cuerda le da lo mismo lo que haya al lado izquierdo, sea una cuerda, una
pared, una persona, o cualquier agente, mientras ejerza la misma fuerza F1 en
el mismo punto.
Hay tres fuerzas que actan sobre el sistema considerado:
u
z
i) El peso W = 1 M g.
2
ii) La fuerza F1 ejercida por la parte izquierda de la cuerda. La magnitud de
esta fuerza es igual a la tensin de la cuerda en el m
o
nimo, que llamaremos
0 . Se tiene que F1 = 0 x .

iii) La fuerza que ejerce el gancho sobre la cuerda. Como la cuerda es exible
la fuerza necesariamente es a lo largo de la tangente de la cuerda. Si a la
magnitud de esta fuerza la llamamos f0 , se tiene que F2 = f0 cos z +

f0 sin x.

CAP
ITULO 4. LAS LEYES DE NEWTON

151

Como nuestro sistema est en equilibrio (no acelera), la suma de las tres fuerzas
a
debe ser nula:
1
W + F1 + F2 = M g 0 x + f0 cos z + f0 sin x = 0 .
z

2
Pero para que un vector sea cero es necesario que cada una de sus componentes
sea nula. Este hecho nos da las siguientes ecuaciones:
componente z:

1
M g + f0 cos = 0
2

componente x:

0 + f0 sin = 0 .

y
De estas dos ecuaciones podemos despejar 0 y f0 , obtenindose
e
0 =

1
M g tan
2

y
f0 =

2
0 +

Mg
2

Notemos cmo para 90 , o sea, a medida que la cuerda se cuelga en forma


o
ms tirante, la tensin de la cuerda tiende a innito. Esto viene, matemtia
o
a
camente, del hecho de que si es muy parecido a /2, la fuerza ejercida por el
gancho tiene una componente vertical muy pequea. Como debe contrarrestar
n
a la unica otra fuerza vertical, el peso de la cuerda, se sigue que |F2 | = f0 debe

ser muy grande. En el l


mite = /2, f0 . Esto tiene importantes consecuencias prcticas. Los cables del tendido elctrico, entre postes, tienen una
a
e
cierta curvatura. Intentar colocarlos completamente horizontales ser un gran
a
riesgo, ya que estar sometidos a una tensin demasiado alta, y cualquier peran
o
turbacin podr romperlos. Si uno desea suspender un cable horizontalmente
o
a
una solucin posible es utilizar cuerdas auxiliares:
o

Solucin que, por cierto, se utiliza en puentes colgantes, por ejemplo.


o

CAP
ITULO 4. LAS LEYES DE NEWTON

152

6. Consideremos una masa m que gira en el


plano x, y, en un c
rculo de radio R y con
una velocidad angular constante, 0 . Encontremos la fuerza neta a la que est soa
metida la masa.
En la seccin 3.3 ya analizamos el moo
vimiento circular y demsotramos que la
aceleracin de la masa m viene dada por
o
2
a(t) = R0 r. De acuerdo a la tercera ley de Newton, el hecho que la masa
m est acelerada implica que sobre ella
e
est actuando una fuerza neta
a

Figura 4.5

2
F = ma = Rm0 r .

Esta fuerza (de magnitud constante) apunta hacia el origen y por esta razn se
o
le denomina fuerza centrpeta.

Debido a la importancia de este resultado lo reiteramos: Una masa m que realiza


un movimiento circular uniforme, est sometida a una fuerza que apunta hacia
a
el centro de giro. La magnitud de esta fuerza centr
peta es
mv 2
,
R
donde R es el radio del c
rculo, 0 la velocidad angular y v = 0 R el mdulo
o
de la velocidad de la part
cula.
2
Fcent = mR0 =

Problema resuelto en clases: 4.6

4.4.

Roce cintico y esttico


e
a

Si un cuerpo se desliza sobre otro, tarde o temprano se detendr a menos que


a
exista una fuerza externa que perpete el movimiento. La fuerza que se opone al
u
deslizamiento relativo entre los dos cuerpos se denomina fuerza de roce cintico. Se
e
origina en la interaccin de ambas supercies en contacto.
o
La fuerza de roce no slo aparece cuando dos cuerpos estn en movimiento relao
a
tivo, sino que tambin puede estar presente cuando los dos cuerpos se encuentran en
e
reposo relativo. En efecto, si, por ejemplo, intentamos deslizar una mesa por el piso,
notamos que aparece una fuerza que impide que este deslizamiento comience. A esta
fuerza se le denomina fuerza de roce esttico.
a

CAP
ITULO 4. LAS LEYES DE NEWTON

153

Tambin existen otras fuerzas de roce que aparecen en diversas circunstancias


e
(por ejemplo, el roce rodante, el roce viscoso, etc), sin embargo, en el presente cap
tulo
centraremos nuestro inters en las fuerzas de roce cintico y esttico.
e
e
a
Se sabe relativamente poco acerca de ambos y es dif cuanticarlos porque
cil
dependen de la naturaleza de los materiales y de propiedades de la supercie como
el pulido, la existencia de xidos en la interfase, etc. Lo que diculta an ms la
o
u
a
cuanticacin de la fuerza de roce es su dependencia de la historia de las supercies:
o
el paso del roce esttico al roce dinmico depende de si las supercies se han deslizado
a
a
previamente o no.
Las fuerzas de roce tienen un origen microscpico. Dos supercies, por suaves que
o
parezcan, a nivel microscpico tienen irregularidades. Estas protuberancias forman,
o
en algunos casos, microsoldaduras, y son el origen de la fuerza adicional que uno
debe aplicar para poder iniciar un movimiento relativo entre los cuerpos. Una vez
que stos estn en movimiento, estas aristas microscpicas se enganchan unas con
e
a
o
otras y dan origen al roce cintico (tambin a veces llamado roce cinemtico o roce
e
e
a
dinmico). Por otra parte, cuando dos supercies estn en reposo relativo, aparecen
a
a
fuerzas entre las molculas de cada supercie, y estas fuerzas son las que hay que
e
vencer para comenzar a desplazar entre s las supercies, dando origen as al roce

esttico.
a
Lo cierto es que no hay una teor completa, de primeros principios, para el roce.
a
Lo cual causa problemas desde el punto de vista terico, porque es un efecto que
o
evidentemente est presente, pero que es dif de modelar en general. En todo caso,
a
cil
numerosos estudios experimentales permiten extraer algunas conclusiones generales
sobre el roce. No se trata de leyes fundamentales de la naturaleza, sino de resultados
fenomenolgicos y cualitativos sobre el roce, sucientes para introducir los efectos del
o
roce en las ecuaciones dinmicas.
a
Consideremos un bloque de masa M que descansa sobre una supercie, el cual
intentamos deslizar aplicando sobre l una fuerza horizontal F , que incrementamos
e
paulatinamente. Designemos por f a la fuerza de roce que aparece debido a la friccin
o
entre las dos supercies y describamos la forma en que t
picamente var esta fuerza.
a
a) Mientras la fuerza horizontal externa F = |F | var desde 0 hasta un cierto
a
(max)
valor fe
, el bloque M no se deplazar. Como no hay aceleracin, la fuerza
a
o
neta horizontal sobre el cuerpo debe ser nula, o sea, debe haber otra fuerza

horizontal sobre el bloque que exactamente cancele a la fuerza F . Esta es la


fuerza de roce esttica f . Se tiene, por lo tanto, que f = F .
a
(max)

b) Cuando la fuerza horizontal externa F sobrepasa cierto valor fe


, la fuerza de
roce no sigue aumentando. Como ahora la componente horizontal de la fuerza
neta no es nula, el bloque comenzar a acelerar. Tan pronto como los cuerpos
a
se deslizan con cierta velocidad relativa, la fuerza de roce se vuelve constante,

CAP
ITULO 4. LAS LEYES DE NEWTON

154
(max)

siendo su magnitud algn valor fc (menor que fe


u
movimiento relativo.

) y su sentido opuesto al

De ah en adelante, si se desea mantener el bloque deslizndose con una veloci


a
dad constante, debe aplicarse una fuerza horizontal de exactamente la magnitud
fc , en la direccin de movimiento.
o
Es decir, el resultado experimental es que, para mover, por ejemplo, un mueble
sobre el piso, cuesta un poco ms comenzar a moverlo que mantenerlo en movia
miento.
Este comportamiento fenomenolgio
co recin descrito, que muestra la
e
fuerza de roce, se muestra en la gura 4.6. Emp
ricamente se ha observado que, para dos supercies (secas) en contacto, tanto la fuerza de
friccin dinmica fc como el mxio
a
a
(max)
mo de la friccin esttica fe
o
a
, son
proporcionales a la fuerza normal
entre ambas supercies, o sea,
fc = c FN
y
(max)
fe
= e FN .

Figura 4.6
FN es la fuerza normal entre las supercies (es decir, perpendicular a la interfase
formada por las dos supercies) y c y e son los coecientes de friccin o de roce. Los
o
coecientes de friccin de alguna manera engloban nuestra ignorancia de los distintos
o
parmetros que intervienen en el problema. Siempre se tiene que el coeciente de
a
roce cinemtico es menor al coeciente de roce dinmico: c < e . Ambas fuerzas de
a
a
roce actan en la direccin paralela a las supercies. El sentido de la fuerza de roce
u
o
esttico es opuesto a la fuerza horizontal neta que acta sobre el cuerpo, mientras que
a
u
el sentido de la fuerza de roce dinmico es siempre opuesto al movimiento relativo (y
a
no a la fuerza) entre las dos supercies.
Ilustremos los conceptos anteriores con un ejemplo.
Problema:
Considere el montaje experimental mostrado en la gura 4.7. Supongamos que
los coecientes de friccin esttico y cinemtico entre la masa M =4 Kg y el plano
o
a
a

CAP
ITULO 4. LAS LEYES DE NEWTON
inclinado son e = 0.4 y c = 0.3, respectivamente.

155

CAP
ITULO 4. LAS LEYES DE NEWTON

156

Qu rango de valores puede tener m para


e
que el sistema se encuentre en equilibrio
esttico? Si la masa m justo sobrepasa ese
a
mximo, con qu aceleracin se mover el
a
e
o
a
bloque sobre el plano?

Solucin:
o

Figura 4.7
Resolvamos primero el problema esttico.
a
Lo primero que hay que tener claro es que el sistema puede estar en equilibrio para
un rango de valores de m. En efecto: si no hay roce, es claro que hay un unico valor

de m (digamos m0 ), tal que el sistema se mantiene en equilibrio. Si m es ligeramente


mayor que m0 , entonces el sistema intentar moverse hacia la derecha en la Fig. 4.7.
a
Pero como hay roce, dicho movimiento no ocurrir, a menos que m sea sucientemente
a
grande para que la fuerza aplicada sobre el otro bloque sea mayor que la fuerza de
roce esttico mxima. Entonces, existe un valor mximo de m tal que el sistema
a
a
a
puede mantenerse en equilibrio. Por argumentos similares, podemos convencernos de
que exsite tambin un valor mnimo de m. El problema ahora es encontrar dichos
e

valores.
La gura 4.8 muestra el diagrama de cuerpo libre del bloque que se encuentra sobre
el plano inclinado. A priori no sabemos en
qu sentido apunta la fuerza de roce fr . La
e
hemos dibujado apuntando a lo largo del
plano hacia abajo; si despus de realizar
e
el clculo fr resulta tener un valor negatia
vo entonces la fuerza de roce en realidad
apunta en el sentido opuesto al mostrado
en la gura. Sea M g el peso, la fuerza
ejercida por la tensin de la cuerda y FN
o
la fuerza normal que ejerce el plano inclinado sobre el bloque. Debido al principio
de accin y reaccin, FN tambin coincide
o
o
e
con la magnitud de la fuerza que el bloque
ejerce sobre el plano.

Figura 4.8

Introduzcamos un sistema de coordenadas cartesianas en que el eje x es paralelo

y el eje y normal al plano inclinado (ver gura 4.8). Como el bloque est en reposo,

a
la fuerza neta sobre el bloque debe ser nula, esto es, tanto la fuerza total a lo largo
del eje x como a lo largo del eje y . Esto nos da las siguientes ecuaciones:

eje x:

M g sin fr = 0

CAP
ITULO 4. LAS LEYES DE NEWTON
eje y :

157

FN M g cos = 0 ,

donde es el ngulo de elevacin del plano inclinado. Como la masa m no acelera, la


a
o
tensin de la cuerda debe ser = mg. Luego, de la primera ecuacin se deduce que
o
o
fr = mg M g sin .
Recordemos que fr puede ser positivo o negativo: fr es positivo si m > M sin y
negativo si m < M sin . Tambin se tiene que
e
|fr | e FN = e M g cos .
De las ecuaciones anteriores se deduce que
mg M g sin = +fr e FN = e M g cos

mg + M g sin = fr e FN = e M g cos

si m > M sin ,
si m < M sin ,

o sea el bloque de masa M no se desliza sobre el plano inclinado si


i) para M sin < m , se cumple que m M (e cos + sin ),
ii) para M sin > m , se cumple que m M (sin e cos ).
Para los valores numricos del enunciado, el bloque no se deslizar por el plano si
e
a
0.61 kg < m < 3.4 kg.
Observemos que, si = 0, resulta M e m M e . En este caso, la supercie
es completamente horizontal, y la masa m es simplemente equivalente a una fuerza
externa mg. Para que el sistema se mantenga en equilibrio se requiere simplemente
que dicha fuerza externa (que puede ir hacia la derecha o hacia la izquierda) no sea
mayor que la fuerza de roce esttico mxima.
a
a
Por su parte, si = /2, se tiene M m M , es decir, el sistema se mantiene
en equilibrio slo si m = M . Lo cual es evidente, ya que cuando la supercie es
o
completamente vertical la fuerza normal sobre el bloque es cero, de modo que el roce
desaparece del problema. Tenemos entonces un sistema de dos masas colgando de los
extremos de una cuerda que pasa por una polea, el cual slo se mantiene en equilibrio
o
si las masas en ambos extremos son iguales.
Analicemos ahora lo que sucede si m sobrepasa (en una magnitud innitesimal)
al valor M (e cos + sin ). En ese caso, el bloque comenzar a deslizarse hacia
a
arriba. La fuerza de roce, por lo tanto, ser
a
fr = c M g cos x .

La fuerza neta sobre el bloque y su aceleracin, en la direccin x, vendrn dados por


o
o
a
Fx = fr M g sin = c M g cos M g sin .

CAP
ITULO 4. LAS LEYES DE NEWTON

158

Fx

=
g(c cos + sin ) .
M
M
Por otra parte, la fuerza neta sobre la masa m y su aceleracin en la direccin vertical,
o
o
sern
a
F = mg .
ax =

F
=
g = ax .
m
m
La ultima igualdad en la ecuacin anterior se debe a que la cuerda es inextensible;

o
por consiguiente, cuando el bloque acelera hacia arriba, la masa m acelerar con la
a
misma magnitud, pero hacia abajo. De las ecuaciones anteriores se deduce que
a =

ax = g

m
M

c cos sin
m
M +1

(4.3)

Este resultado tambin lo podemos escribir de otra manera. Recordemos que m soe
brepasa en una magnitud innitesimal al valor M (e cos + sin ), luego
m = M (e cos + sin ) ,

(4.4)

o sea,

m
= e cos + sin .
M
Sustituyendo esto en la expresin para ax se obtiene
o
ax = g

(e c ) cos
.
1 + e cos + sin

(4.5)

Con los valores numricos del enunciado se obtiene ax 0.047 g.


e
Note que la tensin de la cuerda es distinta en el caso estacionario que en el caso
o
dinmico. En el primer caso es = mg, mientras que en el segundo viene dada por
a
= m(g ax ).
En el caso dinmico, el l
a
mite = /2 es problemtico, pues (4.5) da ax = 0,
a
lo que es imposible, pues si m supera ligeramente el valor de equilibrio (que en este
l
mite es igual a M ), el sistema deber acelerar. El problema es que, en rigor, (4.4)
a
deber ser m = M (e cos + sin ) + , con 1. En ese caso, la expresin
a
o
(4.5) adquiere un trmino adicional de orden . Dicho trmino es irrelevante para
e
e
ngulos arbitrarios, siendo el trmino dominante el que aprarece en (4.5), es decir,
a
e
la aceleracin es esencialmente independiente de la diferencia entre m y su valor
o
de equilibrio. Pero para ngulos muy grandes, en particular = /2, la situacin se
a
o
invierte, es el trmino independiente de el que es despreciable, y ax , tal como
e
sugiere la intuicin, esto es, la aceleracin depende exclusivamente de la diferencia de
o
o
las masas m y M .

CAP
ITULO 4. LAS LEYES DE NEWTON

159

Problema resuelto en clases: 4.35


Problema resuelto en clases: 4.32

4.5.

Problemas

1. Un automvil de 2000 kg movindose a 80 km/h puede llevarse al reposo en


o
e
75 m mediante una fuerza de frenado constante:
a) Cunto tiempo tardar en detenerse?
a
a
b) Cul es la fuerza necesaria para detener el coche en esa distancia? Quin
a
e
o qu ejerce esa fuerza horizontal que detiene al coche?
e
2. Una carga de 2 toneladas se levanta mediante una gra.
u
a) Inicialmente, durante cierto intervalo de tiempo, la carga sube con una
aceleracin a = 1.3 m/s2 . Cul es la tensin del cable que la soporta?
o
a
o
b) Despus de un breve per
e
odo de aceleracin, la carga sigue elevndose con
o
a
una velocidad constante. Cul es la tensin del cable en ese caso?
a
o
3. Dos bloques unidos por una cuerda que pasa por una polea sin rozamiento, descansan sobre planos lisos
como se muestra en la gura 4.9.
a) En qu sentido se mover el
e
a
sistema?
b) Cul es la aceleracin de los
a
o
bloques?
c) Cul es la tensin de la cuera
o
da?

Figura 4.9

4. Una pelota de 2 kg cae libremente llegando, en cierto instante, a tener una rapidez de 6 m/s. Qu fuerza vertical constante se debe aplicar para detenerla en
e
los prximos 5 m? Qu fuerza vertical constante se debe aplicar para detenerla
o
e
en los prximos 5 s?
o

CAP
ITULO 4. LAS LEYES DE NEWTON

160

5. Qu fuerza F debe aplicarse al carro


e
de masa M (ver gura adjunta) para
que el carro de masa m2 no suba ni
baje?
m2
Respuesta: F = g (M + m1 + m2 ) m1

Figura 4.10
6. Considere un pndulo que consiste en
e
una masa m colgada de un hilo de largo . En presencia de un campo gravitacional constante, al sacar el pndulo
e
de su posicin de equilibrio y soltarlo,
o
ste oscilar. Encuentre la aceleracin
e
a
o
de la masa m en el instante en que el
pndulo forma un ngulo con la nore
a
mal.
Si 1, demuestre que
d2 (t)
2
+ 0 (t) = 0 ,
dt2
con 0 =

Figura 4.11

g/.

7. Considere una masa m adosada a un resorte de constante de restitucin k. Sea


o
x = 0 la posicin de equilibrio del sistema. De acuerdo a la Ley de Hook, al
o
desplazar la masa m una distancia x desde su posicin de equilibrio, la fuerza
o
ejercida por el resorte sobre la masa es F = kx. Demuestre que
d2 x(t)
2
+ 0 x(t) = 0 ,
dt2

con 0 =

k/m. Compare este resultado con el del problema anterior.

8. Un cuerpo de 500 g desliza por un plano inclinado liso. El cuerpo parte del
reposo y durante el tercer segundo recorre una distancia de 120 cm. Encuentre
el ngulo de inclinacin del plano.
a
o

CAP
ITULO 4. LAS LEYES DE NEWTON

161

9. Una esfera de masa m es mantenida en la posicin A por dos cuerdas


o
(ver gura 4.12). Sea TA la tensin
o
de la cuerda indicada. Se corta la
cuerda horizontal y el pndulo oscie
la hasta la posicin B. Cul es la
o
a
razn de las tensiones TB /TA ?
o
Respuesta:

TB /TA = cos2 .
Figura 4.12

10. Considere el montaje mostrado en


la gura 4.13, con M =1,650 kg,
m=0,150 kg y d0 =4 m. El sistema
est en reposo cuando d = d0 = 4 m.
a
Cunto tiempo transcurrir antes
a
a
de que la masa m llegue a la base de
M?
Figura 4.13
11. Un objeto se encuentra sobre un
plano liso sin roce y es sometido a
una fuerza F que var en funcin
a
o
del tiempo de acuerdo al grco que
a
se acompaa. Si la masa del objeto
n
es m, obtenga y graque las siguientes magnitudes:
Figura 4.14
a) Aceleracin del objeto en funcin del tiempo.
o
o
b) Velocidad del objeto, si ste parte del reposo.
e
c) Posicin del objeto en funcin del tiempo.
o
o
12. Una pesa calibrada en Newtons se coloca sobre una plataforma mvil y se hace
o
deslizar con una rapidez constante de 14 m/s sobre un terreno ondulado (ver
gura 4.15). Sobre la pesa se coloca una caja que pesa 500 N.
a) Cuando la plataforma pasa sobre la cresta de una colina con radio de
curvatura de 100 m, cul es la lectura de la pesa?
a
b) Cuando la plataforma pasa por la parte inferior de una hondonada con
radio de curvatura de 80 m, cul es la lectura de la pesa?
a

CAP
ITULO 4. LAS LEYES DE NEWTON

162

Figura 4.15
Respuesta: (parte b) 625 N.
13. Un bloque de masa M es tirado hacia una muralla vertical mediante el
uso de una cuerda y poleas como se
muestra en la gura. El bloque se
desliza sin roce sobre la supercie.
La fuerza con que se tira la cuerda es F , el largo de la cuerda es 2L
y la separacin inicial entre el bloo
que y la muralla es L. Determine el
tiempo que transcurre hasta que se
encuentren la punta de la cuerda y
el bloque.
14. Un plato cnico de ngulo caraco
a
ter
stico gira uniformemente entorno a su eje, el cual se mantiene
en posicin vertical. Una piedrecilla
o
de masa m rota solidariamente con
el plato. Suponiendo que no hay roce entre la piedrecilla y la supercie
del plato, calcule el radio de la rbio
ta circular que describe la piedrecilla.

Figura 4.16

Figura 4.17

15. Una persona se para sobre una balanza dentro del ascensor y observa que sta
e
registra un peso igual a un 70 % de su peso normal. Si el ascensor y el pasajero
tienen masas M y m respectivamente, calcule la tensin a la que est sometido
o
a
el cable que sujeta el ascensor. Compare esta tensin con la que se producir
o
a
si el ascensor acelera con la misma magnitud pero en sentido opuesto.

CAP
ITULO 4. LAS LEYES DE NEWTON

163

16. Considere el montaje mostrado en la


gura 4.18. Suponga que las masas de
la polea y del hilo, as como el roza
miento son despreciables. Se conocen
las masas m, M y el ngulo de la cua.
a
n
Encuentre la aceleracin de la cua.
o
n
mg sin
Respuesta: a = M + 2m(1 cos ) .

Figura 4.18
17. Dos masas m y M se encuentran unidas por una cuerda de masa despreciable y
largo . En estas condiciones ambas realizan un movimiento circular uniforme
(en un plano horizontal) en torno al as llamado centro de masas del sistema.

Suponga que el per


odo del movimiento rotatorio es T . Encuentre la distancia
entre la masa m y el centro de giro (para resolver esta parte del problema no
es necesario conocer la denicin de centro de masas). Calcule la tensin de la
o
o
cuerda que une ambas masas.
2 2
mM
Respuesta:

.
=
m+M
T
18. Una cua lisa de masa M se desliza
n
bajo la accin de una fuerza horizontal
o
F . Sobre ella se coloca un bloque de
masa m.
a) Dibuje todas las fuerzas que
actan sobre cada una de las
u
masas.
b) Determine el valor de F para
que el bloque ms pequeo no
a
n
resbale sobre la cua.
n
19. Dos bloques idnticos y de masa m
e
posan sobre una supercie horizontal
pulida. Uno de ellos es tirado mediante una cuerda en cuyo extremo libre
se aplica una fuerza horizontal igual a
M g. El otro bloque es tambin tirado
e
horizontalmente mediante una cuerda
pero en cuyo extremo libre cuelga una
bola de masa M . Determine cual de
los bloques se mueve ms rpido si
a a
ambos parten del reposo simultneaa
mente.

Figura 4.19

Figura 4.20

CAP
ITULO 4. LAS LEYES DE NEWTON

164

20. Un pintor que pesa 900 Newtons trabaja en una silla colgante en un edicio de
altura. Al terminar su turno debe volver al ultimo piso para bajar a la calle.

Para subir con la silla tira de la cuerda de tal forma que la fuerza que l
e
ejerce sobre el asiento de la silla es de
500 Newtons. La silla misma pesa 300
Newtons.
a) Cul es la aceleracin del pintor
a
o
y de la silla?
b) Cul es la fuerza total sobre el
a
soporte de la polea?
Respuestas: a)

a = 2g/3 ; b)

Figura 4.21

Ftot = 2000 N.

21. Considere el montaje mostrado en la gura 4.22. La masa del cuerpo # 1 es


n = 4 veces la del cuerpo # 2. Suponga que las masas de las poleas y de los
hilos, as como el rozamiento son des
preciables por su pequeez. Cuando el
n
cuerpo # 2 se suelta, la masa # 1 se
encuentra a una altura h. Cul es la
a
aceleracin de la masa # 2 mientras m1
o
baja? Cul es la altura mxima del
a
a
suelo H a la que subir la masa # 2?
a
(La altura mxima no es 2h!)
a
Respuesta: H = 6hn/(n + 4) .
Figura 4.22
22. Una masa m se encuentra apoyada sobre una cua de masa M y ngulo de
n
a
elevacin . La cua se puede desplao
n
zar horizontalmente sin roce sobre un
plano. Dos gu restringen el movias
miento de la masa m de manera que
sea slo en direccin vertical. No hay
o
o
roce entre la masa m y la cua como
n
tampoco entre las gu y la masa m.
as

Figura 4.23

CAP
ITULO 4. LAS LEYES DE NEWTON

165

a) Encuentre la relacin que existe entre la aceleracin vertical am de la masa


o
o
m y la aceleracin horizontal aM de la cua.
o
n
b) Haga los diagramas de cuerpo libre de la masa m y de la cua M .
n
c) Encuentre la aceleracin aM de la cua.
o
n
d) Si entre la cua y el suelo hay roce cunto es el valor m
n
a
nimo que debe
valer el coeciente de roce esttico e para que la cua no acelere?
a
n
23. Considere dos masas M y m unidas por
un hilo que pasa por una polea ideal tal
como se muestra en la gura adjunta.
Inicialmente la masa M se sujeta con
un hilo auxiliar (que no se muestra en
la gura) y el sistema se encuentra en
reposo. En cierto instante el hilo auxiliar se corta. Demuestre que la aceleracin de la masa M es (con el eje z
o

apuntando hacia arriba):


a=

4M + 2m
g .
z
4M + m

Demuestre que esta expresin da el vao


lor correcto en los l
mites M m y
m M.
24. Dos objetos 1 y 2, de igual masa, estn
a
atados a los extremos de una cuerda
ideal de largo L. El conjunto descansa sobre un disco que gira en un plano
horizontal con velocidad angular constante, en torno a su centro (ver gura).
Suponga que no existe friccin entre el
o
disco y el objeto 1, pero existe friccin
o
entre el objeto 2 y la supercie del disco. Los coecientes de friccin esttico
o
e
y cintico entre la masa 2 y el disco son
e
e y c , respectivamente.

Figura 4.24

Figura 4.25

Se observa que cuando el disco gira con velocidad angular 0 , la cuerda se


mantiene tensa y alineada en la direccin radial. En esta condicin el objeto
o
o
2 est en reposo a una distancia R del eje de rotacin. Cuando la velocidad
a
o

CAP
ITULO 4. LAS LEYES DE NEWTON

166

angular es mayor que 0 el objeto 2 (y tambin el 1) resbala sobre el disco.


e
Calcule el valor de 0 .
25. Tal como el campo gravitacional ejerce sobre una masa m una fuerza, un campo
elctrico E ejerce una fuerza sobre una carga q. Esta ultima viene dada por
e

F =qE.
(En el sistema internacional de unidades SI, la unidad para la carga es el
Coulomb [C] y la del campo eletrico Volt/metro= Newton/Coulomb, siendo
c
las abreviaciones [V/m]=[N/C]. Un campo de 1 [V/m] ejerce sobre una carga
de 1 [C] una fuerza de 1 [N].)
Considere un electrn, inicialmente en reposo, que es acelerado entre dos placas
o
(un condensador) separadas por una distancia de 1 cm. En el espacio entre las
dos placas hay un campo elctrico de 900 Volt/cm.
e
a) Cul es su velocidad terminal (la velocidad con que emerge del primer
a
condensador)?
b) Suponga ahora que el electrn de la parte a), despus de ser acelerado y
o
e
emerger (por un pequeo agujero) del espacio entre las dos placas, ingresa
n
a una regin de largo L = 3cm en que existe un campo elctrico transversal
o
e
de magnitud |E | = 30 Volt/cm. Cul ser el ngulo de deexin con
a
a a
o
que emerger el electrn del segundo condensador? (Ver gura 4.26). (En
a
o
este problema Usted puede despreciar la interaccin gravitatoria, es decir,
o
puede suponer que g = 0. La carga de un electrn (universalmente denotao
da con la letra e) es e = 1, 601019 [C] y su masa me = 9, 111031 [Kg].)

Figura 4.26

26. Un pulso de iones de Cs+ (simplemente ionizados) que han sido acelerados
desde el reposo por un campo elctrico de 1 (statvolt/cm) a lo largo de 0,33 cm,
e

CAP
ITULO 4. LAS LEYES DE NEWTON

167

tarda un tiempo t = 87 109 s para recorrer 1 mm despus del proceso de


e
aceleracin (ver gura 4.27).
o
a) Encuentre la masa del Cs+ .
b) Si en lugar de Cs+ se realiza el experimento con deuterones, cunto ser el tiempo
a
a
de traves t ?
a
c) Suponiendo que los protones y
los neutrones tienen la misma
masa, encuentre la masa de un
neutrn.
o
d) Con este dispositivo experimental, ser posible disa
tinguir entre deuterones y
part
culas ?

Figura 4.27

(Un deutern es un ncleo atmico formado por un protn y un neutrn; una


o
u
o
o
o
part
cula es equivalente a un ncleo de un tomo de He y consiste en dos prou
a
tones y dos neutrones. El ncleo de cesio consta de 58 protones y 84 neutrones,
u
+ corresponde a un tomo de cesio que ha perdido un electrn).
el in Cs
o
a
o
27. Considere una carga q que en el instante t = 0 se encuentra en el origen y en
reposo. A partir de t = 0 se le aplica un campo elctrico alterno de la forma
e
E = E0 sin (t) x .

Encuentre la ecuacin diferencial que describe el movimiento de la carga y


o
encuentre la expresin ms general para la posicin x(t).
o
a
o
28. Un bloque de masa M sube por un
plano inclinado cuyo ngulo de elea
vacin es . Los coecientes de roce
o
esttico y cintico entre la masa M
a
e
y el plano son e y c , respectivamente.

Figura 4.28

a) Cul es la altura mxima que alcanza el bloque, si parte con velocidad v0


a
a
desde la base del plano?
b) Qu condicin debe satisfacerse para que el bloque vuelva a descender?
e
o

CAP
ITULO 4. LAS LEYES DE NEWTON

168

c) En caso de cumplirse la condicin anterior, con qu velocidad llegar a la


o
e
a
base del plano inclinado?

29. Una masa de 100 kg se empuja a


lo largo de una supercie en la cual
el roce es despreciable mediante una
fuerza F , de modo que su aceleracin es de 6 m/s2 (ver gura). Una
o
masa de 20 kg desliza a lo largo de
la parte superior de la masa de 100
kg y tiene una aceleracin de 4 m/s2
o
(por lo tanto desliza hacia atrs resa
pecto a la masa de 100 kg).

Figura 4.29

a) Qu fuerza de rozamiento ejerce la masa de 100 kg sobre la masa de 20


e
kg?
b) Cul es la fuerza neta sobre la masa de 100 kg? Cul es la fuerza F ?
a
a
c) Una vez que la masa de 20 kg se cae de la masa de 100 kg, cul es la
a
aceleracin de la masa de 100 kg?
o
30. Sea el coeciente de roce esttico
a
entre la masa m y el carro. Cul es
a
la fuerza m
nima que debe aplicarse al carro para que la masa m no
caiga?
Respuesta: F min = (M + m)g/ .

31. Las masas A y B son de 10 y 5 Kg


respectivamente. El coeciente de
roce de A con la mesa es = 0.2.
Encuentre el m
nimo valor de la masa C que impide el movimiento de A.
Encuentre la aceleracin de A si se
o
saca C.

Figura 4.30

Figura 4.31

CAP
ITULO 4. LAS LEYES DE NEWTON

169

32. Una carretera est peraltada de moa


do que un automvil, desplazndose
o
a
a 80 Km/h, puede tomar la curva de
30 m de radio, incluso si existe una
capa de hielo equivalente a un coeciente de friccin aproximadamente
o
Figura 4.32
cero.
Determinar el intervalo de velocidades a que un automvil puede tomar esta
o
curva sin patinar, si los coecientes de friccin esttica y cinemtica, entre la
o
a
a
carretera y las ruedas, son e = 0.3 y c = 0.26, respectivamente.

33. Cul es el mximo valor que puede


a
a
tener m3 para que m1 no se caiga
si el coeciente de friccin esttico
o
a
entre m1 y m2 es e , y el de friccin
o
cinemtica entre m2 y la mesa es c ?
a
Respuesta:
mmax =
3

(m1 + m2 ) c +e
1e

si e < 1
si e > 1

.
Figura 4.33

34. Un bloque de masa M , inicialmente en reposo, resbala por un plano inclinado


cuyo ngulo de elevacin es . Despus de recorrer una distancia D el cuerpo
a
o
e
lleva una velocidad igual al 50 % de la velocidad que habr adquirido en aua
sencia de roce. Encuentre una expresin para el coeciente de roce cinemtico
o
a
entre el plano y el bloque.
35. Sea c el coeciente de roce cintie
co entre un escobilln, cuya masa es
o
m, y el piso. Un hombre ejerce una
fuerza F a lo largo del palo del escobilln. Encuentre |F | en funcin de
o
o
. Existe una solucin para todo
o
entre 0 y 90 ? (El barrendero avanza con velocidad uniforme.)

Figura 4.34

CAP
ITULO 4. LAS LEYES DE NEWTON
36. Una part
cula de masa M descansa
sobre un plano inclinado que forma
un ngulo con la horizontal. Si el
a
coeciente de roce esttico es e , ena
cuentre la m
nima fuerza horizontal
Fmin transversal a la pendiente del
plano, que se requiere para que la
part
cula comience a moverse.

Respuesta:

Fmin =

Mg
0

170

Figura 4.35

2 cos2 sin2 si e > tan


e
si e < tan

37. Considere un paquete, de masa m,


que se mueve sin roce y con rapidez v0 sobre una supercie de hielo.
En cierto instante el paquete entra
en contacto con el tablero horizontal de un trineo de masa M , que a su
Figura 4.36
vez puede deslizarse sin roce sobre el
hielo.
Suponga que el coeciente de roce entre el paquete y el trineo es y que el
paquete se desliza sobre el trineo hasta nalmente quedar en reposo con respecto
a ste.
e
a) Una vez que el paquete queda en reposo con respecto al trineo, cul es la
a
velocidad del trineo?
b) Cunto tiempo demora el paquete en quedar en reposo con respecto al
a
trineo?
c) Evale el momento lineal del paquete antes de que entre en contacto con el
u
trineo y comprelo con el momento lineal del conjunto (trineo ms paquete)
a
a
una vez que el paquete est en reposo respecto al trineo.
a
(El momento lineal de un objeto es el producto de su masa y velocidad).
38. Con dos bloques A y B se arman las conguraciones I y II que se indican en la
gura adjunta. Suponga que las cuerdas y poleas tienen masas despreciables y
el coeciente de roce es constante y es el mismo entre todas las supercies en
contacto. El valor de las fuerzas aplicadas FI y FII es tal que el bloque A se
mueve con velocidad constante en ambas situaciones. Calcule el cuociente entre
el mdulo de FI y FII .
o

CAP
ITULO 4. LAS LEYES DE NEWTON

171

Figura 4.37
39. Considere un cuerpo que cae en la atmsfera. El aire se opone al movimiento
o
con una fuerza que es proporcional al cuadrado de la velocidad, es decir
Froce = kvv ,

v = |v| .

Encuentre la velocidad terminal.


40. Cuando un cuerpo cae en un l
quido y el ujo es laminar (es decir, no es turbulento), el u se opone al movimiento con una fuerza que es proporcional
do
a la velocidad, es decir
Froce = v ,

v = |v| .

Encuentre la velocidad terminal. (El coeciente depende del u y de la


do
forma del objeto).
41. Sea el coeciente de roce cinemtia
co que acta entre las supercies de la
u
masa m y las cuas (ver gura adjunn
ta). Entre las cuas y el suelo el roce
n
es nulo. Suponga que el valor del roce
es tal que el sistema no se encuentra en
equilibrio (es decir, las cuas se sepan
ran y el bloque baja). Sea el ngulo,
a
M la masa de las cuas y m la man
sa del bloque. Determine la aceleracin
o
del bloque m.

Figura 4.38

CAP
ITULO 4. LAS LEYES DE NEWTON

172

42. Sobre un plano inclinado liso, que forma un ngulo con la horizontal, se
a
desliza un bloque partiendo del reposo.
Despus de recorrer una distancia D,
e
el bloque entra en un tramo rugoso. El
bloque se detiene luego de recorrer una
distancia D en dicho tramo. Calcule el
coeciente de roce cintico entre el bloe
que y la supercie rugosa.

Figura 4.39

43. Cul es el mximo valor que puede tener m3 para que m1 no se caiga? Considere
a
a
el coeciente de friccin esttico entre m1 y m2 igual a e , y los dos coecientes
o
a
de friccin cinemtica (entre m2 y el plano horizontal, y entre m3 y el plano
o
a
inclinado) iguales a c .
m1
m2
g

m3

44. Un sujeto de masa MA est parado frente a un bloque de masa MB , y ambos


a
se encuentran sobre una pista de hielo. El sujeto lanza, con una velocidad v0
respecto a l mismo, una pelota de masa m hacia el bloque. La pelota se desplaza
e
horizontalmente hasta llegar al bloque, con el cual choca elsticamente. Con
a
qu velocidad vB queda movindose el bloque luego del impacto?
e
e

MA

MB
m

45. Acrisio, rey de Argos, ha decidido limpiar su honor. Dnae, su esposa, hab sido
a
a
seducida por Zeus, dando a luz a Perseo. Acrisio desaar a los dioses. Encierra
a
a Dnae y a Perseo en un cofre, y empuja ste hacia el mar con una fuerza
a
e
F horizontal, constante. El mar se encuentra a una distancia 2L de Acrisio.

CAP
ITULO 4. LAS LEYES DE NEWTON

173

Partiendo desde el reposo, recorre primero una distancia L sin contratiempos,


en l
nea recta. Pero al llegar a la distancia L, un fuerte viento enviado por
Zeus azota la costa, aplicando una fuerza Fv sobre el cofre, tambin horizontal,
e
formando un ngulo con F (ver gura). A pesar de ello, Acrisio contina
a
u
empujando el cofre con la misma fuerza, y logra nalmente llegar al mar.
Cunto tiempo tard Acrisio en llegar al mar? Cul es el mdulo de la veloa
o
a
o
cidad del cofre al momento de llegar al mar?
Vista lateral
escudo de Acrisio

zona de viento
enviado por Zeus

(en el suelo mientras


empuja el cofre)

Fv

mar

L
Vista desde arriba

Fv

mar

46. Dos cuerpos se encuentran unidos por una polea sobre una cua, como indica
n
la gura. El cuerpo B cuelga verticalmente, y el cuerpo A est sobre el plano
a
inclinado, que forma un ngulo con el suelo. El cuerpo A est abierto en su
a
a
parte superior, y recibe una masa de agua por unidad de tiempo. El cuerpo
B tiene masa M , y el cuerpo A (cuando no tiene agua) tambin.
e
a) Si inicialmente el cuerpo A no tiene agua, cunto tiempo tarda en recibir
a
suciente agua para que el sistema se mantenga en reposo?
b) Despus de llegar al estado anterior, el agua sigue cayendo sobre el cuerpo
e
A. Si el coeciente de roce esttico entre el cuerpo A y el plano inclinado
a
es e , cunto tiempo adicional [es decir, despus del estado descrito en
a
e
a)] tarda el sistema en comenzar a acelerar?

CAP
ITULO 4. LAS LEYES DE NEWTON

174

A
B

47. Un cuerpo de masa m se encuentra en reposo sobre una supercie horizontal


lisa, en un punto O. En t = 0 se comienza a aplicar sobre l una fuerza F1
e
constante, horizontal hacia la derecha. Cuando ha recorrido una distancia D,
adems de F1 , una segunda fuerza F2 , horizontal hacia la izquierda, comienza
a
a aplicarse sobre el cuerpo. Determine F2 para que el cuerpo se detenga luego
de haber recorrido una distancia L desde el punto O.

F1
O

m
D
L

48. Una part


cula P de masa m puede moverse slo sobre la supercie de un cono
o
invertido, unida a una cuerda sin masa, de cuyo otro extremo cuelga un cuerpo
Q de masa M . La cuerda es paralela a la supercie del cono, y sale del cono
por un pequeo oricio en su vrtice (ver gura). P describe un movimiento
n
e
circular uniforme de radio R. Si el ngulo de apertura del cono es , encuentre
a
el mdulo de la velocidad que debe tener P para que el cuerpo Q permanezca
o
en reposo.

R
m

CAP
ITULO 4. LAS LEYES DE NEWTON

175

49. Hay asuntos que slo un pingino puede solucionar. Mantener todo bajo control
o
u
es un trabajo de tiempo completo, pero si alguien sabe de sacricios, es un
pingino. No hay tiempo que perder: deslizarse por ese plano inclinado es el
u
unico modo de llegar rpidamente al mar y reportarse a Skipper a la brevedad.

a
Kowalski y Cabo se deslizan, partiendo desde el reposo, por la pendiente. Pero,
claro, hay pinginos y pinginos. Kowalski se desliz por un sector con hielo,
u
u
o
de modo que el roce con la supercie es depreciable, pero Cabo lo hizo por
una zona sin hielo. El resultado era esperable: al llegar al nal de la pendiente,
despus de recorrer una distancia D, la velocidad de Cabo era un 50 % de la
e
velocidad de Kowalski. Suponiendo que Kowalski y Cabo tienen la misma masa
M , cunto es el coeciente de roce cintico entre Cabo y la supercie?
a
e

4.6.

Solucin a algunos de los problemas


o

Solucin al problema 12a


o
Al pasar la plataforma por la cresta de la colina hay dos fuerzas actuando sobre
la caja:
i) El peso, W = M g. (Hemos elegido al eje z apuntando hacia arriba, M es la
z

masa de la caja.)
ii) La reaccin de la pesa sobre la caja: Fr = Fr z .
o

La fuerza neta es, por lo tanto,


Fneta = (Fr M g) z .

Por otra parte, sabemos que la caja est realizando un movimiento circular de radio
a
R con rapidez constante, o sea, hay una fuerza neta sobre la caja que actua hacia el
centro del c
rculo (la fuerza centr
peta), que es
Fcent =

M v2
z.

CAP
ITULO 4. LAS LEYES DE NEWTON

176

La fuerza centr
peta y la fuerza neta deben ser iguales, es decir, se tiene que
Fr M g =

M v2
.
R

Despejando Fr se obtiene
Fr = M g
= 500N

v2
gR

142
9, 81 100

400 N .

Solucin al problema 16
o
Observe primero que, al moverse la cua hacia la derecha, el bloque m se mon
ver en diagonal (hacia la derecha y hacia abajo). Sea rm el vector de traslacin de
a
o
m cuando la cua se traslada en una magnitud s. Se tiene (ver gura 4.40) que
n
rm = s(1 cos ) x s sin y .

Figura 4.40
Por supuesto que la aceleracin de la cua M y del bloque m estn relacionados.
o
n
a
Si la aceleracin de la cua es
o
n

rM = a x ,

entonces

rm = a (1 cos) x a sin z .

(4.6)

Sea la tensin de la cuerda y R la fuerza que la cua ejerce sobre el bloque m.


o
n
Debido a que no hay roce entre las superecies, esta fuerza de reaccin R es normal
o
al plano inclinado.

CAP
ITULO 4. LAS LEYES DE NEWTON

177

La gura 4.41 muestra el diagrama de


cuerpo libre para la masa m. Las componentes horizontal y vertical de la fuerza
neta que acta sobre el bloque m son
u
(m)

Fx

= cos R sin

(4.7)

y
(m)

Fz

= mg + sin + R cos , (4.8)

Figura 4.41

respectivamente.
Usando la segunda ley de Newton y las ecuaciones (4.6), (4.7) y (4.8), se encuentran las relaciones
cos R sin = m a (1 cos)
(4.9)
y
mg + sin + R cos = m a sin

(4.10)

Sobre la cua actan 4 fuerzas:


n
u
i) El peso M g.
z

ii) Una fuerza (de reaccin) R que el suelo ejerce sobre la cua. Esta fuerza, cuya
o
n
magnitud no nos interesar, acta en la direccin +.
a
u
o
z
iii) Una fuerza que el bloque m ejerce sobre la cua. Por el principio de accin
n
o
esta fuerza es R, o sea, las componentes horizontal y vertical son R sin y
R cos , respectivamente.
iv) La fuerza ejercida por la roldana sobre la
cua (que es igual a la fuerza ejercida por
n
la cuerda sobre la roldana). De la gura 4.42 se deduce que la fuerza total que
ejerce la cuerda sobre la roldana es
Fc = (1 cos ) sin .
x
z

Figura 4.42

La cua slo se mueve a lo largo de la horizontal; por eso slo nos interesa esa
n o
o
componente de la fuerza neta. Usando la segunda ley de Newton se obtiene
R sin + (1 cos ) = M a .

(4.11)

CAP
ITULO 4. LAS LEYES DE NEWTON

178

Las tres ecuaciones de movimiento (4.9), (4.10) y (4.11) con las tres incgnitas a,
o
y R, permiten resolver el problema. Sumando (4.9) y (4.11) se obtiene
= ma(1 cos ) + M a

(4.12)

Multiplicando (4.9) por cos y (4.10) por sin y sumando ambas ecuaciones se
obtiene
= mg sin + ma(cos 1)
(4.13)
De (4.12) y (4.13) se deduce nalmente que
a=

mg sin
.
M + 2m(1 cos )

Solucin al problema 22
o
La relacin entre las aceleraciones es am = aM tan . Los diagramas de cuerpo libre
o
de la masa m y de la cua se muestran en la gura 4.43. FN es la fuerza entre la
n
masa m y la cua.
n

Figura 4.43
Debido a que no hay roce esta fuerza es normal al plano incliado de la cua. Fr es
n
la fuerza que la gu ejerce sobre el bloque m (tal fuerza es perpendicular a la gu
a
a.)
Fp es la fuerza que el piso ejerce sobre la cua; en ausencia de roce esta fuerza es
n
perpendicular al piso.
Las ecuaciones de movimiento para la masa m y la cua son:
n
mg FN cos = mam
y
FN sin = M aM .
Usando la relacin entre las aceleraciones am y aM , podemos despejar aM , obteo
nindose
e
m tan
.
aM = g
M + m tan2

CAP
ITULO 4. LAS LEYES DE NEWTON

179

Si, debido al roce entre el suelo y la cua el sistema est en equilibrio, entonces
n
a
la suma de las fuerzas sobre m debe ser nula. Esto permite evaluar FN de inmediato:
FN cos = mg .
Al diagrama de cuerpo libre de la cua hay que agregar una fuerza de roce fr horin
zontal (apuntando hacia la izquierda). Que la suma de las fuerzas horizontales sobre
la cua sean nulas nos da la relacin
n
o
FN sin = fr ,
o sea,
mg tan = fr .
Por otra parte, la fuerza de roce debe satisfacer la relacin
o
fr e Fp = e (M g + FN cos ) = e (M + m)g .
De las relaciones anteriores se desprende que
mg tan = min (M + m) g ,
e
o sea,
min =
e

m
tan .
m+M

Solucin al problema 24
o
Del hecho que la velocidad angular es constante y las masas 1 y 2 siguen trayectorias
circulares, se deduce que la fuerza neta que acta sobre ellas es
u
2
F1 = m0 (R + L) r

y
2
F2 = m0 R r ,

respectivamente. Aqu m es la masa de cada una de part

culas y r es un vector unitario

que apunta en la direccin radial.


o
La unica fuerza radial real que acta sobre la masa 1 es la que ejerce la cuerda,

u
luego
2
= m 0 (R + L) ,
donde es la tensin de la cuerda.
o
Sobre la part
cula 2 actan dos fuerzas radiales: la tensin de la cuerda r y la
u
o

fuerza de roce fr r . Se tiene

2
fr = m0 R

CAP
ITULO 4. LAS LEYES DE NEWTON

180

o sea,
2
2
fr = + m0 R = m0 (2R + L) .

Para que las masas no se deslicen la fuerza de roce debe satisfacer la desigualdad
fr e mg. De las dos ultimas ecuaciones se deduce que

2
m0 (2R + L) e mg .

La velocidad angular l
mite a partir de la cual las masas comienzan a deslizarse es,
por lo tanto,
e g
0 =
.
2R + L
Solucin al problema 26
o
Sea M la masa, a la aceleracin y t el tiempo que tardan las part
o
culas de
en atravesar el condensador. La carga de cada in de cesio es q = e, donde
o
e = 1, 60 1019 [C] es la carga de un electrn (ver problema 25). Durante el proceso
o
de aceleracin, la fuerza que acta sobre cada in es F = qE0 . Usando la segunda ley
o
u
o
de Newton se obtiene que q E0 = M a. La aceleracin del tomo de cesio (mientras se
o
a
mueve al interior del condensador) es, por lo tanto, a = q/(M E0 ). El movimiento es
uniformemente acelerado.
Durante el intervalo de tiempo [0, t ] el in alcanza a recorrer una distancia
o
Cs+

s1 =

1 2
at = 0, 33 cm,
2

siendo la velocidad con que emerge del condensador v1 = at . A continuacin los


o
iones de cesio atraviesan con esa velocidad constante una regin de ancho s2 = 0, 1 cm,
o
tardando para ello un tiempo t = 87 109 s. Se tiene que
v1 t = at t = s2 ,
o sea
t =

s2
.
at

Por otra parte

2s1
.
a
Eliminando t de las dos ultimas ecuaciones se encuentra

t2 =

a=

s2
2
.
2s1 (t)2

Igualando las dos expresiones que tenemos para la aceleracin podemos despejar M
o
(la masa de cada in de cesio):
o
M=

2|e|E0 s1 (t)2
= 2, 4 1025 Kg .
s2
2

CAP
ITULO 4. LAS LEYES DE NEWTON

181

Cada in de Cs+ est formado por 58 protones, 84 neutrones y 57 electrones. La


o
a
masa de los electrones es despreciable frente al de los protones y neutrones y por
consiguiente, lo ignoraremos. La masa de un neutrn es muy parecida a la de un
o
protn y, en primera aproximacin, podemos suponer que son iguales. En lo que a
o
o
masa respecta, el in de cesio lo podemos pensar como un aglomerado de 58+84=142
o
nucleones. (Nucleones es el nombre genrico que se le da a los protones y neutrones).
e
Dividiendo la masa del in de cesio por 142 se encuentra que la masa de un nuclen es
o
o
27 Kg, valor que diere en 1 % del valor medido usando
aproximadamente 1,6910
otros mtodos.
e
Al acelerar deuterones (un protn + un neutrn) en lugar de iones de cesio, slo
o
o
o
cambia la masa ya que, igual que en el caso del cesio, la carga neta del deutern
o
es +|e| (o sea, la fuerza que acta sobre la part acelerada en ambos casos es la
u
cula
misma). El tiempo de traves t es proporcional a M luego, al usar deuterones en
a
lugar de iones de cesio, el tiempo de traves ser
a
a
td = tCs

142
108 s .
2

El dispositivo experimental no es capaz de distinguir entre deuterones y part


culas
. La part
cula (2 protones + 2 neutrones) tiene el doble de la masa del deutern
o
y tambin el doble de la carga neta. Estas dos modicaciones se cancelen en cuanto
e
a la aceleracin respecta, siendo por consiguiente ambas iguales.
o
Solucin al problema 27
o
La fuerza que acta sobre la carga (ver problema 25) es
u
F (t) = q E(t) = E0 sin(t) x .

Usando la segunda ley de Newton obtenemos las ecuaciones de movimiento:


m x(t) = qE0 sin(t)

m y (t) = 0

y
m z (t) = 0 .

De las dos ultimas, usando las condiciones iniciales se deduce que

y(t) = z(t) = 0 t ,
o sea, el movimiento slo ocurre a lo largo del eje x.
o

CAP
ITULO 4. LAS LEYES DE NEWTON

182

Integremos la primera ecuacin de moo


vimiento. Se tiene
t

x(t) = x(0) +

qE0
sin(t) dt
m

t
qE0
= 0
cos(t)
m
0
qE0
=
(1 cos(t)) .
m

La posicin de la carga en funcin del


o
o
tiempo se obtiene integrando la ultima

ecuacin:
o
x(t) = x0 +
= 0+
=

qE0
m

qE0
m

t
0

(1 cos(t)) dt

1
sin(t)

t
0

qE0
(t sin(t)) .
m 2

Figura 4.44

La gura 4.44 muestra un grco de la fuerza, la velocidad y la posicin de la


a
o
carga en funcin del tiempo.
o
Solucin al problema 32
o
De acuerdo con el enunciado, si la rapidez del coche es v0 = 80 km/h, no
actuar ninguna fuerza de roce. Como
a
la trayectoria del automvil es circular,
o
se tiene que el movimiento es acelerado y, por lo tanto, sobre el coche acta
u
una fuerza neta hacia el centro de giro
O (la fuerza centr
peta):
Fc =

2
mv0
r.

Figura 4.45

Las fuerzas reales que actan sobre el auto (y cuya suma da origen a la fuerza
u
centr
peta) son la fuerza de gravedad
Fg = mg z

CAP
ITULO 4. LAS LEYES DE NEWTON

183

y la fuerza normal que la carretera ejerce sobre el coche:


FN = FN cos z FN sin r .

Por supuesto que


Fc = Fg + FN ,
o sea,

2
mv0
r = mg z + FN cos z FN sin r .

R
Igualando las componentes se deduce que

FN cos = mg
y

2
mv0
.
R
Tomando el cuociente entre estas ecuaciones se encuentra una expresin que nos
o
permite encontrar el ngulo del peralte de la carretera :
a

FN sin =

tan =

2
v0
.
Rg

Sea v1 la mxima velocidad que el automvil puede tener sin que se deslice
a
o
lateralmente por la carretera. Si el automvil avanza con rapidez v1 , entonces adems
o
a
de la fuerza de gravedad y la fuerza normal, actuar tambin una fuerza de roce
a
e
esttica Fr :
a
Fr = Fr cos r Fr sin z .

Cuando el automvil avanza con velocidad mxima v1 , el valor de la fuerza de roce


o
a
tomar el valor mximo posible Fr = e FN . (Observe que el coeciente de roce que
a
a
debe usarse es el esttico y no el cinemtico.) Se tiene
a
a
Fc =

2
mv1
r = mg + FN cos z FN sin r Fr cos r Fr sin z ,

o sea

2
mv1
= FN sin Fr cos
R

y
0 = mg + FN cos Fr sin .
Con Fr = e FN se encuentra
2
mv1
= FN (sin + e cos )
R

CAP
ITULO 4. LAS LEYES DE NEWTON

184

y
mg = FN (cos e sin ) .
Eliminando FN y despejando v1 de las dos ultimas ecuaciones se obtiene nal
mente
2
v1 = Rg

2
v0 + Rge
2
Rg e v0

2
2
Este resultado es vlido mientras Rg > e v0 . Cuando Rg < e v0 , el coche nuna
ca se deslizar lateralmente hacia afuera y la velocidad mxima a la que se puede
a
a
transitar por la carretera, en ese caso, es innita.

Sea v2 la m
nima velocidad con que se puede transitar sobre la carretera sin
deslizarse lateralmente hacia el interior. El anlisis en este caso es anlogo al anterior,
a
a
excepto que la fuerza de roce esttica ahora acta en la direccin opuesta. Para v2 se
a
u
o
encuentra
2
v2 = Rg

2
v0 Rge
2
Rg + e v0

2
2
Este resultado es vlido mientras Rge < v0 . Cuando Rge > v0 , el coche nunca
a
se deslizar lateralmente hacia el interior, pudiendo permanecer incluso en reposo
a
(siendo, en ese caso, v2 = 0).

Para los valores numricos del enunciado las velocidades mxima y m


e
a
nima con
que se puede transitar por la carretera son v1 = 123 km/h y v2 = 59 km/h.
Solucin al problema 39
o
Sea v la velocidad terminal que un cuerpo adquiere al caer en la atmsfera. Al
o
caer con la velocidad terminal el cuerpo se mover con velocidad constante. O sea,
a
la aceleracin y la fuerza neta sobre el cuerpo deben ser nulas. Las unicas fuerzas
o

que actan sobre el cuerpo son la fuerza de gravedad Fg = mg y la fuerza de roce


u
z
2
Froce = kv v = kv z . Se tiene
2
Fg + Froce = mg + kv z = 0 ,
z

o sea,
v =
Solucin al problema 41
o

mg
.
k

CAP
ITULO 4. LAS LEYES DE NEWTON

185

Denotemos por a = a z la acelera


cin del bloque m. Las fuerzas vero
ticales sobre el bloque m nos dan la
ecuacin de movimiento
o
mg+2FN cos +2Fr sin = ma .
Como los bloques estn en movia
miento relativo, la fuerza de roce es
de origen cinemtico y se tiene que
a

Figura 4.46

Fr = FN .
La fuerza que el bloque m ejerce sobre la cua viene dada por FN Fr . La compon
nente horizontal de esta fuerza nos entrega la ecuacin de movimiento
o
Fr cos + FN sin = M b
donde b es la magnitud de la aceleracin de las cuas. Las dos aceleraciones no son
o
n
independientes sino que estn relacionadas por
a
a
= tan .
b
Tenemos cuatro ecuaciones con cuatro incgnitas. Despejando la aceleracin a obteo
o
nemos
M cos cos + sin 1
a=g 1+2
.

m sin sin cos

Para = tan , la aceleracin es nula.


o
Solucin al problema 42
o

Durante el primer tramo de largo D la part


cula acelera uniformemente y despus
e
desacelera uniformemente quedando en reposo despus de recorrer otro tramo de
e
largo D. Es evidente que la aceleracin durante el segundo tramo debe tener la misma
o
magnitud que en el primero, siendo el signo el opuesto. En otras palabras, la fuerza
neta F1 que acta sobre la masa en el tramo 1 debe ser la opuesta de la fuerza neta
u
en el tramo 2, F2 .
En el primer tramo la unica fuerza a lo largo del plano inclinado es la componente

en esa direccin del peso, esto es, F1 = mg sin .


o
En el segundo tramo aparece adicionalmente la fuerza de roce fr . Esta, para dar
una fuerza neta F2 = F1 debe ser
fr = 2 F1 = 2mg sin .

CAP
ITULO 4. LAS LEYES DE NEWTON
Por otra parte, la fuerza de roce (cinemtica) es
a
fr = c mg cos .
Igualando las dos expresiones para fr y despejando c se obtiene, nalmente,
= 2 tan .

186

Cap
tulo 5

Trabajo y energ
a
versin 28 mayo 2012
o

Podemos considerar los cap


tulos anteriores como un progresivo camino de simplicacin en la descripcin del movimiento. Primero, observamos que, para describir
o
o
el movimiento de una part
cula, requerimos conocer su posicin, velocidad y aceleo
racin. En una dimensin stas son tres funciones del tiempo; en tres dimensiones,
o
o e
nueve. Eventualmente, introduciendo Matemtica adicional (derivadas e integrales),
a
aprendemos que no necesitamos tantas funciones, sino slo tres (digamos, las posio
ciones x(t), y(t) y z(t) para todo tiempo). E introduciendo el concepto adicional de
vectores, podemos reducir esto a una sola funcin, el vector posicin para todo tiemo
o
po. A continuacin aprendimos que, en realidad, no necesitamos conocer la posicin,
o
o
o la aceleracin, para todo tiempo, sino la fuerza. Tambin es un vector, y por lo
o
e
tanto tambin son tres funciones, como la posicin, pero la fuerza no necesariamente
e
o
depende del tiempo, y es por tanto un punto de partida mucho ms sencillo para
a
describir el movimiento.
En este cap
tulo aprenderemos que es posible hacer una simplicacin adicional:
o
en realidad tampoco necesitamos la fuerza, que equivale a tres funciones escalares,
sino que basta con una unica funcin escalar: la energ Esto permitir una gran sim
o
a.
a
plicacin en la descripcin del movimiento, una simplicacin que dar pie, eveno
o
o
a
tualmente, a una formulacin mucho ms elegante de la Mecnica. Y no slo ms
o
a
a
o
a
elegante, sino tambin ms poderosa y generalizable a otros mbitos de la F
e
a
a
sica. En
este curso, sin embargo, nos limitaremos a explorar las consecuencias ms bsicas
a a
de la simplicacin y elegancia que mencionamos. El resto, ser tema de cursos ms
o
a
a
avanzados.

187

CAP
ITULO 5. TRABAJO Y ENERG
IA

5.1.

188

Trabajo y energ para movimientos en una dimena


sin
o

Consideremos una part


cula de masa m, restringida a moverse a lo largo del eje z ,

siendo z(t) y v(t) la posicin y velocidad de la part


o
cula a medida que transcurre el
tiempo. En particular, sean zi y vi la posicin y velocidad de la part
o
cula en el instante
ti , y zf y vf la las mismas magnitudes en el instante tf . Supongamos adems que, a
a
medida que la part
cula se traslada, ejercemos sobre ella una fuerza F (z), fuerza que
podr depender de la posicin z.
a
o
Analicemos varios casos:
a) Si la part
cula, excepto por la fuerza que le estamos aplicando, es libre, entonces
acelerar. Si la fuerza F (z) = F0 es constante (es decir, no depende de la
a
posicin), entonces la aceleracin tambin lo ser, tenindose F0 = ma0 . De
o
o
e
a
e
acuerdo a la cinemtica de un objeto uniformemente acelerado, en el intervalo
a
de tiempo [t, t + t], la posicin y velocidad de la part
o
cula en el instante t + t
sern
a
1
z(t + t) = z(t) + v(t) t + a0 (t)2 ,
2
v(t + t) = v(t) + a0 t .
Las variaciones de la posicin y la velocidad son, entonces,
o
1
z = z(t + t) z(t) = v(t) t + a0 (t)2
2
y
v = v(t + t) v(t) = a0 t .

Multipliquemos ahora la primera ecuacin por F0 . Nunca hemos necesitado


o
hasta ahora la cantidad F0 z, pero ya veremos la utilidad de hacerlo. Usemos,
adems, consecutivamente, la segunda ley de Newton y la expresin para v.
a
o
De esta manera se obtiene
1
F0 z = F0 v(t) t + F0 a0 (t)2
2
1
= m a0 v(t) t + m (a0 t)2
2
1
= m v(t) [ v(t + t) v(t) ] + m [ v(t + t) v(t) ]2
2
1
1
F0 z = m v 2 (t + t) m v 2 (t) .
2
2
Como la fuerza es constante, las ecuaciones anteriores son vlidas para cualquier
a
t, en particular para t = ti y t + t = tf , en cuyo caso
1
1
2
2
F0 (zf zi ) = mvf mvi .
2
2

CAP
ITULO 5. TRABAJO Y ENERG
IA

189

Observemos con cuidado este resultado: slo conocemos la posicin inicial, la


o
o
posicin nal, y la fuerza, que es constante. Sabemos que el movimiento eno
tre zi y zf es complicado (la velocidad aumenta linealmente con el tiempo, la
posicin aumenta cuadrticamente), pero nada de eso es relevante ahora. Podeo
a
mos armar, slo con esta informacin, que si escogemos dos puntos arbitrarios
o
o
de la trayectoria, zi y zf , la cantidad F0 (zf zi ) ser igual a la diferencia de
a
una funcin, la misma funcin (mv 2 /2), evaluada en dichos puntos. Esto es,
o
o
en cierto modo, extraordinario. En la bsqueda de una descripcin adecuada
u
o
del movimiento hab
amos concluido que tomar dos puntos arbitrarios de una
trayectoria es insuciente, vindonos en la necesidad de denir velocidad y acelee
racin instantneas, y en denitiva, introducir los elementos de toda un rea de
o
a
a
la Matemtica, el Clculo Innitesimal. Ahora, sin embargo, estamos diciendo
a
a
exactamente lo opuesto: a usted le basta tomar el principio y el nal de la trayectoria, eventos que pueden estar arbitrariamente separados en el tiempo. Se
podr objetar que este resultado es restringido, pues no estamos describiendo
a
el movimiento completo, sino slo calculando algo muy particular, F0 (zf zi ),
o
y adems para un caso en que la fuerza es constante. Veremos, sin embargo,
a
que ambas objeciones se pueden responder satisfactoriamente, y por tanto, el
resultado que acabamos de obtener es el punto de partida de la formulacin ms
o
a
elegante de la Mecnica que hemos sugerido.
a
Como dijimos, la cantidad F0 (zf zi ) resulta ser igual a la diferencia de cierta
funcin, evaluada en ambos extremos de la trayectoria. A dicha funcin la llao
o
mamos energa. Espec

camente, como en este caso dicha energ depende de la


a
velocidad, la denominaremos energa cintica, denotndose habitualmente con

e
a
las letras K o T . Por otra parte, a la cantidad F0 (zf zi ), que es el producto entre la fuerza y el desplazamiento, se le denomina trabajo, y se le denota
usualmente con la letra W . Es decir, el resultado para una part
cula sometida
a una fuerza constante se puede reescribir
Wzi zf = Kf Ki .
En el lenguaje del cap
tulo anterior, habr
amos dicho que ejercer una fuerza
sobre una part
cula se traduce en un cambio de velocidad de la misma. Ahora,
decimos que ejercer un trabajo sobre una part
cula se traduce en un cambio
de su energ cintica. El trabajo realizado es, de hecho, exactamente igual al
a
e
cambio de su energ cintica. Independiente de cunto tiempo le tome.
a
e
a
En F
sica, trabajo y energ es escencialmente lo mismo. Son conceptos distina
tos, pero al menos tienen las mismas unidades: en el sistema internacional de
unidades SI, mv 2 /2 y F0 z tienen unidades de newton metro, que denimos
como joule (J):
1 joule = 1 newton metro ,

CAP
ITULO 5. TRABAJO Y ENERG
IA
o sea,
1J=1Nm=1

190

kg m2
.
s2

b) Si la fuerza no es constante, entonces subdividamos la trayectoria de la part


cula
en N intervalos de tamao z. Denotemos las distintas posiciones por z1 , z2 ,
n
z3 , . . ., zN , zN +1 , siendo zi = z1 y zf = zN +1 . Si en cada intervalo j la fuerza
se mantiene relativamente constante, podemos usar el resultado de la parte a),
o sea,
Wzj zj+1 = F (zj ) (zj+1 zj ) = F (zj ) z = Kj+1 Kj ,
siendo natural denir el trabajo total como la suma de los trabajos ejercidos
sobre la part
cula en cada intervalo de tiempo.
Sumando la contribucin de todos los intervalos se obtiene
o
N
j=1

F (zj ) z = (K2 K1 ) + (K3 K2 ) + (K4 K3 ) + + (KN +1 KN )


= KN +1 K1 ,

o sea,
N

Wzi zf =

j=1

F (zj ) z = Kf Ki .

La expresin anterior es exacta en el l


o
mite N , de modo que el tamao
n
de los intervalos z se torna innitesimalmente pequeo. En este caso, z es
n
un dz, y la sumatoria es una integral, tenindose
e
f

Wzi zf =

F (z) dz = Kf Ki .

Observamos que hemos obtenido exactamente el mismo resultado que para una
fuerza constante, y por lo tanto lo que dijimos en el
tem anterior sigue siendo
cierto: independiente del tiempo que transcurra entre dos mediciones, independiente incluso de cmo var la fuerza entre dichas dos mediciones (y por lo
o
e
tanto el detalle de posiciones, velocidades y aceleraciones intermedias), cuando
calculamos el trabajo ejercido por la fuerza a lo largo de la trayectoria, dicho
trabajo resulta ser igual simplemente a la diferencia de una cierta funcin (la
o
energ cintica), evaluada en el punto nal y en el punto inicial.
a
e
c) Supongamos ahora que la part
cula no es libre, sino que est inmersa en un
a
campo gravitacional constante g = g. Levantemos la part
z
cula desde zi hasta zf , partiendo desde el reposo y volviendo a dejarla en reposo. Elevamos la

CAP
ITULO 5. TRABAJO Y ENERG
IA

191

part
cula aplicando una fuerza de manera que sta suba con una velocidad conse
tante. Mientras la part
cula va subiendo, su aceleracin es nula, luego tambin
o
e
la fuerza neta que acta sobre ella. De lo anterior se desprende que la fuerza
u
que debemos ejercer para elevar la part
cula es F (z) = +mg. El trabajo que
nosotros realizamos es la integral de esta fuerza a lo largo de la trayectoria, es
decir,
Wzi zf = +mg (zf zi ) .
Observamos que, nuevamente, el trabajo ejercido es igual a la diferencia de una
cierta funcin (mgz) evaluada en los puntos inicial y nal de la trayectoria.
o
Siguiendo la analog con el caso anterior, entonces, diremos que esta funcin
a
o

es tambin una energ (tiene las unidades adecuadas, por supuesto). Esta, sin
e
a
embargo, es una energ de carcter distinto. El trabajo no se ha traducido en
a
a
un cambio de energ cintica, porque como la fuerza neta es cero, la velocidad
a
e
es la misma en ambos extremos de la trayectoria. Sin embargo, es claro que algo
ha cambiado en la part
cula, porque despus de aplicada la fuerza la part
e
cula se
encuentra ms arriba. Diremos que lo que ha ocurrido es un cambio de energa
a

potencial, porque cambia la potencialidad de la part


cula para realizar trabajo
o de adquirir energ cintica.
a
e
En efecto, al dejar caer la part
cula sin restricciones desde zf hasta zi , adquirir una velocidad que, de acuerdo a las ecuaciones de la cinemtica de la ca
a
a
da
a
e
libre, es vf = 2g(zf zi ). Para esta velocidad, la energ cintica es
K=

1
1
2
mvf = m 2g(zf zi ) = mgzf mgzi .
2
2

Pero ste es exactamente igual al cambio de energ de la part


e
a
cula al elevarse.
Entonces podemos decir que, al ejercer trabajo sobre ella hasta elevarla una
altura zf zi , aunque no cambi su energ cintica, s cambi algo en ella,
o
a
e

o
porque ahora, si la dejamos libre, es capaz de llegar al punto inicial con una
velocidad mayor. Por ello decimos que la energ que cambia al elevarla es
a
potencial, que no se maniesta directamente.
Otra forma en que puede manifestarse esta potencialidad de la part
cula consiste en hacer que ella realice trabajo. Por ejemplo, podemos atarla a una cuerda
de modo que, por medio de un sistema de poleas, eleve otra masa.
Observemos, incidentalmente, que la manera en que describimos un sistema es
completamente distinta ahora que hemos introducido el concepto de energ
a:
ejercemos un trabajo sobre la part
cula, que se traduce en un cambio de su
energ potencial. Y al dejarla caer, esa energ potencial se traduce en energ
a
a
a
cintica al volver al suelo. En denitiva, el trabajo ejercido por nosotros se
e
convirti en energ cintica, a travs de sucesivas transformaciones.
o
a
e
e

CAP
ITULO 5. TRABAJO Y ENERG
IA

192

De acuerdo a lo desarrollado ms arriba, tambin en este ejemplo podemos


a
e
expresar el trabajo Wzi zf como una diferencia de cierta magnitud evaluada
en el punto nal menos la misma magnitud evaluada en el punto de partida:
Wzi zf = U (zf ) U (zi ) ,
donde U (z) es la energa potencial. (Se designa usualmente con las letras U o

V .) Para una part


cula de masa m en el campo gravitacional constante g = g,
z
que viene dada por
U (z) = U0 + mgz ,
con U0 cierta constante arbitraria, que depende de en qu punto diremos que
e
la energ potencial es nula. Veremos en breve que, cuando la part
a
cula est soa
metida a otros campos de fuerza, tambin puede denirse una energ potencial
e
a
respectiva. Por ello, podemos tambin especicar que la anterior corresponde a
e
la energa potencial gravitatoria.

Hemos introducido una constante U0 , ya que el trabajo slo depende de la


o
diferencia de la energ potencial entre dos puntos, y esta constante es, en
a
ese sentido irrelevante. U0 corresponde simplemente a la energ potencial de
a
la part
cula cuando sta se encuentra en z = 0. Cuando denimos energ
e
a
cintica, ya que tambin el trabajo depende slo de su diferencia, podr
e
e
o
amos
haber introducido una constante aditiva K0 . No lo hicimos, ya que tiene mucho
sentido decir que la part
cula, cuando est en reposo, tiene energ cintica cero.
a
a
e
En el caso de la energ potencial gravitatoria, en cambio, no existe ninguna
a
razn de peso para pensar que debe ser cero cuando z = 0. Por ejemplo, si
o
la energ potencial indica la capacidad de ejercer trabajo, o de adquirir
a
energ cintica al llegar al suelo luego de una ca libre, podr
a
e
da
amos pensar
en la siguiente situacin: digamos que z = 0 corresponde no al suelo, sino a
o
la supercie de una mesa que est a cierta altura h respecto al suelo. Si una
a
manzana se encuentra sobre la mesa, es cierto que si la hacemos rodar hasta el
borde de la mesa, y cae, adquirir una energ cintica. Eso signica que ten
a
a
e
a
una energ potencial en primer lugar. Por lo tanto, no ser correcto decir
a
a
que su energ potencial es cero sobre la mesa. Vemos, entonces, que la altura
a
z = 0 no tiene por qu coincidir con la altura z para la cual U (z) = 0, y por
e
ende U0 = 0 en general. Pero nada impide insistir en poner U (z = 0) = 0. En
el ejemplo de la manzana sobre la mesa, lo unico que sucede es que su energ

a
potencial en el suelo ser negativa, la diferencia de potencial ser negativa,
a
a
U (zf )U (zi ) = mgh, y por tanto la diferencia de energ cintica ser positiva,
a
e
a
K(zf ) K(zi ) = U (zi ) U (zf ) = mgh, y la velocidad nal de ca ser lo
da
a
esperado. O sea, nada extrao. Lo importante es tener presente que el cero de
n
las alturas no tiene por qu corresponder al cero de la energ potencial. Pero,
e
a
para todos los efectos prcticos, lo unico relevante en Mecnica Clsica son las
a

a
a

CAP
ITULO 5. TRABAJO Y ENERG
IA

193

diferencias de energ potencial, de modo que, si simplica las ecuaciones o es


a
conveniente por otras razones, no hay problema en poner U0 = 0.
Notemos nalmente otro aspecto importante: en el anlisis anterior, supusimos
a
que la part
cula sub con velocidad constante dese zi a zf , es decir, la fuerza
a
ejercida por nosotros es constante. Sin embargo, usando el mismo argumento
del caso b), si la fuerza no es constante basta dividir la trayectoria en intervalos
ms pequeos, donde la fuerza s sea constante. La conclusin ser la misma de
a
n

o
a
b): el trabajo total ser simplemente la diferencia de las energ potenciales en
a
as
el punto nal e inicial. No es importante que la part
cula suba con velocidad
constante, entonces: si parte del reposo, la elevamos, y la dejamos en reposo en
su posicin nal, el trabajo realizado es el mismo.
o
Ms an: en el proceso anterior lo unico importante es que el campo de fuerzas
a u

tenga asociada una energ potencial, para poder decir que, en cada intervalo,
a
F (z)z = U (zf ) U (zi ). Por lo tanto, para cualquier campo de fuerzas, podemos calcular el trabajo para llevar la part
cula de un punto a otro, para el
caso de fuerza aplicada constante, y los resultados seguirn siendo vlidos si la
a
a
fuerza no es constante.
d) Consideremos un resorte de constante
de restitucin k, acostado sobre una suo
percie horizontal sin roce y con un extremo empotrado en una pared (ver gura 5.1). Supongamos adems que el
a
sistema inicialmente se encuentra en reposo, con el resorte teniendo su largo
natural.

Figura 5.1

Evaluemos el trabajo que debemos realizar para alargar (lentamente) el resorte


en una magnitud x0 . La fuerza que debemos aplicar para lograr nuestro objetivo
ahora no es constante, sino que aumenta a medida que el resorte se estira:
F (x) = kx x .

(Esta fuerza es la opuesta a la que el resorte ejerce sobre la masa, que, de


acuerdo a la Ley de Hooke, es kx.) El trabajo que debemos realizar para
x
alargar el resorte, desde x = 0 hasta x = x0 , viene dado por

x=0

x0

x0

x=x0

W0x0 =

F (x) dx =

x dx .

kx dx = k
0

CAP
ITULO 5. TRABAJO Y ENERG
IA

194

Ya sabemos que la integral f (x) dx no


es otra cosa que el rea bajo la curva
a
del grco de la funcin f (x) . Para el
a
o
presente caso, la funcin corresponde a
o
una recta que pasa por el origen (ver
gura 5.2), siendo el rea bajo la curva
a
1 2
x0 .
2

Figura 5.2

Luego, el trabajo que debe realizarse para expandir el resorte hasta x0 es


1 2
kx .
2 0
(Se obtiene el mismo resultado si, en lugar de alargarlo, el resorte se comprime
en una magnitud x0 .)
W0x0 =

Si hubiramos desplazado el resorte desde una posicin arbitraria xi a otra xf ,


e
o
es claro que el trabajo realizado ser
a
Wxi xf =

1
k x2 x2
i
f
2

Una vez ms, el trabajo realizado es igual a la diferencia de una funcin, evaa
o
luada en los puntos extremos de la trayectoria.
Adems, tambin en este ejemplo, el trabajo realizado por nosotros sobre la
a
e
part
cula no se maniesta en un cambio de su energ cintica, sino lo que cambia
a
e
es el estado del sistema. En el nuevo estado, el sistema tiene la potencialidad
(al permitir que el resorte vuelva a su largo natural) de realizar trabajo, o
de entregarle a la part
cula adosada al resorte una energ cintica. Decimos
a
e
entonces que el trabajo realizado se traduce en un cambio de energ potencial.
a
Como en este caso el campo de fuerzas corresponde a un resorte, decimos que
es una energa potencial elstica.

a
Si la energ potencial de un resorte la
a
denimos por
1
U (x) = U0 + kx2 ,
2
donde x = 0 corresponde a la posicin
o
de equilibrio del resorte y x es la magnitud en que ste se comprime o se alarga,
e
entonces nuevamente
W0z0 = U (z0 ) U (0) .
Figura 5.3

CAP
ITULO 5. TRABAJO Y ENERG
IA

195

(La constante aditiva U0 en la expresin para la energ potencial nuevamente


o
a
no aparece en el resultado nal; la podr
amos haber elegido igual a cero. De
hecho as lo haremos en lo sucesivo, respetando la intuicin de que la energ

o
a
potencial de un resorte en su largo natural deber ser nula.) La gura 5.3
a
muestra el grco U (x) para la energ potencial de un resorte, correspondiendo
a
a
a una parbola.
a
e) Consideremos ahora una part
cula que se mueve a lo largo de una recta (el
eje x) sobre una mesa y supongamos que, adems de la fuerza que nosotros

a
ejercemos sobre ella, la unica otra fuerza se debe al roce (cintico). La fuerza

e
de roce es fr = mgc (pero note que sta aparece slo cuando la part
e
o
cula
se est moviendo y observe adems que la direccin en que apunta siempre es
a
a
o
contraria a la direccin de movimiento). El trabajo que debemos realizar para
o
empujar la part
cula, con velocidad constante, desde xi hasta xf es
W = fr (xf xi ) = (mgc ) (xf xi ) .
En este caso, sin embargo, el trabajo que realizamos no se maniesta en un
cambio de algo que podr
amos denominar energ potencial. (A pesar de que
a
el trabajo result, como en ocasiones anteriores, igual a la diferencia de una
o
funcin evaluada en los extremos de la trayectoria.)
o
Podemos comprobar la armacin anterior con un ejemplo sencillo: si la part
o
cula tiene inicialmente una velocidad v0 , al dejarla libre en xi se detendr, por
a
2
efecto del roce, al cabo de una distancia d = v0 /(2g). Si, en cambio, la empujamos con velocidad constante hasta xf , y la soltamos en este segundo punto,
la part
cula se detendr al cabo de la misma distancia d. Es decir, an cuando
a
u
efectuamos trabajo sobre la part
cula, el estado del sistema no cambia: al dejar
de aplicar la fuerza, el sistema hace lo mismo. No como en el caso gravitatorio
o elstico, en que liberar al sistema desde dos posiciones distinta signica que
a
la part
cula efectivamente adquiere mayor velocidad, ya sea al llegar al suelo o
al pasar por la posicin de equilibrio del resorte, respectivamente.
o
Vemos, entonces, que no es posible denir un concepto anlogo a la energ
a
a
potencial cuando se trata del roce.
De hecho, podemos distinguir dos problemas adicionales, aunque
ntimamente
relacionados con el anterior, que aparecen cuando hay roce y que hace que la
situacin sea distinta que en los dos ultimos casos:
o

i) En primer lugar, el trabajo que debe hacerse para llevar la part


cula desde
xi hasta xf , cuando hay roce, depende del camino que uno elija para
ello. Esto es profundo, y est directamente relacionado con el hecho de que
a
el trabajo no se pueda escribir, como vimos, como una magnitud que slo
o
dependa del punto inicial y nal. En efecto, supongamos que xf est a la
a

CAP
ITULO 5. TRABAJO Y ENERG
IA

196

derecha de xi . Ya vimos que, al llevar la part


cula directamente desde xi a
xf , el trabajo que debemos realizar es W = mgc (xf xi ). Pero si antes
la empujamos hacia la izquierda en una distancia L, y recin desde ah la
e

empujamos al punto nal xf , el trabajo ser W = mgc (2L + xf xi ).


a
O sea, el trabajo no slo depende del punto inicial y nal sino que tambin
o
e
del camino.
ii) Otra caracter
stica del trabajo que se hace contra el roce es que ste no es
e
recuperable como energ mecnica. Ya observamos que la masa tardar el
a
a
a
mismo tiempo en detenerse, independiente de cunto tiempo apliquemos
a
una fuerza, de modo que el trabajo que hemos hecho, en realidad, no se
ha convertido en nada util desde el punto de vista mecnico. Esto tambin

a
e
est relacionado con el hecho de que no hay ninguna energ potencial
a
a
que haya cambiado en el intertanto. Entonces el trabajo que efectuamos
simplemente desapareci? En realidad no, sino que se disipa como calor,
o
calentando las supercies en contacto. (En rigor s es posible, utilizando

una mquina trmica, recuperar la energ disipada, pero de todos modos


a
e
a
las leyes de la Termodinmica implican que la recuperacin del trabajo,
a
o
aunque se hiciera, ser slo parcial. Una vez que hay roce presente, siempre
a o
parte del trabajo realizado se va a disipar irremediablemente en forma de
calor no aprovechable mecnicamente.)
a
Los ejemplos unidimensionales anteriores sugieren lo siguiente:
Denicin: El trabajo realizado por una fuerza F (z) que acta sobre alguna part
o
u
cula
es
F (zj ) (zj+1 zj ) = F (z) dz ,
W =
j

donde la suma (o integral) se realiza a lo largo de la trayectoria que recorre la part


cula.
El trabajo W que se entrega a un sistema, cuando no hay roce, se maniesta en un
cambio de la energ del sistema. Hasta el momento hemos identicado las siguientes
a
formas de energ
a:

a) Energ cintica de una part


a
e
cula de masa m. Esta viene dada por
1
K = mv 2 ,
2
y se maniesta en el movimiento de la part
cula. Cuando la part
cula est en
a
reposo, su energ cintica es cero.
a
e

b) Energ potencial. Esta es una energ que se debe a la existencia de un campo


a
a
de fuerzas al cual la part
cula est sometida. En los ejemplos
a

CAP
ITULO 5. TRABAJO Y ENERG
IA

197

posicin de la part
o
cula. La energ potencial depende de la posicin, y slo
a
o
o
aparece cuando la part
cula no es libre, sino que est sometida a un campo de
a
fuerzas. La expresin para la energ potencial depende del campo de fuerzas
o
a
espec
co de que se trate. Hasta el momento hemos analizado dos casos:
i) Campo gravitacional uniforme, F (z) = mg = mg, en cuyo caso la
z
energ potencial es
a
U (z) = mg (z z0 ) ,
donde z0 es un lugar que arbitrariamente hemos jado como el cero de la
energ potencial.
a
ii) Campo de fuerzas de un resorte, F (x) = kx, en cuyo caso la energ
x
a
potencial es
1
U (x) = kx2 .
2
Cuando hay roce, parte (o toda) la energ entregada al sistema (por medio del
a
trabajo), puede disiparse. Esta energ se maniesta en un aumento de la temperatura
a
de las supercies que rozan entre s En este caso, el trabajo W se transforma en calor
.
Q.
Las observaciones anteriores se pueden resumir en el siguiente Principio de Conservacin:
o
Conservacin de la energ
o
a:
Al entregarle a una part
cula un trabajo W , entonces
W = (Kf Ki ) + (Uf Ui ) + Q ,

(5.1)

o sea, el cambio de la energ cintica, ms el cambio de la energ potencial, ms la


a
e
a
a
a
energ disipada como calor es igual al trabajo (energ aplicado al sistema.
a
a)
La ecuacin de conservacin de la energ hay que manejarla con precaucin, pues
o
o
a
o
no se puede estar seguro de que uno hayan identicado todas las posibles formas
de energ De hecho, a medida que avancemos en el estudio de la f
a.
sica, en varias
oportunidades nos veremos forzados a reinterpretar esa ecuacin o agregarle trminos
o
e
adicionales.
Lo importante es tener presente que la expresin anterior dice algo extraordio
nariamente importante: que aplicar trabajo sobre un sistema f
sico signica alterar
algo en el mismo, un algo que llamamos energ y que el cambio de energ es
a,
a
exactamente igual al trabajo realizado. A menos que haya roce, en cuyo caso el calor
disipado es exactamente igual a la diferencia entre el trabajo efectuado y el cambio
de energ
a.

CAP
ITULO 5. TRABAJO Y ENERG
IA

198

Este Principio de conservacin de la energa es una armacin muy general acerca


o

o
de los sistemas f
sicos, mecnicos o no. Insistimos en la importancia de notar que este
a
principio de conservacin no dice, tal como est formulado, que la energ no cambia,
o
a
a
sino algo ms general: la energ de un sistema slo puede cambiar si se efecta trabajo
a
a
o
u
sobre l.
e
Podemos, sin embargo, dejar el principio anterior en una forma ms familiar,
a
reescribiendo la ecuacin (5.1) tambin se de la siguiente manera:
o
e
Kf + Uf = Ki + Ui + W Q .
A ambos lados aparece la suma K + U , evaluada al inicio o al nal de la trayectoria.
Denimos entonces la energa mecnica,

a
E =K +U .
Ms an, podemos notar lo siguiente: para un sistema cerrado (es decir, un sistema
a u
sobre el cual no hay fuerzas externas) sin roce,
Ef = Ei ,

(5.2)

es decir, la energa mecnica se conserva.

a
Es importante distinguir que lo anterior se cumple slo para sistemas cerrados.
o
Ojo: esto no signica que no haya fuerzas presentes. De hecho, la energ potencial
a
contiene informacin sobre fuerzas presentes en el sistema. Pero un sistema cerrado
o
es aqul sobre el cual no hay fuerzas externas actuando. Por ejemplo, consideremos
e
una pelota cayendo al suelo. La pelota no es un sistema cerrado, porque hay una
fuerza sobre ella, el peso, que la acelera. El peso efecta trabajo sobre la pelota, y
u
naturalmente se produce un cambio de su energ (cintica en este caso). Si miramos
a
e
slo la pelota, concluimos que su energ no se conserva. Pero si consideramos ahora
o
a
el sistema compuesto por la pelota y la Tierra, el peso de la pelota es una fuerza interna, descrita en la energ potencial gravitatoria, y la energ mecnica se conserva.
a
a
a
(Naturalmente, todo esto depende de qu pregunta estemos interesados en responder,
e
y qu aproximaciones sean relevantes, ya que en rigor s hay fuerzas externas sobre el
e

sistema Tierra-pelota, por ejemplo la atraccin gravitatoria del Sol, y si no podemos


o
despreciarla, entonces ya no ser un sistema cerrado.)
a
Pero volvamos a la ecuacin (5.2). El lado izquierdo depende slo del estado nal
o
o
del sistema, y el lado derecho depende slo del estado inicial del sistema. Vemos aqu
o
,
expl
citamente, lo que notamos en el primer ejemplo unidimensional que tratamos:
independiente del tiempo que medie entre dos eventos, independiente de los detalles
de la trayectoria, de aceleraciones y desaceleraciones, de avances y retrocesos durante
la misma, el concepto de energ nos permite ver el sistema de modo global, y armar
a
que, por encima de todos los detalles intermedios, hay algo en la evolucin del sistema
o
que no cambia, algo invariante, y eso es precisamente la energ del sistema. Ya que
a

CAP
ITULO 5. TRABAJO Y ENERG
IA

199

el instante inicial y el nal de la trayectoria son arbitrarios, la energ mecnica es la


a
a
misma en todo instante.
En F
sica, como veremos a lo largo de este curso y muchos otros en el futuro,
las leyes de conservacin son extremadamente importantes. Podemos intuir por qu,
o
e
ya que si, en medio de toda la complejidad y variabilidad de un sistema f
sico, que
naturalmente evoluciona constantemente, somos capaces de encontrar algo que se
mantiene invariante, ello nos acerca a una gran simpleza conceptual en la descripcin
o
del sistema.
Podemos apreciar lo anterior considerando una part
cula de masa m, en reposo
a una altura h del suelo. Deseamos calcular la velocidad v con la que llega al suelo.
Este problema lo hemos resuelto muchas veces, pero utilicemos el lenguaje que hemos
adquirido en este cap
tulo. Tomemos como estado inicial el instante en que la part
cula
justo comienza a caer desde el reposo, y el estado nal el instante en que llega al suelo.
Entonces sus energ mecnicas inicial y nal son
as
a
Ei = mgh ,
1
Ef = mv 2 ,
2
donde hemos supuesto que el cero de la energ potencial coincide con el nivel del
a
suelo. El principio de conservacin de la energ nos dice que, siendo el sistema cerrado
o
a

y no habiendo roce, Ei = Ef , de lo que se sigue que v = 2gh, el resultado esperado.


Por supuesto que deb ser el mismo resultado de siempre. Pero notemos que, en
a
el lenguaje de fuerzas (Cap. 4), la manera de enfrentar este problema es escribir la
segunda ley de Newton con la fuerza correspondiente (el peso); luego integrar una vez
para obtener la velocidad; luego integrar de nuevo para obtener la posicin; imponer
o
las condiciones iniciales de posicin y velocidad; calcular, de la ecuacin de itinerario
o
o
para la posicin, el tiempo que tarda la part
o
cula en llegar al suelo; reemplazar dicho
tiempo en la ecuacin de itinerario para la velocidad, para obtener nalmente la
o
velocidad de ca
da.
Es claro que la descripcin en trminos de energ es enormemente ms sencilla.
o
e
a
a
Esto no es magia ni casualidad, sino que se debe precisamente al poder contenido en
haber encontrado una cantidad conservada (la energ en el sistema.
a)
Problema resuelto en clases: 5.4
En los ejemplos anteriores hemos resuelto las velocidades nales de un cuerpo en
ca libre o en una mquina de Atwood (problema 5.4), usando la conservacin de
da
a
o
energ entre un determinado estado inicial y otro nal, donde la separacin temporal
a
o
entre ellos es arbitraria. Esto ha sido una gran ventaja, y nos da una manera muy
sencilla de resolver problemas. Podr
amos pensar, sin embargo, que hemos perdido
informacin, precisamente porque ya no hay informacin sobre el tiempo y la evolucin
o
o
o
detallada del sistema para llegar desde dicho estado inicial al nal.

CAP
ITULO 5. TRABAJO Y ENERG
IA

200

No es as La clave es notar que, precisamente porque la energ es constante,


.
a
podemos usar ese hecho tanto para dos eventos arbitrariamente alejados en el tiempo,
como para dos eventos arbitrariamente cercanos. Tenemos un modo de decir eso en
lenguaje matemtico: que la energ sea constante, signica que su derivada temporal
a
a
es cero, es decir
dE
=0.
(5.3)
dt
Consideremos, por ejemplo, la part
cula anterior en ca libre. Para un instante
da
de tiempo arbitrario, su energ mecnica ser la suma de sus energ cintica y
a
a
a
a
e
potencial:
1

E(t) = mz(t)2 + mgz(t) .


2
Entonces, (5.3) da
z(m + mg) = 0 .
z
Puesto que en general z = 0, la ecuacin anterior equivale a

o
m = mg
z

...

que es precisamente la segunda ley de Newton para esta part


cula!
Es decir, la conservacin de la energ no slo permite resolver problemas con
o
a
o
gran elegancia, gracias a que podemos comparar instantes arbitrariamente separados
en el tiempo sin preocuparnos de los detalles intermedios. Adems, la conservacin
a
o
de la energ es equivalente a la segunda ley de Newton, y por lo tanto tiene toda
a
la informacin que necesitamos para describir el movimiento en todo instante si lo
o
requerimos.
Por cierto, hemos mostrado lo anterior slo para un caso particular, la ca libre.
o
da
Es fcil convencerse de que (5.3) tambin da las ecuaciones de movimiento correctas
a
e
para cada masa en la mquina de Atwood, y en muchos otros sistemas. Podremos
a
mostrar que efectivamente es as para un sistema arbitrario (bajo ciertas restriccio
nes) un poco ms adelante en este Cap
a
tulo, pero antes necesitamos generalizar los
resultados obtenidos anteriores al caso de tres dimensiones.

5.2.

Trabajo para un movimiento en tres dimensiones

Consideremos ahora una part


cula libre de masa m que se mueve en el espacio
tridimensional y cuya posicin y velocidad en el instante t son r y v, respectivamente.
o
Apliquemos sobre esa part
cula, durante un intervalo de tiempo innitesimal dt, una
fuerza F .
De acuerdo a la segunda ley de Newton,
F = ma .

CAP
ITULO 5. TRABAJO Y ENERG
IA

201

Durante el intervalo de tiempo innitesimal dt, la part


cula se desplaza una distancia
dr = v dt .
Puesto que ahora la fuerza y el desplazamiento son vectores, es natural pensar que la
cantidad equivalente a F (z)dz en el caso unidimensional est dada por el producto
a
punto entre ellos.
Haciendo pues el producto punto entre la fuerza y el desplazamiento, se obtiene
F dr = mv a dt .

(5.4)

Para evaluar el producto punto v(t) a(t). La aceleracin, por su parte, hace que la
o
velocidad en un tiempo posterior sea distinta, v(t + dt). Inspirados por los resultados
unidimensionales, que representan el trabajo con la energ cintica, consideremos la
a
e
velocidad al cuadrado en el instante t + dt. Se tiene
v 2 (t + dt) = v(t + dt) v(t + dt)

= (v(t) + a(t) dt) (v(t) + a(t) dt)

= v(t) v(t) + v(t) a(t) dt + a(t) v(t) + a(t) a(t) (dt)2

= v 2 (t) + 2v(t) a(t) dt + (trminos del orden de (dt)2 ) ,


e
o sea,

1
1
v(t) a(t) dt = v 2 (t + dt) v 2 (t) .
2
2
Con este resultado y la expresin (5.4) obtenemos que
o
1
1
F dr = mv 2 (t + dt) mv 2 (t) .
2
2

La ultima ecuacin nos indica que el cambio de energ cintica de una part

o
a
e
cula sobre
la cual acta una fuerza durante el intervalo de tiempo [t, t + dt] (pero por lo dems
u
a
es libre), es igual al producto punto de esa fuerza y el desplazamiento realizado por
la part
cula en ese mismo intervalo. Lo anterior sugiere denir el trabajo, en el caso
tridimensional, como el producto punto del vector fuerza y el vector desplazamiento.
Si el movimiento no ocurre durante un intervalo de tiempo innitesimal, sino
entre dos instantes ti y tf , podemos usar la ecuacin anterior siempre que el intervalo
o
se divida en muchos intervalos pequeos y luego se sumen los trabajos y los cambios
n
en la energ cintica de cada uno de los intervalos. De esta manera se obtiene
a
e
f

W =
i

F (r ) dr =

1
1
mv 2 (tf ) mv 2 (ti ) .
2
2

Este resultado es el anlogo tridimensional de la situacin considerada en la seccin


a
o
o
anterior, en las partes a) y b), para el movimiento unidimensional.

CAP
ITULO 5. TRABAJO Y ENERG
IA

202

Hacemos notar que el hecho de que lo unico que interesa para la energ son los

a
estados inicial y nal es robusto, y sigue siendo cierto cuando el problema ocurre en
ms de una dimensin.
a
o
Denicin: El trabajo realizado por una fuerza F (r) que acta sobre alguna part
o
u
cula
viene dado por
W =

F (r ) dr ,

(5.5)

donde la integral se evala a lo largo del camino recorrido por la part


u
cula.
Tcnicamente, la integral anterior, en que el integrando es un vector a lo largo
e
de una trayectoria, se denomina integral de lnea. Los aspectos formales relacionados

con este tipo de integrales corresponder verlos en cursos superiores de Clculo, pero
a
a
para efectos de nuestro curso corresponde a un concepto muy sencillo. Consideremos
una part
cula desplazndose sobre una trayectoria arbitraria, en la direccin indicada
a
o
por el vector dr, como sugiere la gura:
F

dr
P

En el punto P , siente una cierta fuerza F . En general, el mdulo y la direccin


o
o
de F ser distinto en cada punto de la trayectoria. Para calcular el trabajo efectuado
a
al llevar la part
cula de A a B basta con hacer, en cada punto de la trayectoria, el
producto punto entre la fuerza y el desplazamiento dr, que es un vector tangente a la
trayectoria en dicho punto P . Dicho producto punto es un nmero real, y lo que queda
u
es simplemente sumar esos nmeros reales a lo largo de la trayectoria. (El siguiente
u
ejemplo mostrar cmo proceder en un caso concreto.)
a o
Volvamos a analizar el concepto energa

potencial para una part


cula inmersa en
un campo gravitatorio uniforme. Consideremos un objeto de masa m, en un
campo gravitatorio constante g = g
z
(el eje z apuntando hacia arriba), y eva
luemos el trabajo que debemos realizar
para trasladarlo (por jar ideas, con velocidad constante) desde el origen hasta el punto P , (x0 , z0 ) (ver gura 5.4).

Figura 5.4

CAP
ITULO 5. TRABAJO Y ENERG
IA

203

El primer problema que enfrentamos al hacer este problema en ms de una dia


mensin es que podemos ir desde el origen a P por distintos caminos. En rigor, esto
o
siempre fue cierto tambin en una dimensin: para unir dos puntos A y B podemos
e
o
ir en l
nea recta, pero si a mitad de camino nos devolvemos y luego retomamos el
curso original, eso ya es otro camino. Con los argumentos dados en la seccin anterior
o
podr
amos convencernos de que el trabajo es invariante antes cambios de direccin
o
dentro de la misma recta, pero ahora, en dos y tres dimensiones, ya no es tan evidente
y necesitamos considerar expl
citamente la posibilidad de tomar diferentes caminos
para llegar de un punto a otro.
Supongamos, para comenzar, que usamos el camino 1 , o sea, primero elevamos
el objeto desde z = 0 hasta z = z0 y luego lo trasladamos hacia el lado, hasta llegar
al punto P . Durante el primer tramo la fuerza que debemos realizar para elevar el
objeto (con velocidad uniforme) es F = mg, siendo el desplazamiento tambin a lo
z
e
largo del eje z , es decir, dr = dz z . Luego, para este primer tramo, el trabajo que

debemos realizar es
z=z0

z=z0

dz = mg z0 .

mg dz = mg

W0Q =

z=0

z=0

Para el segundo tramo la fuerza sigue siendo F = mg; el desplazamiento, sin embarz
go, ahora es a lo largo del eje x, es decir, dr = dx x. El producto punto entre la fuerza

y el desplazamiento es cero (por ser uno ortogonal al otro). Luego, para trasladar el
objeto desde Q a P no se requiere realizar ningn trabajo. Concluimos que el trabajo
u
total, a lo largo del camino 1 , es
W1 =

F (r ) dr = mgz0 .

Evaluemos ahora el trabajo que debemos realizar para llevar el mismo objeto desde
el origen al punto P a lo largo del camino 2 . La fuerza que debemos aplicar sigue
siendo F = mg; el desplazamiento, sin embargo, es a lo largo de la direccin del
z
o
vector unitario s, o sea, dr = ds s. Luego se tiene que

F dr = (mg z ) (ds s) = mg sin ds .

Concluimos que el trabajo total, a lo largo del camino 2 , viene dado por
W2 =

F (r ) dr = mg(sin )

ds = mgL sin ,
2

donde L es el largo del camino. Pero L sin = z0 , luego los trabajos a lo largo de los
caminos 1 y 2 son iguales.
No es dif demostrar que tambin el trabajo que se debe realizar para llevar
cil
e
el objeto desde el origen al punto P a lo largo del camino 3 es igual a mgz0 . En
efecto, para trasladar el objeto a lo largo de tramos horizontales no se requiere hacer

CAP
ITULO 5. TRABAJO Y ENERG
IA

204

trabajo, mientras que para los tramos verticales el trabajo siempre es mgh, donde h
es la diferencia de altura del tramo vertical.
De hecho, el mismo argumento nos sirve para convencernos de que para cualquier
trayectoria que una el origen con P , el trabajo ser siempre W = mgz0 . Para una
a
trayectoria arbitraria, siempre se podr escribir el vector desplazamiento dr en todo
a
punto como una componente vertical y otra horizontal. La componente horizontal
siempre dar un trabajo nulo, por ser ortogonal a la fuerza, y la vertical dar simplea
a
mente la integral unidimensional entre 0 y z0 , o sea mgz0 .
Por lo tanto, nos damos cuenta de que, igual que en el caso unidimensional, el
trabajo ejercido en presencia de este campo de fuerzas se puede escribir simplemente
como la diferencia de una cierta funcin evaluada en los extremos de la trayectoria. A
o
dicha funcin la podemos llamar energ potencial, y para una part
o
a
cula en la posicin
o
r = x + z es
x
z
U (x, z) = mgz + U0 ,
con U0 una constante que depende de dnde se ha escogido el cero de la energ
o
a
potencial (usualmente z = 0). La energ potencial es, entonces, independiente del
a
valor x.
El hecho de que el trabajo realizado sea independiente de la trayectoria escogida
es extremadamente no trivial e importante. De hecho que la razn misma por la cual
o
podemos denir una energ potencial es que no importa la trayectoria. En efecto, si
a
vemos una part
cula a una altura h respecto a cierto nivel de referencia, no tendr ni
a
siquiera sentido decir que su energ potencial es mgh, porque si la energ potencial
a
a
tiene que ver con el trabajo realizado, cmo sabemos cunta energ tiene si no
o
a
a
sabemos por dnde se fue? La unica respuesta posible es que slo tiene sentido denir
o

o
energ potencial cuando el trabajo es independiente de la trayectoria.
a
Cuando un campo de fuerza tiene la propiedad de que el trabajo realizado para
llevar una part
cula entre dos puntos cualesquiera, es independiente del camino usado
para unir tales puntos, entonces se dice que el campo de fuerzas es conservativo. En
el ejemplo anterior hemos mostrado que el campo gravitacional es un ejemplo de un
campo conservativo.
La fuerza de roce es un ejemplo de una fuerza no conservativa. Al empujar un
cajn de masa M por el suelo de una habitacin de un lugar a otro, el trabajo realizado
o
o
ser proporcional al largo L del camino que para ello se elige, siendo W = c M gL. Al
a
no ser el roce una fuerza conservativa, no se puede introducir una energ potencial
a
para esta fuerza (ya que no existe una funcin que slo dependa de los puntos nal
o
o
e inicial, y cuya diferencia sea igual al trabajo). El trabajo que se realiza contra la
fuerza de roce se transforma en calor. La reconversin de energ calrica a energ
o
a
o
a
mecnica puede hacerse slo recurriendo a alguna mquina y, aun as no en forma
a
o
a
,
completa.
Otro modo de ver lo anterior es notar que si una part
cula parte de una posicin
o
inicial r0 , sigue una cierta trayectoria, y vuelve a r0 , el trabajo necesario en un campo

CAP
ITULO 5. TRABAJO Y ENERG
IA

205

conservativo es cero. Es decir, para cualquier trayectoria cerrada,


r0

Wr0 r0 =

r0

F dr = 0 .

r0

En efecto, siendo conservativo, el trabajo no depende de la trayectoria, y por tanto


salir de r0 y volver a r0 es equivalente a la trayectoria consistente en no salir nunca
de r0 , para lo cual evidentemente no hay que hacer ningn trabajo.
u
Por ejemplo, en un campo gravitatorio, elevar una masa una altura h signica
ejercer un trabajo mgh, y devolverse al nivel inicial implica un trabajo mgh, de
modo que el trabajo total es cero. Pero al arrastrar un bloque sobre una supercie
horizontal con roce una distancia L, el trabajo a realizar es c mgL, y al volver a
la posicin original es nuevamente c mgL, ya que tanto la fuerza como el desplao
zamiento cambian de signo, de modo que el trabajo total no es nulo, y por tanto el
campo de roce no es conservativo.

5.3.

Potencia

Al trabajo por unidad de tiempo se denomina potencia. En el sistema SI, la unidad de


la potencia se denomina watt [W] y corresponde a 1 joule por segundo, es decir,
1W = 1

J
s

Ejemplo: Considere un motor elctrico de 0.4 kW (esto corresponde a aproximadae


mente al motor de una juguera). Cunto tiempo tardar tal motor, mediante un
a
a
sistema de poleas, en levantar un automvil de 600 kg en un metro?
o
Solucin: El trabajo requerido para levantar el automvil es
o
o
W = mgh = 600 9.81 1

kg m2
6000 J .
s2

El motor es capaz de entregar 400 J por segundo (estamos suponiendo una eciencia
de un 100 %), luego, para realizar un trabajo de 6000 J tardar 6000/400 = 15 s.
a

CAP
ITULO 5. TRABAJO Y ENERG
IA

5.4.

206

Ejemplos

A continuacin ilustremos los conceptos anteriores aplicndolos en algunos proo


a
blemas concretos.
Ejemplo 1
Considere un bloque de masa M que incide con velocidad v0 sobre un resorte (ver
gura 5.5) y lo comprime. Cul ser la
a
a
mxima compresin que en algn instante
a
o
u
llega a tener el resorte?
Figura 5.5
El bloque, al comprimir el resorte, realiza trabajo. Este trabajo, que se transforma
en energ potencial del resorte, lo hace a costa de su energ cintica. La mxima
a
a
e
a
compresin se logra cuando el bloque llega a estar en reposo. En ese caso, toda la
o
energ cintica se habr transformado en energ potencial, o sea,
a
e
a
a
1
1
2
M v0 = kx2 .
2
2 0
En la ecuacin anterior, x0 es la mxima compresin que llega a tener el resorte.
o
a
o
Despejando x0 se encuentra que
x0 =

M
v0 .
k

(Insistimos en notar la gran elegancia con la cual podemos resolver este tipo de
problemas al usar la conservacin de la energ De otro modo, hubiramos tenido
o
a.
e
que escribir la fuerza sobre la masa debida al resorte, integrar una vez la ecuacin de
o
fuerza para obtener la velocidad, poner la condicin inicial, encontrar el tiempo T que
o
tarda el bloque en llegar al reposo, integrar nuevamente para encontrar la posicin en
o
funcin del tiempo, poner la condicin inicial correspondiente, y evaluar en el tiempo
o
o
T para encontrar nalmente la compresin del resorte. . . )
o
Ejemplo 2
Un bloque de masa m resbala por un
plano inclinado, partiendo del reposo
desde una altura h. Sea el ngulo
a
de elevacin y el coeciente de roce
o
cinemtico entre el bloque y el plano.
a
Con qu velocidad llegar el bloque al
e
a
pie del plano inclinado?

Figura 5.6

CAP
ITULO 5. TRABAJO Y ENERG
IA

207

Inicialmente el bloque tiene slo una energ potencial U = mgh (el cero de la energ
o
a
a
potencial lo hemos elegido en la base del plano inclinado). Al llegar el bloque abajo,
2
o
ste tiene slo energ cintica K = mvf /2. Usando el principio de conservacin de
e
o
a
e
la energ se tiene que la energ cintica nal debe ser igual a la energ potencial
a
a
e
a
inicial menos la energ disipada por el roce. Esta ultima es Q = m g (cos ) L =
a

m g h/ tan . Se tiene
1
mv 2 = mgh mgh/ tan ,
2 f
o sea,

.
tan
La ecuacin anterior es vlida si e tan . Si la condicin anterior no se cumple
o
a
o
la part
cula no resbala. Observe cmo, en el l
o
mite = 90 , la velocidad vf tiende al
resultado de la ca libre.
da
Observemos tambin que, en ausencia de roce, el resultado es simplemente vf =
e

2gh, el mismo resultado que en ca libre. Da lo mismo si el bloque cae verticalda


mente o por un plano inclinado, la velocidad nal de ca es, en mdulo, la misma.
da
o
Esto puede parecer sorprendente, pero no es sino una consecuencia de que el campo sea conservativo, de modo que el trabajo ejercido por el campo gravitatorio no
depende de la trayectoria, sino slo de la diferencia de elevaciones.
o
2
vf = 2gh 1

Problema resuelto en clases: 5.5

5.5.

Resultados utiles para sistemas conservativos

En esta seccin obtendremos un par de resultados extremadamente utiles, geneo

rales y poderosos relacionados con sistemas conservativos. Escencialmente, observaremos que, para tales sistemas, basta con conocer su energ potencial, y que ella es
a
suciente para conocer mucha informacin sobre la dinmica de un sistema conservao
a
tivo, an cuando no podamos resolver sus ecuaciones de movimiento expl
u
citamente.
Supongamos que tenemos un sistema del que slo sabemos la energ potencial.
o
a
Por supuesto, esto implica de inmediato que el sistema es conservativo, pues de otro
modo ni siquiera podr
amos denir una energ potencial.
a
Por jar ideas, digamos que la energ potencial de una part
a
cula de masa m en
este sistema viene dada por la expresin
o
U (z) = a

1 3
z b2 z
3

(5.6)

donde a y b son ciertas constantes positivas. Encuentre el campo de fuerza F (z) que
da origen a tal energ potencial.
a

CAP
ITULO 5. TRABAJO Y ENERG
IA

208

Sea F (z) la fuerza que el campo ejerce sobre la part


cula. Para llevar la part
cula
desde el origen al punto z deberemos ejercer sobre la part
cula una fuerza de igual
magnitud pero sentido opuesto. Por consiguiente, el trabajo que debemos realizar es
z

W (z) =

F (z) dz .
0

Este trabajo es igual a la diferencia de la energ potencial entre el origen y el lugar


a
z, o sea, U (z) U (0). Como U (0) = 0, podemos igualar W (z) con U (z), obtenindose
e
z

U (z) =

F (z) dz = a
0

1 3
z b2 z
3

Como el proceso de integracin es el inverso del proceso de derivacin (ver guo


o
ra 5.7), se tiene que la fuerza debe ser menos la derivada de la energa potencial, o

sea,
dU (z)
.
(5.7)
F (z) =
dz
Usando esta relacin se encuentra que, para el presente problema, el campo de fuerzas
o
es
F (z) = a b2 z 2 .
(5.8)
La relacin (5.7) es muy importante. Como mencionamos en la introduccin a
o
o
este cap
tulo, la relacin (5.7) corresponde a la culminacin de un largo proceso, que
o
o
comenzamos al comienzo de este curso, por simplicar progresivamente la descripcin
o
del movimiento, pasando por las ecuaciones de itinerario, el clculo diferencial, el lgea
a
bra vectorial, las leyes de Newton y luego la conservacin de la energ Ahora vamos
o
a.
un paso ms all. Ya sab
a
a
amos que la energ mecnica contiene toda la informacin
a
a
o
sobre el sistema, pues dE/dt = 0 equivale precisamente a las leyes de Newton. Ahora
nos damos cuenta de que, en realidad, no es necesaria ni siquiera toda la energ
a
mecnica, sino slo una parte de ella, la energ potencial. Lo cual es evidente, pues
a
o
a
la energ cintica siempre tiene la misma forma, mv 2 /2, independiente del campo
a
e
de fuerzas. De modo que lo que hace realmente diferente a un sistema de otro es
su energ potencial. Y la ecuacin (5.7) dice lo mismo, matemticamente: basta la
a
o
a
energ potencial para conocer la fuerza, y una vez conocida la fuerza podemos aplicar
a
toda la maquinaria matemtica que ya conocemos para estudiar la evolucin de un
a
o
sistema f
sico.
Pero el concepto de energ potencial es extremadamente util no slo por lo
a

o
anterior. Graquemos la energ potencial (5.6).
a

CAP
ITULO 5. TRABAJO Y ENERG
IA

209

Figura 5.7

Figura 5.8
Una part
cula en un campo de fuerzas se dice que se encuentra en equilibrio si
la fuerza sobre ella es nula. Para una part
cula en el campo de fuerzas anterior, la
fuerza F (z) es nula cuando z = b [ver Ec. (5.8)]. En la gura 5.8 se han indicado
dichos puntos. Note que esos puntos coinciden con un mximo y un m
a
nimo de la
energ potencial. Esto tena que ser as por (5.7): siempre los puntos de equilibrio
a

,
correspondern a extremos de la curva de energ potencial. De modo que, aunque
a
a
el equilibrio se dene en trminos de la fuerza, ni siquiera es necesario calcular sta.
e
e
Basta simplemente con mirar la curva de energ potencial, y todos los mximos y
a
a
m
nimos corresponden a puntos de equilibrio.
Se dice que un sistema en equilibrio es estable si al alejarlo levemente del punto
de equilibrio la fuerza que aparece lo acelera nuevamente hacia dicho punto. De lo
contrario, el equilibrio es inestable. En otras palabras, el equilibrio es estable si, en la
vecindad del punto de equilibrio, la fuerza tiene signo opuesto al desplazamiento, y es
inestable si la fuerza tiene el mismo signo que el desplazamiento. Pero, nuevamente,

CAP
ITULO 5. TRABAJO Y ENERG
IA

210

esto es evidente simplemente observando la Fig. 5.8. En efecto, consideremos un punto


ligeramente a la derecha de z = b. En ese punto, la derivada de la energ potencial
a
es positiva, por lo tanto la fuerza es negativa [Eq. (5.7)], es decir, apunta hacia la
izquierda. As que efectivamente, la fuerza se opone al desplazamiento, evitando que

el sistema se aleje del punto de equilibrio. Lo mismo sucede si consideramos un punto


ligeramente a la izquierda de z = b. Para z = b, en cambio, sucede lo opuesto.
Si estamos ligeramente a la derecha, la derivada de la energ potencial es negativa,
a
luego la fuerza es positiva, apuntando por tanto hacia la derecha. De modo que la
fuerza tiene el mismo signo que el desplazamiento, y hace que el sistema se aleje an
u
ms del punto de equilibrio.
a
Concluimos, as que no slo la informacin sobre puntos de equilibrio, sino toda la
,
o
o
informacin sobre estabilidad e inestabilidad del equilibrio, se encuentra simplemente
o
observando la curva de la energ potencial.
a
Ms an, notemos que, para todos los efectos prcticos, la energ potencial
a u
a
a
representada en la Fig. 5.8 podr ser perfectamente un riel sobre el cual se desliza
a
una bolita sometida a un campo gravitatorio constante. Si el riel tiene exactamente la
forma de la gura, entonces la energ potencial de la bolita mientras circula por el riel
a
ser exactamente igual a (5.6), y por tanto la fuerza sobre la bolita ser precisamente
a
a
(5.8). Esto es extraordinario, porque en realidad no sabemos a qu corresponde la
e
fuerza (5.8). Podr ser cualquier cosa: la fuerza entre dos molculas, una fuerza entre
a
e
cargas elctricas distribuidas de alguna manera adecuada, una fuerza gravitacional
e
resultante por la presencia de cierta distribucin de masas. . . Pero no importa, sea cual
o
fuere el sistema f
sico espec
co, todos se comportan en algn sentido de la misma
u
manera, porque tienen la misma energ potencial. Y ahora dicho comportamiento
a
es completamente evidente, intuitivo, porque es claro de inmediato que si ponemos
una bolita en el riel de la Fig. 5.8, en los puntos z = b, la bolita se va a quedar
quieta (equilibrio). Y tambin es evidente que si la bolita se desplaza ligeramente
e
respecto a z = b, sta volver a su posicin original (equilibrio estable), as como
e
a
o

tambin es evidente que si la bolita se desplaza ligeramente respecto a z = b no lo


e
har (equilibrio inestable).
a
Hemos entonces ganado una intuicin poderos
o
sima. De ahora en adelante, seremos capaces de decir muchas cosas sobre un sistema simplemente observando la curva
de energ potencial. Y ni siquiera sabemos de qu sistema f
a
e
sico real se trata. En la
prctica, es lo mismo que si fuera una bolita sobre un riel en un campo gravitatorio
a
constante.
El ejemplo anterior corresponde a una situacin sencilla, donde el sistema evoluo
ciona slo en una dimensin. En ms dimensiones es lo mismo. La energ potencial
o
o
a
a
ya no ser una curva, sino una supercie, una sbana con cimas y valles, y en cada
a
a
uno de esos mximos y m
a
nimos sabremos que el sistema quedar en equilibrio, y que
a
las cimas sern puntos de equilibrio inestable, y los valles puntos de equilibrio estable.
a
Sin haber escrito ninguna ecuacin, e independiente del sistema f
o
sico espec
co de
que se trate.

CAP
ITULO 5. TRABAJO Y ENERG
IA

211

Supongamos ahora que la part


cula de masa m se encuentra en el punto de equilibrio estable y que en cierto instante la sacamos levemente de su punto de equilibrio,
para dejar que luego se mueva en el campo de fuerza. Encontremos expl
citamente la
fuerza que el campo ejerce sobre la part
cula.
Si alejamos la part
cula una distancia + desde el punto de equilibrio estable
z = b, (con b), entonces la fuerza que aparece es
F (b + ) = a b2 (b + )2 = a(2b + 2 ) 2ab .

As la fuerza en la vecindad del m


,
nimo tiene sentido contrario al desplazamiento,
y adems es proporcional al desplazamiento, con una constante de proporcionalidad
a
k = 2ab. Pero este tipo de fuerzas, F () = k, ya las hemos encontrado antes:
es la ley de Hooke, la misma fuerza de un resorte. Por lo tanto, este sistema f
sico
desconocido, que quizs corresponde a molculas interactuando, quizs corresponde
a
e
a
a estrellas en una galaxia, quizs corresponde a electrones en un campo elctrico, del
a
e
cual lo unico que conocemos es su energ potencial, este sistema f

a
sico arbitrario se
comporta, en la vecindad de su punto de equilibrio estable, exactamente igual a un
resorte.
Una observacin tremendamente importante, porque de nuevo signica que, sin
o
tener que resolver ninguna ecuacin, sin ni siquiera saber qu tipo de sistema f
o
e
sico
estamos estudiando, podemos al menos decir algo: cerca del punto de equilibrio estable, el sistema es equivalente a un oscilador armnico. . . el cual ya resolvimos! O sea,
o
podemos asegurar que el sistema oscilar en torno al punto de equilibrio estable, que
a
dicha oscilacin ser independiente de la amplitud inicial, y que tendr un per
o
a
a
odo
igual a
m
m
= 2
.
T = 2
k
2ab
Insistimos en que ste es un resultado independiente del sistema f
e
sico de que se
trate. De hecho z podr ser cualquier cosa: una posicin real, un ngulo, un ujo,
a
o
a
una corriente elctrica, etc. No importa, sea lo que sea, eso oscilar armnicamente
e
a
o
con per
odo T .
Es ms, podemos mostrar que lo anterior es cierto para un sistema completamente
a
arbitrario. En efecto, en la vecindad de un punto x0 , una funcin f (x) cualquiera,
o
pero razonablemente bien comportada en dicha regin (continua, todas sus derivadas
o
existen, etc.), tendr un desarrollo de Taylor de la forma
a
f (x) = f (x0 ) +

1 df
1! dx

x=x0

(x x0 ) +

1 d2 f
2! dx2

x=x0

(x x0 )2 +

(5.9)

Ahora bien, si f (x) corresponde ahora a una energ potencial U (x), y x0 a un punto
a
de equilibrio estable, la expresin anterior queda, en la vecindad de x0 , es decir, para
o
x = x0 + , con /x0 1,
U (x) = U (x0 ) +

dU
dx

+
x=x0

1 d2 U
2 dx2

2 ,
x=x0

(5.10)

CAP
ITULO 5. TRABAJO Y ENERG
IA

212

despreciando trminos de orden superior.


e
En (5.10), U (x0 ) es simplemente una constante; dU/dx F (x) es la fuerza
sobre la part
cula, que, siendo x0 un punto de equilibrio, se anula en dicho punto, y
por lo tanto el segundo trmino es cero. Y d2 U/dx2 |x=x0 es alguna constante, que de
e
hecho es la curvatura de la energ potencial en x0 . Pero si x0 es un m
a
nimo, entonces
la curvatura en x0 es positiva, de modo que podemos denir
d2 U
dx2

x=x0

k > 0.

De este modo, (5.10) se puede reescribir simplemente como


1
U () = U (x0 ) + k2 ,
2

(5.11)

que no es sino la energ potencial de un resorte de constante de restitucin k.


a
o
Lo anterior, matemticamente, no signica otra cosa que, cerca de un m
a
nimo,
cualquier curva diferenciable se puede escribir como una parbola. Pero f
a
sicamente,
signica que un sistema completamente arbitrario siempre, en la vecindad de un

punto de equilibrio estable, oscilar armnicamente. Esta es la razn por la cual el


a
o
o
oscilador armnico, siendo un problema en principio sencillo, es extraordinariamente
o
importante en toda la F
sica: porque estudiando el oscilador armnico, el cual se
o
puede resolver analticamente, se gana intuicin y se obtienen resultados vlidos para

o
a
cualquier otro sistema f
sico, incluyendo sistemas (la abrumadora mayor que no se
a)
pueden resolver exactamente, bastando para ello que tengan algn punto de equilibrio
u
estable. Y es la razn tambin por la cual, ms adelante en este curso, dedicaremos
o
e
a
un cap
tulo completo a estudiar solamente el oscilador armnico.
o
Problema resuelto en clases: 5.16
Problema resuelto en clases: 5.26

CAP
ITULO 5. TRABAJO Y ENERG
IA

213

Hemos hecho hincapi en que el concepto de energ potencial tiene toda la informacin
e
a
o
necesaria para describir la dinmica de un sistema conservativo. Sin embargo, hay un detalle
a
que no hemos abordado: las ecuaciones de Newton son vectoriales, y la conservacin de
o
energ es una ecuacin escalar. En otras palabras, la energ potencial es una funcin
a
o
a
o
escalar, pero la fuerza es un vector, y por tanto no es evidente que la energ potencial
a
tenga realmente toda la informacin necesaria. Si el problema es unidimensional, es claro
o
que (5.7) es suciente, y as ha sido en los ejemplos que hemos estudiado en este cap

tulo,
pero qu sucede en dos o tres dimensiones? Cmo es posible que una funcin escalar
e
o
o
reemplace completamente a una funcin vectorial?
o
Lo que hay que tener claro es que la expresin (5.7) viene en realidad de (5.5), la cual
o
relaciona una cantidad vectorial (la fuerza) con una escalar (el trabajo). (5.7) corresponde
a la inversin de la ecuacin (5.5); uno esperar entonces, que sea posible modicar (5.7)
o
o
a,
para recuperar una expresin vectorial. Para hacerlo necesitamos matemtica que an no
o
a
u
hemos visto en este curso, pero podemos contar que, en ms de una dimensin, la derivada
a
o
en (5.7) debe ser reemplazada por el gradiente, que es una especie de derivada vectorial:
F (r ) = U (r ) ,

(5.12)

donde, dada una funcin g(r ),


o
g(r ) =

g
g
g
x+

y+

z.

x
y
z

(5.13)

El operador diferencial g/x se denomina derivada parcial de g respecto a la coordenada


x, y signica simplemente lo siguiente: g es una funcin del vector r, es decir, de las tres
o
coordenadas x, y y z. g/x signica derivar g respecto a x como si fuera la unica variable,

considerando y y z constantes. Anlogamente para los otros dos operadores, g/y y g/z.
a
Lo importante es tener presente que, an en tres dimensiones, es cierto que una unica
u

funcin escalar (la energ potencial) contiene toda la informacin necesaria, a travs de
o
a
o
e
(5.12) en el caso general, o (5.7) en el caso unidimensional. Por lo tanto, el concepto
de energ potencial efectivamente nos permite describir un sistema f
a
sico de un modo
much
simo ms sencillo.
a

5.6.

Problemas

1. Una masa de 2 kg se lleva desde un punto


A al punto B. Los vectores de posicin de
o
los puntos A y B son:
xA = ( + 3) m
x
z
xB = 5 m
x
Todo el sistema est inmerso en un cama
po gravitatorio constante g. Encuentre
z
el trabajo realizado por la gravedad a lo
largo de cada uno de los tres caminos indicados en la gura adjunta.

Figura 5.9

CAP
ITULO 5. TRABAJO Y ENERG
IA

214

2. Una bomba de agua debe elevar 200 litros de agua por minuto desde un pozo,
cuyo nivel de agua est a 6 m de profundidad, para luego lanzarla con una
a
velocidad de 9 m/s. Suponiendo que no hay prdidas de energ de ningn tipo,
e
a
u
qu trabajo por minuto debe realizar el motor que acciona la bomba? Cul
e
a
es la potencia del motor?
(Una mquina que realiza un trabajo de 1 Joule = 1 kg m2 /s2 por segundo,
a
tiene una potencia de 1 Watt = 1 [W].)
3. Sobre una part
cula de masa m = 0.25 kg, que se mueve a lo largo del eje x,

acta una fuerza F = F (x) x, donde la magnitud F (x) depende de la posicin


u

o
x del modo indicado en la gura 5.10.
a) Determine el trabajo realizado por esta fuerza sobre la part
cula si ella se
traslada desde x = 0 a x = 3 m.
b) Si la part
cula en el instante t = 0 se encuentra en reposo en x = 2 m,
qu velocidad tendr al llegar a x = 6 m?
e
a

Figura 5.10
Respuestas: a) 4 J; b) v=4 m/s.
4. El sistema mostrado en la gura adjunta se abandona, partiendo del reposo,
cuando el bloque de masa m1 est a una distancia d por encima del suelo.
a
Desprecie el roce.

CAP
ITULO 5. TRABAJO Y ENERG
IA

215

a) Encuentre la aceleracin de la masa


o
mayor. (m1 > m2 .)
b) Usando el resultado de la parte (a),
encuentre la velocidad con que la
masa mayor llega al suelo.
c) Suponiendo que todo el trabajo realizado sobre el sistema se transforma
en energ cintica, calcule la velocia
e
dad de la masa mayor justo antes de
que choque contra el suelo.

Figura 5.11

5. Considere un cuerpo compuesto de N masas mj , situados en los lugares rj ,


con j = 1, 2, 3, . . . , N . Demuestre que la energ potencial de tal cuerpo, en un
a
campo gravitacional constante, se puede evaluar suponiendo que toda su masa
M = m1 + m2 + + mN est concentrada en su centro de masas, dado por
a
rcm

1
(m1 r1 + m2 r2 + + mN rN ) .
M

6. Un bloque cuya masa es m = 6 kg se desliza hacia abajo por un plano inclinado


rugoso, partiendo del reposo. El ngulo de elevacin del plano es = 60 y los
a
o
coecientes de roce esttico y cinemtico son 0.2 y 0.18, respectivamente.
a
a
a) Describa todas las fuerzas que actan sobre el bloque y determine el trabajo
u
realizado por cada una de ellas, si el bloque se desliza 2 m (a lo largo del
plano).
b) Cul es el trabajo neto realizado sobre el bloque?
a
c) Cul es la velocidad del bloque despus de recorrer una distancia de 2 m?
a
e
Resuelva el problema dos veces: la primera suponga que el sistema consiste slo
o
del bloque y, por lo tanto, las fuerzas de roce son parte de las fuerzas externas;
la segunda vez suponga que el sistema consiste del bloque y el plano inclinado,
en cuyo caso la disipacin de energ por las fuerzas de roce deben considerarse
o
a
como calor.

CAP
ITULO 5. TRABAJO Y ENERG
IA

216

7. Se desea levantar lentamente una masa M


hasta una altura h, usando el sistema de
poleas mostrado en la gura adjunta.
a) Cul es la fuerza que debe aplicara
se?
b) Qu trabajo se realiza?
e
c) Cul es el cambio en energ potena
a
cial de la masa?

Figura 5.12

8. Un bloque de m = 5 kg se sujeta contra un resorte de constante k = 1000 N/m,


comprimindolo en d0 = 8 cm. Al dejar el bloque en libertad, el resorte al
e
expandirse empuja el bloque a lo largo de una supercie horizontal rugosa con
coeciente de roce cinemtico = 0.2.
a
a) Cul es el trabajo realizado por el resorte sobre el bloque mientras el
a
resorte se extiende desde la posicin comprimida hasta la posicin de equio
o
librio?
b) Cul es el trabajo realizado por el roce sobre el bloque cuando ste recorre
a
e
los 8 cm hasta la posicin de equilibrio?
o
c) Cul es la velocidad del bloque cuando el resorte pasa por su posicin de
a
o
equilibrio?
d) Si al pasar por la posicin de equilibrio el bloque se despega del resorte,
o
qu distancia alcanzar a recorrer antes de detenerse?
e
a
e) Si el bloque se mantiene sujeto al resorte, cul es la extensin mxima
a
o
a
que llegar a tener el resorte?
a

CAP
ITULO 5. TRABAJO Y ENERG
IA

217

9. Un pndulo de masa m colgado de un hilo de largo , se eleva hasta formar un


e
a
ngulo 0 = 90 con la normal y luego se deja en libertad.
a) Encuentre la energ cintica de la masa pendular cuando el pndulo pasa
a
e
e
por su posicin de equilibrio.
o
b) Demuestre que la tensin de la cuerda, para = 0 , es 3 veces el peso de
o
la masa pendular.
10. Considere el campo de fuerza dado por
x
F (r ) = F0 x + F0 y .

a
Evale el trabajo que debe realizarse para
u
llevar una part
cula de masa m desde el
origen hasta el punto A a lo largo de los
dos caminos indicados en la gura adjunta. El campo de fuerzas es conservativo?

^
y
y0

A
2

x0

^
x

Figura 5.13

11. Una caja, de masa 10 Kg, descansa sobre la cubierta horizontal de una mesa.
El coeciente de friccin entre la caja y la supercie de la mesa es 0.4. En
o
cierto instante se aplica sobre ella una fuerza F = F0 x, adquiriendo la caja una

2
aceleracin constante a = 2 x [m/s ].
o

a) Determine F0 .
b) Determine el trabajo realizado por la fuerza F cuando la caja se ha trasladado una distancia de 5 m.
c) Evale la diferencia entre el trabajo realizado sobre la part
u
cula y el calor Q
disipado por el roce. Demuestre que esta diferencia coincide con la energ
a
cintica nal de la caja.
e
12. Una masa m resbala, sin roce y debido a
la gravedad, por la supercie de una semiesfera de radio R. La masa parte desde
la cspide sin velocidad inicial. Sea P el
u
punto en el cual la masa se separa de la
semiesfera. Encuentre el ngulo de elevaa
cin 0 del punto P .
o
Respuesta: sin 0 = 2/3 .

Figura 5.14

CAP
ITULO 5. TRABAJO Y ENERG
IA

218

13. Sobre una cinta transportadora caen 5 kg de material por segundo ( = dm/dt =
5 kg/s). Suponiendo que no hay prdidas de energ de ningn tipo en todo el
e
a
u
sistema que impulsa la cinta transportadora, encuentre la fuerza F que debe
aplicarse para mantener la cinta trasladndose con una velocidad constante v0 =
a
3 m/s. Cul es la m
a
nima potencia que debe tener el motor para hacer avanzar
la cinta transportadora?

Figura 5.15

Respuesta:

P =

dW
2
= v0 = 45 W .
dt

14. Considere dos masas m unidas por una varilla de largo L que no tiene peso. Inicialmente el sistema est apoyado en una paa
red, formando un ngulo 0 con la normal
a
(vea gura 5.16). El sistema comienza a
resbalar sin roce debido a la gravedad. A
qu altura la masa # 1 se separa de la
e
pared vertical?
Respuesta:
h = 2h0 /3 , donde h0 =
L sin 0 es la altura inicial.

Figura 5.16

15. Una moneda se desliza sobre un tramo


horizontal pulido. Luego entra en un tramo cil
ndrico convexo de radio R=1 m. La
moneda pierde contacto con la supercie
cil
ndrica a un ngulo de 30 con respeca
to a la vertical medido desde el vrtice del
e
cilindro. Calcule la rapidez con que se desplazaba la moneda en el tramo horizontal.

Figura 5.17

CAP
ITULO 5. TRABAJO Y ENERG
IA

219

16. La fuerza electrosttica que ejerce una carga elctrica Q sobre otra carga q viene
a
e
dada por la as llamada Ley de Coulomb:

F =

qQ
r ,

r2

donde r = r r es el vector de posicin de q respecto a Q. Notemos que si las

o
dos cargas tienen el mismo signo la fuerza entre las cargas es repulsiva.
Considere una carga Q ja en el origen y una carga q, que inicialmente se
encuentra en el lugar r0 .
a) Encuentre el trabajo que debe realizarse para trasladarla desde r0 hasta
r1 .
b) Repita la parte a) para varios caminos simples y demuestre que siempre
obtiene el mismo resultado (en otras palabras, el campo de fuerzas es
conservativo).
c) Demuestre que la energ potencial (electrosttica) de la carga q viene dada
a
a
por
qQ
.
U (r) = +
r
En qu lugar se ha elegido el cero para la energ potencial?
e
a
17. Considere una carga Q ja en el origen y otra carga q, del mismo signo, que
se acerca a Q a lo largo de la recta que las une. Si q tiene una energ cintica
a
e
K cuando la separacin entre las cargas es muy grande (innita), encuentre la
o
m
nima distancia a la que q se acercar a Q.
a
Para resolver este problema use el resultado para la energ potencial obtenido
a
en el problema anterior.
18. Considere la conguracin de caro
gas mostrada en la gura 5.18. Las
cargas +Q estn jas en los lugares
a
x = a, y = z = 0 mientras que la
carga +q puede deslizarse slo a lo
o
largo del eje x.

Figura 5.18

a) Encuentre una expresin para la fuerza F (x) que acta sobre la carga +q.
o
u
b) Encuentre la energ potencial U (x) y graf
a
quela. (Especique claramente
dnde eligi el cero para la energ potencial.)
o
o
a

CAP
ITULO 5. TRABAJO Y ENERG
IA

220

c) Se comportar este sistema como un oscilador armnico para pequeos


a
o
n
desplazamientos de q en torno al origen? (Un sistema se comporta como
un oscilador armnico si, al desplazar el sistema de su posicin de equio
o
librio, aparece una fuerza proporcional al desplazamiento pero de sentido
contrario ejemplo, ley de Hooke.)
19. Considere una part
cula de masa m
y carga q restringida a moverse a
lo largo del eje x. Adems, dos car
a
gas +Q se ubican jamente sobre el
eje y a una distancia a del origen,

tal como lo muestra la gura 5.19.


a) Encuentre la energ potencial
a
U (x) del sistema de cargas en
funcin de x.
o

Figura 5.19

(b) Encuentre la fuerza electrosttica F (x) que acta sobre la carga q.


a
u
(c) Evale la derivada dU (x)/dx y demuestre que sta coincide con F (x).
u
e
(d) Con qu velocidad pasar la part
e
a
cula por el origen si parte desde el
innito con velocidad cero?
(e) Se comportar este sistema como un oscilador armnico para pequeos
a
o
n
desplazamientos de q en torno al origen? Si su respuesta es armativa,
encuentre el per
odo del movimiento peridico.
o
Respuestas:
2Qq
; b)
a) U (x) =
a2 + x2

F (x) =

(a2

2Qqx
x ; d) v =

+ x2 )3/2

4Qq
.
ma

20. Un bloque de 2 Kg, situado a una altura de 1 m, se desliza por una rampa curva
y lisa, partiendo del reposo. Al terminarse la rampa, el bloque resbala 6 m sobre
una supercie horizontal rugosa antes de llegar al reposo.
a) Cul es la velocidad del bloque en la parte inferior de la rampa?
a
b) Cul es el trabajo realizado por la fuerza de roce sobre el bloque?
a
c) Cunto vale el coeciente de roce cinemtico entre el bloque y la supercie
a
a
horizontal?

Figura 5.20

CAP
ITULO 5. TRABAJO Y ENERG
IA

221

21. En un parque de entretenciones un carro de masa m = 100 kg se desliza (sin


roce) por una rampa desde una altura h, ingresando a un loop de radio R = 3 m.
La altura h es la m
nima que se requiere para que el carro no se salga de la
v Emergiendo del loop el carro ingresa a la regin de frenado, donde en un
a.
o
trayecto de largo L, el coeciente de roce cinemtico es c = 0, 2. Sin embargo,
a
el carro no alcanza a detenerse durante la primera pasada, sino que pasa de
largo y despus de colisionar con un resorte de constante k = 500 N/m, vuelve
e
a ingresar a la regin de frenado quedando en reposo al centro de ella (o sea, en
o
el punto C, ver gura 5.21).
a) Encuentre la velocidad del
carro en el punto B.
b) Encuentre h.
c) Encuentre L.
d) Encuentre la mxima coma
presin que alcanza a tener
o
el resorte.

Figura 5.21

22. Una part


cula de masa m se mueve sobre una mesa rugosa a lo largo de un
c
rculo de radio R. La part
cula est amarrada a un extremo de un hilo de largo
a
R, cuyo otro extremo est jo al centro del c
a
rculo. La velocidad de la part
cula
inicialmente es v0 . Debido al roce con la mesa, la part
cula se ir frenando.
a
Despus de completar una vuelta, su velocidad es v0 /2.
e
a) Encuentre el trabajo realizado por la friccin durante una vuelta. Exprese
o
el resultado en funcin de m, v0 y R.
o
b) Encuentre el valor del coeciente de roce cinemtico.
a
c) Cuntas vueltas dar la part
a
a
cula antes de detenerse?
23. Una masa m se cuelga de dos resortes en serie, de constantes de
restitucin k1 y k2 , tal como se
o
muestra en la gura 5.22a. Encuentre la frecuencia de oscilacin para pequeas vibraciones
o
n
(verticales) de la masa m.
Repita el clculo para el caso en
a
que los dos resortes estn en paa
ralelo (ver gura 5.22b).

Figura 5.22

CAP
ITULO 5. TRABAJO Y ENERG
IA

222

24. Supongamos que la funcin energ poo


a
tencial U (x) en funcin de x, para una
o
part
cula que slo se puede mover a lo laro
go del eje x, viene dada por el grco mos
a
trado en la gura 5.23
a) Identique los puntos de equilibrio e
indique si son estables o inestables.
b) Para qu valor de x la fuerza tiene
e
su valor (mdulo) mximo?
o
a
c) Describa en palabras el movimiento, a medida que transcurre el tiempo, de una part
cula de energ total
a
E1 . (Especique claramente las condiciones iniciales que est suponiena
do.)

Figura 5.23

25. Suponga que la energ potencial entre dos tomos de una molcula diatmica
a
a
e
o
viene aproximadamente dada por la expresin
o
U (r) = U0

a
r

12

a
r

donde r es la separacin entre los tomos y a y U0 son constantes.


o
a
a) Graque la energ potencial.
a
b) Para qu separacin r los tomos estarn en equilibrio? El equilibrio es
e
o
a
a
estable o inestable?
c) Suponga que los tomos tienen la misma masa m0 . Con qu frecuencia
a
e
vibrar la molcula al alejar el sistema levemente de su posicin de equilia
e
o
brio?
d) Si la molcula est en su estado de equilibrio, cul es la m
e
a
a
nima energ
a
que habr que entregarle a la molcula para disociarla, es decir, separarla
a
e
en sus dos tomos constituyentes?
a
26. La fuerza gravitatoria entre dos masas m1 y m2 viene dada por
F =

Gm1 m2
r ,

r2

donde G es la as llamada constante gravitatoria y r = r es el vector que une

r
los centros de masa de ambas masas. El valor experimental de G es
G = 6.6720 1011 m3 / s2 kg .

CAP
ITULO 5. TRABAJO Y ENERG
IA

223

a) Demuestre que el trabajo que debe hacerse para alejar las dos masas desde
una separacin r1 a una separacin r2 (r2 > r1 ), viene dado por
o
o
W = Gm1 m2

1
1

r1 r2

b) A partir del resultado anterior, demuestre que si el cero de la energ


a
potencial se elige en r = , entonces la energ potencial en funcin de la
a
o
distancia entre las dos masas viene dada por
U (r) =

Gm2 m1
.
r

c) Suponga ahora que m1 = MT es la masa de la Tierra y m2 = m es la masa


de un objeto pequeo. Si tal objeto se encuentra sobre la supercie terrestre
n
y se eleva una pequea distancia h, demuestre que la energ potencial
n
a
cambia en U = mgh. Note que de esta manera usted ha encontrado una
expresin para la aceleracin de gravedad g en trminos del radio de la
o
o
e
Tierra RT , la masa MT y la constante de gravitacin G.
o
d) Encuentre la masa de la Tierra suponiendo que el radio de la Tierra es
aproximadamente 6380 km. (Ignore la rotacin de la Tierra.)
o
e) Encuentre la velocidad de escape, es decir, la velocidad m
nima que debe
impartirse a una masa m (inicialmente en reposo sobre la supercie terrestre) para que ella pueda alejarse del campo gravitatorio de la Tierra.
(Ignore la rotacin de la Tierra.)
o
f ) Hasta qu distancia mxima se podr alejar el pequeo objeto si su veloe
a
a
n
cidad inicial es la mitad de la velocidad de escape?
27. Un bloque de masa M se apoya sobre un platillo de masa m sujeto a un resorte
vertical de constante k y largo natural 0 . Al colocar el platillo con la masa M
sobre el resorte este se comprime teniendo, en equilibrio, un largo .
Comprimamos ahora el resorte otro poco de manera que inicialmente se encuentra contra a un largo < . En cierto instante se suelta el resorte, permitiendo
do
que ste se expanda.
e

CAP
ITULO 5. TRABAJO Y ENERG
IA

224

a) Evale .
u
b) Demuestre que si el resorte en algn
u
instante supera el largo natural 0 ,
entonces el bloque se separa del platillo.
c) Cul es el m
a
nimo valor de la contraccin (0 ) que debe tener el
o
resorte antes de soltarlo para que el
bloque alcance a separarse del platillo?

Figura 5.24

d) Encuentre la altura mxima alcanzada por el bloque (en todos los casos,
a
cuando se separa y cuando no se separa del platillo).
Respuesta: c) (0 ) = 2g(M + m)/k.
28. En una feria de entretenciones hay un juego que consiste en que los participantes
abordan un carro en el punto P (ver gura 5.25), caen en ca libre una altura
da
h hasta el punto A, luego recorren un cuarto de circunferencia (AB) de 2 m de
radio y una recta (BC) de 5 m, todo esto sin roce. En el punto C se ingresa
a una zona de 8 m de largo con coeciente de roce c = 0.5. Como zona de
seguridad, hay una distancia (DE) de 5 m sin roce, concluyendo la pista en un
gran resorte cuya constante elstica es k = 6 104 N/m. La masa del carro,
a
con los pasajeros, es de 500 Kg.
a) Calcule hasta cuntos metros por sobre el punto A se puede dejar caer el
a
carro para que ste se detenga en la zona de desaceleracin CD.
e
o
Suponga ahora que el operador del juego sube el carro hasta 6 m sobre A y lo
deja caer desde all
.
b) Encuentre el lugar en que el carro quedar sin velocidad (por primera vez).
a
c) Encuentre el lugar en que el carro quedar nalmente en reposo.
a
d) Calcule el trabajo realizado por la fuerza elstica del resorte para detener
a
el carro (por primera vez).
e) Calcule la aceleracin del carro en el instante en que el resorte lo detiene.
o

CAP
ITULO 5. TRABAJO Y ENERG
IA

225

Figura 5.25

29. Considere el montaje mostrado en


la gura adjunta. Suponga que las
dos masas tienen el mismo valor y
que 0 coincide con el largo natural
del resorte cuya constante de restitucin es k = 5mg/0 . Suponga
o
adems que la masa desliza sin roce
a
sobre la supercie y que en el instante mostrado en la gura el sistema
se encuentra momentneamente en
a
reposo.

Figura 5.26

a) Demuestre que cuando la masa que se desliza por la supercie se haya


desplazado en una cantidad x = 30 /4 hacia la derecha, esta se levantar de
a
la supercie.
b) Demuestre que en el momento en que la masa se separa del plano la velocidad es v = 19g0 /32.
30. Considere dos pequeas masas iguales m
n
unidos mediante cuerdas ideales de longitud = 1, 5 m, como se indica en la gura
adjunta. El sistema rota con velocidad angular uniforme . El ngulo que la cuerda
a
atada al brazo (de longitud L = 4 m) forma con la vertical es de 60 . Encuentre
el ngulo que la otra cuerda hace con
a
la vertical y encuentre la razn entre las
o
tensiones de cada cuerda.

Figura 5.27

CAP
ITULO 5. TRABAJO Y ENERG
IA
31. Dos cuerpos A y B, de masas m y 2m, repectivamente, se unen mediante una cuerda ideal. El cuerpo A posa sobre una mesa de supercie spera (coeciente de roce
a
c ) mientras que B se deja caer como se
muestra en la gura 5.28. No hay roce entre la cuerda y el punto de contacto con
el borde de la mesa. Calcule el ngulo
a
formado por la cuerda que sostiene la masa B y la horizontal cuando el bloque A
comienza a resbalar. El largo de la cuerda
entre el borde de la mesa y el cuerpo B es
L.
32. Dos monos, llamados Patn y Logo, de

igual masa m estn agarrados de una cuera


da que pasa por encima de una polea (sin
roce), frente al Museo del Louvre. Habiendo escuchado el rumor de que en el museo
hay una famosa pintura de una supuesta congnere con una enigmtica sonrisa,
e
a
el mono Pat decide subir por la cuerda
n
hasta una posicin que le permita mirarla
o
por la ventana. Para ello debe remontar
una altura h.

226

Figura 5.28

Figura 5.29

a) Analice como el movimiento del mono Pat afecta la posicin del mono
n
o
Logo.
b) Calcule el trabajo que debe realizar el mono Pat para llevar a cabo su
n
propsito.
o

CAP
ITULO 5. TRABAJO Y ENERG
IA

227

33. Considere dos masas m unidas por un hilo de largo 2L, que caen con el hilo estirado en forma horizontal. Despus de caer
e
una distancia L el centro del hilo choca
con un clavo, correspondiendo de ah en

adelante la trayectoria de las dos masas a


un movimiento circular. Si el hilo se corta cuando la tensin llega tener el valor
o
max = 7mg/2, encuentre el ngulo que
a
en ese instante forma el hilo con la horizontal (ver gura 5.30).
Figura 5.30
34. Un carro de masa m se desliza sobre el riel mostrado en la siguiente gura:
k
m
A
R

k
E

El carro parte en el punto A, a una altura h del suelo, y sujeto a un resorte


de constante elstica k, el cual est comprimido una distancia x0 respecto a la
a
a
posicin de equilibrio. En cierto instante, el resorte se descomprime, el carro
o
se suelta de l, y comienza a deslizar por el riel, sin roce, hasta el punto B,
e
donde comienza a describir una circunferencia de radio R. Luego llega al punto
C, donde comienza el unico sector rugoso del riel, hasta alcanzar el punto D.

El coeciente de roce cintico del riel en dicho sector es . Finalmente, el carro


e
recorre un sector adicional liso hasta llegar al punto E, donde se encuentra con
un segundo resorte de constante elstica k.
a
a) Si la distancia h es ja, cul debe ser la compresin m
a
o
nima xm del primer
resorte para que el carro no se salga de la v Qu condicin debe cumplir
a?
e
o
h para que xm sea distinta de cero?
b) Considere ahora que el primer resorte est comprimido en dicha distancia
a
m
nima xm . Si al nal del trayecto el segundo resorte alcanza una compresin mxima x1 = xm /2, cul es la longitud L del sector del riel con
o
a
a
roce?

CAP
ITULO 5. TRABAJO Y ENERG
IA

228

c) Si x0 = xm y L est dado por el valor calculado en la parte anterior, pero


a
la altura inicial del carro es 2h, cul es la compresin mxima x1 del
a
o
a
segundo resorte al nal del trayecto?
35. Considere un campo de fuerzas unidimensional tal que, si una part
cula se encuentra en una posicin x > 0, entonces ella experimenta una fuerza dada por
o
F (x ) =

a
bx x ,

x2

con a y b ciertas constantes positivas.


a) (30 %) Calcule el trabajo que ejerce el campo para llevar una part
cula
desde una posicin x0 a una posicin x.
o
o
b) (15 %) Muestre que la energ potencial de la part
a
cula se puede escribir
de la forma
a
b
U (x) = + x2 .
x 2
Dnde se ha escogido el cero de la energ potencial?
o
a
c) (15 %) Encuentre la distancia X respecto al origen tal que una part
cula
colocada en X se halla en equilibrio.
d) (40 %) Muestre, a partir de la expresin para la energ potencial U (x),
o
a
que una part
cula colocada en la vecindad de X, x = X + , con X,
se comporta como un oscilador armnico. Encuentre la frecuencia de dicho
o
oscilador armnico.
o
36. Para modelar el cambio de estructura del compuesto NH4 LiSO4 , se ha propuesto
un modelo en que el in SO4 se puede considerar sometido a un potencial, debido
o
a los otros tomos del compuesto, de la forma
a
U () =

2 4 3
+ + ,
2
4
3

donde corresponde a una variable con unidades de longitud, y proporcional


a un ngulo de desplazamiento, y los coecientes , y se pueden ajustar a
a
partir de datos experimentales.
Considere un in SO4 en el potencial anterior, con = 1 J/m2 , = 2 J/m4
o
y = 2 J/m3 .
Muestre que el in SO4 tiene dos puntos de equilibrio estable, en = 1 y
o
= 2 , y un punto de equilibrio inestable, en = 3 . Determine 1 , 2 y 3 .
Si el in SO4 tiene masa M , encuentre la frecuencia de oscilacin del in SO4
o
o
o
en torno a cada uno de los puntos de equilibrio estable.

CAP
ITULO 5. TRABAJO Y ENERG
IA

229

37. Un cuerpo de masa m se encuentra inicialmente a una altura h, en un punto A,


sobre un riel liso. Al deslizar y llegar al suelo, ingresa a una trayectoria circular
de radio R, cuyo punto ms alto es el punto B. Si las velocidades en A y B
a
satisfacen vB = 2vA , cul es el valor de la fuerza normal en B?
a

A
A

38. Un bloque de masa m se encuentra inicialmente en reposo a una altura h del


suelo. El bloque desciende por un plano inclinado, el cual tiene un coeciente
de roce cintico y forma un ngulo con el suelo. Al llegar al punto ms bajo
e
a
a
del plano inclinado, en el punto B, el bloque contina por el riel, siguiendo una
u
trayectoria circular de radio R. Al completar el trayecto circular, el bloque contina con una trayectoria horizontal, hasta el punto C, ubicado a una distancia
u
L de B.
Cul debe ser el valor de h para que el bloque no se salga nunca de la v Si el
a
a?
bloque parte ahora desde el punto D, que est a 4 veces la altura del punto A,
a
cunto tiempo tarda el bloque desde que completa el trayecto circular hasta
a
llegar a C?
D

4h

B
L

39. Una carga elctrica de magnitud q > 0 se encuentra a distancia ja d del suelo.
e
Sobre la misma l
nea vertical se coloca una segunda carga elctrica Q > 0,
e
de masa M . Si el sistema se encuentra en un campo gravitatorio constante g,

CAP
ITULO 5. TRABAJO Y ENERG
IA

230

dirigido hacia el suelo, encuentre la posicin de equilibrio estable de la carga Q,


o
y la frecuencia de oscilaciones en torno a dicho punto, cuando Q se desplaza en
direccin vertical.
o

q
d

5.7.

Solucin a algunos de los problemas


o

Solucin al problema 12
o
Cuando la masa m haya resbalado hasta formar un ngulo con la horizontal, la
a
energ potencial (gravitatoria) habr cambiado en
a
a
Upot = mg h = mg(R R sin ) .
Como no hay roce, este cambio de energ potencial debe coincidir con la energ
a
a
cintica que adquiere la masa m, o sea, debe cumplirse la relacin
e
o
1
mv 2 = mgR (1 sin ) .
2
Esto nos permite encontrar la velocidad v de la masa en funcin de :
o
v=

2gR(1 sin ) .

La masa m recorre un arco de c


rculo de radio R, luego la fuerza centr
peta (que
apunta en la direccin ) es
o
r
Fcent =

mv 2
r.

(Tambin hay una fuerza tangencial que, sin embargo, aqu no es necesario evaluar.)
e

Las unicas fuerzas reales que actan sobre m son la normal N r y el peso mg.

z
(Nuevamente hemos elegido al eje z apuntando hacia arriba.) La componente radial

CAP
ITULO 5. TRABAJO Y ENERG
IA

231

de la fuerza neta es (N mg sin ) r. Esta debe coincidir con la fuerza centr

peta, o
sea,
mv 2
= N mg sin .

R
Despejando N se obtiene
N

mv 2
1
= mg sin 2mgR(1 sin )
R
R
= mg (3 sin 2)

= mg sin

La masa m inicia su movimiento en el pice, en cuyo caso = 90 , siendo la fuerza


a
normal que ejerce la semiesfera sobre la masa N = mg. A medida que transcurre el
tiempo, disminuye y luego tambin N . Cuando sin = 2/3, la fuerza normal se hace
e
cero, siendo ese el lugar en que la masa m se separa de la semiesfera.
Solucin al problema 14
o
Supongamos por un momento que la
part
cula 1 nunca se despega de la pared.
Cuando la part
cula 1 haya bajado desde
h0 hasta una altura y, entonces, por conservacin de energ
o
a
mg(h0 y) =

1
1
m y 2 + m x2 .

2
2

Sabemos que x2 +y 2 = L2 . Derivando esta


relacin se deduce que 2xx + 2y y = 0, o
o

sea,
x
y = x .

Figura 5.31

Sustituyendo esto en la ecuacin de cono


servacin de la energ se encuentra la reo
a
lacin
o
x2
1
1 L2
1
m x2 + m x2 2 = x2 2 .

2
2
y
2 y
De esta ultima ecuacin podemos despejar la velocidad de la part

o
cula 2 en funcin
o
de la altura a la que se encuentra la part
cula 1:
2g
x2 = 2 (h0 y) y 2 .

L
La velocidad de la part
cula 2 depende de y. Observemos que la rapidez con que se
mueve la part
cula 2 es nula cuando y = h0 y tambin cuando y = 0, luego en algn
e
u
lugar entremedio debe tener un mximo.
a
mg(h0 y) =

CAP
ITULO 5. TRABAJO Y ENERG
IA

232

Encontremos el valor y = h para el cual x tiene su mximo. Para ello debemos

a
2 . Igualando la derivada de f (y)
encontrar el mximo de la funcin f (y) = (h0 y)y
a
o
a cero se encuentra
2h0 y 3y 2 = 0 .
Despejando y se encuentra y = h = 2h0 /3. Es claro que cuando la part
cula 1 llegue
a esa altura, se desprender de la pared (si es que no hay un mecanismo que evite
a
que eso ocurra). La razn es la siguiente: el unico elemento que ejerce una fuerza
o

horizontal sobre el sistema (las dos masas con la varilla) es la pared vertical. Mientras
y > h la part
cula 2 acelera (la rapidez x aumenta) en la direccin +, luego la pared

o
x
debe ejercer sobre el sistema una fuerza en esa direccin. Cuando y < h entonces
o
la part
cula 2 desacelera (x vuelve a disminuir); eso implica que la pared ejerce una

fuerza en la direccin sobre el sistema, lo que a su vez slo es posible si existe


o
x
o
algn mecanismo que sujete a la part
u
cula 1 a la pared vertical. Si tal mecanismo no
existe, entonces la part
cula 1 se separa de la pared.
Solucin al problema 18
o
De acuerdo a la ley de Coulomb, las cargas Q de la izquierda y de la derecha ejercen
sobre q una fuerza
qQ
x

F1 =
(a + x)2
y
qQ
F2 =
x,

(a x)2
respectivamente. La fuerza total F (x) que acta sobra la carga q es la suma vectorial
u
de las dos fuerzas F1 y F2 , por lo tanto,
F (x) = F1 + F2 = qQ

1
1

(a + x)2 (a x)2

x.

Para encontrar la energ potencial tambin podemos evaluar primero la energ poa
e
a
tencial de q con cada una de las cargas Q separadamente, para luego hacer la suma
(escalar) de ellas. La energ potencial de una carga q a una distancia r de otra carga
a
Q viene dada por (ver problema 14) U (r) = qQ/r. Usando esta expresin se encuentra
o
que la energ potencial de la carga q, cuando sta se encuentra en el lugar x, es:
a
e
U (x) = U1 (x) + U2 (x) =

qQ
qQ
+
.
|a + x| |a x|

La energ potencial es cero cuando x . La gura 5.32 muestra un grco de


a
a
la funcin U (x).
o

CAP
ITULO 5. TRABAJO Y ENERG
IA

233

Figura 5.32
De la gura se deduce que x = 0 es un punto de equilibrio estable del sistema. Para
pequeos desplazamientos, o sea para |x| a, se tiene
n
1
1
+
=
a+x ax
qQ
x x2
1 + 2 +
a
a a
2qQ 2qQ 2
+ 3 x = U0 +
a
a

U (x) = qQ

qQ
a
1+

1+

x
a

+ 1

x
a

x x2
+
a a2

2qQ 2
x .
a3

De la ecuacin anterior se deduce que, para pequeos desplazamientos de q desde el


o
n
origen, la energ potencial es cuadrtica (es decir, similar a la expresin que se ten
a
a
o
a
para una masa adosada a un resorte).
La fuerza que acta sobre q al desplazarla levemente de su posicin de equilibrio es
u
o
F (x) =

4qQ
dU (x)
= 3 x.
dx
a

Esta fuerza es anloga a la ley de Hooke: es proporcional y apunta en sentido contrario


a
al desplazamiento. El papel de la constante de restitucin k lo juega 4qQ/a3 . Luego, al
o
desplazar la carga q levemente de su punto de equilibrio, sta oscilar armnicamente
e
a
o
con un per
odo
T = 2
donde m es la masa de la carga q.

ma3
,
4qQ

CAP
ITULO 5. TRABAJO Y ENERG
IA

234

Solucin al problema 25
o
La gura adjunta muestra el grco de
a
la energ potencial. Para r > a la pena
diende es positiva, para r = a es nula, mientras que para r < a es negativa. La fuerza entre los dos tomos de la
a
molcula es dU (r)/dr. Cuando la derie
vada es nula (para r = a), la fuerza tambin es nula, luego la separacin r = a coe
o
rresponde a un punto de equilibrio. Para
r > a, dU (r)/dr > 0 y, por consiguiente,
F (r) < 0. En palabras: si la separacin
o
de los dos tomos de la molcula es maa
e
yor que a, entonces la fuerza entre ellas
ser atractiva.
a

Figura 5.33

Lo contrario ocurre para r < a: en ese caso dU (r)/dr < 0 y por consigiente F (r) >
0, o sea, la fuerza que aparece tratar de alejar a los dos tomos (aumentar r).
a
a
Resumiendo, cada vez que el sistema se desplaza de su posicin de equilibrio, aparece
o
una fuerza que trata de llevar al sistema nuevamente a su posicin de equilibrio. (Es
o
precisamente esto ultimo lo que caracteriza a un punto de equilibrio estable.)

Sea F12 la fuerza que acta sobre el tou


a
mo 1 debido al tomo 2 y F21 la fuerza que
a
acta sobre el tomo 2 debido al tomo 1.
u
a
a
Por supuesto que, de acuerdo al principio
de accin y reaccin (tercera ley de Newo
o
ton) F12 = F21 . Sea O un origen y r1 y
r2 los vectores de posicin de cada uno de
o
los tomos (ver gura 5.34).
a
Las ecuaciones de movimiento, de acuerdo
a la segunda ley de Newton, son:

m0 r1 = F12
y

Figura 5.34

m0 r2 = F21 .

Restando una de la otra se obtiene

m0 (r2 r1 ) = F21 F12 = 2F21 .

(5.14)

CAP
ITULO 5. TRABAJO Y ENERG
IA

235

La fuerza que acta sobre la part


u
cula 2 debida a la part
cula 1 es
F21 =

dU (r)
r,

dr

donde r = | r | = | r2 r1 |. Como estamos suponiendo que la molcula no rota, se


e
tiene que el vector unitario r , que apunta a lo largo de la l

nea que une a ambos


tomos, no variar a medida que transcurre el tiempo. Se tiene entonces que
a
a
r r2 r1 = r
r
y

r = r2 r1 = r r .

Sustituyendo la ultima ecuacin en (5.14) se obtiene

o
m0 r r = 2F21 .

(5.15)

Evaluemos F21 . Se tiene:


dU (r)
r

dr
a
U0
= 12
a
r

F21 =

a
r

r.

Escribamos r de la forma r = a+s. De esta manera, s = 0 corresponder a la molcula


a
e
en su posicin de equilibrio. Si los tomos se desplazan slo levemente de su posicin
o
a
o
o
de equilibrio, entonces |s| << a. En este caso
F21

U0
= 12
a

a
a+s

a
a+s

s
s 6
U0
1+
1
1+
a
a
a
s
s
U0
16 1
r

17
12
a
a
a
U0
72 2 s r + o(s2 ) .

= 12

(5.16)

Sutituyendo este resultado en (5.15), se obtiene


m0 r r = 72

U0
sr.

Cancelando a ambos lados r y usando el hecho que r = s, se tiene


2
s + 0 s = 0 ,

(5.17)

CAP
ITULO 5. TRABAJO Y ENERG
IA
con
2
0 = 72

236

U0
2m
a

.
0

La ecuacin diferencial (5.17) corresponde a la de un oscilador armnico. Ya sabemos


o
o
que en ese caso, la magnitud s (el alejamiento de un tomo de su posicin de equilibrio)
a
o
realizar oscilaciones armnicas, siendo el per
a
o
odo de tales oscilaciones
T =

2
=
0

a2 m 0
.
18 U0

De la gura 5.33 tambin se deduce que para disociar a la molcula, es decir, para
e
e
separar los tomos a una distancia r , se debe entregar al sistema una energ
a
a
al menos igual a U0 .
Solucin al problema 28
o
a) Si la masa m parte de una altura h, entonces su energ (antes de entrar a la regin
a
o
de desaceleracin) es
o
1
E = mv 2 = mgh .
2
Al atravezar toda la zona de desaceleracin, la energ disipada en calor es Q =
o
a
a
c mg CD. Para que la masa m quede en reposo en D, toda su energ debe ser
disipada como calor, o sea,
mgh = c mg CD .
Despejamos h:
h = c CD = 0, 5 8 [m] = 4 [m] .
Luego, se puede dejar caer el carro, como mximo, desde 2 metros sobre A.
a
c) Ahora h = 8 [m]. La mitad de la energ se disipar durante la primera pasada
a
a
por la regin de desaceleracin y el resto se disipar en la segunda pasada. El carro
o
o
a
m quedar nalmente en reposo en el punto C.
a
b) Despus de emerger de la regin de desaceleracin por primera vez, la energ del
e
o
o
a
carro ser E1 = mgh/2. Esta tendr que ser la energ potencial del resorte cuando
a
a
a
est comprimido con el carro detenido:
e
1
mgh
= kx2 ,
2
2 0
donde x0 es la compresin mxima del resorte. El carro se detendr por primera
o
a
a
vez a una distancia x0 a la derecha del punto E. Despejando x0 se encuentra (con
g = 10 [m/s2 ]),
x0 =

mgh
=
k

500 10 8
[m] = 0, 816 [m] .
6 104

CAP
ITULO 5. TRABAJO Y ENERG
IA

237

d) El trabajo realizado por la fuerza elstica del resorte para detener el carro es igual
a
a la energ con que incidi sobre el resorte, mgh/2 = 500 10 8/2 [J]= 20000 [J].
a
o
Tambin podemos encontrarla evaluando la integral
e
x0

kx dx =

W =
0

1 2
kx
2

x0

=
0

1 2 1
kx = 6 104 (0, 816)2 [J] .
2 0 2

e) La fuerza que ejerce el resorte cuando est comprimido es kx0 x, donde x apunta
a

hacia la derecha. La aceleracin del carro, por lo tanto, ser


o
a
kx0
x = 97, 92 [m/s2 ] ,

m
aproximadamente 10 veces la aceleracin de gravedad.
o
a=

Solucin al problema 33
o
Despus de chocar el hilo con el clavo y al formar un ngulo con la horizontal, la
e
a
energ potencial de cada masa habr disminu en mgL (1 + sin ). Esta ser la
a
a
do
a
energ cintica que tendr cada masa, es decir,
a
e
a
1
mv 2 = mgL (1 + sin ) .
2
Esta relacin nos permite encontrar la velocidad v = v():
o
v 2 = 2gL (1 + sin ) .
Como cada masa est recorriendo un c
a
rculo sabemos que la fuerza radial neta (la
fuerza centr
peta) que se est ejerciendo sobre ella es
a
mv 2
r = 2mg (1 + sin ) r .

L
Las unicas fuerzas reales que estn siendo ejercidas sobre cada masa son la fuerza

a
debido a la tensin del hilo y la fuerza de gravedad:
o
Fcent =

Freal = r mg z .

La componente radial de esta fuerza es + mg sin . Esta debe coincidir con la


fuerza centr
peta, o sea,
+ mg sin = 2mg (1 + sin ) .
El hilo se corta si el ngulo es tal que = 7mg/2. Llamando 0 a ese ngulo se
a
a
tiene
7
mg + mg sin 0 = 2mg (1 + sin 0 ) .
2
A partir de esta relacin se encuentra que sin 0 = 0, 5, o sea, 0 = 30 .
o

Cap
tulo 6

Momento lineal y colisiones


versin 26 mayo 2011
o

6.1.

Conservacin del momento lineal


o

Consideremos N part
culas, cuyas masas y vectores de posicin vienen dados por
o
{mj }j y {rj }j , con j = 1, 2, . . . , N . Supongamos que sobre cada part
cula j algn
u
agente externo ejerce una fuerza Fj . Supongamos adems que las part
a
culas interactan entre s siendo fji la fuerza que ejerce la part
u
,
cula i sobre la part
cula j. Por
supuesto, debido al principio de accin y reaccin, fij = fji , o, en palabras, la fuerza
o
o
que ejerce la part
cula j sobre la i es la opuesta a la que la part
cula i ejerce sobre la
j.
Usando la segunda ley de Newton, podemos escribir la ecuacin de movimiento de la
o
part
cula j. Se tiene
N

fji = mj rj .

Fj +
i=1

Al lado izquierdo se tiene la fuerza total que acta sobre la part


u
cula j, mientras que
al lado derecho se tiene el producto de la masa por la aceleracin de esa part
o
cula.
Sumando las ecuaciones de movimiento de todas las part
culas (es decir, sumando
sobre j), se obtiene

fji =
mj rj .
(6.1)
Fj +
j

i,j

Note que, debido al principio de accin y reaccin,


o
o

238

ij

fji = 0, es decir, la suma de

CAP
ITULO 6. MOMENTO LINEAL Y COLISIONES

239

todas las fuerzas internas es nula. Denamos la magnitud


rcm

m1 r1 + m2 r2 + + mN rN
1
=
m1 + m2 + + mN
M

mj rj ,

(6.2)

j=1

donde M m1 + m2 + + mN es la masa total. Derivando la ultima ecuacin dos

o
veces respecto al tiempo, queda
N

mj rj .

M rcm =
j=1

Sustituyendo esta expresin en (7.1), y usando la relacin


o
o

ij

fji = 0, se obtiene

Fj = M rcm .
j=1

u
culas,
La suma j Fj corresponde a la fuerza externa total que acta sobre las part
magnitud que denotaremos por Ftot . De esta manera, nalmente

Ftot = M rcm .

(6.3)

Esta ultima ecuacin no depende de las fuerzas internas que actan entre las distin
o
u
tas part
culas. Al vector rcm se le denomina posicin del centro de masas. La ecuao
cin (6.3) ensea que la suma de las fuerzas externas que actan sobre un sistema
o
n
u
de part
culas acelera al centro de masas como si toda la masa del sistema estuviese centrada all Esta es la segunda vez que encontramos que el centro de masas es
.
un punto privilegiado. Antes, pudimos mostrar que, para efectos de eneg potencial
a
gravitatoria, un cuerpo se puede considerar efectivamente como una part
cula, con
toda la masa ubicada en el centro de masas (Problema 5.5). Ahora hemos encontrado
un resultado an ms general: cuando una fuerza externa cualquiera, no solamente
u
a
gravitatoria, acta sobre un sistema de part
u
culas, se puede considerar que toda la
masa est concentrada en el centro de masas, y el problema se reduce nuevamente a
a
la evolucin de una unica part
o

cula, dada por (6.3).


Si la fuerza (externa) total sobre el sistema de part
culas es nula, o sea, si Ftot =
Fj = 0, entonces el centro de masas no acelera. En ese caso, la velocidad del
j
(0)

centro de masas es constante (designemos por vcm a tal velocidad). Por otra parte, la
velocidad del centro de masas se puede obtener derivando la ecuacin (6.2) respecto
o
al tiempo:
N
1
(0)

mj rj = vcm .
rcm =
M
j=1

CAP
ITULO 6. MOMENTO LINEAL Y COLISIONES

240

Introduciendo la denicin de momento lineal pj = mj rj de cada part


o
cula, la ultima

ecuacin se puede reescribir de la forma


o
(0)
Ptot p1 + p2 + + pN = M vcm .

(6.4)

Es decir, por una parte, es natural denir el momento lineal total del sistema, Ptot ,
como la suma de los momentos lineales de cada part
cula. Lo interesante es que dicho
momento lineal total es igual al momento lineal de una unica part

cula de masa M (la


masa total) movindose con la velocidad del centro de masas. Es decir, nuevamente,
e
esta vez para efectos del momento lineal total del sistema, se puede considerar que
toda la masa del cuerpo est concentrada en un unico punto, el centro de masas.
a

De (6.3) y (6.4), entonces, se sigue que, en ausencia de fuerzas externas,


dPtot
=0,
dt

(6.5)

Ptot = constante .

(6.6)

o bien
Esta ecuacin se conoce con el nombre de Principio de Conservacin del Momento
o
o
Lineal. En palabras: cuando la fuerza externa total que acta sobre un sistema de
u
part
culas es cero entonces la suma de los momentos lineales de las distintas part
culas
se mantiene constante en el tiempo.
Este principio de conservacin es, como el de la energ poderoso. Notemos que
o
a,
podemos tener un sistema arbitrario de part
culas, donde todas estn interactuando
a
entre s y con una fuerza externa. La fuerza individual sobre cada part

cula no tiene
por qu ser cero en general, y por lo tanto las part
e
culas cambiarn su velocidad, y por
a
tanto su momento lineal, constantemente. Sin embargo, a pesar de eso, si la fuerza
externa total sobre el sistema es cero, entonces la suma de los momentos lineales de
todas las part
culas no var en el tiempo. A pesar de la evolucin complicada que
a
o
puede tener el sistema internamente, hay algo, el momento lineal total que se conserva.
Esto se suma entonces a la conservacin de la energ que tambin se satisface para
o
a,
e
sistemas cerrados.
Ilustremos los conceptos anteriores con un ejemplo.
Problema: Considere dos masas, m = m0 y M = 3m0 , sobre las cuales no actan
u
fuerzas externas. Supongamos que en el instante t = 0, la part
cula m se encuentra
en el origen y en reposo, y que la part
cula M se encuentra en rM (0) = 2 [m]
x
movindose con una velocidad vM (0) = 4 [m/s]. Supongamos adems que existe
e
y
a
cierta interaccin entre las part
o
culas y, como consecuencia de ella, ambas aceleran.
Si en el instante t0 = 5 [s] la part
cula m se encuentra en rm (t0 ) = (2 8) [m],
x
y
en qu lugar se encontrar la otra masa?
e
a

CAP
ITULO 6. MOMENTO LINEAL Y COLISIONES

241

Solucin: Como no hay fuerzas externas, el centro de masas se mueve con velocidad
o
constante. Encontremos primero la velocidad del centro de masas y la posicin de
o
ste en los instantes t = 0 y t = 5 [s]. En efecto
e
rcm (0) =

m rm (0) + M rM (0)
3m0 2
x
3
=
= x [m]

m+M
m0 + 3m0
2

y
vcm (0) =

m vm (0) + M vM (0)
3m0 4
y
=
= 3 [m/s] .
y
m+M
m0 + 3m0

Como la velocidad del centro de masas es constante, en el instante t0 = 5 [s] el centro


de masas se encontrar en
a
rcm (t0 ) = rcm (0) + vcm (0) t0 =

3
x + 3 5

y
2

[m] .

Por otra parte, en el instante t0 la posicin del centro de masas viene dada por
o
rcm (t0 ) =
=

mrm (t0 ) + M rM (t0 )


m+M
1
m0 (2 8) + 3 m0 rM (t0 )
x
y
x
y
= (2 8 + 3rM (t0 )) [m] .
m0 + 3 m0
4

Igualando las dos ecuaciones para rcm (t0 ) podemos despejar rM (t0 ):
3
1
(2 8 + 3rM (t0 )) [m] =
x
y
x + 3 5

y
4
2

[m] ,

de donde se deduce que


rM (t0 ) =

1
(8 + 68) [m] .
x
y
3

Graque en el plano (, y ) los vectores rm , rM y rcm en los instantes t = 0 y t = 5 s!


x
Notemos cmo hemos sido capaces de resolver este problema poseyendo escasa
o
informacin. Sabemos que existe alguna interaccin entre los cuerpos, pero no coo
o
nocemos el detalle. No importa. En tanto no existan fuerzas externas, el momento
lineal total se conservar, y eso es suciente para determinar la posicin de la segunda
a
o
masa, conociendo la posicin de la primera.
o
Problema resuelto en clases: 6.10

CAP
ITULO 6. MOMENTO LINEAL Y COLISIONES

6.2.

242

Colisiones

Analicemos primeramente colisiones en una dimensin. Considere dos part


o
culas de
masas m y M , restringidas a moverse (sin roce) a lo largo del eje x y estudiemos

algunos casos particulares.


a) Supongamos que la part
cula M incide desde la izquierda con velocidad +v0 y se
mueve hacia la part
cula m, que inicialmente se encuentra en reposo. Suponga
que las dos part
culas colisionan, quedando una adosada a la otra, formando
una unica part

cula de masa (M + m). Con qu velocidad v se mover esta


e

a
nueva part
cula despus de la colisin?
e
o
Para resolver este problema usamos el principio de conservacin del momento
o
lineal. Sobre el sistema no hay fuerzas externas actuando, luego el momento
lineal se conserva. El momento lineal total antes de la colisin es
o
(i)

Ptot = M v0 + m 0 = M v0 ,
mientras que despus de la colisin es
e
o
(f )

Ptot = (M + m) v .
Estas dos expresiones deben ser iguales, luego
v=

M
v0 .
M +m

En el l
mite M m (el caso de una locomotora chocando con una mosca) la
velocidad despus de la colisin es v v0 (la velocidad de la locomotora). En
e
o

el l
mite M m (el caso de una mosca chocando contra una locomotora en
reposo) la velocidad despus de la colisin es v 0.
e
o

La energ cintica antes y despus de la colisin no es la misma, siendo la


a
e
e
o
diferencia
1
1
2
(M + m)2 M v0
v
2
2
1
m
2
= M v0
.
2
M +m

Q Kf Ki =

A esta diferencia se le suele llamar el valor Q de la reaccin. Cuando, como en el


o
presente ejemplo, Q = 0, la colisin se dice que es inelstica. Si Q < 0, la colisin
o
a
o
es endotrmicaparte de la energ que el sistema ten antes de la colisin se
e
a
a
o
difunde a grados de libertad internos del sistema (por ejemplo, se pierde como
calor, que en el fondo es entregar energ cintica a las molculas). Si Q > 0 la
a
e
e
colisin (o reaccin) es exotrmica.
o
o
e

CAP
ITULO 6. MOMENTO LINEAL Y COLISIONES

243

Notemos que es muy interesante el hecho de que en este ejemplo la energ no


a
se conserva. Y no importa, porque la conservacin de momentum tiene toda la
o
informacin necesaria para resolver completamente el sistema.
o
b) Consideremos ahora el caso de una colisin en que se conserva la energ cintio
a
e
ca (o sea, una colisin con Q = 0) y adems se conserva la identidad de las
o
a
part
culas (es decir, el nmero de ellas y sus masas). Tales colisiones se denou
minan elsticas. Nuevamente supongamos que la part
a
cula M incide desde la
izquierda, a lo largo del eje x, con velocidad +v0 y que choca con la part

cula
m, que inicialmente se encuentra en reposo. Encontremos la velocidad nal vm

y vM de cada una de las part

culas.
Notamos que en este caso, a diferencia del anterior, son dos las incgnitas a
o
encontrar. Si slo se conservara el momentum, pero mgicamente en este
o
a
caso se conserva tambin la energ lo cual nos proporciona precisamente la
e
a,
ecuacin adicional que necesitamos para resolver completamente el problema.
o
Invocamos, entonces, la conservacin del momento lineal y de la energ (cintio
a
e
ca). Se tiene
M v0 = mm + M vM
v

1
1
1
2
M v0 = mm + M vM .
v2
2
2
2
2
De estas dos ecuaciones podemos despejar las dos incgnitas vm y vM . Se obtiene
o

vm =

2M
v0
M +m

vM =

M m
v0 .
M +m

Analicemos varios casos l


mites:
i) M m (o sea, una locomotora con velocidad v0 colisionando con una
pelota de tenis en reposo). En este l
mite, vM = v0 y vm = 2v0 (es decir,

la locomotora seguir como si nada hubiese pasado mientras que la pelota


a
de tenis sale disparada con una velocidad igual al doble de la velocidad de
la locomotora).
ii) M m (o sea, una pelota con velocidad v0 colisionando con una locomotora en reposo). En este l
mite, vm 0 y vM = v0 (es decir, la locomotora

prcticamente no se mover y la pelota rebota con una rapidez igual a la


a
a
incidente). Que la velocidad vM en este caso sea negativa signica que la

part
cula se mueve en la direccin .
o
x

CAP
ITULO 6. MOMENTO LINEAL Y COLISIONES

244

iii) M = m (o sea, la colisin central de dos bolas de billar de igual masa).


o
En ese caso vm = v0 y vM = 0 (es decir, la part

cula incidente queda en


reposo mientras que la otra part
cula sale disparada con velocidad v0 ).
Si la colisin no ocurre a lo largo de una l
o
nea recta, sino que ocurre en un plano,
los conceptos involucrados son los mismos y lo unico que cambia es que la ley de

conservacin del momento lineal, siendo una ecuacin vectorial, nos dar una relacin
o
o
a
o
para cada una de las componentes.
Observemos, sin embargo, que en el caso bidimensional las ecuaciones de conservacin son tres (una para la energ dos para el momentum una en cada como
a,
ponente), pero las incgnitas son cuatro (dos componentes de velocidad para cada
o
part
cula). El problema, entonces, no tiene solucin unica. Esto signica, por ejemplo,
o
que cuando dos bolas de billar colisionan, el ngulo en que ellas emergen no se puede
a
determinar a partir de las ecuaciones de conservacin. Lo cual es natural, porque para
o
nosotros las bolas son part
culas, sin estructura. El ngulo con que emergen depende
a
en realidad del punto exacto en que la supercie de una bola colisiona con la otra.
Resolver este problema es en realidad bastante ms complicado, pero es posible (ver
a
Sec. 6.6).
Otro modo de resolver el problema, manteniendo la descripcin en trminos de
o
e
part
culas puntuales, es dar algn tipo de informacin adicional, a n de eliminar una
u
o
incgnita. Demos un ejemplo de ello.
o
Problema: Considere una masa m1 que choca elsticamente contra una masa m2
a
originalmente en reposo. Suponga que despus del choque la masa incidente m1 emere
ge en una direccin perpendicular a su direccin original (ver gura 6.1). Encuentre:
o
o
a) El ngulo con que emerge m2 , en funcin de m1 y m2 .
a
o
b) La velocidad v1 con que emerge m1 .
c) La velocidad v2 con que emerge m2 .

Figura 6.1

CAP
ITULO 6. MOMENTO LINEAL Y COLISIONES

245

Solucin: La energ y el momento lineal total, antes y despus de la colisin son:


o
a
e
o
1
2
Ei = m1 v0 ,
2
p i = m 1 v0 x ,

1
1
2
2
m 1 v1 + m 2 v2 ,
2
2
pf = m1 v1 z + m2 v2 cos m2 v2 sin .

x
z
Ef =

Debido a que el choque es elstico, se conserva la energ total y el momento lineal


a
a
total; esto nos da las ecuaciones:
2
2
2
m 1 v0 = m 1 v1 + m 2 v2 ,

m1 v0 = m2 v2 cos
y
m1 v1 m2 v2 sin = 0 .

Estas son tres ecuaciones con tres incgnitas (v1 , v2 y ). Resolviendo este sistema
o
de ecuaciones se encuentra:
m2 m1
,
v1 = v0
m2 + m1
v2 = v0

2m2
1
m2 (m2 + m1 )

y
tan =

v1
=
v0

m2 m1
.
m2 + m1

Note que el problema slo tiene solucin si m2 m1 .


o
o
En particular, notemos que si las masas son iguales, = 0. Por lo tanto, el ngulo
a
relativo entre ambas masas despus de la colisin es /2. Uno puede demostrar que
e
o
esto es un hecho general (ver siguiente ejercicio).
Ejercicio: Demostrar que, despus de una colisin elstica entre part
e
o
a
culas de
igual masa, sus trayectorias nales forman un ngulo recto.
a
Problema resuelto en clases: 6.11

CAP
ITULO 6. MOMENTO LINEAL Y COLISIONES

6.3.

246

Impulso

De acuerdo a la segunda ley de Newton F = dp/dt. Multiplicando por dt e


integrando se obtiene
f

F dt =
i

dp = pf pi ,

o sea, el cambio de momento que sufre una part


cula en cierto intervalo de tiempo
es igual a la integral de F dt durante ese intervalo. A la integral F dt se le llama
impulso.
Cuando ocurre una colisin entre dos objetos, el tiempos de colisin puede ser
o
o
muy pequeo, el momento transferido sin embargo puede ser grande. Por ejemplo,
n
cuando una bolita de acero, al caer desde una altura h, rebota (elsticamente) por
a

z
una supercie dura. Al llegar a la supercie la bolita tendr la velocidad 2gh y
a

justo despus del rebote 2gh z . El cambio de momento es, por lo tanto,
e
p = pf pi = 2m

2gh z .

Este cambio de momento debe ser igual al impulso transmitido por el suelo a la bolita,
es decir,
f

F (t) dt = 2m

2gh z .

Podemos intuir que la interaccin entre la part


o
cula y el suelo ser complicada,
a
y que en cada instante el suelo ejerce una fuerza sobre la part
cula que depende
del tiempo. En efecto, seguro que dicha fuerza, cuando recin est empezando el
e
a
contacto, y justo cuando est terminando, es cero; y por tanto debe tener algn
a
u
mximo entremedio. Ser interesante poder calcular ese mximo. Sin embargo, el
a
a
a
hacer rebotar una bolita en el suelo slo nos da informacin sobre la integral de
o
o
dicha fuerza en el tiempo. A pesar de ello, veremos a continuacin que slo con
o
o
algunas suposiciones razonables podemos obtener bastante informacin acerca de este
o
problema.
Primero, notemos que, de acuerdo a lo que
hemos dicho, durante la colisin, t
o
picamente la fuerza que ejerce un cuerpo sobre
tiene el comportamiento como el mostrado en la gura 6.2. Antes de la colisin la
o
fuerza es nula. En el instante t0 1 los
cuerpos entran en contacto, la fuerza aumenta rpidamente llegando a un mximo
a
a
en cierto instante t0 para luego decrecer
nuevamente a cero (instante t0 + 2 ). Los
cuerpos estn en contacto durante un ina
tervalo de tiempo 1 + 2 .

Figura 6.2

CAP
ITULO 6. MOMENTO LINEAL Y COLISIONES

247

El rea bajo la curva F (t) es el impulso (momento) transmitido de un cuerpo al


a
otro.
Observando la curva anterior, notamos que un modelo razonable para la fuerza
es una parbola invertida, centrada en torno a un cierto tiempo t0 , que es cuando se
a
tiene la fuerza mxima. Como es una curva simtrica, la fuerza ser distinta de cero
a
e
a
en un cierto intervalo de ancho 2 en torno a t0 . Lo unico que falta para determinar

completamente la parbola es su valor mximo, que denimos como F0 . La unica


a
a

curva que cumple con las condiciones anteriores es la siguiente:


F (t) =

F0 1
0

1
2

(t t0 )2

para t0 < t < t0 +


para t < t0 y t > t0 +

En este caso, entonces, los cuerpos se mantienen en contacto durante un tiempo


2, siendo en el instante t = t0 la fuerza mxima (F0 ) entre los cuerpos. Ahora
a
que tenemos un modelo razonable para la evolucin temporal de la fuerza, podemos
o
continuar. El impulso transmitido es
t0 +

p =

F0
t0

1
(t t0 )2
2

4
dt = F0 .
3

Con este modelo para F (t), en el caso de la bolita obtenemos


2m

4
2gh = F0 .
3

De esta relacin se deduce que la fuerza mxima entre la bolita y la supercie de


o
a
la cual rebota depende del tiempo durante el cual los dos cuerpos se mantienen
en contacto. Tanto menor es este intervalo de tiempo, tanto mayor resulta ser F0 .
Veamos un ejemplo numrico. Supongamos que una bolita de 50 g se hace caer desde
e
una altura de 1 m. Supongamos adems que el intervalo de tiempo durante el cual los
a
dos cuerpos se mantienen en contacto es aproximadamente 10 ms, o sea, 2 = 102 s.
Con g = 10 m/s2 se tiene:
F0 =

3
0.05 2 10 1 [N] 67 [N] ,
102

fuerza que es ms de cien veces mayor que el peso de la bolita. Si la bolita en lugar de
a
acero es de goma, el tiempo de colisin aumenta considerablemente (en un orden de
o
magnitud y ms), siendo la fuerza F0 , por lo tanto, menor en ese mismo factor. Esto
a
explica por qu es posible que si se lanza una bolita de acero y una bolita de goma
e
de iguales masas y con la misma velocidad contra un vidrio, la bolita de acero rompe
el vidrio, mientras que la de goma rebota sin provocar dao. Toda la diferencia tiene
n
que ver con que el mismo cambio de momentum ocurre en un tiempo menor, y a su
vez el tiempo est determinado por la rigidez de la bolita.
a

CAP
ITULO 6. MOMENTO LINEAL Y COLISIONES

6.4.

248

Problemas

1. Un muchacho de m1 = 40 kg se encuentra sobre una plataforma mvil de


o
m2 = 10 kg junto a dos ladrillos de m = 5 kg. La plataforma puede desplazarse, sin roce, sobre un terreno horizontal. En cierto instante el muchacho lanza
horizontalmente uno y despus el otro ladrillo hacia atrs de la plataforma, con
e
a
una velocidad v0 = 2 m/s respecto a l mismo. Qu velocidad v adquirir la
e
e
a
plataforma?
Respuesta:

v = v0

m(2M + 3m)
= 0, 348
(M + m)(M + 2m)

m/s ,

con M m1 + m2 .
2. Se lanza un proyectil con una velocidad inicial de 20 m/s formando un ngulo
a
respecto a la horizontal. En el transcurso de su vuelo, el proyectil explota
de 30
rompindose en dos partes, una de las cuales tiene el doble de masa que la otra.
e
Los dos fragmentos aterrizan simultneamente. El fragmento ms ligero aterriza
a
a
a 20 m del punto de lanzamiento y en la direccin y sentido en que se dispar el
o
o
proyectil. Dnde aterrizar el otro fragmento?
o
a
3. Una part
cula de masa m y velocidad v0
choca contra otra idntica en reposo. La
e
part
cula en reposo tiene un resorte de
constante de restitucin k, al cual queda
o
adosada la part
cula incidente (ver gura
adjunta). Describa el movimiento del sistema despus de la colisin lo ms detae
o
a
lladamente posible.

Figura 6.3

4. Un bloque de madera de 1 kg se sujeta


a un resorte de constante de restitucin
o
k = 200 N/m. El conjunto descansa sobre
una supercie lisa, como se ve en la gura.
Se dispara contra el bloque un proyectil de
20 g, quedando adosado al bloque. El resorte alcanza una compresin mxima de
o
a
13.3 cm.
a) Encuentre la velocidad del proyectil
antes del choque.
b) Qu fraccin de la energ mecnica
e
o
a
a
inicial se pierde en el choque?

Figura 6.4

CAP
ITULO 6. MOMENTO LINEAL Y COLISIONES

249

5. Una granada, que se encuentra en reposo, explota partindose en dos fragmene


tos. Sean m1 y m2 las masas de los dos fragmentos. Demuestre que las energ
as
cinticas de los fragmentos vienen dadas por
e
T1 =

m2 Q
m1 + m2

m1 Q
,
m1 + m2
donde Q es la diferencia entre la energ cintica total del sistema despus y
a
e
e
antes de la explosin. (Q es la energ liberada por la explosin de la granada.)
o
a
o
T2 =

6. Un ncleo de 236 U (masa=236 amu), inicialmente en reposo, se divide en dos


u
fragmentos de masas 146 y 90 amu. El valor Q de la reaccin es de 190 MeV.
o
Encuentre la velocidad y energ (en MeV) de cada uno de los fragmentos.
a
(1 amu=1.7 1027 Kg, 1 MeV=1.61013 J.)
7. Un ascensor sube en un pozo a razn de 2 m/s. En el instante en que el ascensor
o
est 18 m del extremo superior del pozo, se deja caer una pelota desde la parte
a
superior del pozo. La pelota rebota elsticamente sobre el techo del ascensor.
a
Hasta qu altura subir la pelota en relacin a la parte superior del pozo?
e
a
o
Resuelva el mismo problema suponiendo que el elevador se mueve hacia abajo
a 2 m/s.
8. Una part
cula A que inicialmente tiene una velocidad v0 choca contra otra que
se encuentra en reposo, desvindose (la part
a
cula incidente) en un ngulo . La
a
velocidad de A despus del choque es v. Por otra parte, la segunda part
e
cula
se desv en un ngulo respecto a la direccin inicial de la primera part
a
a
o
cula.
Demuestre que
v sin
.
tan =
v0 v cos
9. Considere una plataforma de ferrocarril de
peso W que puede rodar sin friccin sobre
o
una v recta y horizontal (ver gura ada
junta). Inicialmente la plataforma se mueve a la derecha con velocidad v0 . Sobre la
plataforma se encuentra de pie un hombre
de peso W .

Figura 6.5

Cul es el cambio de velocidad de la plataforma si el hombre corre a la izquiera


da, de modo que su velocidad con relacin a la plataforma es u cuando est a
o
a
punto de saltar del extremo izquierdo?

CAP
ITULO 6. MOMENTO LINEAL Y COLISIONES

250

10. Un objeto de masa m = 5 kg, resbala por


una rampa de masa M =10 kg, partiendo
desde una altura h = 2 m. La rampa puede deslizarse, sin roce, sobre el suelo. Una
vez que la masa llega abajo, con qu vee
locidad se mueve la rampa?
Figura 6.6
11. Un satlite, que se mueve en l
e
nea recta con una rapidez v respecto a algn
u
sistema de referencia, recoge part
culas (polvo) csmicas (que, supondremos,
o
se encuentran en reposo en el sistema de referencia en cuestin) a una tasa
o
dM/dt = k v, donde k es una constante. Si en el instante t = 0 la velocidad del
satlite es v0 y su masa es M0 , encuentre v(t).
e
12. Sobre una supercie horizontal sin friccin se encuentran en reposo N bloques
o
de igual masa m colocados uno al lado del otro en l
nea recta separados por
una pequea distancia. (Los bloques estn enumerados de izquierda a derecha
n
a
como N , N 1, N 2, . . . , 2, 1.) Desde la izquierda incide sobre ellos un
bloque de masa M > m, con velocidad v0 . Suponga que todos los choques son
perfectamente elsticos.
a

Figura 6.7
a) Calcule la velocidad nal de cada bloque despus que han ocurrido todos
e
los choques.
b) Muestre expl
citamente (realizando los clculos antes y despus de todos
a
e
los choques) que se conserva el momento lineal y la energ cintica del
a
e
sistema.
13. Aldo y su hermano Pablo han ido a patinar a una laguna helada. Al mediod
a
Aldo se detiene para abrir el paquete del almuerzo, cuando observa que Pablo
se aproxima directamente hacia l con una rapidez v. Antes del impacto Aldo
e
le lanza a su hermano el paquete en direccin horizontal, el cual es atrapado
o
en el aire por Pablo. Si ambos hermanos tienen la misma masa M , y la masa
del paquete es m, calcule el valor m
nimo de la rapidez con la cual Aldo debe
lanzar el paquete para que su hermano no choque con l.
e

CAP
ITULO 6. MOMENTO LINEAL Y COLISIONES
14. Considere un pndulo consistente de
e
una masa m colgada de un hilo de
largo L. Suponga que el pndulo inie
cialmente parte con el hilo en posicin horizontal. Al llegar la masa
o
al punto inferior (punto O de la gura 6.8), choca elsticamente con
a
una masa M = 2m que se mueve
con velocidad v0 x. El pndulo re
e
bota (hacia atrs) llegando tener coa
mo amplitud mxima nuevamente la
a
horizontal.

251

Figura 6.8

a) Encuentre la rapidez inicial v0 en funcin de m, M , L y g.


o
b) Cul es la velocidad de M despus del choque?
a
e
15. Considere la conguracin mostrada
o
en la gura adjunta. Suponga que
en el instante t = 0 todas las masas estn en reposo. La masa #1,
a
despus de avanzar una distancia
e
h/2, colisiona inelsticamente con la
a
masa m quedando adosada a ella.
Cul es la velocidad de la masa #1
a
justo antes de la colisin? Cul es la
o
a
velocidad de la masa #2 justo despus de la colisin? Con qu veloe
o
e
cidad llega la masa #2 al suelo?
16. Un proyectil de masa m = 20 g,
que se desplaza con velocidad v, choca contra un bloque de masa M =
0, 48 kg que se encuentra en reposo
sobre una supercie lisa. El proyectil
queda incrustado en el bloque.

Figura 6.9

Figura 6.10

A continuacin, el sistema proyectil bloque resbala sobre una zona donde el


o
coeciente de roce cintico con el bloque es c = 0, 3. El sistema se detiene
e
despus de haber resbalado 10 m sobre esa zona rugosa. Encuentre la velocidad
e
inicial del proyectil.
17. Sobre una plataforma horizontal lisa (sin roce) se colocan en l
nea recta 99
bloques de igual volumen y masas 2m, 3m, 4m, . . . , 99m y 100m, separadas
entre s por una distancia a (ver gura 6.11). Desde la izquierda incide una

CAP
ITULO 6. MOMENTO LINEAL Y COLISIONES

252

part
cula de masa m con velocidad v0 . Todos los choques son perfectamente
elsticos.
a
a) Calcule la velocidad de la part
cula de masa 2m inmediatamente despus
e
de la primera colisin.
o
b) Calcule la velocidad de la part
cula de masa 2m inmediatamente despus
e
que experimenta el segundo choque.
c) Despus de un tiempo sucientemente largo se observa que ningn bloque
e
u
permanece sobre la plataforma. Cuntos bloque cayeron al lado izquierdo
a
y cuntos al lado derecho?
a

Figura 6.11
18. Sobre un plano liso se encuentran tres
discos iguales (de radio R y masa M ).
Al disco A, que incide con velocidad v0
choca simultnea y elsticamente con
a
a
los discos B y C, tal como se muestra
en la gura 6.12. Los discos B y C inicialmente se encuentran en reposo con
los centros separados en una distancia
2R+2a. Suponga que no hay roce entre
los bordes de los discos cuando estn
a
en contacto. Encuentre la velocidad del
disco A despus de la colisin.
e
o
Respuesta:

vf = v0

2 2
,
6 2

Figura 6.12

con =

R+a
.
R

19. Un objeto de masa m resbala sobre la


supercie lisa de una cua M . La
n
cu`a reposa sobre una supercie tamn
bin lisa (ver gura). Originalmente el
e
objeto se encuentra en reposo a una altura h medida desde el tramo horizontal.
Figura 6.13

CAP
ITULO 6. MOMENTO LINEAL Y COLISIONES

253

a) Calcule las velocidades de la cua y de la masa m una vez que m ha llegado


n
al tramo horizontal de la cua y se desplaza hacia la derecha.
n
b) Posteriormente, la masa m choca elsticamente con la parte posterior de
a
la cua. Calcule la rapidez de m y M despus del choque.
n
e
20. Una bola de goma se deja caer sobre
una cpula semiesfrica de radio R. La
u
e
bola se suelta a una altura H desde el
suelo y a una distancia b con respecto
a la vertical que pasa por el centro de
la cpula. La bola choca elsticamente
u
a
con la cpula. Calcule la altura h de
u
rebote de la bola con respecto al suelo.
Determine el valor mximo de b para
a
que la respuesta tenga sentido.

Figura 6.14

21. En la gura se muestran las direcciones incidente y de rebote de un cuerpo


que choca contra una pared sin roce. El
cuerpo incide con rapidez v0 y con una
direccin que forma un ngulo con la
o
a
normal de la pared. El cuerpo emerge
con rapidez v0 (con < 1). Determine la direccin ( en la gura) con que
o
emerge el cuerpo y el vector impulso
que la pared imprime al cuerpo.

Figura 6.15

22. Una bala de masa 5 gramos atravieza


un saco de virutas de 1 kg de masa. El
saco cuelga de un cordel de 2 m de largo. A consecuencia del impacto el saco
entra en movimiento y se detiene cuando el cordel forma un ngulo de 12 con
a
la vertical. Calcule la rapidez de la bala una vez que ha emergido del saco
si su velocidad antes de la colisin es
o
v0 = 1000 m/s.

Figura 6.16

CAP
ITULO 6. MOMENTO LINEAL Y COLISIONES

254

23. A y B son dos esferitas de igual masa m engarzadas en el eje horizontal. B


est unida a un resorte ideal de largo natural l0 y rigidez (constante de resa
titucin) k. Inicialmente B est en reposo, el resorte en direccin vertical y
o
a
o
sin deformacin. A se desliza con velocidad v desconocida, choca con B y amo
bas permanecen unidas tras la colisin. Calcular v, si en el instante en que el
o
conjunto se detiene el ngulo tiene un valor de 60 . Suponga que el roce es
a
despreciable.

Figura 6.17

24. Dos carros B y C, de masas m y 2m respectivamente, se encuentran inicialmente


en reposo sobre una v horizontal recta, separados por una distancia L. Un
a
tercer carro A, de masa 2m, que se desplaza con velocidad v0 hacia la derecha
embiste al carro B desde la izquierda (ver gura). Suponga que todos los choques
que ocurren son elsticos. En estas condiciones,
a
a) Demuestre que el carro B choca dos veces con el carro A.
b) Calcule el tiempo que media entre estos dos choques.

Figura 6.18

CAP
ITULO 6. MOMENTO LINEAL Y COLISIONES

255

25. Una cuerda (exible), de masa M y


largo L se deja caer sobre una pesa.
Inicialmente el extremo inferior de la
cuerda justo toca la pesa y la cuerda
se encuentra en reposo. Demuestre que
la lectura de la pesa justo antes de que
caiga sobre ella el ultimo segmento de

la cuerda, ser W = 3M g.
a
Figura 6.19
26. Considere nuevamente a los hermanos del Prob. 13 sobre una pista de hielo.
Los hermanos tienen masa M , y el paquete del almuerzo, masa m. Aldo est en
a
reposo con el paquete del almuerzo, y Pablo se mueve con velocidad constante
vP , pero no directamente hacia l (ver gura). En cierto instante, Aldo lanza el
e
almuerzo a Pablo, con un ngulo respecto a la direccin en que Pablo viaja.
a
o
El almuerzo, luego de avanzar con velocidad constante, llega justo a Pablo.

Pablo

muro

Aldo
a) Encuentre las velocidades de Aldo y Pablo luego de que Pablo ha atrapado
el paquete del almuerzo.
b) Si Pablo sigue avanzando hacia la derecha, llegar al nal de la pista tarde
a
o temprano, donde un muro de contencin lo espera (ver gura). Con
o
qu rapidez, respecto a la pista de hielo, debe lanzar el almuerzo Aldo
e
para que su hermano no choque con el muro de contencin?
o
c) Calcule el valor Q de la colisin.
o

6.5.

Solucin a algunos de los problemas


o

Solucin al problema 6
o
Las energ cinticas totales antes y despus del proceso de sin son
as
e
e
o
Ki = 0

CAP
ITULO 6. MOMENTO LINEAL Y COLISIONES

256

1
1
2
2
m 1 v1 + m 2 v2 .
2
2
La reaccin es exotrmica, es decir, la energ nal es superior a la energ inicial en
o
e
a
a
Q = 190 MeV. Se tiene entonces que
(2)

(1)

Kf = Kf + Kf

(2)

(1)

Kf Ki = Kf = Kf + Kf

=Q.

La conservacin del momento lineal nos da la relacin


o
o
m 1 v1 = m 2 v2 .
Usando esta relacin se deduce que
o
(1)

(2)
Kf

De las relaciones

1
2
2 m1 v1
1
2
2 m2 v2

(1)

Kf

(2)

Kf + Kf

m2
.
m1

=Q

y
(1)

Kf

(2)
Kf

se deduce que

m2
m1

(1)

m2
Q = 117, 5
m1 + m2

MeV

(2)

m1
Q = 72, 5
m1 + m2

MeV .

Kf
y
Kf
De la relacin
o

m2
1
2
= m1 v1 =
Q
2
m1 + m2

(1)

Kf
se deduce que
2
v1 =

m2
2Q
146 2 190 1, 6 1013 J
2, 46 1014
=
m1 m1 + m2
90 236 1, 7 1027 Kg

o sea,

m
.
s
Para la velocidad del fragmento mas pesado se obtiene
v1 = 1, 57 107

v2 =

m
m1
.
v1 = 0, 97 107
m2
s

m
s

CAP
ITULO 6. MOMENTO LINEAL Y COLISIONES

257

Solucin al problema 11
o
Sea v(t) la velocidad y M (t) la masa del cohete (incluyendo la del polvo recolectado). La conservacin del momento lineal nos da la relacin
o
o
M (t)v(t) = M (t + dt)v(t + dt) .
Para un intervalo de tiempo dt innitesimal se tiene que
M (t + dt) = M (t) +
y
v(t + dt) = v(t) +

dM
dt
dt

dv
dt .
dt

Usando estas relaciones se obtiene


M (t)v(t) = M (t + dt)v(t + dt)
dv
dM
dt v(t) +
dt
=
M (t) +
dt
dt
dM
dM dv
dv
dt +
(dt)2
= M (t)v(t) + M (t) dt + v(t)
dt
dt
dt dt
Despreciando el trmino de orden (dt)2 , se obtiene
e
M (t) dv =

dM
v(t) dt .
dt

Pero dM/dt = kv, luego


M (t) dv = kv 2 dt .

Multiplicando esta relacin por v = v(t) se obtiene


o

M (t)v(t) dv = kv 3 dt .
Pero como el momento lineal se conserva, se tiene que M (t)v(t) = M0 v0 , donde M0
y v0 son la masa y velocidad del satlite en el instante t = 0. Usando esto, la ultima
e

ecuacin queda
o
dv
k
=
dt .
v3
M0 v0
Integrando la ultima ecuacin desde el instante t = 0 (en que la velocidad es v0 ) hasta

o
el instante t (en que la velocidad es v(t)) se obtiene
v
v0

1
dv =
v3

1
2
2v
1
2

1
1
2
v 2 v0

v
v0

=
=

k
M0 v0
k
M0 v0

k
t
M0 v0

dt
0
t

t
0

CAP
ITULO 6. MOMENTO LINEAL Y COLISIONES

258

Despejando v(t) se obtiene nalmente


v0

v(t) =

1+

2kv0 t
M0

Solucin al problema 14
o
Para que el pndulo vuelva a su posicin inicial, el choque elstico en la parte inferior
e
o
a
debe ocurrir con momentum total nulo. Luego, despus de la colisin la masa M
e
o
tendr una velocidad v0 x.
a

La velocidad v1 con que la masa m llega abajo, se obtiene de la relacin (consero


vacin de energ
o
a):
1
2
mgL = mv1 ,
2
de donde
v1 = 2gL .
Para que la colisin ocurra con momento lineal cero la rapidez de la masa M debe
o
satisfacer la relacin
o
mv1 = M v0 ,
de donde se obtiene para v0 el resultado
v0 =

m
m
v0 =
M
M

2gL .

Solucin al problema 15
o
En todo instante la part
cula #2 se mueva con la mitad de la velocidad de #1. Justo
antes del choque la velocidad vi de la masa #1 viene dada por (conservacin de la
o
energ
a):
h
1
1
vi 2 5
2
2
M g = (M )vi + M
= M vi ,
4
2
2
2
8
o sea,
2gh
2
.
vi =
5
Mientras ocurre el choque (o sea, durante un brev
simo intervalo de tiempo [0, ])
la cuerda ejerce una gran fuerza sobre la masa #1 y #2 transmitiendo un cambio de
momento lineal p:

(t) dt ,

p =
0

CAP
ITULO 6. MOMENTO LINEAL Y COLISIONES

259

donde (t) es la tensin de la cuerda en funcin del tiempo. Este impulso aumenta el
o
o
momento lineal de la masa #1 en la magnitud p y disminuye el momento lineal de
la masa #2 en el doble de esa misma magnitud. Si vf es la velocidad de la masa #1
justo despus de la colisin, entonces la ecuacin de conservacin del momento lineal
e
o
o
o
para la part
cula #1 (junto con la masa m) da la relacin
o
M vi = (M + m)vf + p .
Para la
M

vf
vi
=M
2p
2
2

Despejando v2 se obtiene

5M
.
5M + 4m
Para encontrar la velocidad v con que la masa #2 llega al suelo usamos nueva
mente el principio de conservacin de la energ
o
a:
v2 = v1

vf
1
1
2
(M + m)vf + M
2
2
2

+ Mg

3h
1
1
= (M + m)(2)2 + M v 2 .
v

4
2
2

Despejando v y usando las expresiones para v2 y v1 se obtiene

v 2 = gh

2M (5M + 3m)
.
(5M + 4m)2

Ejercicio: Verique que tanto para m = 0 como para m , la ultima ecuacin

o
entrega el valor correcto para v .

Solucin al problema 19
o
El momento lineal inicial es cero, y como no hay fuerza externa actuando sobre el
sistema, ser nulo siempre. Cuando m llega abajo ( a la parte horizontal), sean vm y
a
vM las velocidades de la masa m y de la cua, respectivamente. La conservacin del
n
o
momento lineal y de la energ nos da las relaciones
a
mvm + M vM = 0
y

Despejando vm

1
1
2
2
mgh = mvm + M vM .
2
2
de estas dos ecuaciones (con las incgnitas vm y vM ) se obtiene
o
vm = 2gh

M
.
M +m

CAP
ITULO 6. MOMENTO LINEAL Y COLISIONES

260

El signo + corresponde a la solucin del problema antes de la colisin de m con la


o
o
pared mientras que la con el signo es la velocidad de m despus del rebote. La
e
velocidad de la cua es
n
vM =

m
m2
vm = 2gh
.
M
M (M + m)

Solucin al problema 23
o
La colisin entre las dos esferitas es un proceso no elstico, por consiguiente, para el
o
a
proceso de colisin slo podemos usar la ley de conservacin del momento lineal. Sea
o o
o
v1 la velocidad de las dos masas unidas justo despues de la colisin. La conservacin
o
o
del momento lineal nos da la relacin
o
mv = (2m)v1 ,
o sea, v1 = v/2. Luego las dos masas permanecen unidas, el resorte se estira y el
sistema queda en reposo cuando el resorte forma un ngulo con la normal (ver
a
gura 6.17).
Una vez ocurrida la colisin, la energ se conserva. Aplicando la ley de consero
a
vacin de energ al sistema que consiste de las dos esferitas y el resorte se obtiene
o
a
1
1
2
(2m)v1 = k (L)2 ,
2
2
donde L es el alargamiento que sufre el resorte. Este viene dado por
L =

2 + (0 tan )2 0 = 0
0

1
1 .
cos

Despejamos v1 de las dos ultimas ecuaciones:

v1 =

k
0
2m

1
1
cos

Con cos = cos 60 = 1/2 se obtiene para v


v = 2v1 = 0

Solucin al problema 24
o

2k
.
m

CAP
ITULO 6. MOMENTO LINEAL Y COLISIONES

261

Usando las relaciones que se encontraron en la seccin 6.2 para el choque elstico
o
a
se deduce inmediatamente que, despus de la primera colisin, el carro A (de masa
e
o
2m) tendr la velocidad
a
v0
2m m
v0 =
vA =
2m + m
3
mientras que la velocidad de B ser
a
vB =

2(2m)
4v0
v0 =
.
2m + m
3

El carro B tardar un tiempo


a
t1 =

3L
L
=
.
vB
4v0

para colisionar con el carro C. Esta colisin tambin es elstica. Despus de esta
o
e
a
e
segunda colisin la velocidad de B ser
o
a
vB =

m 2m
vB
4v0
vB =
=
.
m + 2m
3
9

El signo negativo indica que B ahora se mueve hacia la izquierda, y por consiguiente,
necesariamente debe colisionar denuevo con el carro A (que sigui movindose hacia
o
e
la derecha con velocidad vA ).
Cuando B colisiona con C, la separacin entre A y B es L = LvA t1 = LL/4 =
o
3L/4. De ah en adelante B se mueve hacia la izquiera con velocidad vB = 4v0 /9,

mientras que A sigue movindose hacia la derecha con velocidad vA = v0 /3. La


e
rapidez relativa con que se acercan es vr = v0 (4/9 + 1/3) = 7v0 /9. La distancia L
ser cubierta en un tiempo
a
t2 =

3L 9
27L
L

=
=
.
vr
4 7v0
28v0

El tiempo total que transcurre entre las dos colisiones de A con B es t1 + t2 , es decir,
ttot = t1 + t2 =

6.6.

12L
.
7v0

Colisin de dos discos


o

Un disco de radio R y masa m, que incide con velocidad v0 , colisiona elsticamente


a
con un disco de masa M y radio R, que inicialmente se encuentra en reposo. La
colisin ocurre con un parmetro de impacto b (ver gura 6.20). Encuentre el ngulo
o
a
a

CAP
ITULO 6. MOMENTO LINEAL Y COLISIONES

262

de dispersin del disco incidente. Suponga que no hay friccin entre los discos y la
o
o
supercie sobre la que se deslizan, y que tampoco hay roce entre los bordes de los
discos mientras estos colisionan. En el lado izquierdo de la gura 6.20 se muestra la
situacin (vista por un observador para el cual inicialmente la part
o
cula M est en
a
reposo) antes de la colisin y al lado derecho la situacin que se tiene despus de la
o
o
e
colisin.
o

Figura 6.20
Solucin anal
o
tica:
Al no haber roce entre los bordes de los
dos discos mientras colisionan, la fuerza de
interaccin entre los discos necesariamente
o
ser perpendicular al per
a
metro, o sea, en
la direccin radial. Sea z la direccin deo

o
nida por la velocidad del disco incidente.
De la gura 6.21 se desprende que el disco
M , que inicialmente est en reposo, desa
pus de la colisin se mover formando un
e
o
a
ngulo con respecto a z , donde viene
a

dado por
b
.
(6.7)
sin =
2R

Figura 6.21
Sean vm y vM las rapideces de los dos discos despus de la colisin. La consere
o
vacin del momento lineal tanto en la direccin y como en la direccin z nos da las
o
o
o
ecuaciones
M vM sin = m vm sin
(6.8)
y
mv0 = M vM cos + m vm cos .

(6.9)

CAP
ITULO 6. MOMENTO LINEAL Y COLISIONES

263

El principio de la conservacin de la energ (recordemos que el choque es elstico)


o
a
a
nos entrega la relacin
o
1
1
1
2
2
2
mv0 = mvm + M vM .
(6.10)
2
2
2
En las ecuaciones (11.2), (11.3) y (11.4), las incgnitas son vm , vM y .
o
Despejemos vM de la ecuacin (11.2) y sustituymosla en las ecuaciones (11.3) y
o
a
(11.4). De esta manera se obtienen las relaciones
mv0 = mvm cos + mvm

sin
sin

cos ,

y
2
2
M mv0 = M mvm + mvm

(6.11)

sin
sin

(6.12)

respectivamente. Reordenando las dos ultimas ecuaciones, stas pueden escribirse de

e
la forma
cos
v0 = vm cos + sin
,
(6.13)
sin
y
m sin2
2
2
v0 = vm 1 +
.
(6.14)
M sin2
Elevando al cuadrado la penltima ecuacin y luego dividindola por la ecuacin
u
o
e
o
(11.7), se obtiene
m sin2
cos 2
=1+
.
(6.15)
cos + sin
sin
M sin2
A partir de esta ecuacin, usando relaciones trigonomtricas elementales se obtiene
o
e
cos2 + 2 cos sin

cos
cos2
+ sin2
sin
sin2

= 1+

m sin2
M sin2

= cos2 + sin2 +

2 cos sin

m sin2
,
M sin2

cos
cos2
m sin2
+ sin2
= sin2 +
.
sin
M sin2
sin2

Multiplicando por sin2 y dividiendo por sin2 queda


2 cos sin

m
cos
+ cos2 = sin2 +
,
sin
M

sin(2)
m
= cos(2) +
,
tan
M

CAP
ITULO 6. MOMENTO LINEAL Y COLISIONES

264

de donde nalmente

sin(2)
.
(6.16)
cos(2)
La ultima ecuacin da el ngulo de dispersin de la part

o
a
o
cula incidente en funcin
o
de , ngulo que a su vez est determinado si se conoce el parmetro de impacto b
a
a
a
(ver ecuacin (11.1)).
o
tan =

m
M

Analicemos brevemente dos casos l


mites:
i) L
mite M .
Cuando la masa del blanco es extremadamente grande comparada con
la masa del disco incidente, entonces
el disco M no se mover y la colisin
a
o
ser especular. Cuando la dispersin es
a
o
especular, el ngulo de rebote del disa
co m respecto a la normal es igual al
ngulo incidente. De la gura 6.21 se
a
observa de inmediato que en ese caso la
relacin entre el ngulo de dispersin y
o
a
o
el ngulo es
a
= 2 .

Figura 6.22

(6.17)

Demostremos que se obtiene el mismo resultado a partir de la ecuacin (6.16).


o
Para M sta queda
e
tan = tan(2) .
(6.18)
Pero

luego

tan(2) = tan( 2) ,
tan = tan( 2) .

(6.19)

De la ultima ecuacin se desprende inmediatamente el resultado (6.17).

o
ii) L
mite M = m.
Cuando M = m entonces la ecuacin (6.16) queda
o
sin(2)
1 cos(2)
2 sin cos
=
1 cos2 + sin2
= cot

tan =

(6.20)

CAP
ITULO 6. MOMENTO LINEAL Y COLISIONES

265

De esta ultima relacin se desprende que

o
+=

.
2

(6.21)

Este es un resultado general: siempre que colisiona un objeto elsticamente con otro
a
de igual masa que inicialmente est en reposo, la suma de los ngulos de dispersin
a
a
o
de ambos objetos ser de 90 .
a

Solucin grca
o
a
Demostremos ahora el resultado dado por la ecuacin (6.16) usando un mtodo gro
e
a
co.
La gura 6.23 muestra los discos antes y despus de la colisin vistos desde dos
e
o
sistemas de referencia distintos: el sistema del laboratorio (en que M inicialmente
est en reposo) y el sistema de coordenadas jo al centro de masas. A pesar de que el
a
concepto de centro de masas se estudiar recien en el siguiente cap
a
tulo, para resolver
el presente problema basta con saber que el sistema de referencia del centro de masas
corresponde al referencial de un observador que se mueve con velocidad uniforme
respecto al laboratorio y para el cual el momento lineal total antes de la colisin es
o
nulo. Tal sistema de referencia es igual al del laboratorio, un sistema de referencia
inercial, es decir, en l tambin se cumplen las leyes de Newton.
e
e

Figura 6.23

CAP
ITULO 6. MOMENTO LINEAL Y COLISIONES

266

Notacin: Para hacer ms transparente lo que sigue, a las magnitudes f


o
a
sicas, cuando
stas estn medidas (observadas) desde el sistema de referencia del centro de masas,
e
e
le agregaremos una prima. A las magnitudes despus de la colisin les agregaremos
e
o
adicionalmente una tilde.
El momento lineal se conserva para ambos observadores. En particular, para el
observador en el sistema de referencia del centro de masas, el momento total de las
part
culas siempre es nulo (en efecto, sta ser, como veremos en el siguiente cap
e
a
tulo,
la denicin del sistema de referencia del centro de masas).
o
Resolvamos primero el problema para el observador del centro de masas. Tal
como se mencion en el prrafo anterior, los momentos lineales de los discos, en todo
o
a
instante, en particular antes y despus de la colisin, deben ser de igual magnitud
e
o
pero apuntar en sentidos contrarios. Si la colisin es elstica entonces, adems, las
o
a
a
magnitudes del momento lineal, antes y despus de la colisin, deben ser iguales. O
e
o
sea, medidos desde el CM, los momentos lineales de los dos discos, antes y despus de
e
la colisin, tienen todos la misma magnitud. Si los cuatro vectores tienen la misma
o
magnitud, las puntas de estos vectores deben estar ubicados todos sobre un c
rculo
(ver gura 6.24).

Figura 6.24

Figura 6.25

En el sistema de referencia del centro de masas es fcil determinar los ngulos de


a
a
dispersin de los dos discos. Cuando no hay roce, la fuerza, y por consiguiente, el
o
cambio de momento lineal de cada disco, debe ser perpendicular a la supercie de
contacto. De la armacin anterior es fcil deducir que la colisin (vista desde el CM)
o
a
o
ser especular (ver gura 6.25). De las guras 6.24 y 6.25 se deduce que el ngulo de
a
a

CAP
ITULO 6. MOMENTO LINEAL Y COLISIONES

267

dispersin de m viene dado por


o
= 2 ,

(6.22)

donde sin = b/(2R).


Volvamos al sistema de referencia del laboratorio. Para pasar del sistema de referencia
del centro de masas al del laboratorio, debemos sumar a todas las velocidades la
velocidad del movimiento relativo entre los dos observadores (esto es, la velocidad
del centro de masas). Para realizar esto grcamente debemos primero transformar
a
la gura 6.24, que es un grco de momentos lineales, a una de velocidades. Pero,
a
para transformar momentos lineales en velocidades basta con dividir a los vectores
momento lineal correspondientes a cada part
cula por su masa. Si los dos discos no
tienen la misma masa, entonces los vectores de velocidad de las part
culas ya no
quedarn sobre el mismo c
a
rculo. Supongamos que M es mayor que m, entonces los
vectores velocidad correspondientes a la masa M sern menores que los del disco m y,
a
en este caso, se obtiene una representacin grca como la mostrada en la gura 6.26).
o
a

Figura 6.26

Figura 6.27

Elijamos como unidad de magnitud para medir las rapideces a la rapidez que la
part
cula m tiene en el sistema de referencia del centro de masas, o sea, la rapidez
(medida desde el sistema de referencia del centro de masas) de la part
cula m, tanto
antes como despus de la colisin, es 1. La rapidez de la part
e
o
cula M , en esas unidades,
es m/M .
Como ya hemos mencionado, para pasar del sistema de referencia del centro de masas al sistema del laboratorio, debemos sumarle a todos los vectores velocidad del

diagrama 6.26, la velocidad relativa de los dos observadores (que es vM ).

CAP
ITULO 6. MOMENTO LINEAL Y COLISIONES

268

En lugar de sumar un vector constante a todos los vectores de un grco resula


ta ms cmodo simplemente mover el origen en la direccin contraria en esa misma
a o
o

magnitud. Trasladndo el origen en la cantida vM se obtiene la gura 6.27. Cuando


a
los vectores velocidad se observan desde el sistema de centro de masas debe usarse el origen Ocm , mientras que si sto se realiza desde el sistema de referencia del
e
laboratorio, hay que usar el origen Olab .
A partir de la gura 6.27 es fcil determinar el ngulo de dispersin . La gua
a
o
ra 6.28 muestra el detalle del tringulo relevante. Se tiene:
a
a = vm sin(2) = sin(2)

b = vM vm cos(2) =

m
cos(2)
M

y
tan =

Figura 6.28

a
=
b

m
M

sin(2)
.
cos(2)

Figura 6.29

Esta ultima ecuacin coincide con el resultado que hab

o
amos obtenido anteriormente en forma anal
tica (ver ecuacin (6.14)).
o
Si M < m, entonces el punto Ocm cae fuera del c
rculo original (ver gura 6.29).
Note que en este caso el ngulo de dispersin siempre ser menor que un valor
a
o
a
mximo dado por
a
M
.
sin max =
m

CAP
ITULO 6. MOMENTO LINEAL Y COLISIONES

269

Problema:
Considere una part
cula de masa m y velocidad v0 que colisiona con otra part
cula
de masa m, inicialmente en reposo. La energ cintica de la part
a
e
cula incidente es
2
cula # 2 emerge de manera que su vector velocidad forma un
E0 = mv0 /2. La part
a
ngulo de 45 (medido en el sistema de referencia del laboratorio) con respecto a la
direccin incidente. Adems de esta informacin se sabe que el choque es inelstico,
o
a
o
a
siendo Q = 0, 18 E0 la energ que desaparece del movimiento relativo (indose a
a
e
los grados de libertad internos del sistema, transformndose, por ejemplo, en calor).
a
Encuentre grcamente los ngulos (hay dos soluciones) en que emerge la part
a
a
cula
# 1.

Cap
tulo 7

Torque, centro de masas y


equilibrio
versin 3 enero 2011
o

Hasta el momento, hemos sido capaces de describir la evolucin de sistemas f


o
sicos
basados exclusivamente en el concepto de part
cula. De hecho, hemos visto que, en
ms de un sentido, existe un punto especial, el centro de masas, tal que un cuerpo,
a
por complejo que sea, puede ser considerado como una part
cula en que toda la masa
est concentrada en el centro de masas. Pero evidentemente reemplazar un sistema
a
complicado por una unica part

cula puede ser correcto en algunos contextos, pero no


en todos. Alguna diferencia debe hacer que tengamos una part
cula, una esfera o un
elefantes de masa M . En este cap
tulo comenzaremos a examinar las consecuencias de
que los cuerpos tengan estructura, y no sean slo part
o
culas puntuales. Una primera
diferencia evidente es que, si colocamos una part
cula puntual sobre una mesa, por
ejemplo, siempre quedar en reposo. Pero si colocamos un lpiz, no siempre es as y
a
a
,
depender de hecho de la manera en que coloquemos el lpiz sobre la mesa. Por tanto,
a
a
el primer problema que abordaremos es el del equilibrio de cuerpos con estructura.
Para ello, sin embargo, deberemos introducir nuevos conceptos tales como el torque
y el producto vectorial.

7.1.

Producto vectorial

Denamos una nueva operacin entre dos vectores, llamada producto vectorial o
o
producto cruz.
Denicin:
o
Sean A y B dos vectores. Entonces denimos el vector C, que es el producto vectorial
de A y B, por:

C = A B |A| |B| sin C ,


(7.1)
270

CAP
ITULO 7. TORQUE, CENTRO DE MASAS Y EQUILIBRIO

271

donde es el ngulo (ms pequeo) entre los dos vectores A y B, y C es un vector


a
a
n
unitario perpendicular al plano engendrado por los vectores A y B.
Hay dos vectores unitarios que son perpendiculares al plano engendrado por los
vectores A y B. Por convencin debe usarse el que se obtiene usando la regla de la
o
mano derecha.
Regla de la mano derecha: Empue la mano y estire el dedo pulgar. Oriente los
n
dedos empuados de manera que apunten a lo largo del ngulo (desde A hacia B);
n
a

entonces el pulgar indica la direccin y sentido del vector C.


o
De la denicin se desprende que el producto cruz de dos vectores es otro vector.
o
Notemos que la denicin del vector C es independiente de cualquier sistema de
o
coordenadas. Es inmediato que
xx=yy =zz =0,

x y = x = z ,

y
y z = y = x

z

y
z x = z = y .

x
Una caracter
stica importante del producto cruz es que no es conmutativo, sino anticonmutativo; en efecto, de la denicin se observa inmediatamente que:
o
A B = B A
El producto cruz es distributivo respecto a la suma de vectores:
A (B + C) = A B + A C

y
(A + B) C = A C + B C .
Evaluemos el producto cruz entre los dos vectores A y B en trminos de sus coordee
nadas. Sean A y B dos vectores
A = (Ax , Ay , Az ) = Ax x + Ay y + Az z

B = (Bx , By , Bz ) = Bx x + By y + Bz z ,

entonces se tiene
A B = (Ax x + Ay y + Az z ) (Bx x + By y + Bz z )

= Ax Bx x x + Ax By x y + Ax Bz x z + Ay Bx y x + Ay By y y +





+Ay Bz y z + Az Bx z x + Az By z y + Az Bz z z



= (Ax By Ay Bx ) + (Ay Bz Az By ) + (Az Bx Ax Bz )


z
x
y

(7.2)

CAP
ITULO 7. TORQUE, CENTRO DE MASAS Y EQUILIBRIO

272

Observemos que la expresin anterior se puede calcular rpidamente notando que


o
a
el producto cruz es cclico, en el siguiente sentido:

x
x

=
y

x
z

=
z

x
x

=
x

De manera que simplemente tomando el orden alfabtico y aplicando permutae


ciones c
clicas, se obtiene el producto cruz entre dos coordenadas cualesquiera. Si las
coordenadas entre las cuales se calcula el producto cruz no estn en orden c
a
clico,
entonces el resultado es con signo opuesto.
Lo anterior nos permite calcular rpidamente una componente cualquiera del
a
producto cruz entre dos vectores en coordenadas cartesianas. Es evidente, entonces,
que la componente z del producto cruz entre A y B es Ax By Ay Bz , sin necesidad de
hacer la expansin en coordenadas cartesianas. Se puede as reobtener (7.2) de modo
o

expedito.
Es interesante notar lo siguiente: considere
el paralelgramo engendrado por dos veco
tores A y B (ver gura 7.1). De (11.15)
es claro que el rea de tal paralelgramo
a
o
viene dada por

Area = |A B| .

Figura 7.1

Ilustremos el uso del producto cruz con dos problemas.


Problema 1: Sean P1 =(2,1,5), P2 =(5,2,8) y P3 =(4,8,2) las coordenadas de los
vrtices de un tringulo. Calcule su rea.
e
a
a
Solucin: El vector que une los puntos P1 y P2 es
o
A = 3 + y + 3 ,
x
z
mientras que el vector que une los puntos P1 y P3 es
B = 2 + 7 3 .
x
y
z
Ahora observe que el mdulo del producto vectorial de los vectores A y B es igual al
o
doble de rea del tringulo, por lo tanto
a
a

Area del =
=

1
|A B|
2
1
| 24 + 15 + 19| 17, 04
x
y
z
2

CAP
ITULO 7. TORQUE, CENTRO DE MASAS Y EQUILIBRIO

273

Notemos entonces que, dados tres puntos, el producto cruz nos permite determinar rpidamente si tres puntos son colineales (si lo fueran, el producto cruz de
a
los vectores A y B calculados ser cero, pues ellos ser paralelos), y el rea del
a
an
a
tringulo que forman en caso que no lo sean. Ambas preguntas no ser tan sencillas
a
an
de responder si no conociramos el concepto de producto cruz.
e
Problema 2: Sean A y B dos vectores
unitarios en el plano x, y , que forman
a
ngulos y con el eje x, respectiva
mente (ver gura 7.2). Evale el producu
to cruz de estos vectores de dos maneras,
una vez usando la denicin y la segunda
o
vez usando la expresin en trminos de las
o
e
coordenadas cartesianas, y de esta manera
encuentre una expresin para sin( + ).
o

Figura 7.2

Solucin: El ngulo entre los vectores A y B es + , luego


o
a
|A B| = |A| |B| | sin( + )| = sin( + ) .
Por otra parte
|A B| = |(cos x sin y) (cos x + sin y )|

= |(cos sin + sin cos ) z | = cos sin + sin cos .

Igualando las dos expresiones anteriores concluimos que


sin( + ) = cos sin + sin cos .
Ejercicios:

1. Encuentre un vector unitario A que sea simultneamente perpendicular a los


a

vectores u = 2 + y z y v = x y + z . Cuntos vectores unitarios A


x

a
existen con esta propiedad?
2. Sea A = x +32. Encuentre un vector en el plano x, y que sea perpendicular
z y

a A.
3. Verique la expansin del producto vectorial triple:
o
A (B C) = B (A C) C (A B)
por expansin directa en coordenadas cartesianas.
o

CAP
ITULO 7. TORQUE, CENTRO DE MASAS Y EQUILIBRIO

274

4. Considere los vectores A = x + y , B = y + z y C = x z .





a) Evale el producto escalar triple A (B C).
u
b) Evale A (B C).
u

5. Encuentre un vector que sea perpendicular al plano que pasa por los puntos
P1 = (1, 1, 1), P2 = (1, 2, 3) y P3 = (2, 3, 1).
Respuesta: C = (4 = 2 z ) con un nmero real no nulo.
x
y
u
6. Encuentre un vector que apunte a lo largo de la interseccin de los planos
o
engendrados por los vectores A = x + 2, B = x 3 y C = z + 2 y,

x
D = y + 2 , respectivamente.

Problema resuelto en clases: 7.2

7.2.

Torque

Hasta el momento sabemos describir el movimiento de part


culas, y sabemos que
los objetos, aunque sean extendidos, se pueden considerar como part
culas. De hecho,
la fuerza total sobre un objeto es igual al cambio de momentum del centro de masas,
como si toda la masa estuviera concentrada en l. Pero evidentemente no puede ser
e
completamente equivalente un cuerpo extendido que una unica part

cula. Analicemos
por ejemplo la siguiente situacin.
o
Consideremos un objeto (por ejemplo, una
barra) que en cierto instante se encuentra
en reposo. Que la fuerza total sobre la barra sea nula, y por lo tanto (usando la segunda ley de Newton) su aceleracin sea
o
nula, no signica que sta no empezar a
e
a
moverse.
Una situacin de ese tipo se muestra en
o
la gura 7.3. La fuerza total (es decir, la
suma vectorial de la dos fuerzas aplicadas
sobre la barra) es nula y efectivamente la
barra como un todo no se trasladar; sin
a
embargo, las dos fuerzas paulatinamente
harn que la barra rote.
a

Figura 7.3

CAP
ITULO 7. TORQUE, CENTRO DE MASAS Y EQUILIBRIO

275

De hecho, es claro que no slo importa la magnitud y direccin de las fuerzas,


o
o
sino tambin dnde estn aplicadas. En efecto, si las fuerzas estn aplicadas como se
e o
a
a
indica en la Fig. 7.3, la barra comenzar a girar. Pero si las mismas dos fuerzas se
a
aplican en el centro de la barra, entonces no slo el centro de masas no acelera, sino
o
que la barra no gira tampoco.
El ejemplo anterior nos muestra que, para describir completamente el comportamiento del objeto, el concepto de fuerza por s slo no basta, y es necesario introducir
o
algn concepto nuevo que obviamente depende de la fuerza, pero tambin del lugar
u
e
donde sta se aplica.
e
Consideremos ahora la palanca mostrada
en la gura 7.4. Ignoremos por un momento el peso de la palanca. Qu fuerza dee
bemos hacer para mantener la palanca en
equilibrio? Ya en la antigedad los griegos
u
conoc la respuesta:
an
F = Mg

x
.
L

Figura 7.4

Resultados emp
ricos como los anteriores se pueden reunir en la as llamada ley

de las palancas. Lo importante para nosotros, en este caso, es que nuevamente no slo
o
importa la fuerza aplicada (el peso de la masa M ), sino el punto donde se aplica.
Para describir situaciones como las descritas en los ejemplos anteriores necesitamos introducir el concepto de torque.
Denicin: El torque que genera una fuerza F respecto a un punto P es
o
rF ,
donde r es el vector que va desde el punto P hasta el lugar donde se aplica la fuerza F .

Para ganar un poco de intuicin acerca de este concepto, consideremos nuevao


mente una situacin como la Fig. 7.3. Consideremos una barra, ja a un pivote en su
o
centro O.

CAP
ITULO 7. TORQUE, CENTRO DE MASAS Y EQUILIBRIO
F

276

F
O

F
(a)

(b)

(c)

Si ponemos el origen del sistema de coordenadas en el punto O, las unicas fuerzas

que ejercen torque son las fuerzas indicadas, F o F segn corresponda. Consideremos
u
ahora el caso (a) ilustrado en la gura anterior. Cada fuerza ejerce el mismo torque
respecto al punto O (pues tanto la fuerza como el vector posicin cambian de signo,
o
lo cual deja invariante el producto cruz). La direccin del torque ejercido por cada
o
fuerza apunta en la direccin que sale del plano de la gura. Notemos, por otro lado,
o
que en la situacin (a) evidentemente la barra comenzar a girar. Si ponemos los
o
a
dedos empuados en el sentido de la rotacin, el pulgar extendido apunta a lo largo
n
o
del eje de giro, tambin en la direccin que sale del plano de la gura. Por tanto, la
e
o
direccin del torque coincide con la direccin del eje de giro.
o
o
En la situacin (b), en cambio, se aplican las mismas fuerzas en mdulo, pero
o
o
una ha cambiado de direccin. El torque ejercido por cada una de las fuerzas tiene
o
igual mdulo, pero signo opuesto, por tanto el torque total es cero. Y, por cierto, en
o
(b) evidentemente la barra no deber girar.
a
Finalmente, en (c), hemos vuelto a una situacin en que la barra puede girar,
o
simplemente cambiando el punto de aplicacin de una de las fuerzas, logrando de este
o
modo que ambos torques tengan el mismo signo.
Todas estas observaciones nos permiten intuir que es el torque la magnitud responsable de hacer girar a los objetos. No es la fuerza: en los casos (b) y (c) tenemos
las mismas fuerzas, pero aplicadas en puntos distintos, y es sa la diferencia que hace
e
que en un caso la barra gire y en el otro no.
Adems, hemos aprendido que el torque apunta en la direccin del eje de giro y
a
o
en el sentido dado por la regla de la mano derecha (si los dedos empuados indican el
n
sentido de la rotacin entonces el pulgar extendido apunta a lo largo del eje de giro).
o
Claramente, el torque que ejerce una fuerza depende de la posicin del punto
o
(digamos Q) donde sta se aplica y del punto (digamos P ) respecto al cual estamos
e
evaluando el torque. Una fuerza F , respecto a puntos distintos, ejerce torques distintos. Y la misma fuerza, calculada respecto al mismo punto, pero aplicada en puntos
distintos [como en los casos (b) y (c) del ejemplo anterior], tambin ejerce torques
e
distintos.
Notemos, adems, que lo unico relevante para efectos de torque es la componente
a

perpendicular de la fuerza al vector posicin r. Si las fuerzas en la gura anterior no


o

CAP
ITULO 7. TORQUE, CENTRO DE MASAS Y EQUILIBRIO

277

fueran perpendiculares, basta considerar sus componentes perpendiculares a la barra,


y proseguir la discusin slo en trminos de ella.
o o
e
Tambin es interesante notar que, en ocasiones, es ms util calcular el torque
e
a
como el producto de la fuerza por el brazo, como se explica en la siguiente gura.
En el objeto mostrado en la gura 7.5 se
aplica una fuerza en el punto Q. La magnitud del torque se puede evaluar, como
hemos dicho, multiplicando el tamao de
n
la fuerza por el brazo. El brazo es la distancia entre el punto P y recta que indica
la direccin de F que pasa por el punto
o
Q. Con respecto al punto P1 el mdulo del
o
torque producido por la fuerza F es F a,
donde F = |F | y a es el brazo. El vector
apunta normal a la hoja de papel en direccin del lector (en la direccin de giro
o
o
que tendr el objeto si pudiera girar en
a
torno a P1 , slo debido a la accin de la
o
o
fuerza F ). Respecto al punto P2 el torque
generado por la fuerza F es nulo, ya que
el brazo es nulo.

Figura 7.5

El conocimiento emp
rico que se tiene sobre palancas, y en general, sobre objetos
en equilibrio (es decir, objetos que no aceleran ni comienzan a rotar) permite enunciar
la siguiente ley:
Si el torque neto ejercido por las fuerzas que actan sobre un cuerpo,
u
respecto a un punto P , es nulo, entonces el cuerpo no cambiar su estado
a
rotacional (o sea, si no estaba rotando en torno a P , no comenzar a rotar
a
en torno a ese punto y si estaba rotando con cierta velocidad angular,
seguir rotando con la misma velocidad angular).
a
Notemos que el enunciado anterior es anlogo a la segunda ley de Newton, reema
plazando fuerza por torque y velocidad por velocidad angular.
Ejercicio: Demuestre que para la situacin mostrada en la gura 7.4, el torque neto,
o
en torno al punto 0, ejercido por las tres fuerzas que actan sobre la palanca, es nulo.
u
(Ignore el peso de la palanca.)

CAP
ITULO 7. TORQUE, CENTRO DE MASAS Y EQUILIBRIO

7.3.

278

Centro de masas

La evaluacin del torque debido al peso de


o
un objeto se simplica considerablemente
si se introduce el concepto de centro de
masas, ya mencionado en cap
tulos anteriores.
Consideremos dos masas m1 y m2 , unidas
por una barra de masa despreciable, dentro de un campo gravitacional g = g.
z
Evaluemos el torque neto debido al peso
de las masas en torno al punto P .

Figura 7.6

Tenemos:

= r1 (m1 g) + r2 (m2 g)
z
z
(m1 r1 + m2 r2 )
(M g)
z
=
M

Sea M = m1 + m2 y denamos
rcm =

1
(m1 r1 + m2 r2 ) ,
M

entonces
= rcm (M g) .
z
O sea, una vez conocido el vector de posicin del centro de masas rcm , podemos
o
evaluar el torque debido a la fuerza de gravedad suponiendo que la masa total del
objeto se encuentra en ese lugar.
El concepto centro de masas ha aperecido en varias oportunidades, y nuevamente
aparece ahora como un punto privilegiado. Analicemos con un poco ms de profuna
didad algunas de sus propiedades.
Para N masas discretas {mj }j en los lugares {rj }j , la posicin del centro de
o
masas viene dada por
N
1
mj rj ,
rcm =
M
j=1

con M = m1 + m2 + + mN .
Consideremos ahora el caso de una distribucin continua de masa, por ejemplo,
o
una lmina:
a

CAP
ITULO 7. TORQUE, CENTRO DE MASAS Y EQUILIBRIO

279

^
y
dy
dx
r
^
x

( x, y )

Consideremos, como muestra la gura, una lmina ubicada sobre el plano x-y.
a
Dividamos la lmina en N pequeos cuadraditos de ancho dx y alto dy. Si estos
a
n
cuadraditos (tcnicamente, elementos de supercie) son sucientemente pequeos,
e
n
entonces deber
amos ser capaces de cubrir toda el rea de la lmina con ellos. Cada
a
a
elemento de supercie tiene un rea dx dy, y por tanto una masa (ri ) dx dy, donde
a
(ri ) es la densidad supercial de masa en el punto ri = (xi , yi ).1 Entonces el centro
de masas se puede calcular como:
rcm =

N
i=1 (ri )ri dx dy
N
i=1 (ri )dx dy

En el l
mite en que los elementos de supercie son innitamente pequeos, la suma
n
discreta sobre i se convierte en una integral sobre la variable continua r. r recorre
todos los puntos del espacio necesarios para cubrir toda la lmina, y podemos por
a
tanto escribir:
1
rcm =
(x + y y ) (x, y) dx dy ,
x

M lmina
a
donde M es la masa total de la lmina, que se puede calcular como
a
M=

lmina
a

(x + y y) (x, y) dx dy .
x

En el caso de un cuerpo slido, con volumen, de densidad (x, y, z), podemos


o
dividirlo en cubos de aristas dx, dy y dz, y el mismo anlisis anterior nos indica que
a
la posicin del centro de masas viene dada por
o
rcm =

1
M

cuerpo

r (x, y, z) dx dy dz ,

con
M=
1

cuerpo

r (x, y, z) dx dy dz .

Normalmente, se denotan por a las densidades lineales, a las densidades superciales y a


las densidades volumtricas.
e

CAP
ITULO 7. TORQUE, CENTRO DE MASAS Y EQUILIBRIO

280

Las integrales que han aparecido en las expresiones anteriores son integrales en dos y
tres dimensiones, que no hemos encontrado antes. Ms adelante en el curso tendrea
mos que introducir tcnicas para calcularlas. De momento, slo nos interesa dejar el
e
o
mensaje de que, si tenemos distribuciones continuas de masa, podemos discretizar la
distribucin y calcular el centro de masas de la nueva distribucin, discreta, de masas.
o
o
Las integrales en dos y tres dimensiones que hemos mostrado no son sino una notacin matemtica conveniente para describir ese proceso, y en la Sec. 7.4 veremos un
o
a
ejemplo de clculo numrico del centro de masas basado en este proceso l
a
e
mite. Salvo
dicho ejemplo, por ahora bastar con saber calcular el centro de masas de cuerpos
a
con distribuciones uniformes y ejes de simetr y para ellos es conveniente revisar los
a,
siguientes ejercicios, que se reeren a importantes propiedades del centro de masas.
Ejercicios:
1. A pesar de que el vector centro de masas rcm depende del origen que se elija
para evaluarlo, la posicin del centro de masas es independiente de la eleccin
o
o
del origen.
Sea rcm el vector posicin del centro de
o
masas de un objeto evaluado usando un
sistema de referencia cuyo origen es O y

rcm el resultado que se obtiene usando


otro sistema de coordenadas cuyo origen
es O . Demuestre que
rcm = r + a ,
cm
donde a es el vector que une los dos or
genes.

Figura 7.7

2. Considere un sistema que consiste de dos masas m y M puntuales separadas


por una distancia a. Demuestre que la posicin del centro de masas del sistema
o
se ubica sobre la recta que las une, encontrndose ms cercano a la masa mayor.
a
a
3. Considere tres masas m1 = m0 , m2 = 3m0 y m3 = 6m0 , ubicadas en los lugares
r1 = x + 3 , r2 = + 2 y r3 = 5 + 3 2 ,

y
x
z
x
y
z
respectivamente. Encuentre la posicin del centro de masas usando los dos proo
cedimientos siguientes:
a) Usando la frmula
o
rcm =

mi ri
.
j mj

CAP
ITULO 7. TORQUE, CENTRO DE MASAS Y EQUILIBRIO

281

b) Encontrando primero el centro de masas del subsistema formado por las


part
culas 1 y 2 solamente y luego encontrando el centro de masas de este
subsistema con la part
cula # 3.
c) Demuestre que, en general, para un sistema de N masas, el centro de masas
se puede encontrar determinando primero el centro de masas de N 1
(N 1)
masas, rcm , y luego evaluando el centro de masas de dos part
culas:
N 1
la part
cula N -sima, y una part
e
cula de masa
i=1 mi ubicada en
(N 1)
rcm .
4. Convnzase de que si un objeto tiene ejes y planos de simetr entonces el
e
a,
centro de masas se ubica en tales planos y ejes. Por ejemplo, de acuerdo a este
resultado, en una esfera, un cilindro recto, etc., el centro de masas se ubicar al
a
centro de tales objetos.
Problema resuelto en clases: 7.4

7.4.

Evaluacin numrica del centro de masas de un seo


e
mic
rculo

Problema: Encontrar numricamente el centro de masas de una lmina semicircular


e
a
de radio R y densidad supercial uniforme 0 .
Solucin: Dividamos el semic
o
rculo en N franjas de ancho dz = R/N y luego aproximemos las franjas por rectngulos (ver gura 7.8). El centro de masa del rectngulo
a
a
j se encontrar en el lugar
a
rj =

j dz

1
dz
2

z=

R (j 1/2)
z

j = 1, . . . , N .

El rea del rectngulo j viene dada por


a
a
Aj = (ancho) (largo)
= dz 2 xj
R

= 2
N
R
= 2
N

R2 R2 (j 1)2 /N 2
2

N 2 (j 1)2

CAP
ITULO 7. TORQUE, CENTRO DE MASAS Y EQUILIBRIO

282

Figura 7.8
Encontremos ahora el centro de masas de los centros de masas de todos los rectngua
los. Se tiene:
a
j rj (masa de rectngulo j)
Rcm =
(masa del semic
rculo)
=
j

4R
N 3

R(j 1/2)
z 0 2

R
N

N 2 (j 1)2

1
0 2

1
R2

N 2 (j 1)2 (j 1/2) z

j=1

= f (N )R ,
z
donde
f (N )

4
N 3

N
j=1

N 2 (j 1)2 (j 1/2)

El valor exacto para Rcm se obtiene para valores grandes de N .


Para valores de N no demasiado grandes podemos evaluar f (N ) con una calculadora (hgalo para N = 1 y N = 2 y compare su resultado con el de la tabla). Para
a
valores grandes de N debemos hacer un pequeo programa y usar una computadora.
n
Un pequeo programa en BASIC que permite evaluar f (N ) (para N = 500) es:
n
PI = 3.1415926
N = 500
S=0
FOR J = 1 TO N
S = S + SQR(N*N (J 1)*(J 1)) * (J 0.5)
NEXT J
F = 4*S / (PI*N*N*N)
PRINT N , F

CAP
ITULO 7. TORQUE, CENTRO DE MASAS Y EQUILIBRIO
N
1
2
3
5
10
20
50
100
200
500
1000
Exacto

f (N )
0.6366
0.5826
0.5344
0.4972
0.4642
0.4456
0.4334
0.4390
0.4268
0.4254
0.42490
0.4244

283

Error relativo %
50
35
26
17
9.4
5.0
2.1
1.1
0.56
0.24
0.12
Figura 7.9

Los resultados que arroja este programa para distintos valores de N , se presentan
en la tabla adjunta. Recurriendo al clculo integral, es posible encontrar el resultado
a
exacto, es decir, el valor de f (); ste resulta ser 4/(3) = 0.4244 . . . En la gura 7.9
e
se muestra un grco del error relativo entre el valor nmerico y el valor exacto en
a
u
funcin de N . A partir de N = 100 el error es menor que un 1 %.
o
Nota: El mtodo numrico empleado aqu para resolver el problema no es el ms
e
e

a
eciente. La bondad del mtodo empleado radica en su simpleza.
e
Ejercicio: Use un procedimiento anlogo para calcular la posicin del centro de masas
a
o
de una semiesfera de radio R y densidad de masa (uniforme) 0 .

7.5.

Equilibrio

Ahora que consideramos sistemas con estructura, las condiciones de equilibrio


para un cuerpo deben ser reformuladas. En efecto, ya sabemos que un objeto o sistema inicialmente en reposo permanecer en reposo si la fuerza neta sobre l es cero.
a
e
Esto es en rigor cierto slo para part
o
culas puntuales. Cuando tenemos sistemas ms
a
complejos, lo anterior slo asegura que el centro de masas permanece en reposo [ver
o
Ec. (6.3)]. Pero eso no basta, porque el cuerpo an podr girar respecto al centro de
u
a
masas. Por lo tanto, para que un cuerpo est en equilibrio debemos exigir, adems,
e
a
que el torque neto sobre l (calculado respecto a cualquier punto) sea tambin nulo.
e
e
Leyes de equilibrio:

CAP
ITULO 7. TORQUE, CENTRO DE MASAS Y EQUILIBRIO

284

Para que un cuerpo est en equilibrio es necesario que se cumplan las


e
siguientes dos condiciones:
i) La fuerza neta sobre el objeto debe ser nula.
ii) El torque neto sobre el objeto debe ser nulo.
Consideremos un objeto (cuerpo r
gido)
formado por N masas {mj } ubicadas en
los lugares {rj } (respecto a un origen O)
y unidas por barras r
gidas sin masas
(ver gura 7.10). Sea Fj la fuerza externa
que acta sobre cada una de las masas mj .
u
A continuacin, usando esta gura, deo
mostraremos dos resultados importantes:
Figura 7.10
Si la fuerza neta sobre un cuerpo es cero entonces el torque neto es independiente del punto respecto del cual se evala. En particular, si el torque
u
es nulo repecto a un punto, tambin lo ser respecto a cualquier otro pune
a
to.
Demostracin:
o
o
Sean {rj } y {r j } los vectores de posicin de las masas {mj } respecto a un origen O
a
y O, respectivamente. Sea adems R el vector que une los puntos O y O. Entonces

=
j

=
j

=
j

rj Fj
(R + rj ) Fj
R Fj +

= R

r j Fj

Fj + = R 0 + = .

El otro resultado importante es el siguiente:


Si la fuerza neta Ftot que acta sobre un cuerpo de masa M no es nula,
u
entonces el punto del cuerpo que es acelerado de acuerdo a la segunda ley
de Newton es el centro de masas. O sea, se tiene que:

Ftot = M rcm .

CAP
ITULO 7. TORQUE, CENTRO DE MASAS Y EQUILIBRIO

285

Demostracin:
o
En primer lugar notemos que las barras que unen las distintas masas en la gura 7.10
transmiten fuerzas. Sea Fij la fuerza que ejerce la masa mj sobre la masa mi . Debido
al principio de accin y reaccin Fji = Fij .
o
o
Por lo tanto, se cumplen las mismas condiciones del sistema estudiado en la
Sec. 6.1, y por tanto podemos armar que (6.3) es vlida.
a
En otras palabras, el lugar geomtrico que cumple con la segunda ley de Newton
e
es el centro de masas. O sea, para analizar el movimiento traslacional, toda la masa se
puede pensar como si estuviese concentrada en el centro de masas, siendo se tambin
e
e
el lugar en que se aplica la fuerza neta. Se sigue entonces que si la fuerza neta que
acta sobre un cuerpo es nula, entonces el centro de masas del cuerpo se traslada con
u
velocidad uniforme (o nula).
Observemos nalmente que, para estudiar sistemas en equilibrio, no es necesario
especicar el punto respecto al cual se est evaluando el torque neto, ya que, de
a
acuerdo a los resultados expuestos anteriormente, si la fuerza neta es nula y el torque
es nulo respecto a un punto, tambin lo ser con respecto a cualquier otro punto.
e
a
Ilustremos el uso de las leyes del equilibrio resolviendo un problema.
Problema:
Una escalera de masa m y largo L se encuentra apoyada contra una pared lisa (o
sea, no hay roce entre la escalera y la pared), formando un ngulo con ella. Una
a
persona de masa M se encuentra sobre la escalera. Cul es el m
a
nimo coeciente
de roce esttico que debe existir entre el suelo y la escalera para que la escalera no
a
resbale, independientemente de la altura a la que se encuentra la persona?
Solucin:
o
Introduzcamos el sistema de coordenadas mostrado en la gura adjunta. Para que el
sistema se encuentre en equilibrio, la fuerza total sobre la escalera debe ser nula. Hay
cuatro fuerzas actuando sobre la escalera:

CAP
ITULO 7. TORQUE, CENTRO DE MASAS Y EQUILIBRIO

286

i) El peso de la escalera mg; esta


z
fuerza se aplica en el centro de masas
de la escalera.
ii) El peso de la persona M g.
z
iii) La reaccin que ejerce la pared sobre
o
la escalera. Como la pared es lisa (no
hay roce) tal fuerza es perpendicular
a la pared: Fp = Fp x.

iv) La reaccin del suelo sobre la escao


lera. Esta es Fs = FN z + fr x, donde

FN es la fuerza normal y fr es la
fuerza de roce.

Figura 7.11

La condicin de que la fuerza total sea nula nos da la relacin:


o
o
mg M g Fp x + FN z + fr x = 0 .
z
z

De aqu se deducen la ecuaciones

FN = (m + M )g

(7.3)

fr = Fp .

(7.4)

y
Evaluemos el torque total en torno al origen. Como la escalera est en equilibrio, el
a
torque neto debe ser nulo. Se tiene:
M gx y + mg

L
sin y Fp L cos y = 0 ,

o sea,

g(M x + m L sin )
2
.
(7.5)
L cos
De las ecuaciones (7.4) y (7.5) se encuentra que la fuerza de roce viene dada por
Fp =

g(2M x + mL sin )
.
2L cos
El valor mximo de la fuerza de roce se obtiene cuando la persona sube hasta la parte
a
ms alta de la escalera (x = L cos ). En ese caso la fuerza de roce es
a
fr =

m
tan .
2
La fuerza de roce fr debe ser menor que el mximo posible, que es e FN , o sea, se
a
debe cumplir la relacin
o
m
g M+
tan e FN = e (M + m)g .
2
fr = g M +

CAP
ITULO 7. TORQUE, CENTRO DE MASAS Y EQUILIBRIO

287

De aqu se deduce que el menor valor posible que puede tener e para que la escalera

no resbale es
2M + m
min =
tan .
e
2(M + m)
Problema resuelto en clases: 7.9 (opcional)
Problema resuelto en clases: 7.10

7.6.

Problemas

1. Al moverse una carga q con velocidad v en presencia de un campo magntico B,


e
acta sobre la part
u
cula una fuerza (la as llamada Fuerza de Lorentz) dada

por
F = q (v B) .
Supongamos que para determinar la direccin y magnitud de un campo magntio
e
co constante, un investigador realiza dos experimentos, midiendo en cada uno
de ellos la fuerza que acta sobre una carga:
u
a) Primero hace pasar la carga q a travs del campo magntico con velocidad
e
e
v = v0 x. El investigador mide una fuerza F = F0 (2 4).

z
y

b) Luego hace pasar la carga q con velocidad v = v0 z , midiendo una fuerza

F = F0 ( 2).
y
x

A partir de estos resultados encuentre el campo magntico B (en funcin de v0 ,


e
o
F0 y q).
Respuesta:

B=

F0
( + 2 + 4) .
x
y
z
qv0

2. Considere una part


cula cuya carga elctrica y masa es q y m, respectivamente.
e
La part
cula se mueve en un campo magntico homogneo B = B0 z . Si en el
e
e

instante t = 0 la part
cula se encuentra en el origen (r(0) = 0) y su velocidad
en ese instante es v(0) = v0 x, encuentre el vector de posicin r(t) en funcin

o
o
del tiempo. (La fuerza que el campo magntico ejerce sobre la part
e
cula viene
dada por la fuerza de Lorentz ; ver problema anterior.) Indicacin: recuerde lo
o
que sabe sobre el movimiento circular uniforme.
3. Demuestre que la posicin del centro de masas de una lmina triangular de
o
a
densidad uniforme se ubica en el lugar donde se cortan las tres transversales de
gravedad del tringulo.
a

CAP
ITULO 7. TORQUE, CENTRO DE MASAS Y EQUILIBRIO

288

4. En cinco de los seis vrtices de un hexgono regular hay una masa m0 . Encuentre
e
a
la posicin del centro de masas.
o
5. Encuentre la posicin del centro de masas
o
de una lmina de densidad (de masa) unia
forme 0 y que tiene la forma indicada en
la gura adjunta.
Figura 7.12
6. Encuentre la posicin del centro de masas
o
de un disco de densidad supercial 0 y
que tiene un agujero circular como se indica en la gura adjunta.
Respuesta: El centro de masas del disco con agujero queda al lado opuesto de
la perforacin y a una distancia a =
o
2 d/(R2 r 2 ) del centro del disco de radio
r
R.

Figura 7.13

7. La gura muestra un letrero luminoso de


masa m que cuelga de una barra (de masa despreciable) que se mantiene horizontal con la ayuda de una cuerda. Calcule la
tensin de la cuerda y la fuerza ejercida
o
por la barra contra la pared.
Figura 7.15
8. Considere una estructura formada por dos
barras uniformes de largos a y b, unidas
de modo que forman un ngulo recto y
a
que cuelga con hilo desde el cielo (ver gura adjunta). Determine el ngulo de
a
la estructura cuando ella se encuentra en
equilibrio.
Figura 7.14
9. Describa un procedimiento que permita determinar experimentalmente la posicin del centro de masas de una lmina plana irregular con densidad desconocida
o
a
(y no necesariamente uniforme).
(Observe que al colgar un cuerpo de un punto P y estar ste en equilibrio, el
e
centro de masas siempre debe estar sobre la normal que pasa por P .)

CAP
ITULO 7. TORQUE, CENTRO DE MASAS Y EQUILIBRIO

289

10. Una barra, cuya masa es de 10 Kg y tiene tres metros de largo, se dobla en 45
a 1 m de uno de los extremos y se cuelga como se indica en la gura adjunta.
La estructura se encuentra en equilibrio gracias a una masa M que se cuelga en
uno de los extremos.
a) Encuentre la tensin T y el valor de
o
M . El equilibrio es estable o inestable?
b) Conteste nuevamente las mismas
preguntas de la parte a), pero asumiendo ahora que la barra al lado
izquierdo, en lugar de estar doblada hacia abajo en 45 , est doblada
a
hacia arriba en 45 .

Figura 7.16

11. Considere una lmina triangular unifora


me, de masa M = 5 Kg, que est sujeta
a
a una pared con una articulacin y colgao
da del cielo con una cuerda, tal como se
muestra en la gura adjunta. Encuentre
la tensin T de la cuerda.
o
Figura 7.17
12. Encuentre la posicin de equilibrio de una
o
varilla de largo L colocada dentro de un
pocillo. Considere al pocillo como una semiesfera de radio R y asuma que entre ste
e
y la varilla no hay roce.
Figura 7.18
13. Se podr fomar una torre con ladrillos
a
(sueltos), uno encima de otro (ver gura), de manera que el ladrillo de ms
a
arriba est desplazado en ms de una
e
a
unidad con respecto al de ms abajo,
a
sin que la torre se desplome ?
Indicacin: Comience el anlisis con los
o
a
ladrillos superiores.

Figura 7.19

CAP
ITULO 7. TORQUE, CENTRO DE MASAS Y EQUILIBRIO
14. Tres tambores del mismo radio estn
a
arrumbados como se indica en la gura adjunta. Encuentre el m
nimo coeciente de roce esttico que debe existir
a
entre los tambores y tambin entre los
e
tambores y el suelo de manera que el
sistema no se derrumbe.

290

Figura 7.20

15. Un tringulo equiltero, de lado a = 10 cm y masa M = 10 kg se sujeta en forma


a
a
r
gida a una polea de radio R = 4 cm. El tringulo acta de contrapeso para
a
u
mantener en equilibrio a una masa m = 1 kg que cuelga de un hilo enrollado en
la polea (ver gura 7.18)
a) Encuentre el valor del ngulo
a
que mantiene el sistema en equilibrio. ( es el ngulo entre la nora
mal y la altura del tringulo.)
a
b) Cul es el mximo valor de m
a
a
para el cual el sistema se mantendr en equilibrio?
a
16. De una pieza metlica cuadrada, de
a
densidad supercial uniforme 0 , se recorta un tringulo issceles de manea
o
ra tal que la lmina resultante quede
a
en equilibrio en cualquier posicin si se
o
sujeta desde el punto P . Cul es la ala
tura del tringulo?
a

Figura 7.21

Figura 7.22

17. Una barra de masa M y largo L, que


puede pivotear libremente en torno
a O, se mantiene en equilibrio con
una masa m y una cuerda, tal como se muestra en la gura adjunta.
Encuentre el ngulo para el caso
a
en que m/M = 0.5. El equilibrio es
estable o inestable?
Figura 7.23

CAP
ITULO 7. TORQUE, CENTRO DE MASAS Y EQUILIBRIO
18. Considere un puente cuyo armazn
o
consiste de 14 soportes de largo a.
(En la gura se observan los 7 soportes de uno de los lados.) Asuma que los soportes slo transmiten
o
fuerzas a lo largo de ellos, o sea, en
cada unin slo se transmiten fuero o
zas y no torques. Encuentre la tensin adicional (al generado por el peo
Figura 7.24
so del puente) que debe soportar cada soporte si por el centro del puente
pasa un camin de peso W .
o
Especique en cada caso si la tensin es de compresin o de traccin.
o
o
o
19. Una cadena de masa M y largo L
se encuentra apoyada (en equilibrio)
sobre un cono recto cuyo ngulo ena
tre la normal y el manto es . Encuentre la tensin de la cadena.
o
Indicacin: Aplique las leyes de
o
equilibrio a un pequeo trozo (inn
nitesimal) de cadena.

Figura 7.25

20. Un objeto formado por tres lminas


a
cuadradas de lado a, homogneas y
e
de igual densidad, descansa sobre
una supercie horizontal apoyado en
dos pivotes colocados en los vrtie
ces del cuadrado inferior (ver gura 7.26).
a) Encuentre la posicin del ceno
tro de masas.
b) Calcule la razn de la magnio
tud de las fuerzas de reaccin
o
de cada pivote.
21. Una regla T de masa M , largo a y barra
transversal b posa sobre un plano horizontal pulido como se indica. Calcule las reacciones normales en cada punto de contacto
con el suelo.

Figura 7.26

Figura 7.27

291

CAP
ITULO 7. TORQUE, CENTRO DE MASAS Y EQUILIBRIO

292

22. Considere una semiesfera homognea de radio R. Demuestre


e
que el centro de masas de la semiesfera est ubicado sobre el eje de
a
simetr y a una distancia b = 3R/8
a
de la base.
Figura 7.28
23. Considere una semiesfera homognea de radio R que se
e
encuentre sobre un plano inclinado.
Existe un roce esttico que evita
a
que la semiesfera se deslice por el
plano. Determine el mximo ngulo
a
a
de inclinacin que puede tener el
o
plano para que la semiesfera no se
d vuelta.
e
24. Considere una semiesfera de radio
R, hecha de un material de densidad 0 , que se encuentra con la parte
curva sobre una supercie horizontal (ver gura adjunta). El centro de
masas de una semiesfera homognea
e
queda sobre el eje de simetr a una
a
distancia b = 3R/8 de la base.

Figura 7.29

Figura 7.30

a) Encuentre la magnitud y direccin del torque, respecto al punto de apoyo,


o
ejercido por la fuerza de gravedad cuando la semiesfera se ladea en un
a
ngulo . Observe que el torque que aparece trata de hacer volver a la
semiesfera a su posicin de equilibrio (o sea, la posicin de equilibrio es
o
o
estable).
b) Coloquemos ahora un cilindro homogneo hecho del mismo material, del
e
mismo radio R y altura h, sobre el cilindro. Determine la posicin del
o
centro de masas del sistema compuesto.
c) Describa en palabras la condicin que debe satisfacer la posicin del centro
o
o
de masas para que la posicin de equilibrio del sistema compuesto siga
o
siendo estable.
d) Encuentre la altura l
mite del cilindro para la cual el sistema compuesto
pierde su estabilidad.

CAP
ITULO 7. TORQUE, CENTRO DE MASAS Y EQUILIBRIO
25. Considere una semiesfera de radio R, hecha de un material de densidad 0 , que se
encuentra sobre una supercie horizontal
y apoyada contra una pared tal como se
muestra en la gura adjunta. El centro de
masas de una semiesfera homognea quee
da sobre el eje de simetr y a una disa
tancia b = 3R/8 de la base. Suponga que,
entre la semiesfera y el suelo el coeciente de roce esttico es = 3/16, mientras
a
que entre la pared y la semiesfera el roce
es nulo.

293

Figura 7.31

a) Haga un diagrama de cuerpo libre para la semiesfera.


b) Encuentre la magnitud y direccin del torque, respecto al punto de apoyo
o
P , ejercido por la fuerza de gravedad cuando la semiesfera est ladeada en
a
un ngulo .
a
c) Encuentre la fuerza de roce entre la semiesfera y el suelo.
d) Encuentre el ngulo de inclinacin mximo max posible para que la esfera
a
o
a
no resbale.
e) Coloquemos ahora un cilindro homogneo, hecho del mismo material, del
e
mismo radio R y de altura h sobre el cilindro. Determine la posicin del
o
centro de masas del sistema compuesto. (1 punto)
f) Encuentre la altura l
mite hmax del cilindro a partir de la cual, para h >
hmax , el sistema compuesto se da vuelta (es decir, pierde su estabilidad).
26. Una semiesfera homognea de masa M y
e
radio R se ha cortado en dos mitades. El
sistema se dispone con las dos mitades, cara a cara, y con la supercie de corte vertical. A n de que las mitades no se separen,
una cuerda sin roce y con masas iguales en
sus extremos, es dispuesta como se indica
en la gura. Determine las masas m
nimas
a atar en los extremos de la cuerda para
que las mitades permanezcan juntas.

Figura 7.32

CAP
ITULO 7. TORQUE, CENTRO DE MASAS Y EQUILIBRIO
27. En los extremos de una barra de masa despreciable se adhieren bolas de mas m y
2m, respectivamente. El sistema posa sobre un tiesto de fondo esfrico resbaloso,
e
de radio igual al largo de la barra. Calcule el ngulo que la barra forma con la
a
vertical.
28. Un vaso cil
ndrico (abierto por arriba), de
radio basal a y altura b, hecho de un material de densidad supercial uniforme, posa
sobre un plano inclinado y no resbala gracias a un tope jo en el plano. Demuestre
que el centro de masas se ubica a lo largo
del eje y a una distancia b2 /(a + 2b) de la
base. Determine el ngulo de inclinacin
a
o
mximo del plano de modo que el vaso no
a
vuelque.
29. En la gura se muestra un cilindro de masa M y radio R, el cual se ata a la muralla
mediante una cuerda. Alrededor de un calado que se le ha hecho al cilindro se enrolla una cuerda ideal. De la cuerda cuelga
una masa m por determinar. Si el coeciente de roce entre el suelo y el cilindro
es , determine la masa mxima a colgar
a
para que el cilindro no rote.
30. Un semicilindro de radio R y peso W se
encuentra en equilibrio esttico sobre un
a
plano horizontal, con un pequeo bloque
n
de peso Q sobre l. El bloque est ligado
e
a
mediente un resorte ideal de largo natural
0 = R y constante elstica k a un punto
a
A en el borde (ver gura). Suponga que no
hay roce entre la supercie del cilindro y la
masa de peso Q. Determine el ngulo de
a
equilibrio. Considere conocida la distancia
D a la que se encuentra el centro de masas
del punto O. Analice con cuidado que pasa
cuando Q es pequeo.
n

Figura 7.33

Figura 7.34

Figura 7.35

Figura 7.36

294

CAP
ITULO 7. TORQUE, CENTRO DE MASAS Y EQUILIBRIO

295

31. En un disco homogneo de densidad supercial de masa y radio R se hacen dos


e
perforaciones, una cuadrada de arista a, y otra circular de radio a. La distancia
desde los bordes interiores de ambas perforaciones (puntos P y Q) al centro
del disco (punto O) es b. Los centros del disco y de ambas perforaciones son
colineales. La arista de la perforacin cuadrada que pasa por Q es perpendicular
o
al trazo P Q. Para qu valor de b el centro de masas del sistema se encuentra
e
en el punto Q?

b
P

O
R

7.7.

Solucin a algunos de los problemas


o

Solucin al problema 12
o
Elijamos el origen y los ejes tal como
se muestra en la gura adjunta. Sea
el ngulo que la varilla forma con la
a
horizontal, o sea, el ngulo ABO es .
a
Por ser AOB un tringulo issceles, se
a
o
tiene que el ngulo AOB tambin es .
a
e
Figura 7.37
Como no hay roce entre las supercies de contacto, las fuerzas de reaccin debe ser
o
perpendiculares a las supercies de contacto. En otras palabras: la fuerza de reaccin
o
F1 en O ser a lo largo del radio OA, mientras que la fuerza de reaccin F2 en B
a
o
ser perpendicular a la varilla. Que la fuerza total horizontal sobre la varilla sea cero
a
nos da la relacin
o
F1 cos(2) = F2 sin .
(7.6)

CAP
ITULO 7. TORQUE, CENTRO DE MASAS Y EQUILIBRIO

296

La relacin correspondiente para la componente vertical es


o
M g = F2 cos + F1 sin(2) .

(7.7)

Para que la varilla est en equilibrio tambin el torque total (respecto a O) debe
e
e
ser nulo. La fuerza F1 no ejerce torque (ya que su brazo es cero); el peso ejerce
un torque g = M g (L/2) cos y , mientras que el torque generado por F2 es 2 =

F2 2R cos y . De esta manera la condicin de que el torque total sea nulo nos da la

o
relacin
o
L
M g cos = 2F2 R cos .
(7.8)
2
Tenemos tres ecuaciones con tres incgnitas. De la ultima ecuacin se deduce inmeo

o
diatamente que
M gL
F2 =
.
(7.9)
4R
De la ecuacin (7.6) se encuentra
o
F1 = F2
Usando (7.9) y (7.10) en (7.7), se tiene
Mg =
o sea,

sin
.
2 cos2 1

M gL
sin 2 sin
cos +
4R
2 cos2 1

(7.10)

4R
2 cos (1 cos2 )
cos
= cos +
=
.
21
L
2 cos
2 cos2 1
La ultima relacin es una ecuacin de segundo grado para cos ; resolvindola se

o
o
e
encuentra nalmente

1 + 1 + 82
.
cos =
4

Solucin al problema 14
o
La gura adjunta muestra las fuerzas que
actan sobre los tambores (las echas con
u
l
neas llenas son fuerzas que actan sobre
u
el tambor inferior, mientras que las echas
con l
neas segmentadas corresponden a fuerzas que actan sobre el tambor superior). Obu
serve que no hay una fuerza horizontal entre
los dos tambores inferiores. Observe tambin
e
que el ngulo entre la horizontal y la reca
ta que une a los centros de un tambor inferior con tambor superior es de 60 , luego
el
sin = 3/2 y cos = 1/2.

Figura 7.38

CAP
ITULO 7. TORQUE, CENTRO DE MASAS Y EQUILIBRIO

297

La unicas fuerzas que producen un torque sobre el tambor inferior (respecto a

su centro) son las dos fuerzas de roce. Como el torque total sobre el tambor inferior
debe ser nulo se deduce que ambas fuerzas de roce deben tener la misma magnitud;
llammosla fr .
e
La fuerza neta vertical sobre uno de los tambores inferiores debe ser nula; esto
nos da la relacin
o

1
3
fr = 0 .
F1 M g F2
2
2
La relacin correspondiente a las fuerzas horizontales es
o

3
1
fr + fr
F2 = 0 .
2
2
Como 2F1 debe ser igual al peso total de los tres tambores se tiene
3
F1 = M g .
2
Tenemos tres ecuaciones con tres incgnitas (en realidad la tercera ecuacin ya es la
o
o
solucin de una de las incgnitas). Para las otras dos incgnitas fr y F2 se encuentra
o
o
o
fr =

3
2

Mg

1
F2 = M g .
2
Si 1 es el coeciente de roce esttico entre el tambor inferior y el suelo, y 2 es el
a
coeciente de roce entre los tambores inferior y superior, entonces, para que el sistema
no se derrumbe, debe cumplirse
fr 1 F1

fr 2 F2 .

De estas desigualdades se deduce, nalmente, que

2 3
1
y
2 2 3 .
3

CAP
ITULO 7. TORQUE, CENTRO DE MASAS Y EQUILIBRIO
Solucin al problema 19
o
El radio del c
rculo que forma la cadena es
R=

L
.
2

Consideremos un trozo de cadena de largo


innitesimal R d. Debido a la curvatura, la
tensin T ejerce sobre el pequeo trozo de
o
n
cadena una fuerza neta FT hacia el centro O
(ver gura 7.39a):
FT = 2T sin

d
2

r = T d r .

Figura 7.39a

No hay roce entre la cadena y el cono, luego


la fuerza que el cono ejerce sobre la cadena
es perpendicular al manto. Sea FN la magnitud de esta fuerza. De la la gura 7.39b se
desprende que
FN = FN cos r + FN sin z .

Por ultimo, la otra fuerza que acta sobre el

u
trozo de cadena, debido a la gravedad, es
Fg =

d
Mg z .

Como el trozo de cuerda est en reposo, la


a
suma de las tres fuerzas debe ser nula, es decir,

Figura 7.39b

FT + FN + Fg = T d r + FN cos r + FN sin z +

Igualando las componentes se obtienen las relaciones


T d = FN cos
y

d
= FN sin .
2
Despejando la tensin se encuentra, nalmente
o
Mg

T =

Mg
.
2 tan

d
Mg z = 0 .

298

CAP
ITULO 7. TORQUE, CENTRO DE MASAS Y EQUILIBRIO

299

Solucin al problema 22
o
Para evaluar la posicin del centro de
o
masas de la semiesfera, la colocamos
con la cara plana sobre el plano x y,
haciendo coincidir el eje con z , y luego

la rebanamos en tajadas de ancho dz


(ver gura).
Evaluemos primero la masa de la rebanada que se encuentra a la altura z. Su
masa es 0 (R2 z 2 ) dz, donde 0 es la
densidad de masa de la semiesfera. El
centro de masa de esta rebanada por
supuesto que queda sobre el eje z a la

altura z.

Figura 7.40

El centro de masas de la semiesfera ser la suma de los centros de masas de cada


a
rebanada pesada con la masa de de cada rebanada, es decir,
zcm =
=

1
M
0
M

R
0

z0 (R2 z 2 ) dz =

1 2 2 1 4
R z z
2
4

Pero M = 20 R3 /3, luego zcm = 3R/8.

=
0

0
M

R
0

(zR2 z 3 ) dz

0 1 4
R .
M 4

CAP
ITULO 7. TORQUE, CENTRO DE MASAS Y EQUILIBRIO

300

Solucin al problema 24
o
a) La masa de la semiesfera es M = 20 R3 /3. El torque en torno al punto de
contacto P viene dado por
= M gb sin x ,

donde x es un vector unitario que, para la situacin mostrada en la gura

o
adjunta, apunta hacia el lector.

Figura 7.41a

Figura 7.41.b

b) Al colocar sobre la semiesfera un cilindro de altura h la posicin del centro de


o
masas es
1
h
5
zcm =
M R + Mc R +
,
M + Mc
8
2
donde Mc = R2 h 0 es la masa del cilindro. Reemplazando las masas de los
cuerpos se obtiene
zcm =

1
+h

2
3R

h2
5 2
R + hR +
12
2

c) Mientras el la posicin del centro de masas del sistema compuesto se encuentre


o
por debajo del centro del semicirculo (punto A), el equilibrio ser estable. (Es
a
fcil convencesrse de que el torque que aparece al ladear el sistema trata de
a
restituir al cuerpo a su posicin de equilibrio). Por lo contrario, si el centro de
o
masas del sistema compuesto se encuentra por encima del punto A, el equilibrio
ser inestable.
a
d) La altura l
mite h0 se obtiene cuando zcm = R. Se tiene
zcm = R =

1
2
3 R + h0

h2
5 2
R + h0 R + 0
12
2

CAP
ITULO 7. TORQUE, CENTRO DE MASAS Y EQUILIBRIO

301

Despejando la altura l
mite se encuentra que sta viene dada por
e
R
h0 = .
2

Solucin al problema 25
o
a) El diagrama de cuerpo libre se muestra en la gura adjunta.
b) La masa de la semiesfera es M = 20 R3 /3. El torque en torno al punto de
contacto P viene dado por
= M gb sin x ,

donde x es un vector unitario que, para la situacin mostrada en la gura

o
adjunta, apunta hacia el lector.

Figura 7.42a

Figura 7.42.b

c) La fuerza Fp que ejerce la pared sobre la semiesfera es en la direccin horizontal


o
+ y su magnitud es tal que el torque total respecto a P es nulo. Luego
y
Fp =

M gh sin
3
y = M g sin y .

R
8

Como la fuerza horizontal total debe ser nula, y la unica otra fuerza horizontal

es la fuerza de roce, se tiene


Fr = Fp =

3
M gh sin
y = M g sin y .

R
8

CAP
ITULO 7. TORQUE, CENTRO DE MASAS Y EQUILIBRIO

302

d) La fuerza de roce no debe sobrepasar el valor M g, o sea,


3
3
M g sin M g =
Mg
8
16
de donde

1
.
2
El ngulo cr
a
tico es, por lo tanto, max = 30 .
sin

e) Al colocar sobre la semiesfera un cilindro de altura h la posicin del centro de


o
masas es
1
5
h
scm =
M R + Mc R +
,
M + Mc
8
2
donde Mc = R2 h 0 es la masa del cilindro. Reemplazando las masas de los
cuerpos se obtiene
scm =

1
+h

2
3R

h2
5 2
R + hR +
12
2

Figura 7.43a

Figura 7.43.b

f) La altura l
mite hmax se obtiene cuando scm = R. Se tiene
scm = R =

1
2
3 R + hmax

h2
5 2
R + hmax R + max
12
2

Despejando la altura l
mite se encuentra que sta viene dada por
e
R
hmax = .
2

CAP
ITULO 7. TORQUE, CENTRO DE MASAS Y EQUILIBRIO

303

Solucin al problema 30
o
Las guras 7.44a y 7.44b muestran los diagramas de cuerpo libre de la masa Q
y el semicilindro, respectivamente. Fr = kx es la fuerza ejercida por el resorte, W el
peso del semicilindro y Q el peso del bloque que se encuentra a una distancia x de O.
La distancia d entre el centro de masas y O la supondremos conocida.

Figura 7.44a

Figura 7.44.b

Debido a que ambos objetos estn en equilibrio se debe tener que la fuerza total
a
sobre cada uno de ellos debe ser nula, y tambin el torque total sobre el semicilindro
e
(en torno a cualquier origen).
Para el bloque Q se obtiene la relacin
o
Ftot = (Fr cos + Fr sin ) Q + (N1 cos z N1 sin x) = 0 .
x
z
z

Igualando las componentes de los vectores de la ultima igualdad y usando el hecho

que Fr = kx, se obtienen las ecuaciones,


kr cos = N1 sin

(7.11)

kx sin = Q N1 cos .

(7.12)

y
La fuerza total que acta sobre el semicilindro (que tambin debe ser nula) es
u
e
W z + N z + (Fr cos Fr sin ) + (N1 cos z + N1 sin x) = 0 .

x
z

Esto nos da las relaciones


N1 cos kx sin W + N = 0
y
N1 sin kx cos = 0 .

(7.13)

CAP
ITULO 7. TORQUE, CENTRO DE MASAS Y EQUILIBRIO

304

Esta ultima ecuacin no da informacin nueva ya que coincide con (7.11). Por ultimo,

o
o

evaluando el torque total (en torno al punto O) que acta sobre el semicilindro, se
u
obtiene:
N1 x W d sin = 0 .
(7.14)
De las cuatro equaciones (7.11), (7.12), 11.28) y (7.14), con las cuatro incgnitas N ,
o
N1 , x y , podemos despejar cos . Realizando el lgebra, se obtiene,
a
cos =

W dk
.
Q2

Que pasa cuando Q es pequeo? Es claro que la solucin obtenida slo tiene sentido
n
o
o
2 > W kd. Al analizar el problema (hgalo!) con ms cuidado se encuentra que
si Q
a
a
= 0 (y, por lo tanto, x = 0) tambin (para todos los valores de Q) es una solucin
e
o
de este problema de equilibrio. Tambin se encuentra que para 0 Q2 < W kd, la
e
solucin = 0 es la unica, siendo estable. Para W dk < Q2 , hay tres soluciones: = 0
o

y = Acos (W dk/Q2 ); siendo la primera de stas inestable, y estables las otras dos.
e
Para comprender mejor lo que est ocurriendo es util analizar el problema tama

bin desde el punto de vista de la energ potencial. Deniendo el origen de la energia


e
a
potencial gravitacioneal cuando = 0, se encuentra que
1
U () = W d (1 cos ) Qx sin + kx2 .
2
El primer trmino al lado derecho es el cambio de la energ potencial gravitacional
e
a
del semicilindro, el segundo el cambio de la energ potencial gravitacional de la masa
a
Q y el tercero la energ potencial del resorte. Con kx = Q sin queda
a
U () = W d(1 cos )

Q2
sin2 .
2k

Para hacer un estudio grco de esta relacin introducimos el parmetro Q2 /(kW d)


a
o
a

o
y denimos U () U ()/(W d); de esta manera la ultima ecuacin queda de la forma
U () = (1 cos )

sin2 .
2

La gura 7.45 muestra el grco para = 0; 0, 5; 1,0 y 2. Para 0 < < 1, el


a
grco tiene un slo minimo, para > 1 el grco tiene dos m
a
o
a
nimos (en 0 ) y un
mximo (en = 0). Para encontrar 0 debemos evaluar la derivada de U () respecto
a
a e igualarla a cero:

CAP
ITULO 7. TORQUE, CENTRO DE MASAS Y EQUILIBRIO

dU ()

= sin 2 sin cos = 0 ,


d
2
o sea,
sin (1 cos ) = 0 .
Esta ecuacin se satisface si sin = 0 o
o
(1 cos ) = 0. La primera de estas
condiciones nos da la solucin = 0
o
mientrs que la segunda entrega las soluciones 0 = Acos (1/), soluciones
que existe slo si 1. Para = 2, se
o
obtiene 0 = 60 .
Figura 7.45

305

Cap
tulo 8

Momento angular
versin 19 agosto 2010
o

En el cap
tulo anterior comenzamos a estudiar sistemas que ya no son part
culas puntuales, sino cuerpos con estructura. Aprendimos que la fuerza neta sobre un
objeto, por s sola, no es suciente para describir el estado de tales sistemas, y que

es necesario introducir una nueva cantidad f


sica, el torque. De este modo, la fuerza
describe el estado traslacional de un objeto, mientras que el torque describe su estado rotacional. De momento, sin embargo, slo nos hemos preocupado de sistemas en
o
equilibrio. Ahora debemos estudiar su dinmica. Para ello, ser necesario introducir
a
a
otra magnitud f
sica, el momento angular , que podemos considerar como el anlogo
a
al momento lineal estudiado en el Cap. 6, pero cuando lo que nos interesa es describir rotaciones y no traslaciones. En este cap
tulo slo pretenderemos enunciar este
o
concepto, mostrar algunos resultados importantes, y ganar intuicin al respecto, de
o
manera que ms adelante en el curso podamos estudiar adecuadamente la evolucin
a
o
de sistemas con estructura.

8.1.

Momento angular de una part


cula

Consideremos una part


cula de masa m y cuya posicin (respecto a algn sistema
o
u
de referencia inercial) viene dada por el vector r. Sea F la fuerza neta que acta sobre
u
la part
cula. Entonces, de acuerdo a la 2a ley de Newton, la ecuacin de movimiento
o
es
dp
.
F =
dt
Tomando el producto cruz con el vector r se obtiene
rF =r

306

dp
.
dt

(8.1)

CAP
ITULO 8. MOMENTO ANGULAR

307

Observemos que
d
dr
dp
dp
(r p) =
p+r
=r
.
(8.2)
dt
dt
dt
dt
La ultima igualdad se deduce del hecho que los vectores dr/dt = v y p son paralelos.

Usando (8.2) en (8.1) se obtiene


= rF =

d
(r p) .
dt

Observemos que la expresin anterior es anloga a la segunda ley de Newton, donde


o
a
en el lado izquierdo la fuerza es reemplazada por el torque (como debe ser, ya que
ahora no slo interesa la posicin de una part
o
o
cula puntual o del centro de masas), y
en el lado derecho el momento lineal p ha sido reemplazado por r p. Inspirados por
dicha analog entonces, denimos el momento angular de una part
a,
cula por
rp.
Entonces
d
.
dt
Igual que en el caso del torque, el momento angular de una part
cula depende del
origen que se use para evaluarlo. Notemos tambin que si el torque que acta sobre
e
u
una part
cula, medido respecto a cierto origen es nulo, entonces el momento angular de
la part
cula, respecto al mismo origen, no variar en el tiempo, es decir, se conservar.
a
a
=

Revisemos algunos ejemplos sencillos. Primero, consideremos una part


cula A (ver
gura) con velocidad constante v = v0 x, movindose a lo largo del eje x:

v0
^
y

rB
O

rA

^
x

De la denicin de momento angular es inmediato que A = rA pA = 0. Si ahora


o
consideramos la part
cula B, de igual masa, movindose con igual velocidad, pero a
e
una distancia b del eje x, es evidente que B = 0. La misma part

cula, entonces,
puede tener momento angular distinto simplemente porque se mueve a lo largo de
otra recta. Esto no es sino consecuencia de lo dicho anteriormente, que el momento
angular depende del origen respecto al cual se evala.
u

CAP
ITULO 8. MOMENTO ANGULAR

308

En el caso recin analizado, la fuerza neta sobre las part


e
culas es cero, luego el
torque es cero, y por lo tanto el momento angular debe ser constante. Para la part
cula
A ello es evidente. En el caso de la part
cular B, si B es constante signica que para
evaluarlo podemos usar cualquier punto de la trayectoria, por ejemplo cuando pasa
por el eje y, en cuyo caso r y p son perpendiculares. Es inmediato entonces que

B = mv0 b.
z
Lo anterior, obviamente, tambin se puede observar en una trayectoria rectil
e
nea
arbitraria, como la indicada en la siguiente gura:
^
y

v0
r

^
x

En este caso, si n es la ordenada en que la recta intersecta al eje y , entonces el

momento angular respecto a O es = mv0 n cos , que es por supuesto constante.


z
Consideremos ahora una part
cula que se mantiene en un movimiento circular
uniforme (con velocidad angular 0 ) mediante un hilo.
Sea R el radio de c
rculo. El momento angular de la part
cula (respecto al centro de
la circunferencia) viene dado por
= mR2 0 z .

La direccin en que apunta es a lo laro


go del eje de giro, y en el sentido dado
por la regla de la mano derecha (los dedos empuados indicando el sentido de la
n
rotacin; el pulgar extendido da el sentio
do del momento angular la misma regla
que encontramos para el sentido del torque).

Figura 8.1

2
El hilo ejerce una fuerza sobre la part
cula (la fuerza centr
peta dada por mR0 r),
pero esta fuerza no ejerce un torque respecto al origen ya que F y r son paralelos.
Debido a que el torque es nulo, el momento angular de la part
cula se conserva (o
sea, a medida que transcurre el tiempo no cambia la magnitud ni la orientacin del
o
vector ).

CAP
ITULO 8. MOMENTO ANGULAR

309

Evaluemos ahora el momento angular para una part


cula movindose de manera
e
arbitraria en un plano.
Supongamos que una part
cula de masa m
se mueve en el plano x ,y y sean r(t), (t)
las coordenadas polares del vector de posicin r(t). La posicin de la part
o
o
cula
vendr dada por
a
r = r ,
r
donde
r = cos x + sin y .

Derivando obtenemos la velocidad


Figura 8.2

v = rr +rr .

Pero
luego

r = (cos x + sin y ) = sin() x + cos() y ,

dt

v = r r + r .

De esta manera, para el momento angular de la part


cula se encuentra la expresin
o

= r p = m r r v = mr r r r + mr 2 r = mr 2 z ,

(8.3)

donde z es el vector unitario perpendicular al plano (x, y) (cuya direccin en que

o
apunta se encuentra usando la regla de la mano derecha).

Observe que si la part


cula se aleja en direccin radial (o sea, = 0 y r = 0)
o

entonces el momento angular es nulo. Slo si el ngulo del vector de posicin cambia
o
a
o
a medida que transcurre el tiempo, el momento angular es no nulo. El momento
angular de una part
cula est relacionado con el aspecto rotacional de su movimiento!
a
Esta conclusin de algn modo era evidente cuando estudiamos el caso de una
o
u
part
cula en movimiento circular, pero es menos evidente cuando consideramos un
movimiento rectil
neo, ya que el momento angular pod o no ser cero. La razn es
a
o
la siguiente: cuando la part
cula se mueve sobre el eje x, vista desde el origen la

part
cula se ve alejarse radialmente, y por lo tanto = 0, y = 0, de acuerdo a
(8.3). Pero cuando la part
cula se mueve a una distancia b del eje x, el ngulo que

a
forma su vector posicin con el eje x s cambia (se acerca asintticamente a cero a
o

o
medida que la part
cula se aleja). Por tanto, respecto al origen, s podr

amos decir
que la part
cula gira, en algnsentido. Lo importante entonces es que el momento
u

CAP
ITULO 8. MOMENTO ANGULAR

310

angular ser distinto de cero cuando la variable angular que describe la posicin de la
a
o
part
cula var Eso puede o no representar una rotacin o un movimiento curvil
a.
o
neo,
en el sentido convencional.
Problema resuelto en clases: 8.7

8.2.

Momento angular de varias part


culas

Consideremos ahora N masas {mj } ubicados en los lugares {rj }. Sean {Fj } la
fuerza externa que acta sobre cada part
u
cula y {fji } la fuerza que la masa i ejerce
sobre la masa j. Por supuesto que debido al tercer principio de Newton, fji = fij .
Supongamos adems que la fuerza que una part
a
cula i ejerce sobre otra part
cula j
es a lo largo de la l
nea que las une (o sea, que la interaccin entre las part
o
culas es
central).
La ecuacin de movimiento (2a ley de Newton) para cada part
o
cula es
dpj
.
dt

fji =

Fj +
i

Tomando el producto cruz con el vector rj se obtiene


rj

fji

Fj +
i

= rj

dpj
.
dt

Por la misma razn discutida en la seccin anterior


o
o
rj

d
dpj
=
(rj pj ) .
dt
dt

Usando esta relacin y sumando sobre j, se obtiene


o

rj Fj +

ji

rj fji =

d
d
(rj pj ) =
dt
dt

rj pj .

Pero
ri fij + rj fji = (ri rj ) fij = 0 ,
ya que (ri rj ) es paralelo a fij . Luego, la doble suma ji rj Fji es nula. De esta
manera, usando las deniciones de momento angular y torque, se obtiene
j =
j

d
dt

j .
j

Sea

j
j

(8.4)

CAP
ITULO 8. MOMENTO ANGULAR

311

y
L

j
j

el torque y el momento angular total del sistema de part


culas, entonces la ecuacin
o
(8.4) queda
dL

=L.
=
(8.5)
dt
En palabras: Si el torque total que acta sobre un sistema (respecto a un punto
u
P ) es nulo, entonces el momento angular del sistema (respecto al mismo punto) no
cambiar. Lo anterior se conoce como la ley de conservacin del momento angular. Las
a
o
fuerzas internas de un sistema pueden cambiar el momento angular de las part
culas
que lo componen, pero no pueden modicar el vector momento angular total.
Ilustremos el uso de la ley de conservacin de momento angular con algunos
o
ejemplos.
Ejemplo 1
Demuestre que un planeta, que se mueve alrededor del sol, barre reas iguales en
a
tiempos iguales, es decir, dA/dt =constante.
Coloquemos el origen de nuestro sistema de coordenadas en el lugar donde est el
a
sol. La fuerza que el sol ejerce sobre los planetas es a lo largo de la direccin radial,
o
por lo tanto, la fuerza atractiva de gravitacin no ejerce torque sobre el planeta. De
o
lo anterior se desprende que el momento angular del planeta debe ser en todos los
instantes el mismo. De hecho, el momento angular se conservar en general, para
a
cualquier fuerza central (que dependa slo de r).
o
Cul es el rea A que barre el planeta
a
a
en un tiempo t ? La respuesta es
A =

t
t
1
| r (v t) | =
|r p| =
.
2
2m
2m

Como = | | se conserva a lo largo de


la trayectoria, se deduce que el rea barria
da en un tiempo t es independiente del
punto de la trayectoria que se considere.
Figura 8.3
Observemos adems que la conservacin del momento angular es vectorial, es
a
o
decir, que debe conservarse tambin su direccin. En particular, para un planeta
e
o

CAP
ITULO 8. MOMENTO ANGULAR

312

orbitando en torno a una estrella el momento angular es perpendicular al plano orbital,


y por ende la conservacin del momento angular implica que el plano orbital debe ser
o
constante y el movimiento del planeta es siempre en dos dimensiones.
Ejemplo 2
Considere una masa M colgada de una varilla r
gida, de masa despreciable y de largo L, que puede girar libremente en torno
al punto O (ver gura adjunta). En el instante t = 0 la masa M explota y una parte
M/2 sale disparada con velocidad v en una
direccin que forma un ngulo con reso
a
pecto a la horizontal. Encuentre la energ
a
cintica de la parte que qued adosada a la
e
o
varilla en el instante inmediatamente posterior a la explosin.
o
Figura 8.4
Observemos que este problema corresponde a una colisin inelstica. Es evio
a
dentemente inelstica ya que la energ cintica cambia despus de la explosin. Y
a
a
e
e
o
como las ecuaciones de Newton no cambian si invertimos el sentido del tiempo, no
pueden distinguir entre el proceso de explosin descrito, o el proceso inverso, de dos
o
masas acercndose hasta quedar adosadas y en reposo en el extremo del pndulo, de
a
e
modo que podemos hablar de una colisin en sentido genrico tambin.
o
e
e
Sobre el sistema (la varilla con la masa colgando) actan las siguientes fuerzas: i)
u
el peso M g, ii) una fuerza F0 que ejerce el eje de giro sobre la varilla y iii) fuerzas
z
originadas por la explosin. En el instante t = 0 el peso no ejerce un torque sobre
o
el sistema respecto a O ya que en ese instante los vectores r y M g son paralelos.
z
La fuerza F0 tampoco ejerce un torque ya que el brazo para esta fuerza es nulo.
Las fuerzas originadas por la explosin son fuerzas internas y por consiguiente no
o
modican el momento angular total del sistema. Concluimos que el momento angular
total antes y justo despus de la explosin deben ser iguales.
e
o
Inicialmente el momento angular es cero. Despus de la explosin el momento
e
o
angular del fragmento que sale disparado es
1 =

M
Lv cos y .

Si la velocidad angular de la varilla en el instante posterior a la explosin es 0 , el


o
momento angular de la masa que qued adosada a la varilla es
o
2 =

M 2
L 0 y .

CAP
ITULO 8. MOMENTO ANGULAR

313

Como la suma de los dos momentos angulares 1 y 2 debe ser nula, se tiene que
M 2
M
Lv cos =
L 0 .
2
2
Despejando 0 se encuentra

v cos
.
L
Finalmente, conociendo la velocidad angular 0 podemos evaluar la energ cintica
a
e
del fragmento que qued adosado a la varilla, en el instante inmediatamente posterior
o
a la explosin:
o
1M 2 2 M 2
L 0 =
v cos2 .
K=
2 2
4
0 =

Antes de analizar un tercer ejemplo debemos demostrar una proposicin imo


portante.
Consideremos nuevamente N part
culas con masas {mj } ubicadas en los
lugares {rj } y con velocidades {vj }.
Sean Rcm y Vcm la posicin y velocidad
o
del ventro de masas. Denotemos por
rj y vj los vectores de posicin y
o
velocidad de la part
cula mj respecto
al centro de masas.
Entonces

Figura 8.5

j =

L=

rj pj =

mj ( rj vj ) .

Por otra parte


rj = Rcm + rj
y
vj = Vcm + vj .
Sustituyendo estas relaciones en la ecuacin anterior se obtiene
o
L =
j

=
j

mj Rcm + rj Vcm + vj
mj Rcm Vcm + M

mj rj
M

Vcm + M Rcm

mj vj
M

+
j

mj rj vj

CAP
ITULO 8. MOMENTO ANGULAR
Pero
j

mj rj
M

y
j

314

= Rcm

mj vj

= Vcm
M
son la posicin y velocidad del centro de masas medidas desde el centro de masas
o
luego ambas sumatorias son nulas. De esta manera la ecuacin anterior queda
o
Proposicin:
o
L = M Rcm Vcm +

= Rcm Pcm + L .

En palabras: El momento angular respecto a cualquier punto O es igual al momento


angular debido a la traslacin del sistema como un todo, es decir, el movimiento del
o
centro de masas con toda la masa concentrada en ese lugar, ms el momento angular
a
(rotacional intr
nseco) del sistema visto desde el centro de masas.
Es importante recalcar que el centro de masas ha aparecido nuevamente como
un punto privilegiado, que nos permite hacer esta descomposicin. Si tomramos
o
a
cualquier otro punto, digamos A, el esta descomposicin del momento angular total
o
en el momento angular de A, ms un momento angular en torno al punto A, no es
a
posible.
Ejemplo 3
Considere dos part
culas de masa m unidas por una barra de masa despreciable
y largo L. Una tercera part
cula, tambin
e
de masa m, colisiona con las anteriores,
quedando adosada a la # 2. Si la velocidad incidente de la masa # 3 es v0 , y sta
e
incide como se muestra en la gura 8.6,
encuentre la posicin de la masa # 1 en
o
funcin del tiempo.
o

Figura 8.6

Observemos, en primer lugar, que en este problema tenemos tres ecuaciones (dos
de momento lineal, una de momento angular), y, en principio, 4 incgnitas (dos cooro
denadas de velocidad para la masa 1, y dos para la masa 2-3). Pero ambas masas
estn unidas por una barra, de modo que sus velocidades no son independientes,
a
y esto resulta en slo 3 incgnitas (la velocidad traslacional del centro de masas y
o
o

CAP
ITULO 8. MOMENTO ANGULAR

315

la velocidad angular de rotacin en torno al mismo), consistente con el nmero de


o
u
ecuaciones. As el problema se deber poder resolver exactamente.
,
a
Lo haremos de dos maneras.
Primero elegiremos el sistema de coordenadas de manera que el eje x coincida con la

recta a lo largo de la cual se mueve el centro de masas del sistema (ver gura 8.7).
Si t = 0 corresponde al instante en que ocurre la colisin, entonces la posicin
o
o
del centro de masas del sistema total (es decir, de las tres masas), tanto antes como
despus de la colisin, vendr dado por
e
o
a
rcm (t) =

v0
tx .

Posterior a la colisin, la barra con masas


o
2m y m en sus extremos, rotar con cierta
a
velocidad angular 0 en torno al centro
de masas. Podemos evaluar 0 usando la
ley de conservacin del momento angular.
o
Antes de la colisin el momento angular
o
del sistema es
Li =

L
mv0
3

z .

Despus de la colisin, para el sistema de


e
o
referencia que estamos usando, la varilla
con las masas slo tiene un momento ano
gular intr
nseco:
Lf =

2L
2L
m 0
3
3

Figura 8.7

L
(2m)
3

L
3

z=

2
m0 L2 z .

Usando la ley de conservacin del momento angular se encuentra que


o
0 =

v0
.
2L

Volveremos a resolver el problema pero eligiendo ahora un sistema de coordenadas


jo en el laboratorio y con el origen coincidiendo con la posicin de la part
o
cula # 2
antes de la colisin. Nuevamente elegimos el eje x a lo largo de la velocidad de la
o

part
cula incidente y el eje y a lo largo de la direccin que tiene la barra antes de la

o
colisin.
o
En este sistema de coordenadas, el momento angular del sistema, antes de la
colisin, es nulo. Despus de la colisin, el momento angular total de la barra con las
o
e
o

CAP
ITULO 8. MOMENTO ANGULAR

316

tres masas, tambin deber ser nulo. El momento angular de este sistema complejo
e
a
que se aleja, se puede evaluar usando la proposicin recin demostrada. Consta de
o
e
dos partes: el momento angular del centro de masas y el momento angular rotacional
intr
nseco.
Como el centro de masas se mueve con velocidad v0 /3, la masa total es 3m y el
brazo (distancia entre el origen y la tangente de la trayectoria del centro de masas)
es L/3, el momento angular del centro de masas ser
a
Rcm Pcm =

v0
L
(3m)
z.

3
3

El momento angular intr


nseco, igual que en el caso anterior, viene dado por
2
L = m0 L2 z .

3
La condicin que la suma de los dos momentos angulares anteriores sea nula, nos da
o
la misma relacin que ya hab
o
amos encontrado:
0 =

v0
.
2L

Para la posicin de la masa #1 se obtiene la expresin


o
o

2L

3
r1 (t) =
v 2L [cos( t) y sin( t) x]
0 tx +
0
0
3
3

para t < 0
para t > 0

Problema resuelto en clases: 8.11 (a)

Problema resuelto en clases: Pndulo simple, resuelto con Ec. (8.5)


e
Problema resuelto en clases: 7.13 (si se dio de tarea anteriormente)

8.3.

Problemas

1. Consideremos un satlite articial, de masa m, que gira en torno a la tierra a


e
lo largo de una rbita el
o
ptica y las distancias mxima y m
a
nima a la supercie
de la tierra son 2555 km y 352 km, respectivamente. La velocidad mxima del
a
satlite es de 29737 km/h. El radio terrestre es igual a 6382 km. Cules sern las
e
a
a
velocidades del satlite en el perigeo (rmin ) y apogeo (rmax ), respectivamente?
e

CAP
ITULO 8. MOMENTO ANGULAR

317

2. Una bala de masa m y velocidad v pasa


a travs de la lenteja de un pndulo de
e
e
masa M , y emerge con velocidad v/2. La
lenteja del pndulo est colgada de una
e
a
cuerda de longitud . Cul debe ser el
a
valor de v para que la lenteja del pndulo
e
describa un c
rculo completo? Cmo se
o
modica el problema si, en lugar de una
cuerda, la lenteja est colgada de una vaa
rilla r
gida sin masa?
Figura 8.8
3. Una part
cula de masa m y velocidad v0
incide sobre una barra de largo L y masa despreciable, que en cada uno de los
extremos tiene una masa m, tal como se
indica en la gura. Suponga que el choque
entre las esferas # 1 y # 2 es elstico y
a
central (frontal). Se mover la part
a
cula
# 1 despus del choque? Si su respuesta es
e
armativa evale su direccin y magnitud.
u
o
4. Una masa m1 se deja caer desde una altura
h sobre un balanc (ver gura 8.10). El
n
balanc puede girar libremente en torno
n
a O en el sentido contrario al reloj. Sobre
el otro extremo del balanc hay una man
sa m2 . Al chocar la masa m1 contra el balanc sta queda adosada a l. Qu fracn, e
e
e
cin de la energ total inicial se disipa
o
a
en la colisin? Desprecie la masa del bao
lanc
n.

Figura 8.9

Figura 8.10

CAP
ITULO 8. MOMENTO ANGULAR
5. Considere dos masas m, unidas por una
varilla de largo L. Esta varilla est sola
dada en su centro a otra varilla, formando un ngulo . El sistema anterior roa
ta con una velocidad angular 0 en torno
a la segunda varilla (ver gura adjunta).
En cierto instante la soldadura se rompe,
desacoplndose el movimiento de las dos
a
varillas. Describa, de ah en adelante, el

movimiento de la varilla con las dos masas.


6. Considere una varilla r
gida, pero de masa despreciable, cuyo largo es L y que
tiene dos masas m, una adosada en uno
de los extremos y la otra al centro (ver
gura). La varilla puede girar libremente en el plano vertical alrededor de un
eje que pasa por el extremo en que no
tiene una masa adosada. Todo el sistema se encuentra en un campo gravitacional constante g = g. Suponga que
z
este sistema inicialmente se encuentra en
reposo en su posicin de equilibrio ineso
table. Una leve perturbacin hace que el
o
sistema salga de su posicin de equilibrio
o
y paulatinamente comienza a caer.

318

Figura 8.11

Figura 8.13

a) Encuentre la velocidad angular = y la aceleracin angular = de la


o
varilla cuando sta forme un ngulo con la vertical.
e
a
b) Encuentre la fuerza que la varilla ejerce sobre el eje cuando la varilla pasa
por la horizontal (es decir, cuando = /2).
7. Una masa m realiza un movimiento circular, con radio R0 , sobre una mesa (sin
friccin), atada a un hilo (ver gura 8.12).
o
Sea 0 la velocidad angular inicial. Cul
a
es el trabajo que debe realizarse (tirando
del hilo) para achicar el radio de giro desde
R0 a R0 /2 ?
2 2
Respuesta: W = 3m0 R0 /2 .

Figura 8.12

CAP
ITULO 8. MOMENTO ANGULAR

319

8. Considere un pndulo cnico (es decir,


e
o
una masa m colgada de un hilo ideal de
largo L), que gira en c
rculos formando
un ngulo 0 con la vertical.
a
a) Con qu velocidad angular gie
rar si el hilo se acorta lentamente
a
hasta llegar a L/2?
b) Que trabajo debe realizarse para
acortar el hilo en esa magnitud?

Figura 8.15

9. Un alambre (de masa despreciable) de


largo 2L se dobla al centro de manera
que forma un ngulo . En cada extremo
a
el alambre tiene una masa m. Este dispositivo se cuelga de un eje tal como
se muestra en la gura adjunta. Calcule
el per
odo de oscilacin del sistema para
o
pequeas oscilaciones en torno a su posin
cin de equilibrio estable. Verique que
o
la expresin general, en los l
o
mites = 0
y = , da los resultados esperados.

Figura 8.16
Para resolver este problema suponga que el sistema est oscilando (con pequeas
a
n
oscilaciones) y evale para un instante arbitrario el torque y el momento anguu
lar. Luego, usando la ecuacin (8.4) demuestre que la variable (t) satisface la
o
ecuacin diferencial de un oscilador armnico.
o
o

10. Considere una varilla de largo L que tiene dos masas M adosadas tal como se
muestra en la gura. Un masa m que
incide con velocidad v0 , choca con el
pndulo quedando adosada a l a una
e
e
distancia h del eje. Determine el impulso
trasmitido por el eje al pndulo durante
e
la colisin. A qu altura debe impactar
o
e
m para que el impulso transmitido por
el eje sea nulo? (en ese caso el eje no se
percata de la colisin).
o

Figura 8.17

CAP
ITULO 8. MOMENTO ANGULAR

320

11. Dos masas m unidas por un hilo de largo L, caen con el hilo horizontal partiendo desde el reposo. Despus de caer una
e
distancia h, una de ellas choca elsticaa
mente con una viga.
a) Determine la velocidad angular
con que girarn las masas en torno
a
a su centro de masas despus de la
e
colisin.
o
b) Encuentre la tensin a la que eso
tar sometido del hilo despus de
a
e
que ha ocurrido la colisin.
o

Figura 8.14

12. Una varilla de largo 2L y masa despreciable se dobla en su punto medio O, de


modo que ambas secciones de la varilla forman un ngulo entre s (ver gura).
a

En cada extremo de la varilla hay una masa m. El sistema se coloca sobre una
mesa horizontal, y se ja a la mesa en el punto O, permitindole rotar en torno
e
a l.
e
Dos masas adicionales, m1 y m2 , chocan simultneamente sobre cada una de las
a
masas anteriores, quedando adosadas a ellas, con velocidades v1 y v2 . La masa
m2 incide perpendicularmente a uno de los segmentos de la varilla, y la masa
m1 incide paralelamente a ese mismo segmento, como indica la gura.
Si m2 = m, y | v1 | = | v2 | = v, Con qu frecuencia angular queda girando el
e
sistema luego de la colisin?
o
Cunto debe valer m1 para que el sistema no gire despus de la colisin?
a
e
o

v1

m1
L
m

L
v2
m2

CAP
ITULO 8. MOMENTO ANGULAR

8.4.

321

Solucin a algunos de los problemas


o

Solucin al problema 6
o
Debido al principio de conservacin de la energ la energ cintica que tiene el
o
a,
a
e
sistema cuando la varilla forma un ngulo con la normal debe ser igual al cambio
a
de energ potencial, o sea, K = U , con
a
1
K = m
2

1
5
+ m(L)2 = mL2 2
2
8

y
L L
cos
2
2

U = mg(L L cos ) + mg

3
= mgL (1 cos ) .
2

De esta manera se deduce que


2 () =

12 g
(1 cos ) .
5 L

Derivando esta relacin encontramos la aceleracin angular, en efecto,


o
o
2 =

12 g

sin .
5 L

Pero = , luego

6g
sin .
5L
Demostremos que el mismo resultado se puede obtener usando la ecuacin de moo
vimiento = d/dt. Cuando la varilla forma un ngulo con la normal, el torque
a
respecto a un origen ubicado en el eje es
==

= mgL sin + mg

3
L
sin = mgL sin .
2
2

Para el momento angular tenemos


= m(L)L + m

L
5
= mL2 .
2
4

Reemplazando estas expresiones en la ecuacin de movimiento se obtiene


o
3
5
mgL sin = mL2 ,

2
4
de donde, nuevamente
==

6g
sin .
5L

CAP
ITULO 8. MOMENTO ANGULAR

322

Supongamos ahora que la varilla est pasando por la horizontal (es decir, =
a
/2). En ese instante el centro de masas (que est ubicado a una distancia 3L/4 del
a
eje) acelera con una aceleracin
o
3
z
acm = |at | |ac | = L 2
z
x
4

3
L
4

x=

9
g( + 2) .
z
x
10

(Observe que la componente z de la aceleracin de la part

o
cula m que est en el
a
extremo de la varilla, cuando sta pasa por la horizontal, es 6g/5, o sea, mayor que
e
g; convnzase de que as deb ser). La fuerza neta que acta sobre la varilla (cuando
e

a
u
pasa por la horizontal) es
Ftot = Feje 2mg .
z
Pero Ftot = (2m)acm , luego
Feje 2mg = 2m
z
de donde se deduce que

9
g( + 2) ,
z
x
10

1
Feje = mg ( 18) .
z
x
5

Solucin al problema 9
o
Denotemos por al ngulo que el pndulo hace respecto a su posicin de equilibrio
a
e
o
(ver gura 8.18).
El momento angular del pndulo ser
e
a
= 2Lm(L) x ,

donde es la velocidad angular del pndu
e
lo (siendo positiva cuando gira en la direccin contraria a los punteros del reloj).
o
Derivando respecto al tiempo se deduce
que
d
= 2mL2 x .

dt
Figura 8.18
El torque de la fuerza gravitacional (respecto a un origen en el eje) es

= mgL sin(/2 ) x mgL sin(/2 + ) x

= 2mgL cos(/2) sin x

CAP
ITULO 8. MOMENTO ANGULAR

323

Sustituyendo las dos relaciones anteriores en la ecuacin de movimiento = d/dt


o
encontramos
2mgL cos(/2) sin = 2mL2 .

Para pequeas oscilaciones en torno de la posicin de equilibrio podemos usar la


n
o
aproximacin sin . De esta manera obtenemos
o
g
cos(/2) = 0 .
L

Esta es la ecuacin de movimiento de un oscilador armnico cuyo per


o
o
odo es
T = 2

g
cos
L
2

Solucin al problema 11
o

Al chocar con la viga la velocidad de la masa m ser v0 = 2gh. El choque


a
z
con la viga es elstico y el hilo que une ambas masas (que no puede ejercer fuerzas
a
transversales a su orientacin) no interviene para nada en ese proceso. Luego la masa
o

m rebotar con la velocidad 2mg z . La otra masa no modica su velocidad mientras


a

ocurre el choque. Por lo tanto, justo despus de la colisin, la velocidad del centro de
e
o
masas (respecto a un observador junto a la viga) ser nula.
a
Para un observador junto a la viga (en el lugar donde ocurrir la colisin), el
a
o
momento angular antes de la colisin es
o
i = Lm

2gh .

Despus de la colisin ser


e
o
a
f = Lcm + ,
f
donde Lcm es el momento angular debido a la traslacin del centro de masas y es
o
f
el momento angular observado desde el centro de masas. Ya que justo despus de la
e
colisin el centro de masas est en reposo, Lcm = 0. Denotemos por 0 la velocidad
o
a
angular del hilo despus de la colisin, entonces
e
o
= 2m
f

L
0
2

mL2
L
=
0 .
2
2

Como el impulso que ejerce la viga no cambia el momento angular del sistema respecto
al punto en que se aplica esa fuerza de percusin, se tiene que el momento angular
o
debe conservarse. Luego
mL2
0 = Lm 2gh ,
2
de donde

8gh
0 =
.
L

CAP
ITULO 8. MOMENTO ANGULAR

324

En este problema el mismo resultado tambin se puede obtener usando la consere


vacin de la energ Despus de la colisin, como el centro de masas est en reposo,
o
a.
e
o
a
toda la energ cintica se debe a la rotacin, siendo sta
a
e
o
e
1
Kr = 2 m
2

L
0
2

1
2
= mL2 0
4

Por otra parte, el cambio de energ potencial es


a
U = 2mgh .
Igualando ambas expresiones obtenemos nuevamente que

8gh
0 =
.
L
Despus de ciertos momentos de reexin, es claro que la ca de las dos masas
e
o
da
en un campo gravitatorio constante, no afecta la tensin del hilo. Por lo tanto, la
o
tensin de la cuerda se debe slo al movimiento rotacional de las dos masas. El radio
o
o
de giro de ellas es L/2. La magnitud de la fuerza centr
peta (que es igual a la tensin
o
del hilo) es
h
2L
Fcent = m0 = 4mg .
2
L

Cap
tulo 9

Rotacin de un cuerpo r
o
gido
versin 20 agosto 2012
o

En los cap
tulos 7 y 8 hemos introducido conceptos necesarios para estudiar
el comportamiento de cuerpos no puntuales. En efecto, a travs de los conceptos
e
de torque y momento angular, hemos podido formular el equivalente a la Segunda
Ley de Newton para rotaciones, punto de partida para comprender la dinmica de
a
cuerpos con estructura. Sin embargo, hasta el momento hemos sido o bien demasiado
particulares (estudiando sistemas compuestos por una part
cula, Sec. 8.1) o demasiado
generales (estudiando sistemas compuestos por muchas part
culas, sin restricciones
adicionales, Sec. 8.2).
En este cap
tulo estudiaremos un sistema espec
co: rotaciones de un cuerpo r
gido, es decir, un sistema compuesto de muchas part
culas, tales que las distancias entre
ellas no cambian. En particular, nos restringiremos a estudiar rotaciones de un cuerpo
r
gido en torno a un eje jo (que no cambia su orientacin a medida que transcurre
o
el tiempo), suciente para obtener una serie de resultados muy importantes. El caso
general, de rotaciones en torno a un eje no jo, tambin se puede resolver, pero ree
quiere un formalismo matemtico que en este momento no tenemos, y por lo tanto
a
ser tratado en cursos ms avanzados.
a
a

9.1.

Las ecuaciones bsicas


a

Estudiar en detalle la dinmica de un sistema de muchas part


a
culas es un problema demasiado complejo, ya que cada una de las part
culas tiene una ecuacin de
o
movimiento, y stas en general estn acopladas, de modo que el problema matemtico
e
a
a
consiste en resolver, digamos, N ecuaciones diferenciales de segundo orden acopladas,
lo cual puede, o bien no tener solucin anal
o
tica, o no ser soluble computacionalmente
en un tiempo razonable.
Por ello, uno podr pensar que estudiar la dinmica de un cuerpo complejo coa
a
mo, por ejemplo, un lpiz, puede resultar una tarea titnica e intil. Sin embargo, si
a
a
u
325


CAP
ITULO 9. ROTACION DE UN CUERPO R
IGIDO

326

bien es cierto un lpiz est compuesto por much


a
a
simas part
culas, tambin es cierto
e
que una buena aproximacin es considerar que todas esas part
o
culas mantienen distancias constantes entre s Es lo que llamamos un slido rgido. Interesantemente,
.
o

esta restriccin simplica enormemente la descripcin del sistema, como veremos.


o
o
En efecto, recordemos primero que, para un sistema arbitrario, la segunda ley
de Newton nos permite, conocidas las fuerzas externas, determinar completamente el
movimiento del centro de masas. Por lo tanto, sin prdida de generalidad podemos
e
considerar que el centro de masas est en reposo. Esto, para un sistema genrico
a
e
como una nube de gas o un uido puede no ayudar mucho, porque an las part
u
culas
tendr una gran libertad de movimiento. Pero para un slido r
an
o
gido, la condicin de
o
que el centro de masas est en reposo signica que el unico movimiento posible que
e

queda son rotaciones en torno al centro de masas, como muestra la siguiente gura:

A
P

B
En la gura anterior, P representa el centro de masas, y se ha dibujado tambin
e
el eje de rotacin. En primer lugar, debe ser claro que existe un eje de rotacin, es
o
o
decir, que para una rotacin arbitraria siempre se puede encontrar un eje respecto
o
a la cual dicha rotacin se realiza. Lo segundo que debe ser claro es que el centro
o
de masas debe estar sobre dicho eje de rotacin. Esto sucede porque el centro de
o
masas est en reposo, como ya hemos dicho. As que la gura anterior representa
a

una situacin completamente general. El tercer punto a notar es que si escogemos


o
un punto arbitrario del cuerpo, digamos A, entonces necesariamente, por tratarse el
movimiento de una rotacin, el punto A describe una trayectoria circular. Ms an, si
o
a u
escogemos cualquier otro punto, por ejemplo B, ambas trayectorias deben ser c
rculos
en planos paralelos entre s Notemos cmo sta es una consecuencia de que todas las
.
o
e
part
culas mantengan distancias constantes entre s
.
Ahora bien, es claro tambin que si el cuerpo es r
e
gido, no importa cun lejos
a
estn del eje de rotacin las part
e
o
culas A, B, o cualquier otra, todas deben completar
sus respectivas trayectorias circulares en el mismo tiempo. Ninguna puede retrasarse
respecto a las dems. Todo el cuerpo completa un giro en un tiempo dado. Esto
a


CAP
ITULO 9. ROTACION DE UN CUERPO R
IGIDO

327

es muy importante, porque nos permite observar que, no importa qu punto del
e
cuerpo consideremos, hay una cantidad f
sica que es comn a todos: la magnitud
u
de la frecuencia angular. Cada punto est a una distancia dada del eje, y por lo
a
tanto sus radios de giro sern en general diferentes; consecuentemente, sus velocidades
a
tangenciales sern en general diferentes, ya que deben completar un giro en el mismo
a
tiempo, y por ende los puntos ms lejanos al eje de rotacin deben desplazarse ms
a
o
a
rpido. Pero la frecuencia angular es la misma: todos los puntos deben completar un
a
giro, es decir, deben barrer un ngulo igual a 2, o una fraccin arbitraria de 2, en
a
o
el mismo tiempo.
Pero la frecuencia angular en realidad es un vector. Para un movimiento circular,
recordemos, la direccin de la frecuencia angular se asigna de acuerdo a la regla de
o
la mano derecha, y es perpendicular al plano de giro de la part
cula. Y como hemos
observado, todos los puntos del cuerpo describen trayectorias circulares en planos
paralelos entre s por lo tanto concluimos que no slo la magnitud de la frecuencia
,
o
angular es la misma para todos los puntos, sino tambin su direccin, que de hecho
e
o
es paralela al eje de rotacin.
o

Este es entonces el primer hecho signicativo que podemos armar, con completa
generalidad, para el movimiento de un slido r
o
gido: siempre se puede considerar que
dicho movimiento consiste en una rotacin en torno a un eje que pasa por el centro
o
de masas; dicha rotacin puede ser caracterizada por una unica frecuencia angular ,
o

cuya direccin est dada por la regla de la mano derecha (para determinarla se puede
o
a
utilizar cualquier punto del cuerpo), y es paralela al eje de rotacin.
o
Todo lo que hemos dicho hasta el momento es completamente general, una vez
que decidimos que estamos trabajando con un slido r
o
gido, y que el centro de masas
est en reposo. Sin embargo, el anlisis de este movimiento an puede ser bastante
a
a
u
complicado, por dos razones: en primer lugar, slo hemos observado que la trayectoria
o
de cada punto debe ser circular, pero eso no signica que tenga que ser uniforme. En
otras palabras, el mdulo de la frecuencia angular no tiene por qu ser constante.
o
e
Nosotros ya sabemos que si hay una aceleracin angular, signica que hay un torque,
o
de modo que, en principio, tenemos las herramientas anal
ticas para estudiar el caso de
aceleracin angular no nula, y lo haremos en este cap
o
tulo, si bien es cierto en muchos
casos consideraremos slo el caso en que la aceleracin angular es cero. En segundo
o
o
lugar, la frecuencia angular puede no ser constante no slo en su magnitud, sino
o
tambin en su direccin. Esto signica que el eje de rotacin puede no ser constante.
e
o
o
Cuando esto sucede, es necesario introducir conceptos adicionales como precesin y
o
nutacin, y el estudio anal
o
tico de las rotaciones de un cuerpo r
gido se complica
enormemente. Es posible hacerlo, pero ello ser materia de cursos ms avanzados.
a
a
En este cap
tulo, entonces, estudiaremos las rotaciones de un cuerpo r
gido en
torno a un eje constante.
Consideremos, pues, un cuerpo slido, que gira con velocidad angular 0 en torno
o


CAP
ITULO 9. ROTACION DE UN CUERPO R
IGIDO

328

a un eje jo que elegiremos como el eje z . El origen lo elegimos en algn lugar sobre el

u
eje. (Observar que el origen de coordenadas no tiene que ser necesariamente el centro
de masas.) El vector velocidad angular se puede escribir entonces en la forma:
0 = 0 z ,

donde 0 = |0 |. La direccin de 0 se elige usando la regla de la mano derecha:


o
si los dedos curvados indican la direccin de rotacin, entonces el pulgar muestra
o
o
la direccin en que apunta 0 . A esa direccin la hemos llamado z , sin prdida de
o
o

e
generalidad.
Para ser concretos, supongamos que el slio
do consta de N masas mj (j = 1, 2, . . . , N ),
ubicadas en los puntos rj , unidas por varillas
r
gidas sin masa (ver gura 9.1).
El vector posicin de cada part
o
cula se puede
descomponer como sigue:
rj = rj + zj z .

La magnitud rj = |rj | es la distancia de la


masa mj al eje de giro.

Figura 9.1
Para determinar la velocidad de cada masa, notamos nuevamente que cada una
describe un movimiento circular en torno al eje z , con frecuencia angular 0 . Por lo

tanto, la rapidez de la masa j es


vj = rj 0 .
En cuanto a la direccin, notamos que, para la gura anterior, vj apunta en la
o
direccin que entra al plano de la hoja, y que por tanto es perpendicular tanto a 0
o
como a rj . Se sigue entonces que, en general,
vj = 0 rj .
Ahora podemos evaluar la energ cintica del slido y tambin la componente
a
e
o
e
del momento angular que apunta a lo largo del eje de rotacin.
o
La energ cintica (debido a la rotacin del slido) viene dada por la suma de
a
e
o
o


CAP
ITULO 9. ROTACION DE UN CUERPO R
IGIDO

329

las energ cinticas de cada una de las masas, o sea:


as
e
N

1
2
mj vj
2
j=1

N
1
2
2
mj rj 0
=
2

K =

j=1

Observe que las coordenadas zj de las distintas masas no intervienen en la expresin


o
para la energ cintica. Algo similar ocurre al evaluar la componente z del momento
a
e
angular. Para una sola part
cula, el momento angular es
= mr v

= mr (0 r )

= m0 r 2 mr (0 r )

= m0 r 2 z mr 0 z

= m0 r 2 z m(z + r ) 0 z

z
= m0 (r 2 z 2 ) z m0 zr

2
= m0 r z m0 zr .

La componente z del momento angular es entonces


2
z = mr 0 .

La componente z del momento angular de todas las part


culas que componen el slido
o
es, por lo tanto,

Lz =

j=1

2
mj rj 0 .

ciEn dos ocasiones ya nos ha aparecido la expresin N mj rj . Es util denir expl


o
j=1
tamente este concepto: Deniremos el momento de inercia del slido en torno a un
o
eje por
N

j=1

2
mj rj .

De esta manera la energ cintica y la componente del momento angular paralela al


a
e
eje de rotacin vienen dadas por
o
1 2
K = I0
2
y
Lz = I 0 .


CAP
ITULO 9. ROTACION DE UN CUERPO R
IGIDO

330

Observemos que, gracias al concepto de momento de inercia, es posible escribir la


energ cintica y el momento angular (en torno al eje de rotacin) de modo anlogo a
a
e
o
a
la energ cintica y el momento lineal de una unica part
a
e

cula de masa m y velocidad


v0 , reemplazando la velocidad por velocidad angular, momento lineal por momento
angular, y reemplazando la masa por el momento de inercia. En otras palabras, el momento de inercia es el equivalente al concepto de masa cuando estudiamos rotaciones
de un cuerpo r
gido.
Lo anterior tambin es cierto cuando la aceleracin angular no es cero. En efecto,
e
o
si sobre un sistema aplicamos un torque en la direccin z, entonces cambiar la
o
a
componente z de su momento angular de acuerdo a la relacin
o
d0
dLz
=I
= I 0 .

dt
dt
Nuevamente, este resultado es equivalente al de una part
cula, reemplazando las cantidades lineales por rotacionales (velocidad lineal por velocidad angular, fuerza
por torque, masa por momento de inercia). Y lo importante es que ahora, dado el
momento de inercia (que siempre se puede evaluar conocida la distribucin de las
o
part
culas que forman el cuerpo), conociendo el torque sobre el cuerpo podemos evaluar su aceleracin angular.
o
z =

En el cap
tulo anterior demostramos que el momento angular de un slido que
o
se mueve en el espacio, respecto a un origen O se puede escribir como una suma de
dos contribuciones: i) el momento angular debido a la traslacin del sistema como un
o
todo, es decir, el movimiento del centro de masas con toda la masa concentrada en
ese lugar y, ii) el momento angular (rotacional intr
nseco) del sistema, L , visto desde
el centro de masas, es decir,
L = Rcm Pcm + L .
Mostraremos a continuacin que se tiene una expresin anloga para la energ cintio
o
a
a
e
ca. Sea vj la velocidad de la part
cula j medida desde un sistema de referencia O,
vj la velocidad de la misma part
cula pero vista desde el sistema de referencia jo al
centro de masas y Vcm la velocidad del centro de masas. Entonces se tiene que
vj = vj + Vcm .
La energ cintica (para el observador O) es
a
e
1
K=
2

N
2
m j vj .
j=1

A partir de las dos ultimas ecuaciones se encuentra que

K =

1
2

N
j=1

mj vj vj


CAP
ITULO 9. ROTACION DE UN CUERPO R
IGIDO

K =

1
2
1
2
1
2

331

N
j=1

mj (vj + Vcm ) (vj + Vcm )


N

mj vj 2 +
j=1

j=1

mj vj

j=1

1
2

mj Vcm vj +

+ Vcm

N
j=1

N
2
mj Vcm
j=1

1
2
mj vj + M Vcm
2

(9.1)

El primer trmino al lado derecho de la ultima ecuacin es la energ cintica del


e

o
a
e
slido vista desde el centro de masas, o sea, corresponde a la energ cintica debido
o
a
e
a la rotacin intr
o
nseca del slido. Como vimos al inicio de la presente seccin, esta
o
o
energ cintica la podemos escribir de la forma
a
e
1
K =
2

j=1

1 2
2
mj vj = I0 .
2

El segundo trmino del lado derecho de la ecuacin (11.16) es nulo ya que ( mj vj )/M
e
o
es la velocidad del centro de masas vista desde el centro de masas. Por ultimo, el ter
cer trmino del lado derecho de la ecuacin (11.16) es la energ cintica de traslacin
e
o
a
e
o
del slido como un todo. Concluimos que la ecuacin (11.16) se puede escribir de la
o
o
forma
1 2
1
2
K = M Vcm + I0 .
2
2

9.2.

Una notacin util


o

Al calcular el momento angular de una part


cula apareci la expresin r(0 r ),
o
o
y de hecho aparece habitualmente en el estudio de rotaciones. En este caso particular, ya hab
amos obtenido, en el Cap. 3, una expresin anal
o
tica para este producto,
pero recordemos que obtenerla en coordenadas cartesianas fue un proceso bastante
engorroso. Debido a que este tipo de expresiones vectoriales, y otras ms complejas,
a
aparecen con mucha frecuencia, es conveniente introducir un modo ms expedito de
a
obtener expresiones anal
ticas para ellas.
La clave de todo ser introducir una nueva notacin. Para ello, notemos primero
a
o
que un vector es conocido si se conocen sus tres coordenadas cartesianas, cada una de
las cuales podemos rotular con un
ndice i, donde i = 1 corresponde a la direccin x,
o
i = 2 a y e i = 3 a z . Si designamos como xi a los correspondientes vectores unitarios


CAP
ITULO 9. ROTACION DE UN CUERPO R
IGIDO

332

(1 = x, x2 = y y x3 = z ), entonces podemos escribir un vector en la forma:


x

Ai xi .

A=
i=1

Lo que proponemos ahora es que, si acordamos que siempre vamos a trabajar en


coordenadas cartesianas, y que siempre trabajaremos en tres dimensiones, entonces
indicar los vectores xi es redundante (porque esa informacin ya est en el sub

o
a
ndice
de la coordenada Ai ), y la suma tambin es redundante (porque siempre se entiende
e
que un vector es una suma de tres trminos, con i = 1, 2 y 3). Por lo tanto, la propuesta
e
es representar un vector con sus coordenadas, de la siguiente forma:
A Ai .
Un vector, entonces, ser representado como una cantidad con un
a
ndice. Esto es
interesante, porque pone de maniesto que un vector se puede considerar simplemente
como una coleccin de tres nmeros, A1 , A2 y A3 . Pero adems, permite una geneo
u
a
ralizacin inmediata a otros objetos matemticos. Por ejemplo, una matriz ser una
o
a
a
coleccin de nueve nmeros, ordenados en tres las y tres columnas. En nuestra nueva
o
u
notacin, podemos representarlo como un objeto con dos
o
ndices, i y j, cada uno de
los cuales va de uno a tres, como en el caso de los vectores:

A11 A12 A13


A21 A22 A23 Aij .
A31 A32 A33

A la inversa, un objeto con ningn


u ndice representar un unico nmero, es decir, un
a

u
escalar.
En resumen, en esta notacin, tenemos las siguientes asociaciones:
o
escalar
vector
matriz

A
Ai
Aij

Notemos ahora que el producto escalar se puede escribir:


3

AB =

Ai Bi .
i=1

Aqu tenemos un
,
ndice repetido (i) y una suma sobre l, desde 1 hasta 3. Como
e
todas las sumas van desde 1 a 3, vamos a simplicar la notacin del siguiente modo:
o
3
i=1

Ai Bi Ai Bi .

La convencin ser entonces la siguiente:


o
a


CAP
ITULO 9. ROTACION DE UN CUERPO R
IGIDO

333

Cada vez que aparezcan ndices repetidos en una expresin, se enten


o
der que hay una suma sobre dicho ndice, y que en dicha suma el ndice
a

repetido recorre los valores 1, 2 y 3.


Esta convencin puede parecer una trivialidad, pero el hecho es que resulta exo
traordinariamente util para realizar clculos complejos que involucran vectores o ma
a
trices. Fue introducida por Einstein, en el contexto de su Teor de la Relatividad
a
General, y se conoce por ello como notacin de Einstein o convencin de suma de
o
o
Einstein.
A un
ndice repetido se dice que es un
ndice contrado. As la convencin de

,
o
suma de Einstein indica simplemente que ndices contrados se suman. A un

ndice
que no esta repetido se le denomina un
ndice libre.
Con la notacin de Einstein, entonces, el producto escalar entre dos vectores se
o
escribe:
A B = Ai Bi .
Notemos que el
ndice de suma (i) podr llamarse de cualquier otra manera, sin
a
afectar el resultado nal ( i Ai Bi = j Aj Bj ). En notacin de Einstein:
o
Ai Bi = Aj Bj .
Tcnicamente, se dice que un
e
ndice contra o de suma es mudo, y podemos camdo
biarlo por cualquier otro (mientras el nuevo
ndice no se est usando en otra parte de
e
la misma expresin porque, recordemos,
o
ndices repetidos se deben sumar).
Observemos tambin que la expresin Ai Bi involucra expresiones con un
e
o
ndice
(vectores), pero el resultado es un escalar, un objeto que como sabemos no tiene

ndices. Pero esto es perfectamente consistente dentro de nuestra notacin! En efecto,


o
los unicos

ndices vectoriales estn contra


a
dos (no son libres), y como eso implica que
se debe sumar sobre ellos, pierden su identidad, al punto que podr llamarse de
an
cualquier otro modo y no importar
a.
Entonces, podemos reformular las asociaciones que hicimos anteriormente:
Un escalar, en realidad, ser cualquier objeto sin
a
ndices libres (a, ai bi , ai bi cj dj ,
etc.).
Un vector ser cualquier objeto con slo un
a
o
ndice libre (ai , ai bj cj , etc.).
Una matriz ser cualquier objeto con dos
a
ndices libres (aij , ai bj , aij bj ck , etc.).
Para escribir el producto cruz con notacin de Einstein, necesitamos escribir una
o
cantidad adicional:
(A B)i = ijk Aj Ak .
ijk es un objeto con tres
ndices, uno ms que una matriz. Podemos imaginarlo
a
como una matriz en tres dimensiones. A estos objetos con tres o ms
a ndices se les


CAP
ITULO 9. ROTACION DE UN CUERPO R
IGIDO

334

denomina tensores. El objeto ijk necesario para denir el producto cruz es el tensor
de Levi-Civita, y se dene como:

1
si (i, j, k) es una permutacin c
o clica de (1, 2, 3)

ijk = 1 si (i, j, k) es una permutacin antic


o
clica de (1, 2, 3)

0
en otro caso (i.e., si algunos de los
ndices son iguales entre s
)

En general, la denominacin de tensores se puede utilizar para objetos con cualo


quier nmero de
u
ndices, de modo que escalares, vectores y matrices se pueden considerar como casos particulares de tensores.

Ejercicio: Muestre que con la denicin del tensor de Levi-Civita y la notacin


o
o
de Einstein, ijk Ai Bj corresponde efectivamente a la coordenada i del producto cruz
A B.
El producto de dos tensores de Levi-Civita (contra
dos en su primer
ndice) se
puede escribir de la siguiente forma:
ijk im = j km jm k ,
donde ij es la delta de Kronecker :
ij =

1 si i = j .
0 si i = j .

Ejercicio: Con las deniciones anteriores, recupere la expresin demostrada en el


o
Cap. 3:
A (B C) = B(A C) C(A B) .
i

9.3.

Momento de inercia

Ya hemos observado, en la Sec. 9.1, que el momento de inercia es de vital importancia para describir las rotaciones de un slido, ya que equivale al concepto de masa.
o
Para un slido constituido de N masas discretas (unidas r
o
gidamente con varillas sin
peso), el momento de inercia viene dado por
N

I=
j=1

2
mj rj .

Para distribuciones de masa continua, la expresin anterior debe sustituirse por una
o
con integrales que adecuadamente describa la situacin. Por ejemplo: si un cuerpo
o


CAP
ITULO 9. ROTACION DE UN CUERPO R
IGIDO

335

slido viene descrito por una densidad de masa (x, y, z) = (r ), entonces el momento
o
de inercia en torno al eje z viene dado por

(x2 + y 2 ) (r ) dx dy dz .

I=
Slido
o

En la ultima expresin (x2 + y 2 ) es el cuadrado de la distancia al eje de la masa del

o
volumen d3 r = dx dy dz ubicado en el lugar r.
A continuacin evaluaremos algunos momentos de inercia sencillos, para cuerpos hoo
mogneos y con al menos un eje de simetr En varios casos, observaremos que poe
a.
demos encontrar el momento de inercia fcilmente dividiendo un cuerpo complicado
a
en cuerpos ms sencillos con momento de inercia conocido.
a

Ejemplo 1:
Evaluemos el momento de inercia de una
varilla de largo L y masa M en torno a un
eje que pasa perpendicularmente por uno
de sus extremos (ver gura 9.2).
Figura 9.2
La densidad lineal de la varilla es = M/L. El trozo de varilla de largo dx que se
encuentra a una distancia x del eje tiene una masa igual a dx y su contribucin al
o
momento de inercia es x2 dx. Sumando todas las contribuciones desde x = 0 hasta
x = L se obtiene
L
1 3L
L3
2
x dx =
I=
x
,
=
3 0
3
0
o sea
I=

M L2
.
3


CAP
ITULO 9. ROTACION DE UN CUERPO R
IGIDO

336

Ejemplo 2:
Evaluemos el momento de inercia de un
anillo de radio R y masa M en torno a
un eje que pasa perpendicularmente por
el centro (ver gura 9.3).
Como toda la masa del anillo est a la
a
distancia R del eje, el momento de inercia
es simplemente
I = M R2 .

Figura 9.3

Ejemplo 3:
Evaluemos el momento de inercia de un disco uniforme de radio R y masa M en torno
a un eje que pasa perpendicularmente por el centro (ver gura 9.4). La densidad
supercial del disco viene dada por = M/(R2 ). Para encontrar el momento de
inercia subdividiremos el disco en anillos innitesimales.
El momento de inercia dI de un anillo de
radio r y ancho dr viene dado por (ver
ejemplo anterior)
dI = (masa del anillo) r 2 .
Pero la masa de tal anillo es
(masa del anillo) = 2r dr ,
luego
dI = 2 r 3 dr .
Figura 9.4
Sumando la contribucin de todos los anillos desde r = 0 hasta r = R se encuentra
o
R

I=

dI =

2 r 3 dr = 2

r4
4

=
0

Sustituyendo la expresin para se obtiene nalmente


o
I=

M R2
.
2

4
R .
2


CAP
ITULO 9. ROTACION DE UN CUERPO R
IGIDO

337

Notemos que el momento de inercia es menor que en el caso del anillo, lo cual
es esperable, ya que ahora la masa se encuentra distribuida a distancias menores o
iguales que R.
Observemos que en todos los ejemplos estudiados, se repite una caracter
stica
comn: el momento de inercia siempre resulta tener, al menos para estos cuerpos
u
sencillos, la siguiente forma: masa, multiplicada por el cuadrado de una longitud
caracter
stica del cuerpo (longitud de la barra, radio del aro, etc.), por un factor
numrico (que depende de la geometr del cuerpo).
e
a
Ejemplo 4:
Encontremos el momento de inercia de una esfera uniforme de radio R y masa M
alrededor de un eje que pasa por el centro.
La densidad de masa de la esfera viene
dada por
M
0 = 4 3 .
3 R
Para encontrar el momento de inercia de
una esfera supongamos que ella est consa
tituida por numerosos discos innitesimales de grosor dz (ver gura 9.5). El radio
del disco innitesimal que se encuentra a

una altura z viene dado por R2 z 2 . El


rea de tal disco es por lo tanto
a

Figura 9.5

A = (R2 z 2 ) .
Para la masa dM (que es el volumen del disco innitesimal multiplicado por su
densidad) se obtiene
dM = 0 (R2 z 2 ) dz .
La contribucin de tal disco al momento de inercia de la esfera es (ver ejemplo 3)
o
dI =

0
1
dM (R2 z 2 ) =
(R2 z 2 )2 dz .
2
2


CAP
ITULO 9. ROTACION DE UN CUERPO R
IGIDO

338

Sumando la contribucin de todos los discos desde z = R hasta z = R se encuentra


o
R

I=

dI =

dI =
R

=
=

0
2

0
2

R
R

0
2

R
R

(R2 z 2 )2 dz

(R4 2R2 z 2 + z 4 ) dz

1
2
R4 z R2 z 3 + z 5
3
5

0
2
1
=
2 R5 R5 + R5
2
3
5
8
= 0 R 5
15

R
R

Sustituyendo la expresin para 0 se obtiene nalmente


o
2
I = M R2 .
5
Nuevamente, el momento de inercia tiene la forma observada: la masa, por el
cuadrado de una distancia caracter
stica (el radio de la esfera), por un factor numrico.
e
Ejemplo 5:
Evaluemos el momento de inercia de una esfera hueca de radio interno Ri y radio
externo Re , hecha de un material de densidad uniforme 0 , para un eje que pasa por
el centro.
Es fcil resolver este problema si se observa que la esfera hueca se puede pensar
a
como dos esferas concntricas sobrepuestas: una de radio Re con densidad 0 y otra
e
de radio Ri con densidad negativa 0 .
El momento de inercia de esta superposicin (que coincide con la de la esfera
o
hueca) viene dado por (ver ejemplo 4)
I=

8
8
8
5
5
5
5
0 Re + (0 ) Ri = 0 (Re Ri ) .
15
15
15

Expresemos el resultado tambin en trminos de la masa de la esfera hueca, que es:


e
e
4
3
3
M = (Re Ri ) 0 .
3
Sustituyendo esta relacin en la expresin para el momento de inercia se encuentra
o
o
5
2 (R5 Ri )
I= M e
3 .
3
5 (Re Ri )


CAP
ITULO 9. ROTACION DE UN CUERPO R
IGIDO

339

Para Ri 0, se recupera, tal como debe ser, el resultado del ejemplo anterior.
En este caso, el cuerpo no tiene una unica longitud caracter

stica (hay un radio


interno y uno externo), de modo que el momento de inercia no tiene exactamente la
misma forma observada en los casos anteriores.
Ejemplo 6:
Evaluemos el momento de inercia de una cscara esfrica de radio R y masa M ,
a
e
para un eje que pasa por el centro.
Para resolver este problema usamos el resultado del ejemplo anterior, evalundolo
a
en el l
mite Ri Re . Para encontrar este l
mite pongamos Ri = Re con muy
pequeo. Se tiene:
n
3
3
3
Re Ri = Re (Re )3

3
Re

3
Re

3
3
Re Re

1
Re

13
Re

2
3Re .

De la misma manera se encuentra que


5
5
4
Re Ri = 5Re .

Reemplazando estos resultados en la expresin para el momento de inercia, e iguao


lando Re con R, se encuentra (para el momento de inercia de una cscara esfrica)
a
e
2
I = M R2 .
3
Ejemplo 7:
Evaluemos el momento de inercia de un
anillo de radio r y masa M en torno a un
eje que coincide con un dimetro del anillo
a
(ver gura 9.6).
Para resolver este problema subdividamos
el anillo en numerosos sectores angulares
innitesimales. La densidad lineal del anillo es = M/(2R). La masa del anillo
del sector comprendido entre y + d
es r d. Su contribucin al momento de
o
inercia del anillo es

Figura 9.6


CAP
ITULO 9. ROTACION DE UN CUERPO R
IGIDO

340

dI = R d (R sin )2 .
Sumando la contribucin de todos los sectores (desde = 0 hasta = 2) se obtiene
o
2

I=

dI =

R3 sin2 d .

Pero

sin2 d = ,

luego
I = R3 =

1
M R2 .
2

Notemos que, de acuerdo a lo observado anteriormente, el momento de inercia


deb ser proporcional a M R2 , igual que en el ejemplo 2. Sin embargo, ahora el eje
a
en torno al cual se calcula el momento de inercia es distinto, y eso tiene el efecto de
cambiar el factor numrico involucrado. Notemos, adems, que en el ejemplo 2 toda
e
a
la masa estaba a una distancia R del eje, y ahora toda la masa est a una distancia
a
menor o igual que R. Por lo tanto, era esperable que el momento de inercia fuese
menor que en el ejemplo 2.
Problema resuelto en clases: 9.1
Problema resuelto en clases: 9.15
(Generalizado para un pndulo f
e
sico arbitrario)
Problema resuelto en clases: 9.10
Problema resuelto en clases: 9.14
(No exactamente, sino discutir el problema del lanzamiento vertical de distintos
objetos rotando, y cmo la energ rotacional crece a expensas de la traslacional.)
o
a
Discusin de la tcnica de patinadoras y clavadistas para aumentar su velocidad
o
e
de rotacin.
o
Problema resuelto en clases: 9.35


CAP
ITULO 9. ROTACION DE UN CUERPO R
IGIDO

9.4.

341

Problemas

1. (Teorema de Steiner o teorema de los ejes paralelos.) Demuestre que el momento


de inercia I para las rotaciones de un cuerpo slido alrededor de un eje L es
o
I = I0 + M R 2 ,
donde I0 es el momento de inercia para rotaciones del slido alrededor del eje
o
paralelo a L que pasa por el centro de masas y R es la distancia de separacin
o
de los dos ejes.
2. Encuentre el momento de inercia de las supercies, de densidad supercial uniforme 0 , mostrados en la gura 9.7 y en torno a los ejes ah indicados.

Figura 9.7

3. Considere un sistema de dos masas m1 y m2 , separadas por una distancia r.


Demuestre que el momento de inercia con respecto al eje que pasa por el centro
de masas en forma perpendicular a la l
nea que los une, viene dado por r 2 ,
donde = m1 m2 /(m1 + m2 ) es la masa reducida del sistema.
4. Encuentre el momento de inercia de los alambres, de densidad lineal uniforme
0 , mostrados en la gura 9.8 y en torno a los ejes ah indicados.


CAP
ITULO 9. ROTACION DE UN CUERPO R
IGIDO

342

Figura 9.8
5. La molcula de metano consiste de un
e
a
tomo de carbono localizado al centro de
un tetraedro regular cuyos vrtices estn
e
a
ocupados por 4 tomos de hidrgeno. La
a
o
distancia C H es de 1.08 . (1 =
A
A
108 cm). Cul es el momento de inercia
a
de la molcula de metano para una rotae
cin alrededor de un eje C H? Las masas
o
de los tomos de hidrgeno y carbono son:
a
o
27 Kg y m =19.9 1027 Kg,
mH =1.68 10
C
respectivamente.

Figura 9.9

6. Una esfera sube rodando un plano inclinado en 30 . Cuando la esfera se encuentra al pie del plano, su centro de masas se traslada con una velocidad de
5 m/s. Hasta dnde subir la esfera por el plano inclinado? Cunto tiempo
o
a
a
tardar en regresar al punto de partida?
a
7. Se enrolla una cuerda alrededor de la plataforma de un carrusel de radio R = 2 m
para echarlo andar. Durante 10 s se tira
de la cuerda con una fuerza de 200 N. Durante ese tiempo el carrusel da una vuelta
completa. Cul es el momento de inercia
a
del carrusel?
Respuesta: I =3183 kg m2 .
Figura 9.10


CAP
ITULO 9. ROTACION DE UN CUERPO R
IGIDO

343

8. Los dos discos mostrados en la gura adjunta tienen masas m y radios R iguales.
El disco superior puede rotar libremente
alrededor de su eje. Una cuerda est enroa
llada alrededor de ambos discos. Encuentre:
a) La aceleracin del centro de masas
o
del disco inferior.
b) La tensin de la cuerda.
o
c) La aceleracin angular de cada disco
o
alrededor de su centro de masas.
9. Una esfera de densidad uniforme 0 y radio r rueda sin deslizarse a lo largo de una
v que posee una vuelta circular de radio
a
R (ver gura 9.12). La esfera inicia su movimiento partiendo, del reposo, desde una
altura h. Cul es la m
a
nima altura h requerida para que la esfera no deje la v
a?
Cul ser la altura h si la bola en lugar
a
a
de rodar se desliza resbalando?

Figura 9.11

Figura 9.12

Respuesta: h = 27 (R r)/10 .
10. Una bola de palitroque, de radio R y masa M , se lanza de manera que inicialmente resbale (sin que ruede) con velocidad v0 . Si el coeciente de roce entre
el suelo y la bola es , qu distancia recorrer la bola antes de que ruede sin
e
a
resbalar? Cul es su velocidad nal?
a
Respuesta: vf = 5v0 /7 .
11. Considere dos poleas (discos) de masas
m1 , m2 y radios R1 , R2 , respectivamente. Con estas poleas se realiza el montaje
mostrado en la gura adjunta (la cuerda
est enrollada en torno a la polea # 2).
a
Encuentre la aceleracin de la masa M .
o
Respuesta:
M + m1
.
a = g
3
M + 2 m1 + 2m2
Figura 9.13


CAP
ITULO 9. ROTACION DE UN CUERPO R
IGIDO

344

12. Una varilla de largo L y masa M puede rotar libremente alrededor de un pivote A. Una bala de masa m y velocidad v impacta contra la varilla en un
punto P alejado una distancia a desde el pivote, quedando incrustada en ella.
a) Encuentre el momento angular alrededor del punto A inmediatamente
antes y despus de la colisin.
e
o
b) Determine el momento lineal del sistema inmediatemente antes y despus de la colisin.
e
o
c) Cul es el valor Q de la colisin,
a
o
es decir, cunta energ es disipada
a
a
durante el proceso?

Figura 9.14

13. Dos nios, cada uno de masa M , estn sentados en los extremos de una barra
n
a
horizontal de largo L y masa m. La barra gira inicialmente con una velocidad
angular 0 alrededor de un eje vertical que pasa por su centro.
a) Cul ser la velocidad angular si caa
a
da nio se mueve una distancia d han
cia el centro de la barra (sin tocar el
suelo)?
b) En cunto cambiar la energ
a
a
a
cintica de rotacin del sistema?
e
o

Figura 9.15

14. Una esfera, un disco y un aro, hechos de materiales homogneos, tienen el mismo
e
radio R y la misma masa M . Los tres objetos se dejan libres desde la parte
superior de un plano inclinado. Los tres objetos parten desde el reposo y ruedan
sin resbalar. El plano tiene un largo L y su inclinacin respecto a la horizontal
o
es .
a) Cules son sus velocidades al llegar al pie del plano inclinado?
a
b) Encuentre la fuerza de roce fr en cada caso.
c) Cunto tarda cada uno de los objetos en llegar a la parte inferior?
a
15. Un aro circular de radio R oscila en torno a un eje horizontal que pasa por A
(ver gura). El eje es normal al plano del aro.


CAP
ITULO 9. ROTACION DE UN CUERPO R
IGIDO

a) Cul ser el largo de un pndulo


a
a
e
simple con igual per
odo de oscilacin que el del aro? (Haga el anlisis
o
a
slo para pequeas oscilaciones.)
o
n
b) Se desea que el aro d una vuelta
e
completa alrededor de A. Cul es
a
la m
nima velocidad angular que debe poseer el aro, en la parte inferior,
para que esto sea posible?

345

Figura 9.16

16. Un aro circular de masa m y radio R descansa sobre una supercie horizontal sin
roce (ver gura, vista desde arriba). Contra el aro se dispara tangencialmente
una bala con velocidad v0 , cuya masa tambin es m. La bala queda incrustada
e
en el aro.
a) Describa el movimiento del sistema
despus del choque.
e
b) Cul es la velocidad del centro de
a
masas del sistema antes y despus
e
del choque?
c) Cul es el momento angular del sisa
tema respecto a su centro de masas
antes del choque?
d) Cul es la velocidad angular con
a
que gira el sistema despus del choe
que?
e) Cunta energ cintica se pierde
a
a
e
en el choque?

Figura 9.17

17. Un aro de masa M y radio r, rueda sin resbalar por la supercie interior de una cinta
circular ja de radio R (ver gura 9.18).
Encuentre el per
odo de este movimiento
para pequeas oscilaciones alrededor de la
n
vertical.
Respuesta:
T = (2)/0 , con

2
0 =

g
.
2(R r)

Figura 9.18


CAP
ITULO 9. ROTACION DE UN CUERPO R
IGIDO

346

18. Considere la mquina de Atwood mostraa


da en la gura adjunta. La polea consta de
un disco uniforme de masa m (que coincide con el valor de la masa ms pequea
a
n
colgada de la mquina) y radio R. El moa
mento de inercia para rotaciones en torno
al eje de un disco es I = mR2 /2. El roce entre la cuerda y la polea hace que esta ultima gire mientras las masas estn en

e
movimiento. Suponga que la cuerda no tiene masa y que no desliza sobre la polea.
La masa 2m parte del reposo desde una
altura h.
Figura 9.19
a) Usando el teorema de conservacin de la energ encuentre la velocidad
o
a,
de la masa 2m cuando sta llega al suelo.
e
b) Encuentre la tensin de la cuerda a ambos lados de la mquina de Atwood.
o
a
Es decir, encuentre 1 y 2 en funcin de m, g y R. (Cuando el momento
o
de inercia de la polea no se puede despreciar (lo que es el caso del presente
problema) entonces la tensin de la cuerda no es la misma a ambos lados
o
de la polea.)
c) Encuentre la tensin de la cuerda que sujeta la polea mientras las masas
o
estn en movimiento.
a
d) Encuentre la tensin de la cuerda que sujeta la polea despus de que la
o
e
masa 2m lleg al suelo (y todas las componentes de la mquina de Atwood
a
a
estn en reposo).
a
19. Considere dos poleas jas unidas por una correa (o cadena) de transmisin
o
tal como se muestra en la gura adjunta. Una masa M colgada por una cuerda
enrollada en la polea #1 pone en movimiento el sistema. Suponga que las poleas
son discos de radio R y tienen una masa tambin igual a M (es decir, el momento
e
de inercia de las dos poleas coinciden, tenindose I = M R2 /2). Note que una
e
correa (o cadena) de transmisin slo puede transmitir una fuerza de traccin.
o o
o
Para el presente problema slo la parte superior de la correa transmite una
o
fuerza entre las poleas.


CAP
ITULO 9. ROTACION DE UN CUERPO R
IGIDO

347

Figura 9.20
a) Encuentre la tensin T de la cuerda.
o
b) Encuentre la aceleracin angular de la polea #1.
o
c) Usando la ley de conservacin de la energa, encuentre la velocidad v que
o

tiene la masa M despus de haber bajado una distancia h. (La masa M


e
parte desde el reposo).

20. Una barra uniforme de largo L apoyada


contra la pared comienza a resbalar (sin
roce). Inicialmente el ngulo que forma
a
con la pared es 0 . Encuentre la altura z
para la cual el extremo A de la barra se
separa de la pared vertical.
2
Respuesta: z = 3 L cos 0 .

Figura 9.21


CAP
ITULO 9. ROTACION DE UN CUERPO R
IGIDO

348

21. Una carretilla de hilo, formada de dos discos y un cilindro de las dimensiones
indicadas en la gura 9.22a, se tira del hilo que tiene enrollado tal como se
muestra en la gura 9.22b. Encuentre la aceleracin de la carretilla de hilo si
o
sta rueda sin resbalar.
e

Figura 9.22
22. Considere un automvil de masa M , cuya geometr se muestra en la gura
o
a
adjunta, y que inicialmente se mueve con velocidad v0 hatx. Suponga que en
cierto instante el automvil frena bloqueando las dos ruedas delanteras. Eno
cuentre la distancia que el automvil alcanza a recorrer durante el frenado si
o
el coeciente de roce cinemtico entre el pavimento y las ruedas es c . Asuma
a
que durante el frenado, las ruedas traseras, en todo instante, estn en contacto
a
con el pavimento, situacin que generalmente se da en la prctica.
o
a
Bajo ciertas condiciones extremas de frenado, el automvil podr elevarse por la
o
a
parte trasera y tumbarse. Encuentre la condicin que debe satisfacerse para que
o
el automvil quede, en todo instante, con las cuatro ruedas sobre el pavimento.
o
Haga tambin un anlisis del proceso de aceleracin del automvil.
e
a
o
o

Figura 9.23


CAP
ITULO 9. ROTACION DE UN CUERPO R
IGIDO

349

23. Considere una varilla r


gida (de masa
despreciable), que en cada uno de sus
extremos tiene adosada una masa m.
La varilla se desplaza inicialmente sin
rotar sobre el plano (x, y), con la velocidad del centro de masas v = v0 x y

con la varilla orientada de manera de


formar un ngulo con el eje x, (ver
a

gura). En cierto lugar una de las masas choca elsticamente con una pared
a
r
gida, tal como se muestra en la gura. Despus de la colisin (el centro de
e
o
Figura 9.24
masas de) la varilla con las masas se
trasladar uniformemente, rotando sia
multneamente con velocidad angular
a
constante.
Desprecie el roce entre las masas m y el plano y suponga tambin que la pared
e
est pulida, es decir, no hay fuerzas de roce entre la masa m y la pared cuando
a
entran en contacto.
a) Determine la velocidad angular de la varilla despus de la colisin.
e
o
b) Encuentre el impulso transmitido al sistema por la pared durante la colisin.
o
c) Verique que el resultado obtenido en la parte b) da los resultados correctos
en los l
mites = 0 y = /2.

24. Uno de los extremos de un resorte ideal


de constante elstica k se ja a una paa
red. El otro extremo se ja al eje de una
rueda cil
ndrica de radio R y masa M .
El resorte se comprime una distancia a,
manteniendo su posicin horizontal.
o
Figura 9.25
Estando el cilindro en contacto con el suelo (supercie rugosa) se suelta ste
e
del reposo. Calcule la velocidad angular del cilindro cuando la elongacin del
o
resorte es nula. Suponga que el cilindro no resbala.
25. Cuatro bolitas idnticas, de masa m cada una, se unen mediante varillas de
e
masa despreciable de largo a de tal forma que las bolas queden ubicadas en los
vrtices de un cuadrado.
e


CAP
ITULO 9. ROTACION DE UN CUERPO R
IGIDO

350

a) Calcule el momento de inercia con


respecto a un eje a lo largo de la diagonal del cuadrado.
b) Calcule el momento de inercia con
respecto a un eje paralelo a uno de
los lados y que pasa por el centro de
ste.
e
c) Si los dos cuadrados se hacen rotar,
cada uno entorno a los ejes descritos
anteriormente y con la misma velocidad angular, determine cual de ellos
(y en que porcentaje) tiene mayor
energ cintica.
a
e
26. Un bloque rectangular y un cilindro se
unen con una varilla como se muestra en
la gura. El cilindro rueda sin resbalar
mienmtras que el bloque desliza sobre el
piso rugoso (c ). Si la masa del cilindro y
del bloque es la misma (M/2) y el radio
del cilindro es R, calcule el tramo recorrido por el sistema desde aquel instante en
que el sistema se desplaza con rapidez v0 .
Compare con el resultado que obtendr si
a
el cilindro se mantiene fuera de contacto
con el piso.

Figura 9.26

Figura 9.27

27. Considere un cubo de arista L y masa que


se desliza von velocidad v0 sobre un plano
horizontal (sin roce). En cierto lugar el cubo se encuentra con un tope.
a) Demuestre que el momento de inercia del cubo respecto a un eje de rotacin que coincide con una de sus
o
aristas es I = 2M L2 /3.

Figura 9.28

(b) Cul es la m
a
nima velocidad que debe tener el cubo para que se vuelque?


CAP
ITULO 9. ROTACION DE UN CUERPO R
IGIDO
28. Una ardilla de masa m corre (aceleradamente) dentro de un cilindro hueco de radio R y masa M . La ardilla en ningn mou
mento resbala y el cilindro posa sobre un
plano rugoso horizontal (sobre el cual rueda sin resbalar). A consecuencia de su movimiento acelerado la ardilla se mantiene
siempre a una altura h del suelo. Determinar la aceleracin con que se traslada
o
el centro de masas del cilindro (que es la
misma con que se traslada la ardilla).
29. Un disco de radio R y masa M , inicialmente en reposo, puede girar libremente
alrededor de un eje vertical jo. Sobre ese
disco se coloca otro disco, de radio r y masa m que inicialmente rota con una velocidad angular 0 . Debido al roce entre los
dos discos el segundo disco eventualmente
quedar en reposo respecto al primero. Si
a
la separacin entre los centros entonces es
o
D, encuentre la velocidad angular nal
con que girarn los dos disco entorno al
a
eje.

351

Figura 9.29

Figura 9.30

30. Una varilla de masa M y longitud L cuelga con uno de sus extremos jo al techo.
La varilla puede rotar libremente entorno
a este punto. Sobre el piso un cuerpo pequeo de masa m choca elsticamente con
n
a
el extremo inferior de la varilla.
a) Determine la velocidad angular de la
varilla inmediatamente despus de la
e
colisin.
o
b) Determine la masa de la varilla si
a consecuencia del choque la masa
incidente queda detenida.

Figura 9.31


CAP
ITULO 9. ROTACION DE UN CUERPO R
IGIDO

352

31. Una cuerda se enrolla entorno a un cilindro. El cilindro se ubica sobre un plano horizontal rugoso () y en contacto con una
pared vertical del mismo material del piso (ver gura). La cuerda se tira con una
fuerza F hacia abajo. Calcular la razn
o
entre las fuerzas normales experimentadas
en el suelo y la pared, respectivamente.

Figura 9.32

32. Considere la conguracin experimental


o
mostrada en la gura adjunta. Suponga
que no hay roce entre la carretilla de hilo
y el plano inclinado. Suponiendo conocidos el momento de inercia I de la carretilla para rotaciones alrededor de su eje, los
radios R y r, el ngulo de inclinacin del
a
o
plano inclinado, encuentre la aceleracin a
o
del eje de la carretilla.

Figura 9.33

33. Un mono se encuentra sobre una plataforma que puede rotar (sin roce) alrededor
de un eje. Inicialmente la plataforma y el
mono se encuentran en reposo. Qu debe
e
hacer el mono para alcanzar los pltanos
a
que estn al otro lado. No hay nada a la
a
mano del mono y supondremos que con
slo soplar no es posible poner en movio
miento la plataforma.

Figura 9.34

34. Considere un pndulo (f


e
sico) formado por
una varilla de largo R y masa M en cuyo
extremo est adosada una esfera de radio
a
R y masa 2M . El pndulo cuelga de uno
e
de los extremos de la varilla.
a) Determine el momento de inercia del
pndulo para rotaciones en torno al
e
punto de suspensin.
o
b) Determine el per
odo de este pndue
lo para pequeas oscilaciones.
n

Figura 9.35


CAP
ITULO 9. ROTACION DE UN CUERPO R
IGIDO

353

35. Una rueda de bicicleta se sostiene del eje


con un hilo (amarrado a un solo lado). El
punto de amarre se ubica a D = 20 cm
del centro de la rueda. El neumtico y la
a
llanta pesan M = 4 kg y tienen un radio R = 30 cm. La rueda se hace girar a
10 rev/s. El eje se orienta (inicialmente)
de manera horizontal.
a) Demuestre que el eje de la rueda
se mantendr en posicin horizontal
a
o
y que sta realizar un movimiento
e
a
circular (coincidiendo el eje de este
movimiento con el hilo). Este movimiento se llama precesin.
o

Figura 9.36

b) Encuentre la velocidad angular de


precesin.
o
36. Un cubo, de lado h y masa m, est colocado sobre una cinta transportadora
a
en la orientacin que se muestra en la gura adjunta. El coeciente de roce
o
esttico entre el bloque y la cinta es e = 0, 5 y el coeciente de roce cintico
a
e
es c = 0, 4. En t = 0, la cinta comienza a moverse, aumentando linealmente
su aceleracin. En t = 20 s su aceleracin es a = 3 m/s2 x y en t = 60 alcanza
o
o

9 m/s2 x. Entre t = 0 y t = 60 s, se observa que el bloque se desplaza en la

direccin x manteniendo su orientacin original (es decir, sin rotar con respecto
o
o
a su centro de masa).

Figura 9.37
a) Calcule la fuerza de roce que acta sobre el bloque cuando la aceleracin
u
o
de la cinta transportadora es 3 m/s2 .
b) Haga un grco de la aceleracin en funcin del tiempo.
a
o
o
c) Considere un sistema de coordenadas solidario al bloque, con origen en su
centro de masa y con el eje x paralelo a la direccin en que se mueve la
o
cinta. Cul es la coordenada x del punto donde acta la fuerza normal
a
u
efectiva cuando la aceleracin de la cinta transportadora es 6 m/s2 x ?
o


CAP
ITULO 9. ROTACION DE UN CUERPO R
IGIDO

354

37. Un disco homogneo de masa M y radio R cuelga desde un pivote O. Un


e
proyectil de masa m es disparado horizontalmente con velocidad v, quedando
incrustado en el borde del disco.
O

m
v

a) Determine el momento angular respecto al punto O antes y despus de la


e
colisin.
o
b) Determine el momento de inercia del sistema respecto al punto O despus
e
de la colisin.
o
c) Cul es la m
a
nima velocidad v para que el disco con la bala incrustada
d una vuelta completa respecto a O?
e
38. Calcule el momento de inercia de un cono slido homogneo de masa M , radio
o
e
basal R y altura h, respecto a un eje que pasa por el eje de simetr del cono.
a
Hint: El volumen de este cono es R2 h/3.

39. Considere una carretilla homognea de masa M , compuesta por dos ruedas de
e
radio Rr y ancho Lr , unidas por el centro por un cilindro de radio Rc = Rr /2 y
largo Rc = 2Lr . Esta carretilla rueda sin resbalar por una supercie horizontal,


CAP
ITULO 9. ROTACION DE UN CUERPO R
IGIDO

355

de tal modo que es el cilindro central (y no las ruedas) el que permanece en


contacto con la supercie (ver gura).

h
g

2 Rc

Lr

Lc

Lr

2 Rr
vista lateral

vista frontal

Eventualmente, la carretilla sube por un plano inclinado, detenindose a una


e
altura h respecto a la supercie horizontal. Si la velocidad angular inicial es 0 ,
determine la altura h.
Cul es el momento angular de la carretilla cuando est a una altura h/2?
a
a
Escriba su resultado en trminos de Rr y Lr .
e
40. Un bloque de masa M y un anillo de masa 2M y radio R, ambos homogneos,
e
cuelgan unidos por una cuerda ideal sin masa, que pasa por una polea (ver
gura). La polea es un disco homogneo de masa M y radio 3R.
e
111111111111111
000000000000000
111111111111111
000000000000000
111111111111111
000000000000000

El sistema completo se suelta desde el reposo. Encuentre la aceleracin del anillo


o
y la del bloque.


CAP
ITULO 9. ROTACION DE UN CUERPO R
IGIDO

356

41. Calcule el momento de inercia del cuerpo mostrado en la gura, respecto a un


eje que pasa por su eje de simetr El cuerpo es similar a una taza con un
a.
platillo: consta de una base cil
ndrica slida de radio R, altura h y masa m.
o
Sobre l hay un cascarn cnico (hueco), de radios r (extremo inferior) y R
e
o o
(extremo superior), altura H y masa M , y es abierto en ambos extremos (slo
o
hay masa en el manto del cono).
Ambas partes, el cilindro que sirve de base y el cascarn cnico, tienen densidad
o o
uniforme.

H
h
r
R

42. Considere un cascarn cnico (hueco), de radios r (extremo inferior) y R (exo o


tremo superior), altura H, y abierto en ambos extremos.
El cascarn gira en torno a su eje de simetr con velocidad angular = z .
o
a

Un bloque de masa m se encuentra sobre la supercie interna del cono mientras


este gira. Si hay un coeciente de roce esttico entre el bloque y el cascarn
a
o
cnico, encuentre la m
o
nima frecuencia angular para que el bloque no se deslice
hacia abajo.
Importante: Resuelva este problema en el sistema de referencia solidario al
cascarn cnico.
o o


CAP
ITULO 9. ROTACION DE UN CUERPO R
IGIDO
z

357

r
R

43. Considere el siguiente sistema, formado por un bloque de masa m y un cilindro


de radio R y masa M .

m
g

M
R
El bloque y el cilindro estn unidos por una cuerda, que se puede enrollar/desenrollar
a
en el cilindro. El bloque est ubicado sobre un plano inclinado un ngulo resa
a
pecto a la horizontal.
Calcule las aceleraciones lineales del bloque y el cilindro.

9.5.

Solucin a algunos de los problemas


o

Solucin al problema 8
o


CAP
ITULO 9. ROTACION DE UN CUERPO R
IGIDO

358

Denamos el eje z apuntando hacia arriba y concentrmonos primeramente en el

e
disco inferior. Sea a = a (con a > 0) la aceleracin lineal del disco inferior. Para
z
o
tal disco, usando la segunda ley de Newton, se encuentra la ecuacin de movimiento
o
Ftot = mg + T z = ma ,
z

o sea,
mg T = ma .

(9.2)

Sea = la aceleracin angular del disco inferior. El torque ejercido por el peso

o
(respecto al centro de masas del disco) es nulo mientras que el torque ejercido por la
cuerda es = T R. Se tiene
= TR =

d
d
= (I) = I = I .

dt
dt

Usando el valor del momento de inercia de un disco I = mR2 /2, se encuentra


1
T = mR .
2

(9.3)

Siendo los dos discos iguales y siendo que el torque sobre el disco superior (respecto
a su centro de masas) es igual al del disco inferior, se concluye que ambos discos se
desenrollarn con la misma aceleracin y velocidad angular.
a
o
La aceleracin lineal a y la aceleracin angular (que es la misma para ambos
o
o
discos) no son magnitudes independientes, sino que estn correlacionadas. En efecto,
a
se tiene que
2R = a .
(9.4)
A partir de las tres ecuaciones (9.2), (9.3) y (9.4), con las tres incgnitas T , a y
o
, se encuentra a = 4g/5 , T = mg/5 y = 2g/(5R) .
Solucin al problema 15
o
Para encontrar las ecuaciones de movimiento de un problema en que la energ se
a
conserva, en muchas ocasiones el mtodo ms fcil consiste en escribir una expresin
e
a a
o
para la energ y derivar sta respecto al tiempo.
a
e
Por ejemplo, para una masa que cae bajo el efecto de la gravedad: la energ total
a
viene dada por
1

E = mgz + mz 2 .
2
Derivando esta expresin respecto al tiempo se encuentra
o
1

0 = mgz + m2z z ,


CAP
ITULO 9. ROTACION DE UN CUERPO R
IGIDO

359

o sea, la ecuacin de movimiento para la ca libre z = g.


o
da

Usemos esta idea para resolver el presente problema. Si es el ngulo de desviaa

cin del anillo respecto a la normal y es su velocidad angular, la energ total del
o
a
sistema vendr dada por
a
1
E = mgR(1 cos ) + I 2 .
2

(9.5)

Aqu m es la masa del aro e I es el momento de inercia respecto al punto de suspensin

o
A. El primer trmino al lado derecho representa el cambio de energ potencial del aro
e
a
(respecto a su posicin de equilibrio) mientras que el segundo es la energ cintica
o
a
e
rotacional en torno a A.
Derivando (9.5) respecto al tiempo se obtiene

0 = mgR sin + I ,
o sea,

mgR sin .
+
I
Usando el teorema de Steiner y el resultado del ejemplo 2 de la segunda seccin,
o
deducimos que el momento de inercia del aro, en torno a A, es I = 2mR2 . Usando
esta relacin, y la aproximacin sin para ngulos pequeos, se obtiene
o
o
a
n
g

=0.
+
2R

Esta ecuacin de movimiento para el aro corresponde a la de un oscilador armnico


o
o
y coincide con la de un pndulo de largo L = 2R. El per
e
odo de oscilacin es
o
T = 2

2R
.
g

Para que el aro d una vuelta completa la energ cintica en la parte inferior
e
a
e
debe coincidir con 2mgR, que es la diferencia de energ potencial que el aro debe
a
sobreponer. Si denotamos por 0 a la velocidad angular en el m
nimo, se tiene
1 2
I = 2mgR ,
2 0
o sea,
0 =

2g
.
R


CAP
ITULO 9. ROTACION DE UN CUERPO R
IGIDO

360

Solucin al problema 18
o
a) El cambio de energ potencial debe ser igual a la energ cintica nal. La masa
a
a
e
2m baja una distancia h mientras que la masa m sube una distancia h. Luego
U = mgh .
Sea v0 la rapidez nal de la masa 2m, justo antes de chocar con el suelo. Entonces
la energ cintica, en ese instante, es
a
e
1
1
1 2
2
2
mv0 + (2m)v0 + I0 ,
2
2
2
donde 0 = v0 /R es la velocidad angular nal de la polea. Usando el valor
I = mR2 /2 para el momento de inercia se encuentra que
K=

2
K = 2mv0 .

Igualando K con U se encuentra


2
v0 =

gh
.
2

b) Los diagramas de cuerpo libre de las dos masas nos dan las ecuaciones de movimiento
1 mg = ma
y

(2m)g 2 = (2m)a ,

donde a es la aceleracin (hacia arriba) de la masa m. Sea 0 la aceleracin


o
o
angular de la polea. El hecho de que la cuerda no resbale sobre la polea nos da
la relacin
o

0 R = a .
Finalmente, evaluando el torque total que acta sobre la polea se encuentra la
u
ecuacin de movimiento para la rotacin
o
o

2 R 1 R = I 0 .

Tenemos cuatro ecuaciones para las cuatro incognitas 1 , 2 , a y 0 . Resolviendo


este sistema de ecuaciones se encuentra
2
a= g,
7
9
1 = mg
7
y
10
2 = mg .
7


CAP
ITULO 9. ROTACION DE UN CUERPO R
IGIDO

361

c) Mientras las masas estn cayendo, como la polea no se desplaza, la fuerza total
a
sobre ella debe ser nula. Por lo tanto, la tensin de la cuerda que sujeta la polea
o
debe ser
26
= 1 + 2 + mg =
mg .
7
d) Consideremos la situacin que se tiene cuando la masa 2m ha tocado el suelo
o
y todo est detenido. El torque total sobre la polea es nulo y, por lo tanto, la
a
tensin de la cuerda que pasa por la polea debe ser la misma a ambos lados,
o
siendo su valor mg. Conclu
mos que la tensin de la cuerda que sujeta la polea,
o
en este caso, es
= 2mg .

Solucin al problema 19
o
Slo la parte superior de la correa de transmisin transmite fuerza. Denotemos sta
o
o
e
por F . La ecuacin de movimiento para la rotacin de la polea #2 es
o
o

F R = I 2 .
Para la polea #1 sta es
e

= I 1 .
2
El diagrama de cuerpo libre para la masa M nos da la ecuacin
o
TR F

Mg T = Ma ,
donde a es la aceleracin (hacia abajo) de la masa M . Los ngulos de rotacin de
o
a
o
ambas poleas no son independientes sino que estn relacionados por
a
1

R
= 2 R .
2

Derivando dos veces respecto al tiempo y cancelando el radio R se obtiene

2 = 22 .

Finalmente, tambin la aceleracin a est relacionada con 1 . En efecto, a = R1 .


e
o
a

De las ecuaciones anteriores podemos despejar las cinco incgnitas a, T , F , 1 y 2 .


o
De esta manera se encuentra
8

a = R 1 =
g,
13
5
Mg
T =
13


CAP
ITULO 9. ROTACION DE UN CUERPO R
IGIDO
y
F =

362

2
Mg .
13

Despus de bajar una distancia h la energ potencial disminuye en U = M gh. Esta


e
a
energ debe transformarse en energ cintica.
a
a
e
Si la velocidad de la masa M es v, entonces las velocidades angulares de las poleas #1
son 1 = v/R y 2 = v/(2R), respectivamente. La energ cintica es, por lo tanto,
a
e
K =
=

1
M v2 +
2
1
M v2 +
2

1 2 1 2
I + I
2 1 2 2
1 1
13
2
2
M R2 (1 + 2 ) = M v 2 .
2 2
16

Igualando esto con la diferencia de energ potencial se encuentra para v la expresin


a
o
v=

16
gh .
13

Solucin al problema 21
o
El momento de inercia de la carretilla, para rotaciones alrededor de su eje de
simetr es
a,
1
1
I=2
m(2R)2 + (2m)R2 = 5mR2 .
(9.6)
2
2
El diagrama de cuerpo libre para la masa M nos da la ecuacin
o
M g T = M aM ,

(9.7)

donde T es la tensin de la cuerda y aM la aceleracin (hacia abajo) de la masa M .


o
o
Aplicando la segunda ley de Newton para el movimiento traslacional horizontal de la
carretilla se encuentra la ecuacin
o
T fr = (4m)aC ,

(9.8)

donde fr es el roce esttico entre la carretilla y la mesa y aC la aceleracin de la


a
o
carretilla. (Este roce es el responsable de hacer que la carretilla ruede sin resbalar).
Sea la aceleracin angular de la carretilla. El torque neto sobre la carretilla debe
o
ser igual al momento de inercia multiplicado por , o sea,
fr (2R) T R = I .

(9.9)

Por supuesto que hay una estrecha relacin entre y aC , que viene dada por el hecho
o
que la carretilla rueda sin resbalar. Esta relacin es
o
2R = aC .

(9.10)


CAP
ITULO 9. ROTACION DE UN CUERPO R
IGIDO

363

Tambin existe una relacin que vincula AM , aC y : La cuerda es inextensible y por


e
o
lo tanto la diferencia entre la aceleracin de la carretilla y la masa M debe coincidir
o
con la aceleracin con que la cuerda se enrolla, es decir,
o
aC aM = R .

(9.11)

Las ecuaciones (9.6), (9.7), (9.8), (9.9), (9.10) y (9.11) resuelven el problema.
Para la aceleracin de la carretilla se obtiene
o
ac = g

2M
.
M + 21m

Solucin al problema 22
o
Introduzcamos el sistema de coordenadas mostrado en la gura 9.23. (El eje y

apunta hacia el interior del papel). Las distintas fuerzas que actan sobre el automvil
u
o
durante el frenado son: Las fuerzas F1 = F1 z y F2 = F2 z que el pavimento ejerce

sobre las ruedas delanteras y traseras, respectivamente; el peso Fp = M g y la


z
fuerza de roce fr = +c F1 x .

Como el automvil no se eleva, la fuerza total en la direccin z debe ser nula, es


o
o
decir,
0 = (F1 + F2 M g) z .

(9.12)
Por otra parte, como el automvil durante el frenado tampoco gira (en torno al
o
eje y ), el torque total (respecto al centro de masas) debe ser nulo. Los torques que

ejercen las cuatro fuerzas son:


1 = +F1 d
y
2 = F2 b
y
p = 0
y
r = fr h = F1 c h .
y
y
La suma de estos torques debe ser nulo, condicin que nos entrega la relacin
o
o
F1 d F2 b F1 c h = 0 .
De las ecuaciones (9.12) y (9.13) se pueden despejar F1 y F2 , obtenindose
e
F1 = M g
y
F2 = M g

b
b + d c h
d c h
.
b + d c h

(9.13)


CAP
ITULO 9. ROTACION DE UN CUERPO R
IGIDO

364

Las ecuaciones anteriores dejan de ser vlidas si c h > d, ya que en ese caso, la
a
fuerza F2 se vuelve negativa, lo que signica que las ruedas traseras dejan de estar en
contacto con el suelo. En otras palabras, las ecuaciones anteriores son vlidas mientras
a
d > c h, relacin que favorece un diseo (del automvil) en que el centro de masas
o
n
o
se ubica en la parte trasera y cerca del suelo.
Conociendo F1 podemos calcular la fuerza de roce fr (que es la fuerza responsable de
la (des)aceleracin del automvil). Para la la aceleracin se obtiene
o
o
o
a=

fr
F1 c
bc
x=

x=g

x = a0 x .

M
M
b + d c h

Finalmente, la distancia D que recorre el automvil durante su frenado es


o
D=

2
v0
2 b + d c h
= v0
.
2a0
2gbc

Observe que en este problema la fuerza neta sobre el automvil no es nula y, por
o
lo tanto, el torque neto respecto a otro origen no es nulo aun cuando lo sea respecto
al centro de masas. Conrme la aseveracin anterior evaluando el torque respecto al
o
punto de contacto de la rueda delantera con el suelo.
Reiteramos: Para determinar si cambiar el estado rotacional de un cuerpo acea
lerado, debe evaluarse el torque total respecto al centro de masas (ver tambin
e
cap
tulo siguiente).
Analicemos ahora el proceso de aceleracin. Sea sta a = a0 x. Supongamos que
o
e

el motor ejerce la fuerza sobre las ruedas traseras y que estas no resbalan. En ese caso
la fuerza de roce (esttica) fr = fr x actuar sobre las ruedas traseras y en direccin
a

a
o
). Mientras el automvil est con las cuatro ruedas sobre el suelo, el torque total
x
o
a
respecto al centro de masas debe ser nulo, o sea,
F1 d F2 b + fr h = 0 .
La fuerza que acelera el automvil es la fuerza de roce, es decir,
o
fr = M a .
En la direccin vertical la suma de todas las fuerzas sobre el automvil debe ser nula:
o
o
F1 + F2 = M g .
Las tres ultimas ecuaciones permiten determinar F1 y F2 :

F1 = M

gb a0 h
,
b+d

F2 = M

gd + a0 h
.
b+d


CAP
ITULO 9. ROTACION DE UN CUERPO R
IGIDO

365

Observe que F1 es mayor que cero slo mientras a0 < gb/h. Para aceleraciones mayoo
res, las ruedas delanteras del automvil pierden contacto con el suelo. La aceleracin
o
o
maxima (si el roce esttico lo permite) viene dada por
a
b
amax = g .
h
Solucin al problema 29
o
El momento angular total debe conservarse. Inicialmente el momento angular es
L i = I1 0 =

1 2
mr
2

0 .

El momento angular una vez que el segundo disco este en reposo respecto al primero
es
Lf = I ,
donde I es el momento de inercia de la conguracin nal para rotaciones alrededor
o
del eje jo. Se tiene
1
I = I1 + I2 = M R 2 +
2

1 2
mr + mD 2
2

donde para I2 hemos usado el teorema de Steiner.


Igualando los dos momentos angulares se deduce
1 2
1
mr 0 =
M R2 +
2
2
o sea,
=

1 2
mr + mD 2
2

mr 2 0
.
mr 2 + M R2 + 2mD 2

Solucin al problema 35
o
En la gura 9.38 la rueda gira en el plano
(x, z), movindose la parte superior de la
e
rueda hacia el lector. El momento angular
debido al giro de la rueda alrededor de su
eje, por lo tanto, apunta en la direccin y .
o
Suponiendo que toda la masa de la rueda
est concentrada en la periferia, su moa
mento de inercia para rotaciones alrededor
de su eje ser
a
I0 = M R 2 .

Figura 9.38


CAP
ITULO 9. ROTACION DE UN CUERPO R
IGIDO

366

El momento angular asociado a este movimiento (para el instante mostrado en la


gura 9.38), respecto al origen O, ser
a
L 0 = I0 y ,

donde es la velocidad angular con que gira la rueda alrededor de su eje (esto es,
= 2 10 s1 ).
El torque total repecto a O es
= M gD x .

Pero, por otra parte,

dL
,
dt
luego el cambio de momento angular (para la situacin mostrada en la gura 9.38)
o
ser
a
=

dL
= M gD x .

dt
Observe que L y dL son perpendiculadt
res (ver tambin gura 9.39). Como L y
e
dL estn en el plano (x, y), el vector L
a
seguir en ese plano, pero cambiar su
a
a
orientacin. En otras palabras, el eje de
o
la rueda girar en el plano (x, y), mana
tenindose horizontal.
e
Figura 9.39
Para deducir la velocidad angular con la cual el eje de la rueda gira alrededor del
eje z , recordemos algunos aspectos del movimiento circular uniforme: Sea r un vector

de largo R que gira en el plano (x, y) con velocidad angular uniforme. En ese caso
v = dr/dt es siempre perpendicular a r. Si conocemos el radio de giro y la rapidez
con que gira podemos determinar la velocidad angular:
=

|dr/dt|
v
=
.
R
|r |

La situacin en el presente problema es anloga. Tenemos un vector L que gira en


o
a
el plano (x, y). Conocemos L = |L| y la rapidez |dL/dt|, luego podemos encontrar la
velocidad angular de precesin con la que gira L:
o
=

|dL/dt|
|L|

gD
M gD
= 2 .
2
MR
R

Tanto ms rpido gira la rueda, tanto ms lenta es la precesin.


a a
a
o

Cap
tulo 10

Fuerzas cticias
versin 20 agosto 2012
o

Desde que introdujimos el concepto de fuerzas, siempre hemos sido cuidadosos


en estudiar cualquier sistema f
sico en los llamados sistemas de referencia inerciales.
La razn es clara: un sistema de referencia inercial es aquel en el cual se cumplen las
o
tres leyes de Newton. En particular, en un sistema inercial los cuerpos aceleran slo
o
si sobre ellos acta una fuerza.
u
Sin embargo, ya observamos, en el comienzo del Cap. 4, que no hay en principio
ninguna razn para evitar describir el universo en un sistema de referencia no inercial.
o
En el Cap. 4 utilizamos el ejemplo de un observador en el interior de un automvil que
o
acelera. En este caso, un rbol en la acera parece acelerar sin que se apliquen fuerzas
a
sobre l, lo cual es inconsistente con las leyes de Newton. Lo mismo suceder en
e
a
cualquier sistema de referencia acelerado: vistos desde dicho sistema, todos los dems
a
cuerpos estarn sujetos a aceleraciones que no se deben a ninguna fuerza debida a
a
ningn otro cuerpo.
u
De hecho, nosotros mismos, d a d estamos en esa situacin: recordemos que
a
a,
o
estamos sobre la supercie del planeta Tierra, que gira en torno a su eje, y adems
a
gira en torno al Sol, el cual adems gira en torno al centro de masas de la V
a
a
Lctea. . . Nunca hemos vivido en un sistema de referencia inercial, y por lo tanto, en
a
rigor, no podemos aplicar las leyes de Newton para describir el Universo tal como lo
vemos nosotros.
Signica eso que nada de lo que hemos hecho hasta el momento tiene sentido?
No, en absoluto. Primero, porque la F
sica es una ciencia experimental, y si las aceleraciones son sucientemente pequeas, o si actan durante un tiempo sucientemente
n
u
pequeo, para que la no inercialidad de nuestro sistema de referencia no sea men
dible, entonces la aproximacin de que nos encontramos en un sistema de referencia
o
inercial es vlida. Segundo, porque es posible, incluso en sistemas de referencia no
a
inerciales, considerar que las leyes de Newton siguen siendo vlidas. El costo es ina

367

CAP
ITULO 10. FUERZAS FICTICIAS

368

troducir fuerzas cticias, responsables de las aceleraciones inexplicables que los


cuerpos tienen cuando son observados desde un sistema no inercial.
Esta segunda opcin es la que exploraremos en el presente cap
o
tulo. Las fuerzas
cticias, entonces, son fuerzas que deben incluirse en la descripcin de un sistema f
o
sico
cuando la observacin se realiza desde un sistema de referencia no inercial y, a pesar
o
de ello, se insiste en usar las leyes de Newton. Estudiaremos dos casos: un sistema de
referencia uniformemente acelerado (equivalente al caso del automvil mencionado
o
ms arriba) y un sistema de referencia jo a un cuerpo que rota uniformemente
a
(equivalente al caso de nosotros sobre la supercie terrestre). El caso general, de
un sistema de referencia sometido a una aceleracin arbitraria es inmanejable con las
o
herramientas de que disponemos hasta el momento, pero con los dos casos sencillos que
revisaremos podremos encontrar algunos resultados importantes y adquirir algunas
intuiciones bsicas.
a

10.1.

Referencial uniformemente acelerado

Sea S : (, y , z ) un sistema de referencia inercial y S : ( , y , z ) un sistema de


x
x
referencia que acelera con aceleracin constante a0 respecto a S. El vector que une
o
los or
genes O y O de ambos sistemas de referencia es
1
R(t) = R0 + V0 t + a0 t2 .
2
Sean r (t) y r (t) los vectores de posicin de una masa m en los sistemas de referencia
o
, respectivamente. La relacin entre r y r es
SyS
o
r = R+r .
Derivando dos veces respecto al tiempo se obtiene

r = R + r = a0 + r ,
o sea,

mr = mr ma0 .

(10.1)

Sea F la fuerza real neta que acta sobre la masa m, es decir, la fuerza que genera
u
de la masa m observada desde un sistema de referencia inercial. En
la aceleracin r
o
otras palabras

F = mr .
Si se insiste en usar la segunda ley de Newton, pero con las magnitudes observadas
desde un sistema de referencia acelerado, se tiene

F = mr ,

CAP
ITULO 10. FUERZAS FICTICIAS

369

pero la fuerza F ahora ya no es F sino que, de acuerdo con la ecuacin (10.1),


o
F = F ma0 .
El trmino ma0 = Fct es la fuerza cticia que hay que agregar a la fuerza real F
e
para poder seguir usando la segunda ley de Newton desde un sistema acelerado con
aceleracin a.
o
Observe que esta fuerza cticia acta como un campo gravitacional constante (adiu
cional al campo gravitacional g que pudiese estar presente).
Ejemplo: Consideremos un pndulo sobre un
e
carro que acelera con aceleracin constante
o
a = a0 x (ver gura 10.1). Encontremos el

ngulo entre la normal y la posicin de


a
o
equilibrio del pndulo.
e
Resolveremos el problema de dos maneras: i)
usando primero el sistema de referencia inercial del observador O y ii) el sistema de referencia acelerado jo a O .

Figura 10.1

i) En el sistema de referencia inercial el diagrama de cuerpo libre de la masa m se


muestra en la gura 10.2a. La fuerza neta que acta sobre la masa m es
u
F = + Fg = cos z + sin x mg z .

En el sistema de referencia inercial la part


cula acelera con una aceleracin a = a x,
o

luego
F = cos z + sin x mg z = ma .

x
Igualando las componentes de esta ecuacin vectorial se obtiene
o
cos = mg
y
sin = ma .
Dividiendo la segunda ecuacin por la primera se deduce nalmente que
o
tan =

a
.
g

CAP
ITULO 10. FUERZAS FICTICIAS

370

Figura 10.2a

Figura 10.2b

ii) Para un observador sobre el carro la masa m no se mueve. Por eso, para O la
fuerza neta sobre la masa m debe ser nula. El diagrama de cuerpo libre en este caso
se muestra en la gura 10.2b. Adems de la fuerza ejercida por la tensin del hilo y
a
o
de la gravedad, debemos agregar la fuerza cticia Fct = ma x. Tenemos

F = + Fg + Fct = 0 ,

o sea
0 = cos z + sin x mg z ma x .

Nuevamente, igualando las componentes de esta ecuacin vectorial se deduce que


o
cos = mg
y
sin = ma ,
o sea, las mismas relaciones encontradas en la parte i).
Para el observador O sobre el carro, tambin podr
e
amos haber simplemente considerado un campo gravitacional efectivo (ver gura 10.3).
ge = g a = g z a x .

Es evidente que el ngulo que ge hace con la


a
normal cumple con la relacin tan = a/g .
o
Si el pndulo realiza pequeas oscilaciones en
e
n
torno a su posicin de equilibrio la frecuencia
o
angular de las oscilaciones ser
a
=
donde es el largo del pndulo.
e

ge
=

Figura 10.3
g 2 + a2
,

CAP
ITULO 10. FUERZAS FICTICIAS

10.2.

371

Referencial en rotacin uniforme


o

Sea S : (, y , z ) un sistema de referencia


x
inercial y S : ( , y , z ) un sistema de refex
rencia que coincide con el sistema S en cierto
instante (por ejemplo, en t = 0), pero que rota con velocidad angular = z constante

en torno al eje z (ver gura 10.4).

Sea A un vector con componentes Ax , Ay , Az


en el sistema de referencia S y componentes
A , A , A en el sistema de referencia S , o
z
y
x
sea,
A(t) = Ax (t) x + Ay (t) y + Az (t) z .

y
A(t) = A (t) x + A (t) y + A (t) z .

x
y
z

Figura 10.4

Los vectores unitarios del sistema de referencia inercial x, y , z son jos, sin embargo,

los vectores unitarios del sistema de referencia rotatorio x , y , z rotan, tenindose

e
x = cos(t) x + sin(t) y

y = sin(t) x + cos(t) y

z = z .

Derivando estos vectores respecto al tiempo se encuentra

x = sin(t) x + cos(t) y = y

y = cos(t) x sin(t) y =

z = 0 .

Evaluemos la derivada del vector A en ambos sistemas de referencia. Por una parte
se tiene




A = Ax x + Ay y + Az z ,
y por otra parte

z
y
x
A = A x + A x + A y + A y + A z + A z
y
z
x

x
y
z
= (A x + A y + A z ) + (A y A x ) .
x
y

CAP
ITULO 10. FUERZAS FICTICIAS

372

Usando las relaciones anteriores y el hecho que


A = z A (t) x + A (t) y + A (t) z = A y A x ,

x
y
z
x
y
podemos escribir






Ax x + Ay y + Az z = A x + Ay y + A z + A ,
x
z
o sea
dA
dt

dA
dt

=
S

+A.

(10.2)

En la ultima expresin los ejes unitarios no aparecen expl

o
citamente, por consiguiente,
es una expresin que tiene una validez general (es decir, no slo para rotaciones en
o
o
torno al eje z ). La ecuacin (10.2) relaciona la derivada temporal de cualquier vector

o
en el sistema de referencia inercial S con la derivada temporal de ese mismo vector,
pero observada desde un sistema de referencia S que rota con velocidad angular
respecto a S.
Apliquemos la ecuacin (10.2) al vector posicin r de una part
o
o
cula de masa m.
Se tiene
dr
dr
+r ,
=
dt S
dt S
o sea,
vS = vS + r .
Ac vS es la velocidad de la part
a
cula m observada por el observador inercial S y vS
es la velocidad de la misma part
cula, pero observada desde el sistema de referencia
rotatorio.
Apliquemos nuevamente la ecuacin (10.2), pero ahora al vector vS . Se tiene
o
dvS
dt

=
S

dvS
dt

+ vS .

(10.3)

Usando la ecuacin (10.3), se obtiene


o
dvS
dt

=
S

=
=

d(vS + r )
dt
dvS
dt

dvS
dt

+ (vS + r)

d( r )
dt

+ vS + ( r)

+ 2 vS + r

El lado izquierdo de la ultima ecuacin es la aceleracin de la part

o
o
cula observada
por el observador inercial S, denotmosla por aS . El primer trmino al lado derecho
e
e

CAP
ITULO 10. FUERZAS FICTICIAS

373

es la aceleracin de la misma part


o
cula pero observada desde el sistema de referencia
, denotmosla por a . De esta manera obtenemos
rotacional S
e
S
maS = maS 2m vS m r

(10.4)

Sea F la fuerza real neta que acta sobre la masa m, es decir, la fuerza que genera
u
la aceleracin aS de la masa m observada desde un sistema de referencia inercial. En
o
otras palabras
F = maS .
Si se insiste en usar la segunda ley de Newton, pero con las magnitudes observadas
desde un sistema de referencia acelerado, se tiene
F = maS ,
pero la fuerza F ahora ya no es F sino que, de acuerdo con la ecuacin (10.4),
o
F = F 2m vS m r

Los trminos
e
Fct = 2m vS m r

(10.5)

son la fuerza cticia que hay que agregar a la fuerza real F para poder seguir usando
la segunda ley de Newton desde un sistema de referencia que rota respecto a un
sistema de referencia inercial con velocidad angular .
El primer trmino de la fuerza cticia dada por la ecuacin (10.5) es la as llamada
e
o

fuerza de Coriolis
FCoriolis = 2m vS
mientras el segundo trmino se llama fuerza centrfuga
e

Fcent = m r

Lo interesante de la fuerza de Coriolis es que ella slo aparece si, en el sistema


o
, la masa se mueve, y en ese caso, es perpendicular a la
de referencia rotacional S
direccin de movimiento. Cuando m est en reposo (es decir, vS = 0) entonces la
o
a
unica fuerza cticia que hay que agregar a la fuerza que se observa en un sistema

inercial, es la fuerza centr


fuga.
Cuando realizamos experimentos en la tierra (laboratorio) siempre asumimos
que un sistema jo al laboratorio representa un sistema de referencia inercial. Sin
embargo, la rotacin de la tierra en torno a su propio eje (con una frecuencia =
o
2/(24 3600) = 7, 27 105 s1 hace que el sistema de referencia no sea inercial y
que, en la prctica, debamos en ocasiones agregar la fuerza cticia (10.5) para obtener
a
una descripcin correcta del sistema. La fuerza de Coriolis es responsable de muchos
o

CAP
ITULO 10. FUERZAS FICTICIAS

374

efectos (a veces sorprendentes) que se observan a nuestro alrededor. Por ejemplo, es


la responsable de la rotacin de los ciclones y de las corrientes marinas o del giro del
o
plano de oscilacin de un pndulo.
o
e
Problema resuelto en clases: 10.1
Problema resuelto en clases: 10.3
Problema resuelto en clases: 10.7

CAP
ITULO 10. FUERZAS FICTICIAS

10.3.

375

Problemas

1. Demuestre que la fuerza centr


fuga que
acta sobre una masa m (si sta es obu
e
servada desde un sistema de referencia rotacional que gira con velocidad angular
respecto a un sistema de referencia inercial) viene dada por
2
Fcentr

fuga = m ,

donde es la distancia entre el eje de rotacin y la masa m y es un vector unitario


o

que apunta el eje hacia la masa m y es perpendicular al eje de giro (ver gura 10.5).
Observe que la fuerza centr
fuga tiene la
misma magnitud y direccin de la fuerza
o
centr
peta slo que apunta en el sentido
o
opuesto.

Figura 10.5

2. En un parque de diversiones, los participantes se sostienen contra la pared de


un cilindro giratorio mientras el suelo se hunde. El radio del cilindro es R = 3m
y el coeciente de roce entre las personas y la pared del cilindro es e = 0, 4.
Determine el nmero m
u
nimo de revoluciones por minuto que se requiere para
que el juego funcione.
Haga el clculo de dos maneras distintas: i) usando un sistema de referencia
a
inercial y ii) usando un sistema de referencia solidario al cilindro.
3. Considere el efecto de la rotacin terrestre sobre el movimiento de un proyectil
o
que se lanza desde la supercie terrestre con velocidad v0 . Suponga que el alcance del proyectil es tal que en todo instante se mueve en un campo gravitacional
constante, es decir, Fg = mg.
a) Demuestre que la velocidad del proyectil viene dada por
v = v0 + g t 2 r .
Todas las magnitudes estn medidas respecto a un observador solidario
a
con la Tierra. Ac r es el vector posicin del proyectil medido desde el
a
o
punto de lanzamiento y es el vector velocidad angular de la Tierra.
Al resolver el problema no se debe incluir la fuerza centr
fuga ya que sta
e
est incluida en el valor local de g que se est usando. Al rotar la Tierra
a
a
no slo se modica la magnitud g sino que tambin su direccin. La fuerza
o
e
o
centr
fuga incluso modica la forma de la Tierra; de hecho, la normal a la
supercie terrestre usualmente no pasa por el centro de la Tierra.

CAP
ITULO 10. FUERZAS FICTICIAS

376

b) Demuestre que, al despreciar trminos del orden 2 , para la aceleracin se


e
o
obtiene la ecuacin
o
a = g 2 g t 2 v0 .
(Nuevamente todas las magnitudes medidas desde un sistema de referencia
solidario a la Tierra). Integre la ultima ecuacin y demuestre que

o
r(t) = v0 t +

1 2 1
g t g t 3 v0 t 2 .
2
3

4. Desde un edicio de altura h = 100 m situado en el Ecuador terrestre, se suelta


una piedra. Debido a la rotacin terrestre,
o
la piedra no caer a lo largo de la normal
a
sino que se desviar levemente de ella. Una
a
vez que llega al suelo, encuentre la magnitud y direccin de la desviacin. Desprecie
o
o
efectos debido al roce viscoso con el aire.
Indicacin: use el resultado obtenido en el
o
problema anterior.
Respuesta: La desviacin es hacia el este
o
y es de magnitud
2
h
3

Figura 10.6

2h
2, 19 cm .
g

5. Desde el Ecuador se lanza un proyectil con velocidad v0 = 500 m/s en la direccin esteoeste, con un ngulo de elevacin = 10 . Encuentre como cambia el
o
a
o
tiempo que el proyectil tarda en volver a chocar con la tierra y el alcance del
proyectil debido a la rotacin terrestre. Para resolver este problema no hay que
o
incluir la fuerza centr
fuga ya que el efecto de ella ya se incluy en el vector la
o
aceleracin de gravedad g, que supondremos constante en magnitud y direccin
o
o
sobre toda la trayectoria. Ignore cualquier efecto debido al roce con el aire y
desprecie correcciones del orden 2 .
Respuesta: El alcance disminuye en
D =

3
4v0
sin
g2

1
sin cos2
3

62, 9 m .

Qu pasa si en lugar de dispararlo de este a oeste se dispara de oeste a este o


e
de sur a norte?

CAP
ITULO 10. FUERZAS FICTICIAS

377

6. Considere un canal de ancho a ubicado sobre la tierra a una latitud > 0,


en direccin paralela al Ecuador. Por el canal uye agua con una velocidad v0 .
o
Demuestre que, para un observador mirando en la misma direccin del ujo
o
del canal, el nivel del agua al lado derecho del canal es superior a la del lado
izquierdo en una cantidad
h =

2av0 sin
,
g

donde es la velocidad angular de la tierra.


7. Un balde con agua gira en torno a su eje
de simetr con velocidad angular . Dea
bido a la rotacin, la supercie del agua
o
no ser plana. Encuentre su forma.
a
Figura 10.7
8. Considere un pndulo cnico de largo , cuya masa gira en el plano horizontal
e
o
en un c
rculo de radio R. Si se ignora la fuerza de Coriolis, la frecuencia angular
del pndulo cnico es 0 = g/, siendo esta independiente del sentido del giro.
e
o
Demuestre que al incluir la fuerza de Coriolis, las frecuencias en ambos sentidos
ya no son iguales, tenindose
e
1 =

g
+ ( sin )2 sin

y
2 =

g
+ ( sin )2 sin ,

donde es la latitud del lugar en que se encuentra el pndulo.


e
9. (Pndulo de Foucault)
e
Al superponer (sumar) las dos soluciones de un pndulo cnico correspondientes
e
o
al mismo radio de giro, pero rotando en sentidos opuestos, se obtiene la solucin
o
de un pndulo simple.
e
a) Demuestre lo anterior en forma expl
cita para un pndulo cnico ignorando
e
o
la fuerza de Coriolis.
b) Al realizar el mismo clculo, pero ahora incluyendo el efecto de Coriolis
a
(ver problema anterior), se encuentra que debido a la rotacin terrestre, el
o
plano de oscilacin del pndulo simple no se mantendr invariante sino que
o
e
a
girar paulatinamente. Demuestre que la velocidad angular con que gira el
a

CAP
ITULO 10. FUERZAS FICTICIAS

378

plano de oscilacin del pndulo viene dado por F = sin , donde es


o
e
la velocidad angular de la tierra en torno a su propio eje y es la latitud
del lugar en que se encuentra el pndulo.
e
Foucault fue el primero en demostrar experimentalmente, con un pndulo
e
muy largo, que el plazo de oscilacin efectivamente gira a medida que
o
transcurre el tiempo. Observe que en el Ecuador el plano de oscilacin no
o
gira, mientras que en los polos da una vuelta completa en 24 horas (despus
e
de 6 horas el plano de oscilacin habr girado en 90 ).
o
a
10. Considere una cua de masa M y ngun
a
lo de elevacin que puede deslizarse
o
sobre un plano horizontal sin roce. Sobre el plano inclinado se encuentra otra
masa m, que a su vez tambin puede
e
deslizarse sobre el plano sin roce. Encuentre la aceleracin del plano inclinao
do M .

Figura 10.8

11. Un nio se encuentra en una ciudad ubicada en una latitud . El nio deja caer
n
n
una pelota desde un punto A, que est a una altura h del suelo. Al llegar al
a
suelo, la pelota rebota, subiendo hasta alcanzar una altura mxima en un punto
a
B.
Encuentre la posicin del punto B, respecto a un sistema de referencia (, y , z ),
o
x
solidario a la supercie terrestre, donde z es perpendicular al suelo, alejndose

a
del centro de la Tierra, y x es paralelo al suelo, apuntando hacia el norte. z = 0

corresponde al nivel del suelo.

Hint 1: Desprecie trminos de orden 2 , donde es la frecuencia angular de


e
rotacin de la Tierra.
o
Hint 2: Utilice la expresin que se encontr en el problema 10.3:
o
o
1
1
r(t) = r0 + v0 t + gt2 ( g)t3 ( v0 )t2 ,
2
3

CAP
ITULO 10. FUERZAS FICTICIAS

379

donde r0 y v0 son la posicin y la velocidad inicial de un proyectil medidas en


o
el sistema de referencia solidario con la supercie terrestre.
12. Una mosca se dirige, segn ella, radialmente hacia el centro de un tornamesa.
u
Una hormiga, en reposo sobre el tornamesa, opina que en realidad la mosca se
dirige al centro describiendo una espiral.
A partir de las expresiones para la fuerza en un sistema de referencia en rotacin uniforme muestre que la hormiga tambin tiene razn. (Desprecie trminos
o
e
o
e
de orden 2 , y suponga que el movimiento de la mosca no tiene componente
vertical.)
Hint: encuentre primero la ecuacin diferencial que satisfacen las componentes
o
de la velocidad de la mosca, vx y vy .
13. Sobre un disco horizontal se coloca un pndulo de largo l y masa m, a una
e
distancia R del centro (ver gura).

l
m

^
z

Si el disco gira respecto a un eje vertical que pasa por su centro, con velocidad
angular = , encuentre el ngulo 0 que forma el pndulo con la vertical.
z
a
e
Suponga que 0 es pequeo, de modo que basta con mantener trminos hasta
n
e
2
orden 0 .
Suponiendo ahora que el pndulo forma un ngulo con la vertical igual a =
e
a
0 + , con 1, encuentre la ecuacin diferencial para .
o

10.4.

Solucin a algunos de los problemas


o

Solucin al problema 3
o
Como la tierra est girando en torno a su propio eje, la fuerza efectiva que acta sobre
a
u
el proyectil ser
a
Fe = m g 2 m v .

CAP
ITULO 10. FUERZAS FICTICIAS

380

(La fuerza centr


fuga Fcentrifuga = m ( r) no debe incluirse ya que su
efecto est considerado en el valor local de la aceleracin de gravedad g). Todas las
a
o
magnitudes en la ecuacin anterior se reeren a variables medidas en un sistema jo a
o
la Tierra, es decir, respecto a un sistema de referencia que rota con velocidad angular
respecto a un sistema de referencia inercial. ( es la velocidad angular de la rotacin
o
de la Tierra alrededor de su propio eje.)
Al incluir la fuerza efectiva podemos seguir usando la segunda ley de Newton. Se
tiene
dv
dr
Fe = m
= mg 2m
,
dt
dt
o sea,
m dv = m g dt 2 m dr .
Integrando (sumando) desde un instante inicial a uno nal se obtiene
f

dv =
i

g dt 2

dr .
i

Sea ti = 0, tf = t, vi = v0 , vf = v, ri = 0 y rf = r. Entonces, evaluando las integrales


de la ultima ecuacin se obtiene

o
v v0 = g t 2 r .

(10.6)

Derivando esta ecuacin respecto al tiempo se encuentra la aceleracin


o
o
a = g 2 v .
Reemplacemos v en esta ecuacin por la expresin dada por (10.6), entonces
o
o
a = g 2 ( v0 + g t 2 r )
= g 2 v0 2

t + O(2 )

Integrando estas expresiones respecto al tiempo se encuentra la velocidad y luego la


posicin en funcin de t:
o
o
a dt =
i

g dt 2 v0

dt 2 g

t dt
i

v v0 = g t 2 v0 t g t 2
v0 dt +

v dt =
i

f
i

r (t) = v0 t +

g t dt 2 v0

t dt g

1 2
1
g t ( v0 ) t2 ( g ) t3 .
2
3

t2 dt

(10.7)

CAP
ITULO 10. FUERZAS FICTICIAS

381

Solucin al problema 5
o
Denamos los ejes del sistema referencia solidario con la Tierra tal como se muestra en la
gura adjunta. El proyectil se dispara desde
P en la direccin esteoeste con un ngulo
o
a
de elevacin , luego la velocidad inicial del
o
proyectil viene dada por
v0 = v0 cos y + v0 sin x .

Los vectores correspondientes a la aceleracin


o
de gravedad (local) y la velocidad angular de
la Tierra vienen dados por
g = g g

Figura 10.9

y
=z.

Reemplazando estas relaciones en (10.7) se obtiene la posicin del proyectil a medida


o
que transcurre el tiempo (medido desde el lugar de lanzamiento):
1
1

r(t) = v0 t cos y + v0 t sin x gt2 x + g t3 y v0 t2 cos x v0 t2 sin y .

2
3
Sea t el instante en que el proyectil vuelve a caer sobre la Tierra. En ese instante se
tiene que
r (t ) = D y ,

donde D es el alcance del proyectil. Evaluando r (t) en el instante t e igualando el


resultado con la expresin anterior, se puede despejar t y D obtenindose
o
e
t =

2 v0 sin
2v0

sin
g + 2 v0 sin
g

2v0
g

sin cos + O(2 ) .

(Estamos despreciando todas las correcciones del orden 2 ). Para D se obtiene la


expresin
o
1
D = vo t cos t3 g + v0 t2 sin .
3
Sustituyendo en esta ecuacin la expresin para t se encuentra (despreciando nueo
o
vamente todos los trminos de orden O(2 )) que
e
D=

2
2v0
v3
sin cos + 4 0 2 sin
g
g

1
sin2 cos2
3

CAP
ITULO 10. FUERZAS FICTICIAS

382

El primer trmino del lado derecho de la ultima ecuacin es el alcance del proyectil
e

o
si se ignora la fuerza de Coriolis; el segundo trmino es la correccin (a primer orden
e
o
en ), que sufre el alcance debido a la rotacin terrestre.
o
Solucin al problema 9
o
a) La posicin en funcin del tiempo de un
o
o
pndulo cnico de largo que recorre un
e
o
c
rculo de radio R viene dada por
r(t) = R cos(t) x + R sin(t) y ,

o
con = g/. Esta solucin corresponde a
un pndulo cnico que gira en la direccin
e
o
o
contraria al reloj. Una solucin que gira en el
o
mismo sentido que el reloj viene dada por
r2 (t) = R cos(t) x + R sin(t) y .

Figura 10.10
Al ignorar la rotacin de la Tierra (es decir, al despreciar la fuerza de Coriolis) las
o
frecuencias angulares para ambos sentidos es la misma.
Al sumar las dos soluciones se obtiene la proyeccin sobre el plano x-y de la
o
solucin correspondiente a un pndulo lineal. En efecto:
o
e
(t) = r1 (t) + r2 (t) = 2R cos(t) x .

El lado derecho corresponde al movimiento de un oscilador a lo largo del eje x con

amplitud 2R (esto es, la proyeccin de la posicin del pndulo sobre el plano horizono
o
e
tal).

CAP
ITULO 10. FUERZAS FICTICIAS

383

b) Al incluir el efecto de la fuerza de Coriolis,


los vectores posicin (en el plano x-y) de los
o
pndulos cnicos, a medida que transcurre el
e
o
tiempo, vienen dados por
r1 (t) = R cos(1 t) x + R sin(1 t) y ,

y
r2 (t) = R cos(2 t) x + R sin(2 t) y ,

con
1 =

g
+ ( sin )2 sin

y
2 =

g
+ ( sin )2 sin ,

Figura 10.11

donde es la latitud del lugar en que se encuentra el pndulo (ver problema 10.8). Al
e
sumar las dos soluciones y usar las relaciones
1
(1 2 )t
cos(1 t) + cos(2 t) = 2 cos
2
y
1
sin(1 t) + sin(2 t) = 2 cos
(1 2 )t
2

cos

1
(1 + 2 )t
2

sin

1
(1 + 2 )t
2

se encuentra

(t) = r1 (t) + r2 (t) = 2R cos

2 sin2 +

g
t [cos(t sin ) x sin(t sin ) y ] .

La expresin delante del parntesis cuadrado corresponde a un movimiento oscilatoo


e
e
e
rio de amplitud 2R y con esencialmente la frecuencia g/. El trmino en parntesis
cuadrado es un vector unitario que indica la direccin de oscilacin. Observe, sin emo
o
bargo, que ese vector unitario rota lentamente en el plano x-y a medida que transcurre
el tiempo. O sea, la direccin de oscilacin de este oscilador (el plano de oscilacin
o
o
o
del pndulo) rotar a medida que avanza el tiempo, siendo la velocidad angular de
e
a
rotacin de este movimiento
o
F = sin .

CAP
ITULO 10. FUERZAS FICTICIAS

384

Solucin al problema 10
o
Resolvamos el problema usando un sistema solidario a la cua. Sea a = a la
n
x
aceleracin de la cua. Al gracar los diagramas de cuerpo libre en el sistema de
o
n
referencia acelerado, debemos agregar las fuerzas cticias. Los diagramas de cuerpo
libre de la masa m y de la cua se muestran en la gura siguiente.
n

Figura 10.12a

Figura 10.12b

En el sistema solidario a la cua, la aceleracin de la masa m es am = ax x a2 z ,


n
o

donde
az
= tan ,
(10.8)
ax
y la aceleracin de la cua M es nula. De los diagramas de cuerpo libre se deducen
o
n
las ecuaciones de movimiento
mg z + FN cos z + FN sin x + ma x = m am

FN sin x + M a x M g z + FN z = 0 .

De la segunda ecuacin se encuentra que FN = M a/ sin , y de la primera se obtienen


o
las componentes az y ax de la aceleracin de m:
o
az = mg FN cos
ax = ma + FN sin .
Usando estas relaciones y (10.8), se encuentra
tan =

mg FN cos
,
ma + FN sin

CAP
ITULO 10. FUERZAS FICTICIAS
de donde, nalmente,
a=

mg
.
M cotg + (m + M ) tan

385

Cap
tulo 11

Gravitacin
o
versin 20 agosto 2012
o

Las contribuciones de Isaac Newton a la F


sica y la Matemtica son numerosas,
a
y en los cap
tulos anteriores y los que siguen exploramos slo algunas de ellas. La
o
sistematizacin de conocimientos previamente adquiridos gracias a cuidadosas obo
servaciones, la formalizacin a travs de las herramientas matemticas adecuadas,
o
e
a
muchas de ellas desarrolladas por el propio Newton, como el Clculo Diferencial, y la
a
extraccin de nuevas consecuencias y predicciones a partir de dicha formalizacin, son
o
o
sin duda logros mayores. Sin embargo, de todas estas contribuciones, probablemente
la que es mayormente reconocida fuera de los c
rculos cient
cos es la Teor de la
a
Gravitacin Universal. Gracias a ella, Newton nos entreg un modelo preciso para
o
o
explicar una variedad abismante de fenmenos. Fenmenos que durante siglos fueron
o
o
considerados completamente desconectados (la ca de una manzana, las mareas, el
da
movimiento de los planetas), o para los cuales no se ten un modelo que los explicaa
ra (las rbitas el
o
pticas de los planetas, la aceleracin constante con la que caen los
o
cuerpos cerca de la supercie terrestre), y otros que ni siquiera se sospechaban (el
retorno de los cometas, la existencia de planetas ms all de Saturno), todos no eran
a
a
sino consecuencia de una sola y simple ecuacin para la fuerza entre dos masas.
o
La Teor de la Gravitacin Universal de Newton es una de las teor ms ima
o
as a
portantes con las que contamos, y su inmensa fama es, sin duda, merecida. En este
cap
tulo estudiaremos algunos aspectos de la misma. Comenzaremos con un complemento matemtico sobre elipses, ya que una de las consecuencias de esta teor es que
a
a
los planetas describen, en torno al Sol, rbitas el
o
pticas (o, en general, cnicas: elipses,
o
parbolas o hiprbolas). Con estos elementos matemticos a mano seremos capaces,
a
e
a
en las pginas siguientes, de reconocer rpidamente la prediccin de que las rbitas
a
a
o
o
debidas a la atraccin gravitacional son siempre cnicas, y podremos caracterizar los
o
o
parmetros geomtricos de las rbitas en trminos de cantidades f
a
e
o
e
sicas (energ moa,
mento angular). Observaremos tambin que es posible describir el complejo problema
e
de la trayectoria de dos cuerpos masivos sujetos a su mutua atraccin gravitacional
o
386


CAP
ITULO 11. GRAVITACION

387

como un problema de un solo cuerpo orbitando en torno a un cuerpo en reposo, y


adems como un problema unidimensional, lo que permitir realizar muchos anlia
a
a
sis de manera muy sencilla. Finalmente, deniremos el campo gravitatorio, nuestro
primer acercamiento al concepto de campo, un concepto fundamental, ampliamente
utilizado en muchas reas de la F
a
sica.

11.1.

Elipses

Como indicamos en la introduccin, una de las predicciones importantes de la


o
teor de la gravitacin es que las rbitas de los planetas en torno al Sol sern cnicas
a
o
o
a o
(elipses, parbolas o hiprbolas). Por ello, comenzaremos este cap
a
e
tulo recordando
algunos resultados matemticos relevantes para estas curvas.
a
Consideremos dos puntos, f1 y f2 ubicados en un plano. Consideremos adicionalmente un tercer punto P (en el mismo plano), y denotemos por r y r a las
distancias de este punto y f1 y f2 , respectivamente. Por denicin, una elipse es el
o
lugar geomtrico de todos los puntos del plano para los cuales r + r = 2a, en que 2a
e
es una constante (mayor que la separacin entre f1 y f2 ).
o
Introduzcamos un sistema de coordenadas cartesiano, con el origen en
el centro de la gura geomtrica y el
e
eje x a lo largo de la recta que une

los dos focos f1 y f2 . Es claro que el


semieje mayor de la elipse es a. Sea
2c la distancia entre los dos focos,
entonces
semieje menor = b =

a2 c2 .
Figura 11.1

Elipse en coordenadas cartesianas


De la gura 11.1 se deduce que
2a = r + r =
=

(c + x)2 + y 2 +

(c2 + x2 + y 2 ) + 2cx +

(c x)2 + y 2

(c2 + x2 + y 2 ) 2cx .

Elevando al cuadrado se obtiene


2(c2 + x2 + y 2 ) + 2 (c2 + x2 + y 2 ) + 2cx

(c2 + x2 + y 2 ) 2cx = 4a2 ,


CAP
ITULO 11. GRAVITACION

388

o sea,
2a2 (c2 + x2 + y 2 ) =
=

(c2 + x2 + y 2 ) + 2cx

(c2 + x2 + y 2 ) 2cx

(c2 + x2 + y 2 )2 4c2 x2 .

Elevando nuevamente al cuadrado queda


4a4 4a2 (c2 + x2 + y 2 ) + (c2 + x2 + y 2 )2 = (c2 + x2 + y 2 )2 4c2 x2 ,
o sea,
c2 x2 = a4 a2 c2 a2 x2 a2 y 2 .

Usando la relacin c2 = a2 b2 se deduce nalmente


o
x2 y 2
+ 2 =1.
a2
b

(11.1)

Elipse en coordenadas polares


Otra representacin util y comn de la elipse es en trminos de coordenadas
o
u
e
polares (r, ), tomando como origen a uno de los focos (ver gura 11.2).
1
Se tiene que
x = r cos c ,
y = r sin
y
y 2 = r 2 (1 cos2 ) .
Sustituyendo estas expresiones en la
ecuacin de la elipse en coordenao
das cartesianas dada por la ecuacin
o
(11.1), se obtiene
Figura 11.2
b2 (r cos c)2 + a2 r 2 (1 cos2 ) = a2 b2 ,

o sea,

b2 r 2 cos2 2b2 rc cos + b2 c2 + a2 r 2 a2 r 2 cos2 = a2 b2 ,


a2 r 2 = (a2 c2 ) b2 + 2b2 cr cos + (a2 b2 ) r 2 cos2
= b4 + 2b2
=

b2 +

a2 b2 r cos + (a2 b2 ) r 2 cos2

a2 b2 r cos


CAP
ITULO 11. GRAVITACION

389

Extrayendo la ra se deduce que


z,
ar = b2 +

a2 b2 r cos .

Introduciendo los parmetros


a
r0

b2
a

(11.2)

(11.3)

y
b2
c
= 1 2 .
a
a
la ecuacin (11.2) se puede escribir de la forma
o

1
1
=
(1 cos ) .
r
r0

(11.4)

(11.5)

Al parmetro se le denomina excentricidad de la elipse. Observemos que siempre


a
se tiene que < 1. Un caso particular es la circunferencia, en cuyo caso b = a, es
decir, la circunferencia tiene excentricidad nula.
Resumen: Las dos formas ms usuales para representar una elipse son:
a
i) Coordenadas cartesianas x, y (con el origen al centro):
x2 y 2
+ 2 =1,
a2
b

(11.6)

donde los parmetros a y b representan a los semiejes mayor y menor, respeca


tivamente.
ii) Coordenadas polares r, (con el origen en uno de los focos):
1
1
=
(1 cos ) .
r
r0

(11.7)

Las ecuaciones (11.3) y (11.4) relacionan los parmetros a, b con r0 , .


a
Si colocamos ahora el origen en uno de los focos, se denomina perigeo a la distancia m
nima entre la elipse y dicho foco, y apogeo a la distancia mxima (no deben
a
confundirse con los semiejes menor y mayor de la elipse!). Denominemos r1 y r2 al
perigeo y al apogeo, respectivamente. Entonces podemos encontrar relaciones entre
los semiejes menor, mayor, y la excentricidad, por un lado, con el apogeo y el perigeo,
por otro. En efecto:
r1 = a c
y
r2 = a + c ,


CAP
ITULO 11. GRAVITACION

390

de donde se deduce que


r1 + r2 = 2 a
y
r1 r2 = a2 c2 = b2 .
En otras palabras, el semieje mayor es el promedio aritmtico
e
r1 + r2
2

a=

y el semieje menor el promedio geomtrico


e
b=

r1 r2

del apogeo y perigeo.


La excentricidad, en trminos de r1 y r2 , es
e
=

r2 r1
.
r1 + r2

Ejercicio: Conociendo el rea de un c


a
rculo y las ecuaciones en coordenadas cartesianas de un c
rculo y de una elipse, demuestre que el rea A de una elipse viene dada
a
por
A = ab ,
donde a y b son sus semiejes.
Una propiedad muy interesante de las elipses es que si desde uno de sus focos
emerge un rayo de luz, y ste se reeja especularmente en la elipse, entonces el rayo
e
pasa necesariamente por el otro foco.
Para demostrar esta propiedad consideremos dos puntos P y P sobre la elipse,

innitesimalmente cercanos. Sean R1 y R1 los rayos que unen los puntos P y P con el

foco f1 y R2 y R2 los rayos que unen los puntos P y P con el foco f2 (ver gura 11.3).

a
Como R1 + R2 = R1 + R2 = 2a, se deduce que P A = P B. Los tringulos P AP y
BP son equivalentes, luego los ngulos y son iguales. Pero = , luego = .
P
a

Esta ultima relacin implica que los rayos R1 y R1 corresponden a rayos reejados

o
en forma especular por la elipse.


CAP
ITULO 11. GRAVITACION

391

Figura 11.3
Ejercicio: Considere la funcin siguiente (en coordenadas polares):
o
1
= 1 cos .
r
Graque r() para = 0, 0.5, 1.0 y 1.5.

11.2.

Las leyes de Kepler

Basndose en precisas mediciones de la posicin de los planetas realizadas por


a
o
Tycho Brahe, Johannes Kepler (15711630) pudo establecer las siguientes leyes para
describir el movimiento de los planetas alrededor del Sol:
1. Los planetas se mueven alrededor del Sol en rbitas el
o
pticas, estando el Sol en
uno de los focos.
2. Cada planeta barre reas iguales en tiempos iguales.
a
3. El cuadrado del per
odo de un planeta alrededor del Sol es proporcional al cubo
del semieje major de su trayectoria.
Estas leyes emp
ricas se conocen como Leyes de Kepler .
A continuacin mostraremos cmo la ley de gravitacin universal de Newton se
o
o
o
puede deducir a partir de dichas leyes.
Primero, debemos notar que, por lo que sabemos, la primera ley de Kepler no
puede ser completamente correcta. Si dos cuerpos son libres excepto por la fuerza
de interaccin que existe entre ellos, entonces stos deben moverse de manera que el
o
e
centro de masas se mantenga en reposo (o en movimiento uniforme). O sea, el Sol
necesariamente tambin tiene que estar movindose. Sin embargo, siendo la masa del
e
e


CAP
ITULO 11. GRAVITACION

392

Sol muy superior a la de los planetas, el centro de masas Sol-planeta esencialmente


coincide con la posicin del Sol y en ese caso resulta ser una muy buena aproximacin
o
o
suponer que el Sol est en reposo en uno de los focos de la elipse. (La masa del Sol
a
es de 1.99 1030 kg mientras que la de la Tierra es de 5.98 1024 kg.)
Ahora bien, tal como se demostr en la Sec. 8.2, el hecho de que un planeta barre
o
a
reas iguales en tiempos iguales es equivalente a decir que el momento angular no
var en funcin del tiempo. Esto a su vez implica que el torque ejercido por el Sol
a
o
sobre el planeta es nulo, lo que a su vez implica que la fuerza entre los dos cuerpos
debe ser a lo largo de la l
nea que los une. En otras palabras, la segunda ley de Kepler
implica que la fuerza entre el Sol y el planeta debe ser radial, en direccin r. Ms
o
a
an, si la energ se conserva, entonces debe existir una energ potencial U (r ), tal
u
a
a
que la fuerza gravitacional es el gradiente de U (r ). Por lo tanto, para que la fuerza
est en la direccin r , necesariamente U debe depender slo de r = | r |, y por lo tanto
e
o
o
la fuerza gravitacional es de la forma
F (r ) = f (r) r ,

con f (r) alguna funcin que depende slo de la distancia entre las part
o
o
culas.
Hasta el momento, hemos usado la segunda ley de Kepler, en conjunto con las
leyes de conservacin del momento angular y de la energ para encontrar la forma
o
a,
que deber tener la fuerza de gravedad entre dos cuerpos. Para encontrar la funcin
a
o
f (r) debemos jarnos en ms datos observacionales. La tabla adjunta muestra algunos
a
de los parmetros orbitales para los planetas pertenecientes al sistema solar.
a
Planeta
Mercurio
Venus
Tierra
Marte
Jpiter
u
Saturno
Urano
Neptuno
Plutn
o

Semieje mayor
u.a.
0.387
0.723
1.000
1.523
5.202
9.554
19.218
30.109
39.60

Per
odo
[s]
7.60 106
1.94 107
3.16 107
5.94 107
3.74 108
9.30 108
2.66 109
5.20 109
7.82 109

Excentricidad

Inclinacin
o

0.205
0.006
0.016
0.093
0.048
0.055
0.046
0.008
0.246

7 00
3 23

1 51
1 18
2 29
0 46
1 46
17 07

Masa
[kg]
3.28 1023
4.83 1024
5.98 1024
6.37 1023
1.90 1027
5.67 1026
8.80 1025
1.03 1026
5.4 1024

(Los astrnomos, para medir distancias, frecuentemente usan la unidad astronmio


o
ca, u.a. Una unidad astronmica es igual al semieje mayor de la rbita terrestre,
o
o
1 u.a.= 1, 495 1011 m.)
De la tabla anterior se sigue que la excentricidad = (r2 r1 )/(r1 + r2 ) de
la mayor de los planetas es bastante pequea, siendo sus rbitas, por lo tanto,
a
n
o
casi circulares. (Al dibujar una elipse con una excentricidad = 0.05 es dif
cil, slo
o
mirndola, darse cuenta de que diere de un c
a
rculo.)


CAP
ITULO 11. GRAVITACION

393

Si suponemos que, en primera aproximacin, las rbitas de los planetas son circuo
o
lares, entonces, a partir de la tercera ley de Kepler, podemos encontrar una expresin
o
para la magnitud de la fuerza gravitacional. En efecto:
La tercera ley de Kepler dice que
T 2 = Kr 3 ,
donde K es una constante (la misma para todos los planetas) y r es el radio de la rbita
o
circular. El radio r de la rbita, la velocidad v y el per
o
odo T estn relacionados
a
por
2r
T =
.
v
Por otra parte la magnitud de la fuerza gravitacional debe coincidir con la fuerza
centr
peta, o sea,
mv 2
,
f (r) =
r
donde m es la masa del planeta. Usando estas tres ecuaciones, despejando v y T , se
encuentra
4 2 m 1
.
f (r) =
K r2
La constante K, de acuerdo a la tercera ley de Kepler, no depende de ninguna propiedad de los planetas, pero si podr depender de alguna propiedad del Sol. Es ms
a
a
o menos evidente, por razones de simetr que si la fuerza gravitacional depende de
a,
la masa m del planeta, entonces debe tener la misma dependencia de la masa del Sol
M . Esta ultima observacin sugiere escribir

o
4 2
= GM ,
K
donde G es una nueva constante que ahora es independiente de las masas del Sol y
del planeta. De esta manera se deduce que la fuerza gravitacional entre dos masas m
y M , separadas por una distancia r, es
F (r ) = G

mM
r.

r2


CAP
ITULO 11. GRAVITACION

394

La ultima ecuacin precisamente es la

o
ley de gravitacin universal de Newton
o
siendo G la asi llamada constante gravitacional. Usando una balanza de torsin (ver gura 11.4), Cavendish en el
o
ao 1798 midi G en el laboratorio.
n
o
El dispositivo experimental fue el siguiente: dos masas m se colocan en los
extremos de una barra r
gida y se cuelgan desde el centro de un alambre (hilo
de torsin) que se sujeta rmemente del
o
cielo. El sistema puede girar libremente
en el plano horizontal, pero tiene una
orientacin para la cual est en equilio
a
brio.

Figura 11.4

Si las masas m se sacan del equilibrio, por ejemplo, dndoles una pequea velocidad
a
n
de rotacin, entonces el sistema comenzar a oscilar tenindose un pndulo de torsin.
o
a
e
e
o
A partir del per
odo de este movimiento oscilatorio se puede deducir la constante de
restitucin de torsin (o sea, el torque que ejerce el hilo de torsin cuando este se gira
o
o
o
en una magnitud ).
La orientacin del pndulo se dedecta con un rayo de luz que es reejado por un
o
e
pequeo espejo adosado a la varilla.
n
El experimento mismo se realiza en dos etapas. Primero se colocan dos masas M
en las posiciones A y se registra la orientacin de la varilla. Luego se rotan las masas
o
M a las posiciones B y se vuelve a registrar la orientacin de la varilla. A partir de
o
la variacin de la orientacin de la varilla en estas dos mediciones se puede deducir
o
o
la fuerza entre m y M .
El experimento es bastante dif ya que la fuerza de atraccin de las dos masas
cil
o
es muy pequea. Por ejemplo, una masa m = 20 kg es atra por otra de masa
n
da
M = 150 kg con una fuerza de slo 2, 3.5 105 g N (esto es, el peso de una masa de
o
23 milligramos) cuando la separacin de sus centros es de 30 cm.
o
El valor actualmente aceptado para el valor de G es:
G = (6.673 0.003) 1011

m3
.
kg s2

Ejercicio: Conocidos el per


odo y el radio de la rbita terrestre alrededor del Sol y
o
usando el valor de G recin dado, determine la masa del Sol.
e


CAP
ITULO 11. GRAVITACION

11.3.

395

Satlites
e

En esta seccin analizaremos algunos aspectos del movimiento de satlites que


o
e
orbitan gravitacionalmente entorno a un objeto masivo. Algo es un satlite si su masa
e
m es mucho menor que la masa M del objeto alrededor del cual orbita. En particular,
los resultados de la presente seccin podrn aplicarse al movimiento de los planetas
o
a
alrededor del Sol, de las lunas alrededor de los planetas y de los satlites articales
e
alrededor de la Tierra. Las trayectorias de todos estos objetos corresponden a elipses
(y c
rculos), o sea, a cnicas con excentricidades 0 < 1.
o
En la seccin 11.1 se relacionaron los parmetros r0 y de una elipse con el
o
a
semieje mayor a y el semieje menor b y tambin con el perigeo r1 = rmin y el apogeo
e
r2 = rmax de ella. A continuacin estudiaremos la dependencia de estos parmetros
o
a
de las constantes de movimiento de la rbita; espec
o
camente del momento angular
orbital y de la energ total E.
a
La condicin m M implica que la masa mayor esencialmente se matendr en
o
a
reposo en uno de los focos de la elipse, lugar en que ubicaremos el origen.
La fuerza sobre la part
cula m (el satlite) viene dada por
e
mM
r.

r2
Esta fuerza, que es conservativa, da origen a una energ potencial
a
F (r) = G

U (r) = G

mM
.
r

Si en cierto instante la posicin y veloo


cidad de la part
cula m es r y v, entonces la energ total del sistema ser
a
a
E =K +U =

1
mM
mv 2 G
.
2
r

Figura 11.1

Sean P1 y P2 los puntos correspondientes al perigeo y apogeo, respectivamente (ver


gura 11.5). Como la energ total se conserva sta debe ser igual a E tanto en el
a
e
apogeo como en el perigeo, o sea,
mM
1
mM
1
2
2
= mv2 G
.
E = mv1 G
2
r1
2
r2

(11.8)

Como la fuerza que acta sobre m es central se tiene que tambin el momento
u
e
angular (respecto al origen) se conserva. Evaluando el momento angular en el apogeo


CAP
ITULO 11. GRAVITACION

396

y perigeo, se encuentra
= mr1 v1 = mr2 v2 .
De esta ecuacin se obtiene
o
v1 =

mr1

v2 =

(11.9)

.
mr2

Sustituyendo esto en (11.8) se encuentra


E=

mM
1 2
2 G r
2m r1
1

(11.10)

E=

1 2
mM
.
2 G r
2m r2
2

(11.11)

De estas dos ecuaciones se deduce


2
2mEr1 = 2 2Gm2 M r1

y
2
2mEr2 = 2 2Gm2 M r2 .

Restndolas se obtiene
a
E(r2 r1 )(r2 + r1 ) = GmM (r2 r1 )
E = G

mM
mM
,
= G
r1 + r2
2a

o sea, el semieje mayor de la rbita viene determinado slo por la energ E:


o
o
a
a = G

mM
.
2E

(11.12)

Encontremos ahora una relacin entre b (el semieje menor de la rbita) y el momento
o
o
angular y energ del satlite. Esta se obtiene restando las ecuaciones (11.11) de
a
e
(11.10):
2
mM
mM
2

+G
=0
2
2 G r
r2
2mr1
2mr2
1
2

1
1
2 r2
r1
2
2

= 2Gm2 M

1
1

r1 r2

2
2
r1 r2
r2 r1
2
2 r 2 = 2Gm M r r
r1 2
1 2


CAP
ITULO 11. GRAVITACION

397

r2 + r1
= 2Gm2 M
r1 r2
2a
2 2 = 2Gm2 M ,
b

o sea,

a2
2
=
.
(11.13)
Gm2 M
2Em
La importancia de las ecuaciones (11.12) y (11.13) radica en que son stas las que
e
relacionan las constantes de movimiento de la rbita con su forma geomtrica.
o
e
b2 =

Otra relacin importante se obtiene usando la segunda Ley de Kepler: como el


o
momento angular se conserva se tiene que la part
cula m barre reas iguales en
a
tiempos iguales. La cantidad de rea que el satlite barre en un intervalo de tiempo
a
e
dt lo podemos evaluar cuando ste se encuentra en el apogeo:
e
1

dA = r2 v2 dt =
dt .
2
2m
De esta relacin se deduce que el rea total, A = ab, la part
o
a
cula m lo barre en un
tiempo T dado por

T ,
ab =
2m
es decir,
4m2 2 a2 b2
T2 =
.
2
Usando (11.13) se obtiene, nalmente,
T2 =

4 2 3
a ,
GM

(11.14)

resultado que no es otro que la tercera ley de Kepler. Pero observe que ahora conocemos la constante de proporcionalidad entre T 2 y a3 .
Deseamos recalcar que las ecuaciones (11.13) y (11.14) son slo vlidas en el l
o
a
mite
m M . Cuando las dos masas son del mismo orden las ecuaciones deben corregirse).
Ilustremos el uso de las ecuaciones anteriores resolviendo un problema:
Problema:
Si la Tierra conservara su energ total, pero perdiera la mitad de su cantidad de
a
momento angular respecto al Sol (por ejemplo, cambindole bruscamente la direccin
a
o
de su velocidad),


CAP
ITULO 11. GRAVITACION

398

a) Qu tanto se acercar al Sol?


e
a
b) Cul ser la distancia de alejamiento mxima (apogeo) de la Tierra en su
a
a
a
nueva rbita?
o
c) Cul ser el largo del ao en ese caso?
a
a
n
d) Cul tendr que ser el ngulo en que se var bruscamente la direccin de la
a
a
a
a
o
Tierra?
Suponga que inicialmente la rbita es circular.
o

Solucin:
o
Sea R el radio de la rbita circular en torno al Sol (estamos suponiendo que el
o
centro de masas del sistema coincide con la posicin del Sol). La rbita circular es un
o
o
y b los semiejes mayor y
caso particular de elipse para la cual a = b = R. Sean a
menor de la elipse nal (despus de haberle cambiado bruscamente su direccin).
e
o
Como la energ de la Tierra no cambia, se tiene que a = a = R. De la ecuacin
a
o
= b/2 = R/2.
(11.13) se deduce que el semieje menor disminuir a la mitad, o sea, b
a
Para el perigeo y apogeo se encuentra

2 3

2 b2 = R
2 R2 /4 = R
R
r1 = a a
2
y

r2 = a +
Como el per
odo slo depende de a
o
(tercera ley de Kepler) y a no cambia, se encuentra que el ao de la
n
Tierra en su nueva rbita seguir
o
a
siendo de 365 d
as.
La magnitud del momento angular
viene dada por la magnitud del momento lineal por el brazo. Como el
momento lineal no cambia, para disminuir el momento angular a la mitad debemos disminuir el brazo a la
mitad. De la gura 11.6 se desprende inmediatamente que el ngulo dea
.
be ser = 60

a2

b2

2+ 3
=R
.
2

Figura 11.6


CAP
ITULO 11. GRAVITACION

11.4.

399

Potencial efectivo

Sea r el vector posicin de m. La fuerza que acta sobre m es


o
u
F (r) = G

mM
r,

r2

(11.15)

y la energ potencial asociada a esta fuerza es


a
U (r) = G

mM
.
r

(11.16)

A continuacin mostraremos que cuando la energ potencial es central, es decir,


o
a
slo depende de la magnitud del vector r y no de su direccin, entonces el problema de
o
o
determinar la trayectoria del satlite se puede reducir a un problema unidimensional.
e
Es evidente que si la energ potencial de una part
a
cula es central, entonces el
campo de fuerzas generador del potencial, es radial. Por otra parte, una fuerza radial
no ejerce torque respecto al origen. Si el torque respecto al origen es nulo, el momento angular de la part
cula no puede alterarse, luego el momento angular (para una
part
cula movindose en un potencial central) es una constante de movimiento (igual
e
que, por ejemplo, la energ total). Que el momento angular sea una constante de moa
vimiento signica que ni la magnitud ni la direccin del momento angular cambian a
o
medida que transcurre el tiempo. Tambin sabemos que la velocidad de la part
e
cula
es siempre perpendicular al momento angular. Como la direccin del momento ano
gular no cambia se concluye que el movimiento de la part
cula necesariamente debe
transcurrir en un plano.
Debido a la importancia de este resultado lo volvemos a remarcar:
En un potencial central la fuerza siempre es solamente radial. Un potencial
central no ejerce un torque sobre la partcula respecto al origen, lo que a

su vez implica que el momento angular de la partcula respecto a tal origen

nunca vara (es una constante de movimiento). Una consecuencia de lo

anterior es que el movimiento de una partcula en un potencial radial

siempre trancurre en un plano.


El papel que juega el momento angular como constante de movimiento es similar al
papel de constante de movimiento que juega la energ total; ambos son magnitudes
a
que, de alguna manera, estn determinadas por las condiciones iniciales del problema.
a
Supongamos, por consiguiente, que conocemos la energ total E, la magnitud del
a
momento angular del sistema y el plano en el que transcurre el movimiento.


CAP
ITULO 11. GRAVITACION

400

Descompongamos la velocidad v(r ) de la


part
cula, cuando ella se encuentra en el lugar r, en una componente radial y una componente perpendicular a r (ver gura 11.7):

v(r ) = vr r + v .

(11.17)

Ac r y son vectores unitarios; el primero


a
en la direccin radial, el segundo perpendicuo
lar a r, pero en el plano de la trayectoria.

Figura 11.7
El mdulo del momento angular de la part
o
cula es
= r m v ,
o sea, una vez conocido la velocidad v queda determinada por la distancia r de la
part
cula al centro:

.
(11.18)
v (r) =
mr
De lo anterior concluimos que el problema queda esencialmente resuelto si logramos
establecer cmo var la distancia r en funcin del tiempo. Una vez conocido r(t), no
o
a
o
slo se conoce la velocidad radial vr = r, sino que tambin, usando la ecuacin (11.18),
o

e
o
la velocidad v . A su vez, conocida la velocidad en funcin del tiempo podemos,
o
integrando, obtener la posicin r (t) de la part
o
cula en todo instante.
De la discusin anterior se desprende que debemos centrar la atencin en resolver el
o
o
movimiento radial de la part
cula de masa m.
Sabemos que la energ total es la suma de la energ cintica y la energ poa
a
e
a
tencial:
E =K +U .
(11.19)
Por otra parte, la energ cintica viene dada por
a
e
1
1
2
2
K = m v 2 = m vr + v ,
2
2
que, usando r = vr y la ecuacin (11.18), queda de la forma

o
1
1
K = m v2 = m
2
2

r2 +

2
m2 r 2

(11.20)

(11.21)

Sustituyendo este resultado en (11.19) se obtiene


1

E = m r2 +
2

2
+ U (r)
2mr 2

(11.22)


CAP
ITULO 11. GRAVITACION

401

Esta es una ecuacin que slo depende del radio r, de la velocidad radial r y conso
o

tantes de movimiento. Esta ecuacin, por lo tanto, se puede reinterpretar como la


o
correspondiente al problema de una part
cula de masa m y energ total E, que se
a
mueve en una dimensin (dada por la coordenada r) en un potencial dado por
o
Ue (r) = U (r) +

2
.
2mr 2

(11.23)

De esta manera hemos reducido el problema original a un problema unidimensional:


el de una part
cula de masa m afectada por una fuerza
Fe (r) =

dUe (r)
mM
2
= G 2 +
.
dr
r
mr 3

(11.24)

Ue (r) se llama el potencial efectivo, mientras que 2 /(2mr 2 ) es el as llamado poten


cial centrfugo. Cuando el momento angular es cero, el potencial efectivo coincide con

el potencial original.
La ecuacin (11.22) podemos escribirla como
o
1

E = mr 2 + Ue (r) .
2
Derivndola respecto al tiempo se obtiene
a
dUe
r ,

(11.25)
dr
de donde, dividiendo por r y usando (11.24), encontramos una ecuacin de movimiento

o
para r(t):
2
mM
.
(11.26)
m r = G 2 +

r
mr 3
Es esta la ecuacin que resolveremos en la prxima seccin para encontrar las trayeco
o
o
torias de los satlites.
e
0 = mrr +

De acuerdo a lo discutido ms arriba, para establecer cmo se comporta el veca


o
tor r, debemos resolver el problema de una part
cula de masa m movindose en el
e
potencial dado por la ecuacin (11.23):
o
Ue (r) =

2
GM m
+
.
r
2mr 2

(11.27)

La energ total E = K + U y el momento angular son constantes de movimiento,


a
es decir, para un problema f
sico en particular, tienen valores jos bien determinados.


CAP
ITULO 11. GRAVITACION

402

A Caso = 0.
Consideremos brevemente el caso de momento angular = 0. Este valor para el
momento angular implica que la part
cula nunca tiene una velocidad tangencial,
o sea, la part
cula siempre se mueve a lo largo de la recta que une M con m.
La gura 11.8 muestra el potencial Ue (r) que, en este caso, coincide con U (r).
Si la energ de la part
a
cula es E < 0, entonces la mxima distancia a la que se
a
puede alejar es r0 .

Figura 11.8 : Potencial efectivo para = 0.


Si la part
cula en un instante est en r0 , entonces su energ total E coincide
a
a
con la energ potencial U (r0 ) y la energ cintica, por lo tanto, es nula
a
a
e
la part
cula se encuentra en reposo. Sin embargo, sobre la part
cula acta una
u
fuerza ya que la pendiente de la energ potencial en r0 no es nula. De hecho,
a
la pendiente es positiva, luego sobre m acta una fuerza negativa que la hace
u
acelerar hacia el origen. A medida que transcurre el tiempo la distancia entre m
y M disminuir progresivamente. La energ cintica de m (y por consiguiente
a
a
e
el mdulo de la velocidad radial), como tambin la fuerza atractiva irn auo
e
a
mentando. Finalmente, despus de transcurrido un tiempo nito, la masa m
e
llegar al origen, teniendo una energ cintica innita. (No debemos preocua
a
e
parnos demasiado por este innito que apareci. Obviamente ningn potencial
o
u
f
sico es de la forma GmM/r hasta r = 0. Todos los objetos f
sicos tienen un


CAP
ITULO 11. GRAVITACION

403

tamao, y a ms tardar cuando la distancia entre los objetos es menor que la


n
a
suma de sus radios, la interaccin cambia de carcter.)
o
a
B Caso = 0.
La gura 11.9 muestra el potencial U (r), el potencial centr
fugo 2 /(2mr 2 ) y el
potencial efectivo Ue (r) para dos valores del momento angular (1 > 2 ). Es
este ultimo, el potencial efectivo, l que es util para analizar el comportamiento

de la variable r = |r | en funcin del tiempo.


o

Figura 11.9: Potencial centrfugo y efectivo para dos valores del momento

angular distintos (no nulos).


Consideremos nuevamente una part
cula con energ E < 0 (ver gura 11.10).
a
La part
cula, en este caso, est restringida a moverse entre r1 y r2 . (Para el caso
a
del movimiento de la tierra alrededor del sol el vector r recorre una trayectoria
el
ptica y las magnitudes r1 y r2 corresponden a la distancia m
nima y mxima
a
de esa elipse). La energ cintica de la part
a
e
cula es K = E U (r), siendo
Kr = E Ue (r) la energ cintica radial y Kt = Ue (r) U (r) la energ
a
e
a
cintica tangencial. Cuando la part
e
cula se encuentra en el perigeo r1 o en el
apogeo r2 , ella no tiene energ cintica radial, pero s tiene una energ cintica
a
e

a
e


CAP
ITULO 11. GRAVITACION

404

tangencial. Note que la energ cintica tangencial es mayor en el perigeo que


a
e
en el apogeo.

Figura 11.10: Potencial efectivo para = 0.


Si la energ es positiva (E > 0), entonces la part
a
cula no est ligada; la part
a
cula
se acerca hasta una distancia m
nima y luego se aleja indenidamente, para
no volver. (Estas trayectorias corresponden a las soluciones hiperblicas del
o
problema; los cometas que provienen de fuera del sistema solar son un ejemplo de
tales trayectorias.) Cuando la energ total es exactamente cero, la trayectoria
a
tampoco es acotada y la trayectoria, como veremos en la siguiente seccin,
o
corresponde a una parbola.
a
Cuando la energ total es igual al m
a
nimo del potencial efectivo, entonces la
part
cula no tiene energ cintica radial, pero s una energ cintica tangencial;
a
e

a
e
tal trayectoria corresponde a la solucin circular del problema gravitatorio.
o

11.5.

Trayectorias de los satlites


e

En esta seccin analizaremos las posibles trayectorias de un satlite de masa m


o
e
cuando es atra gravitacionalmente por un cuerpo masivo de masa M de acuerdo
do
a la ley de gravitacin universal.
o


CAP
ITULO 11. GRAVITACION

405

Del analisis hecho en la seccin anterior (ver ecuacin 11.26) se desprende que
o
o
debemos estudiar la ecuacin de movimiento
o
m = G
r

mM
2
+
.
r2
mr 3

(11.28)

El momento angular es una constante de movimiento y viene determinada por las


condiciones iniciales del problema, es decir, para un problema en particular no cambia
su valor a medida que transcurre el tiempo.
Sabemos que al menos algunas de las soluciones de esta ecuacin deben ser elipses.
o
En coordenadas polares la ecuacin de una elipse es
o
1
1
=
(1 cos ) .
r
r0
Esto sugiere que, en lugar de analizar una ecuacin para r en funcin de t, ser
o
o
a
mejor buscar una ecuacin para w = w() 1/r en funcin de , pues tal ecuacin
o
o
o
probablemente sea simple.
Realicemos esos cambios de variable. Tenemos
r=

d 1
1 dw()
1 dw d
d
r=
= 2
= 2
.
dt
dt w
w
dt
w d dt

Pero

w2

==
,
=
dt
mr 2
m

luego
r=

dw
.
m d

Derivando nuevamente respecto al tiempo se obtiene


r=

d dw
dt d

d dw
d d

d
d2 w
d2 w d
2
=
= 2 w2
.
dt
m d2 dt
m
d2

Sustituyendo esta expresin en (11.28) y usando que w = 1/r, se obtiene


o

o sea,

2 2 d2 w
2
w
= GmM w2 + w3 ,
m d2
m
1
d2 w
+w =
.
2
d
r0

En la ecuacin anterior se introdujo r0 denido por


o
r0

2
.
GM m2

(11.29)


CAP
ITULO 11. GRAVITACION

406

Tal como sospechbamos, la ecuacin diferencial para w en funcin de , efectivamente


a
o
o
es muy simple. Si el lado derecho fuese nulo, la solucin ser
o
a
w() = A cos() ,
donde A es una constante. (Elegir la funcin seno en lugar coseno slo redene el lugar
o
o
desde el cual estamos midiendo los ngulos.) Es claro que al agregarle la constante
a
1/r0 a la ultima expresin, se encuentra una solucin de (11.29). Por ultimo, si en

o
o

lugar de A introducimos dado por


= A r0 ,
entonces la expresin para r() = 1/w() queda de la forma
o
1
1
=
(1 cos ) .
r()
r0

(11.30)

Hemos encontrado una expresin para la distancia r en funcin del ngulo polar. Como
o
o
a
es usual para ecuaciones diferenciales de segundo grado, la solucin general tiene dos
o
constantes (en este caso r0 y ) que deben determirse a partir de las condiciones
iniciales.
La ecuacin (11.30), de acuerdo al valor de psilon, corresponde a las distintas
o
e
secciones cnicas:
o
=0

corresponde a un c
rculo

0<<1

corresponde a una elipse

=1

corresponde a una parbola


a

1 < corresponde a una hiprbola


e

11.6.

El campo y potencial gravitacional

La ley de gravitacin universal de Newton nos da la fuerza de atraccin debido a


o
o
la gravedad que acta entre dos masas, en efecto,
u
F (r ) = G

mM
r

r2

es la fuerza que acta sobre la masa m debido a la masa M donde r es la separacin


u
o
entre las masas y r es un vector unitario que apunta de M a m.

Es conveniente introducir el concepto de campo gravitacional para describir el


efecto que una masa M introduce en su entorno. Denimos el campo gravitacional de
la masa M por
M

g(r ) G 2 r .
r


CAP
ITULO 11. GRAVITACION

407

Si en el lugar r de este campo gravitacional generado por la masa M colocamos una


masa m, entonces sobre m actuar una fuerza
a
F = mg(r ) .
Un campo es un ente que est denido en todos los puntos del espacio que nos interesa.
a
Como g es un vector, el campo gravitacional se dice que es un campo vectorial.
No slo existen campos vectoriales, tambin existen campos escalares, tensoriales,
o
e
etc.. Un ejemplo de un campo escalar es la temperatura. Supongamos que en todos los
puntos de una pieza puedo medir la temperatura T , entonces T (r ) dene un campo
escalar (el campo de temperatura) de la pieza. Que un campo sea vectorial signica
que en cada punto no slo tiene una magnitud sino que tambin una direccin.
o
e
o
El campo gravitacional satisface el principio de superposicin: si distintas masas
o
m1 , m2 , . . . , mN ubicados en los lugares r1 , r2 , . . . , rN generan campos gravitacionales
g1 , g2 , . . . , gN , entonces el campo gravitacional en el punto r ser
a
mj
(r rj ) .
g(r ) =
gj (r rj ) = G
|r rj |3
j

Cuando se tiene un campo de fuerzas conservativo, resulta muy conveniente introducir el concepto de energ potencial. El campo gravitacional es conservativo y
a
en el Cap. 5 demostramos que la energ potencial de dos masas M y m, separadas
a
por una distancia r, viene dado por
mM
.
r
La constante aditiva arbitraria se ha elegido de manera que la energ potencial sea
a
nula cuando las dos masas estn separadas por una distancia innita (r ). La
a
relacin anterior sugiere denir otro campo, un campo escalar, por
o
U (r) = G

(r) = G

M
.
r

(r ) se llama el potencial gravitatorio de la masa M . Si se coloca una masa m en el


potencial gravitatorio de una masa M , entonces la energ potencial del sistema es
a
U (r) = m(r) ,
donde r es la separacin entre las masas m y M .
o
El potencial gravitacional tambin satisface el principio de superposicin: si dise
o
tintas masas m1 , m2 , . . . , mN ubicados en los lugares r1 , r2 , . . . , rN generan potenciales gravitacionales 1 , 2 , . . . , N , entonces el potencial gravitacional total en el
punto r ser
a
mj
.
(r ) =
j (r rj ) = G
|r rj |
j


CAP
ITULO 11. GRAVITACION

408

Ilustremos los conceptos anteriores con dos problemas.


Problema 1:
Considere dos masas, de valor m y 2m,
que se encuentran separados por una
distancia 2a (ver gura adjunta).
a) Calcule el valor de la energ potencial
a
gravitacional de una masa m ubicada en
el punto medio entre las dos.
b) Cul es la fuerza gravitacional ejercida sobre m ?
a

Figura 11.11

c) Cul es la velocidad de escape ve ? Depender sta de la direccin?


a
ae
o
Solucin
o
La energ potencial de una masa m ubicada en el punto central es
a
U = m = G

2mm
3mm
mm
G
= G
.
a
a
a

La fuerza que acta sobre m es


u
F = m g = m G

m
2m
mm
x + G 2 x = G 2 x .

a2
a
a

La velocidad de escape es la m
nima velocidad que debe darse a la part
cula para que
se aleje llegando hasta innito (se escape del sistema). Para que la masa m escape
su energ debe ser no negativa. La m
a
nima energ que debe tener es por lo tanto
a
E = 0, o sea, la energ cintica debe ser de igual magnitud que la energ potencial
a
e
a
pero de signo contrario:
1
3mm
2
K = m ve = G
.
2
a
Finalmente, para la velocidad de escape, que no depende de la direccin, se encuentra
o
ve =

6Gm
.
a


CAP
ITULO 11. GRAVITACION

409

Problema 2:
Considere dos part
culas, de masas m y M , que inicialmente estn en reposo y sepaa
radas por una distancia muy grande (innita). Demuestre que en cualquier instante
su velocidad relativa de acercamiento atribuible a la atraccin gravitacional es
o
v=

2G(m + M )
,
D

donde D es la distancia que las separa.


Solucin
o
Situmonos en el sistema del centro de masas. Si en cierto instante las velocidades de
e
las dos masas (observadas desde el centro de masas) son vm y vM , stas cumplen con
e
mvm = M vM .
Para la energ cintica de ambas part
a
e
culas se obtiene
m
1
1
1
2
2
2
K = mvm + M vM = mvm 1 +
2
2
2
M

Inicialmente tanto la energ potencial (la separacin es grande) como la cintica


a
o
e
son nulas. Como la energ total se conserva, esta deber ser nula en todo instante.
a
a
Cuando las part
culas estn separadas por una distancia D la energ potencial es
a
a
U = G

mM
,
D

y, por lo tanto, la energ cintica debe ser de la misma magnitud pero de signo
a
e
contrario:
m
mM
1
2
=G
K = mvm 1 +
.
2
M
D
Despejando vm se obtiene
2GM 2
2
.
vm =
D(m + M )
La velocidad relativa es igual a v = vm + vM , luego
v=

2GM 2
m
+
D(m + M ) M

2GM 2
=
D(m + M )

2G(m + M )
.
D


CAP
ITULO 11. GRAVITACION

11.7.

410

El caso elctrico: la ley de Coulomb


e

Es interesante, en este punto, mostrar brevemente la gran analog existente entre la


a
fuerza de gravedad entre dos masas y la fuerza elctrica entre dos cargas q1 y q2 .
e
En efecto, la fuerza electrosttica sobre una part
a
cula 2, de carga q2 , debido a una part
cula 2, de carga q1 , est dada por la ley de Coulomb que, en el sistema de unidades MKS, es
a
F =

1 q1 q2
r,

40 r2

donde r es un vector unitario que apunta de la part

cula 1 a la 2.
El campo y potencial electrosttico asociados a una carga Q son, respectivamente,
a
E(r ) =

1 Q
r

40 r2

(r ) =

1 Q
.
40 r

La relacin entre E y es:


o
r

(r ) = (r0 )

r0

E(r ) dr .

Finalmente, al colocar una carga q en el campo gravitacional de una carga Q, entonces


la fuerza sobre q y su energ potencial vienen dadas por
a
F (r ) = q E(r )
y
U (r) = q(r) .
Las expresiones anteriores son formalmente iguales a las correspondientes al caso gravitatorio, con los siguientes reemplazos:
G
m, M
g

1
,
40
q, Q ,
E.

Una diferencia importante e interesante es que en la electrosttica las cargas pueden ser
a
positivas y negativas, mientras que en el caso gravitatorio todas las masas son positivas. Este
hecho da origen a numerosas situaciones y fenmenos interesantes que no pueden darse en el
o
caso gravitatorio (por ejemplo, el concepto de dipolo puntual ). Adems, en el caso gravitatorio
a
la fuerza entre dos masas (que siempre tienen el mismo signo) es siempre atractiva, mientras
que en el caso electrosttico la fuerza entre dos cargas del mismo signo es siempre repulsiva.
a
Esto se maniesta en el cambio de signo que hay que introducir entre la constante gravitatoria
G y la constante que aparece en la ley de Coulomb.


CAP
ITULO 11. GRAVITACION

11.8.

411

Campo gravitacional de una cscara esfrica


a
e

Considere una cscara esfrica de radio R


a
e
de densidad de masa uniforme. Si la masa total de la cscara es M , entonces su
a
densidad supercial de masa ser
a
=

M
.
4R2

Deseamos encontrar el potencial gravitacional en el punto P debido a la cscaa


ra esfrica. Para ello primero evaluamos
e
la contribucin de un anillo de ancho R d
o
que forma un ngulo con la l
a
nea que une
P con el centro de la esfera. La masa del
anillo es
dm = (2R sin ) R d .
Figura 11.12
Todos los puntos del anillo estn a la misma distancia D = (R sin )2 + (r R cos )2
a
del punto P , luego la contribucin del anillo al potencial gravitatorio es
o
d = G

dm
= 2GR2
D

sin d

(R sin )2 + (r R cos )2
sin d
= 2GR2
.
2 + r 2 2rR cos
R
Sumando la contribucin de todos los anillos obtenemos el potencial total:
o

sin d
.
+ r 2 2rR cos
0
0
La integral la podemos evaluar con un cambio de variable. Denotemos cos por s.
Entonces
s = cos
ds
= sin ,
d
o sea
sin d = ds .
=

d = 2GR2

R2

Observando que si = 0 entonces s = 1 y si = , entonces s = 1, podemos


reescribir la integral:

I=
0

sin d
=
2 + r 2 2rR cos
R

1
1

ds

.
2 + r 2 2rRs
R


CAP
ITULO 11. GRAVITACION

412

Ejercicio: Demuestre que


d
dx

b
,
a + bx =
2 a + bx

donde a y b son constantes. Como corolario de este ejercicio se tiene que


x2
x1

1
2
a + bx
dx =
b
a + bx

x2

=
x1

2
[ a + bx2
b

a + bx1 ] .

Usando ahora el resultado del ejercicio propuesto (con a = R2 + r 2 , b = 2rR,


x1 = 1 y x2 = 1), se obtiene
I

1
1

ds

R2 + r 2 2rRs

2
R2 + r 2 + (2rR)(1)
2rR
1
[(R + r) |R r|]
rR

=
=

R2 + r 2 + (2Rr) 1

Distinguiendo con cuidado los dos casos r > R y R < r, se obtiene que
2
si r < R
R
I=
.
2
si r > R
r

De esta manera, para el potencial de una cscara esfrica de radio R y masa M se


a
e
encuentra la expresin
o

para R > r
G M
R
(r) =
.

M
G r
para r > R
Algunas observaciones importantes respecto a este resultado:

a) Para r > R, o sea, cuando el punto P se encuentra fuera de la cscara esfrica,


a
e
el potencial gravitacional que sta ejerce es idntica a la que se hubiese obtenido
e
e
si toda la masa de la cscara se ubica en el origen.
a
b) Si el potencial afuera es el mismo al de una masa puntual, entonces tambin el
e
campo gravitacional lo ser, o sea, la fuerza que la cscara esfrica ejercer sobre
a
a
e
a
una masa m (si sta se encuentra en el exterior) ser
e
a
F = G

mM
r.

r2


CAP
ITULO 11. GRAVITACION

413

c) En el interior de la cscara esfrica el potencial gravitacional es constante (no


a
e
depende de la posicin). Esto signica que el campo gravitacional ah es nulo,
o

o sea, si colocamos una masa m en el interior, sobre ella la cscara esfrica no


a
e
ejercer ninguna fuerza gravitacional.
a
Resumen:
Para una cscara esfrica de radio R y masa M se tiene
a
e
GM

r2 r
g(r ) =

0
GM
r
(r ) =
GM
R

(para r > R)
(para r < R)
(para r > R)
(para r R)

La gura 11.13 muestra un grco de la intensidad del campo gravitacional y el


a
potencial para una cscara esfrica.
a
e
Notemos que el potencial es una funcin continua de r. Debe ser as ya que
o
,
el potencial est dado por una integral sobre r (del campo gravitatorio). El campo
a
gravitacional, en cambio, puede ser discontinuo, y de hecho lo es en este caso, como
muestra la Fig. 11.13.

Figura 11.13
A continuacin presentaremos una demostracin alternativa de que una cscara
o
o
a
esfrica no ejerce ningn campo gravitacional al interior de ella.
e
u


CAP
ITULO 11. GRAVITACION
Algunos preliminares: Al intersectar un
a
ngulo (innitesimal) d con un c
rculo de
radio r, el largo del arco es ds = r d (ver
gura 11.14). Si tal arco se inclina en un
a
ngulo , el largo del arco (l
nea segmentada) ahora es ds = rd/ cos .

414

Figura 11.14

Estos conceptos se pueden generalizar para ngulos slidos en tres dimensiones. Paa
o
ra ellos tomemos una esfera de radio r y
consideremos un rea A (que puede tea
ner forma irregular) sobre la supercie. Si
unimos todos los puntos del per
metro del
rea con el centro obtenemos un ngulo
a
a
slido.
o
En forma anloga a lo que ocurre para
a
ngulos en un plano, denimos el ngua
a
lo slido por A/r 2 . El ngulo slido
o
a
o
completo (en el espacio tridimensional) es,
por lo tanto, 4. A la inversa, si tenemos
un ngulo slido innitesimal d e intera
o
ceptamos este con una esfera de radio r, el
rea denido por la interseccin ser dA =
a
o
a
r 2 d. Si el rea la inclinamos en un ngua
a
2 d/ cos .
lo , su rea ser dA = r
a
a
Volviendo a la cscara esfrica de radio R
a
e
y masa M , evaluemos el campo gravitacional en un punto P que se encuentra en
su interior (ver gura 11.16).
Consideremos un ngulo slido d desde
a
o
el punto P y hacia los dos lados. El ngua
lo slido intersecta a la cscara esfrica en
o
a
e
los puntos A y B (ver gura 11.16). La inclinacinde las reas en A y B es en el miso
a
mo ngulo ya que ABO es un tringulo
a
a
issceles. El rea que el ngulo slido ino
a
a
o
2
tersecta en A es r1 d/ cos mientras que
2
en B es r2 d/ cos . Sea la densidad
supercial de masa de la cscara esfrica.
a
e

Figura 11.15

Figura 11.16


CAP
ITULO 11. GRAVITACION

415

El campo gravitacional que el rea A genera en P es


a
gA =

2
r1 d
cos

d
1

2 n = cos n .
r1

Ac n es un vector unitario que apunta de P a A. En forma anloga se obtiene que


a
a
el campo gravitacional que el rea B genera en P es
a
gB =

2
r2 d
cos

d
1
n

2 ( ) = cos n.
r2

Se observa que el campo gravitacional de las reas A y B se cancela exactamente. Lo


a
mismo ocurre con cualquier otro ngulo slido. De la discusin anterior se concluye
a
o
o
que al interior, el campo gravitacional generado por una cscara es necesariamente
a
nulo.

11.9.

Campo gravitacional de una esfera slida


o

Evaluemos el campo gravitacional en algn punto al exterior de una esfera slida


u
o
de masa M y radio R. Para ello es conveniente pensar que la esfera est compuesta de
a
muchas cscaras esfricas (como una cebolla). Ya sabemos que el campo gravitacional
a
e
de cada cscara es el mismo que el que se obtiene al concentrar toda la masa de la
a
cscara en el centro. La suma de todas las cscaras (que es la esfera slida), por lo
a
a
o
tanto, generar un campo gravitacional igual al de una masa (equivalente a la suma
a
de las masas de todas las cscaras) concentrada en el centro. O sea, si el punto P
a
est al exterior de la esfera, a una distancia r del centro, el campo gravitacional ser
a
a
g(r) = G

M
r.

r2

Un razonamiento anlogo permite encontrar el potencial gravitacional (para un punto


a
en el exterior): (r) = GM/r.


CAP
ITULO 11. GRAVITACION

416

Si el punto P se encuentra al interior de


la esfera se puede proceder de una manera
parecida. Dividamos nuevamente la esfera en numerosas cscaras esfricas. Todas
a
e
las cscaras esfricas con un radio mayor
a
e
que r no contribuyen al campo gravitacional en P (pues P est al interior de ellas).
a
Las capas con radio menor que r las podemos concentrar en el centro. El campo
gravitacional por lo tanto ser
a
m
r,

r2
donde m es la masa de las cscaras interiores, esto es,
a
g(r) = G

m=M

Figura 11.17

r3
.
R3

De manera anloga se procede para encontrar el potencial gravitatorio al interior de


a
una esfera slida.
o
Resumen: Para una esfera slida homognea, de radio R y masa M , se tiene
o
e
GM

(para r > R)
r2 r
g(r ) =
GM r
R3 r

(para r < R)
GM
(para r > R)
r

(r ) =
GM
r2

3 R2
(para r R)
2R

La gura 11.18 muestra un grco de la intensidad del campo gravitacional y el


a
potencial para una cscara esfrica.
a
e


CAP
ITULO 11. GRAVITACION

417

Figura 11.18

Densidad media de la Tierra


Problema: Determinar la densidad media de la Tierra suponiendo concocidos el valor de la constante de la gravitacin G, la aceleracin de la gravedad g y el radio
o
o
terrestre R.
Solucin: Sobre una masa m colocada en la supercie de la Tierra acta una fuerza
o
u
igual a mg. Por otra parte, el campo gravitacional de la Tierra para un punto sobre
su supercie se puede evaluar suponiendo que toda la masa de la Tierra est en el
a
centro, es decir,
GM
|g(R)| = 2 ,
R
donde M es la masa total de la tierra. Conociendo el campo garvitacional podemos
evaluar la fuerza que acta sobre una masa m; esta es
u
m|g(R)| =

GM m
.
R2

Igualando las dos expresiones para la fuerza se obtiene para g la expresin


o
g=

GM
.
R2

Esta ecuacin relaciona la aceleracin de la gravedad g=9,81 m/s2 con la constante


o
o
gravitacional G y la masa y radio terrestre M y R, respectivamente.
La densidad media de la Tierra viene dada por
=
=

3M
3g
M
=
=
3
V
4R
4GR
3 981, 0
g
5, 5
.
4 6, 67 108 6, 4 108
cm3


CAP
ITULO 11. GRAVITACION

418

(Para referencia: la densidad del hierro, que es el principal elemento del ncleo teu
3 .)
rrestre, es de 7,86 g/cm

11.10.

Problemas

1. Jpiter tiene doce lunas conocidas, cuatro de las cuales fueron descubiertas por
u
Galileo. Estos son los radios y per
odos de las primeras cuatro:

R (km)
180.000
422.000
671.000
1.072.000

T (d
as)
0,498
1,769
3,551
7,155

a) Obedecen estos cuatro satlites a la tercera ley de Kepler?


e
b) Con estos datos y el valor de G, encuentre la masa de Jpiter.
u
c) El dimetro de Jpiter es de 142.900 Km. Cul es su densidad media?
a
u
a
2. Las rbitas de dos satlites terrestres A y B son el
o
e
pticas, siendo R y 4R,
respectivamente, sus ejes mayores.
a) Cul es el cuociente entre sus energ mecnicas totales?
a
as
a
b) Cul es el cuociente entre sus per
a
odos?
c) Qu puede decirse del cuociente entre sus momentos angulares?
e
3. Un satlite articial recorre una trayectoria circular 320 Km por encima de la
e
supercie terrestre.
a) Cul ser su velocidad?
a
a
b) Con qu frecuencia girar alrededor de la Tierra?
e
a


CAP
ITULO 11. GRAVITACION

419

4. Dos satlites articiales de igual mae


sa orbitan alrededor de un planeta.
S1 se mueve en una rbita circuo
lar de radio 108 [m] y per
odo de
5 d
as, S2 se mueve en una rbita
o
el
ptica de radios rmin = 108 [m] y
rmax = 2 108 [m].
a) A partir de los datos para la rbio
ta circular, calcule la masa del planeta.
b) Encuentre el per
odo de S2 .

Figura 11.19

c) Cul satlite es ms veloz al pasar por B? Cul tiene mayor energ


a
e
a
a
a?
d) Calcule el cuociente entre las velocidades que S2 tiene en B y en A.
e) Qu maniobra deber ordenar el puesto de mando para poner S2 en la
e
a
o
rbita de S1 ?
5. Un satlite es geoestacionario si para un observador jo sobre la Tierra este no
e
se mueve. La rbita de tal satlite necesariamente deber coincidir con el plano
o
e
a
del Ecuador terrestre. Determine el radio de la rbita del satlite.
o
e
Respuesta: R 42.000 km.
6. A qu distancia de la Tierra debe colocarse un cuerpo en la l
e
nea dirigida
hacia el Sol de tal manera que la atraccin gravitacional solar contrarreste a la
o
atraccin de la Tierra? El Sol est a 15 107 Km de distancia y su masa es de
o
a
MS = 3, 24 105 MT (MT es la masa de la Tierra = 5, 97 1024 Kg). Analice el
problema incluyendo el efecto introducido por la rotacin de la Tierra alrededor
o
del Sol (o sea, tanto la Tierra como el cuerpo giran alrededor del Sol una vez al
ao).
n
Para encontrar la solucin (aproximada) use el hecho de que MT /MS 1.
o
Respuesta:

rR

MT
3MS

es la solucin que se tiene si el cuerpo se encuentra entre la Tierra y el Sol. Hay


o
dos soluciones adicionales si se permite que r sea mayor que R. Convnzase de
e
sto y encuntrelas.
e
e

7. Dos satlites A y B giran alrededor de la Tierra en la misma rbita circular


e
o
(de radio R), pero uno al lado opuesto de la Tierra respecto al otro. Se desea


CAP
ITULO 11. GRAVITACION

420

interceptar el satlite B con un proyectil lanzado desde A. Estudie el problema


e
e indique velocidad y direccin en que debe lanzarse el proyectil para lograr su
o
objetivo. D al menos 3 soluciones distintas.
e
8. El perigeo, punto ms prximo de la rbita de un satlite, se halla a 320 Km
a
o
o
e
de la supercie terrestre, y el apogeo, punto ms alejado, a 2400 Km.
a
a) Cul es el semieje mayor de la rbita del satlite?
a
o
e
b) Cul es la excentricidad de su rbita?
a
o
c) Si el satlite tiene una masa de 15 Kg, cul es su energ total?
e
a
a
d) Cul es su velocidad en el apogeo?
a
e) Cul es su velocidad en el perigeo?
a
f) Cul es su momento angular?
a
9. Una pequea masa m cae hacia el Sol partiendo del reposo desde una distancia
n
igual al radio de la rbita terrestre. Determine el tiempo de ca usando slo
o
da
o
las leyes de Kepler.
10. Se dice que la rbita de un satlite es heliosincrnica si pasa diariamente por
o
e
o
los mismos lugares a la misma hora. Suponiendo que la altura m
nima de un
satlite es de 200 km por sobre la supercie terrestre, encuentre los radios de
e
todas las rbitas heliosincrnicas circulares.
o
o
11. Considere la parbola y = ax2 .
a
a) Encuentre su foco.
b) Traslade el sistema de coordenadas de manera que el nuevo origen coincida
con el foco calculado en la parte a).
c) Introduzca coordenadas polares (r, ), midiendo desde el m
nimo de la
parbola y demuestre que la ecuacin de la parbola queda de la forma
a
o
a
1
1
= (1 cos )
r
s
con = 1 y s algn valor constante.
u
12. Un satlite gira sobre el Ecuador, en el mismo sentido que la Tierra, en una
e
trayectoria el
ptica con rmin = 200 km y rmax = 500 km. Se desea poner este
satlite en rbita geoestacionaria. El cohete del satlite es capaz de acelerarlo
e
o
e
con una aceleracin a = 50 m/s2 . En qu instantes y durante cunto tiempo
o
e
a
se deben prender los motores para lograr el propsito?
o
13. Calcule el per
odo de rotacin de la Luna en torno a la Tierra sabiendo que el
o
radio de su rbita es 60.3 veces el radio de la Tierra.
o


CAP
ITULO 11. GRAVITACION

421

14. Un cometa de masa m se dirige (cae) radialmente hacia el sol. Observaciones astronmicas permiten estableo
cer que la energ mecnica
a
a
total del cometa es nula, es decir, E = 0. El cometa se estrella contra Venus, cuya masa es m. Supongamos adems
a
que la trayectoria de Venus es
circular, de radio R0 . A consecuencia del choque, el cometa
y Venus forman un solo astro
que llamaremos Vennus.

Figura 11.20

a) Calcule la rapidez v0 y el per


odo de Venus antes de la colisin.
o
b) Calcule la energ mecnica de Venus en su rbita antes de chocar con el
a
a
o
cometa.
c) Calcule la velocidad radial y el momento angular de Vennus inmediatamente despus de la colisin.
e
o
d) Determine la energ mecnica de Vennus y exprsela en trminos de m,
a
a
e
e
y v0 .
e) Demuestre que la rbita de Vennus es el
o
ptica y determine el semieje mayor
de la rbita.
o
f ) Determine si el ao para los venusianos se ha acortado o alargado a
n
causa del choque con el cometa.
Respuesta parte f): La razn del per
o
odo de Vennus y Venus es
1+
T
=
T
1 + 4

3/2

15. Un proyectil de masa m se lanza tangencialmente a la supercie de la Tierra.


Suponiendo que no hay resistencia del aire (como en la Luna), calcular la rapidez
v0 con que el proyectil debe ser lanzado para que orbite en forma circular y
rasante a la Tierra. Compare esta velocidad con la velocidad de escape. Calcule
el momento angular y la energ del proyectil en esta situacin.
a
o
16. El proyectil del problema anterior ahora es lanzado tangencialmente a la super
cie de la Tierra con una rapidez v0 , con 1 < < 2. Calcule la distancia
radial del perigeo y apogeo y la excentricidad de la bita.
o


CAP
ITULO 11. GRAVITACION

422

17. Dos part


culas de igual masa se unen mediante una cuerda ideal de longitud h. El par es
atra gravitacionalmente por un planeta de
do
masa M . La distancia entre el planeta y la
part
cula mas cercana es R, con h R.
a) Despreciando la fuerza de atraccin eno
tre las dos part
culas, calcule la tensin
o
de la cuerda si ellas caen sobre el planeta con la cuerda estirada y dispuesta
radialmente.
b) Ahora tome en cuenta la atraccin grao
vitacional entre las dos masas. Demuestre que para que la cuerda no est tene
sa la masa de cada part
cula debe ser
m = M (h/R)3 .

Figura 11.21

18. Marte tiene un dimetro medio de 6.800 Km, la masa de Marte es 0.107 MT .
a
a) Cul es la densidad media de Marte comparada con la de la Tierra?
a
b) Cul es el valor de g en Marte?
a
c) Cul es la velocidad de escape en Marte?
a
19. La estrella enana Sirio B tiene un radio que es 1/50 del radio solar, a pesar de
tener aproximadamente la misma masa que el sol.
a) Cul es el valor de g en la supercie de Sirio B? b) Cul ser la densidad
a
a
a
media de Sirio B?
20. Una esfera uniforme de densidad 0 y radio
R1 tiene una cavidad esfrica de radio R2 .
e
Encuentre el potencial en el punto r (ver gura 11.22).
21. Cul ser el peso suyo si el radio de la Tierra
a
a
doblara su valor,
a) mantenindose la masa de la Tierra igual
e
a la actual?
b) mantenindose la densidad promedio de la
e
Tierra igual a la actual?

Figura 11.22

22. Dos esferas de plomo, de 1 m de radio, estn en contacto.


a
a) Cul es la magnitud de la fuerza de atraccin mutua?
a
o
b) Qu velocidad tendr en el instante de contacto si partieran de posicioe
an
nes muy separadas en el espacio y cayeran una contra la otra? (plomo =


CAP
ITULO 11. GRAVITACION

423

11.3 g/cm3 ).
23. Un satlite de masa m = 5 kg es lanzado a una rbita circular cuyo per
e
o
odo es
de 120 minutos. Ignore la rotacin de la Tierra y tambin cualquier efecto del
o
e
roce viscoso del satlite con el aire.
e
a) Calcule la razn entre el radio de la rbita del satlite y el radio terrestre.
o
o
e
b) Calcule la energ m
a nima requerida para poner al satlite en dicha bita.
e
o
De esta energ qu fraccin se uso para subirlo y que fraccin para
a
e
o
o
darle la velocidad requerida?
24. Una nave csmica se dirige hacia la Luo
na a lo largo de una trayectoria parablica que casi toca a la supercie luo
nar. En el momento de mxima aproa
ximacin un motor de frenado, en un
o
breve lapso, modica la velocidad de la
nave de manera que esta entre en una
o
rbita circular alrededor de la Luna.
Encuentre la velocidad de la nave justo
antes y despus del frenado. La masa y
e
el radio lunar son M = 7, 34 1022 kg y
R = 1, 74 106 m, respectivamente.

Figura 11.23

25. Se taladra un tnel liso y recto a travs de un planeta esfrico cuya densidad de
u
e
e
masa 0 es constante. El tnel pasa por el centro del planeta y es perpendicular
u
al eje de rotacin del mismo. El planeta rota con una velocidad angular detero
minada de modo que los objetos dentro del tnel no tienen aceleracin relativa
u
o
al tnel. Hallar la relacin entre o y para que esto sea cierto.
u
o
26. Demuestre que en un conducto excavado en la Tierra, siguiendo una cuerda
cualquiera, (no siguiendo necesariamente un dimetro), el movimiento de un
a
objeto ser armnico simple. (Desprecie efectos de roce y de la rotacin de la
a
o
o
Tierra.) Encuentre el per
odo del movimiento.
27. Discuta el origen de las mareas. Por qu se presentan dos mareas y no una
e
sola?


CAP
ITULO 11. GRAVITACION

424

28. Encuentre la rbita de una part


o
cula
que se mueve en un campo central
que genera una fuerza
F (r) =

Gm1 m2
2
+ 3 .
r2
r

Si es pequeo con respecto al mon


mento angular muestre que la rbio
ta corresponde a una elipse cuya
orientacin precesa lentamente. Eno
cuentre el ngulo en que cama
bia la orientacin del semieje mao
yor en un per
odo. (Indicacin: Reo
pita lo hecho en la seccin 11.5 pero
o
con la nueva expresin para F .)
o

Figura 11.24

29. a) Demuestre que para escapar de la atmsfera de un planeta una condicin que
o
o
debe cumplir una molcula es que tenga una velocidad tal que v > 2GM/r,
e
siendo M la masa del planeta y r la distancia de la molcula al centro del
e
planeta.
b) Determine la velocidad de escape para una part
cula atmosfrica a 1000 km
e
sobre la supercie de la Tierra.
c) Haga lo mismo para la Luna.
30. Considere una esfera de radio a y hecha de materia homognea y que al interior
e
tiene una cavidad esfrica concntrica de radio b.
e
e
a) Haga un grco de la fuerza de gravitacin F ejercida por la esfera sobre
a
o
una part
cula de masa m, localizada a una distancia r del centro de la
esfera.
b) Haga un grco de la energ potencial gravitacional U (r) de la masa m
a
a
en funcin de r.
o
c) Demuestre que en el l
mite b a vuelve a obtener el resultado correspondiente a una cscara esfrica.
a
e
d) Demuestre que en el l
mite b 0 vuelve a obtener el resultado correspondiente a una esfera slida.
o


CAP
ITULO 11. GRAVITACION

425

31. Considere un recipiente (cascarn)


o
semiesfrico. Demuestre que en
e
cualquier punto del plano que pasa
por el borde del recipiente (a modo
de tapa, regin punteada), el campo
o
gravitatorio es perpendicular a dicho plano.
Figura 11.25
32. Considere una semiesfera slida de masa M y radio R, ja. Cul es el trabajo
o
a
que se debe realizar para llevar una pequea masa m desde el centro de la base
n
hasta el innito?
33. Una part
cula de masa m se encuentra situada sobre el eje de simetr de un
a
anillo de masa M y radio R, a la distancia d del plano del anillo.
a) Encontrar la fuerza con que el anillo atrae a la masa m, en trminos de G,
e
M , m, R y d.
b) Discuta en particular los casos d = 0 y d , y convnzase de que se
e
obtiene lo que uno espera intuitivamente.
c) Si la masa m se mueve a lo largo del eje y parte del innito (desde una
distancia muy grande) con velocidad nula, con que velocidad pasar por
a
el centro del anillo? (Suponga que el anillo no se mueve, es decir, hay una
fuerza externa que lo mantiene jo).

34. Considere un satlite de masa m que gira en una rbita circular de radio R an
e
o
torno a un planeta de masa M m.
a) Determine la velocidad del satlite.
e
b) Suponga que el satlite es interceptado por un proyectil, tambin de mae
e
sa m, que se desplaza radialmente hacia el planeta. Sea v1 = v1 r la

velocidad del proyectil justo antes del impacto. Suponga que el choque
es completamente inelstico (es decir, el proyectil y el satlite forman un
a
e
solo cuerpoo despus de la colisin). Encuentre la velocidad que tiene el
e
o
proyectilsatlite justo despus del choque.
e
e
c) Determine la m
nima rapidez v1 que debe tener el proyectil justo antes
del choque para que el cuerpo proyectilsatlite logre escapar del campo
e
gravitacional del planeta.


CAP
ITULO 11. GRAVITACION

426

35. Considere un casquete esfrico jo,


e
muy delgado, de densidad uniforme, radio R y masa M , que posee dos oricios
que lo perforan en posiciones diametralmente opuestas. Una masa puntual
m se encuentra inicialmente en reposo
a una distancia 3R de su centro sobre
la l
nea que une las perforaciones. Encuentre el tiempo que tarda la masa m
en cruzar el casquete de un extremo al
otro.

Figura 11.26

36. Dos cscaras esfricas del mismo raa


e
dio R y masa M se encuentran separados (sus centros) por una distancia r.
a) Haga un grco esquemtico
a
e
de la energ potencial gravia
tatoria a lo largo de la recta
A, B.
b) Si una part
cula de masa m
se suelta en reposo desde el
centro de la esfera #1, con
qu velocidad llega a la supere
cie de la esfera #2?

Figura 11.27

(c) Cul es la mxima separacin r a la que pueden estar las esferas para que
a
a
o
la masa m llegue a la supercie de la cscara esfrica #2?
a
e
37. Un satlite de masa m orbita en torno a un planeta de masa M m. La rbita
e
o
es circular, de radio R.

R
A


CAP
ITULO 11. GRAVITACION

427

Cuando pasa por el punto A, los motores se encienden y su rapidez aumenta en


un factor , de modo escencialmente instantneo. La direccin de la velocidad
a
o
no cambia, de modo que sigue siendo tangente al c
rculo de radio R. Luego los
motores se vuelven a apagar.
a) (20 %) Calcule la energ total del satlite Ef despus de que los motores
a
e
e
se apagan, en trminos de G (la constante de gravitacin), las masas de
e
o
los cuerpos y R.
b) (15 %) En qu tipo de rbita queda el satlite si = 6/5? Cul es el
e
o
e
a
mximo valor de para que el satlite quede en una rbita cerrada en
a
e
o
torno al planeta?
c) (50 %) Si r es la distancia entre el satlite y el planeta, encuentre los valores
e
de r tales que la energ total Ef es igual al potencial efectivo gravitatorio
a
Ue (r).
d) (15 %) Qu representan f
e
sicamente las soluciones encontradas en el punto
anterior?
38. Considere dos discos homogneos, a y b, de densidad supercial de masa y
e
radios Ra y Rb , respectivamente. Ambos discos estn orientados paralelamente,
a
a una distancia L, y con sus respectivos centros sobre el eje x.

b
Ra
Rb
^
x

Considere adems una part


a
cula puntual de masa m que slo puede moverse a
o
lo largo del eje x, entre ambos discos.

Calcule la energ potencial gravitatoria de dicha part


a
cula cuando est a una
a
distancia x del disco a.
Si la part
cula se suelta desde el reposo en el centro del disco a, con qu veloe
cidad llega al centro del disco b?
39. Considere un satlite de masa 2m orbitando en torno a un planeta de masa
e
M 2m, en una rbita circular de radio R.
o


CAP
ITULO 11. GRAVITACION

428

En un determinado instante, el satlite se desintegra en dos trozos de masa m.


e
La velocidad de cada uno de los trozos, inmediatamente despus de la desintee
gracin, forma un ngulo con la velocidad del satlite inmediatamente antes
o
a
e
de ella.

2m

m

Encuentre la velocidad de cada trozo inmediatamente despus de la desintegrae


cin, y la excentricidad de la rbita en la que queda cada uno de los trozos.
o
o
40. Considere el siguiente modelo para Saturno: una esfera homognea de radio R y
e
masa M , y un anillo en su plano ecuatorial, de radio Ra y masa Ma . Si el eje z

se dene como el eje perpendicular al plano ecuatorial que pasa por el centro de
Saturno, y z = 0 corresponde al centro de ste, calcule el campo gravitacional
e
g(z) en todos los puntos del eje z .

^
z

R
Ra

Ma

41. Un planeta P , de masa mp , describe una rbita el


o
ptica en torno a una estrella
de masa M mp . Los semiejes mayor y menor de la rbita son a y b, reso
pectivamente. Cuando el planeta se encuentra en el afelio, es impactado por un


CAP
ITULO 11. GRAVITACION

429

cuerpo C, de masa mc M . Si el impacto es frontal y perfectamente inelstico,


a
qu velocidad debe tener el cuerpo C para que el sistema formado por P y C
e
quede describiendo una rbita circular en torno a la estrella?
o

mp

mc

11.11.

Solucin a algunos de los problemas


o

Solucin al problema 4
o
(a) La fuerza gravitacional debe coincidir con la fuerza centr
peta. Sea M la masa
del planeta y m la masa del satlite, entonces
e
G

mv 2
Mm
r,

r=

R2
R

donde v = 2R/T es la velocidad del satlite en su rbita circular. De las


e
o
ecuaciones anteriores se deduce que
M=

4 2 1024
4 2 R3
=
kg 3, 2 1024 kg .
GT 2
6, 67 1011 (5 24 3600)2

(b) De acuerdo a la tercera ley de Kepler


T1
=
T2

a1
a2

3/2

donde a1 y a2 son los semiejes mayores de las trayectorias el


pticas de los satlie
8 [m] y a = (r
8 [m],
tes. De acuerdo al enunciado a1 = 10
2
min +rmax )/2 = 1, 510
luego a1 /a2 = 2/3. Para T2 se obtiene
T2 = T1

a2
a1

3/2

= 5 (1, 5)1 , 5 d 1, 19 d .
as
as


CAP
ITULO 11. GRAVITACION

430

(c) La energ total y el semieje mayor de un satlite estn relacionados por la


a
e
a
ecuacin
o
mM
E = G
.
2a
La energ es inversamente proporcional al semieje mayor, pero observe que
a
debido al signo menos, la rbita que tiene el mayor a tiene tambin mayor
o
e
energ (es menos negativa). Conclu
a
mos que el satlite S2 es el que tiene mayor
e
energ En el punto B ambos satlites tienen la misma energ potencial, luego
a.
e
a
la energ cintica de S2 es mayor que la de S1 . El satlite S2 es mas rpido que
a
e
e
a
S1 cuando pasan por B.
(d) Se tiene que (todo evaluado en el punto B)
a1
2
E2
=
= .
E1
a2
3
Por otra parte, para la rbita circular U = 2K1 , o sea,
o
E1 = U + K1 = U
Se tiene

U
Mm
U
=
= G
.
2
2
2R2

U + K2
2
E2
=
= .
E1
U/2
3

De aqu se deduce que K2 = 2U/3. Luego

2U/3
4
v2
K2
=
= = 2 .
2
K1
U/2
3
v1
(e) Para pasar de la bita el
o
ptica a la circular, el satlite S2 debe, cuando se
e
encuentra pasando por el punto B, prender los motores y frenar hasta bajar
la velocidad de v2 a v1 .

Solucin al problema 9
o
La trayectoria de la masa m que cae hacia el Sol es el l
mite de una elipse en que el
semieje menor b tiende a cero. En ese caso la trayectoria es una l
nea recta estando el
Sol en uno de los extremos. El semieje mayor de tal elipse es a = R/2, donde R = aT
es el radio (igual al semieje mayor) de la trayectoria circular de la Tierra. De acuerdo
a la tercera ley de Kepler
Tm
=
TT

a
aT

3/2

= 23/2 = 0, 3536 .


CAP
ITULO 11. GRAVITACION

431

El tiempo t0 que demora la masa m en llegar al sol es la mitad del per


odo de su
o
rbita, o sea,
Tm
TT
t0 =
=
0, 3536 64, 5 d .
as
2
2
Solucin al problema 14
o
(a) Sea M la masa del Sol, entonces, igualando la fuerza gravitacional con la fuerza
centr
peta
GM m
mv 2

r = 0r

2
R0
R0
se obtiene
2
v0 =

GM
.
R0

(b) La energ menica de Venus (antes de la colisin) es


a
a
o
Ei =

GM m 2 1
GM m
2
+ mv0 =
2 .
R0
2
2R0

(c) Como el cometa (cuando est lejos) se mueve radialmente hacia el sol, no tiene
a
momento angular (respecto al origen en el Sol). Luego el momento angular de
Vennus es el mismo que el de Venus
L = R0 mv0 .

Esto nos permite encontrar la componente de la velocidad de Vennus justo


despus de la colisin. El momento angular justo despues de la colisin es
e
o
o
L = R0 (m + m)v .
Como el momento angular se conserva se deduce que
v =

v0
.
1+

La conservacin del momento lineal en la direccin radial hay que darse cuenta
o
o
que la interaccin entre Venus y el cometa son fuerzas internas y, por lo tanto,
o
para calcular la velocidad del cometa podemos ignorar el efecto introducido por
la interaccin entre el cometa y Venus. El cometa tiene energ nula, luego,
o
a
K = U = +

1
GM m
2
= mvC .
R0
2


CAP
ITULO 11. GRAVITACION

432

(vC es la velocidad del cometa justo antes de la colisin ignorando el efecto


o
introducido por Venus). Se deduce que
2
vC =

2GM
.
R0

Aplicamos ahora la conservacin del momento lineal a lo largo de la direccin


o
o
radial
mvC = (m + m)vr ,
donde vr es la velocidad de Vennus justo despus de la colisin. Se deduce que
e
o
vr =

vC .
1+

(d) La energ mecnica de Vennus (la evaluamos justo despus del choque) es
a
a
e
Ef

= U +K =
= (1 + )
=

GM m
2R0

GM m(1 + ) 1
2
2
+ m(1 + )(v + vr )
R0
2

GM m 1
+ (1 + ) 2
R0
2
1 + 4
1+

= Ei

1+

1 + 4
1+

1
(1 + )2

GM m
R0

(e) la rbita de Vennus obviamente ya no es un c


o
rculo. Como la energ es negativa
a
debe, por lo tanto, ser el
ptica. Se tiene que
af
af
Ei
=
=
.
Ef
ai
R0
Aqu ai y af son los semiejes mayores de las rbitas de Venus y Vennus, respec
o
tivamente. Se deduce que
a f = R0

Ei
1+
= R0
.
Ef
1 + 4

(f) Usando la tercera ley de Kepler podemos calcular la razn del per
o
odo de Vennus
y Venus:
af 3/2
1 + 3/2
T
=
=
.
T
r0
1 + 4

Solucin al problema 17
o


CAP
ITULO 11. GRAVITACION

433

(a) La fuerza neta que acta sobre la part


u
cula ms cercana, llammosla #1, es
a
e
F1 =

GM m
r + Tr .

R2

La fuerza neta que acta sobre la otra part


u
cula (#2) es
F2 =

GM m
r Tr .

(R + h)2

Como el hilo que las une es inextensible, ambas part


culas aceleran con la misma
aceleracin a = a. Se tiene
o
r
F1 = ma =
r
y
F2 = ma =
r

GM m
r + Tr .

R2

GM m
r Tr .

(R + h)2

Despejando la tensin del hilo T se encuentra que


o
T =

GM m
2

1
1

2
R
(R + h)2

Con h R se obtiene la expresin


o
T =

GM mh
.
R3

(b) Para que la cuerda no quede tensa la fuerza gravitacional entre las part
culas
debe coincidir con T , o sea,
Gmm
.
T =
h2
Despejando m de las dos ultimas ecuaciones se encuentra

m=M

h
R

Solucin al problema 32
o
El trabajo que debemos realizar es independiente del camino que elijamos para llevar
la masa m del centro de la basa de la semiesfera hasta el innito. Elijamos un camino


CAP
ITULO 11. GRAVITACION

434

recto paralelo a la base. Sea ste el


e
eje x y denotemos por y al eje que

coincide con el eje de simetr de la


a
semiesfera (ver gura).
Sea F (x) la fuerza gravitacional que
acta sobre la masa m cuando esu
ta se encuentra sobre el eje x en la

posicin x. Esta fuerza tendr dos


o
a
componentes
F (x) = Fx (x) + Fy (x) .
x
y

Figura 11.28

La componente y de la fuerza no nos interesa pues al evaluar el trabajo con la expresin


o
W =

F (x) (dx x)

se observa que tal componente es perpendicular al desplazamiento y, por lo tanto,


no gura al evaluar el producto punto. Para evaluar Fx (x) coloquemos una segunda
semiesfera idntica en la parte inferior (ver gura l
e
nea punteada). Por simetr la
a
fuerza que ejerce esta segunda esfera es
F (x) = Fx (x) Fy (x) ,
x
y
o sea, la componente x es la misma, pero la componente y cambia de signo. El efecto
de las dos semiesferas es
F + F = 2Fx (x) .
x
Pero, por otra parte, las dos semiesferas forman una esfera completa de masa 2M , y
para ese caso sabemos que la fuerza es (ver seccin 11.9)
o

G(2M )m x

(para x > R)

x2
mg(x ) =
G(2M )m

x

(para x < R)
R3
Igualando las dos expresiones se encuentra que
GM m
(para x > R)
x2
Fx (x) =
GM m
R3

(para x < R)

Esto es exactamente el mismo resultado que se tendr para una esfera slida completa
a
o
de masa M y radio R. Podemos entonces usar los resultados que se encontraron en la
seccin 11.9 para la esfera slida. El trabajo lo podemos evaluar usando el potencial
o
o


CAP
ITULO 11. GRAVITACION

435

gravitatorio . El potencial gravitatorio (con el cero del potencial en el innito) al


centro de una esfera slida es
o
3GM
(0) =
.
2R
Luego eltrabajo para llevar la masa m del origen al innito es
W =m

3GM
.
2R

Solucin al problema 35
o
Sobre la part
cula m no se ejerce ninguna fuerza cuando se encuentra al interior del
casquete. Si v0 es la velocidad con que llega a la supercie, entonces el tiempo de
traves es t0 = 2R/v0 . Para calcular la velocidad v0 hacemos uso del hecho de que
a
la energ debe conservarse. La energ de m en el punto de partida es
a
a
Ei =

GM m
,
3R

mientras que cuando llega a la supercie es


Ef =

GM m 1
2
+ mv0 .
R
2

Igualando las dos expresiones se deduce que


2
v0 =

4GM
.
3R

Al interior del casquete no hay fuerzas sobre la masa m y, por lo tanto, su velocidad
se mantiene contante. Para el tiempo de traves se obtiene
a
t0 =

3R3
.
GM

Cap
tulo 12

Fluidos
versin 10 septiembre 2012
o

12.1.

Conceptos preliminares

Un uido es una substancia incapaz de soportar fuerzas de cizalla. Es sta la propiedad


e
que distingue a un slido de un uido. En la
o
gura 12.1 se muestra una placa, la cual se
intenta deslizar hacia la derecha mediante la
aplicacin de una fuerza F . Un pasador slido
o
o
evita que esto ocurra. Sin embargo, cuando
el pasador es sustituido por un l
quido o un
gas, la placa comenzar a deslizarse (aun paa
ra fuerzas F pequeas). El uido no es capaz
n
de ejercer una fuerza de cizalla para mantener el equilibrio.

pasador

placa

Figura 12.1

La densidad de una sustancia es la razn entre su masa y volumen: = m/V .


o
C, es 1.00 g/cm3 (es el valor mximo de la densidad del
La densidad del agua, a 4
a
agua).
Los uidos se dividen en dos subclases: los l
quidos y los gases. Los l
quidos se caracterizan por ocupar un volumen denido independiente del volumen del recipiente
que lo contiene. Un gas, por otra parte, tiende a expandirse y a ocupar todo el volumen del recipiente que lo contiene. La compresibilidad del uido es otra propiedad
marcadamente distinta en los l
quidos y en los gases. Un l
quido es bastante incompresible y en la gran mayor de las aplicaciones se puede suponer que su densidad es
a

constante. Lo opuesto es cierto para los gases. Estos son sustancias muy compresibles

436

CAP
ITULO 12. FLUIDOS

437

y generalmente no se puede suponer que su densidad sea constante.


A pesar de que los uidos estn constituidos por molculas, en el presente cap
a
e
tulo
se tratan como un medio continuo. El uso de los aspectos macroscpicos de un uido
o
est justicado cuando el camino libre medio (es decir, la distancia media que alcanza
a
a recorrer una molcula del uido antes de colisionar con otra) es mucho menor que
e
las distancias involucradas del sistema bajo consideracin.
o
Sea F una fuerza que acta en forma perpendicular sobre un rea A. Se dene
u
a
la presin P por la relacin
o
o
F
P
.
A
Considere un uido en reposo (por ejemplo, un vaso de agua, una piscina o
una lago). Al sumergir un objeto en el uido, ste ejercer una fuerza sobre las
e
a
supercies del objeto. La fuerza por unidad de rea (o presin) que ejerce un uido
a
o
sobre los objetos (o supercies) con las que est en contacto, tiene varias propiedades
a
importantes:
a) La fuerza que un uido en reposo ejerce sobre una supercie es siempre perpendicular a ella. Esto est directamente relacionado con el hecho de que un uido es
a
incapaz de ejercer una fuerza de cizalla.
b) Un uido, en un punto particular, ejerce la misma presin en todas las direcciones
o
(Principio de Pascal). En otras palabras, la presin es una magnitud escalar. Si
o
sumergimos en el uido un cubo innitesimal, la fuerza sobre todas las caras del
cubito ser la misma, siendo su magnitud F = P A. Aqu A es el rea de una de
a

a
las caras del cubito y P es la presin del uido en el lugar donde se encuentra el
o
cubo (estamos despreciando variaciones de la presin sobre distancias del tamao
o
n
del cubito).
c) Los lugares isobricos (de igual presin) en un uido en reposo (y de densidad
a
o
constante) son los planos horizontales. En la gura 12.2, en los puntos A, B, C,
D y E la presin es la misma. Tambin la presin es igual en los puntos F , G, H
o
e
o
e I. La presin es mayor en puntos ubicados a mayor profundidad. En el punto J
o
la presin es menor que en el punto F .
o

CAP
ITULO 12. FLUIDOS

438

J
H

aire
F

lquido

Figura 12.2

12.2.

La presin atmosfrica P0
o
e

La presin en la supercie de un uido que se encuentra en un recipiente abierto


o
a la atmsfera no es nula, sino igual a la presin atmosfrica. Esta ultima se debe a
o
o
e

que estamos inmersos en un uido (compresible) constituido por el aire. La atmsfera


o
de la Tierra ejerce una presin sobre todos los objetos con los que est en contacto,
o
a
en particular sobre otros uidos. La presin atmosfrica sobre la supercie terrestre
o
e
la denotaremos por P0 , y es igual a la presin ejercida por el peso de toda la columna
o
de aire que est por encima.
a
P0 no es despreciable o insignicante como algunas personas suelen creer. Por el
contrario, la presin atmosfrica juega un papel importante en numerosos aparatos y
o
e
mquinas de la vida diaria.
a
Antes de continuar digamos algo sobre las unidades de la presin:
o
En el sistema SI, la unidad de presin es el Pascal: 1 Pa = 1 N/m2 . A 105 Pa se
o
5 Pa. Observe que 1 bar es aproximadamente la
le suele llamar bar , o sea 1 bar = 10
presin que ejerce una masa de 1 kg si sta est apoyada sobre un rea de 1 cm2 . En
o
e
a
a
efecto,
9.81 N
= 0.981 105 N/m2 = 0.981 bar.
1 Kg/cm2 =
0.0001 m2
Tambin observe que 1 kg es la masa de una columna de agua de 10 m de altura y
e
2 de seccin transversal.
1 cm
o
Otra unidad frecuentemente usada para medir la presin es la atmsfera (atm).
o
o
1 atm corresponde a la presin promedio que ejerce la atmsfera terrestre a nivel del
o
o
mar. Experimentalmente se encuentra que sta es aproximadamente 1.013 105 N/m2
e
= 1.013 bar. Se dene la atmsfera estndar por
o
a
1 atm = 1.0135 105 N/m2 = 1.0135 bar .

CAP
ITULO 12. FLUIDOS

439

O sea, y esto es util recordar, 1 atm es aproximadamente igual a un bar e igual a

la presin que ejerce el peso de una masa de 1 kg sobre 1 cm2 , que a su vez es igual
o
a la presin adicional ejercida por una columna de agua a 10 metros de altura.
o
La palma de una mano tiene un rea de aproximadamente 100 cm2 , luego la
a
fuerza que ejerce la atmsfera sobre la palma extendida es aproximadamente igual a
o
la que ejercer una masa de 100 kg apoyada sobre ella. La fuerza sobre la palma es
a
balanceada por una fuerza igual y contraria aplicada sobre el dorso de la mano.
Considere un tubo de 1 m de largo y seccin transversal A, cerrado por uno de los
o
extremos. Llenemos el tubo con mercurio y vaco
coloquemos el tubo, con el extremo abierto presin 0
hacia abajo, en un recipiente con mercurio.
Observaremos que el nivel de mercurio se sih = 760 mm
tuar a aproximadamente 760 mm del nivel
a
del recipiente (ver gura 12.3). El extremo presin P0
superior del tubo queda al vac
o.
Apliquemos la segunda ley de Newton a la
Hg
columna de mercurio (que sobresale de la supercie del l
quido en el recipiente). Cules
a
Figura 12.3
son las fuerzas que actan sobre ella?
u
Hay slo dos: por una parte est la presin que el uido que est en el recipiente
o
a
o
a
ejerce sobre el mercurio que est en el tubo: tal fuerza es F1 = P0 A; por otra, est el
a
z
a
z
a
peso del mercurio al interior de la columna, F2 = AhHg g. Como el uido est en
reposo la fuerza neta debe ser nula, o sea
P0 A = AhHg g .
La densidad del mercurio es Hg = 13.6 g/cm3 . Con esto obtenemos para P0 el valor
P0 1.014 105 Pa = 1 atm .
La fuerza que eleva al mercurio al interior del tubo es la presin atmosfrica! El
o
e
dispositivo que acabamos de describir es un barmetro de mercurio. La altura de la
o
columna de mercurio mide la presin atmosfrica. La presin atmosfrica promedio a
o
e
o
e
nivel del mar corresponde a 760 mm de mercurio.
Al repetir el mismo experimento, pero con una columna de agua, la altura
ser 13.6 veces mayor (recuerde que la densidad del mercurio es 13.6 g/cm3 y la
a
del agua 1 g/cm3 ). Multiplicando los 76 cm por 13.6 se obtienen 10.34 m. Este dato
es muy importante, ya que interviene en varias aplicaciones tecnolgicas. Por ejemplo,
o
al intentar elevar agua de un pozo (cuya supercie est en contacto con el aire que
a
nos rodea) succionando por el extremo superior de un tubo largo, slo se tendr xito
o
ae
si el nivel de agua no est a ms de 10.34 metros de profundidad (en la prctica
a
a
a

CAP
ITULO 12. FLUIDOS

440

esta altura es menor ya que el agua comienza a hervir bastante antes de llegar a los
10.34 metros).

12.3.

Principio de Arqu
medes

^
A
Al sumergirnos en un uido, la presin
o
z
P0
aumenta. Evaluemos este aumento de presin
o
para un uido incompresible (l
quido) de densidad . para ello consideremos el uido cong

h
tenido en un paralelep
pedo imaginario de altura h y rea A. Una de las caras de rea
a
a
A la ubicamos de manera que coincida con
la supercie del l
quido mientras que la otra
queda a una profundidad h (ver gura 12.4).
Por lo dicho en la seccin anterior, la presin
o
o
P = P (h) es slo una funcin de la profundio
o
Figura 12.4
dad h.
Es claro que las fuerzas netas horizontales ejercidas por el uido externo sobre el
paralelep
pedo son nulas, de lo contrario el uido del paralelep
pedo acelerar lo
a
que estar en contradiccin con la suposicin de que el uido se encuentra en reposo.
a
o
o
Las fuerzas que actan sobre el paralelep
u
pedo en la direccin vertical son: i) la
o
fuerza que el aire ejerce sobre la cara superior, que es F1 = P0 A, ii) la fuerza
z
que el uido (exterior) ejerce sobre la cara inferior, que es F2 = P (h)A y iii) la
z
fuerza debida al peso del paralelep
pedo con su uido. Esta fuerza de gravedad es
F3 = (Ah)g. Como el paralelep
z
pedo est en equilibrio, la fuerza total debe ser
a
nula, es decir,
0 = F1 + F2 + F3 = (P0 A + P (h)A Ahg) .
z
De la ecuacin anterior se deduce que
o
P (h) = P0 + gh ,
donde P0 es la presin atmosfrica que acta sobre la supercie del uido. Observe
o
e
u
que el aumento de la presin con la profundidad es igual a la presin ejercida por el
o
o
peso de la columna del uido que se encuentra por encima.
Estamos en condiciones de demostrar el Principio de Arqumedes:

Al sumergir un cuerpo parcial o totalmente en un uido aparece una fuerza llamada empuje que acta sobre el cuerpo y apunta en la direccin
u
o
opuesta a la gravedad. La magnitud del empuje es Fe = gV , donde es
la densidad del uido y V es el volumen del uido que fue desplazado por
el cuerpo.

CAP
ITULO 12. FLUIDOS
Para demostrar este principio observe primeramente que la fuerza que el l
quido ejerce
^
sobre cada parte de la supercie del cuerpo z
sumergido o parcialmente sumergido es independiente del material de que est hecho.
a
Por lo tanto, en lo que a empuje respecta,
podemos reemplazar la parte sumergida del
cuerpo A por un l
quido igual al l
quido que
lo rodea (ver gura 12.5). Si es la densidad del l
quido y Vs el volumen de la parte sumergida del cuerpo A, entonces el peso
del cuerpo B es W = Vs g. Por supuesz
to que el cuerpo B estar en equilibrio, por
a
consiguiente la fuerza de empuje que el l
quido exterior ejerce sobre B debe exactamente
contrarrestar el peso. Luego la fuerza de empuje es Fe = Vs g.
z

441

Figura 12.5

Ms an, el cuerpo B est en equilibrio neutro (es decir, dentro del l


a u
a
quido lo
podemos trasladar a cualquier punto y orientarlo en cualquier direccin, quedando en
o
reposo), luego la fuerza de empuje debe estar actuando como si estuviera aplicada en
el centro de gravedad de B. Esto es un dato de importancia para analizar el equilibrio
de objetos otantes o sumergidos.
Ejemplo: Considere tres cubos del mismo tamao, adheridos tal como se muestra en
n
la gura 12.6. El material del cual estn hea
chos los dos cubos A y B es 1 = 0.5 g/cm3 ,
mientras que el cubo C est hecho de un maa
terial de densidad 2 = 2 g/cm3 . Observe que
la densidad media de los tres cubos es igual
a la del agua ( = 1 g/cm3 ) y, por lo tanto,
al sumergirlo en agua, la fuerza de empuje
exactamente cancela el peso. Cul ser la
a
a
orientacin de equilibrio estable que el objeo
to adquirir cuando est otando rodeado
a
a
de agua?

B
A
C

Figura 12.6

Las unicas fuerzas que estn actuando sobre el objeto son el peso W y el empuje

a
Fe . Ya sabemos que ambas fuerzas tienen la misma magnitud y apuntan en direcciones
opuestas y, por lo tanto, la fuerza neta sobre el objeto es nula. Pero para que se
encuentre en equilibrio tambin el torque neto debe ser nulo. Esto se logra slo si
e
o
ambas fuerzas son colineales (actan a lo largo de la misma recta). Encontremos los
u

CAP
ITULO 12. FLUIDOS

442

puntos en que actan las dos fuerzas.


u
La gravedad acta en el centro de masas.
u
El centro de masas de los cubos A y B se
encuentra en a y el centro de masas de C se
encuentra en b. El centro de masas del objeto completo se encontrar sobre la recta que
a
une a con b. Como el cubo C tiene el doble
de masa de los dos cubos A + B juntos, el
centro de masas del objeto completo se ubicar ms cerca de b que de a. En la gura 12.7
a a
hemos designado el centro de masas del objeto completo con el nmero 1. Se tiene que
u
b, 1 = a, b/3.
La fuerza de empuje, por otra parte,
acta en el centro de masas que se obtiene
u
al sustituir los tres cubos por agua (en la gura lo hemos designado con el nmero 2).
u

A
2

1
C

b
L
Figura 12.7

Nuevamente el centro de masas de los cubos A + B se encuentra en a, mientras


que el de C se encuentra en b. El centro de masas de los centros de masas nuevamente
se encontrar sobre la recta a, b. Pero ahora los cubos A + B pesan el doble de lo que
a
pesa C, luego el centro de masas ahora estar ms cerca de a que de b. De hecho, el
a a
centro de masas cuando los tres cubos estn hechos de agua debe estar sobre el plano
a
de simetr indicado en la gura con una l
a
nea punteada.
En resumen, la fuerza de gravedad acta en 1 y el empuje acta en 2. Para que
u
u
no haya torque sobre el sistema la recta a, b debe orientarse a lo largo de la vertical.
Concluimos que el ngulo de la gura 12.6 debe coincidir con el de la gura 12.7. Se
a
deduce inmediatamente que tan = 1/2. Convnzase de que el equilibrio es estable
e
cuando el punto 2 est sobre el punto 1 e inestable cuando 1 est sobre 2.
a
a

12.4.

La frmula baromtrica
o
e

Considere N molculas de un gas connadas en un volumen V y a una temperae


tura T . Si la ecuacin de los gases ideales es aplicable se tiene que
o
P V = N kB T .
Aqu P es la presin del gas y kB = 1.38 1016 erg/K es la constante de Boltzmann.

o
Sea m la masa de cada molcula, entonces
e
kB T
N m kB T
=
,
V m
m
donde es la densidad de masa del gas. De esta relacin se deduce que, mientras
o
la temperatura se mantenga constante, la presin de un gas es proporcional a su
o
P =

CAP
ITULO 12. FLUIDOS

443

densidad. En particular, si 0 y P0 son la densidad y presin de la atmsfera al nivel


o
o
del mar (z = 0) y (z)y P (z) son las mismas magnitudes, pero a una altura z (por
sobre el nivel del mar), entonces
P0
0
=
.
P (z)
(z)
Por otra parte (ver gura 12.8), la presin
o
a una altura z es la misma que la que hay a
una altura z + dz ms la presin ejercida por
a
o
el peso del gas que hay entre las alturas z y
z + dz, o sea,
P (z) = P (z + dz) + (z)g dz .

^
z
z+dz
g

Esta ecuacin se puede reescribir de la forma


o
g0
dP
= (z)g =
P (z) .
(12.1)
dz
P0
Figura 12.8

Esta es la ecuacin diferencial que gobierna el comportamiento de la presin


o
o
atmosfrica (a temperatura constante). Para resolver esta ecuacin debemos antes
e
o
discutir la funcin exponencial.
o

La funcin exponencial
o
La ecuacin diferencial del tipo
o
df (t)
= f (t) ,
f(t) =
dt

(12.2)

donde es una constante (real o compleja), aparece frecuentemente en las ciencias naturales (y tambin en las ciencias econmicas). Es muy importante discutir y analizar
e
o
sus soluciones.
Una ecuacin diferencial es una ecuacin que involucra una funcin y sus deo
o
o
rivadas (primera, segunda, etc.). La derivada de ms alto orden que aparece en la
a
ecuacin dene el orden de la ecuacin diferencial. La ecuacin diferencial (12.2) es
o
o
o
de primer orden.
Nos interesa encontrar la solucin ms general de (12.2). Un resultado importante
o
a
de la teor de ecuaciones diferencial (y que no demostraremos aqu es que la solucin
a
)
o
general de una ecuacin diferencial de orden n tiene n constantes arbitrarias. En otras
o
palabras, sabremos que tenemos la solucin general de la ecuacin (12.2) si sta tiene
o
o
e
una constante que se puede elegir arbitrariamente. Una vez que se ha encontrado la
solucin general, la constante arbitraria se elige de manera que la solucin corresponda
o
o
a la solucin del problema planteado (o sea, cumpla con las condiciones iniciales).
o

CAP
ITULO 12. FLUIDOS

444

Ejemplo: Consideremos la ecuacin diferencial z = a0 . Esta es una ecuacin difeo

o
2 /2. La solucin
rencial de segundo orden. La solucin general es z(t) = z0 + v0 t + a0 t
o
o
general tiene dos constantes arbitrarias z0 y v0 , las que deben elegirse de manera que
la solucin corresponda a la situacin f
o
o sica concreta que se est considerando.
a
Denamos la funcin exp(t) mediante la serie
o
exp(t) = 1 +

t2 t3
t
+ + + .
1! 2! 3!

(12.3)

Es evidente que su derivada es igual a la funcin, es decir,


o
d
exp(t) = exp(t) .
dt
Ejercicio: Demuestre que la funcin f (t) = A exp(t), donde A es una constante
o
arbitraria, es la solucin general de la ecuacin
o
o
f(t) = f (t) .
Como consecuencia del ejercicio anterior concluimos que la solucin general de la
o
ecuacin (12.1) es
o
g0
z ,
P (z) = A exp
P0
donde la constante arbitraria A se determina exigiendo que la presin en z = 0 sea P0 .
o
Esto nos da la condicin A = P0 . De esta manera obtenemos la frmula baromtrica
o
o
e
P (z) = P0 exp

g0
z
P0

Reiteramos que este resultado, que nos da la presin baromtrica en funcin de la


o
e
o
altura, es slo aproximadamente correcto ya que, contrariamente a nuestra suposicin,
o
o
la temperatura de la atmsfera normalmente disminuye a medida que uno se eleva.
o
Ejercicio: Demuestre que la funcin f (t) = exp(1 t) exp(2 t) es una solucin de la
o
o
ecuacin diferencial
o
f(t) = (1 + 2 )f (t) .
Por consiguiente, f (t) = exp(1 t) exp(2 t) debe poder escribirse de la forma
f (t) = A exp((1 + 2 )t). Demuestre que en ese caso A = 1, o sea
exp(1 t) exp(2 t) = exp((1 + 2 )t) .
Observe que esta relacin justica la introduccin de la notacin
o
o
o
exp(t) = et .

(12.4)

CAP
ITULO 12. FLUIDOS

445

La funcin et = exp(t) se llama funcin exponencial.


o
o
Ejercicio: Evaluando la serie (12.3) para t = 1, demuestre que e = 2.718 . . .
Problemas (relacionados con la funcin exponencial)
o
1. Suponiendo que la atmsfera tiene una temperatura constante, determine la
o
presin atmosfrica a 10 km de altura. (La densidad del aire, en la vecindad de
o
e
la supercie terrestre, a 20 C, es aproximadamente 0 = 1.29 kg/m3 .)
2. Considere un cilindro de radio R sobre el cual se apoya una cuerda. Sea e el
coeciente de roce esttico entre la cuerda y el cilindro. Suponga que en uno de
a
los extremos de la cuerda est colgando una masa M . Cul es la m
a
a
nima masa
que debe colgarse en el otro extremo para que la cuerda no resbale?
Respuesta: m = M ee .
3. La cantidad de ncleos de un elemento radiactivo que decae en un intervalo
u
[t, t ] es proporcional al nmero de ncleos no deca
u
u
dos que se ten inicialmente
a
(en el instante t). Demuestre que la armacin anterior implica que
o
N (t) = N0 et ,
donde N (t) es el nmero de ncleos en el instante t que no ha deca
u
u
do, N0 la
misma magnitud pero en el instante t = 0 y es una constante positiva (la
as llamada constante de desintegracin.)

o
Para el caso en que = 0.01 s1 , determine el tiempo que debe transcurrir para
que decaiga la mitad de los ncleos.
u
4. Suponga que cierto banco (en el pa de las maravillas) para intereses a una
s
tasa de 100 % anual sobre los depsitos, y ms an, los paga en forma continua,
o
a u
sumando los intereses al capital depositado. Si una persona deposita $1000,
cunto le devolver el banco al cabo de un ao?
a
a
n
Respuesta: $ 2 718.28. . . = e 1000.

CAP
ITULO 12. FLUIDOS

12.5.

446

Tensin supercial
o

Entre dos molculas de un uido actan fuerzas. Estas fuerzas, llamadas fuerzas
e
u
de van der Waals o fuerzas cohesivas son de origen elctrico. Una de las caracter
e
sticas
de estas fuerzas es que su alcance es muy pequeo (rpidamente se desvanecen cuando
n
a
la distancia entre las molculas es dos o tres veces su tamao); otra caracter
e
n
stica es
que mientras las molculas no se traslapan, la fuerza es atractiva.
e
El efecto neto de las fuerzas de cohesin
o
sobre una molcula que est en el interior del
e
a
l
quido es nulo, pero no as para una molcu
e
la que se encuentra en la supercie (ver gura 12.9). Para poner una molcula en la sue
percie hay que realizar un trabajo. O sea, la
existencia de una supercie en un uido introduce una energ potencial. Esta energ
a
a
es proporcional a la supercie y se tiene que

Figura 12.9

dW = dA .
Aqu es una constante que depende del uido y se llama tensin supercial

o
y dA es un elemento (innitesimal) de supercie. En realidad la tensin supercial
o
depende de las dos substancias que estn en contacto. La siguiente tabla da valores
a
de la tensin supercial para algunos casos.
o
Substancia
Agua
Agua
Agua
Hg
Hg
Alcohol met
lico
Glicerol C3 H8 O3
Solucin jabonosa
o

En contacto con
aire
aire
aire
vac
o
aire
aire
aire
aire

Para medir la tensin supercial se pueo


de usar el dispositivo mostrado en la gura 12.10. Un alambre movible, inicialmente
sumergido, se tira lentamente, extrayndolo
e
del l
quido (con una pel
cula del l
quido adosada). Midiendo la fuerza F se puede deducir
. En efecto, al mover el alambre movible a
una altura h a h+dh, el trabajo que se realiza
es dW = F dh.

Temp. C
0
20
80
20
20
20
20
20

[N/m]
0.0756
0.07275
0.0626
0.475
0.436
0.0227
0.0634
0.025
F

Figura 12.10

CAP
ITULO 12. FLUIDOS

447

Por otra parte, la supercie de la pel


cula aumenta en dA = 2L dh (el factor 2 se
debe a que la pel
cula tiene una supercie a cada lado). Se tiene
=

dW
F dh
F
=
=
.
dA
2L dh
2L

Problema: Deseamos encontrar la diferencia de presin entre el interior y exterior


o
de una pompa de jabn de radio R = 1 cm.
o
Si, soplando con una pajita, aumentamos el radio de la pompa de R a R + dR,
entonces la supercie aumenta en
dA = 2 (4(R + dr)2 4R2 ) = 16R dR .
El factor 2 nuevamente se debe a que hay que considerar tanto la supercie interior
como exterior de la pompa. El cambio de energ debido al aumento de la supercie
a
es por lo tanto
dW = dA = 16R dR .
Por otra parte, podemos evaluar el trabajo directamente, multiplicando el desplazamiento dR por la fuerza P 4R2 , es decir,
dW = P 4R2 dR .
Igualando las dos ultimas expresiones se encuentra la diferencia de presin

o
P =

4
.
R

Con = 0.025 N/m y R = 0.01 m se obtiene P = 10 N/m2 . Si se deja de soplar


por la pajita, la pompa se desina.
Observe que la presin al interior de una pompa de jabn es mayor tanto ms
o
o
a
pequeo es su radio. De esta observacin se deduce que al juntarse una pompa de
n
o
jabn grande con una pequea, la pequea inar a la ms grande. De esta manera
o
n
n
a
a
la pompa grande aumentar su tamao mientras que la pequea disminuir: en otras
a
n
n
a
palabras, la ms grande absorber a la ms pequea.
a
a
a
n

CAP
ITULO 12. FLUIDOS

12.6.

448

Capilaridad

La fuerza entre molculas de dos subse

tancias distintas se llama fuerza de adhesin.


o

Consideremos una pequea cantidad de l


n
quido (medio #2) en contacto con una supercie
slida plana (medio #3) y ambos en contaco
to con un gas (medio #1) (ver gura 12.11).
Sea {i,j }, con i, j = 1, 2, 3 las tensiones superciales para las distintas interfases de la
gura 12.11.

Si la fuerza de adhesin (entre el l


o
quido
y el slido) es mucho mayor que la fuerza de
o
cohesin (entre las molculas del l
o
e
quido), entonces el l
quido tender a esparcirse sobre el
a
slido (ver gura 12.11a). En este caso se dice
o
que el l
quido moja al slido.
o
Figura 12.11
Por otra parte, si la fuerza de adhesin es mucho menor que la fuerza de cohesin,
o
o
entonces el l
quido tender a concentrarse, adquiriendo una forma compacta tipo gota
a
(ver gura 12.11b).
Como resultado de esta competencia entre las distintas fuerzas de adhesin y
o
cohesin, se forma un ngulo de contacto bien caracter
o
a
stico entre el l
quido y el slio
do. Experimentalmente se determina que este ngulo de contacto para las substancias
a
aguavidrio es aproximadamente 0 , mientras que para mercuriovidrio = 140 .
Considere la l
nea a lo largo de la cual conviven las tres fases. Conocemos la
magnitud y la direccin de la fuerza sobre proveniente de la tensin supercial del
o
o
l
quido. Por el principio de accin y reaccin, el slido ejercer sobre el l
o
o
o
a
quido una
fuerza de la misma magnitud pero en direccin opuesta. Esta fuerza es la que hace
o
subir un uido por un capilar.
Consideremos un tubo jo, de dimetro
a
interior muy pequeo 2r y con un extremo
n
inmerso verticalmente en un l
quido cuya tensin supercial es . El largo de la l
o
nea en
este caso es 2r. La fuerza que el tubo ejerce
sobre el l
quido a travs de la tensin supere
o
cial es
F = (2r) cos ,
donde es el ngulo de contacto del l
a
quido con el material del tubo. Esta fuerza debe compensarse exactamente con el peso del
l
quido (que est por sobre el nivel exterior).
a

2r

Figura 12.12

CAP
ITULO 12. FLUIDOS

449

El peso del l
quido que subi por el tubo capilar es
o
Fg = 0 (r 2 h)g ,
donde 0 es la densidad del l
quido. Igualando las dos fuerzas se obtiene para la altura
mxima h a la que sube el l
a
quido la expresin
o
2 cos
h=
.
0 gr
Ejemplo: Los xilemas que trasportan los nutrientes en una plante t
picamente tienen
3 cm. Evaluemos la altura mxima a la que podrn llegar los nutrientes.
un radio de 10
a
a
Supondremos que el ngulo de contacto = 0 y para la densidad y tensin supercial
a
o
del l
quido usaremos la del agua.
Usando la frmula expuesta ms arriba se encuentra que h 1.5 m. La capilario
a
dad es efectivamente uno de los mecanismos que las plantas usan para elevar la savia,
sin embargo, no puede ser el mecanismo responsable para elevar el agua de las ra
ces
hasta la punta de los rboles grandes (cuya altura puede superar los 100 metros), ya
a
que para ello los xilemas tendr que tener un dimetro 100 veces menor.
an
a

12.7.

Fluidos en movimiento

Consideraciones preliminares
Los uidos en movimiento se pueden clasicar con respecto a varios aspectos.
Uno de ellos es la compresibilidad. La hidrodinmica se preocupa de estudiar el ujo
a
de uidos incompresibles, mientras que la aerodinmica analiza los ujos de uidos
a
compresibles. Notamos, sin embargo, que incluso los gases pueden aproximadamente
como incompresibles mientras su velocidad no supere a la tercera parte de la velocidad
del sonido.
Otro aspecto clasicatorio se introduce respecto al roce interno. Se tiene el ujo
de un uido ideal si se ignoran todos los efectos debido al roce interno (es decir, se
ignora la viscosidad del uido). En caso contrario se estar considerando ujos de
a
lquidos y gases reales.

La trayectoria de un pequeo elemento de


n
uido dene una lnea de corriente o lnea de

ujo. A su vez todo un haz de l


neas de ujo dene un tubo de ujo (ver gura 12.13).
Tambin podemos clasicar los uidos en moe
vimiento con respecto al comportamiento de
sus l
neas de corriente. Si stas no var a
e
an
medida que transcurre el tiempo se tiene un
ujo estacionario o ujo laminar ; en caso
contrario, el ujo es turbulento.

2
1

A2
v2

A1
v1

Figura 12.13

CAP
ITULO 12. FLUIDOS

450

En un ujo laminar, dos l


neas de corriente cercanas entre s en cierto lugar,

se mantendrn cercanas en todas partes. Tambin dos l


a
e
neas de corriente del uido
nunca se cruzan.
Cuando el ujo es turbulento entonces elementos de uido que inicialmente estn
a
innitesimalmente cerca pueden llegar a estar separados por distancias macroscpicas
o
a medida que transcurre el tiempo. El ujo del uido en este caso es catico y se
o
forman remolinos errticos (llamadas tambin corrientes parsitas).
a
e
a

Flujo turbulento

Flujo laminar

Figura 12.14
La disipacin de energ es mucho mayor cuando el ujo es turbulento que cuando
o
a
es laminar.
Ecuacin de continuidad
o
Consideremos un tubo de ujo como, por ejemplo, el que se muestra en la gura 12.13. Sean A1 , 1 y v1 el rea transversal del tubo, la densidad y velocidad del
a
uido en la entrada del tubo y A2 , 2 y v2 las mismas magnitudes pero a la salida del
tubo. Para un ujo estacionario, la cantidad de uido que ingresa por el tubo durante
un intervalo de tiempo dt debe coincidir con la que emerge en ese mismo intervalo
por el otro extremo, luego
1 A1 v1 dt = 2 A2 v2 dt ,
relacin a la que se denomina ecuacin de continuidad. Cuando el ujo es incompreo
o
sible, la densidad no cambia (o sea, 1 = 2 ), luego, para uidos incompresibles, la
ecuacin de continuidad es
o
A1 v1 = A2 v2 .
Ecuacin de Bernoulli
o
En lo que sigue consideraremos el ujo estacionario de un uido ideal incompresible. Sean P1 y P2 las presiones a la entrada y salida de un tubo de ujo, respectivamente. Evaluemos el trabajo neto en el punto de entrada realizado por la presin
o

CAP
ITULO 12. FLUIDOS

451

sobre el uido que est al interior del tubo. En un tiempo dt la seccin transversal
a
o
inicial avanza una distancia v1 dt, siendo el trabajo sobre el uido
W1 = F1 v1 dt = P1 A1 v1 dt .
Por otra parte, el uido que emerge del tubo realiza un trabajo igual a
W2 = F2 v2 dt = P2 A2 v2 dt .
La diferencia es el trabajo neto realizado sobre el uido:
dW = W1 W2 = (P1 A1 v1 P2 A2 v2 ) dt .
Este trabajo neto hecho sobre el uido debe ser igual al cambio de energ (potencial
a
y cintica) del uido:
e
dW = dU + dK .
Si z1 es la altura del uido a la entrada del tubo y z2 la altura a la salida, el cambio
de energ potencial es
a
dU = (A2 v2 dt)z2 g (A1 v1 dt)z1 g .
El cambio de energ cintica es
a
e
1
1
2
2
dK = (A2 v2 dt)v2 (A1 v1 dt)v1 .
2
2
De las ecuaciones anteriores se obtiene
(P1 A1 v1 P2 A2 v2 ) dt = [(A2 v2 dt)z2 g (A1 v1 dt)z1 g]
1
1
2
2
+ (A2 v2 dt)v2 (A1 v1 dt)v1 .
2
2
Usando la ecuacin de continuidad, se encuentra
o
1
2
2
P1 P2 = g(z2 z1 ) (v2 v1 ) ,
2
o sea, para cualquier punto a lo largo de un tubo de ujo,
1
P + gz + v 2 = constante .
2
Esta ultima relacin, consecuencia directa del teorema de conservacin de la energ

o
o
a,
se conoce con el nombre de ecuacin de Bernoulli . Es importante recalcar que la
o
ecuacin de Bernoulli recin deducida es slo vlida para uidos ideales, o sea aplicable
o
e
o
a
slo a situaciones en las cuales la viscosidad es despreciable.
o

CAP
ITULO 12. FLUIDOS

12.8.

452

Aplicaciones del Principio de Bernoulli

Supondremos impl
citamente que en todos los casos analizados en la presente
seccin que el uido bajo consideracin es ideal y que el ujo es estacionario. En la
o
o
prctica los resultados obtenidos aqu sern slo una primera aproximacin al problea

a o
o
ma estudiado. Para una descripcin ms precisa es necesario incluir en el formalismo
o
a
los efectos introducidos por la viscosidad.
Problema 1: Un tambor de altura h y rea
a
A, parado y abierto por la tapa superior (es
decir, en contacto con la atmsfera), se eno
cuentra lleno de agua. Asuma que en la parte
inferior del manto se abre un tapn de seccin
o
o
transversal a. Cunto tiempo tardar en vaa
a
ciarse el tambor?

A
1

Solucin: Apliquemos la ecuacin de Bero


o
noulli en los puntos 1 y 2, en la parte superior del uido en el tambor y una vez que ha
emergido del tambor (gura 12.15). En ambos lugares la presin del uido es igual a la
o
presin atmosfrica P0 .
o
e

Figura 12.15

Elijamos el origen del eje vertical en la base del tambor. De acuerdo a la ecuacin
o
de Bernoulli se tiene
1
P0 + gh + 0 = P0 + 0 + v 2 ,
2
donde v es la velocidad del uido a la salida del tambor. La velocidad, por lo tanto,
es
v = 2gh .
Esta ultima relacin se llama teorema de Torricelli . Observe que la velocidad del

o
uido es la misma que la que adquiere un objeto cuando cae una distancia h.
Supongamos ahora que en cierto instante el uido dentro del tambor est a una
a
altura z. El volumen de uido que emerge en un tiempo dt es av dt, lo que hace bajar
el nivel del tambor en dz = av dt/A. Tenemos que

dz
a
a
= v=
dt
A
A

2gz ,

o, escribindolo de otra forma,


e
dz
a
=
z
A

2g dt .

CAP
ITULO 12. FLUIDOS

453

Integrando la ultima ecuacin desde que se comienza a evacuar el tambor hasta que

o
est vac se obtiene:
e
o,
t=T
dz
a
=
dt
2g
A
z
t=0
z=h
0
T
a
2z 1/2
=
2gt
A
h
0

a
2 h=
2gT .
A
z=0

El tiempo que demora en evacuarse el tambor es


T =

2A
a

Problema 2: Considere un sifn consisteno


te de un tubo con un dimetro constante de
a
10 cm, con el cual se extrae agua de una represa. Con las alturas mostradas en la gura 12.16, evale el ujo que pasa por el tubo.
u

h
.
2g
4
^
z
h 1= 2 m
h 2= 6 m

1
3

Solucin: Apliquemos la ecuacin de Bero


o
noulli en los puntos 1 y 2. Se tiene que
1
P0 + g(h2 h1 ) + 0 = P0 + 0 + v 2 ,
2

agua

Figura 12.16

donde v es la velocidad del agua al interior del tubo. Como el uido es incompresible
y el dimetro del tubo no cambia, la velocidad para un uido ideal al interior del
a
tubo en todos los lugares es la misma. Para la velocidad v se obtiene
v=

2g(h2 h1 ) .

El volumen de agua que pasa por el tubo en un tiempo dt es


dV = Av dt ,
donde A es la seccin transversal del tubo. Sustituyendo los valores del enunciado se
o
obtiene

dV
= (0.05)2 2 9.81 4 m3 /s 70 litros/s .
dt
Cul es la presin en el punto 3 (al interior del tubo, a la altura del nivel de
a
o
agua del tranque)?

CAP
ITULO 12. FLUIDOS

454

Para responder esta interrogante aplicamos la ecuacin de Bernoulli en los puntos


o
2 y 3. Tenemos
1
1
P0 + 0 + v 2 = P3 + g(h2 h1 ) + v 2 .
2
2
Ac P3 es la presin del agua en el punto 3. Se obtiene
a
o
P3 = P0 g(h2 h1 ) .
Una columna de agua de 10 metros corresponde a aproximadamente la presin ato
mosfrica P0 . Por lo tanto, g(h2 h1 ) = 0.4P0 . Luego P3 0.6P0 .
e
Anlogamente, para la presin en el punto 4 se obtiene
a
o
P4 = P0 gh2 0.4P0 .
Observe que h2 no puede sobrepasar los 10 metros, ya que de lo contrario la columna
de agua se corta.
Otras aplicaciones
i) Atomizador:
Al pasar una corriente de aire por encima de un tubo abierto, se reduce la presin al interior del tubo. Si la velocidad
o
del aire es v, la presin P justo encima del
o
tubo es
1
P = P0 v 2 .
2
La disminucin de presin provoca que el
o
o
l
quido suba por el tubo. Una vez que el
l
quido llega a estar en contacto con la corriente de aire, ste se atomiza. Este prine
cipio es usado en las botellas de perfume
y en los aspersores de pintura.

aire
P0

Figura 12.17

ii) Tubo de Venturi:


Al hacer pasar un l
quido por una tuber estrechada, en el lugar constreido
a
n
baja la presin. La disminucin de la preo
o
sin permite determinar la velocidad del
o
uido.
Apliquemos la ecuacin de Bernoulli
o
en los puntos 1 y 2 (gura 12.18).

P1

P2
2

A1

A2

Figura 12.18

CAP
ITULO 12. FLUIDOS

455

Si la tuber es horizontal (o sea, no hay cambios en la energ potencial del


a
a
uido) se tiene que
1 2
1 2
P1 + v1 = P2 + v2 .
2
2
Por otra parte, la ecuacin de continuidad nos da la relacin
o
o
A1 v1 = A2 v2 .
De las ecuaciones anteriores se deduce que
v2 = A1

2(P1 P2 )
.
(A2 A2 )
1
2

Si el ujo es sucientemente alto, el tubo de Venturi puede usarse para bombear.


Por ejemplo, los extractores de saliva usados por los dentistas se basan en este
principio.
iii) Efecto Magnus:
Consideremos un cilindro (o una esfera) en un uido en movimiento. Si el cilindro rota en torno a un eje perpendicular a
la corriente del uido, y adems hay roce
a
viscoso entre el cilindro y el uido, entonces el cilindro arrastrar al uido haciendo
a
que las velocidades del uido a ambos lados del cilindro no sean iguales. En el caso
mostrado en la gura adjunta, la velocidad
es mayor arriba que abajo.

F
1

Figura 12.19

De acuerdo a la ecuacin de Bernoulli, la presin en el lugar 1 ser inferior que


o
o
a
en el lado 2 (P1 < P2 ). Esta diferencia de presin genera una fuerza neta sobre
o
el cilindro hacia arriba.
Es este efecto, llamado efecto Magnus, el responsable de los as llamados efec
tos que pueden observarse en numerosos juegos de pelota. Justamente para
aumentar el efecto las pelotas no deben ser completamente suaves en la supercie (pelusas en la pelota de tenis).

CAP
ITULO 12. FLUIDOS

456

iv) Bomba de chorro (jet) de agua.


Por una tobera inyectora P se hace ingresar agua a alta velocidad en una cmaa
ra. De esta manera se genera una disminucin de la presin en la vecindad de P ,
o
o
lo que a su vez permite aspirar el aire de
un recipiente. El l
mite inferior a que puede bombear este dispositivo (usando agua
y a temperatura ambiente) es de aproximadamente P 2.7 104 Pa (la 1/40 ava
parte de la presin atmosfrica).
o
e

agua

aire
P

Figura 12.20

12.9.

Viscosidad

Entre las distintas molculas de un uido actan fuerzas de adhesin. Por esta
e
u
o
razn, cuando uyen y distintas partes del uido se mueven con velocidades relativas,
o
aparecen fuerzas de roce interno, tambin llamada viscosidad. A pesar de que los uie
dos no maniestan resistencia a fuerzas de cizalla, la viscosidad hace que s presenten

cierta resistencia al deslizamiento.


Otra consecuencia de la viscosidad es que la velocidad del uido que est en
a
contacto con una supercie (de un slido) es nula (con respecto a la supercie).
o
En esta seccin slo analizaremos casos en que el ujo es laminar.
o o
Consideremos dos placas paralelas
de rea A, separadas por una distana
cia D y con un uido entre ellas. Una
de las placas la mantenemos ja y la
otra se mueve (paralelamente) con velocidad v0 (ver gura 12.21). El uido en contacto con la placa superior se
mueve con velocidad v0 , mientras que
el que est en contacto con la placa ina
ferior est en reposo.
a

^
z

rea A
v0

Fr

en reposo

Figura 12.21
Newton experimentalmente encontr que para muchos uidos la fuerza que se
o
debe realizar para mantener la placa en movimiento es
Fr = A

dv
v0
= A
,
D
dz

o sea, es proporcional al rea A y al gradiente (derivada) de la velocidad. La consa


tante de proporcionalidad es la viscosidad dinmica. Los uidos que cumplen con
a

CAP
ITULO 12. FLUIDOS

457

esta relacin se llaman uidos newtonianos. La siguiente tabla da la viscosidad para


o
algunas substancias:
Fluido
Agua
Agua
Agua
Alcohol et
lico
Glicerina
Glicerina
Aire
Aire
Aire
Helio

Temp. C
0
20
100
20
0
20
-31.6
20
230
20

viscosidad [Ns/m2 ]
1.79 103
1.00 103
0.28 103
1.2 103
12.11
1.49
1.54 105
1.83 105
2.64 105
1.94 105

(Otra unidad usada para medir la viscosidad es el poise [P]: 1 [P] = 10 [Ns/m2 ].)
De la tabla se observa que la viscosidad es mucho mayor para los l
quidos que para
los gases. Tambin se observa una fuerte dependencia de la temperatura. Para los
e
l
quidos la viscosidad disminuye al aumentar la temperatura, mientras que para los
gases aumenta.
Flujo laminar en tubos
El efecto de la viscosidad en el ujo de uidos por tubos de seccin redonda
o
es de gran importancia en muchas aplicaciones. Consideremos aqu un caso: el ujo

estacionario de un l
quido newtoniano por un tubo horizontal de largo L y radio R.
Sean P1 y P2 las presiones del l
quido en los dos extremos del tubo y determinemos el
perl de velocidad v(r) del uido al interior del tubo y el ujo por unidad de tiempo.
Sea v(r) la velocidad del uido al interior del tubo. Sabemos que v(R) = 0, o
sea, el uido en contacto con el tubo est en reposo. Consideremos ahora el uido
a
encerrado al interior de un cilindro de radio r (ver gura 12.22). Llamemos A al uido
interior y B al uido que est ubicado a distancia mayores que r. El rea de contacto
a
a
del uido A con B es 2rL.
Fluido A

P
1

Fluido B

P
2

R
L

Figura 12.22

CAP
ITULO 12. FLUIDOS

458

La fuerza que B ejerce sobre A es, por lo tanto,


Fr = (2rL)

dv(r)
x.

dr

Observe que dv/dr es negativo y, por lo tanto, la fuerza que el uido exterior ejerce
sobre A es contraria a la direccin del uido. Como el ujo es estacionario, la fuerza
o
total sobre el uido A debe ser nula, o sea, la fuerza ejercida por las presiones P1
y P2 sobre el cilindro interno debe cancelar exactamente a la fuerza Fr debida a la
viscosidad:
P1 r 2 x P2 r 2 x + Fr = 0 .

De esta manera se deduce que


dv
P1 P2
=
r.
dr
2L
Integrando sobre r y jando la constante de integracin de manera que v(R) = 0 se
o
encuentra el perl de velocidades al interior del tubo (ecuacin de Poiseuille):
o
v(r) =

P1 P2 2
(R r 2 ) .
4L

Este perl es de forma parablica.


o
Conocido el perl de velocidades podemos evaluar el ujo dV /dt (la cantidad
de uido que atraviesa la seccin transversal del tubo por unidad de tiempo). La
o
cantidad de uido que pasa entre dos cilindros concntricos de radios r y r + dr en
e
un tiempo dt es (2r dr)v(r) dt. Sumando sobre todos los cilindros (integrando sobre
r) se obtiene la cantidad de uido dV que pasa por el tubo en un tiempo dt:
R

(2r dr)v(r) dt .

dV =
0

Se obtiene

dV
P1 P2
= 2
dt
4L

R
0

r(R2 r 2 ) dr =

P1 P2
R4 .
8L

Observe que la cantidad de agua que se puede hacer pasar por un tubo aumenta
dramticamente cuando se aumenta su dimetro. Aumentar la diferencia de presin
a
a
o
en un factor 2 aumenta el ujo en ese mismo factor; aumentar el dimetro en un
a
factor 2 (sin aumentar la diferencia de presin) aumenta el ujo en un factor 16.
o
Tambin podemos escribir la ultima ecuacin como sigue:
e

o
P = P1 P2 =

8L dV
,
R4 dt

o sea, la prdida de presin al pasar un ujo dV /dt por un tubo es proporcional a su


e
o
largo L y a la viscosidad e inversamente proporcional a la cuarta potencia de R.

CAP
ITULO 12. FLUIDOS

459

Flujo laminar alrededor de una esfera


Usando matemticas ms avanzadas se puede evaluar la fuerza de roce Fr debido
a
a
a la viscosidad que acta sobre una esfera de radio R cuando sta se mueve respecto
u
e
a un uido con velocidad v0 . Si el ujo es laminar la fuerza es (ley de Stokes)
Fr = 6rv0 .
Esta ecuacin, midiendo la velocidad terminal de esferas cayendo en el uido, permite
o
determinar su coeciente de viscosidad.

12.10.

Problemas

1. El rey Hiern de Siracusa pidi a Arqu


o
o
medes que examinara una corona maciza
que hab ordenado hacer de oro puro. La corona pesaba 10 kg en el aire y
a
9.375 kg sumergida en agua. Arqu
medes concluy que la corona no era de puro
o
oro. Asumiendo que en su interior conten plata, cunto oro ten la corona
a
a
a
3 ; la de la plata, 10.5 g/cm3 .
de Hiern? La densidad del oro es 19.3 g/cm
o
2. Considere un vaso de agua lleno hasta el borde, con un trozo de hielo otando
en l. Por supuesto que el hielo, al otar, sobrepasar por encima del borde del
e
a
vaso. A medida que el hielo se derrite. Se derramar el vaso?
a
Suponga ahora que en el mismo vaso ota un pequeo barco de juguete hecho
n
de latn. Suponga adems que el barquito tiene un pequeo oricio por el cual
o
a
n
penetra agua, haciendo que el barquito lentamente se llene de agua. Durante
este proceso, o sea mientras el barco se llena de agua pero an no se hunde, el
u
nivel del agua del vaso baja, queda a igual altura o sube? Cuando nalmente
el barquito se hunde, que pasa con el nivel del agua?
3. Considere un cilindro de masa M , rea A y altura h, que ota parado en un
a
l
quido de densidad 0 .
a) Hasta qu altura estar sumergido el
e
a
cilindro en el l
quido?
b) Si el recipiente que contiene el l
quido
es muy grande (por ejemplo, un lago),
qu trabajo debe realizarse para sacar
e
el cilindro del l
quido?
c) Var la respuesta si el recipiente
a
que contiene el l
quido es un tambor
cil
ndrico de rea A0 ?
a

A
h

Figura 12.23

CAP
ITULO 12. FLUIDOS
4. Considere una varilla de madera muy
liviana, de largo L, seccin transversal
o
A y densidad , que se hace otar en el
agua (designe la densidad del agua por
0 ).
a) Convnzase de que no es posible
e
que la varilla ote parada.
b) Para lograr que la varilla ote
parada, agregumosle una masa
e
puntual m en el extremo inferior.
Cul es la m
a
nima masa m que
debe agregarse para lograr el objetivo?

460

Figura 12.24

5. Considere un vaso comunicante de


2 cm2 de seccin transversal que cono
tiene mercurio ( = 13.6 g/cm3 ). A un
lado se echan 360 gramos de glicerina
( = 1.2 g/cm3 ) y en el otro 1/4 de
litro de alcohol ( = 0.8 g/cm3 ). En- glicerina
cuentre el desnivel d que existe entre los
niveles superiores de la glicerina y el alcohol. Haga un grco cualitativo de la
a
presin hidrosttica en funcin de la
o
a
o
profundidad para cada uno de los dos
brazos del vaso comunicante (graque las dos curvas en el mismo grco).
a
6. Considere un cilindro de seccin A y
o
altura h que se encuentra otando en
la interfase de dos uidos de densidades 1 y 2 , respectivamente (1 > 2 ).
Encuentre la densidad del cilindro si
ste se encuentra sumergido en el uie
do 1 en una magnitud d.

alcohol

mercurio

Figura 12.25

Figura 12.26
7. Qu volumen de helio se requiere si debe elevarse un globo con una carga de
e
800 kg (incluido el peso del globo vac
o)? Las densidades del aire y del helio,
a la presin de una atmsfera, son aire = 1.29 kg/m3 y He = 0.18 kg/m3 ,
o
o
respectivamente.

CAP
ITULO 12. FLUIDOS

461

8. Una varilla de largo L y densidad 1 ota en un l


quido de densidad 0 (0 >
1 ). Un extremo de la varilla se amarra a un hilo a una profundidad h (ver
gura adjunta).

aire

a) Encuentre el ngulo .
a

b) Cul es el m
a
nimo valor de h para el cual la varilla se mantiene en
posicin vertical?
o
c) Si A es la seccin transversal de
o
la varilla, encuentre la tensin del
o
hilo.

fluido

hilo

Figura 12.27

9. Considere las tres mediciones mostradas en la gura adjunta:

P1

P2

P3

Figura 12.28
P1 es el peso de un recipiente con agua con un objeto sumergido en l.
e

P2 es el peso cuando el objeto est sumergido en el agua, pero colgado de una


a
cuerda sin que toque el fondo del recipiente.
P3 es el peso del recipiente con agua.
Encuentre la densidad promedio del objeto.

10. En un canal horizontal, de ancho b, uye agua con velocidad v, siendo el nivel
de agua h. Asuma que en cierto lugar el canal se ensancha en una pequea
n
cantidad db. Demuestre que el nivel del agua cambiar en
a
dh =

hv 2
db .
b(gh v 2 )

Note que si v 2 < gh el nivel del agua sube.

CAP
ITULO 12. FLUIDOS
11. Un corcho cil
ndrico de masa m1 y seccin transversal S1 ota en un l
o
quido
de densidad . El corcho est conectaa
do por medio de una cuerda sin masa,
de largo L, a un cilindro de aluminio
de masa m2 y seccin transversal S2 .
o
El cilindro de aluminio puede deslizarse sin roce por un oricio hermtico en
e
el fondo del tiesto. Calcular la profundidad h a la que debe hallarse la base
del corcho para que el sistema de los
dos cilindros est en equilibrio. La pree
sin atmosfrica, juega algn rol?
o
e
u

462

S1
h

m2

S2

Figura 12.29

12. Un prado es regado con un regador hechizo que consiste en una botella plstia
ca, con numerosos agujeros de 1 mm
de dimetro, acostada sobre el prado
a
y conectada aun a manguera. Asuma
que una bomba de agua se encarga de
generar un ujo de agua constante de
0.2 litros por segundo. Cuntos agua
jeros debe tener la botella para que el
agua llegue a mojar el prado a 8 metros de distancia de la botella? Cul
a
es la presin al interior de la manguera
o
si sta tiene una seccin transversal de
e
o
2?
4 cm
13. Un tubo de largo L, lleno de agua, gira en el plano horizontal en torno a un
eje vertical que pasa por uno de sus extremos. En el extremo junto al eje, el
tubo est abierto, coincidiendo por lo
a
tanto la presin del uido con la preo
sin atmosfrica. El tubo gira con veo
e
locidad angular constante . Si en el
otro extremo, en cierto instante, se abre
un pequeo oricio, con qu velocidad
n
e
emerger el agua del tubo? (Especia
que la rapidez y direccin de la velocio
dad.)

m1

Vista lateral

Vista frontal

Figura 12.30

L
Figura 12.31

CAP
ITULO 12. FLUIDOS
14. Para abastecer de agua a
una casa de dos pisos se
recurre a un hidropack.
Este sistema consiste en
una depsito subterrneo,
o
a
una bomba y un cilindro
con agua y aire. La bomba inyecta agua a presin
o
al cilindro, que en su parte superior queda con aire comprimido. Un medidor de presin detiene la
o
bomba cuando la presi n
o
del cilindro alcanza el valor
deseado (el mismo medidor
vuelve a encender la bomba cuando la presin baja
o
de cierto nivel).

463

llave

5m

1m

aire
comprimido

depsito
bomba
Figura 12.32

Si el nivel del agua en el cilindro se sita 1 metro por debajo del suelo, calcule la
u
presin necesaria en el aire comprimido para que una llave de 1 cm2 de seccin,
o
o
a una altura de 5 metros sobre el suelo, entregue un caudal de 12 litros por
minuto. (La seccin transversal del cilindro es grande respecto a la de la llave.)
o
Tambin encuentre la presin del aire al interior del cilindro.
e
o
15. La fuerza de sustentacin de un avin
o
o
moderno es del orden de 1000 N por
metro cuadrado de ala. Suponiendo que
el aire es un uido ideal y que la velocidad del aire por debajo del ala es
de 100 m/s, cul debe ser la velocidad
a
requerida por sobre el ala para tener la
sustentacin deseada? (La densidad del
o
aire es 1.3 kg/m3 .)

Figura 12.33

16. Un bombero lanza agua con su manguera hacia un incendio formando un ngulo
a
de 45 con la horizontal. El agua que emerge del pistn penetra horizontalmente
o
por una ventana del tercer piso que se encuentra a una altura h = 10 metros.
La manguera que transporta el agua desde el carro bomba tiene un dimetro D
a
de 6 cm y concluye en un pistn cuya abertura tiene un dimetro d de 1.5 cm.
o
a
a) Cuntos litros de agua emergen del pistn por minuto?
a
o
b) Cul es la presin P que debe soportar la manguera (en atmsferas)?
a
o
o

CAP
ITULO 12. FLUIDOS

464

17. Considere la tuber que lleva el agua


a
de una represa hacia una turbina. Suponga que la bocatoma se encuentra a
10 metros bajo el nivel de las aguas y
que la turbina se encuentra 80 metros
por debajo de ese nivel. Al inicio, es decir a la salida de la represa, la tuber
a
tiene un dimetro de 40 cm. Suponga
a
que el uido se comporta como un uido ideal.

10 m

80 m

a) Cul es el dimetro mximo que


a
a
a
puede tener la tuber en su exa
tremo inferior para que no se produzcan cortes de la columna de
agua al interior de la tuber
a?

Figura 12.34

b) Cul ser la cantidad de agua que pasar en ese caso por la tuber y
a
a
a
a
cul la velocidad del agua emergente?
a
c) Si el proceso de generacin de energ elctrica usando la presente turo
a e
bina fuese 100 % eciente, cul ser la potencia de esta central? Esto
a
a
corresponde al consumo promedio de cuntas casas?
a
d) Haga un grco cualitativo de la presin al interior de la tuber en funcin
a
o
a
o
de la altura. Cmo cambia esta presin si la seccin de la tuber en el
o
o
o
a,
punto emergente, se disminuye a la mitad? A la centsima parte?
e
18. Considere una tuber de una calefaccin. En el stano su dimetro es de 4.0 cm
a
o
o
a
y en el segundo piso, 5 metros ms arriba, la tuber tiene un dimetro de slo
a
a
a
o
2.6 cm. Si en el stano una bomba se encarga de bombear el agua con una
o
velocidad de 0.5 m/s bajo una presin de 3.0 atmsferas, cul ser la rapidez
o
o
a
a
de ujo y la presin en el segundo piso?
o
19. Suponga que el nivel de un l
quido
(agua) en un tambor tiene una altura h. A una altura b se hace una pequea perforacin lateral que permite
n
o
que el agua emerja horizontalmente. A
qu altura debe hacerse la perforacin
e
o
para que el alcance d del agua se mxia
mo?
Respuesta: b = h/2.

Figura 12.35

CAP
ITULO 12. FLUIDOS

465

20. En un torrente de agua se sumerge un tubo doblado, tal como se muestra en la


gura adjunta. La velocidad de la corriente con respecto al tubo es v = 2.5 m/s.
La parte superior del tubo se encuentra a h0 = 12 cm sobre el nivel del agua
del torrente y tiene un pequeo agujero.
n
A qu altura h subir el chorro de agua que sale por el agujero?
e
a

h
h0
v

Figura 12.36
21. Considere una masa esfrica homognea en equilibrio hidrosttico. Sea RT el
e
e
a
radio y 0 la densidad de masa.
a) Muestre que la presin a una distancia r del centro viene dada por
o
p=

2 2
G(R2 r 2 ) .
3 0

b) Evale la presin al centro de la Tierra. RT = 6.3 108 cm y densidad


u
o
uniforme promedio 0 = 5.5 g/cm3 .
22. En un baln el gas en su interior se encuentra a una presin P . Demuestre que
o
o
la velocidad con que escapa el gas, al abrir la vlvula, es
a
v=

2(P P0 )
,

donde es la densidad del gas y P0 la presin atmosfrica. (Esta ecuacin se


o
e
o
conoce por ley de Bunsen.)
23. Considere una prensa hidrulica (ve gura adjunta). Sean R1 = 25 cm y R2 =
a
150 cm los radios de los mbolos de bombeo y de presin, respectivamente.
e
o

CAP
ITULO 12. FLUIDOS

466

Si de la palanca que acta sobre el mbolo de bombeo se tira con una fuerza
u
e
F1 = 100 [N] (ver gura), qu fuerza ejercer el mbolo de presin sobre el
e
a
e
o
objeto S?

F2

F1

L
mbolo
de presin

vlvula
de retorno

mbolo
de bombeo

vlvula

estanque
de reserva

vlvula

Figura 12.37
24. Se quiere confeccionar aluminio poroso (algo as como queso suizo) que se

mantenga en suspensin en agua. Determine la razn entre el volumen de los


o
o
poros y el volumen del aluminio poroso. (La densidad del aluminio slido es
o
3 .)
= 2700 kg/m
25.

Considere un cuerpo l
quido de densidad
uniforme 0 , que se mantiene unido debido
a la gravedad y que gira con una velocidad
angular 0 . Si bien el cuerpo es esfrico si
e
0 = 0, cuando 0 = 0 (pero no demasiado
grande), el cuerpo adquiere la forma de un
esferoide oblato. Demuestre que si la desviacin de la esfericidad es pequea, entonces
o
n
2
3 0
R1 R2
=
,
R
8 0 G

donde R R1 R2 . Evale (R1 R2 )/R


u
para la Tierra y comprelo con el valor expea
rimental, que es 1/298.4.

R1

O
R2

Figura 12.38

CAP
ITULO 12. FLUIDOS
26.

Considere la situacin mostrada en la guo


ra 12.39. Un cilindro de radio R y largo L
evita que el agua de cierto recipiente se rebase. El cilindro se puede mover libremente.
La densidad del cilindro es tal que, cuando
el agua llega a la parte superior del cilindro,
la posicin del cilindro es la mostrada en la
o
gura. Encuentre la fuerza que ejerce el agua
sobre el cilindro. Encuentre la densidad del
material del que est hecho el cilindro.
a
Respuesta: Fagua

27.

467

= 0 gR L x +
2
2

^
z

cilindro

^
x

R
agua
0

Figura 12.39

3
1 z .

Considere una caja de dimensiones a, b y h,


llena de agua. Todos los lados de la caja estn
a
rmemente unidos entre s excepto uno de
,
los lados laterales (de dimensin bh). Evale
o
u
la magnitud de la fuerza exterior m
nima con
que debe presionarse ese lado contra el resto
de la caja para que el agua no escurra. Si la
fuerza se aplica en un solo lugar, encuentre la
posicin en la que debe aplicarla.
o

agua
F
h
b
a

Figura 12.40

28. Un mol de aire en condiciones normales (a nivel del mar y a 20 C de temperatura) ocupa un volumen de 22.4 litros. Estime la densidad del aire si gran parte
de l est constituido por nitrgeno. (Resp.: 1.28 kg/m3 .)
e
a
o
Cul es el m
a
nimo volumen que debe tener un globo de helio ( = 0.18 kg/m3 )
para levantar un veh
culo de 1200 kg?

29. Dos globos esfricos inados con aire, ambos


e
de radio R, se unen mediante una cuerda de
longitud L. Los dos globos se mantienen bajo
el agua con el punto medio de la cuerda jo
al fondo. Calcular la fuerza de contacto entre
los globos.

L /2

L /2

Figura 12.41

CAP
ITULO 12. FLUIDOS
30. Una varilla yace en el fondo de un recipiente con agua formando un ngulo de
a
60 con la vertical. La varilla es de seccin uniforme y est formada por dos
o
a
pedazos iguales en longitud pero de distinta densidad. La densidad de una de
las porciones de la varilla es la mitad
de la del agua. Determine la densidad
de la otra porcin.
o

468

30 o

Figura 12.42
a

31. Considere un bloque de hielo ( =


920 kg/m3 ) en forma de L, formado
de tres cubos de 25 cm por lado. Mediante un peso se desea sumergir el hielo en agua como se indica en la gura.
Determine la masa del peso y la ubicacin en el hielo donde deber adherirse
o
a
de modo que el hielo se mantenga justo
sumergido lo ms estable posible.
a

Figura 12.43

32. Considere un sistema de vasos comunicantes formado por dos tubos de seccin transversal de 50 cm2 que estn
o
a
unidos por un tubito corto de seccin
o
transversal muy pequea (o sea, para
n
efectos de este problema podemos despreciar la cantidad de uido que se encontrar en el tubito). Inicialmente en
a
este sistema de vasos comunicantes se
encuentran dos litros de agua.

Figura 12.44

hielo

a) Encuentre la altura en que se encontrarn las interfases entre los l


a
quidos y
el aire en cada uno de los tubos si en uno de los tubos se le agregan 2 litros
de un l
quido cuya densidad es = 0.8 g/cm3 .
b) Para la situacin descrita en la parte a), encuentre la presin en el fondo
o
o
de los vasos comunicantes.
c) Encuentre la altura en que se encontrarn las interfases entre los l
a
quidos
y el aire en cada uno de los tubos si en uno de los tubos, en lugar de 2, se
le agregan 3 litros de un l
quido cuya densidad es = 0.8 g/cm3 .

CAP
ITULO 12. FLUIDOS

469

33. Un tubo horizontal por el que uye l


quido de densidad 0 a razn de Q m3 /s,
o
se bifurca en dos ramas en el plano vertical, una superior y otra inferior, de
secciones transversales a1 = a2 = a, abiertas a la atmsfera (ver gura). Si la
o
distancia entre las ramas es h, determinar:
a) Las cantidades q1 y q2 de l
quido (en m3 /s) que uyen por ambas ramas.
b) La condicin que debe cumplir Q para que haya ujo en la rama superior.
o

q1
g

Q
h

q2

Figura 12.45
34. Una gotita de agua de 1 mm de radio se pulveriza en gotitas de 104 mm
de radio. En qu factor aumenta la energ supercial (debido a la tensin
e
a
o
supercial)?
35. La gura 12.46 muestra un tubo de Pitot, instrumento que se usa para medir
la velocidad del aire. Si el l
quido que
indica el nivel es agua y h = 12 cm,
encuentre la velocidad del aire. La densidad del aire es aire = 1.25 kg/m3 .
Respuesta: v0 = 43.4 m/s = 156 km/h.

aire

h
agua

Figura 12.46
36. Considere dos placas planas de vidrio, separadas por una distancia de 0.1 mm,
con un extremo sumergidas en agua en forma vertical. Qu distancia se elee
var el agua entre las placas debido a la capilaridad?
a
37.

Encuentre la velocidad terminal que adquiere una esfera de cobre de 0.5 cm


de dimetro, cuando cae en agua (Cu = 8.92 g/cm3 ). En qu factor disminuye
a
e
la velocidad terminal si el dimetro se achica en un factor 10?
a

CAP
ITULO 12. FLUIDOS
38.

470

Considere un oleoducto de 5 km y 50 cm de dimetro por el cual se desea


a
3 por segundo. Si uno de los extremos est abierto a la presin
bombear 1 m
a
o
atmosfrica, qu presin P1 debe existir en el otro extremo? Suponga que
e
e
o
la densidad del petrleo es = 950 kg/m3 y el coeciente de viscosidad es
o
aproximadamente = 0.2 Pas. Cul es la potencia dW/dt (energ por unidad
a
a
de tiempo) disipada por la friccin interna originada por la viscosidad?
o
Respuesta: P1 7.5 atm; dW/dt 650 kW.

39. Considere una varilla homognea de largo L, seccin transversal A y densidad


e
o
de masa , tocando el fondo de un recipiente que contiene dos l
quidos de densidades 1 y 2 , con 1 < 2 . Las alturas de cada l
quido en el recipiente son h1
y h2 , respectivamente.

h1

h2

Encuentre el ngulo con la horizontal que forma la varilla cuando est en


a
a
equilibrio.
40. Un chorro de agua cil
ndrico cae desde una llave por accin de la gravedad.
o
Considerando un dimetro inicial d0 , y si el chorro cae inicialmente con velocidad
a
v0 , a qu distancia por debajo de la llave el chorro ha disminuido su dimetro
e
a
a un dcimo de su dimetro inicial?
e
a
41. Considere una varilla de seccin transversal constante sumergida en un recio
piente que contiene dos l
quidos, de densidades 1 y 2 , con 1 < 2 . La varilla
est compuesta de dos trozos de longitudes iguales, pero densidades distintas
a
y , con < . La punta inferior de la varilla est en contacto con el
1
a
2
1
2
fondo del recipiente, y la parte superior de la varilla intersecta la interfase entre
ambos l
quidos. Se sabe adems que la varilla tiene un segmento de longitud R
a
contenido en el l
quido de densidad 1 . Determine la longitud L del segmento de
la varilla contenido en el l
quido de densidad 2 , si la varilla est en equilibrio.
a

CAP
ITULO 12. FLUIDOS

471

111111111111111111111111111
000000000000000000000000000
111111111111111111111111111
000000000000000000000000000
111111111111111111111111111
000000000000000000000000000

12.11.

Solucin a algunos de los problemas


o

Solucin al problema 8
o
El largo a de la parte de la varilla
sumergida es a = h/ sen . La fuerza
de empuje se aplica en el lugar a/2 y
la fuerza de gravedad en el lugar L/2
(medidos desde O).
Sea A la seccin transversal de la
o
varilla. Entonces la fuerza de empuje
viene dada por
Fe = 0 Aag = 0 A
z

Fe
aire

rea A
a

h
0

Fg
O

fluido

h
g .
z
sen

La fuerza de gravedad es

Figura 12.47

Fg = 1 LAg .
z

El torque ejercido por ambas fuerzas respecto a O debe ser nulo, o sea,
L
a
Fe cos = Fg cos .
2
2
Simplicando se obtiene
Fe a = Fg L .
Sustituyendo las expresiones par Fe y Fg se deduce que
0 Aa2 g = 1 AL2 g ,
o sea
0

h2
= 1 L 2 .
sen2

hilo

CAP
ITULO 12. FLUIDOS

472

Despejando se encuentra nalmente que


sen =

0 h
.
1 L

Si el lado derecho de la ultima ecuacin es mayor o igual a uno, la varilla se man


o
tendr en posicin vertical. El m
a
o
nimo valor de h para que la varilla est en posicin
e
o
vertical es
1
.
hmin = L
0
La tensin del hilo se obtiene exigiendo que la fuerza total sea nula. De esta
o
manera se obtiene que
h
g 1 LAg
sen
0
0
1 = Mg
1
1
1

T = Fe Fg = 0 A
= ALg1

donde M es la masa de la varilla.


Solucin al problema 16
o
a) Si v es velocidad con que emerge el agua del pistn, la velocidad hacia arriba
la
o
ser v/ 2. El agua alcanza a subir una altura h, luego su velocidad es
a
v = 2 gh = 20 m/s .
La cantidad de agua V que emerge del pistn en t = 60 segundos es
o
V = vt

d
2

1
= 20 60 3.14 (0.015)2 m3 = 212 litros .
4

b) Usemos el teorema de Bernoulli para comparar el ujo del agua justo a la salida
del pistn con el ujo en la manguera justo detrs del pistn. No hay cambio en la
o
a
o
energ potencial. Como la seccin transversal de la manguera es 16 veces mayor
a
o
que la abertura del pistn, la velocidad del agua en la manguera ser 16 veces
o
a
menor que la velocidad emergente v. A la salida del pistn la presin es la presin
o
o
o
atmosfrica, que ignoraremos en el presente clculo, ya que slo estamos interesae
a
o
dos en la presin adicional p que debe soportar la manguera debido al agua que
o
uye en su interior. Se tiene
v
1
p + 0
2
16

1
= 0 v 2 .
2

CAP
ITULO 12. FLUIDOS

473

Ignorando la energ cintica del agua al interior de la manguera (convnzase de


a
e
e
que modica el resultado nal en menos de un 0.5 %), se obtiene
kg
m2
1
1
kg
,
p = 0 v 2 = 1000 3 400 2 = 2 105
2
2
m
s
m s2
lo que corresponde a aproximadamente 2 atmsferas.
o
Solucin al problema 27
o
Elijamos el eje z a lo largo de la vertical, con el origen al fondo de la caja sobre

la tapa mvil. La presin a una altura z es P (z) = 0 g(h z). Dividamos la tapa
o
o
en franjas horizontales de largo b y ancho (altura) dz. La fuerza que ejerce el uido
sobre la franja que est a la altura z es
a
dF = P (z)b dz .
Sumando (integrando) la fuerza que el l
quido ejerce sobre cada una de las franjas se
obtiene la fuerza total
h

P (z)b dz = 0 gb

F =

1
(h z) dz = 0 bgh2 .
2

Para encontrar a qu altura h0 debemos aplicar esta fuerza sobre la tapa, evae
luemos el torque que ejerce el uido sobre la tapa respecto al origen. El torque que
el uido ejerce sobre la franja que est a la altura z es
a
d = zP (z)b dz .
Sumando (integrando) el torque que el l
quido ejerce sobre cada una de las franjas se
obtiene el torque total
h

zP (z)b dz = 0 gb
0

z(h z) dz =

1
0 bgh3 .
6

Para que la tapa est en equilibrio el torque que ejerce la fuerza total externa F debe
e
coincidir en magnitud con , es decir,
F h0 = ,
o sea

1
1
0 bgh2 h0 = 0 bgh3 .
2
6
De esta ecuacin se deduce nalmente que h0 = h/3.
o

CAP
ITULO 12. FLUIDOS

474

Solucin al problema 33
o
La relacin de Bernoulli se puede aplicar entre los puntos A y B1 y tambin entre
o
e
A y B2 . Por transitividad, la relacin de Bernoulli tambin es vlida entre los puntos
o
e
a
B1 y B2 . Se tiene
1 2
1 2
P1 + gh1 + v1 = P2 + gh2 + v2 .
2
2
Pero P1 = P2 = P0 (la presin atmosfrica), h1 = h y h2 = 0, luego
o
e
1 2
1 2
v1 = gh + v2 .
2
2

q1
B1

^
z
g

Q
h

B2

q2
Figura 12.48
Los ujos que circulan por la rama superior e inferior vienen dados por q1 = av1
y q2 = av2 , respectivamente. Tambin se tiene que Q = q1 + q2 . De las relaciones
e
anteriores se deduce que
Q2 2a2 gh
q1 =
2Q
y
Q2 + 2a2 gh
.
q2 =
2Q
Para que circule l
quido por la rama superior se debe tener que
Q > a 2gh .

Cap
tulo 13

Oscilador armnico
o
versin 5 noviembre 2012
o

13.1.

2
La ecuacin diferencial x(t) + 0 x(t) = 0
o

La ecuacin diferencial que gobierna el comportamiento de un oscilador armnico


o
o
simple es
2
x(t) + 0 x(t) = 0 .

(13.1)

Esta es una ecuacin diferencial lineal de segundo orden. Comenzaremos este cap
o
tulo
exponiendo algunos resultados generales relativos a este tipo de ecuaciones, resultados
que sern de gran utilidad para nuestros propsitos.
a
o
Sean x1 (t) y x2 (t) dos soluciones cualesquiera de cierta ecuacin difeo
rencial. Tal ecuacin diferencial es lineal si x1 (t) + x2 (t) tambin es
o
e
solucin, donde y son constantes (reales o complejas) arbitrarias.
o
Ejercicio: Demuestre que la ecuacin diferencial del oscilador armnico es lineal
o
o
El orden de la derivada ms alta da el orden de la ecuacin diferencial. La
a
o
solucin general de una ecuacin diferencial de orden n tiene n constantes
o
o
arbitrarias (que luego deben ser determinadas usando las condiciones de
borde).
La ecuacin diferencial del oscilador armnico es de segundo orden, por lo tanto,
o
o
la solucin general tiene dos constantes arbitrarias. Sean x1 (t) y x2 (t) dos soluciones
o
cualesquiera (distintas) de (13.1). Como la ecuacin diferencial (13.1) es lineal, se tiene
o
que la funcin xg (t) = x1 (t) + x2 (t), con y constantes arbitrarias, tambin es
o
e
solucin. Pero observe que la solucin xg (t) tiene dos constantes arbitrarias y, por
o
o
475


CAP
ITULO 13. OSCILADOR ARMONICO

476

lo tanto, debe ser una solucin general del problema. En otras palabras, todas las
o
posibles soluciones de (13.1) deben ser de la forma xg (t); las distintas soluciones se
diferencian slo por los valores de y .
o
En el lenguaje tcnico se dice que las soluciones de la ecuacin diferencial (13.1)
e
o
forman un espacio vectorial de 2 dimensiones, siendo x1 y x2 dos vectores particulares de ese espacio. Los dos vectores x1 (t) y x2 (t) (si uno de ellos no es mltiplo
u
del otro) forman una base del espacio vectorial. Cualquier otro vector (o sea, solucin
o
de (13.1)) es una combinacin lineal de los vectores base, es decir, es de la forma
o
x1 (t) + x2 (t).
Sabemos que las funciones
x1 (t) = cos(0 t)

(13.2)

x2 (t) = sen(0 t)

(13.3)

y
son dos soluciones particulares de (13.1). Estas dos funciones (y de hecho as se hace

frecuentemente) pueden ser tomadas como los dos vectores base del espacio vectorial
formado por las soluciones de (13.1). Cualquier otra solucin x(t) de la ecuacin
o
o
diferencial del oscilador armnico puede escribirse de la forma
o
x(t) = a cos(0 t) + b sen(0 t) .
Las constantes a y b se determinan a partir de las condiciones iniciales.
Observe que no es necesario elegir las funciones (13.2) y (13.3) como vectores base
del espacio vectorial; de hecho, cualquier otro par de soluciones (mientras una no sea
mltiplo de la otra) tambin habr servido. Lo interesante es que las funciones (13.2)
u
e
a
y (13.3) no son las funciones ms convenientes para usar como base. Existe un par de
a
soluciones de (13.1) que, si se usan como base, simplican notoriamente los clculos.
a
En lo que sigue de esta seccin introduciremos esta nueva base, estudiaremos algunas
o
de sus propiedades y la relacionaremos con la base dada por las funciones (13.2) y
(13.3).

Consideremos la funcin
o
z(t) = et = exp(t) .
Al derivar z(t) dos veces se obtiene
z(t) = et


CAP
ITULO 13. OSCILADOR ARMONICO

477

y
z (t) = 2 et = 2 z(t) .

Observe que esta ultima ecuacin se puede escribir de la forma

o
z (t) 2 z(t) = 0 .

Esta ecuacin es idntica a la del oscilador armnico si se identica


o
e
o
2
2 = 0 ,

lo que es equivalente a la relacin


o
= i0 ,

con i

1. Observe que acabamos de demostrar que las funciones


x1 (t) = ei0 t

(13.4)

x2 (t) = ei0 t

(13.5)

y
son dos soluciones particulares de la ecuacin diferencial del oscilador armnico, o sea,
o
o
de (13.1). Resulta que stas son las funciones ms convenientes para generar todas
e
a
las dems soluciones de (13.1). Cualquier solucin x(t) de (13.1) se puede escribir de
a
o
la forma
x(t) = ei0 t + ei0 t ,
donde las constantes y se determinan a partir de las condiciones iniciales. (Las
constantes y , generalmente, resultan ser nmeros complejos.)
u
Determinemos las relaciones entre las dos bases. Como cos(0 t) es solucin de
o
(13.1) debe poder escribirse de la forma
cos(0 t) = c1 ei0 t + c2 ei0 t .

(13.6)

Determinemos las constantes c1 y c2 . Derivando (13.6) se encuentra que


0 sen(0 t) = i0 c1 ei0 t i0 c2 ei0 t ,
o sea,
sen(0 t) = i c1 ei0 t c2 ei0 t .
Evaluando (13.6) y (13.7) para t = 0 se obtiene
1 = c1 + c2
y
0 = i(c1 c2 ) .

(13.7)


CAP
ITULO 13. OSCILADOR ARMONICO

478

De estas relaciones se deduce que c1 = c2 = 1/2. De esta manera hemos demostrado


que
1 i0 t
cos(0 t) =
e
+ ei0 t
(13.8)
2
y
1 i0 t
e
ei0 t .
(13.9)
sen(0 t) =
2i
Tambin podemos escribir exp(i0 t) y exp(i0 t) en funcin de cos(0 t) y sen(0 t).
e
o
Usando las relaciones anteriores no es dif demostrar que
cil
ei0 t = cos(0 t) + i sen(0 t)

(13.10)

ei0 t = cos(0 t) i sen(0 t) .

(13.11)

y
Por ultimo, sustituyendo en (13.10) 0 t por encontramos una de las ms bellas

a
ecuaciones de la matemtica
a
ei + 1 = 0 ,
relacin que combina de manera simple los ms importantes nmeros de esa ciencia:
o
a
u

0, 1, , e e i = 1.
Ejercicio: Demuestre que el mdulo de los nmeros complejos exp(i0 t) y exp(i0 t)
o
u
es uno, es decir, demuestre que
|ei0 t | = |ei0 t | = 1 .

13.2.

El oscilador armnico simple


o

Cada vez que la ecuacin dinmica de un sistema tiene la forma


o
a
2
x(t) + 0 x(t) = 0 ,

estaremos en presencia de un oscilador armnico.


o
Ejemplo: Consideremos un pndulo de largo R. Elijamos el origen en el punto de
e
suspensin. El momento angular y el torque (en torno al origen) vienen dados por
o


CAP
ITULO 13. OSCILADOR ARMONICO

479

l = mR(R)
y
= Rmg sen .
Por otra parte

dl

=
= mR2 ,
dt
luego

mR2 = Rmg sen .

Esta relacin se puede escribir de la forma


o

m
g sen = 0 .
+
R
Figura 13.1
2
Denotando g/R por 0 y restringindonos a pequeos ngulos, de manera que
e
n a
podamos usar la aproximacin sen , se obtiene
o
2

+ 0 = 0 .

(13.12)

La constante 0 est relacionada con el periodo T del movimiento por la relacin


a
o
0 T = 2.
Conocer la ecuacin dinmica de un sistema permite, en principio, conocer la
o
a
evolucin temporal del mismo. Para encontrar la solucin expl
o
o
cita del problema se
procede generalmente de la siguiente manera: i) se busca la solucin general de la
o
ecuacin dinmica; ii)las constantes arbitrarias de la ecuacin general se determinan
o
a
o
exigiendo que la solucin cumpla con las condiciones de borde (iniciales) del problema.
o
Ilustremos el procedimiento con nuestro ejemplo concreto. Supongamos que en
el instante t = 0 el ngulo y la velocidad angular del pndulo son 0 y , respeca
e
tivamente. Deseamos encontrar una expresin expl
o
cita para (t). Resolveremos este
problema de dos maneras:
a) Sabemos que la solucin general de (13.12) puede escribirse de la forma
o
(t) = a cos(0 t) + b sen(0 t) .
Determinaremos las constantes a y b. Para ello derivemos primero la ultima ecua
cin respecto al tiempo. Se obtiene
o

(t) = a 0 sen(0 t) + b 0 cos(0 t) .


CAP
ITULO 13. OSCILADOR ARMONICO

480

Evaluando las dos ultimas ecuaciones para t = 0, y usando las condiciones iniciales,

se obtiene
(0) = a = 0
y

(0) = b0 = .

La solucin expl
o
cita se obtiene sustituyendo los valores de a y b, que se deducen
de estas relaciones, en la solucin general:
o
(t) = 0 cos(0 t) +

sen(0 t) .
0

b) Como vimos en la seccin anterior, en lugar de cos(0 t) y sen(0 t) tambin poo


e
demos usar las soluciones particulares exp(i0 t) y exp(i0 t) como base. O sea,
otra forma de escribir la solucin general de (13.12) es
o
(t) = exp(i0 t) + exp(i0 t) .
Determinaremos las constantes y . Para ello, nuevamente, derivemos la solucin
o
general:

(t) = i0 exp(i0 t) i0 exp(i0 t) .


Evaluando estas dos ultimas ecuaciones para t = 0, y usando las condiciones

iniciales, se obtiene
(0) = 0 = +
y

(0) = = i0 i0 .

Despejando y de estas dos relaciones:


1
2
1
= =
2
=

0 + i
0
0 i

,
.

Sustituyendo estos valores en la solucin general se obtiene


o
(t) =

1
2

0 i

exp(i0 t) +

1
2

0 + i

exp(i0 t)

Demostremos ahora que las expresiones encontradas en las partes a) y b) son equivalentes. En efecto, reordenando los trminos de la solucin encontrada en la parte b)
e
o


CAP
ITULO 13. OSCILADOR ARMONICO

481

se encuentra que
1
1
(exp(i0 t) + exp(i0 t)) i
(exp(i0 t) + exp(i0 t))
2
0 2
ei0 t + ei0 t
ei0 t ei0 t
= 0
+
2
0
2i

sen(0 t) .
= 0 cos(0 t) +
0

(t) = 0

Los dos procedimientos dan exactamente el mismo resultado. En el presente ejemplo, el segundo mtodo result ser ms engorroso, ms largo y menos transparente
e
o
a
a
y ciertamente no se observa ninguna ventaja al haber introducido la base con exponenciales complejas. Sin embargo, en las secciones siguientes, al estudiar problemas
levemente ms complejos, la ventaja de usar las exponenciales complejas en lugar del
a
seno y coseno resultar ms evidente.
a a

13.3.

El oscilador armnico atenuado


o

Ejemplo: Consideremos una masa m adosada a un resorte de constante de restitucin


o
k. Supongamos que la masa m slo se pueo
de desplazar a lo largo del eje x. Sea x(t)

la posicin de m, siendo x = 0 la posicin


o
o
de equilibrio. Supongamos adems que sobre
a
el sistema acta una fuerza de roce que es
u
proporcional a la velocidad x (pero de signo

contrario), o sea
fr = x(t)

k
m

^
x

O
Figura 13.2

(con > 0) .

Usando la segunda ley de Newton se deduce que la posicin x(t) satisface la


o
siguiente ecuacin diferencial
o
m(t) = k x(t) x(t) .
x

Introduciendo las constantes 0 k/m y /2m se encuentra que la relacin


o
dinmica para este oscilador armnico con roce es
a
o
2
x + 2 x + 0 x = 0 .

Esta es la ecuacin diferencial del oscilador armnico atenuado.


o
o

(13.13)


CAP
ITULO 13. OSCILADOR ARMONICO

482

Ejercicio: Demuestre que la ecuacin diferencial (13.13) es lineal.


o
Deseamos encontrar la solucin general de la ecuacin (13.13). Sabemos que, si
o
o
encontramos dos soluciones particulares distintas de (13.13) (denotmoslas por x1 (t)
e
y x2 (t)), entonces la solucin general vendr dada por
o
a
x(t) = x1 (t) + x2 (t) ,
donde las constantes y se eligen de manera que la solucin satisfaga las condiciones
o
iniciales.
Procederemos de acuerdo al siguiente esquema: primero encontraremos la solucin
o
general de (13.13) y luego determinaremos las constantes arbitrarias de la solucin geo
neral de manera de obtener la solucin particular que, en t = 0, satisface las siguientes
o
condiciones iniciales:
x(0) = x0
y
x(0) = v0 = 0 .

Ansatz (o hiptesis de trabajo): Busquemos soluciones del tipo x(t) = et , donde


o
es una constante por determinar. Derivando el Ansatz dos veces se obtiene
x(t) = et ,

y
x(t) = 2 et .

Sustituimos estas relaciones en (13.13),


2
2 et + 2 et + 0 et = 0 ,

o sea,
2
2 + 2 + 0 = 0 .

Resolviendo esta ecuacin de segundo grado para se encuentra


o
=

2
2 0 .

(13.14)

Debemos distinguir tres casos:


i) Caso > 0 (oscilador armnico supercr
o
tico).
En este caso la ecuacin (13.14) nos entrega dos soluciones distintas de la ecuao
cin diferencial, stas son
o
e
2 2
x1 (t) = e(+ 0 )t


CAP
ITULO 13. OSCILADOR ARMONICO

483

y
x2 (t) = e(

2
2 0 )t

La solucin general, por lo tanto, es


o
2 2
2 2
x(t) = e(+ 0 )t + e( 0 )t .
Determinando y de manera que la solucin general anterior cumpla con las
o
condiciones iniciales x(0) = x0 y x(0) = 0, se encuentra

x(t) =

x0
2

1+

2
0

e(+

2
2 0 )t

La gura 13.3 muestra cualitativamente


el comportamiento del oscilador en este caso. En el caso supercr
tico la friccin es muy
o
grande y la masa m no oscila. Imag
nese una
bolita colgada de un resorte sumergida en un
frasco con miel.

2
0

e(

2
2 0 )t

x (t)
x0

0
t

Figura 13.3

ii) Caso < 0 (oscilador armnico subcr


o
tico).

En este caso la ecuacin (13.14) tambin nos da dos soluciones distintas:


o
e
2 2
x1 (t) = e(+i 0 )t = et eit
y
x2 (t) = e(i

2
0 2 )t

= et eit ,

con

2
0 2 .

La solucin general viene dada por


o
x(t) = et eit + eit

Evaluando y de manera que la solucin cumpla las condiciones de borde


o
x(0) = x0 y x(0) = 0, se encuentra

x(t) =

x0 t
e
2

1+

= x0 et cos(t) +

eit + 1

sen(t) .

eit
(13.15)


CAP
ITULO 13. OSCILADOR ARMONICO
La gura 13.4 muestra cualitativamente
el comportamiento del oscilador en este caso.
En el caso subcr
tico la friccin es relativao
mente pequea y la masa m oscila. Note que
n
a medida que transcurre el tiempo la amplitud de la oscilacin decae exponencialmente.
o

484
x (t)
x0

0
t

Figura 13.4
iii) Caso = 0 (oscilador armnico cr
o
tico).
Este caso es levemente ms complicado, ya que la ecuacin (13.14) nos da slo
a
o
o
una solucin:
o
x1 (t) = et .
Debemos, de alguna manera, encontrar otra solucin para poder construir la
o
solucin general.
o
Ejercicio: Demuestre que la otra solucin de la ecuacin diferencial
o
o
x + 2 x + 2 x = 0

es la funcin
o
x2 (t) = t et .
Usando el resultado del ejercicio se encuentra que, para un oscilador armnico
o
atenuado cr
tico, la solucin general viene dada por
o
x(t) = ( + t) et .
Para que la solucin cumpla con las condiciones de borde se determina que sta
o
e
viene dada por
x(t) = x0 (1 + t) et .
(13.16)
Observe que, independiente de las condiciones iniciales, el oscilador armnico ateo
nuado paulatinamente siempre se acercar a su posicin de equilibrio, es decir, para
a
o
t , siempre x(t) 0.
Ejercicio: Demuestre que la solucin (13.16) tambin se puede obtener a partir de
o
e
(13.15) poniendo 0 = + y realizando el l
mite 0.


CAP
ITULO 13. OSCILADOR ARMONICO

13.4.

485

El oscilador armnico forzado


o

Agregumosle al oscilador armnico atenuado una fuerza armnica externa Fe de una


e
o
o
frecuencia , es decir,
Fe = F0 cos(t) .
Situaciones de este tipo se dan con gran frecuencia en la naturaleza.
La ecuacin diferencial para el oscilador en este caso es
o
2
x + 2 x + 0 x =

F0
cos(t) .
m

(13.17)

Ejercicio: Demuestre que la ecuacin diferencial anterior no es lineal, o sea, la suma


o
de dos soluciones ya no sigue siendo solucin.
o
Si el lado derecho es nulo (o sea, F0 = 0), entonces la ecuacin coincide con la anao
lizada en la seccin anterior. En este caso conocemos la solucin general. Denotemos
o
o
esta solucin general (de la ecuacin homognea) por xh (t). Tal solucin tendr dos
o
o
e
o
a
constantes arbitrarias.
Sea xp (t) una solucin particular cualquiera de (13.17), entonces la solucin geo
o
neral ser
a
x(t) = xh (t) + xp (t) .
Efectivamente, es fcil demostrar que x(t) es solucin de (13.17). De que es la solucin
a
o
o
general se desprende del hecho de que sta, por ser la ecuacin diferencial de segundo
e
o
orden, debe tener dos constantes arbitrarias, las que x(t) efectivamente tiene (las de
la funcin xh (t)).
o
En general, la solucin x(t) tiene un comportamiento complejo. Sin embargo, pao
ra tiempos grandes (t ) la solucin xh (t) siempre desaparece, quedando slo la
o
o
solucin particular xp (t). Observe que xp (t) es independiente de las condiciones inio
ciales. Todas las soluciones del problema, para t , terminarn siendo idnticas.
a
e
Cuando esto ocurre, se dice que se ha llegado al estado estacionario. Las oscilaciones
iniciales del oscilador, que son altamente irregulares, y que si dependen de las condiciones iniciales, se llama es transiente. Para muchos problemas prcticos la solucin
a
o
que interesa es la del estado estacionario.
En lo que sigue encontraremos la solucin xp (t) que es la correspondiente al estado
o
estacionario. Por ser algebraicamente mucho ms simple, usaremos extensivamente
a
las funciones exponenciales complejas. La fuerza externa la reemplazaremos por la
expresin
o

Fe = F0 eit .
En otras palabras, en lo que sigue encontraremos la solucin estacionaria de la ecuao
cin diferencial
o
F0 it
2

e .
(13.18)
x + 2 x + 0 x =


CAP
ITULO 13. OSCILADOR ARMONICO

486

Observe que la parte real de Fe corresponde a la fuerza externa Fe = F0 cos(t), luego,


al tomar la parte real de esta ecuacin diferencial, obtenemos la ecuacin (13.17); y
o
o
a su vez, la parte real de x(t) corresponder a la solucin estacionaria de (13.17).

a
o
Hagamos el siguiente Ansatz:
x(t) = A eit ,

o sea, analicemos si (13.18) puede tener una solucin de este tipo. Aqu A es una conso

tante que eventualmente habr que determinar. Derivamos x(t) respecto al tiempo:
a

x(t) = iA eit

x(t) = 2 A eit .

Sustituyendo esto en (13.18) se obtiene


F0 it
e ,
m
F0
2
,
2 A + 2iA + 0 A =
m

2
2 A eit + 2iA eit + 0 A eit =

o sea, nuestro Ansatz es una solucin slo si


o o
A=

2
(0

F0 /m
.
2 ) + 2i

Observe que A es un nmero complejo.


u
Cualquier nmero complejo A se puede escribir de la forma
u
A = AR + iAI = |A| ei = |A| cos + i|A| sen ,
donde |A| es el mdulo y la fase del nmero complejo. Conociendo la parte real e
o
u
imaginaria de A se pueden encontrar el mdulo y la fase usando las relaciones
o
|A| =

A2 + A2
R
I

y
tan =

AI
.
AR

Usando las expresiones anteriores para el nmero complejo A se encuentra que ste
u
e
puede escribirse de la forma
A=

F0 /m
(2

2
0 )2

+ 4 2 2

exp i arctan

2
2
0


CAP
ITULO 13. OSCILADOR ARMONICO

487

Hemos encontrado una solucin particular de (13.18):


o
F0 /m

x(t) =

(2

2
0 )2

4 2 2

exp it + i arctan

2
2
0

La solucin estacionaria de (13.17) es la parte real de x(t), o sea,


o

x(t) =

F0 /m
(2

2
0 )2

4 2 2

cos t + arctan

con
|A| =
y

2
2
2 0

= |A| cos(t + ) ,

F0 /m
(2

2
0 )2 + 4 2 2

tan =

2
2 .
2 0

Observe que la solucin estacionaria, o sea, despus de que el transiente ha desapao


e
recido, oscila con la misma frecuencia con que la fuerza externa est forzando el
a
sistema. Observe, sin embargo, que las dos oscilaciones (la se la fuerza externa y la de
la respuesta del sistema) no van es fase, sino que estn desfasados en . La amplitud
a
con que oscila el sistema en el estado estacionario viene dada por |A|.
Resonancias
Analicemos con ms detalle la amplitud con que oscila un oscilador armnico forzado
a
o
en su estado estacionario. La gura 13.5(a) muestra la amplitud |A| en funcin de la
o
frecuencia con que se est forzando el oscilador. Las distintas curvas corresponden
a
a distintos parmetros del coeciente de roce 2/0 . cuando el roce es pequeo, la
a
n
amplitud llega a ser muy grande cuando la frecuencia con que se fuerza el oscilador
es parecida a la frecuencia natural del oscilador 0 . Estas grandes respuestas de un
sistema a est
mulos pequeos se conoce con el nombre de resonancias.
n
Para pequeo, la amplitud mxima de la resonancia viene dada por
n
a
|A|max =

F0
.
2m0

La friccin , aun cuando es pequea, no puede despreciarse. De lo contrario se


o
n
obtienen resultados absurdos; la amplitud del oscilador se ir incrementando indea
nidamente.


CAP
ITULO 13. OSCILADOR ARMONICO

10

0
=0.1
=0.2
=0.5
=1.0

8
6

=0.1
=0.2
=0.5
=1.0

m02|A|/F0

488

-/2

4
2
0
0

-
0

/0

/0

(a)

(b)
Figura 13.5

La gura 13.5(b) muestra el comportamiento de la fase . Observemos que si la


frecuencia con que se fuerza el sistema es mucho menor que la frecuencia natural del
sistema 0 , entonces el est
mulo y la respuesta del sistema esencialmente estarn en
a
fase; cuando 0 , las dos magnitudes estarn completamente desfasadas. Cuando
a
el sistema entra en resonancia 0 , el desfase entre el est
mulo y la respuesta del
.
sistema es de 90

13.5.

Osciladores armnicos acoplados


o

Considere la conguracin mostrada en la


o
gura 13.6. Las masas estn restringidas a
a
moverse a lo largo del eje x. Analicemos la

forma en la cual oscila este sistema.

^
x

Figura 13.6
Sean x1 y x2 los desplazamientos de las dos masas respecto a sus posiciones de
equilibrio. Las ecuaciones de movimiento para estas masas son:
M x1 = kx1 + k(x2 x1 )

(13.19)

M x2 = kx2 k(x2 x1 ) .

(13.20)

y
(Note que (x2 x1 ) es el alargamiento neto del resorte central respecto al largo
que tiene cuando el sistema est en equilibrio.) Las ecuaciones (13.19) y (13.20) son
a
dos ecuaciones diferenciales de segundo orden acopladas (la segunda derivada de x1


CAP
ITULO 13. OSCILADOR ARMONICO

489

depende no slo de x1 , sino que tambin de x2 , y lo mismo ocurre para la segunda


o
e
derivada de x2 ). Sumando y restando las dos ecuaciones diferenciales obtenemos
M (1 + x2 ) = k(x1 + x2 )
x

y
M (2 x1 ) = 3k(x2 x1 ) .
x

Denamos dos nuevas variables 1 y 2 por


1 = x1 + x2 ,
2 = x2 x1 .
Con estas deniciones las dos ultimas ecuaciones diferenciales se pueden escribir de

la forma

M 1 = k1
y

M 2 = 3k2 .

Observe que estas ecuaciones ya no estn acopladas y que cada una de ellas corresa
ponde a la de un oscilador armnico simple. Las soluciones generales vienen dadas
o
por
1 (t) = A cos(1 t) + B sen(1 t)
y
2 (t) = C cos(2 t) + D sen(2 t) ,
con
1

k
M

y
2

3k
= 3 1 .
M

Conociendo 1 y 2 en funcin del tiempo tambin conocemos el comportamiento de


o
e
x1 y x2 :
A
B
C
D
1
cos(1 t) +
sen(1 t) +
cos(2 t) +
sen(2 t)
x1 (t) = [1 (t) + 2 (t)] =
2
2
2
2
2
y
A
B
C
D
1
cos(1 t) +
sen(1 t)
cos(2 t)
sen(2 t) .
x2 (t) = [1 (t) 2 (t)] =
2
2
2
2
2


CAP
ITULO 13. OSCILADOR ARMONICO

490

Esta solucin general tiene cuatro constantes arbitrarias (A, B, C y D), las que se
o
determinan exigiendo que la solucin cumpla con las cuatro condiciones iniciales (la
o
posicin y velocidad de cada una de las masas). Por ejemplo, si en t = 0, x1 (0) =
o
x2 (0) = 0, x1 (0) = v0 y x2 = 0, entonces las constantes resultan ser A = C = 0,

B = v0 /1 y D = v0 /2 .
Debido a que la razn entre las frecuencias 1 y 2 no es un nmero racional, el
o
u
sistema, en general, no manifestar un comportamiento peridico.
a
o
Modos normales
Si en el problema anterior hacemos oscilar el sistema de manera que C = D = 0,
entonces la posicin de ambas masas vendr dada por
o
a
x1 (t) = x2 (t) =

B
A
cos(1 t) +
sen(1 t) .
2
2

Observe que en ese caso ambas masas oscilan juntas (en fase) y que el movimiento de
cada una de ellas es armnico (con per
o
odo T1 = 2/1 ).
Algo parecido ocurre cuando el sistema oscila de manera que A = B = 0. En este
caso
C
D
x1 (t) = x2 (t) =
cos(2 t) +
sen(2 t) .
2
2
Nuevamente ambas masas oscilan juntas, pero en sentido opuestos (en contrafase) y el
movimiento de cada una de ellas es armnico (con per
o
odo T2 = 2/2 ). Estos modos
de oscilacin armnicos del sistema se conocen con el nombre de modos normales.
o
o
Un concepto util en la discusin de sistemas ms complejos es el de grados de

o
a
libertad. Los grados de libertad de un sistema son el nmero de variables que se
u
requieren para describir el sistema. Por ejemplo: una masa m restringida a moverse a
lo largo de una recta tiene un grado de libertad. La misma part
cula, si su movimiento
est connado a un plano, tendr dos grados de libertad. Un sistema consistente de
a
a
tres part
culas que pueden moverse en un plano, tiene 6 grados de libertad. Dos
part
culas en el espacio tridimensional unidas por una barra r
gida poseen 5 grados
de libertad.
A continuacin resumiremos, sin demostracin, algunas caracter
o
o
sticas generales
que presentan todos los sistemas consistentes de osciladores armnicos acoplados.
o
i) Un sistema de osciladores armnicos acoplados de N grados de libertad se deso
cribe con N funciones {xj (t)}. Las ecuaciones dinmicas son ecuaciones diferena
ciales de segundo orden y generalmente estn acopladas.
a
ii) Siempre es posible introducir nuevas variables {j (t)} cuyas ecuaciones diferenciales son de la forma
2

j + j j = 0 ,


CAP
ITULO 13. OSCILADOR ARMONICO

491

o sea, corresponden a osciladores armnicos simples. Las variables j (t) son como
binaciones lineales de las variables {xj (t)}. Los mtodos generales para encontrar
e
estas nuevas variables sern materia de cursos ms avanzados. Sin embargo, en
a
a
muchas situaciones simples no es dif encontrarlos por simple inspeccin.
cil
o
iii) Algunas de las frecuencias j pueden ser nulas, en cuyo caso la ecuacin difereno

cial es simplemente j = 0. Los modos normales de frecuencia nula corresponden


a la traslacin o rotacin del sistema como un todo.
o
o
iv) Cada una de estas nuevas variables da origen a un modo normal. Un sistema con
N grados de libertad tiene N modos normales (algunos de ellos pueden tener
frecuencia nula).
v) La solucin de las ecuaciones diferenciales para las variables j (t) son inmediatas.
o
En total se tendrn 2N constantes arbitrarias.
a
vi) Siempre es posible despejar xj (t) en funcin de las funciones {j (t)} (en el lenguao
je tcnico, el movimiento del sistema, en general, es una suma superposicin
e
o
de los distintos modos normales). De esta manera se encuentra la solucin general
o
del problema. Las constantes arbitrarias se determinan exigiendo que la solucin
o
cumpla con las condiciones iniciales. Hay 2N condiciones iniciales: xj (0) y xj (0)

para los j = 1, 2, . . . , N grados de libertad.


vii) Cuando slo se excita un unico modo normal, todas las part
o

culas se movern
a
armnicamente y con la misma frecuencia. Cuando se excitan dos o ms modos
o
a
normales es forma simultnea, las part
a
culas no se movern armnicamente y el
a
o
movimiento, en general, ni siquiera ser peridico.
a
o
viii) Frecuentemente, en sistemas no demasiado complejos, es posible no slo identio
car algunos o todos los modos normales, sino que tambin encontrar las frecuene
cias respectivas por simple inspeccin del problema.
o

Ejemplo:
Consideremos la conguracin mostrada en
o
la gura 13.7. Las tres masas slo pueden moo
verse a lo largo del anillo de radio R. Los resortes, todos con constante de restitucin k,
o
tambin siempre se deforman a lo largo de la
e
circunferencia. Encontraremos todos los modos normales con sus frecuencias.

k
R

Figura 13.7
El sistema tiene tres grados de libertad y, por lo tanto, existirn tres modos
a


CAP
ITULO 13. OSCILADOR ARMONICO

492

normales. Uno de ellos tiene frecuencia cero y corresponde a una rotacin (r


o
gida)
uniforme de las tres masas a lo largo del anillo.
Es evidente que otro modo normal de oscilacin del sistema es el mostrado en
o
la gura 13.8: una de las tres part
culas queda quieta y las otras dos se mueven en
sentidos opuestos.
1

2
t =0

T/ 4

T/2

3T / 4

Figura 13.8
No es dif encontrar la frecuencia angular de este modo. De los resortes que
cil
unen las part
culas que se mueven, uno se acorta en una magnitud a = R y el otro se
alarga en 2a; la fuerza neta sobre la part
cula es, por lo tanto, 3ka. Para la frecuencia
de este modo de vibracin se obtiene = 3k/m.
o
Otro modo normal se encuentra si la part
cula 2 se mantiene quieta y las part
culas
1 y 3 oscilan movindose en direcciones opuestas (ver gura 13.9). Por supuesto que
e
este modo de oscilacin tiene la misma frecuencia que el modo anterior (en el lenguaje
o
tcnico se dice que los dos modos son degenerados).
e
1

2
t =0

T/ 4

T/2

3T / 4

Figura 13.9
Pareciera que existe un cuarto modo, en que la part
cula 3 se mantiene quieta
y las part
culas 1 y 2 oscilan, movindose en direcciones opuestas (ver gura 13.10).
e
Efectivamente este tambin es un modo normal, pero no es uno distinto; en efecto,
e
la superposicin de los modos mostrados en las guras 13.8 y 13.9 generan el modo
o
mostrado en la gura 13.10. En el lenguaje tcnico se dice que el modo de la gura
e
13.10 no es un modo independiente sino que es una combinacin lineal de los modos
o
normales mostrados en las guras 13.8 y 13.9.


CAP
ITULO 13. OSCILADOR ARMONICO

493

2
t =0

T/ 4

3T / 4

T/2

Figura 13.10

13.6.

Modos normales de una cuerda

Consideremos una cuerda de largo L, sin masa, bajo la tensin , que al centro tiene
o
adosada una masa m y analicemos el movimiento transversal de la masa en ausencia de
gravedad. Sea u(t) el desplazamiento transversal de la masa en funcin del tiempo. En
o
todo momento supondremos que el ngulo de la cuerda con la horizontal es pequeo,
a
n
es decir, que
u(t)
= tan .
L/2

m
u

L
Figura 13.11
Adems supondremos que la tensin no var debido a la pequea elongacin
a
o
a
n
o
que sufre la cuerda cuando est deformada. La fuerza transversal neta sobre la masa
a
m debida a la tensin de la cuerda es
o
F = 2 sen 2 2
La segunda ley de Newton nos da la relacin
o
m =
u

4
u,
L

4
u
=
u.
L/2
L


CAP
ITULO 13. OSCILADOR ARMONICO

494

o sea,
2
u + 1 u = 0 ,

con

4
.
Lm
Concluimos que la masa m oscilar armnicamente con frecuencia 1 .
a
o
2
1 =

(13.21)

Consideremos ahora dos masas m adosadas a la cuerda en forma equiespaciada.


Este sistema ahora tiene dos grados de libertad y, por lo tanto, tendr dos modos
a
normales de oscilacin: uno en que las dos part
o
culas oscilan en fase y otro en que
oscilan en contrafase (ver gura 13.12).

1111
0000
1111
0000
1111
0000
1111
0000
1111
0000
1111
0000

L /3

111
000
111
000
111
000 modo 1
111
000
111
000
111
000

1111
0000
1111
0000
1111
0000
1111
0000
1111
0000

L /3

L /3

111
000
111
000 modo 2
111
000
111
000
111
000

m
Figura 13.12
En el modo 1, la fuerza transversal que acta sobre cada masa es
u
F = sen = =

u
3
= u.
L/3
L

El desplazamiento de cada masa satisfar la ecuacin de movimiento (segunda ley de


a
o
Newton)
3
m = u ,
u
L
que es la ecuacin de un oscilador armnico con frecuencia angular
o
o
2
1 =

3
.
Lm

(13.22)


CAP
ITULO 13. OSCILADOR ARMONICO

495

En el modo 2, la fuerza transversal que acta sobre cada masa es


u
F = sen sen = =

u
2u
9

= u.
L/3
L/3
L

La ecuacin de movimiento de cada masa (segunda ley de Newton) en este caso es


o
m =
u

9
u.
L

Nuevamente es la ecuacin de un oscilador armnico, pero ahora con la frecuencia


o
o
angular
9
2
.
(13.23)
2 =
Lm
Generalicemos los resultados anteriores y consideremos N masas m adosadas en
forma equiespaciada a la cuerda. Denamos el eje x a lo largo de la cuerda cuan
do est en su posicin de equilibrio y elijamos el cero coincidiendo con el extremo
a
o
izquierdo de la cuerda (el otro extremo de la cuerda estar en x = L). La posicin
a
o
longitudinal de la masa j ser
a
L
.
(13.24)
xj = j
N +1
El sistema tiene N grados de libertad y por lo tanto existirn N modos normales. En
a
lo que sigue encontraremos los N modos normales con sus frecuencias respectivas.
Para ello, observemos primero que en los dos casos estudiados anteriormente, con
una y dos masas, la forma de los modos normales parece sinusoidal. En los casos del
unico modo normal con una part

cula (Fig. 13.21), y del modo 1 para dos part


culas
(Fig. 13.23), la forma parece ser sinusoidal, con un cuarto de la longitud de onda
contenida entre los extremos de la cuerda. Para el caso del modo 2 con dos part
culas
(Fig. 13.23) la forma tambin es sinusoidal, pero entre los extremos de la cuerda hay
e
media longitud de onda (el doble de la anterior). Lo anterior sugiere que, en general,
los modos normales tienen, en la coordenada espacial, una forma sinusoidal. Puesto
que la amplitud debe ser cero en ambos extremos, la dependencia debe ser de la forma

sen(x/L), con un nmero natural. Esta es slo la amplitud, porque adems las
u
o
a
masas deben oscilar armnicamente en el tiempo, lo cual se consigue multiplicando la
o
amplitud anterior por una cierta funcin del tiempo, u(t). El razonamiento anterior
o
nos conduce a introducir, como candidatos a modos normales, a las N funciones
y (x, t) = u(t) sen

x
L

(13.25)

con = 1, 2, 3, . . . , N .
Consideremos un particular (por ejemplo = 3) y desplacemos las N part
culas transversalmente en una distancia uj (t) = y (xj , t). La gura 13.13 muestra esquemticamente la situacin que se tiene en este caso.
a
o


CAP
ITULO 13. OSCILADOR ARMONICO

496

=3

y(x)

u2
x2

^
x

m
0

L
L
N +1
Figura 13.13

Encontraremos la ecuacin de movimiento de la part


o
cula j. Los ngulos que la
a
cuerda al lado izquierdo y derecho de la part
cula j forman con la horizontal son

uj uj1
L/(N + 1)
uj+1 uj
,
L/(N + 1)

respectivamente (ver gura 13.14).

L
N +1
m

uj

uj + 1

u j 1
x j 1

xj

xj+ 1

Figura 13.14
La fuerza transversal neta que acta sobre la part
u
cula j es
F = sen + sen ( ) (2uj uj+1 uj1 )

N +1
.
L


CAP
ITULO 13. OSCILADOR ARMONICO

497

La ecuacin de movimiento para la part


o
cula j es, por lo tanto,
mj =
u

(N + 1)
(2uj uj+1 uj1 ) .
L

Pero uj (t) = y (xj , t), luego


uj = u(t) sen

xj = u(t) sen
L

j
N +1

(j + 1)
+ sen
N +1

j
cos
N +1
N +1

uj+1 + uj1 = u(t) sen


= 2 sen

(j 1)
N +1

y
2uj uj+1 uj1 = 2u(t) 1 cos

N +1

sen

j
N +1

Con estas relaciones la ecuacin de movimiento para la part


o
cula j queda
m =
u

2 (N + 1)
1 cos
L

N +1

u,

N +1

o sea,
2
u + u = 0

con
2
=

2 (N + 1)
1 cos
mL

(13.26)

Observe que sta resulta ser la de un oscilador armnico y que es independiente de


e
o
j, o sea, todas las masas oscilarn armnicamente con la misma frecuencia. En otras
a
o
palabras, el movimiento ser el de un modo normal de vibracin del sistema. Haciendo
a
o
variar se obtienen los distintos modos de vibracin.
o
Ejercicio: Demuestre que la ecuacin (13.26), para N = 1 (y = 1) coincide
o
con (13.21) y que para N = 2 (con = 1 y 2) coincide con (13.22) y (13.23),
respectivamente.
Ejercicio: Demuestre que para enteros > N no se obtienen nuevos modos de
oscilacin.
o
A continuacin estudiaremos el caso de una cuerda de largo L, pero con una
o
densidad lineal de masa uniforme . La masa de tal cuerda es L.


CAP
ITULO 13. OSCILADOR ARMONICO

498

Para obtener la cuerda con masa tomaremos el l


mite N y m 0 de
manera que la masa total de la cuerda sea L, o sea,
L

m0

tal que N m = L

En este l
mite, para las frecuencias , se tiene
2
=

2 N 2
1 2 2
1 1
L(N m)
2 N2

22
,
L2

o sea,
=

Esta ultima ecuacin da las frecuencia de los modos normales de una cuerda de largo

o
L, densidad lineal y bajo tensin . Hay innitos modos normales, todos ellos
o
mltiplos enteros de una frecuencia fundamental
u
1 =

13.7.

Problemas

1. La aceleracin de la gravedad var con la posicin sobre la Tierra debido a


o
a
o
su rotacin y a que el globo terrqueo no es exactamente esfrico. Esto fue
o
a
e
descubierto por primera vez en el siglo XVII, cuando se observ que un reloj
o
pendular, cuidadosamente ajustado para marcar el tiempo correcto en Par
s,
perd alrededor de 90 s por d cerca del Ecuador.
a
a
a) Demuestre que una pequea variacin de g produce una pequea modin
o
n
cacin del per
o
odo del pndulo T dada por
e
1 g
T
=
.
T
2 g
b) Cunto deber variar g para lograr explicar la diferencia del per
a
a
odo de
un pndulo entre Par y el Ecuador?
e
s
2. Una masa de 2 kg se sujeta a un resorte de constante de fuerza k = 10 N/m
que descansa sobre una supercie horizontal lisa. Otra masa de 1 kg se desliza
a lo largo de la supercie hacia la primera a 6 m/s.
a) Hallar la amplitud de la oscilacin si las masas realizan un choque perfeco
tamente inelstico y ambas quedan adosadas al resorte. Cul es el per
a
a
odo
de oscilacin?
o


CAP
ITULO 13. OSCILADOR ARMONICO

499

b) Hallar la amplitud y per


odo de la oscilacin si el choque es perfectamente
o
elstico.
a
c) En cada caso encuentre una expresin para la posicin x de la masa sujeta
o
o
al resorte en funcin del tiempo, admitiendo que el choque se produce en
o
el instante t = 0.
3. Un resorte de constante de fuerza k =
100 N/m cuelga verticalmente de un soporte. En su extremo inferior (que se encuentra a una distancia l0 del techo) se engancha
una masa de 0.5 kg, que luego (en el instante
t = 0) se suelta, desde el reposo. La masa comenzar a oscilar en torno a un nuevo punto
a
de equilibrio x0 .

l0

x0

Figura 13.15

a) Encuentre el nuevo punto de equilibrio x0 .


b) Con qu per
e
odo oscilar la masa m alrededor de x0 ?
a
c) Encuentre la energ cintica y el potencial en funcin del tiempo. (Espea
e
o
cique claramente los or
genes usados para especicar las energ potenas
ciales.)
d) Encuentre la velocidad mxima que llegar a tener la masa m mientras
a
a
oscila.
4. En una cuenca esfrica de radio r se deslie
za una masa m1 una pequea distancia s1 ,
n
siendo s1 r. Una segunda masa m2 se desplaza en sentido opuesto hasta una distancia
s2 = 3s1 (tambin s2 r).
e
a) Si las masas se dejan libres en el mismo
instante y resbalan sin roce, en dnde
o
se encontrarn?
a

r
m1
s1

m2
s2

Figura 13.16

b) Si la colisin es elstica, cundo volo


a
a
vern las masas nuevamente a estar en
a
reposo y en qu lugar?
e
5. Un bloque de madera se desliza sobre una supercie horizontal lisa. El bloque
est sujeto a un resorte que oscila con per
a
odo de 0.3 s. Un segundo bloque descansa en su parte superior. El coeciente de roce esttico entre los dos bloques
a
es = 0.25.
a) Si la amplitud de oscilacin es 1 cm, se deslizar el bloque situado encima?
o
a
b) Cul es la mayor amplitud de oscilacin para la cual no se deslizar el
a
o
a
bloque de encima?


CAP
ITULO 13. OSCILADOR ARMONICO

500

6. Una variable x(t) se comporta armnicamente. Si en t = 0, la posicin, la


o
o
velocidad y aceleracin vienen dadas por x(0) = 1 cm, v(0) = 2 cm/s y a(0) =
o
4 cm/s2 , respectivamente. Encuentre la posicin x(t) y la velocidad v(t) para
o
t = 6 s.
7. La gura 13.17 muestra un tubo de seccin constante A y forma de U, abierto a la
o
atmsfera. El tubo est lleno hasta el nivel ino
a
dicado por una l
nea a trazos con un l
quido
incompresible que uye a travs del tubo con
e
un rozamiento despreciable. La longitud total
de la columna de l
quido es L. Demuestre qeu
si se hace descender la supercie del l
quido
en uno de los brazos de la U y luego se deja libre, el nivel del uido oscilar armnicamente
a
o
alrededor de su posicin de equilibrio con un
o
per
odo dado por T = 2 L/2g.

Figura 13.17

8. Encuentre (aproximadamente) el menor valor de la frecuencia angular que podria tener un oscilador armnico x(t), si lo que se conoce es que x(0) = 0,
o
v(1 s) = 2 cm/s y a(2 s) = 4 cm/s2 .
9. Suponga que una variable x(t) var armnicamente con una frecuencia angular
a
o
0 = 2 s1 .
a) Encuentre la posicin x y la velocidad v en el instante t = 3 s si x(0) = 1 cm
o
y x(1 s) = 1 cm.
b) Repita lo mismo pero con las condiciones de borde x(1 s) = 1 cm y v(1 s) =
4 cm/s.
c) Repita lo mismo pero ahora con las condiciones de borde x(0) = 2 cm y
v(2 s) = 4 cm/s.
10. Se cuelga una masa M de un resorte y se pone en movimiento oscilatorio vertical,
con una amplitud de 7 cm. La frecuencia de las oscilaciones es de 4 Hz. Al
llegar M a la posicin ms baja, se le coloca encima una pequea piedrecita.
o
a
n
Supongamos que la masa de la piedrecita es tan pequea que no tiene mayor
n
efecto sobre la oscilacin.
o
a) A qu distancia por encima de la posicin de equilibrio perder contacto
e
o
a
la piedrecita con la masa M ?
b) Cul es la velocidad de la piedrecita cuando se separa de la masa M ?
a


CAP
ITULO 13. OSCILADOR ARMONICO

501

11. Un pndulo simple de 50 cm de largo cuele


ga del techo de un vagn que se acelera con
o
una aceleracin a = 7 m/s2 en direccin horio
o
zontal. Encuentre el per
odo del pndulo para
e
pequeas oscilaciones en torno a su posicin
n
o
de equilibrio.

Figura 13.18

12. Considere una variable x(t) que satisface la ecuacin de un oscilador armnico
o
o
atenuado. Suponga que 0 = 1 rad/s y que se tienen las siguientes condiciones
iniciales: x(0) = 2 cm, v(0) = 0 cm/s.
a) Encuentre la solucin si = 2.20 . Graque la solucin en el intervalo
o
o
0 < t < 20 s.
b) Repita lo mismo de la parte (a), pero con = 0 .
c) Repita lo anterior, pero ahora con = 0.50 .
d) Repita lo de las partes (a), (b) y (c), con las condiciones iniciales x(0) =
0 cm y v(0) = 50 cm/s.
13. Considere dos pndulos idnticos acoplados.
e
e
Las ecuaciones de movimiento en ese caso vienen dadas por:

1111111111111111111111111111
0000000000000000000000000000
1111111111111111111111111111
0000000000000000000000000000
1111111111111111111111111111
0000000000000000000000000000
1111111111111111111111111111
0000000000000000000000000000

m1 = mg1 (1 2 )

m2 = mg2 + (1 2 )

La constante acopla los dos osciladores


armnicos. Si = 0 (o sea, si el acoplamieno
to se hace cero) cada pndulo oscila indepene
dientemente del otro.

Figura 13.19

a) Introduzca las nuevas variables


1 (t) = 1 (t) + 2 (t)
2 (t) = 1 (t) 2 (t)

y demuestre que stas var armnicamente con las frecuencias


e
an
o
0 =

g/

respectivamente, donde = 2/(m).

1 =

2
0 + ,


CAP
ITULO 13. OSCILADOR ARMONICO

502

b) Demuestre que la solucin general se puede escribir de la forma


o
1
[A cos(0 t + ) + B cos(1 t + )]
2
1
2 (t) = [A cos(0 t + ) B cos(1 t + )]
2
1 (t) =

Las constantes A, B, y se determinan con las condiciones de borde.


c) Sea 0 = 1 rad/s y = 0.1 s2 . Encuentre la solucin para el caso en que
o

1 (0) = 0 , 2 (0) = 1 (0) = 2 (0) = 0. Graque 1 (t) y 2 (t).

d) Repita lo anterior, pero para el caso en que 1 (0) = 2 (0) = 0 y 1 (0) =


2 (0) = 0.

e) Repita lo anterior, pero para el caso en que 1 (0) = 2 (0) = 0 y 1 (0) =


2 (0) = 0.

f ) Para el caso (c) el movimiento de cada pndulo consiste en un movimiento


e
oscilatorio cuya amplitud tambin var peridicamente. Sea la frecuene
a
o
cia angular de la variacin peridica de la amplitud. Encuentre .
o
o
14. Pndulo fsico: Considere un objeto de masa M , que puede oscilar alrededor de
e

un eje que lo atraviesa. Sea I el momento de inercia para rotaciones alrededor


de ese eje y la distancia entre el eje y el centro de masas del objeto. Encuentre
el per
odo T para pequeas oscilaciones alrededor de su posicin de equilibrio.
n
o
Demuestre que un pndulo simple equivalente, es decir, uno que tenga el mismo
e
per
odo, tiene un largo
I
.
0 =
k
m
k
m
15. Considere la conguracin mostrada en la o
gura 13.20 Las cuatro masas slo pueden moo
verse a lo largo del anillo de radio R. (Los resortes tambin siempre se deforman a lo largo
e
de la circunferencia.) Encuentre la frecuencia
de los modos normales de oscilacin.
o

m
R

Figura 13.20

16. Considere una masa m resbalando sin roce (en presencia de la aceleracin de
o
gravedad g y ) a lo largo de un perl de la forma

y(x) = x3 x2 ,
con = 1 m2 y = 3/2 m1 . Graque y(x). Si la masa realiza pequeas
n
oscilaciones en torno al m
nimo local, encuentre el per
odo T de tal movimiento.


CAP
ITULO 13. OSCILADOR ARMONICO

503

17. Una masa de 2 kg oscila colgada de un resorte de constante de restitucin k =


o
1 . El sistema es forzado
400 N/m. La constante de amortiguamiento es = 1 s
por una fuerza sinusoidal de amplitud F0 = 10 N y frecuencia angular = 10
rad/s.
a) Cul es la amplitud de las oscilaciones en el rgimen estacionario?
a
e
b) Si se var la frecuencia de la fuerza impulsora, a qu frecuencia se proa
e
ducir la resonancia?
a
c) Encuentre la amplitud de las vibraciones en la resonancia.
18. Considere una masa m = 50 g que oscila sujeta a un resorte de constante de
restitucin k = 100 N/m. Suponga que hay algn dispositivo que atena las
o
u
u
oscilaciones con una fuerza que es proporcional a la velocidad (o sea, estamos
en presencia de un oscilador armnico atenuado). Con un cronmetro se mide
o
o
el per
odo de oscilacin; ste resulta ser igual a 2.1 s.
o
e
a) Cunto valen 0 y ?
a
b) En cunto disminuir la amplitud mxima de oscilacin entre dos ciclos
a
a
a
o
consecutivos?
19. Una masa m = 1 kg cuelga de un resorte de constante de restitucin k = 200
o
1 . En el instante t = 0
N/m. La constante de amortiguamiento es = 1 s
comienza a actuar sobre la masa una fuerza F = F0 sin(t), con F0 = 2 N y
= 10 s1 .
a) Si x(0) = 0 y v(0) = 0, encuentre x(t) para t = 1 s, t = 100 s y t = 1000 s.
b) Encuentre la energ disipada en un ciclo cuando el oscilador se encuentra
a
en el rgimen estacionario.
e
20. Una masa m descansa sobre una mesa horizontal lisa (sin roce). El movimiento
de la masa est restringido a desplazamientos a lo largo del eje x. Sobre la masa
a

acta una fuerza F (t) = F0 sin(t).


u
x
a) Encuentre la aceleracin a(t) y la velocidad v(t) de la masa, si en el instante
o
t = 0 se encontraba detenida.
b) Encuentre la posicin x(t) si adems se sabe que x(0) = 0. Demuestre que
o
a
el movimiento es armnico con una amplitud A = F0 /(m 2 ).
o
c) La masa ahora se sujeta adicionalmente a un resorte de constante de restitucin k. (La orientacin del resorte tambin es a lo largo del eje x).
o
o
e

Compare el movimiento que tiene ahora con el que ten cuando no estaba
a
unida al resorte.


CAP
ITULO 13. OSCILADOR ARMONICO

504

21. (Pndulo de torsin)


e
o
Suponga que un extremo de un alambre
metlico est rmemente adosado del ciea
a
lo de una pieza y del otro cuelgan dos esferas slidas tal como se mustran en la gura
o
adjunta. Al girar las esferas con el alambre
en un ngulo (alrededor del eje formaa
do por el alambre), el alambre ejercer un
a
torque que har que las esferas retornen
a
a la posicin de equilibrio. El torque que
o
ejerce el alambre es
=
donde es una constante (que depende del
largo, dimetro y material de que est hea
a
cho el alambre).

Figura 13.21

Para este problema suponga que = 1250 g cm2 /s2 . Si las esferas son de aluminio (Al = 2, 7 g/cm3 ), qu dimetro deben tener las esferas para que el
e a
per
odo sea exactamente de un segundo? (El momento de inercia de una esfera
slida de masa M y radio R para una rotacin alrededor de un eje que pasa por
o
o
su centro es I = 2mR2 /5).
22. Una masa de m = 0.5 kg, despus de caer una distancia h = 5 m, se adosa a un
e
resorte (largo) de constante k = 2 kg/s2 . El sistema resultante viene gobernado
por la ecuacin de movimiento
o
2

z (t) + 0 z(t) + 20 z(t) = 0

o sea, corresponde a un oscilador armnico amortiguado cr


o
tico. La magnitud
z(t) mide la posicin de la masa m respecto al punto de equilibrio y 0 = k/m
o
es la frecuencia natural del sistema.
La solucin general est dada por la relacin
o
a
o
z(t) = (A + Bt)e0 t
donde A y B son constantes que se ajustan con las condiciones iniciales.
(Para los clculos numricos que siguen, use para la aceleracin de gravedad el
a
e
o
2)
valor g = 10 m/s


CAP
ITULO 13. OSCILADOR ARMONICO

505
m

a) Determine A y B usando las condiciones iniciales.


b) Sea t0 el instante en que el resorte
tiene su mxima compresin. Evale
a
o
u
t0 . (Elija el cero del tiempo en el instante en que la masa colisiona con el
resorte).

resorte
amortiguado

c) Haga un grco esquemtico de la


a
a
funcin z(t).
o
d) Cul ser la energ total disipada
a
a
a
por el amortiguador?
23. Considere dos cilindros que giran rpidaa
mente en sentidos contrarios tal como se
muestra en la gura adjunta. Sobre estos
cilindros se coloca un tabln de masa M y
o
densidad uniforme. Sea d la distancia entre los dos cilindros y sea el coeciente
de roce cinemtico entre el tabln y los cia
o
lindros. Demuestre que el movimiento del
tabln es armnico. Encuentre el per
o
o
odo
del movimiento.

Figura 13.22

M
d

Figura 13.23

24. Considere dos masas m1 y m2 unidas por un resorte de largo natural o y


constante de restitucin k. Supongamos que el movimiento de ambas masas
o
est restringido a lo largo de la recta que los une.
a
Sean x1 (t) y x2 (t) las posiciones de las masas m1 y m2 , respectivamente.
a) Demuestre que x1 (t) y x2 (t) satisfacen las ecuaciones diferenciales acopladas
m1 x1 (t) = k[x2 (t) x1 (t) 0 ]

m2 x2 (t) = k[x2 (t) x1 (t) 0 ]

b) Denamos las variables 0 (t) y 1 (t) por


m1 x1 (t) + m2 x2 (t)
m1 + m2
1 (t) = x2 (t) x1 (t) 0
0 (t) =


CAP
ITULO 13. OSCILADOR ARMONICO

506

Demuestre que las variables 0 (t) y 1 (t) satisfacen las ecuaciones diferenciales (desacopladas)
0 = 0

1 + 2 1 = 0

con
2 = k

m1 + m2
.
m1 m2

c) Demuestre que la solucin ms general del problema se puede escribir de


o
a
la forma
m2
(0 + C cos(t) + D sin(t))
m1 + m2
m2
x2 (t) = A + Bt +
(0 + C cos(t) + D sin(t))
m1 + m2
x1 (t) = A + Bt

d) Denamos 0 y por 0 = k/m y = m2 /m1 . Exprese en trminos


e
de 0 y . Cunto vale en trminos de 0 si ? Coincide esto
a
e
con lo que usted intu Cuanto vale en trminos de 0 si = 1?
a?
a
e
e) Sea 0 = 8 cm y 0 = 1 rad/s y = 1 (o sea, m1 = m2 ). Encuentre
la solucin que satisface las siguientes condiciones iniciales: x1 (0) = 0,
o
x2 (0) = 10 cm y v1 (0) = v2 (0) = 0. Graque x1 (t) y x2 (t) en un mismo
grco para el intervalo 0 < t < 15 s.
a
f) Repita lo mismo de la parte (e) pero para las condiciones iniciales x1 (0) =
0, x2 (0) = 8 cm, y v1 (0) = 4 cm/s y v2 (0) = 0.
g) Repita la parte (f) pero con = 10.
h) Repita la parte (f) pero con = 0.1.
25. Considere tres part
culas de masa m que
slo pueden moverse a lo largo del eje x
o

y estn unidas por resortes de largo naa


tural 0 y constantes de restitucin k (ver
o
gura).

l0
m

l0
m

11111111111111111111
00000000000000000000
11111111111111111111
00000000000000000000
11111111111111111111
00000000000000000000

Figura 13.24
Sean x1 (t), x2 (t) y x3 (t) las posiciones de las tres masas en funcin del tiempo.
o
a) Demuestre que x1 (t), x2 (t) y x3 (t) satisfacen las ecuaciones diferenciales
acopladas
m1 = k(x2 x1 0 )
x

m2 = k(x3 + x1 2x2 )
x
m3 = k(x2 x3 + 0 )
x


CAP
ITULO 13. OSCILADOR ARMONICO

507

b) Intoduzca las nuevas variables denidas por


0 = (x1 + x2 + x3 ) , 1 = (x1 x3 ) + 20

2 = (x1 2x2 + x3 ).

Demuestre que estas nuevas variables satisfacen las ecuaciones diferenciales


desacopladas
2
1 + 1 1 = 0 y

k/m y 2 = 31 . Interprete!

2
2 + 2 2 = 0,

0 = 0 ,

con 1 =

c) Demuestre que la solucin general al problema se puede escribir de la forma


o
x1 (t) = A + Bt + C cos(1 t + 1 ) + D cos(2 t + 2 ) 0

x2 (t) = A + Bt 2D cos(2 t + 2 )

x3 (t) = A + Bt C cos(1 t + 1 ) + D cos(2 t + 2 ) + 0


Las constantes A, B, C, D, 1 y 2 se eligen de manera que la solucin
o
satisfaga las condiciones de borde. Convnzase de que, en general, las cone
diciones de borde determinan a las seis constantes.
d) Suponga que 0 = 5 cm y 1 = 1 rad/s. Encuentre la solucin que satisface
o
las siguientes condiciones iniciales: x1 (0) = 8 cm, x2 (0) = 0, x3 (0) = 8
cm, v1 (0) = v2 (0) = v3 (0) = 0. Graque en un mismo grco x1 (t), x2 (t)
a
y x3 (t) en el intervalo 0 < t < 15 s.
e) Repita lo mismo que la parte (d), con las condiciones iniciales x1 (0) = 4
cm, x2 (0) = 2 cm, x3 (0) = 6 cm y v1 (0) = v2 (0) = v3 (0) = 0.

f) Repita lo mismo que la parte (d), con las condiciones iniciales x1 (0) = 8
cm, x2 (0) = 0 cm, x3 (0) = 5 cm y v1 (0) = v2 (0) = v3 (0) = 0.

g) Repita lo mismo que la parte (d), con las condiciones iniciales x1 (0) = 5
cm, x2 (0) = 0 cm, x3 (0) = 5 cm, v1 (0) = v2 (0) = 0 y v3 (0) = 3 cm/s.
26. Considere un resorte, de constante de restitucin k, que conecta dos masas,
o
M y m restringidas a moverse a lo largo del eje x. Encuentre la frecuencia de

oscilacin de tal sistema.


o
27.

Suponga que la energ potencial de cierta molcula diatmica viene razonaa


e
o
blemente bien descrita por la expresin
o
1
U (r) = U0
2

r0
r

12

r0
r

con U0 = 2 eV (eV es una unidad de energ usada en la f


a
sica atmica llamada
o
9 m).
electron-volt) y r0 = 0,5 nm (1 nm=10
a) Demuestre que r = r0 es la separacin de equilibrio de la molcula.
o
e


CAP
ITULO 13. OSCILADOR ARMONICO

508

b) Graque U (r) en el rango 0,4 nm < r < 0,7 nm.


c) Desarrolle el potencial U (r) en torno a r = r0 , es decir exprese U (r) de la
forma
1
U (r) = c0 + c1 s + c2 s2 +
2
donde s = r r0 y encuentre los coecientes c0 , c1 y c2 .

d) Convnzase de que la fuerza para pequeos desplazamientos de los tomos


e
n
a
respecto a su posicin de equilibrio (que ocurre para s = 0) viene dada por
o
F (s) = ks. Evale k.
u
e) Si las masas de los tomos son m1 = m2 = m, cul ser la frecuencia
a
a
a
vibracional de la molcula?
e

28. Considere cuatro masas iguales unidas por


resortes de constante de restitucin k tal
o
como se muestra en la gura. Las masas
slo se pueden mover en el plano en que se
o
ubican. Usando argumentos de simetr
a,
describa algunos de los modos normales
de vibracin y encuentre la frecuencia de
o
ellos. Cuntos modos normales tiene este
a
sistema? Cuntos de ellos tienen frecuena
cia cero?

m
k
k

k
k

k
Figura 13.25

29. Un reloj de los abuelos se basa en un pndulo de longitud 1 m. El reloj se


e
atrasa 1 segundo por d En cunto se debe corregir la longitud del pndulo?
a.
a
e
30. Un resorte de constante de resitucin k
o
= 2 dina/cm y largo en reposo 0 se encuentra adosado rmemente a la base de
un recipiente (ver gura). El recipiente
est lleno de agua.
a
Suponga ahora que en el instante t = 0
se le adosa al extremo superior una esfera slida homognea de radio R = 1 cm,
o
e
hecha de un material ms liviano que el
a
agua, y que la esfera luego se suelta (o sea,
en el instante t = 0 la longitud del resorte
es 0 y la esfera se suelta en reposo). Se
observa que la esfera realiza oscilaciones
armnicas de amplitud A = 0,8 cm.
o

t= 0

l0

Figura 13.26


CAP
ITULO 13. OSCILADOR ARMONICO

509

a) Encuentre la densidad de la esfera.


b) Encuentre el per
odo T del movimiento armnico que la esfera realiza una
o
vez que se suelta.
(Al desarrollar el problema ignore los efectos debidos al roce viscoso entre la
esfera y el agua.)
31. El pndulo de la gura est formado por
e
a
una barra de masa despreciable y longitud L. La masa del extremo inferior se
mantiene unida a un resorte de constante k dispuesto horizontalmente y jo, por
su otro extremo a una pared. Cuando el
pndulo se encuentra en posicin vertical
e
o
la longitud del resorte es la de su largo natural. Calcule la frecuencia del sistema
para pequeos desplazamientos en torno
n
a la posicin de equilibrio. Verique su reo
sultado analizando el l
mite de algunos sistemas conocidos.

L
k
m
Figura 13.27

32. Considere un cilindro de radio R y densidad , con una perforacin cil


o
ndrica de
radio R/2, tal como se muestra en la gura. El cilindro rueda sin resbalar sobre una
supercie horizontal realizando pequeas
n
oscilaciones en torno a su posicin de equio
librio. Encuentre el per
odo de las oscilaciones.

Figura 13.28
33. Considere el siguiente sistema de dos masas iguales M y tres resortes, con
extremos jos, como se muestra en la gura:

k1

k1

^
x

El resorte central tiene constante de restitucin k, y los laterales constante de


o
restitucin k1 .
o


CAP
ITULO 13. OSCILADOR ARMONICO

510

Encuentre la ecuacin de movimiento para cada una de las masas, desacplelas


o
o
para encontrar los modos normales, y encuentre la frecuencia de cada uno de
ellos. Dibuje los modos normales.
34. Resuelva nuevamente el problema 13.29, pero agregando una segunda masa m,

ubicada en el punto medio de la barra, y unida a la pared con un resorte de

constante k. Ambos resortes se encuentran en su largo natural cuando el pndulo


e
est en posicin vertical.
a
o

111111111111111
000000000000000
111111111111111
000000000000000
111111111111111
000000000000000
111111111111111
000000000000000
111111111111111
000000000000000
L /2
_
111111111111111
000000000000000
_
k
111111111111111
000000000000000
m
111111111111111
000000000000000
111111111111111
000000000000000
111111111111111
000000000000000
L /2
111111111111111
000000000000000
k
111111111111111
000000000000000
111111111111111
000000000000000
m
111111111111111
000000000000000
111111111111111
000000000000000

35. Un sistema formado por dos masas y dos resortes cuelga del techo, en presencia
de la gravedad, como muestra la gura:

x1

x1

a) (20 %) Encuentre las ecuaciones de movimiento para las coordenadas x1 y


x2 , denidas como muestra la gura. Suponga que los resortes tienen largo
natural l0 .
b) (20 %) Debido a la gravedad, las masas no estarn en equilibrio cuando
a
(0)
(0)
(0)
x1 = x2 = l0 , sino para ciertos valores x1 = x1 , x2 = x2 . Encuentre x1
(0)
y x2 .


CAP
ITULO 13. OSCILADOR ARMONICO

511
(0)

c) (10 %) Dena nuevas coordenadas i = xi xi , que representan el desplazamiento de cada masa respecto a su posicin de equilibrio respectiva.
o
Encuentre las ecuaciones de movimiento para 1 y 2 .
d) (50 %) Muestre que la combinacin lineal 1 + 2 desacopla las ecuaciones
o
de movimiento, para cierto . Encuentre los valores de que desacoplan
las ecuaciones, y encuentre las frecuencias asociadas a los modos normales
respectivos.
36. Considere un resorte sin masa, de constante k, sobre un plano inclinado un
a
ngulo respecto a la horizontal. Inicialmente, el resorte se encuentra en su
largo natural. En t = 0 se cuelga del resorte una masa m y se suelta desde el
reposo.
Encuentre la nueva posicin de equilibrio del resorte. Si x(t) representa la diso
tancia entre la posicin de la masa y la posicin de equilibrio, encuentre la
o
o
ecuacin diferencial para x(t), la frecuencia de oscilacin, y encuentre tambin
o
o
e
expl
citamente x(t) para las condiciones iniciales dadas. Desprecie el roce entre
la masa y el plano inclinado.
k

37. Considere un sistema formado por dos masas m1 = m2 = m y dos resortes de


constantes 3k y 2k, oscilando sobre una supercie horizontal sin roce y unido a
una pared por uno de sus resortes, como indica la gura:

3k

m1

2k

m2

^
x

Si x1 y x2 representan el desplazamiento de m1 y m2 respecto a sus respectivas posiciones de equilibrio, encuentre las ecuaciones de movimiento para x1
y x2 . Muestre que la combinacin lineal x1 + x2 desacopla las ecuaciones de
o


CAP
ITULO 13. OSCILADOR ARMONICO

512

movimiento, para ciertos valores de . Encuentre los valores de que desacoplan las ecuaciones, y encuentre las frecuencias asociadas a los modos normales
respectivos.

13.8.

Solucin a algunos de los problemas


o

Solucin al problema 8
o
La forma general de la solucin para un oscilador armnico simple es
o
o
x(t) = A cos(t) + B sin(t).
La condicin x(0) = 0 implica que A = 0, luego queda
o
x(t) = B sin(t).
Derivando se obtiene
v(t) = B cos(t) y

a(t) = 2 B sin(t)

Aplicando las condiciones de borde se encuentra que


v(1) = B cos() = 2
y
a(2) = 2 B sin(2) = 2 2 B sin() cos() = 4.
Formando el cuociente entre las dos ultimas ecuaciones obtenemos

La gura 13.29 muestra un grco del lado izquierdo y derecho de esta ecuacin.
a
o
La interseccin de menor frecuencia ocurre para 3,43 s1 .
o
sin() =

Solucin al problema 15
o
El sistema tiene 4 grados de libertad y por lo tanto existen cuatro modos normales.
Sean j , j = 1, 2, 3, 4 los cuatro ngulos de las cuatro masas respecto a sus posiciones
a
de equilibrio. Los cuatro modos normales se encuentran por simple inspeccin del
o
problema.
i) Uno de los modos normales tiene frecuencia nula (1 = 0) y corresponde a la
rotacin uniforme y simultnea de las cuatro masas a lo largo del anillo, o sea,
o
a
1 (t) = 2 (t) = 3 (t) = 4 (t) = t.


CAP
ITULO 13. OSCILADOR ARMONICO

513

sin()

1
0

1/

3
4
5

0
3.2 3.25
0.05

3.3

3.35

3.4

3.45

3.5 3.55

3.6

0.1
0.15
0.2
0.25

sin()

0.3
0.35

1/

0.4
0.45

Figura 13.29

ii) Otro modo normal se obtiene si las part


culas 1 y 3 se mantienen en reposo y
las part
culas 2 y 4 oscilan con la misma amplitud pero en sentido contrario, o
sea 1 (t) = 3 (t) = 0, 2 (t) = 4 (t), t. Al desplazar la masa 2 en un ngulo
a
uno de los resortes se comprime y el otro se alarga en una magnitud R. La
fuerza sobre la masa ser igual a 2kR, luego la frecuencia de este modo normal
a
ser 2 = 2k/m.
a
iii) Otro modo normal se obtiene si las part
culas 2 y 4 se mantienen en reposo y
las part
culas 1 y 3 oscilan con la misma amplitud pero en sentido contrario.
Por simetr este modo tiene la misma frecuencia que el modo normal anterior
a
(3 = 2k/m).
iv) El cuarto modo normal se obtiene si las cuatro masas oscilan con la misma
amplitud, 1 y 3 en la misma direccin y 2 y 4 en la direccin contraria, es decir,
o
o
1 (t) = 3 (t) = 2 (t) = 4 (t), t. Al desplazarse una masa en un ngulo ,
a


CAP
ITULO 13. OSCILADOR ARMONICO
Modo #1

#3

#2

4
2

#4

3
1

Tiempo

514

Figura 13.30

uno de los resortes se acorta y el otro se alarga en una magnitud 2R. La fuerza
sobre la masa ser, por lo tanto, igual a 4kR. Luego la frecuencia de oscilacin
a
o
es 2 = 4k/m.
La gura 13.30 muestra esquemticamente el movimiento de las cuatro masas
a
para los cuatro modos normales.
Solucin al problema 21
o
Al girar el alambre con las esferas en un ngulo = el torque es
a
z
= .
z
El torque genera un cambio del momento angular del sistema. Se tiene
d
d

z
I z = I ,
=
dt
dt
donde I es el momento de inercia de las dos esferas para rotaciones alrededor del eje
z (que coincide con el alambre).

De las dos ecuaciones anteriores se deduce que


=

I =


CAP
ITULO 13. OSCILADOR ARMONICO
o sea,

515

+ 0 = 0

2
odo T se obtiene
con 0 = /I. Para el per

T = 2

I
.

Usando el teorema de Steiner, para el momento de inercia se encuentra la expresin


o
I=2

2
14 4 3 2 56
14
mR2 + mR2 = mR2 =
R R = R5 .
5
5
5 3
15

Usando esto en la expresin anterior para el per


o
odo y despejando R se encuentra
R5 =

15 T 2
= 0, 99986
56 4 3

cm5 .

O sea, con esferas de dimetro igual a 1 cm, este pndulo tendr un per
a
e
a
odo de
1 segundo.
Solucin al problema 22
o
a) Sea x0 la magnitud que el resorte se comprimir respecto a su largo natural una
a
vez que llegue al equilibrio. Se tiene que
kx0 = mg
o sea,

mg
0.5 10
=
m = 2.5 m.
k
2
La velocidad v0 de la masa cuando choca con el resorte viene dada por
x0 =

v0 =

2gh =

2 10 5

m
m
= 10 .
s
s

Por consiguiente, las condiciones iniciales son


x(0) = x0 = 2.5

m y

x(0) = v0 = 10

La frecuencia angular natural del sistema es 0 =


expresin
o
z(t) = (A + Bt)e0 t

m
.
s

k/m = 2 1 . Derivando la
s

se obtiene
z(t) = (B A0 B0 t)e0 t .


CAP
ITULO 13. OSCILADOR ARMONICO

516

0.6
0.5
0.4
0.3
0.2
0.1

z(t)

0.1

0.2
0.3
0.4

Figura 13.31

Evaluando estas expresiones en t = 0 se obtiene


z(0) = A y

z(0) = B A0 .

Usando las condiciones iniciales se encuentra para A y B los valores


A = x0 = 2.5 m
y
B = A0 + z(0) = (2.5 2 10)

m
m
= 5
.
s
s

b) La velocidad z(t) es nula cuando (B A0 B0 t) = 0. De esta relacin se

o
deduce que ello ocurre en el instante
to =

1
A

=
0 B

1
2.5

2 (5)

s=1s.

c) La gura 13.31 muestra el grco de la posicin z(t) en funcin del tiempo.


a
o
o
d) Del cambio de energ potencial U = mg(h + x0 ),
a
1 2
kx
2 o
queda como energ potencial del resorte; el resto se disipa. Por lo tanto, la
a
energ disipada es
a


CAP
ITULO 13. OSCILADOR ARMONICO

1
Q = mgh + mgx0 kx2
2 0
1
= mgh + kx2
2 0
1
1
=
10 5 + 2 (2.5)2
2
2

517

Joule = 31.25 Joule

Solucin al problema 30
o
Una vez que se adosa la esfera al resorte el nuevo punto de equilibrio del resorte
sube en una magnitud D que se puede evaluar de la relacin
o
4
kD = R3 (0 )g ,
3
donde 0 = 1 g/cm3 es la densidad del agua. Observe que la amplitud de la oscilacin
o
coincidir con D, o sea, D = A = 0.8 cm. Despejando se encuentra
a
= 0

3kA
.
4R3 g

Ahora k/g = 2 dina/g = 2 gramos, luego


= 1

3 2 0.8
4

g/cm3 = 0.618 g/cm3 .

El per
odo del movimiento viene dado por
T = 2

m
,
k

donde m = 4R3 /3 = 4 0.618/3 g = 25.9 g. Para el per


odo se encuentra T =
1.75 s.

Cap
tulo 14

Ondas
versin 5 noviembre 2012
o

El oscilador armnico resuelto en el Cap


o
tulo anterior, representa, como hemos
dicho, un problema de gran importancia en F
sica, debido a que puede modelar todos
los sistemas cerca de un punto de equilibrio estable. Sin embargo, hay una sutileza
en esta armacin: todo el trabajo del Cap
o
tulo anterior slo es aplicable a sistemas
o
discretos, es decir, compuestos por una o ms part
a
culas. En este Cap
tulo, mostraremos que los conceptos anteriores pueden extenderse tambin a sistemas continuos
e
(cuerdas de guitarra, lminas metlicas, volmenes de aire encerrados en una cavidad,
a
a
u
agua en un recipiente, etc.). En tal caso, el concepto de oscilacin es generalizado al
o
de onda, y la ecuacin del oscilador armnico que corresponde a la ecuacin de
o
o
o
movimiento en sistemas discretos ser reemplazada por la ecuacin de ondas, que
a
o
tambin es de gran importancia y est relacionada con sistemas f
e
a
sicos de muy distinto tipo. Y la razn es la misma: la ecuacin de ondas permite describir sistemas
o
o
(continuos) cuando stos son perturbados levemente respecto a una conguracin de
e
o
equilibrio estable.

14.1.

La ecuacin de ondas
o

Considere una cuerda de densidad lineal , muy larga, bajo tensin . Denamos
o
el eje x a lo largo de la cuerda cuando sta est en su posicin de equilibrio y llamemos

e
a
o
a esta direccin horizontal.
o
Sea u(x, t) la deformacin de la cuerda en el lugar x e instante t, es decir, u(x, t)
o
es la desviacin transversal de la cuerda, en el lugar x e instante t, respecto a su
o
posicin de equilibrio. Encontraremos la ecuacin que debe satisfacer u(x, t). Para
o
o
ello, debemos encontrar la ecuacin de movimiento para cada elemento innitesimal
o
de la cuerda, lo cual a su vez requiere conocer la fuerza sobre dicho segmento de
cuerda. Supondremos en este tratamiento que la fuerza se debe exclusivamente a la
tensin de la cuerda, despreciando otras fuerzas como la gravedad, por ejemplo.
o
518

CAP
ITULO 14. ONDAS

519

u (x, t )

^
x

Figura 14.1
Consideremos un pedazo innitesimal de la cuerda entre x y x + dx y evaluemos
la fuerza que acta sobre l. En cada extremo, la magnitud de la fuerza es y la
u
e
direccin es tangente a la cuerda (ver gura 14.2, que es una ampliacin del sector
o
o
entre x y x+dx de la gura 14.1). (Estamos suponiendo que la tensin de la cuerda no
o
var si ella se deforma; esto es cierto slo si las deformaciones son suaves y pequeas.)
a
o
n
Sea (x) el ngulo entre la tangente a la cuerda en el lugar x y el eje x. La
a

componente horizontal de la fuerza en el extremo izquierdo es Fx = cos((x)).


Por deformaciones suaves y pequeas entendemos aquellas para las cuales, en todo
n
instante, (x) 1, x. Despreciando trminos de segundo orden en se tiene que
e
cos = 1, luego Fx = . Usando la misma argumentacin se encuentra que la fuerza
o
que sobre el extremo derecho del trozo ejerce el resto de la cuerda es Fx = . La
fuerza total en la direccin x que acta sobre el trozo es, por lo tanto, cero. O sea, el
o
u
trozo de cuerda (entre x y x + dx) no acelerar horizontalmente y por consiguiente el
a
movimiento ser slo en la direccin transversal (vertical).
a o
o
z

(x)
u(x+dx)

u(x)

x
x

x+dx

Figura 14.2
La fuerza en la direccin vertical sobre el extremo izquierdo del trozo de cuerda
o
es Fz = sen((x)). A primer orden en , el seno y la tangente de son iguales, o
sea,
u(x, t)
sin((x)) tan((x)) =
.
x

CAP
ITULO 14. ONDAS

520

De esta manera, para la fuerza vertical en el extremo izquierdo del trozo, se obtiene
Fz (x) =

u(x, t)
.
x

De manera anloga, para la fuerza sobre el extremo derecho se encuentra


a
Fz (x + dx) =

u(x + dx, t)
.
x

La fuerza neta en la direccin vertical, por lo tanto, es


o
tot
Fz = Fz (x) + Fz (x + dx) =

u(x + dx, t) u(x, t)

x
x

Usando la segunda ley de Newton se obtiene


(dx)

u(x + dx, t) u(x, t)


2 u(x, t)
tot

= Fz = Fz (x) + Fz (x + dx) =
2
t
x
x

Dividiendo por dx se obtiene

1 u(x + dx, t) u(x, t)


2 u(x, t)
2 u(x, t)
=

,
=
t2
dx
x
x
x2

y nalmente la

2 u(x, t)
2 u(x, t)
v2
=0,
t2
x2

(14.1)

o
o
con v /. La ecuacin de ondas (14.1) es una ecuacin de derivadas parciales
de segundo orden.
La importancia del estudio de esta ecuacin y sus soluciones se debe a que aparece
o
en numerosas reas de la f
a
sica: Electromagnetismo (ondas electromagnticas, luz,
e
ondas de radio, radar, etc.), Acstica (ondas sonoras), Sismolog (ondas s
u
a
smicas),
etc.

14.2.

Solucin de la ecuacin de ondas


o
o

En esta seccin analizaremos las soluciones de la ecuacin de ondas para una


o
o
cuerda de largo innito.
Proposicin: Sea f (z) una funcin arbitraria pero continua, con derivada continua
o
o
y acotada. Entonces f (x vt) es solucin de la ecuacin de ondas.
o
o

CAP
ITULO 14. ONDAS

521

Demostracin: Denotemos por f y f la primera y segunda derivada de f (respecto


o
a su argumento). Entonces
f (x vt)
x
2 f (x vt)
x2
f (x vt)
t
2 f (x vt)
t2

= f (x vt)
=

f (x vt)
= f (x vt)
x

= f (x vt) (v)
= v

f (x vt)
= vf (x vt) (v) = v 2 f (x vt)
t

De las ecuaciones anteriores se deduce que efectivamente f (xvt) satisface la ecuacin


o
de ondas.
Ejercicio: Sea g(z) una funcin arbitraria pero continua, con derivada continua y
o
acotada. Demuestre que g(x + vt) tambin es una solucin de la ecuacin de ondas.
e
o
o
Sea f (x) la funcin mostrada en la gura 14.3. Observe que el grco de f (x vt)
o
a
es igual al de f (x), excepto que se ha trasladado una cantidad vt hacia la derecha.
En otras palabras, f (x vt) corresponde a un pulso u onda que se traslada hacia
la derecha. Observe que no es la cuerda la que se traslada, sino que el pulsola
deformacin. Anlogamente g(x + vt) corresponde a una onda o pulso que se traslada
o
a
hacia la izquierda (hacia ).
x
f(x)

vt
f(xvt)
x

Figura 14.3
La velocidad con que estos pulsos se propagan a lo largo de la cuera es v =

Principio de superposicn
o

/.

CAP
ITULO 14. ONDAS

522

Tiempo

La ecuacin de ondas 14.1 es lineal, es deo


cir, si f1 (x, t) y f2 (x, t) son soluciones, entonces tambin u(x, t) = f1 + f2 , con y
e
arbitrarios es solucin (demustrelo como
o
e
ejercicio).
La gura 14.4 muestra la deformacin de
o
la cuerda al encontrarse dos pulsos que viajan en direccin contraria.
o

Figura 14.4
Ejercicio: Como caso particular, suponga que v = 1 cm/s, que las funciones f1 y f2
vienen dadas por f1 (s) = 1/(1 + s2 ) y f2 (s) = s/(1 + s2 ), y que = = 1. Graque
u(x, t) = f1 (x vt) + f2 (x + vt) en el intervalo x [10, 10] cm para t = 5, 1, 0,
+1 y +5 s.
Proposicin: Sea u0 (x) la deformacin de la cuerda en el instante t = 0. Si la
o
o
velocidad transversal de la cuerda en todos los lugares en ese instante es nula, entonces
la solucin de la ecuacin de onda que cumple estas condiciones iniciales es
o
o
u(x, t) =

1
[u0 (x vt) + u0 (x + vt)] .
2

Demostracin: Es inmediato que u(x, 0) = u0 (x). Por otra parte, derivando respecto
o
al tiempo se tiene
u(x, t)
1 u0 (x vt) u0 (x + vt)
1
=
+
= [vu (x vt) + vu (x + vt)] ,
0
0
t
2
t
t
2
de donde se deduce que u(x, 0) = 0.

Proposicin: (demustrela como ejercicio)


o
e
Sea u0 (x) la deformacin y v0 (x) la velocidad transversal de la cuerda en el inso
tante t = 0. Entonces la solucin de la ecuacin de onda que cumple estas condiciones
o
o
iniciales es
u(x, t) =

1
1
(u0 (x vt) + u0 (x + vt)) + (f (x + vt) f (x vt)) ,
2
2v

donde f (s) es una funcin tal que


o
df (s)
= v0 (s) .
ds
Ondas viajeras sinusoidales

CAP
ITULO 14. ONDAS

523

De particular importancia prctica son las ondas viajeras sinusoidales: cos(kx


a
t) y sen(kx t), donde k = /v. Las ondas electromagnticas emitidas por una
e
estacin de radio o las ondas superciales en el mar, la onda acstica emitida por un
o
u
diapasn, etc., en primera aproximacin, esencialmente son ondas viajeras sinusoidao
o
les. En este cap
tulo slo analizaremos la situacin de ondas sinusoidales viajando a
o
o
lo largo de una cuerda.
La constante k es el nmero de onda y es la frecuencia angular . La frecuencia
u
angular est relacionada con el per
a
odo T con que se repite la oscilacin en un lugar
o
jo: T = 2/. Es claro que vT = y, por lo tanto, el nmero de onda est relacionado
u
a
con la longitud de onda por k = 2/. La onda viajera sinusoidal tambin la
e
podemos escribir de la forma
cos(kx t) = cos(k(x vt)) .
Las ondas con el signo viajan hacia la derecha, mientras que las con el signo +
hacia la izquierda.
Analicemos ahora lo que pasa cuando la cuerda, digamos en el lugar x = 0,
cambia de densidad. Como las densidades son distintas, tambin lo sern las velocie
a
dades de propagacin. Sean stas v1 y v2 , a la izquierda y a la derecha de x = 0,
o
e
respectivamente.
Supongamos que una onda viajera sinusoidal, de amplitud 1 y frecuencia 0 ,
se acerca de la izquierda al punt ox = 0. Tal onda vendr dada por la expresin
a
o
cos(k1 x 0 t), con k1 = 0 /v1 .
Una vez que la onda llega al punto de interfase, lo que sucede es que parte de la
onda es transmitida y parte es reejada. En el regimen estacionario, la cuerda en el
lado izquierdo () y en el lado derecho (+), sern de la forma
a
u (x, t) = cos(k1 x t) + A cos((k1 x + 0 t))
y
u+ (x, t) = B cos(k2 x 0 t) ,
donde A es la amplitud de la onda reejada, mientras que B es la amplitud de la
onda transmitida. Note que la frecuencia de la onda no cambia al pasar de un medio
a otro; lo que cambia es el vector de onda (o equivalentemente, la longitud de onda).
Evaluemos A y B.
En todo instante las deformaciones de la cuerda u y u+ , para x = 0, deben
coincidir, o sea,
u (0, t) = u+ (0, t) .
A partir de esta ecuacin se encuentra que
o
1+A=B .

(14.2)

CAP
ITULO 14. ONDAS

524

Otra condicin que debe cumplirse es que en ningn punto (en particular en x = 0)
o
u
la cuerda puede tener una punta, es decir, en todas partes la derivada espacial de
la cuerda debe ser continua:
u (x, t)
x

=
x=0

u + (x, t)
x

.
x=0

Pero
u (x, t)
x

x=0

= k1 sen(k1 x 0 t) + Ak1 sen((k1 x + 0 t))

x=0

y
u (x, t)
x

= Bk2 sen(0 t) .
x=0

Luego la exigencia de que la cuerda no tenga una punta en x = 0 nos da la relacin


o
k1 (1 A) = k2 B .

(14.3)

De las ecuaciones (14.2) y (14.3) se encuentra que

1 2
k1 k2
v2 v1
A=
=
=

k1 + k2
v2 v1
1 + 2
y

2 1
2v2
2k1
=
=
B=
.
k1 + k2
v2 + v1
1 + 2

En las expresiones anteriores, 1 y 2 son las densidades de la cuerda a la izquierda


y derecha del punto x = 0, respectivamente.

CAP
ITULO 14. ONDAS

525

Casos particulares:
i) 1 = 2 . En este caso la densidad no sufre variaciones y nada ocurren en x =
0. La magnitud de la onda reejada es nula (A = 0), mientras que la onda
transmitida tiene la magnitud de la onda incidente (B = 1).
ii) 2 1 . En este caso la amplitud de la onda transmitida es casi nula (B 0),
mientras que la onda reejada tiene prcticamente la misma magnitud que la
a
onda incidente, pero con un cambio de fase de 180 (A 1).

iii) 2 = . Esta es la situacin que se tiene si la cuerda est empotrada en x = 0.


o
a
En ese caso no hay ona transmitida y la amplitud de la reejada es A = 1.
iv) mu2 1 . En este caso la amplitud de la onda transmitida es B 2, mientras que la onda reejada tiene prcticamente la misma magnitud que la onda
a
incidente.
v) mu2 = 0. En este caso a la derecha de x = 0 no hay nada, es decir la cuerda de la
ezquierda tiene un extremo libre en x = 0 (eso s debe haber algn mecanismo
,
u
que mantenga la tensin de la cuerda). Como no hay cuerda a la derecha, no
o
existe onda transmitida; slo hay una onda reejada de magnitud A = 1.
o
Energ transportada por una onda viajera sinusoidal
a
Si una masas m oscila armnicamente con una frecuencia angular y amplitud
o
A, su energ total (cintica ms potencial) viene dada por
a
e
a
E=

1
m 2 A2 .
2

Ejercicio: Demuestre expl


citamente que el resultado anterior se cumple para las
oscilaciones de una masa colgada de un resorte y para un pndulo.
e
Supongamos que a lo largo de una cuerda de densidad se desplaza una onda
viajera del tipo u(x, t) = A cos(kx t). Consideremos un segmento de la cuerda de
largo d. La masa de tal segmento es d. Cada segmento de la cuerda realiza un
movimiento armnico simple de amplitud A, luego su contribucin a la energ total
o
o
a
es
1
dE = (d) 2 A2 .
2
La energ total de un sector de la cuerda, de longitud igual a la longitud de onda, es
a
1
E = 2 A2 .
2

CAP
ITULO 14. ONDAS

526

Como la onda avanza aprecisamente una distancia en un intervalo de tiempo igual


al per
odo T , se deduce que la potencia, o sea, la energ por unidad de tiempo,
a
transmitida por la onda es
S=

dE
E
2 2 1
=
=
A = v 2 A2 .
dt
T
2T
2

Reexin en un extremo jo
o
Consideremos una cuerda tensa de densidad que va desde ahsta x = 0,
punto en el cual est empotradad. Que la cuerda en x = 0 signica que u(0, t) = 0, t.
a
Supongamos que un pulso f (x vt) viaja a lo largo de la cuerda hacia x = 0. (Para
t < 0 el pulso est a la izquierda de x = 0 y avanza hacia el origen; si la cuera no
a
estuviese empotrada, en el instante t = 0 el pulso pasar por el origen.) Qu pasa
a
e
con el pulso cuando ste se acerca al origen? Cmo evoluciona el pulso para t > 0?
e
o
Para contestar estas preguntas consideremos el siguiente problema: Una cuerda
que va desde hasta con dos pulsos convergiendo hacia x = 0, uno desde la
derecha y otro desde la izquierda (ver gura 14.5):
u(x, t) = f (x vt) f ((x + vt)) .
v
x
0
v

Figura 14.5
Note que la cuerda en x = 0 nunca se mueve; la superposicin de los dos pulsos,
o
en todo instante, da un desplazamiento total nulo en el origen. El lado izquierdo de
la cuerda del segundo problema es idntico al problema original: ambas satisfacen la
e
misma ecuacin de movimiento, tienen la misma condicin inicial para t (un
o
o
pulso f (x vt)) y ambas cuerdas tienen las mismas restricciones (u(0, t) = 0 t).
Como la solucin es unica, sabemos cmo se mover la cuerda del problema original
o

o
a
en todo instante: igual a la de la parte izquierda del segundo problema. para t 0,
a lo largo de la parte izquierda viajar un pulso invertido (f ((x + vt))) hacia la
a
izquierda.
Consideremos nuevamente la cuerda tensa de densidad empotrada en x = 0.
Pero ahora, en lugar de un pulso, supongamos que una onda viajera u(x, t) = cos(kx
t) viaja continuamente desde la izquierda hacia x = 0. La onda viajera sinusoidal,
en x = 0, se reejar, invirtiendo su amplitud. La deformacin de la cuerda, por lo
a
o

CAP
ITULO 14. ONDAS

527

tanto, ser
a
u(x, t) = cos(kx t) cos((kx + t)) = 2 sen(t) sen(kx) .
Esta ultima ecuacin ya no corresponde a una onda viajera, sino que a una onda

o
estacionaria: estamos en presencia de una onda sinusoidal sen(kx), cuya amplitud
2 sen(t) oscila en el tiempo. Para una onda estacionaria las crestas de las ondas no
se trasladan. Una onda estacionaria tiene nodos, que son los lugares de la cuerda que
se mantienen en reposo mientras transcurre le tiempo. La separacin entre dos nodos
o
es igual a la mitad de la longitud de la onda.
Reexin en un extremo libre
o
Consideremos una cuerda tensa de densidad que va desde hasta x = 0,
punto en el cual tiene un extremo libre. Aunque el extremo sea libre, debe existir
algn mecanismo que mantenga la tensin de la cuerda, por ejemplo, mediante un
u
o
hilo extremadamente no (de densidad despreciable) e innitamente largo. Que la
cuerda est libre en x = 0 signica que ella puede desplazarse verticalmente en x = 0,
e
pero debe cumplir la relacin
o
u(x, t)
x

=0,

t ,

x=0

o sea, en todo instante la tangente a la cuerda en x = 0 debe ser horizontal. Si esta


condicin no se cumpliera, sobre un pedazo de cuerda innitesimal en el extremo x = 0
o
actuaria una fuerza transversal innita, lo que generar aceleraciones ininitamente
a
grandes.
Supongamos que un pulso f (x vt) viaja a lo largo de la cuerda hacia x = 0.
Qu pasa con el pulso cuando ste se acerca al extremo libre? Cmo evoluciona el
e
e
o
pulso para t > 0?
Para contestar estas preguntas consideremos nuevamente una cuerda que va desde
hasta , con dos pulsos convergiendo hacia x = 0, uno desde la derecha y otro
desde la izquierda, tal como se muestra en la gura 14.6:
u(x, t) = f (x vt) + f ((x + vt)) .
v

v
x
0

Figura 14.6

CAP
ITULO 14. ONDAS

528

Observe que la pendiente de la cuerda en x = 0 de esta superposicin de pulsos


o
siempre es horizontal. Tambin observe que el lado izquierdo de la cuerda del segundo
e
problema es idntico al problema original: ambas satisfacen la misma ecuacin de
e
o
movimiento, tienen la misma condicin inicial para t (un pulso f (x vt)), y
o
ambas cuerdas tienen las mismas restricciones (pendiente horizontal en x = 0). Como
la solucin es unica, sabemos cmo debe moverse la cuerda del problema original: igual
o

o
que la parte izquierda de la cuerda del segundo problema. En particular concluimos
que para t 0, a lo largo de la parte izquierda viajar un pulso f ((x + vt)) hacia
a
la izquierda.
Repitamos lo anterior, pero ahora con una onda viajera u(x, t) = cos(kx t)
que incide continuamente desde la izquierda sobre el extremo libre. La onda viajera
snusoidal, en x = 0, se reejar manteniendo la misma amplitud. La deformacin de
a
o
la cuerda, por lo tanto, ser
a
u(x, t) = cos(kx t) + cos((kx t))
= 2 cos(t) cos(kx) .

Esta ecuacin corresponde nuevamente a una onda estacionaria: una onda sinusoidal
o
cos(kx), cuya amplitud 2 cos(t) oscila en el tiempo.

14.3.

Ondas estacionarias en una cuerda de largo L

Consideremos una cuerda de largo L, bajo tensin , cuyos extremos se encueno


tran empotrados en x = 0 y x = L.
Una caracter
stica de las ondas estacionarias es que poseen nodos, que son los
lugares de la cuerda que siempre estn en reposo. Al empotrar la cuerda en dos de
a
sus nodos, obtenemos las ondas estacionarias posibles en tal cuerda.
La distancia entre dos nodos siempre es /2, luego no todas las longitudes de
onda son posibles, sino que slo las que cumplen con
o
L=n

,
2

donde n es un entero positivo. Por otra parte, la longitud de onda y la frecuencia


= 1/T con que oscila la onda estacionaria estn relacionadas por
a
= vT =

v
v
= 2 .

De las relaciones anteriores se deduce que las frecuencias angulares posibles son:
n = 2

n
v
=
v.
n
L

CAP
ITULO 14. ONDAS

529

Ac v es la velocidad con que se propaga una onda transversal por la cuerda, magnitud
a
relacionada con la tensin y la densidad lineal de masa por
o

v=

La desviacin transversal de la cuerda cuando oscila en el modo estacionario


o
correspondiente al entero n es
un (x, t) = An sen(kn x) cos(n t) ,
donde An es la amplitud mxima de la oscilacin, kn es el nmero de onda
a
o
u
kn =

2
n
=
n
L

y n es la frecuencia angular
n = n

v
=n
L

con
1 =

= n1 ,

Observemos que el origen del tiempo se ha elegido en un instante en que la


cuerda ten su deformacin mxima y, por consiguiente, estaba momentneamente
a
o
a
a
en reposo. La expresin ms general para la desviacin transvesal de la cuerda cuando
o
a
o
oscila en el modo estacionario correspondiente al entero n es
un (x, t) = sen(kn x)(An cos(n t) + Bn sen(n t)).
Puesto que todas las partes de la cuerda oscilan armnicamente, los distintos
o
estados estacionarios corresponden a distintos modos normales del sistema. El modo
con n = 1 se llama modo fundamental y el modo correspondiente al entero n se llama
n-simo modo normal.
e
Resumen:: Para una cuerda de largo L con extremos jos en x = 0 y x = L, los
modos normales vienen dados por:
un (x, t) = An sen(nx/L) cos(n1 t) ,
con
1 =

n = 1, 2, 3, . . . ,

Ejercicio: Demuestre que para una cuerda de largo L con un extremo jo en x = 0


y un extremo libre en x = L, los modos normales de oscilacin vienen dados por
o
un (x, t) = An sen

(2n 1)x
2L

cos((2n 1)1 t) ,

n = 1, 2, 3, . . . ,

CAP
ITULO 14. ONDAS

530

con
1 =

2L

Ejercicio: Demuestre que para una cuerda de largo L con extremos libres en x = 0
y x = L, los modos normales de oscilacin vienen dados por
o
un (x, t) = An sen

nx
cos(n1 t) ,
L

con
1 =

n = 1, 2, 3, . . . ,

La gura 14.7 muestra esquemticamente la forma en la cual oscila una cuerda


a
en los distintos modos normales. La tabla 14.1 da un resumen de las frecuencias de
los distintos modos normales de una cuerda de largo L para las tres situaciones que
pueden presentarse: ambos extremos jos, ambos extremos libres, y un extremo jo
y el otro libre.

Extremos libres en x=0 y x=L

Extremo fijo en x=0


y libre en x=L

Extremos fijos en x=0 y x=L

n =1
0

n =2

n =3

n =4

Figura 14.7

CAP
ITULO 14. ONDAS

531

Cuadro 14.1Frecuencias de los distintos modos de oscilacin de una cuerda de largo L.


o
Modo de oscilacin
o
Fundamental
2o armnico
o
3er armnico
o
o armnico
4
o
5o armnico
o

n
1
2
3
4
5

Extremos jos
1
21
31
41
51

Extremos jo libre
1
31
51
71
91

Extremos libres
1
21
31
41
51

Frecuencia del
modo fundamental

1 = v/2L

1 = v/4L

1 = v/2L

Frecuencia del
modo n

n = n1

n = (2n 1)1

n = n1

14.4.

Desarrollo de Fourier

La ecuacin de ondas es lineal, es decir, una superposicin (suma) de soluciones


o
o
tambin es una solucin. En particular, una superposicin de los distintosmodos nore
o
o
males tambin ser un modo de oscilacin de la cuerda. Ms an, todo movimiento
e
a
o
a u
posible de una cuerda se puede describir como una superposicin de los distintos moo
dos normales. Por ejemplo, para una cuerda de largo L con extremos jos, cualquier
solucin se puede escribir de la forma
o
u(x, t) =

(An cos(n1 t) + Bn sen(n1 t)) sen(nx/L) .

(14.4)

n=1

Observe que las frecuencias de los distintos modos son mltiplos enteros de la freu
cuencia fundamental 1 y, por lo tanto, el movimiento resultante de la superposicin
o
necesariamente ser peridico. Los coecientes An y Bn se determinan de manera que
a
o
la solucin cumpla con las condiciones iniciales. Ilustremos este procedimientos con
o
un ejemplo.
Supongamos que se toma la cuerda desde el centro, se la desplaza una distancia
h y luego se suelta. En ese caso las condiciones iniciales son
u(x, 0) =

2h
Lx

2h

2h
Lx

para 0 < x < L/2


para L/2 < x < L

(14.5)

y
u(x, t)
t

=0.
t=0

(14.6)

CAP
ITULO 14. ONDAS

532

Derivando (14.4) se obtiene


u(x, t)
=
t

n1 (An sen(n1 t) + Bn cos(n1 t)) sen(nx/L) .

n=1

Evalundola en t = 0 se encuentra
a
0=

u(x, t)
t

=
t=0

n1 Bn sen(nx/L) .

n=1

La unica forma de satisfacer esta relacin para todo x es que todos los coecientes

o
Bn sean cero. De esta manera queda

u(x, t) =

An cos(n1 t) sen(nx/L) .

n=1

Para t = 0 se tiene
u(x, 0) =

An sen(nx/L) .

n=1

Multipliquemos ambos lados por sen(nx/L) e integremos sobre x entre x = 0 y


x = L. De esta manera se obtiene
L

u(x, 0) sen(nx/L) dx =
0

n=1

Pero

sen(nx/L) sen(nx/L) dx .

An
0

sen(nx/L) sen(nx/L) dx =
0

L
nn .
2

Ejercicio: Demuestre la ultima relacin. Indicacin: escriba las funciones seno en

o
o
trmino de exponenciales complejas e integre.
e
Tenemos, pues:
L

u(x, 0) sen(nx/L) dx =
0

n=1

L
L
An nn = An .
2
2

Despejando An se encuentra
An =

2
L

u(x, 0) sen(nx/L) dx .
0

As conociendo u(x, 0) podemos evaluar los coecientes An .


,

CAP
ITULO 14. ONDAS

533

Ejercicio: Demuestre que si u(x, 0) viene dado por (14.5), entonces


An =

0
n1
8h
(1) 2
2 n2

si n es par
.
si n es impar

Usando los resultados anteriores se encuentra qeu el desplazamiento de la cuerda


vendr dado por
a
u(x, t) =
n impar

n1
8h
n
(1) 2 cos(n1 t) sen
x
2 n2
L

8h
1

= 2 cos(1 t) sen
x cos(31 t) sen

L
9

3
1
x +
cos(51 t) sen
L
25

5
x
L

Esta serie nos da el desplazamiento de la cuerda en todos los lugares y en todos los
instantes.
La serie converge rpidamente, es decir,
a
bastan unos pocos trminos para obtener
e
un resultado relativamente preciso. La gura 14.8 muestra el resultado que se obtiene
con 8 trminos. De la gura se desprende que
e
despus de un per
e
odo T = 2/1 la cuerda volver a su posicin inicial. La situacin
a
o
o
mostrada en la gura 14.8 corresponde al movimiento de una cuerda de guitarra si sta se
e
pulsa exactamente al centro.
En el problema 14.22 se pide resolver este mismo problema pero usando el mtodo
e
grco introducido en la seccin 14.2.
a
o

14.5.

t= 0
t= 0.1 T
t= 0.2 T
t= 0.3 T
t= 0.4 T
t= 0.5 T

Figura 14.8

Problemas

1. Sabemos que si una onda viajera sinusoidal, que se propaga a lo largo de una
cuerda, llega a un punto donde cambia de densidad, entonces parte de la onda
es reejada y parte es transmitida. Demuestre que la energ transportada (por
a
unidad de tiempo) por la onda reejada ms la energ de la onda transmitida
a
a
coincide con la energ de la onda incidente.
a

CAP
ITULO 14. ONDAS

534

2. La gura 14.9 muestra la forma de un pulso


que en t = 0 viaja en una cuerda en el sentido positivo del eje x con rapidez v = 1 m/s.
Dibuje la forma de la cuerda en los instantes t = 1, 2, 3, 4, 5, y 6 segundos. En todos
los casos indique el sentido de la velocidad
transversal de todos los puntos de la cuerda.
En cada uno de los instantes, qu puntos del
e
pulso tienen velocidad nula?

111
000
111
000
111
000
111
000
111
000
111
000
111
000
111
000
111
000
111
000
111
000
111
000
111
000
111
000
111
000
111
000
111
000
111
000
111
000

Figura 14.9

3. Una cuerda homognea, de densidad y largo L, est ja en el extremo x = 0


e
a
y libre en el extremo x = L. Sea la tensin de la cuerda.
o
Usando tres alleres se clava la cuerda a una pared vertical de manera de producir una deformacin triangular centrada en x = L/2, tal como se muestra en
o
la gura adjunta.

alfiler
11
00
11
00
11
00
11
00
11
00
11
00
11
00
11
00
11
00
11
00
11
00
11
00
11
00
11
00
11
00
11
00
11
00
11
00
11
00
11
00
11
00
11
00
11
00

t= 0

b
2a
L

11
00
11
00
11
00
11
00
11
00
11
00
11
00
11
00
11
00
11
00
11
00
11
00
11
00
11
00
11
00
11
00
11
00
11
00
11
00
11
00
11
00
11
00
11
00

t> 0
v

2a

2a

b /2

Figura 14.10
En t = se liberan simultneamente los tres alleres e inmediatamente se obsera
va que la perturbacin original se desdobla en dos pulsos triangulares idnticos,
o
e
a
que viajan en sentidos opuestos con velocidad v = /. La amplitud mxima
de cada pulso es b/2 y su extensin 2a.
o
(a) Graque la forma de la perturbacin en t1 = L/v y en t2 = 2L/v.
o
(b) Haga un grco que indique la magnitud y direccin de la velocidad transa
o
versal de cada segmento de la cuerda en t1 = L/v y en t2 = 2L/v.

CAP
ITULO 14. ONDAS

535

4. Suponga que la funcin


o
u(x, t) = u0 sen[2(1x 5t)]
representa la deformacin de una cuerda, con u0 = 0.3 cm, y donde x se mide
o
en cent
metros y t en segundos. Encuentre la longitud de onda, el per
odo, la
velocidad, la direccin de propagacin, la amplitud de la onda y la energ
o
o
a
trasportada por la onda.
5. Entre dos torres de un andarivel, separadas por 100 metros, cuelga un cable de
acero cuyo peso es de 25 kg. Al recibir un golpe transversal cerca de uno de los
extremos, el pulso creado demora 3 s en volver. Cul es la tensin del cable?
a
o
6. Una cuerda de acero de longitud 80 cm y masa 10 g est ja en ambos extremos
a
y sometida a una tensin de 500 N.
o
Considere que la cuerda oscila en el modo fundamental y que la amplitud en el
punto medio es 0.3 cm.
(a) Encuentre la frecuencia con que oscila la cuerda. (Debido a que el acero
ofrece resistencia a las deformaciones, las frecuencias deducidas con las expresiones deducidas para cuerdas exibles no correspondern exactamente
a
a las de la cuerda de acero. Sern levemente mayores o menores?
a
(b) Escriba la funcin que describe la forma de la cuerda para todo valor de x
o
y para todo tiempo t. Suponga que en t = 0 todos los puntos de la cuerda
se encuentran en la posicin de mxima amplitud.
o
a
(c) Calcule la velocidad transversal de la porcin de la cuerda ubicada a 20 cm
o
de uno de sus extremos, en el instante en que la cuerda pasa por la posicin
o
de equilibrio.
7. Una cuerda de largo L y masa M cuelga del cielo.
(a) Muestre que un pulso transversal tarda un tiempo 2 L/g para recorrer
toda la cuerda.
(b) Usando el resultado de la parte (a), encuentre el tiempo que tarda el pulso
en viajar la mitad inferior de la cuerda.
(c) Cmo se modica el resultado si en el extremo inferior de la cuerda se
o
cuelga una masa m?
8. Un alambre de acero usado para confeccionar las cuerdas de un piano es capaz de
sostener un peso de aproximadamente 25.000 kg por cm2 de seccin transversal.
o
Cul es el mximo largo que podr tener la cuerda correspondiente al Do
a
a
a
central del piano? Depender este largo del grosor de la cuerda? Si la cuerda
a
tiene un dimetro de 0.8 mm, cul es la tensin de la cuerda? (La frecuencia
a
a
o
del Do central es de 262 Hz.)

CAP
ITULO 14. ONDAS

536

9. Sea v = / la velocidad de propagacin de ondas transversales en una cuerda,


o
donde es la tensin y la densidad lineal de la cuerda. Cules de las siguientes
o
a
expresiones corresponden a un movimiento posible de una cuerda con extremos
jos en x = 0 y x = L?
u(x, t) = u0 sen(kx) sen(t), con k = 2/L y = kv.
u(x, t) = u0 sen(k2 x) sen(2 t) + 2u0 sen(k4 x) sen(4 t), con n = n1 =
nv/L y kn = n /v.
u(x, t) = u0 sen(k2 x) sen(2 t) + 2u0 sen(k3 x) sen(3 t), con n = n1 =
nv/L y kn = n /v.
u(x, t) = u0 x(L x) sen(t), con = v/L.
u0
u0
u(x, t) =

.
1 + (x vt)2 1 + (x + vt)2
u(x, t) =

[f (2nL+xvt)f (2nLxvt)], donde f (s) es una funcin


o

n=

continua y derivable arbitraria (pero que para argumentos grandes decae


a cero).
En los casos en que el movimiento sea peridico, indique el per
o
odo.
10. La ecuacin = v relaciona la longitud de onda de una perturbacin armnio
o
o
ca con su frecuencia y su velocidad v. Esta relacin tambin es vlida para
o
e
a
las ondas electromagnticas que se propagan en el vac La velocidad con que
e
o.
se propagan las ondas electromagnticas es de c = 3 108 m/s.
e
(a) Cul es la longitud de onda de las ondas electromagnticas emitidas por
a
e
la radio Beethoven? La frecuencia de esa estacin de radio es de 96.6 MHz.
o
(b) Cul es la frecuencia de las ondas electromagnticas de longitud de onda
a
e
de 3 cm (microondas)?
(c) Para estudiar la estructura cristalina de los slidos frecuentemente se usan
o
rayos X (que tambin son ondas electromagnticas). La razn del uso
e
e
o
de tales ondas electromagnticas se debe a que su longitud de onda es
e
del orden de la separacin entre tomos vecinos del cristal. Sabiendo que
o
a
la separacin entre tomos vecnios en un cristal es de aproximadamente
o
a
2 = 2 108 cm = 0.2 nm, encuentre la frecuencia t
A
pica de una onda
electromagntica correspondiente a un rayo X.
e

CAP
ITULO 14. ONDAS
11. El pulso mostrado en la gura 14.11 se propaga hacia la derecha donde la cuerda tiene
un extremo libre. La velocidad del pulso es
de 1 m/s. Graque la forma de la cuerda en
los instantes t = 1, 2, 3, 4, 5 y 6 s. Haga un
grco de la velocidad del extremo libre en
a
funcin del tiempo.
o

537

Figura 14.11

12. Las frecuencias propias (o de resonancia) de tres modos normales de oscilacin


o
sucesivos de una cuerda exible son 60, 100 y 140 Hz, respectivamente.
(a) La cuerda tiene extremos jos, libres o uno jo y uno libre?
(b) A qu armnicos (o sea, especique los valores de n) corresponden estas
e
o
frecuencias de resonancia?
13. Una cuerda con ambos extremos jos tiene modos de resonancia (normales)
sucesivos, cuyas longitudes de onda son 0.54 m y 0.48 m.
(a) Cul es el n de estos armnicos?
a
o
(b) Cul es la longitud de la cuerda?
a
14. Un oscilador (por ejemplo diapasn) unido al extremo de una cuerda tensa,
o
genera en ella ondas transversales con una frecuencia de 500 Ha. La potencia
entregada a la cuerda por el oscilador es de 0.3 Watts. La densidad lineal de
masa de la cuerda es = 0.01 kg/m y est sometida a una tensin de 1000 N.
a
o
Encuentre la velocidad de propagacin, la longitud de onda y el nmero de
o
u
onda de la perturbacin generada. Encuentre la expresin que describe a la
o
o
perturbacin en la cuerda. Exprese la amplitud de la perturbacin en trminos
o
o
e
de las magnitudes numricas conocidas.
e
15. Dos cuerdas exibles de diferente densidad lineal estn unidad extremo con
a
extremo y sometidas a la misma tensin . La velocidad de propagacin para
o
o
ondas transversales en la primera cuerda es el doble que en la segunda. Cuando
una onda armnica que viaja en la primera cuerda es reejada en el punto de
o
unin entre ambas, la onda reejada tiene la mitad de la amplitud de la onda
o
transmitida. Si la amplitud de la onda incidente es A, cul es la amplitud de
a
las ondas reejada y transmitida? Qu fraccin de la potencia incidente es
e
o
transmitida?
16. Considere una cuerda de masa M y largo L, que se sujeta en un extremo y
se hace girar (como un aspa de helicptero) con una frecuencia angular 0 .
o
Demuestre que la velocidad de propagacin para ondas transversales a lo largo
o
de la cuerda es
0
L 2 x2 ,
v(x) =
2

CAP
ITULO 14. ONDAS

538

donde x es la distancia a lo largo de la cuerda medida desde el eje de rotacin.


o
17. Una cuerda de 4 m de largo, con ambos extremos jos, oscila en una superposicin del tercer, cuarto y sexto modo normal de oscilacin. La velocidad de
o
o
propagacin para ondas transversales es de 20 m/s.
o
(a) Cul es el per
a
odo de las oscilaciones de la cuerda?
(b) Cmo se podr eliminar el cuarto modo de oscilacin sin afectar a los
o
a
o
otros dos modos normales?
(c) Cunto vale el per
a
odo de oscilacin de la cuerda una vez que quede oscio
lando slo en una superposicin del tercer y sexto modo normal?
o
o
18. Demuestre que la velocidad de una onda transversal a lo largo de un resorte de
constante de restitucin k y estirado hasta tener un largo L, viene dada por
o
v=

k(L L0 )L
,
M

donde M es la masa del resorte y L0 su largo natural.


19. Considere una onda viajera que se propaga a lo largo de una cuerda de densidad
de masa y bajo tensin . Debido a procesos de disipacin, paulatinamente las
o
o
amplitudes de la oscilacin disminuirn en tamao. Suponga que la deformacin
o
a
n
o
de la cuerda viene dada por la relacin
o
u(x, t) = Aex cos(kx t) .
Encuentre la potencia transportada por esta onda en el punto x y comprela
a
con la potencia en el lugar x = 0.
20. Considere una cuerda de densidad y largo
L, sometida a una tensin , que en el instano
te t = 0 tiene la forma mostrada en la gura
y la velocidad transversal de cada porcin de
o
ella es nula. A medida que el tiempo avanza, la forma de la perturbacin cambia, pero
o
se observa que el movimiento es peridico.
o
Cul es el per
a
odo del movimiento? Justique su respuesta.

L
1
0
1
0
1
0
1
0
1
0
1
0
1
0

1
0
1
0
1
0
1
0
1
0
1
0
1
0

Figura 14.12

CAP
ITULO 14. ONDAS

539

21. Tres segmentos de cuerda de densidad estn


a
atados tal como se muestra en la gura 14.13.
Suponga que se conocen las distancias L1 y
L2 y el ngulo . Un pulso que parte de A
a
tarda un tiempo TB para llegar a B y un
tiempo TC para llegar a C. Encuentre la longitud de la cuerda L3 . Encuentre la tensin
o
de la cuerda L1 .

1111
0000
1111
0000
1111
0000
1111
0000
1111
0000
1111
0000
A
L1

L3

1
0

1
C 0
1
0
1
0
1
0
1
0
1
0

L2
B

11111
00000
11111
00000
11111
00000

Figura 14.13
22. Considere una cuerda de largo L que inicialmente tiene la forma mostrada en la gura 14.14 y cuya velocidad transversal es nula.
Determine grcamente la forma de la cuerda
a
en los instantes t = 0, t = 0.1T y t = 0.2T ,
donde T es el per
odo del movimiento de la
cuerda.

1
0
1
0
1
0
1
0
1
0
1
0
1
0

1
0
1
0
1
0
1
0
1
0
1
0
1
0

Figura 14.14

23. Suponga que cierta onda (armnica) se propaga a lo largo de una cuerda con una
o
velocidad v = 12 m/s (en la direccin +). La longitud de onda es = 0.4 m.
o
x
La densidad lineal de masa de la cuerda es 0 = 15 g/cm. La amplitud de la
onda es de 3 cm.
(a) Determine la frecuencia de la onda.
(b) Determine la tensin de la cuerda.
o
(c) Determine la mxima velocidad transversal de un punto de la cuerda.
a
(d) Determine la potencia propagada a lo largo de la cuerda.
(e) Determine la funcin u(x, t) que representa la deformacin de la cuerda en
o
o
el lugar x e instante t.
24. Considere una cuerda horizontal de longitud L, ja en ambos extremos, sometida a una tensin . En dicha cuerda, la densidad de masa a una distancia x
o
de uno de los extremos es
(x) = 0
donde 0 , 0 son constantes.

x
+ 0 ,

CAP
ITULO 14. ONDAS

540
( x)

Encuentre el tiempo T0 que tarda un pulso en llegar desde un extremo de la


cuerda al otro.
Encuentre una expresin para T0 vlida para 0 /0 1, y usando dicha expreo
a

sin, muestre que en el l


o
mite 0 0 se recupera el resultado esperado.
25. Considere una cuerda de largo L, densidad lineal de masa y sometida a una
tensin , libre por ambos extremos. Inicialmente, se le da a la cuerda una
o
deformacin gaussiana centrada a una distancia 2L/3 del extremo izquierdo,
o
como muestra la gura:

2L/3

L/3

Es decir, la deformacin de la cuerda est dada por:


o
a
2 / 2

u(x, 0) = Ae(x2L/3)

donde x es la distancia desde el extremo izquierdo, y A y son ciertas constantes.


En t = 0 se suelta la cuerda. Encuentre la deformacin de la cuerda para todo
o
tiempo t > 0, en trminos de la velocidad v de propagacin de ondas en la
e
o
cuerda.
Haga una representacin esquemtica de la forma de la cuerda para t = L/v y
o
a
t = 2L/v.
26. Determine el desarrollo de Fourier para un pulso cuadrado de altura 1 y ancho
1/2, sobre una cuerda de largo 1, ja en ambos extremos, como se muestra en
la gura:

CAP
ITULO 14. ONDAS

541
f(x)

0
1/4

3/4

Graque, en un mismo grco, el desarrollo de Fourier usando los primeros N


a
trminos de la serie, con N = 1, N = 3, N = 5 y N = 15.
e
27. Considere una cuerda de largo igual a 60 cm, ja en ambos extremos. La frecuencia del modo fundamental (primer armnico) es 1 . Cul es la frecuencia
o
a
del tercer armnico? Cul es la distancia entre dos nodos consecutivos cuando
o
a
se encuentra oscilando en dicho tercer armnico?
o
28. Considere una cuerda horizontal ja en uno de sus extremos y libre en el otro.
Sobre la cuerda se propagan dos pulsos triangulares (tringulos rectngulos) de
a
a
base a y altura b, en direcciones opuestas. Inicialmente los pulsos estn uno en
a
cada extremo de la cuerda, como muestra la gura:
11
00
11
00
11
00
11
00
11
00
11
00
11
00
11
00
11
00
11
00
11
00
11
00
11
00
11
00
11
00
11
00
11
00
11
00
11
00
11
00
11
00
11
00
11
00

b
a

a
L

Si la velocidad de propagacin de ondas en la cuerda es v y la distancia entre


o
ambos extremos es L, dibuje la forma de la cuerda en los tiempos L a /v,
2
L
a /v, L/(2v), (L a)/v, L/v y 2L/v.
2
2

CAP
ITULO 14. ONDAS

14.6.

542

Solucin a algunos de los problemas


o

Solucin al problema 3
o
Despus de transcurrido un tiempo t1 = L/v, tanto el pulso que viaja hacia la
e
izquierda como el que viaja hacia la derecha son reejados (por el extremo jo y libre
respectivamente), estando de nuevo en el punto de partida. Sin embargo, el reejado
por el extremo jo se habr invertido mientras que el reejado por el extremo libre
a
no. La suma (superposicin) de los dos pulsos, en el instante t1 , dar una deformacin
o
a
o
nula de la cuerda.
Despus de un tiempo t2 (medio per
e
odo para una cuerda con un extremo libre
y otro jo) el pulso ser el inverso del pulso inicial. La gura adjunta muestra la
a
deformacin de la cuerda y su velocidad transversal para varios instantes.
o

Deformacin

Tiempo
t=0

t=L/(8v)

t=3L/(4v)

t=L/v

t=5L/(4v)

t=7L/(4v)

t=2L/v

Figura 14.15

Velocidad

CAP
ITULO 14. ONDAS

543

Solucin al problema 5
o
La densidad lineal del cable es 0 = 25/100 kg/m = 0.25 kg/m. Si el pulso
demora 3 s en viajar 200 m, entonces la velocidad de propagacin es v = 200/3m/s =
o
66.6 m/s. Conocidos 0 y v podemos encontrar la tensin:
o
= v 2 = 0.25 (66.6)2 N 1109 N .
Solucin al problema 7
o
Sea z el eje vertical con z = 0 y z = L correspondiendo a los extremos inferior y

superior de la cuerda, respectivamente.


a) La tensin de la cuerda, que se debe a su propio peso, no es constante siendo, por
o
lo tanto, tambin la velocidad del pulso a lo largo de la cuerda no constante. Para
e
la tensin y densidad lineal de la cuerda se tiene (z) = M gz/L y = M/L. De
o
esta manera, para la velocidad del pulso, cuando se encuentra a una altura z, se
obtiene

= gz .
v(z) =

El tiempo dt que tarda el pulso en recorrer la distancia entre z y z + dz es dt =


dz/v(z). Sumando sobre todos los dz de la cuerda se encuentra
L

t0

dt =
0

1
dz ,
gz

o sea,
1
t0 =
g

z 1/2 dz = 2

z
g

L
,
g

=2
0

donde t0 es el tiempo que tarda el pulso en recorrer toda la extensin de la cuerda.


o
b) Usando la expresin anterior con L L/2 se encuentra el tiempo que tarda el
o
pulso en recorrer la mitad inferior. Observe que esto no es la mitad del tiempo
evaluado en la parte a).
c) Cuando a la cuerda se le agrega una masa m en su extremo la respuesta es
t0 = 2

L
g

1+

m
M

Demuestre que si la masa de la cuerda M es despreciable en comparacin con la


o
masa m que se le cuelga, entonces de la ecuacin anterior se obtiene el resultado
o
o
esperado t0 = L /0 , donde 0 = mg es, en ese caso, la tensin constante.

CAP
ITULO 14. ONDAS

544

Solucin al problema 9
o
Los requisitos que debe cumplir una funcin u(x, t) para que ella describa la
o
deformacin de una cuerda con extremos jos en x = 0 y x = L son:
o
i) u(x, t) debe ser solucin de la ecuacin de onda
o
o
2 u(x, t)
2 u(x, t)
v2
=0.
t2
x2
ii) u(0, t) = u(L, t) = 0 t.
Analicemos si cada na de las funciones propuestas cumple con estos requisitos.
a) u(x, t) = u0 sen(kx) sen(t), con k = 2/L y = kv, cumple con ambos requisitos
y porlo tanto corresponde aun movimiento posible de la cuerda. El periodo del
movimiento es T = L/v (la mitad del modo fundamental); la cuerda oscila en su
segundo armnico (n = 2).
o
b) u(x, t) = u0 sen(k2 x) sen(2 t) + 2u0 sen(k4 x) sen(4 t), con n = n1 = nv/L y
kn = n /v, cumple con ambos requisitos y por lo tanto corresponde a un movimiento posible de la cuerda. En este caso la cuerda oscila en una superposicin del
o
segundo (n = 2) y cuarto (n = 4) armnicos. El periodo es nuevamente T = L/v.
o
c) u(x, t) = u0 sen(k2 x) sen(2 t) + 2u0 sen(k3 x) sen(3 t), con n = n1 = nv/L y
kn = n /v, cumple con ambos requisitos y por lo tanto corresponde a un movimiento posible de la cuerda. En este caso la cuerda oscila en una superposicin
o
del segundo (n = 2) y tercer (n = 3) armnicos. El periodo ahora es el del modo
o
fundamental T = 2L/v.
d) u(x, t) = u0 x(L x) sen(t), con = v/L, cumple con el requisito ii), pero no
es solucin de la ecuacin de ondas, por lo tanto no corresponde a un movimiento
o
o
posible de la cuerda.
e)
u(x, t) =

u0
u0

,
1 + (x vt)2 1 + (x + vt)2

es solucin de la ecuacin de onda y cumple con la condicin u(0, t) = 0 t, pero


o
o
o
no cumple con u(L, t) = 0 para t = 0, luego no corresponde a un movimiento
posible de la cuerda.
f) u(x, t) =
o
n= [f (2nL + x vt) f (2nL x vt)], donde f (s) es una funcin
continua y derivable arbitraria, es solucin de la ecuacin de ondas. Tambin es
o
o
e

CAP
ITULO 14. ONDAS

545

evidente que cumple con u(0, t) = 0


0 t. En efecto,
u(L, t) =

n=

t. Demostremos que tambin u(L, t) =


e

f [(2n + 1)L vt]

n=

f [(2n 1)L vt] .

Cambiando en la segunda sumatoria el


ndice mudo n 1 = n se observa que
ambas sumatorias efectivamente son iguales. Dejaremos al lector que demuestre
que u(x, t + T ) = u(x, t) con T = 2L/v.
Solucin al problema 12
o
Del hecho de que las frecuencias 60, 100 y 140 no son mltiplos de enteros suu
cesivos de cierta frecuencia fundamental, se deduce qeu la cuerda tiene un extremo
libre y otro jo. Planteando las relaciones (2n 1)0 = 60 Hz, (2n + 1)0 = 100 Hz y
(2n + 3)0 = 140 Hz, se encuentra que 0 = 20 Hz y n = 2. Los modos normales que
corresponden a estas frecuencias son los con n = 2, 3 y 4, siendo sus frecuencias 30 ,
50 y 70 , respectivamente.

Cap
tulo 15

Ondas sonoras
versin 5 noviembre 2012
o

Se conoce como sonido a la sensacin que, percibida por el o


o
do, tiene su origen
en el movimiento vibratorio de los cuerpos, el cual es transmitido por un medio, por
ejemplo, el aire. En el vac no hay propagacin de sonido: para su propagacin, el
o
o
o
sonido requiere de un medio.

15.1.

Propagacin del sonido


o

La gura 15.1 muestra en forma esquemtica la distribucin, para varios instantes,


a
o
de las molculas del aire en una regin del espacio por la que atraviesa una onda
e
o
sonora. Los lugares en que el aire est comprimido (alta densidad de puntos) se
a
alternan con regiones en las que el aire est enrarecido (baja densidad de puntos).
a
Si en un instante observamos la densidad de aire (o sea, centramos nuestra atencin en una franja en particular) notaremos que sta var espacialmente en forma
o
e
a
peridica. La distancia entre dos mximos sucesivos de la densidad es la longitud de
o
a
onda.
Por otra parte, si en un lugar jo observamos las variaciones temporales de la
densidad del aire (o sea, en algn lugar realizamos un corte vertical de la gura 15.1),
u
observaremos que sta tambin variar peridicamente. El tiempo que transcurre
e
e
a
o
entre dos mximos sucesivos de la densidad es lo que se denomina per
a
odo de la onda.
Las siete franjas de la gura 15.1 corresponden a siete instantes equiespaciados dentro
de un per
odo.
Es importante comprender que al propagarse una onda de sonido, las part
culas
del aire realizan slo un pequeisimo movimiento oscilatorio. En la gura 15.1 se
o
n
han destacado tres molculas con un pequeo c
e
n rculo. Notemos que a medida que
transcurre el tiempo, las tres molculas oscilan, volviendo despues de un per
e
odo a
la posicin original. De hecho, la ultima franja de la gura 15.1 (correspondiente al
o

instante t = T ) es idntica a la primera (t = 0). Ms an, jndose en la gura 15.1


e
a u
a
546

CAP
ITULO 15. ONDAS SONORAS

547

con mayor detencin es posible seguir las oscilaciones de todo un grupo de molculas y
o
e
observar cmo el grupo oscila como un todo y es comprimido y luego descomprimido
o
al paso de una onda sonora. Esto es un efecto general y sucede en todos los lugares
de cualquier medio que transmite sonido.

Figura 15.1
En presencia de sonido, las part
culas del aire, en promedio, no se desplazan
una distancia neta, aun cuando el sonido contina alejndose a 340 m/s de la fuente
u
a
sonora. Algo anlogo ocurre con las olas en el agua: una gaviota que ota en el mar
a
slo describe un pequeo movimiento de ascenso y descenso al paso de una ola; aunque
o
n
la ola se propaga, sta no arrastra la gaviota con ella.
e

CAP
ITULO 15. ONDAS SONORAS

548

De qu tamao son las oscilaciones de las molculas de aire al paso de una onda
e
n
e
sonora? Para un sonido de 100 dB (decibeles) que, como veremos, corresponde aproximadamente a la intensidad del sonido de una orquesta sinfnica tocando fortsimo,
o

la amplitud de las oscilaciones es de slo aproximadamente 0.01 mm.


o
Es bien sabido que el sonido se propaga no slo a travs del aire, sino tambin
o
e
e
a travs de todos los cuerpos slidos y sustancias l
e
o
quidas y gaseosas. De hecho, la
mayor de los slidos y l
a
o
quidos son buenos transmisores del sonido.
No todas las sustancias propagan el sonido con la misma facilidad: existen uenos
y malos transmisores del sonido. Los gases y las sustancias porosas y sueltas, como el
algodn y la lana, son malos conductores del sonido, razn por la cual se usan como
o
o
aislantes acsticos. Por su parte, las sustancias poco compresibles, como el acero y el
u
agua, son buenas conductoras del sonido.

15.2.

Velocidad del sonido

Numerosas experiencias de la vida cotidiana ensean que el sonido se propaga a


n
una velocidad menor que la de la luz. Por ejmplo, el ruido producido por un avin a
o
turbina que pasa a gran altura, pareciera no provenir del lugar donde se ve el avin,
o
sino de uno ya superado. Otro ejemplo se presenta durante las tempestades elctricas.
e
Despus de caer un rayo, transcurre un lapso antes de que se escuche el trueno. De
e
hecho, es usual medir el tiempo que transcurre entre la observacin del relmpago y
o
a
el trueno, para estimar la distancia a que se encuentra la tormenta.
Los ejemplos anteriores muestran claramente que el sonido se propaga en el aire
con una velocidad nita. En el aire, a una temperatura de 15 C, el sonido se propaga con una velocidad de 340 m/s. Hemos indicado la temperatura del aire, porque
experimentalmente se encuentra que la velocidad de propagacin del sonido depeno
de ligeramente de ella por encima de los 0 CV, por cada grado que aumente la
temperatura, la velocidad se acrecienta en aproximadamente 0.6 m/s.
La tabla 15.1 recopila la velociad de propagacin del sonido en varias sustancias.
o
De ella inferimos que, en general, la velocidad del sonido es bastante mayor en los
l
quidos y slidos que en los gases, siendo una excepcin el gas de hidrgeno.
o
o
o
En el caso de los gases se observa que, a medida que aumenta su peso espec
co,
disminuye la velocidad de propagacin del sonido.
o
No es tan sencillo encontrar una relacin similar para la velocidad de propagao
cin del sonido en slidos. Adems de la densidad , el otro factor que determina la
o
o
a
velocidad del sonido es la elasticidad e del medio. Mientras ms r
a gido el material,
tanto mayor es e. Se puede demostrar que la velocidad del sonido (la cual en el presente cap
culo denotaremos con la letra c) en un medio viene dada por la relacin
o
o
c = e/. A grandes rasgos: a medida que la densidad del slido aumenta, la velocidad del sonido tiende a disminuir; sin embargo, sta no es una caracter
e
stica de
validez universal.

CAP
ITULO 15. ONDAS SONORAS

549

Hidrgeno (0 C)
o
1260
C)
Aire (0
331
Aire (100 C)
387
Anh
drido carbnido
o
254
Ox
geno (0 C)
316
Vapor de agua (130 C)
450
C)
Agua (20
1480
Alcohol
1168
Glicerina
1950
Madera
4500
Hielo
3200
C)
Cobre (20
3560
Aluminio
5040
Plomo
1200
Vidrio
hasta 5300
Acero
5200
Goma vulcanizada
35
Cuadro 15.1Velocidad del sonido en m/s.

En la seccin anterior hemos visto que en el aire el sonido se propaga en forma


o
de una onda de compresin y rarefaccin. Lo mismo es cierto para la propagacin del
o
o
o
sonido en todos los dems medios. Al paso de un sonido, el (usualmente pequeo)
a
n
desplazamiento oscilatorio de una part
cula del medio coincide con la direccin de
o
propagacin de la onda. A este tipo de ondas se las conoce por el nombre de ondas
o
longitudinales.

15.3.

La ecuacin de ondas
o

En esta seccin determinaremos la ecuacin dinmica que gobierna el comportao


o
a
miento de las ondas sonoras en un gas. Para ello usaremos algunas relaciones termodinmicas de los gases, que aqu presentaremos sin demostracin. La relacin
a

o
o
P V = N kB T

(15.1)

es la as llamada ecuacin de estado de los gases ideales, y relaciona el volumen V

o
que ocupan N molculas, con la temperatura T en (Kelvin) y la presin del gas P . kB
e
o
es una constante (la constante de Boltzmann) y su valor es kB = 1.3807 1023 J/K.
Sea m la masa de cada molcula, entonces
e
P =

kB
N m kB
T = T ,
V m
m

(15.2)

CAP
ITULO 15. ONDAS SONORAS

550

donde es la densidad de masa del gas. Esta ultima ecuacin muestra que para un

o
gas, a temperatura constante, la presin es proporcional a su densidad.
o
Al comprimir rpidamente un gas, su temperatura aumenta (es fcil observar esto
a
a
al usar un bomb de bicicleta). Cuando la compresin de un gas ocurre rpidamente
n
o
a
de manera que ste no alcance a termalizarse con su entorno, entonces las magnitudes
e
y P no son proporcionales sino qu ecumplen la relacin
o
P = constante ,

(15.3)

donde es una constante que depende de la naturaleza del gas. A temperatura ambiente, para el aire como tambin para el O2 y el N2 , = 1.40.
e
A continuacin, usando las ecuaciones anteriores, deduciremos la ecuacin dinmio
o
a
ca para las ondas sonoras. Supongamos que en cierto lugar del espacio, el aire es
perturbado levemente por una onda sonora que avanza a lo largo del eje x. La presin
o
del aire y su densidad sern entonces funciones de la posicin x y del tiempo t, que
a
o
designaremos por P (x, t) y (x, t), respectivamente.
Como la onda sonora genera compresiones y rarefacciones rpidas del aire, ste
a
e
no alcanza a termalizarse y, por lo tanto, se tiene que
P (x, t)(x,t) = constante .
Derivando respecto a x se obtiene
P
P
=
.
x
x

(15.4)

Cuando las variaciones son pequeas, como lo es en el caso del sonido, podemos
n
escribir
P
P0

,
(15.5)
x
0 x
donde P0 y 0 son la presin y densidad media del aire (los valores en ausencia de
o
sonido).
Consideremos ahora un tubo imaginario de aire, de rea A orientado a lo largo de
a
la direccin de propagacin del sonido. Cuando no hay sonido, las molculas de aire
o
o
e
estarn en reposo, es decir, no realizarn movimiento colectivo alguno. Por otra
a
a
parte, cuando hay sonido, las molculas de aire s realizarn un movimiento colectivo.
e

a
Sea u(x, t) el desplazamiento en el instante t de las molculas que estn en el plano
e
a
transversal que pasa por x, respecto a la posicin que ten en ausencia de sonido
o
an
(ver tambin la gura 15.2).
e

CAP
ITULO 15. ONDAS SONORAS

551

u(x, t )

u( x+ dx , t )

x+ dx

^
x

Figura 15.2
Apliquemos la segunda ley de Newton a un segmento de grosor dx. La masa del
aire del segmento es (A dx)0 , su aceleracin 2 u(x, t)/t2 , mientras que la fuerza
o
neta (ejercida por la presin atmosfrica) es
o
e
F = P (x)A P (x + dx)A = A

P (x, t)
dx .
x

La segunda ley de Newton nos da la relacin


o
A0

P (x, t)
2u
dx ,
dx = A
2
t
x

o sea,
0

2u
P0
dx =
dx ,
2
t
0 x

Pero
N = 0 A dx = (x, t)A[dx + u(x + dx, t) u(x, t)] ,
luego
(x, t) =

0
.
1 + u
x

Como |u/x| 1, tenemos que


(x, t) = 0 1

u
x

Derivando respecto a x se obtiene

2u
= 0 2 .
x
x

(15.6)

CAP
ITULO 15. ONDAS SONORAS

552

Reemplazando este resultado en (15.6), se obtiene


0
o sea

P0 2 u
2u
= 0 2 ,
t2
0 x

2u
2u
c2 2 = 0 ,
t2
x

(15.7)

con
c=

P0
.
0

(15.8)

La ecuacin (15.7) es la ecuacin dinmica que estbamos buscando: es la ecuacin


o
o
a
a
o
de ondas! Esta ecuacin es formalmente idntica a la ecuacin que obtuvimos en
o
e
o
el cap
tulo anterior para los desplazamientos transversales en una cuerda tensa. La
magnitud c, por lo tanto, es la velocidad de propagacin de las ondas de compresin
o
o
en el aire. Usando la ecuacin (15.2) podemos reescribirla de la forma
o
c=

kB T
.
m

(15.9)

Con esta relacin es posible entender algunas cosas. Por ejemplo, una molcula
o
e
de ox
geno pesa 16 veces ms que una molcula de hidrgeno y para el hidrgeno
a
e
o
o
= 1.41, valor casi idntico al del ox
e
geno. Luego, de acuerdo a (15.9), la velocidad
del sonido en el hidrgeno deber ser 4 veces mayor que en el ox
o
a
geno. Esto se cumple
extraordinariamente bien (ver tabla 15.1).
Conociendo la masa de una molcula de ox
e
geno, y midiendo podemos encontrar
la velocidad del sonido. La masa de un protn es 1.6725 1024 g, y por lo tanto, la
o
masa de una molcula de hidrgeno aproximadamante
e
o
mH2 3.34 1024 g .
Usando (15.9), para la velocidad del sonido (en hidrgeno) a una temperatura T =
o
C = 273 K, se obtiene
0
c=

m
1.41 1.38 1023 273 m
= 1261
,
27
3.34 10
s
s

resultado que prcticamente coincide con el valor experimental.


a

15.4.

Frecuencia

Llamaremos tono al sonido producido por fuentes sonoras que oscilan de manera
regular (oscilaciones sinusoidales). Lo ms caracter
a
stico de un tono es su bien denido
grado de elevacin o altura, fcilmente identicable.
o
a

CAP
ITULO 15. ONDAS SONORAS

553

La elevacin de un tono depende de la prefuencia de la perturbacin peridica


o
o
o
introducida por la fuente sonora en el medio en que el sonido se propaga. Los tonos
agudos corresponden a frecuencias mayores que los tonos graves.
Hay varias maneras de representar grcamente una onda sonora. Una
a
consiste en medir, en un lugar jo y
a medida que transcurre el tiempo, las
uctuaciones de la densidad del aire inducidas por un sonido. Por ejemplo, si
en el lugar en que se realiza la medicin se escucha un tono de una frecueno
cia bien denida, se encontrar que la
a
densidad del aire variar peridicamena
o
te alrededor del valor de equilibrio 0 .
Son estas reiterativas uctuaciones las
que se muestran esquemticamente en
a
la gura 15.3.
Una representacin grca distinta
o
a
se obtiene si las mediciones de la densidad se realizan en un unico instan
te, pero en lugares distintos (a lo largo de la direccin de propagacin). En
o
o
ese caso, un sonido de frecuencia bien
denida tambin corresponder a uce
a
tuaciones regulares de la densidad, uctuaciones que e repiten despus de una
e
distancia (la longitud de onda). La
gura 15.4 muestra esta representacin
o
para dos tonos de frecuencias distintas,
el sonido de frecuencia mayor se muestra en la parte inferior. A medida que la
frecuencia del sonido aumenta, la longitud de onda disminuye.

Figura 15.3

Figura 15.4
Para estudiar experimentalmente los sonidos y obtener un registro de ellos se
usa generalmente un micrfono, dispositivo que transforma las variaciones de presin
o
o
(que son proporcionales a las variaciones de densidad) en variaciones de una corriente
elctrica. Al amplicarlas, estas corrientes pueden ser desplegadas en un osciloscopio
e
o ser escuchadas con un parlante.

CAP
ITULO 15. ONDAS SONORAS

554

Cmo var la presin del aire, en funcin del tiempo, para distintos tipos de
o
a
o
o
sonidos? En la gura 15.3 se mostr en forma esquemtica cmo var la densidad del
o
a
o
a
aire (el grco para la presin es idntico pues ambas magnitudes son proporcionales),
a
o
e
a medida que trascurre el tiempo, para un tono de frecuencia bien denida.
Por otra parte, un ruido por
ejemplo, el ruido producido al llenar
una baera genera uctuaciones de
n
la presin, que var en este caso, irreo
a,
gularmente (ver gura 15.5).
Llamaremos tono puro o tono simple a un sonido para el cual el grco
a
de presin en funcin del tiempo viene
o
o
dado por una funcin sinusoidal; esto
o
signica que la presin P var a medio
a,
da que transcurre el tiempo, de acuerdo
a la expresin
o
Figura 15.5

P (t) = P0 + P cos(2t) .

(Denotaremos por P a la amplitud de


es un escalar y no un
la oscilacin; P
o
vector unitario.)
La gura 15.6 muestra un grco
a
presin-tiempo para un tono puro.
o
Cuando el tono no es simple y es,
por ejemplo, como el mostrado en la
gura 15.3, entonces se habla de tono
compuesto.

Figura 15.6
Es claro que la longitud de onda de un sonido es igual al producto de la velocidad
con que se propaga por su per
odo, o sea, es la distancia que alcanza a recorrer un
mximo de densidad en un per
a
odo:
= cT = c

1
.

(15.10)

La elevacin de un tono que escuchamos depende de la frecuencia con que vibra


o
la fuente sonora. El o humano es capaz de percibir tonos cuyas frecuencias var
do
an
entre 16 Hz y aproximadamente 20.000 Hz o sea, 20 kHz (kilohertz). Con la edad,
la posibilidad de escuchar frecuencias altas disminuye de manera signicativa, siendo
el l
mite superior para varones de edad avanzada de slo unos 5.000 Hz.
o

CAP
ITULO 15. ONDAS SONORAS

555

La gura 15.7 muestra el teclado completo de un piano moderno, indicando el


nombre que se da a las distintas notas, la frecuencia del sonido por ellas generadas
(en Hz) y la notacin musical que se usa para representarlas en un pentagrama.
o

Figura 15.7

15.5.

Intensidad

Hemos denido un tono como el sonido producido por una fuente sonora que
realiza oscilaciones peridicas. Adems de la frecuencia hay otras dos propiedades
o
a
bsicas que caracterizan a un tono; stas son su intensidad y su timbre. En la presente
a
e
seccin analizaremos algunos aspectos relacionados con la intensidad.
o
Como sabemos, un sonido puede tener un amplio rango de intensidades. Un sonido
dbil puede ser no audible; por otro lado, un sonido intenso puede producir dolor, e
e
incluso lelgar a romper el t
mpano.
Cmo podr
o
amos medir la intensidad de un sonido? Es bien sabido que un aumento de la presin del aire conlleva un aumento de su densidad. Esto permite usar
o
las variaciones de la presin del aire generadas por una onda sonora para caracterio
zar su intensidad. En la prctica se usa el valor efectivo (dado por la raz del valor
a

CAP
ITULO 15. ONDAS SONORAS
Fuente de sonido

556
Presin
o

Nivel de Potencia acstica


u
intensidad L relativa
Umbral de sonido
0.00002 Pa
0 dB 1
Respiracin
o
0.000063 Pa
10 dB 101
Ruido de hojas
0.0002 Pa
20 dB 102
Cuchicheo
0.00063 Pa
20 dB 103
Conversacin (separacin 1 m)
o
o
0.002 Pa
40 dB 104
Viol (pian
n
simo)
0.0036 Pa
45 dB 104.5
Ruido en sala de clase
0.0063 Pa
50 dB 105
Altoparlante (fuerte)
0.02 Pa
60 dB 106
Ruido callejero fuerte
0.063 Pa
70 dB 107
Grito fuerte
0.2 Pa
80 dB 108
Orquesta sinfnica (fort
o
simo)
2.0 Pa
100 dB 1010
Discoteca
6.3 Pa
110 dB 1011
Bocina de camin (a 1 m)
o
6.3 Pa
110 dB 1011
Turbina de avin
o
20.0 Pa
120 dB 1012
Martillo neumtico
a
20.0 Pa
120 dB 1012
Umbral del dolor
63.0 Pa
130 dB 1013
Cuadro 15.2Intensidad del sonido producido por varias fuentes.

cuadrtico medio)
a
Pe (x) =

(P (x, t) P0 )2 dt .

Para un tono
puro P (x, t) = P0 + p cos(kx t), el valor de la presin efectiva es

o
/ 2.
Pe (x) = P
En el sistema de unidades M KS, la unidad de presin es el Pascal. Recordemos
o
que un Pascal equivale a proximadamente un cien milsimo de la presin atmosfrica.
e
o
e
Cuando se escucha un tono, la presin atmosfrica que acta sobre el t
o
e
u
mpano
no es uniforme, sino que var peridicamente. Experimentalmente se encuentra que,
a
o
para que el o sea capaz de percibir un sonido, la variacin de la presin efectiva
do
o
o
debe ser al menos de unos 0.00002 Pa. Esta presin umbral es peque
o
nsima de slo
o
2 1010 atmsferas y muestra la extraordinaria sensibilidad de nuestro sistema
o
auditivo.
Otra unidad usada comnmente para caracterizar el nivel de intensidad de un
u
sonido es el decibel (dB). La tabla ?? muestra la magnitud de las variaciones de presin
o
atmosfrica generadas por varias fuentes de sonido, como tambin su equivalencia en
e
e
dB. El nivel de intensidad (para designarlo usaremos la letra L) viene denido por
L = 20 log10

Pe
Pu

dB ,

CAP
ITULO 15. ONDAS SONORAS

557

donde Pu 0.00002 Pa es la presin efectiva para el umbral de sonido.


o
La potencia acstica, por unidad de rea, viene dada por
u
a
I(x) =

2
Pe
,
0 c

donde c es la velocidad del sonido y 0 la densidad de masa del medio en que ste
e
se propaga. (La densidad del aire, en la vecindad de la supercie terrestre, a 20 C es
aproximadamente 0 = 1.29 kg/m3 ).
Tambin se muestra en la tabla la potencia acstica relativa de los distintos
e
u
sonidos.
Evaluemos Pe para una suma de dos perturbaciones sinusoidales:

P (x, t) = P0 + P1 cos(k1 x 1 t) + P2 cos(k2 x 2 t) .


Para esta magnitud al cuadrado se tiene
1
(P( x, t) P0 )2 dt
0
1

[P1 cos(k1 x 1 t) + P2 cos(k2 x 2 t)]2 dt


=
0
1 2
1 2
=
P1 cos2 (k1 x 1 t) dt +
P cos2 (k2 x 2 t) dt
0
0 2
2
P1 P2 cos(k1 x 1 t) cos(k2 x 2 t) dt
+
0
= (Pe,1 )2 + (Pe,2 )2 + trmino de interferencia
e

(Pe )2 =

Si las frecuencias son diferentes, el trmino de interferencia no contribuye ya que el


e
integrando es oscilatorio. De esta manera se obtiene que
(Pe )2 = (Pe,1 )2 + (Pe,2 )2 .
Como la intensidad por unidad de rea de un sonido es proporcional a (Pe )2 , se
a
obtiene que la intensidad de dos sonidos superpuestos es simplemente la suma de la
intensidad de cada uno de ellos por separado.
El rango dinmico del o es enorme: Entre el umbral de audibilidad y el uma
do
bral del dolor, las ondas sonoras dieren en un factor 1013 , en cuanto a la energ
a
transportada por unidad de tiempo.
Cabe sealar que la sensibilidad del o depende fuertemente de la frecuencia del
n
do
sonido (ver gura 15.8), sienod mxima para frecuencias de entre 500 y 6.000 Hz. El
a
o es relativamente sordo para frecuencias mucho ms altas o mucho ms bajas que
do
a
a
stas. Por ejemplo, para un sonido de 20 Hz, la variacin m
e
o
nima de presin requerida
o
para o es casi 104 veces mayor que para un sonido de 1000 Hz. Las l
rlo
neas continuas
en la gura 15.8 unen puntos correspondientes a est
mulos sonoros de tonos puros que

CAP
ITULO 15. ONDAS SONORAS

558

el o humano percibe como de la misma intensidad. As un sonido de 20 Hz debe


do
,
tener una intensidad de alrededor de 90 dB para que se peciba como si tuviera la
misma intensidad que un sonido de 1000 Hz a 30 dB.

Figura 15.8
Lo expuesto en los prrafos anteriores corresponde a resultados que pueden obtea
nerse experimentalmente, midiendo las propiedades de los sonidos con instrumentos
f
sicos. Esto, sin embargo, no siembpre coincide necesariamente con lo que una persona percibe. Analicemos entonces lo que subjetivamente tal persona escucha en lo
que a intensidad del sonido se reere.
Si consideramos un sonido de 1000 Hz, para un humano, la intensidad del sonido
aumenta aproximadamente al doble cada vez que hay un aumento de intensidad de
10 dB. O sea: de dos sonidos que dieren en 20 decibeles, uno es percibido como si
fuera cuatro veces ms intenso que el otro; si dieren en 30 decibeles, uno parece ser
a
ocho veces ms intenso que el otro. La sensacin de intensidad es proporcional al
a
o
logaritmo del est
mulo!

15.6.

Propagacin del sonido


o

Reexin y absorcin
o
o
Volvamos a las ondas sonoras y analicemos algunos fenmenos que pueden modio
carlas en su trayecto desde la fuente sonora hasta su deteccin por nuestro sistema
o
auditivo. Uno de ellos es la reexin. Una onda de sonido se reeja especularmente a
o

CAP
ITULO 15. ONDAS SONORAS

559

Material

Frecuencia (Hz)
125 250 500 1000 2000 4000
Pared de ladrillos
0.02 0.03 0.03 0.04 0.05 0.06
Pared de ladrillos estucada y pintada
0.01 0.01 0.01 0.01 0.02 0.02
Paneles de madera terciada
0.60 0.30 0.10 0.09 0.09 0.09
Piso de madera
0.15 0.11 0.10 0.07 0.06 0.07
Cortinaje grueso
0.14 0.35 0.55 0.72 0.70 0.66
Alfombra gruesa sobre piso de concreto 0.02 0.06 0.15 0.40 0.60 0.60
Vidrio de una ventana
0.30 0.20 0.20 0.10 0.07 0.04
Butaca (sin ocupar)
0.20 0.40 0.60 0.70 0.60 0.60
Butaca ocupada
0.40 0.60 0.80 0.90 0.90 0.90
Silla metlica o de madera
a
0.02 0.03 0.03 0.06 0.06 0.05
Cuadro 15.3Coeciente de absorcin de sonido para algunos materiales. Un coeciente
o
de absorcin igual a 1 signica absorcin total.
o
o

lchocar con un objeto de al menos el tamao de su longitud de onda, interpuesto en


n
su camino. El sonido se reeja bien en supercies duras y r
gidas, y mal en supercies
porosas, blandas y deformables. Al incidir el sonido sobre, por ejemplo, un cortinaje,
hay varios fenmenos que causan que se absorba la energ de la onda sonora, siendo
o
a
el ms importante el debido al roce viscoso con el aire.
a
Para vislumbrar lo que ocurre en el interior del cortinaje ilustremos primero el
efecto del roce en un caso ms familiar. Consideremos un r no muy turbulento.
a
o
A pesar de que el r uye y en el centro su velocidad puede llegar a ser de varios
o
metros por segundo, cerca de la orilla el agua estar prcticamente en reposo. Se tiene
a a
pues que en distintos sectores del r el agua avanza con distintas velocidades. Este
o
movimiento relativio de aguas vecinas ocasiona roce y disipa energ de movimiento.
a
(Si no fuera por este roce, el agua de un r deber correr cada vez ms rpido a
o
a
a a
medida que se acerca al mar.) Algo anlogo ocurre con el aire del cortinaje. El aire
a
muy cercano a las bras est siempre prcticamente en reposo. El sonido que incide
a
a
sobre el cortinaje slo puede inducir a moverse a las molculas del aire que no estn
o
e
a
muy cerca de las bras. Por consiguiente, en el interior del corinaje hay distintos
sectores del aire con distintas velocidades; la friccin, consecuencia de la viscosidad
o
del aire, se encarga de disipar la energ de la onda sonora.
a
Existen adems otros mecanismos que contribuyen a la atenuacin de una onda
a
o
sonora cuando es reejada por una supercie. Sin entrar en ms detalles, mencionamos
a
algunos de ellos:
a) La transmisin trmica de energ desde la onda sonora a la supercie reectora.
o e
a
b) La conduccin de sonido desde el reector hacia otras estructura y ambientes.
o

CAP
ITULO 15. ONDAS SONORAS

560

c) La deformacin no elstica que la onda sonora reeja en el reector.


o
a
La tabla 15.4 muestra la fraccin de la energ de un sonido que es absorbida al
o
a
reejarse en diversos materiales. Se observa que, para un sonido de 1000 Hz, una
pared slida de ladrillos reeja del orden de un 98 % del sonido que incide sobre ella,
o
mientras que un cortinaje grueso absorbe cerca de un 72 %. Por ello en una sala
de conciertos hay que evitar exagerar en felpas y alfombras para disminuir el ruido
ambiente, ya que esto podria causar serios problemas de acstica, al eliminarse, en
u
muy alto grado, los rebotes de los sonidos en la sala.
Observemos cmo distintos materiales absorben de manera distinta a los sonidos.
o
Por ejemplo, una pared de madera terciada absrobe ecientemente los sonidos de
frecuencia baja y slo en un grado mucho menor los de frecuencias altas. Lo contrario
o
ocurre con un cortinaje grueso: ste es un absorbente ms efectivo para sonidos de
e
a
frecuencias altas.
Acstica de salas
u
Volvamos a la reexin de las ondas sonoras. Como es fcil imaginarse, tal reexin
o
a
o
juega un papel importante en la acstica de salas y piezas. La percepcin sonora en
u
o
una sala de conciertos depende de la intensidad y de la relacin temporal entre el
o
sonido directo el que viaja en l
nea recta desde la fuente sonora hasta el receptor
y el sonido indirecto el reejado por las paredes de la sala.
Emp
ricamente, una diferencia de tiempo entre el sonido directo y el indirecto
menor que 0.05 s resulta ser acsticamente favorable. Las reexiones en este caso no
u
son molestas para entender la voz hablada, todo lo contraio, aumentan la intensidad
del sonido que llega al o y, en el caso de la msica, favorecen la amalgamacin de
do
u
o
los sonidos, contribuyendo al colorido musical.
Por otra parte, diferencias de tiempo entre el sonido directo y el indirecto aproximadamente mayores que 0.05 s son perjudiciales para entender la voz hablada y
para la ptima percepcin de la msica, si ambos tienen intensidades similares. En
o
o
u
efecto, 0.05 segundos es aproximadamente el tiempo m
nimo que requiere el o para
do
percibir el sonido original y el reejado como dos sonidos diferentes. Si el tiempo es
menor, los dos sonidos, el directo y el indirecto, se confunden y el o los acepta
do
como uno solo.
Ejercicio
Demuestre que lo expuesto en el prrafo anterior limita el radio de las conchas acstia
u
cas a aproximadamente ocho metros.
Cuando el sonido es reejado en forma reiterativa se tiene lo que se conoce como
reverberacin. Paredes paralelas que reejan bien el sonido favorecen este fenmeno.
o
o
La reverberacin en las salas de conciertos se puede minimizar recurriendo a suo
percies inclinadas o absorbentes. Cuando la reverberacin es excesiva, situacin que
o
o
ocurre en la mayor de las grandes iglesias, los distintos tonos de una melod comiena
a

CAP
ITULO 15. ONDAS SONORAS

561

zan a confundirse y si, por ejemplo, hay un coro cantando, al pblico le parecer que
u
a
la ejecucin es poco precisa. La reverberacin puede incluso llegar a ser tan seria que
o
o
termina por desorientar a los mismos msicos.
u
La gura 15.9 muestra un caso t
pico de la evolucin temporal a la que est exo
a
puesto un oyente en una sala de conciertos cuando en el escenario algn msico toca
u
u
con su instrumento un sonido continuo durante el intervalo temporal [ti , tf ].

Figura 15.9
A pesar de que el sonido comienza a emitirse en el instante ti , ste llega recin en
e
e
el instante td hasta el oyente. El producto de la velocidad del sonido c por la diferencia
de tiempo (td ti ) es igual a la distancia L que separa al oyente del msico con su
u
instrumento, o sea, L = c (td ti ).
Analicemos detalladamente la gura 15.9. Durane el intervalo de tiempo [ti , td ],
el oyente slo escucha el ruido de fondo, que necesariamente siempre est presente.
o
a
Para el caso mostrado en la gura, ste es de unos 30 dB. En el instante td , el
e
oyente percibe el sonido directo del instrumento; supondremos que la intensidad del
sonido directo corresponde a unos 70 decibeles. Si la sala no tuviese paredes o un
cielo, el nivel de intensidad se mantendr en ese valor. En una sala, sin embargo,
a
las sucesivas reexiones incrementarn paulatinamente la intensidad del sonido. En
a
el instante tr llega al oyente el primer sonido reejado. Tal como ya se seal, es
n o
importante que el lapso transcurrido entre td y tr no sea superior a 1/20 de segundo,
para que el oyente no perciba el escaln. Para el ejemplo mostrado en la gura,
o
la primera reexin aumenta la intensidad del sonido en unos 3 decibeles (lo que
o

CAP
ITULO 15. ONDAS SONORAS

562

equivale a aproximadamente un factor 2 en la energ sonora). Sucesivas reexiones


a
mltiples siguen aumentando la intensidad hasta que la energ por unidad de tiempo
u
a
del sonido absorbido por las paredes, el alhajamiento y las personas presentes en la
sala iguale a la potencia sonora que emana del instrumento. Para nuestro ejemplo,
esto ocurre cuando la intensidad del sonido (en el lugar en que se encuentra el oyente)
llega a los 80 decibeles. Ntese que la energ sonora en este nivel de intensidad es 10
o
a
veces mayor que la del sonido directo (que slo era de 70 dB), en otras palabras, las
o
mltiples reexiones del sonido son fundamentales para incrementar su intensidad.
u
De acuerdo aon nuestras supsciciones, el msico deja de tocar su instrumento el
u
d deja de llegar el sonido directo hasta
instante tf . Sin embargo, recin en el instante t
e
el oyente, percibiendo ste una pequea dsiminucin en la intensidda (recordemos
e
n
o
que para el ejemplo mostrado en el grco, el sonido directo es slo un 10 % de la
a
o
r , deja de llegar la primera reexin
intensidad total). Posteriormente, el el instante t
o
producindose una nueva disminucin de la intensidad. El sonido seguir siendo toe
o
a
davia bastante intenso ya que (como se ha indicado) la mayor parte de la intensidad
proviene de las reexiones mltiples (la reverberacin). El decaimiento del sonido es
u
o
relativamente lento.
Es usual caracterizar la reverberacin por el tiempo T que tarda la intensidad de
o
un sonido en decaer 60 dB una vez que se apaga la fuente de sonido. Para el ejemplo

mostrado en la gura 15.9, T = t td , o sea, aun cuando la intensidad no puede


disminuir en 60 decibeles debido al ruido de fondo, para determinar el tiempo de
reverberacin se extrapola el decaimiento de la intensidad como si el ruido de fondo
o
no existiese, hasta que la disminucin es de 60 dB.
o
En contraposicin a lo que ocurre en una sala, al aire libre, la ausencia de reexioo
nes mltiples hace que el sonido que se esxcucha sea mucho menos intenso. Tambin,
u
e
una vez que el msco ha dejado de tocar y el sonido directo deja de llegar al oyente
u

(o sea, para instantes posteriores a td ), la intensidad del sonido abruptamente baja


hasta coincidir con el ruido de fondo.
Hoy en d las salas de conciertos son diseadas para que tengan cierta determia,
n
nada reverberacin (un valor considerado ptimo para el tipo de espectaculos para
o
o
el que se est diseando la sala). El valor ptimo para msica de cmara es menor
a
n
o
u
a
que para una gran orquesta u pera. Naturalmente, la cantidad de pblico presente
o
u
en la sala tambin afecta a este valor. Una reverberacin optima ayuda a que los
e
o
distintos ejecutantes de una pieza musical se escuchen mutuamente. Cuando no hay
reverberacin, el sonido parece no tener vida, se desvanece rpidamente y los distintos
o
a
ejecutantes pueden tener dicultades para escucharse mutuamente, hacindose dif
e
cil
una ptima presentacin (situacin usual en conciertos realizados al aire libre).
o
o
o
Se ha determinado que, para frecuencias de entre 500 Hz y 1000 Hz, el valor
o
ptimo de T para una sala de conciertos es de entre 1.6 y 2 segundos. El valor t
pico
de T para una sala de clases es de 0.5 s; para la faosa sala de peras Alla Scala de
o
Miln, T 1.5 s; para la del Teatro Coln de Buenos Aires, considerada entre las
a
o
cinco mejores salas de conciertos del mundo, T 1.8 s; y para la inmensa Catedral

CAP
ITULO 15. ONDAS SONORAS

563

de Colonia, este valor es T 13 s.


Es claro que el tamao de una sala afecta directamente el tiempo T : cuando ms
n
a
grande la sala, tanto mas demora el sonido en viajar entre las paredes y, por lo tanto,
el tiempo de reverberacin T aumenta. Una sala para msica de cmara, que para
o
u
a
su ptima ejecucin requiere tiempos de reverberacin del orden de 1 s, debe tener
o
o
o
un volumen de entre 500 y 5000 m3 . Para la representacin de una pera, el tiempo
o
o
de reverberacin ptimo es de alrededor de 1.6 s, lo que se logra en salas con un
o o
volumen de entre 10000 y 30000 m3 . En salas grandes jhay que cuidar que el tiempo
transcurrido entre el sonido directo y la primera reexin no sea mayor que 1/20 s (de
o
otro modo, los dos sonidos no se amalgaman sino que se escuchan como dos sonidos
separados). Esto usualmente se logra suspendiendo desde los cielos de la sala y del
escenario, grandes paneles reectores. Los paneles ubidcados sobre el escenario son
adems fundamentales para que los msicos se escuchen bien unos a otros. Si estos
a
u
panesles se encuentran suspendidos a ms de 6 m, se hace muy dif ejecutar una
a
cil
obra musical en forma completamente coordinada.
Atenuacin del sonido
o
La experiencia cotidiana muestra que, a
medida que nos alejamos de una fuente de
sonido, su intensidad disminuye. La principal razn de esto es puramente geomtrica.
o
e
Si rodeamos la fuetne sonora con cscaras
a
esfricas concntricas imaginarias (ver gue
e
ra 15.10, una de radio R y otra de radio 2R,
observamos que la onda sonora, al llegar a
la cscara exterior, debe repartirse sobre una
a
supercie mucho mayor que sobre la cscara
a
interior.
Figura 15.10
Como el rea de la esfera exterior es cuatro veces el rea de la interior, la razn
a
a
o
de la potencia sonora por unidad de area entre ambas esferas, tambien ser cuatro.
a
Como 3 decibeles corresponden a un factor 2 en la potencia acstica, un factor 4
u
corresponder a 6 dB. O sea, al alejarnos de la fuente sonora aumentando la distancia
a
al dobrle, la intensidad del sonido, por razones puramente geomtricas, disminuye en
e
6 decibeles.
En general, para una fuente puntual, en ausencia de reexiones yrefracciones, la
intensidad del sonido diminuye, por razones geomtricas,s en forma proporcional al
e
inverso de la distancia al cuadrado.
La viscosidad del aire tambin afecta a la intensidad de la onda sonora en su
e
propagacin libre. Como ya sabemos, un sonido genera pequeas oscilaciones de fraco
n

CAP
ITULO 15. ONDAS SONORAS

564

ciones de un mil
mietro de las molculas del aire (recordemos las oscilaciones de las
e
tres molculas mostradas en la gura 15.1). Al analizar cuidadosamente esa gura se
e
observa que laas molculas no oscilan en la misma direccin en el mismo instante; de
e
o
hecho, moluclas ubicadas en regiones separadas por media longitud de onda siempre
e
tienen velociades opuesta. Esto signica que tambin durante la propagacin libre de
e
o
una onda sonora hay, entre distintos sectores del medio, velocidades relativas. Pero,
como ya hemos visto, cuando distintos sectores de aire se mueven unos respecto a los
otros, debido a la viscosidad habr necesariamente disipacin de energ (roce).
a
o
a
Afortunadamente para sonidos de inters musical, estas regiones o sectores en
e
movimiento relativo estn relativamente distantes unas de las otras (separados por
a
media longitud de onda) y, por consiguiente, el efecto de la viscosidad es mucho
menor que en el caso del cortinaje mencionado anteriormente, donde las distancias
involucradas eran las que hay entre bras vecinas. Sin embargo, cuando la longitud
de onda es pequea (como ocurre en los sonidos de altas frecuencias), la absorcin de
n
o
la intensidad debida a la viscosidad del aire es importante.
Frecuencia del Sonido
La tabla adjunta muestra la
distancia que alcanza a recorrer
el sonido en el aire antes de perder la mitad de su intensidad debido al roce viscoso (en la tabla
no est considerado el efecto de la
a
disminucin de la intensidad del
o
sonido debido al factor geomtrie
co).

Distancia

1 kHz
10 kHz
60 kHz
100 kHz
1 MHz

22.2 km
220 m
6.14 m
2.2 m
2.2 cm

Notemos cmo la distancia disminuye rpidamente a medida que aumenta la


o
a
frecuencia del sonido. Esto explica por qu slo se percibe el retumbar grave como
e o
efecto de una fuerte explosin ocurrida lejos.
o
Superposicin de ondas sonoras
o
El hecho que la ecuacin de ondas sea lineal, signica que una combinacin lineal
o
o
de soluciones tambin ser solucin. Esto se conoce con el nombre de principio de
e
a
o
superposicin.
o

CAP
ITULO 15. ONDAS SONORAS

565

El principio de superposicin permite eno


tender un fenmeno interesante. En la guo
ra 15.11 se muestra la representacin grca
o
a
de la presin de dos sonidos que tienen la
o
misma amplitud y casi la misma frecuencia
(situacin que se presenta a menudo al ao
nar un piano o una guitarra). Cmo percibe
o
el o el sonido de ambas cuerdas pulsadas
do
simultneamente?
a

Figura 15.11
De acuerdo al principio de superposicin, debemos sumar ambas perturbaciones.
o
Tambin se muestra tal suma en la gura. Obserbamos que la suma consiste en oscie
laciones con esencialmente la misma frecuencia que las ondas originales, moduladas
por una oscilacin de frecuencia mucho menor. La amplitud de la onda-suma no es
o
constante sino que var peridicamente.
a
o
Ejercicio:
Muestre que la suma de ambas ondas consiste en rpidas oscilaciones con frea
cuencia = (1 + 2 )/2, cuya amplitud viene modulada por una frecuencia mucho
ms lenta, igual a (2 1 ).
a
Estas variaciones de amplitud de la onda suma originan una variacin en la
o
intensidad del sonido que es fcil de percibir y se conoce con el nombre de pulsaciones
a
o batimentos. Al anar una guitarra, dos cuerdas estarn bien anadas si al hacer
a
sonar con ellas simultneamente la misma nota, no se escuchan pulsaciones. Si, por
a
ejemplo, se perciben pulsaciones de 2 Hz, ser porque la frecuencia de las dos cuerdas
a
dieren en esa magnitud.

15.7.

Timbre

Una de las caracter


sticas de un tono que an no hemos abordado es el timbre, la
u
propiedad que permite distinguir, por ejemplo, entre una nota tocada por una auta
y la misma nota ejecutada por un viol
n.
Al pulsar, por ejemplo, una cuerda de guitarra, el movimiento resultante no corresponder al asociado a un modo normal puro, sino que ser el resultado de una
a
a
superposicin de numerosos armnicos. Aun cuando el movimiento de la cuerda se deo
o
ba a una suma de distintos modos de oscilacin (de frecuencias 1 , 21 , 31 , 41 , etc.),
o
el movimiento resultante tendr la frecuencia 1 , siendo, por consiguiente, tambin
a
e
sa la frecuencia del tono generado por ella.
e
Ser posible invertit la argumentacin anterior? Al observar un movimiento pea
o
ridico complejo de frecuencia 1 , podr descomponerse este movimiento en una
o
a
superposicin de modos normales que tengan frecuencias que sean mltiplos enteros
o
u

CAP
ITULO 15. ONDAS SONORAS

566

de 1 ? La respuesta es s procedimiento que se conoce con el nombre anlisis de


,
a
Fourier. (Esencialmente coincide con lo realizado en la seccin 16.5).
o
Mostremos algunos ejemplos concretos. En las guras 15.12, 15.13 y 15.14 se presenta la descomposicin de Fourier (o descomposicin espectral) de tres movimientos
o
o
peridicos compuestos (por ejemplo, la presin de tonos musicales en funcin del
o
o
o
tiempo).
En cada caso, en la parte superior se despliegan las variaciones de la presin (en
o
funcin del tiempo) generadas por los tonos y en la parte inferior, su descomposicin
o
o
en armnicos. En todos los casos se graca la funcin original sobre un intervalo temo
o
poral que abarca cuatro per
odos completos. Para simplicar la discusin, nos hemos
o
restringido a mostrar ejemplos para los cuales la variable original es una superposicin de a lo sumo los cuatro primeros armnicos. Al lado derecho de cada uno de los
o
o
armnicos, se indica la amplitud con que contribuye al tono original.
o

Figura 15.12

CAP
ITULO 15. ONDAS SONORAS

567

Figura 15.13

Figura 15.14
;
Observe que:
El tono compuesto resultante de la superposicin de los armnicos es siempre un
o
o
tono cuya frecuencia coincide con la del primer armnico (ambos tienen la misma
o

CAP
ITULO 15. ONDAS SONORAS

568

periodicidad). Lo anterior es incluso cierto cuando el 1er armnico est ausente,


o
a
es decir, cuando contribuye con una amplitud nula (ver gura 15.14).
Las amplitudes de los distintos armnicos mostrados en las guras 15.12 y 15.13
o
son iguales. A pesar de ello, la superposicin da origen a tonos compuestos que
o
en ambos casos tienen una evolucin temporal bastante distinta. La diferencia
o
entre los dos ejemplos radica en que el segundo armnico en la gura 15.13,
o
se ha desplazado en 1/4 de longitud de onda respecto al caso mostrado en la
gura 15.12.
Los tonos compuestos mostrados en las guras anteriores, por tener todos la misma frecuencia, los escuchamos como teniendo todos la misma altura. Sin embargo, no
todos ellos tendrn el mismo timbre. Cul es la propiedad de un tono que permite
a
a
que exista una amplia gama de timbres? La respuesta a este interrogante fue descubierta por Hermann von Helmholtz (1821-1894):
Para sonidos continuos, el timbre de un tono depende slo de la
o
amplitud de los distintos armnicos que lo componen.
o
El o es bastante sensible a la intensidad (o amplitud) de los distintos armnido
o
cos de un tono compuesto, siendo, por otra parte, esencialmente insensible a la fase
(o corrimiento) con que ellos aparecen en su descomposicin. De acuerdo a este
o
notable descubrimiento de von Helmholtz, deducimos que los tonos mostrados en las
guras 15.12 y 15.13 tienen el mismo timbre. Los sonidos generados por la onda mostrada en la gura 15.13 y 15.14, dieren en el timbre, pues, en su descomposicin
o
espectral, los distintos armnicos que lo componen aparecen con intensidades distino
tas.
La elevacin o altura de un tono resulta determinada slo por su
o
o
frecuencia . Lo anterior sigue siendo vlido aun cuando en la
a
descomposicin espectral del tono, el armnico fundamental de freo
o
cuencia est ausente.
e
En la caracterizacin del timbre de un sonido continuo, lo unico relevante para
o

el o humano son las amplitudes de los distintos armnicos de su descomposicin


do
o
o
espectral. Por esta razn, para analizar el timbre, en lugar de mostrar el comportao
miento temporal de un tono compuesto, basta gracar las amplitudes de los distintos
armnicos que lo conguran.
o

CAP
ITULO 15. ONDAS SONORAS

569

La gura 15.15 muestra la intensidad relativa (en decibeles) de los distintos armnio
cos en que se descompone un sonido t
pico
de un clarinete. En este ejemplo se observan
26 armnicos distintos. Para los armnicos de
o
o
orden bajo, dominan los con n impar; lo que
da origen al caracter
stico sonido aterciopelado del clarinete.
Figura 15.15

15.8.

Consonancia y disonancia

El descubrimiento de las relaciones existentes entre la msica, la matemtica y


u
a
la f
sica se remonta al siglo VI antes de Cristo, poca en que la Escuela de Pitgoras
e
a
realiz un exhaustivo estudio de la cuerda vibrante (monocuerda). Los descubrimieno
tos de Pitgoras y sus disc
a
pulos fueron el punto de partida de todos los estudios
posteriores de la armon en la msica y, sin lugar a dudas, inuyeron en forma
a
u
importante en lo que hoy d conocemos como la msica del mundo occidental.
a
u
Pitgoras se dio cuenta de que si una cuerda tensa se acortaba a la mitad, entonces
a
la frecuencia del sonido aumentaba al doble, elevndose su sonido en una octava. Si,
a
por ejemplo, el tono de la cuerda entera corresponde a un Do, las oscilaciones de la
mitad de la cuerda generan el Do siguiente, una octava ms agudo (para distinguirlo
a
del primer Do, le hemos agregado una comilla).
El descubrimiento importante de la escuela de Pitgoras consisti en percatarse
a
o
de que los intervalos musicales ms consonantes con respecto a la cuerda totales decir,
a
los que suenan ms agradables al o al sonar conjuntamentese obtienen cuando,
a
do
al acortarla, el pedazo de cuerda que oscila corresponde a una fraccin (irreductible)
o
n/m de la cuerda completa, en la que tanto el numerador n como el denominador m
son enteros pequeos. Cuanto ms pequeos son estos enteros, tanto ms consonante
n
a
n
a
se percibe el sonido simultneo de los dos sonidos.
a
Al permitir que vibre la mitad de la cuerda, la frecuencia aumenta en un factor
2/1, siendo el intervalo entre los dos sonidos lo que se conoce con el nombre de octava.
Algunas notas de la escala natural mayor
Al permitir que vibren 2/3 de la longitud de una cuerda, la frecuencia aumenta
en un factor 3/2. El intervalo entre los sonidos nuevo y original se denomina quinta
justa o quinta perfecta. El intervalo Do-Sol corresponde a una quinta justa, y por lo
tanto se tiene

CAP
ITULO 15. ONDAS SONORAS

570

Sol
3
= .
Do
2
Esta ultima ecuacin permite, conociendo la frecuencia del Do, determinar la

o
frecuencia del Sol.
Continuando de la misma manera, el cuociente que sigue en complejidad es 3/4.
Al acortar una cuerda a 3/4 de su longitud original, la frecuencia del sonido emitido
aumenta en un factor 4/3. En este caso, el intervalo musical entre el tono original
y el nuevo se denomina cuarta justa. El intervalo Do-Fa, como tambin el Sol-Do,
e
corresponden a cuartas justas. En este caso se tiene
4
Fa
= .
Do
3
Si las frecuencias de dos sonidos estn en la razn 1:1, entonces tienen la misma
a
o
frecuencia, y se dice que estn al unsono.
a

El un
sono, la octava, la quinta y la cuarta justa, son los intervalos musicales
ms consonantes y tienen su origen en las fracciones ms simples: 1/1, 1/2, 2/3 y
a
a
3/4, respectivamente. Estos intervalos desempean un papel especial en la armon
n
a
musical.
En la mayor parte de la msica del mundo occidental, el desarrollo armnico y
u
o
meldico de las piezas musicales transcurre en una tonalidad y en torno a una nota
o
central o eje, la as llamada tnica. La ultima nota de la melod de una pieza musical

a
coincide, generalmente, con la tnica.
o
Las notas musicales a las que se llega partiendo de la tnica por medio de un
o
intervalo de quinta y cuarta justa tienen nombres especiales: se llaman dominante
y subdominante de la tonalidad, respectivamente. Si la tnica es el Do, entonces la
o
dominante corresponde a la nota Sol y la subdominante al Fa.
Evaluemos la razn entre las frecuencias de las notas Sol y Fa. Usando lgebra
o
a
elemental se encuentra que
Sol
Sol Do
3 3
9
Sol
=
=
= =
Fa
Fa
Do F a
2 4
8
(Observe que hemos usado la razn de dos notas para referirnos en realidad al cuoo
ciente de sus frecuencias.) Los enteros que intervienen en la ultima fraccin ya no son

o
tan pequeos y tal intervalo (que se llama segunda mayor ) suena disonante.
n
Volvamos a la monocuerda y acortmosla ahora a 4/5 de su longitud original. La
e
frecuencia de la cuerda aumentar en un factor 5/4. Esta fraccin an est constitu
a
o u
a
da
por enteros relativamente pequeos y los dos sonidos, separados por un intervalo
n
que se suele llamar tercera mayor, efectivamente se perciben como consonantes. El
intervalo Do-Mi corresponde a una tercera mayor y, por supuesto, se tiene que
5
Do
= .
Mi
4

CAP
ITULO 15. ONDAS SONORAS

571

El acorde perfecto mayor


Hasta aqu hemos establecido las frecuencias (relativas) de las notas Do, M i, F a, Sol

y Do . Cmo podemos obtener las dems notas de la escala musical?


o
a
Para ello introduzcamos la nocin de acorde. El acorde ms importante de la
o
a
msica occidental es el acorde perfecto mayor; por ejemplo, el dado por las notas
u
Do M i Sol. El placer producido al escucharlo se debe a que la razn entre las
o
frecuencias, 4:5:6, slo involucra nmeros enteros pequeos1 .
o
u
n
Los acordes perfectos mayores, basados en la tnica, subdominante y dominante,
o
permiten encontrar las frecuencias de las dems notas de la escala musical mayor (ver
a
problemas de las seccin 15.11). La gura 15.16 muestra las razones para las notas
o
sucesivas de la escala Do mayor en la anacin natural o justa.
o

Figura 15.16

15.9.

Oscilaciones de una columna de aire

Las oscilaciones de una columna de aire cil


ndrica completamente encerrada son
en muchos aspectos anlogas a las oscilaciones de una cuerda con ambos extremos
a
1

Se podr argumentar que un acorde formado por tres notas cuyas frecuencias estn en la razn
a
a
o
3:4:5, debe producir un sonido an ms placentero que el acorde perfecto mayor. Esto es efectivamente
u
a
as Pero observemos que ese acorde es esencialmente el mismo que el acorde perfecto mayor: al
.
trasladar la nota ms grave del acorde 3:4:5 a la octava superior (el 3 se transforma en 6), se obtiene
a
un acorde cuyas frecuencias estn en la razn 4:5:6. El acorde Sol Do M i est compuesto por
a
o
a
notas cuyas frecuencias estn en la razn 3:4:5 tal acorde es simplemente una variacin (en el
a
o
o
lenguaje tcnico, la segunda inversin) del acorde perfecto mayor Do, M i, Sol. Cuando las notas
e
o
estn en la razn 1:2:3 no se tiene realmente un acorde, ya que la segunda nota es esencialmente la
a
o
misma nota que la primera, slo que trasladada en una octava. Lo mismo sucede con notas en la
o
relacin 2:3:4; en este caso, la tercera nota esencialmente coincide con la primera.
o

CAP
ITULO 15. ONDAS SONORAS

572

jos. Recordemos que ambos movimientos satisfacen la misma ecuacin diferencial


o
la ecuacin de ondas. La diferencia ms importante entre estos dos sistemas es que,
o
a
en un caso, la cuerda realiza oscilaciones transversales, mientras que, en el otro caso
la masa de aire realiza oscilaciones longitudinales.
En los dems aspectos, las oscilaciones de una columna de aire cerrada en ambos
a
extremos, que denotaremos por cc, y las oscilaciones de una cuerda con ambos
extremos jos, tienen muchas semejanzas. En efecto:
1. Tanto en la cuerda como en la columna de aire, el movimiento es nulo en los
extremos.
2. Ambos sistemas poseen varios modos de oscilacin, llamados modos normales
o
o armnicos. Estos modos tienen frecuencias mltiplos de la del modo de oscio
u
lacin fundamental (o primer armnico). La gura 15.17 da una idea de cmo
o
o
o
oscila el aire en el interior de la columna en el modo fundamental y en el segundo
modo.
3. La longitud de onda de la oscilacin correspondiente al n-simo modo, en ambos
o
e
casos viene dada por
n =

2L
.
n

Figura 15.17
4. La relacin que existe entre la frecuencia y la longitud de onda , en ambos
o
casos, es
=

c
.

Para la columnda de aire, c es la velocidad del sonido en ese medio.


A partir de las dos ultimas ecuaciones, deducimos que

(15.11)

CAP
ITULO 15. ONDAS SONORAS

573

n = n

c
,
2L

(15.12)

es decir, la frecuencia es inversamente proporcional al largo del tubo.


En la gura ?? se muestra la presin del aire en el interior de la columna cuando
o
sta oscila en el modo fundamental y en el segundo armnico. La presin es alta
e
o
o
(por encima de la presin ambiental p0 ) si el aire est comprimido, y baja, cuando
o
a
est enrarecido.
a
La velocidad del aire para una columna de aire cc siempre es nula en los extremos. En la gura ??, velocidades positivas indican que el aire se mueve hacia la
derecha; velocidades negativas, que se mueve hacia la izquierda. Cuando la presin
o
alcanza uno de sus valores extremos (es decir, es m
nima o mxima), el aire se encona
trar momentneamente en reposo. Algo similar ocurre con las oscilaciones de una
a
a
cuerda (o las oscilaciones de un columpio): cuando el desplazamiento desde su posicin
o
de equilibrio es mximo, el sistema se encontrar momentneamente en reposo.
a
a
a
Columna de aire abierta por un extremo
Analicemos las oscilaciones que pueden generarse en una columna de aire cil
ndrica, cerrada en un extremo y abierta en el otro, que denotaremos por ca.
La clave para poder analizar este problema consiste en darse cuenta de que el
aire ubicado en la vecindad del extremo abierto del cilindro, por estar en contacto con
el exterior, estar (en primera aproximacin) sometido en todo instante a la presin
a
o
o
atmosfrica.
e
Ahora notemos que en la columna de aire cc, cuando la columna de aire oscila
en el modo fundamental, la presin en el centro nunca var coincidiendo siempre
o
a,
con la presin atmosfrica p0 . Esto signica que, si la columna cc oscila en su modo
o
e
fundamental, la podemos cortar por la mitad, y alejar una de las mitades sin que se
modique el modo de oscilacin de la parte restante. En particular, para el pedazo de
o
= L/2 que queda, no hay cambio de la longitud de onda
cilindro ca de longitud L
y, por consiguiente, tampoco en la frecuencia. Ahora bien, en la columna de aire cc
la longitud de onda del modo fundamental es el doble del largo L. Al acortarla por
la mitad, la longitud de onda del modo fundamental resulta ser cuatro veces el largo
L del cilindro que queda, es decir, = 4L .
Reiteramos que la columna de aire cc cercenada al centro es precisamente un
cilindro ca. De esta manera, hemos encontrado la frecuencia de las oscilaciones
correspondientes al modo fundamental (n = 1) de tal sistema:
c
.
4L
La gura ?? muestra la velocidad del aire y la presin en el interior del tubo
o
(cerrado en el extremo O y abierto en L ). Notemos que estas guras coinciden exac1 =

CAP
ITULO 15. ONDAS SONORAS

574

tamente con la mitad izquierda de las correspondientes al modo fundamental de la


columna cc, mostradas en las guras ?? y ??.
Para obtener el segundo armnico del sistema ca podemos usar un truco
o
parecido. Notemos que la columna de aire cc, al oscilar en el segundo armnico, en
o
los lugares L/4 y 3L/4, la presin nunca var coincidiendo siempre con la atmosfrio
a,
e
ca. Luego, podemos cortar el tubo en el lugar 3L/4 y descartar la parte ms corta,
a
sin modicar las vibraciones del aire en el interior de la parte ms larga. De esta
a
manera, encontramos la frecuencia de oscilacin para el segundo armnico de una
o
o
columna cil
ndrica de aire ca. Designemos su longitud con la letra L . Se deduce
que 3/4 = L . La frecuencia de este segundo armnico es, por lo tanto,
o
3c
c
=
= 31 .

4L
Continuando del mismo modo, se encuentran las frecuencias de todos los dems
a
modos de oscilacin de una columna de aire ca. Las frecuencias posibles resultan
o
ser los mltiplos impares de la frecuencia del modo fundamental 1 = v)(4L), es decir:
u
2 =

n = (2n 1)

c
= (2n 1)1 ,
4L

(15.13)

con n = 1, 2, 3 . . .
La gura ?? muestra la presin y la velocidad del aire dentro del cilindro cuando la
o
columna oscila en el segundo (n = 2) y tercer (n = 3) modo normal, respectivamente.
Columna de aire abierta por ambos extremos
Analicemos ahora las oscilaciones que pueden generarse en una columna de aire
cil
ndrica aa (abierta por ambos extremos).
En este caso, en la vecindad de ambos extremos (abiertos) del cilindro, la presin
o
del aire ser en todo instante igual a la presin atmosfrica p0 . Nuevamente podemos
a
o
e
descubrir cules sern los modos de oscilacin en este caso, cortando el cilindro cc
a
a
o
en lugares apropiados (donde la presin es p0 ).
o
En la gura ?? se observa que el modo fundamental del cilindro cc no posee
dos nodos de presin, es decir, lugares en que la presin del aire no var pero que
o
o
e,
el segundo armnico si los tiene. Cortemos el cilindro cc (al oscilar en el segundo
o
armnico) en los lugares x = L/4 y x = 3L/4. De este modo, la seccin central
o
o
dar el modo fundamental de un tubo cil
a
ndrico aa de largo L = L/2. Notemos que
= 2L, o sea, la longitud de onda es dos veces el largo L. La frecuencia del modo
fundamental para un tubo abierto en ambos lados ser dada por
a
v
,
2L
resultado que coincide con el obtenido del tubo cil
ndrico cc. La gura ?? muestra
la presin y la velocidad del aire para el modo de oscilacin fundamental en el interior
o
o
de una columna cil
ndrica aa.
1 =

CAP
ITULO 15. ONDAS SONORAS
Modo de Oscilacin
o
Fundamental
2o modo
3er modo
4o modo
5o modo
Frecuencia del
modo fundamental
Frecuencia del
modo n
Cuadro 15.4Frecuencias de

n
1
2
3
4
5
1
n

575

Cilindro cc
1
21
31
41
51
c
1 =
2L

Cilindro ca
1
31
51
71
91
c
1 =
4L

Cilindro aa
1
21
31
41
51
c
1 =
2L

n = n1

n = (2n 1)1

n = n1

los modos normales de oscilacin de una columna de aire.


o

Para obtener el segundo armnico, es casi evidente que la columna de aire tendr un
o
a
nodo de presin adicional al centro. La gura ?? muestra cmo se comportan la preo
o
sin y la velocidad del aire en este caso. La longitud de onda del segundo armnico
o
o
coincide con el largo L del tubo, siendo la frecuencia 2 = 21 .
Continuando de esta manera se encuentra que, para el caso de un cilindro abierto
en ambos extremos, las frecuencias de los distintos modos de oscilacin son expresadas
o
por
c
= n1 ,
(15.14)
2L
con n = 1, 2, 3 . . . La tabla adjunta recapitula los resultados de esta seccin. Compreo
a
la con la tabla anloga para las oscilaciones de una cuerda tensa mostrada en el
a
cap
tulo anterior.
n = n

15.10.

Efecto Doppler

Considere un observador que escucha el sonido de una fuente sonora. Supongamos


que la fuente sonora genera un sonido continuo de frecuencia 0 . Si la separacin
o
entre la fuente y el observador var en funcin del tiempo, entonces la frecuencia que
a
o
escuchar el observador no ser 0 . Este fenmeno se conoce por efecto Doppler.
a
a
o
Supongamos que una fuente de sonido ja emite un sonido de frecuencia 0 y que
el observador se observa con velocidad v. (Suponemos adems que no corre viento, es
a
decir, el aire est en reposo para la fuente). La separacin de dos mximos de presin
a
o
a
o
sucesivos de la onda sonora es 0 = c/0 , donde c es la velocidad del sonido. Pero
como el observador va al encuentro de ellos, el tiempo que tarda en recibir a dos
mximos sucesivos ser menor, y por lo tanto la frecuencia mayor (ver gura ??). Un
a
a
simple anlisis muestra que la frecuencia que escucha el observador ser
a
a

CAP
ITULO 15. ONDAS SONORAS

576

c+v
.
(fuente ja, observador se acerca).
c
Si en lugar de acercarse, el observador se aleja de la fuente, se debe cambiar el signo
de v. En tal caso la frecuencia resulta ser
= 0

cv
.
(fuente ja, observador se aleja).
c
Consideremos el otro caso. Supongamos que el observador se encuentra jo y que
la fuente de sonido, que emite un sonido de frecuencia 0 , se mueve hacia el observador
con una velocidad v (ver gura ??). En este caso, debido a que la fuente se acerca,
la longitud de onda (es decir, la distancia entre dos mximos sucesivos de la presin)
a
o
no ser c/0 , sino que algo menor. Un anlisis simple muestra que la frecuencia que
a
a
escucha el observador en este caso ser
a
= 0

c
.
(observador jo, fuente se acerca).
cv
Si en lugar de acercarse, la fuente se aleja, entonces se debe cambiar el signo de v.
En tal caso, la frecuencia resulta ser
= 0

= 0

c
.
c+v

(observador jo, fuente se aleja).

Cono de Mach
Al propagarse un objeto con velocidad supersnica, la envolvente de las ondas sonoras
o
que emite el objeto formarn un cono (llamado cono de Mach). No hay sonido en el
a
exterior del cono. La envolvente se mueve con la velocidad del sonido. Cuando la
envolvente llega a un observador, este escuchar la t
a
pica explosin supersnica. El
o
o
ngulo del cono de Mach viene dado por
a
sin =

15.11.

c
.
v

Problemas

1. El o humano puede escuchar sonidos cuyas frecuencias var entre 16 y


do
an
20 000 Hz. Encuentre la longitud de onda de estas ondas sonoras.
2. Las variaciones de presin para el umbral del sonido son de slo 0,00002 Pa. Para
o
o
comprender lo que signica en la prctica esta peque
a
nsima magnitud, recuerde
cmo se tapan los o
o
dos al descender rpidamente en un ascensor o bajar en
a
automvil una cuesta; basta descender verticalmente unos veinte metros para
o
sentirlo. Demuestre que el cambio de presin de 2 1010 atm ya se produce al
o
descender slo 1,6 milsimo de un mil
o
e
metro!

CAP
ITULO 15. ONDAS SONORAS
3.

577

a) Demuestre que la suma de dos sonidos de 0 dB tiene un nivel de intensidad


de 3dB.
b) Tres sonidos por separado, en cierto lugar, tienen un nivel de intensidad de
40, 70 y 80 decibeles, respectivamente. Encuentre el nivel de intensidad que
se senti en ese lugar si las tres fuentes de sonido sonaran simultneamente.
a
a

4. Suponga que en una orquesta, 30 violinistas tocan su viol con la misma inn
tensidad. El nivel de intensidad del sonido que se escucha en cierto lugar de la
sala es de 66 dB. Cul ser la intensidad en ese lugar si slo tocara un viol
a
a
o
n?
5. Considere una onda sonora (plana) sinusoidal:

P (x, t) = P0 + P cos(kx t).

(Denotaremos por P a la amplitud de la oscilacin: en este problema, a ser un


o

a
escalar y no un vector unitario.)
a) Demuestre que la amplitud u de las oscilaciones de las molculas del medio,

e
al paso de la onda sonora viene dada por
u=

P
,
0 c

donde c es la velocidad del sonido y 0 su densidad de masa.


b) Demuestre que la potencia acstica I transmitida por unidad de rea, viene
u
a
dada por
1
I = c0 2 u2 .

2
(Note la similitud de esta expresin con la ecuacin para la energ transo
o
a
portada por una onda sinusoidal en una cuerda, deducida en el cap
tulo
anterior.)
c) Demuestre que otra expresin alternativa para la potencia acstica por
o
u
unidad de rea es
a
I=
6.

P2
.
20 c

a) Las notas musicales Fa - La - Do corresponden a un acorde perfecto


mayor. Usando este hecho, demuestre que la razn entre las frecuencias de
o
las notas La y Do es 5:3.
b) Tambin el acorde Sol - Si - Re es un acorde perfecto mayor. Determine
e
la razn de frecuencias entre el Re y el Do.
o

CAP
ITULO 15. ONDAS SONORAS

578

c) Determine la razn de frecuencias entre el Mi y el Re. (En la escala mayor,


o
con anacin natural, no todos los intervales segunda mayor son iguales!)
o
7. Determine el valor de la velocidad del sonido en gas de nitrgeno si ste se
o
e
C.
encuentra a una temperatura de 30
8. Considere las oscilaciones de la columna de aire al interior de un tubo. Demuestre que la frecuencia de un modo normal particular de oscilacin var con un
o
a
cambio de temperatura de acuerdo a la relacin
o

T
=

2T
9.

a) Encuentre una expresin para el desplazamiento u(x, t) de las molculas


o
e
de aire al interior de un tubo ca, cuando este est oscilando en su tercer
a
modo normal.
b) Derivando con respecto al tiempo, encuentre una expresin para la velocio
dad de las molculas al interior del tubo.
e
c) Usando la relacin
o
(x, t) = 0 1

u
x

encuentre una expresin para la presin p(x, t).


o
o
d) Convnzase de que los resultados anteriores estn en concordancia con lo
e
a
mostrado en la gura ??.
10. Un automvil, cuyo chofer toca la bocina, se acerca con velocidad v a una pared.
o
El eco que escucha el chofer es una tercera mayor ms agudo que el sonido de
a
la bocina. Encuentre la velocidad v.
11. Una persona A genera sonido induciendo oscilaciones en el aire encerrado en el
interior de un tubo cil
ndrico ca de 20 cm de largo.
a) Si las oscilaciones inducidas corresponden al segundo modo normal de oscilacin, determine la frecuencia del sonido emitido.
o
b) Supongamos que la persona A se encuentra sobre un carrito abierto que
se mueve con velocidad v hacia una gran pared. Cul es la frecuencia del
a
eco que escucha A?. (Suponga que no corre viento).
c) Una persona B se encuentra entre el carrito y la pared. Con qu frecuencia
e
escuchar B el sonido directamente emitido por A y el eco que rebota en
a
la pared?

CAP
ITULO 15. ONDAS SONORAS

579

12. Un plomero debe destapar un tubo que se encuentra obstru


do. Al soplar por
encima de la apertura del tubo, escucha un sonido de 200 Hz. A qu distancia
e
de la apertura se encuentra la obstruccin?
o
13. Demuestre que una onda sinusoidal de 1000 Hz, con vibraciones de presin
o
Pe = Pu = 0.00002 Pa, corresponde aproximadamente a una intensidad de
I 1012 W/m2 .
14. Para un sonido de 1000 Hz, la sensibilidad del o humano es de aproximadado
mente 1012 W/m2 . Encuentre la amplitud de las oscilaciones de las molculas
e
de aire para un sonido de esa intensidad. Observe que las amplitudes de las
oscilaciones son menores que el tamao de las molculas de aire.
n
e
15. Cul ser la intensidad del sonido (en decibeles) de una bocina de camin cuya
a
a
o
potencia acstica es de 5 Watt, si se la escucha a 5 m de distancia? Desprecie
u
los efectos (refraccin, difraccin, etc.) que modican a la onda sonora durante
o
o
su propagacin.
o
16. Un d de verano, en que corre un viento sur de 30 km/hora, un automovilista
a
viaja a 100 km/h hacia el norte. Al hacer sonar la bocina, el chofer al interior
del automvil la escucha emitiendo un sonido de 440 Hz. Con qu frecuencia
o
e
percibe el sonido una persona, parada junto a la carretera, antes y despus de
e
que haya pasado el automvil?
o
17. Una persona escucha el sonido de una campana de una iglesia que est a 2 km
a
de distancia. La campana emite un sonido de 500 Hz. Adems corre un viento
a
de 50 km/hora, transversal a la l
nea que une a la persona con la iglesia.
a) Cunto tarda el sonido para propagarse desde la iglesia hasta la persona?
a
b) Con qu frecuencia escuchar la persona el sonido?
e
a
18. Un automvil se mueve con velocidad v hacia un observador A jo. Se hace
o
sonar la bocina del automvil, cuya frecuencia es 0 . Encuentre la frecuencia
o
del sonido detectado por A, si corre un viento con velocidad v en la misma

direccin en que avanza el automvil.


o
o
19. Una persona observa un avin supersnico que vuela horizontalmente con veloo
o
cidad constante. 10 segundos despus de pasar por encima de su cabeza, escucha
e
la explosin supersnica del cono de Mach. Si en ese instante el avin forma
o
o
o
un ngulo de 30 con la normal, cul es la velocidad del avin y a qu altura
a
a
o
e
est volando?
a
20. Dos fuentes de sonido puntuales, separadas por una distancia D = 4 m, se perciben separadamente, en el punto medio entre las dos, como teniendo intensidades
sonoras L de 70 y 50 decibeles, respectivamente.

CAP
ITULO 15. ONDAS SONORAS

580

a) Qu intensidad sonora L (en decibeles) tendr el sonido si se escuchan


e
a
ambas fuentes en forma simultnea?
a
b) En qu lugar entre ambas fuentes uno debe ubicarse para escuchar las
e
dos fuentes con la misma intensidad? (Para resolver este problema, slo
o
considere los aspectos geomtricos del problema, despreciando efectos como
e
la reexin, refraccin, absorcin del sonido por objetos en el entorno, y
o
o
o
tambin la atenuacin del sonido por mecanismos disipativos.)
e
o
21. Un observador A deja caer una fuente sonora desde la azotea de un edicio. Sea
0 la frecuencia del sonido que emite la fuente.
Sea t = 0 el instante en que la fuente sonora comienza a caer. Determine la
frecuencia (t) con que A percibe el sonido. (Observe que hay un tiempo que
transcurre entre el instante en que se emite el sonido y el instante en que A lo
escucha.)

15.12.

Solucin a algunos de los problemas


o

Solucin al problema 2
o
La frmula baromtrica que nos da la presin de la atmsfera en funcin de la altura
o
e
o
o
o
es
P (h) = P0 exp

g0
h ,
P0

donde P0 es la presin atmosfrica y 0 = 1.29 kg/m3 es la densidad del aire a nivel


o
e
del mar. Derivando esta expresin se obtiene
o
dP (h)
g0
= g0 exp
h .
dh
P0
Evaluando en h = 0 (esto es en la supercie terrestre), se encuentra que
dP = g0 dh.
Usando esta relacin se deduce nalmente que la altura dh para la cual la presin
o
o
5 Pa es
cambia en dP = 2 10
dh =

2 105
dP
=
[m] 1.6 106 [m] = 0, 0016[mm].
g0
9, 81 1, 29

Solucin al problema 5
o

CAP
ITULO 15. ONDAS SONORAS

581

a) Derivando P (x, t) respecto a x se obtiene


P (x, t)

= kP sin(kx t).
x
Pero (de acuerdo a la ecuacin (15.5))
o
P0
P

,
x
0 x
luego

0
=
kP sin(kx t) .
x
P0

(15.15)

De esta relacin deducimos que


o
(x, t) = 0 +

0
P cos(kx t) ,
P0

o sea, la amplitud de las variaciones de densidad es


= 0

P0
.

P0

(15.16)

Tambin sabemos que (ver deduccin de la ecuacin de ondas presentada en la


e
o
o
seccin 15.3)
o
(x, t) = 0 1

u
x

Combinando las ecuaciones (15.15) y (15.16) se obtiene que

P
u(x, t)
=
cos(kx t) .
x
P0
Integrando se encuentra la funcin u(x, t) que describe el desplazamiento de las
o
molculas
e
u(x, t) =

P
sin(kx t) .
kP0

La amplitud de estas oscilaciones de las molculas de aire es, por lo tanto,


e

CAP
ITULO 15. ONDAS SONORAS

582

u=

P
.
kP0

u=

P
.
0 c

Pero k = /c y c2 0 = P0 , luego

b) Consideremos ahora un cilindro (imaginario) de seccin transversal A y longitud


o
L = cT = , con el eje del cilindro orientado a lo largo de la direccin de
o
propagacin del sonido. La masa del aire encerrado en ese cilindro es m =
o
A0 . Cada molcula realiza un movimiento armnico de frecuencia angular
e
o
y amplitud u. Por lo tanto, la energ total (debido al sonido) de las molculas

a
e
encerradas por el cilindro es
E=

1
m 2 u2 .

Esta energ en un tiempo T = 2/ avanza en una distancia (que es justaa,


mente el largo del cilindro), luego la potencia acstica propagada a lo largo del
u
cilindro es
1
1
1
1
1
E
= m 2 u2 = A0 2 u2 = Ac0 2 u2 .

T
2
T
2
T
2
De esta manera se encuentra que la potencia acstica por unidad de rea viene
u
a
dada por
1
I = c0 2 u2 .

2
c) Usando los resultados de la parte (a) y (b) encontramos
I=

P2
.
2c

Por ultimo, recordemos que una onda sinusoidal para Pe = P / 2, luego

tambin se puede escribir


e
I=

Pe
,
c

que es precisamente la expresin que se us en la seccin 15.5.


o
o
o

CAP
ITULO 15. ONDAS SONORAS

583

Solucin al problema 10
o
Sea 0 la frecuencia con la que la bocina emite el sonido. Al acercarse el automvil
o
con velocidad v a la pared, un observador junto a ella escuchar el sonido de la bocina
a
con una frecuencia
c
.
cv
Con esa misma frecuencia la pared hace rebotar el sonido. La persona en el
automvil que ahora se acerca a una fuente de sonido (1 ) con velocidad v lo escuo
char con frecuencia
a
1 = 0

c+v
c+v
= 0
.
c
cv
De acuerdo al enunciado, la razn entre las frecuencias de los sonidos 2 y 0 es
o
5/4 (corresponde a un intervalo de tercera mayor), luego,
2 = 1

c+v
5
=
.
4
cv

Despejando, se encuentra que la velocidad del automvil es v = c/9 136 km/h.


o
Solucin al problema 20
o
a) El nivel de intensidad sonora L y la intensidad (potencia) por unidad de rea I
a
estn relacionadas por
a
L = 20 log 10

Pe
Pu

[db] = 10 log 10

I
[dB] .
I0

Recordemos que cuando hay dos (o ms) sonidos, la magnitud aditiva es la


a
potencia I y no el nivel de intensidad L. Para un sonido
L1 = 50 [dB] = 10 log10

I1
[dB]
I0

se encuentra que I1 = I0 105 . Anlogamente para el sonido de 70 dB se encuentra


a
que I2 = I0 107 . Al escuchar ambas fuentes conjuntamente, intensidad ser I =
a
7 , lo que corresponder a un nivel de intensidad
I1 + I2 = 1.01 I0 10
a
L = 10 log 10

I
[dB] = 10 (7 + log10 1.01) [dB] 70.04 [dB]
I0

CAP
ITULO 15. ONDAS SONORAS

584

b) Para una fuente sonora puntual se tiene que I(r) = P/(4r 2 ), donde P es la
potencia acstica de la fuente. Conociendo las intensidades I de las fuentes en
u
el punto medio r = D/2, podemos deducir la potencia de ellas:
P1 = 4I1

D2
= I0 105 D 2
4

y
P2 = 4I2

D2
= I0 107 D 2 .
4

Sea x la distancia de la fuente #1 a la que debemos situarnos para escuchar


ambas con la misma intensidad sonora (en ese caso la distancia a la fuente
sonora #2 es (D x)). Para que las dos fuentes sonoras se escuchen con el
mismo nivel sonoro, debe cumplirse
P2
P1
=
.
2
x
(D x)2
Despejando x se encuentra

1
x
=
,
D
1
donde P2 /P1 = 100, o sea,
x
1 10
=
.
D
99
Tenemos dos soluciones: x1 = +D/11 corresponde a una solucin en que el
o
observador se encuentra entre las dos fuentes (m
as cerca de la fuente #1) y x2 = D/9, solucin es que el observador se eno
cuentra al lado izquierdo de la fuente #1 (ver gura ??.
Solucin al problema 21
o
Sea t = 0 el instante en que se suelta la fuente sonora desde la azotea. Despus de transcurrido un tiempo t, la fuente habr ca una distancia x(t) = gt2 /2
e
a do
y tendr una rapidez v(t) = gt. El sonido que la fuente emite en ese instante, el
a
observador en la azotea eventualmente lo escuchar teniendo una frecuencia
a
= 0

c
.
c + gt

CAP
ITULO 15. ONDAS SONORAS

585

Es importante, sin embargo, darse cuenta que tal sonido el observador no lo escuchar en el instante t, sino que en un instante levemente posterior (ya que el sonido
a
aun debe recorrer la distancia x). En efecto, el sonido emitido en el instante t, el observador recin lo escuchar en el instante t = t + t , donde t = x(t)/c es el tiempo
e
a
que el sonido tarda en recorrer la distancia x(t). La relacin entre t y t es
o
t = t +

x(t)
gt2
=t+
.
c
2c

Despejando t se encuentra
2gt
gt
= 1 + 1 +
.
c
c
Sustituyendo esta expresin en la expresin para se obtiene nalmente
o
o
(t ) = 0

c
=
+gt

0
1+

2gt
c

Cap
tulo 16

Relatividad especial
versin 27 agosto 2012
o

16.1.

Problemas (Galileo-Newton)

Para todos los problemas siguientes, se debe realizar el diagrama de espaciotiempo que describa la situacin descrita, y las respuestas deben ser encontradas
o
geomtricamente.
e
1. Un observador O se halla en la berma de un camino. A una distancia L hay
un retn policial. En t = 0, un automvil pasa con velocidad v0 , mayor que
e
o
la permitida, junto a O. El observador O le avisa al conductor del automvil,
o
gritndole, que hay polic ms adelante. Pero O se demora en gritar, y el aviso
a
as a
le llega al automovilista justo cuando pasa frente a la autoridad.
Encontrar el tiempo t0 que tard O en enviar el aviso desde que el automvil
o
o
pas a su lado.
o
2. Dos polillas, P1 y P2 , se encuentran juntas, posadas en una rama. En t = 0 la
polilla P1 vuela hacia la derecha con velocidad v0 , sin notar que a una distancia
D acecha un murcilago M , en reposo respecto a la polilla P2 . En el mismo
e
instante t = 0 en que la polilla comienza a volar, el murcilago M emite una
e
onda sonora que viaja hacia P1 , rebota en ella mientras est en vuelo, y vuelve
a
a M.
Cunto tiempo tarda la onda sonora en llegar a la polilla P1 ?
a
A qu distancia del murcilago se encuentra la polilla P1 cuando la onda sonora
e
e
vuelve al murcilago?
e

586

CAP
ITULO 16. RELATIVIDAD ESPECIAL

P1

587

v0

P2
D

3. Consideremos un tren que se acerca con velocidad v0 hacia la casa de una


paloma que se encuentra a una distancia D. En t = 0, con el n de salvar su
vida, la paloma emprende vuelo hacia el tren, con velocidad constante vp , con
la intencin de posarse sobre la ventanilla del conductor y lograr que detenga
o
el tren, evitando que rompa su casita.
A qu distancia estar el tren de la casita cuando la paloma llega a l? Cundo
e
a
e
a
llegar la paloma al tren?
a
4. Un automovilista pasa al frente de un retn policial con velocidad v. Un tiempo
e
t0 despus, un polic sale en su persecusin, con velocidad constante vp > v.
e
a
o
A qu distancia del retn se encuentran el polic y el infractor?
e
e
a
5. Dos amigos se encuentran en t = 0 en x = 0, uno en reposo (O) y el otro
movindose con velocidad v0 hacia la derecha (O ). En t = t0 , O le grita a
e
O . Cuando lo escucha, O le responde con otro grito. Dnde se encuentra O
o
cuando O escucha el grito de respuesta? Cunto tiempo transcurri entre que
a
o
O lanz su grito y escuch la respuesta?
o
o
6. Susana y Fabiola son parte del equipo femenino de posta de su escuela. Su
amiga Paulina se encuentra en x = 0, al lado de la pista atltica, alentndolas.
e
a
En t = 0, Susana se encuentra tambin en x = 0, con el testimonio, movindose
e
e
con velocidad vS respecto a Paulina. En ese mismo instante, Fabiola se encuentra
a una distancia d de ambas, movindose con velocidad vF < vS . Eventualmente,
e
Susana alcanza a Fabiola para entregarle el testimonio, y dicho encuentro sucede
a una distancia xA de Paulina. Cul es la velocidad de Fabiola?
a
7. Un canario se encuentra a una distancia D a la izquierda de un observador O,
ambos en reposo. En t = 0 un automvil pasa junto a O, con velocidad v0 hacia
o
la derecha. En su interior hay una jaula con una canario hembra. El canario, en
t = 0, intenta llamar la atencin de la hembra emitiendo un silbido. Cuando la
o
hembra canario escucha el canto, inmediatamente responde.
En qu instante escucha el observador O el canto de la hembra canario?
e

CAP
ITULO 16. RELATIVIDAD ESPECIAL

588

v0

16.2.

Problemas (Einstein)

Los primeros 24 problemas de esta seccin se encuentran en un documento aparte.


o
25. Resuelva nuevamente el problema 1 de la Sec. 16.1, en su versin relativista,
o
suponiendo que ahora el aviso no se realiza gritando, sino enviando una seal
n
luminosa.
26. Un observador O se mueve con velocidad v0 hacia la derecha respecto a un
observador O en reposo. En t = 0, O pasa junto a O. O nos relata que, en su
sistema de referencia, un tiempo T despus del encuentro con O dos destellos
e
de luz son emitidos simultneamente, uno por O, y el otro por O . El destello
a
emitido por O es recibido por O en un tiempo TO (medido por O ); el destello
emitido por O es recibido por O en un tiempo TO (medido por O). TO y TO
estn medidos respecto a t = 0.
a
a) Construya el grco de Loedel que representa la situacin antes descrita.
a
o
e
b) Encuentre los valores de TO y TO . Exprese su resultado en trminos de los
a
ngulos 2 (entre los ejes temporales de ambos sistemas de referencia) y
(entre el eje espacial x y la l
nea de vida de un destello que viaja hacia
la derecha).
27. Ana y Valeria eran no slo muy buenas tortugas, sino adems grandes amigas.
o
a
Por eso, aprovechando las liquidaciones pre-navideas, decidieron reunirse para
n
ir de compras. Mauricio, el caracol del jard al enterarse de esto se acerc rpin,
o a
damente a Valeria. Puedo ir contigo?, le pregunt. Mauricio era vecino de
o
Valeria, y aprovechaba cada oportunidad de ver a Ana, por quien sent algo
a
ms que una tierna amistad. Valeria no cre que un invertebrado como Mauria
a
cio fuera un buen partido para Ana (despus de todo, aparte de un caparazn,
e
o
no ten mucho en comn), pero Ana ya era adulta, y a sus 103 aos (que
an
u
n
coquetamente insist en declarar como 93) pod tomar sus propias decisiones.
a
a
De modo que acept que Mauricio la acompaara.
o
n
Ana y Valeria se levantaron temprano ese d para alcanzar a hacer todo lo que
a
deb hacer. Quedaron de reunirse junto a las ligustrinas, y se aproximaron a
an

CAP
ITULO 16. RELATIVIDAD ESPECIAL

589

una velocidad relativa entre ellas vt , cercana a la velocidad de la luz. Mauricio


iba junto a Valeria, muy concentrado, cuidando de no quedarse atrs ni un
a
mil
metro, viajando a la misma velocidad que Valeria. Pero Mauricio estaba
demasiado emocionado, y cuando las amigas tortugas se encontraban a una
distancia L (en el sistema de referencia de Mauricio), apur el paso. Valeria vio
o
cmo Mauricio se alejaba con velocidad vM respecto a ella. Iba tan rpido, de
o
a
hecho, que Valeria observ que el tiempo que le tom a Mauricio encontrarse con
o
o
Ana fue un tercio del tiempo que le tom a ella misma encontrarse con su amiga
o
(ambos tiempos medidos en su propio sistema de referencia, por supuesto).
En el sistema de referencia de Valeria, cunto tiempo le tom a Mauricio ena
o
contrarse con Ana? Y en el sistema de referencia de Mauricio? En el sistema de
referencia de Valeria, qu distancia la separa de Ana cuando sta se encuentra
e
e
con Mauricio? Escriba sus respuestas en trminos de L, vt y vM .
e
Resuelva el problema usando grcos de Loedel.
a

Valeria

Mauricio

Ana

28. Un observador O se aleja hacia la derecha con velocidad v0 de un observador O


en reposo. En t = t = 0 ambos observadores coinciden. El observador O viaja
sobre un carrito, y a una distancia L de l (medida en el sistema de referencia
e
0
) se encuentra una linterna que apunta hacia la izquierda. (Ver gura.)
O
O

v0

L0

En su sistema de referencia, O observa que en el instante t = 0 la linterna

emite un destello, y un tiempo 0 despus emite un segundo destello.


e
O, en cambio, observa que el primer destello le llega al mismo tiempo que el
segundo destello es emitido.

CAP
ITULO 16. RELATIVIDAD ESPECIAL

590

Construya el diagrama de Loedel para la situacin planteada, y usndolo, deo


a

muestre que, para que dicha situacin sea posible, debe cumplirse que c0 = L .
o
0
Observe que dicha condicin es independiente de la velocidad relativa entre los
o
observadores.
29. Considere un pin 0 cuya energ cintica es T , que decae en vuelo en dos fotoo
a
e
nes que se emiten a lo largo de la recta denida por su movimiento. Encuentre
algebraicamente la energ de ambos fotones.
a
De los siguientes cuatro diagramas de energ y momentum, indique cul reprea
a
senta mejor la situacin planteada, y justique por qu cada uno de los otros
o
e
no puede ser correcto. En cada gura, las echas con punta llena representan el
estado inicial, y las echas con punta vac representan el estado nal.
a

^
E

(A)

^
pc

^
E

(C)

^
pc

^
E

(B)

^
pc

^
E

(D)

^
pc

30. Un observador en un sistema de referencia S observa una explosin que ocurre


o
en la posicin x1 . Una segunda explosin ocurre un tiempo despus, en la
o
o
e
, que se mueve a lo largo del eje +x
posicin x2 . En el sistema de referencia S
o
con velocidad v respecto a S, ambas explosiones ocurren en el mismo punto del
espacio. Considere t = t = 0 el instante en que ambos sistemas de referencia
coinciden.

CAP
ITULO 16. RELATIVIDAD ESPECIAL

591

Encuentre la velocidad v entre los sistemas de referencia, y la separacin de


o
) entre las explosiones.
tiempo (medida en S
31. Considere los siguientes diagramas de energ y momentum. En cada gura,
a
las echas con punta llena representan el estado inicial, y las echas con punta
vac representan el estado nal. Describa las colisiones representadas por cada
a
diagrama, indicando las part
culas involucradas y su estado de movimiento (direccin de movimiento, relacin cualitativa entre las magnitudes de las energ
o
o
as
y momentos de las part
culas involucradas, etc.).
^
E

^
pc

(A)

P1

^
E

P2

(C)

^
pc

(B)

P1

^
E

^
E

K0

^
pc

P2

(D)

^
pc

P1

^
E

P2

(E)

^
pc

32. En la orilla de la recin remodelada laguna del Parque Quinta Normal, Sergio
e
desea observar cmo su hijo Ignacio se aleja en un bote en direccin a la otra
o
o
orilla, donde don Esteban, dueo de los botes los espera. Ignacio se encuentra
n

CAP
ITULO 16. RELATIVIDAD ESPECIAL

592

en el extremo posterior del bote, y en t = 0 se encuentra junto a su padre.


Sergio, en todo caso, era un padre aprehensivo, y se puso a pensar en cun
a
lejos estar la otra orilla, y se dio cuenta de que pod averiguarlo haciendo
a
a
el siguiente experimento: tom una linterna y al mismo tiempo que el bote
o
parti, envi un destello luminoso en direccin a don Esteban, de tal modo
o
o
o
que el destello lleg donde don Esteban, segn Sergio, al mismo tiempo que el
o
u
extremo delantero del bote.
Si se sabe que la velocidad del bote es v y que la longitud propia del bote
es L0 , cul es la distancia L entre Sergio y don Esteban, en el sistema de
a
referencia de Sergio? Para Sergio, obviamente el trayecto se hizo interminable,
pero para Ignacio, que iba disfrutando el viaje, no. Cunto tiempo dur el viaje
a
o
(es decir, cunto tiempo transcurri desde que la linterna es encendida hasta
a
o
que el extremo delantero del bote llega donde don Esteban) segn Ignacio?
u

Don Esteban

Sergio
Ignacio

También podría gustarte